You are on page 1of 369

RISE USMLE NEPAL HELP OTHERS SO THAT GOD WILL HELP YOU.

Con ten ts
HELP OTHERS SO THAT GOD WILL HELP YOU.

Review ers vii


Preface ix
Ack n ow ledgm en ts xi

1. THE BEGINNING OF LIFE: PREGNANCY THROUGH


PRESCHOOL 1
I. Ch ildbirth an d th e Postp artu m Period 1
II. In an cy: Birth to 15 Mon th s 3
III. Th e Toddler Years: 15 Mon th s–2½ Years 5
IV. Th e Presch ool Ch ild: 3–6 Years 6
Review Test 8

2. SCHOOL AGE, ADOLESCENCE, SPECIAL ISSUES OF


DEVELOPMENT, AND ADULTHOOD 15
I. Sch ool Age: 7–11 Years 15
II. Adolescen ce: 11–20 Years 16
III. Sp ecial Issu es in Ch ild Develop m en t 18
IV. Early Adu lth ood: 20–40 Years 19
RISE USMLE NEPAL

V. Middle Adu lth ood: 40–65 Years 19


Review Test 20

3. AGING, DEATH, AND BEREAVEMENT 25


I. Agin g 25
II. Stages o Dyin g an d Death 27
III. Bereavem en t (Norm al Grie ) Versu s Com p licated Bereavem en t
(Dep ression ) 27
Review Test 29

xiii
xiv Contents

4. GENETICS, ANATOMY, AND BIOCHEMISTRY


OF BEHAVIOR 34
I. Th e Gen etics o Beh avior 34
II. Th e Neu roan atom y o Beh avior 35
III. Neu rotran sm ission 37
IV. Biogen ic Am in es 38
V. Am in o Acid Neu rotran sm itters 41
VI. Neu rop ep tides 41
Review Test 42
HELP OTHERS SO THAT GOD WILL HELP YOU.

5. BIOLOGICAL ASSESSMENT OF PATIENTS WITH


PSYCHIATRIC SYMPTOMS 49
I. Overview 49
II. Measu rem en t o Biogen ic Am in es an d Psych otrop ic Dru gs 49
III. Evalu atin g En docrin e Fu n ction 50
IV. Neu roim agin g an d Electroen cep h alogram Stu dies 50
V. Neu rop sych ological Tests 51
VI. Oth er Tests 52
Review Test 53

6. PSYCHOANALYTIC THEORY AND DEFENSE MECHANISMS 57


I. Overview 57
II. Freu d’s Th eories o th e Min d 57
III. De en se Mech an ism s 58
IV. Tran s eren ce Reaction s 58
Review Test 61
RISE USMLE NEPAL

7. LEARNING THEORY 65
I. Overview 65
II. Habitu ation an d Sen sitization 65
III. Classical Con dition in g 66
IV. Op eran t Con dition in g 66
Review Test 69

8. CLINICAL ASSESSMENT OF PATIENTS WITH BEHAVIORAL


SYMPTOMS 74
I. Overview o Psych ological Testin g 74
II. In telligen ce Tests 74
Contents xv

III. Ach ievem en t Tests 75


IV. Person ality Tests 76
V. Psych iatric Evalu ation o th e Patien t with Em otion al Sym p tom s 76
Review Test 79

9. SUBSTANCE-RELATED DISORDERS 83
I. Su bstan ce-related Disorders: Def n ition s, Ep idem iology, an d
Dem ograp h ics 83
HELP OTHERS SO THAT GOD WILL HELP YOU.

II. Stim u lan ts 84


III. Sedatives 85
IV. Op ioids 87
V. Hallu cin ogen s an d Related Agen ts 88
VI. Clin ical Featu res o Su bstan ce-related Disorders 89
VII. Man agem en t 90
Review Test 91

10. NORMAL SLEEP AND SLEEP DISORDERS 98


I. Norm al Awake an d Sleep States 98
II. Classif cation o Sleep Disorders 100
III. In som n ia 100
IV. Breath in g-related Sleep Disorder (Sleep Ap n ea) 102
V. Narcolep sy 102
VI. Man agem en t o Sleep Disorders 103
Review Test 104

11. SCHIZOPHRENIA SPECTRUM AND OTHER PSYCHOTIC


DISORDERS 111
RISE USMLE NEPAL

I. Sch izop h ren ia 111


II. Oth er Psych otic Disorders 114
Review Test 116

12. DEPRESSIVE DISORDERS AND BIPOLAR AND RELATED


DISORDERS 121
I. Overview 121
II. Classif cation o Dep ressive an d Bip olar Disorders 122
III. Etiology 124
IV. Man agem en t 126
Review Test 127
xvi Contents

13. ANXIETY DISORDERS, SOMATIC SYMPTOM DISORDERS,


AND RELATED CONDITIONS 132
I. An xiety Disorders 132
II. Som atic Sym p tom an d Related Disorders 135
III. Factitiou s Disorder, Factitiou s Disorder Im p osed on An oth er, an d
Malin gerin g 136
Review Test 137
HELP OTHERS SO THAT GOD WILL HELP YOU.

14. NEUROCOGNITIVE, PERSONALITY, DISSOCIATIVE, AND


EATING DISORDERS 144
I. Neu rocogn itive Disorders 144
II. Person ality Disorders 148
III. Dissociative Disorders 149
IV. Obesity an d Eatin g Disorders 150
Review Test 153

15. PSYCHIATRIC DISORDERS IN CHILDREN 160


I. Au tism Sp ectru m an d Related Disorders 160
II. Atten tion Def cit/ Hyp eractivity Disorder, Op p osition al Def an t Disorder
an d Con du ct Disorder 161
III. Oth er Disorders o Ch ildh ood 163
Review Test 164

16. BIOLOGIC THERAPIES: PSYCHOPHARMACOLOGY 169


RISE USMLE NEPAL

I. Overview 169
II. An tip sych otic Agen ts 169
III. An tid ep ressan t Agen ts 171
IV. Mood Stabilizers 175
V. An tian xiety Agen ts 175
VI. Psych oactive Medication s in Pregn an cy 176
VII. Electrocon vu lsive Th erapy an d Related Th erap ies 177
Review Test 179

17. PSYCHOLOGICAL THERAPIES 187


I. Psych oan alysis an d Related Th erap ies 187
II. Beh avioral Th erap ies 188
III. Oth er Th erap ies 189
Review Test 191
Contents xvii

18. THE FAMILY, CULTURE, AND ILLNESS 195


I. Overview o th e Fam ily 195
II. Dem ograp h ics an d Cu rren t Tren ds 195
III. Cu ltu re in th e Un ited States 197
IV. Am erican Su bcu ltu res 198
Review Test 200

19. SEXUALITY 205


HELP OTHERS SO THAT GOD WILL HELP YOU.

I. Sexu al Develop m en t 205


II. Th e Biology o Sexu ality in Ad u lts 206
III. Sexu al Dys u n ction 207
IV. Parap h ilias an d Parap h ilic Disorders 209
V. Illn ess, In ju ry, an d Sexu ality 210
VI. Agin g an d Sexu ality 210
VII. Dru gs an d Sexu ality 211
VIII. Th e Hu m an Im m u n odef cien cy Viru s an d Sexu ality 211
Review Test 213

20. AGGRESSION AND ABUSE 220


I.Aggression 220
II.Abu se an d Neglect o Ch ildren , th e Elderly, an d Im p aired Person s 221
III.Ph ysical an d Sexu al Abu se o Dom estic Partn ers 223
IV. Th e Role o th e Ph ysician in Su sp ected Ch ild, Elder, an d Dom estic
Partn er Abu se 224
V. Sexu al Aggression : Rap e an d Related Crim es 224
Review Test 226
RISE USMLE NEPAL

21. THE PHYSICIAN–PATIENT RELATIONSHIP 232


I. Medical Practice 232
II. Adh eren ce 233
III. Th e Clin ical In terview 235
Review Test 238

22. PSYCHOSOMATIC MEDICINE 248


I. Stress an d Health 248
II. Psych ological Stress in Sp ecif c Patien t Pop u lation s 250
III. Patien ts with Ch ron ic Pain 251
IV. Patien ts with Acqu ired Im m u n e Def cien cy Syn drom e 252
Review Test 253
xviii Contents

23. LEGAL AND ETHICAL ISSUES IN MEDICINE 258


I. Legal Com p eten ce 258
II. In orm ed Con sen t 259
III. Con f den tiality 260
IV. Rep ortable Illn esses 260
V. Eth ical Issu es In volvin g HIV In ection 261
VI. In volu n tary an d Volu n tary Psych iatric Hosp italization 261
VII. Advan ce Directives 262
VIII. Death an d Eu th an asia 262
HELP OTHERS SO THAT GOD WILL HELP YOU.

IX. Medical Malp ractice 263


X. Im p aired Ph ysician s 264
Review Test 265

24. HEALTH CARE IN THE UNITED STATES 278


I. Health Care Delivery System s 278
II. Costs o Health Care 280
III. Paym en t or Health Care: Health In su ran ce 280
IV. Dem ograp h ics o Health in th e Un ited States 283
Review Test 285

25. MEDICAL EPIDEMIOLOGY 290


I. Medical Ep idem iology: In ciden ce an d Prevalen ce 290
II. Research Stu dy Design 291
III. Qu an ti yin g Risk 291
IV. Bias, Reliability, an d Validity 293
V. Clin ical Probability an d Attack Rate 296
Review Test 298
RISE USMLE NEPAL

26. STATISTICAL ANALYSES 306


I. Elem en ts o Statistical An alyses 306
II. Hyp oth esis Testin g 309
III. Statistical Tests 310
Review Test 312

Comprehensive Examination 317


Index 353
Th e Begin n in g o Li e:
c ha pte r
1 Pregn an cy Th rou gh
Presch ool
HELP OTHERS SO THAT GOD WILL HELP YOU.

Typical Board Question


Th e eld erly gran d ath er o two ch ild ren , on e aged 2 years an d on e aged 4 years, h as recen tly
died . Th e ch ildren’s gran d ath er was very in volved in th e care o th e ch ildren . Wh en th e
ch ildren are told ab ou t th e death , th e typ ical p ercep tion o th eir gran d ath er’s death or th e
2-year-old an d th e 4-year-old , resp ectively, is m ost likely to be th at th e death
(A) is aban don m en t; is p u n ish m en t
(B) is p u n ish m en t; is aban don m en t
(C) is p u n ish m en t; is irreversib le
(D) is ab an d on m en t; is irreversib le
(E) is irreversib le; is p u n ish m en t
(See “An sw ers an d Explan ation s” at th e en d of th e ch apter.)

I. CHILDBIRTH AND THE POSTPARTUM PERIOD


A. Birth rate in the United States and cesarean birth
1. Abou t 4 m illion ch ildren are born each year in th e Un ited States.
2. Abou t on e-th ird o th ese b irth s are by cesarean section .
3. Th e n u m ber o cesarean birth s declin ed du rin g th e 1990s, p artly in resp on se to in creasin g
RISE USMLE NEPAL

evid en ce th at wom en o ten u n dergo u n n ecessary su rgical p roced u res. From 2000 to 2009,
Th e rate was in creasin g bu t sin ce 2009 th e rate h as been declin in g again .

B. Premature birth
1. Premature births an d very premature births are d e in ed as th ose ollowin g a gestation o less
th an 37 an d 32 completed weeks , resp ectively.
2. Prem atu re b irth p u ts a ch ild at greater risk or dyin g in th e irst year o li e an d or em o-
tion al, beh avioral, an d learn in g p roblem s as well as physical an d intellectual disabilities .
3. Prem atu re b irth s, wh ich are associated with low in com e, m atern al illn ess or m aln u trition ,
an d you n g m atern al age, occu r in alm ost twice as m an y n on -Hisp an ic A rican -Am erican
in an ts th an n on -Hisp an ic Wh ite in an ts.

C. Infant mortality
1. Low socioeconomic status , wh ich is related in p art to eth n icity, is associated with p rem a-
tu rity an d h igh in an t m ortality (Table 1.1).
2. In p art, becau se th e Un ited States d oes n ot h ave a system o h ealth care or all citizen s
p aid or by th e govern m en t th rou gh taxes, p rem atu rity an d in an t m ortality rates in th e
Un ited States are h igh as com p ared to th e rates in oth er develop ed cou n tries (Figu re 1.1).

1
2 BRS Behavioral Science

t a b l e 1.1 Ethnicity and Infant Mortality in the United States (2010)

Ethnic Group Infant Deaths Per 1,000 Total Live Births

All ethnic groups 6.14


Asian or Pacific Islander 4.27
Non-Hispanic White 5.18
Hispanic 5.25
Native American 8.28
Non-Hispanic Black (African American) 11.46
From Matthews TJ, MacDorman M. Infant mortality statistics from the 2010 period: Linked birth/infant death set. Natl Vital Stat Rep. 2013;62(8):1–26.
HELP OTHERS SO THAT GOD WILL HELP YOU.

3. Th e Apgar score (named for Dr. Virginia Apgar but useful as a mnemonic): A—appearance
(color), P—pulse (heartbeat), G—grimace (reflex irritability), A—activity (muscle tone),
R—respiration (breathing regularity), qu an ti ies p h ysical u n ction in g in p rem atu re an d
u ll-term n ewborn s (Table 1.2) an d can be u sed to p redict th e likelih ood o im m ediate
su rvival.
Th e in an t is evalu ated 1 m in u te an d 5 (or 10) m in u tes a ter birth . Each o th e ive m easu res
can h ave a score o 0, 1, or 2 (h igh est score = 10). Score >7 = n o im m in en t su rvival th reat; score
<4 = im m in en t su rvival th reat.

D. Postpartum maternal reactions


1. Baby blues
a. Man y wom en exp erien ce a typ ical em otion al reaction called “baby blues ” or “p ostp ar-
tu m b lu es” lastin g u p to 2 weeks a ter ch ildb irth .
b. Th is reaction resu lts rom psychological factors (e.g., th e em otion al stress o ch ild b irth ,
th e eelin gs o ad ded resp on sibility) as well as physiological factors (e.g., ch an ges in
h orm on e levels, atigu e).

Ja pa n 2.3

S we de n 2.5

S pa in 3.2
RISE USMLE NEPAL

Ge rma ny 3.4

Ita ly 3.4

Fra nce 3.6

Aus tra lia 4.1

Unite d Kingdom 4.2

Ca na da (2009) 4.9

Unite d S ta te s 6.1

0 1 2 3 4 5 6
Ra te pe r 1,000 live births
FIGURE 1.1. Infant mortality rates: Selected Organisation for Economic Co-operation and Development countries, 2010
(Selected countries). (From: International comparisons of infant mortality and related factors: United States and Europe.
Natl Vital Stat Rep. 63 (5), 2014 Figure 1.)
Chapter 1 The Beginning of Life: Pregnancy Through Preschool 3

t a b l e 1.2 The Apgar Scoring System

Score
Measure 0 1 2

Heartbeat Absent Slow (<100/min) Rapid (>100/min)


Respiration Absent Irregular, slow Good, crying
Muscle tone Flaccid, limp Weak, inactive Strong, active
Color of body and Both body and extremities Pink body, blue extremities Pink body, pink extremities
extremities pale or blue
Reflexes, e.g., heel prick No response Grimace Foot withdrawal, cry,
HELP OTHERS SO THAT GOD WILL HELP YOU.

or nasal tickle sneeze, cough

c. Man agem en t in volves em otion al su p p ort rom th e p h ysician as well as p ractical su g-


gestion s or ch ild care.
2. Major depressive disorder with peripartum onset an d brief psychotic disorder with post-
partum onset (p ostp artu m p sych osis) are m ore seriou s reaction s th an p ostp artu m blu es
an d are treated with an tidep ressan t an d an tip sych otic m edication s (Table 1.3) (an d see
Ch ap ters 11 an d 12).
Wom en wh o h ave exp erien ced th ese reaction s on ce are at risk or h avin g sim ilar
reaction s a ter su bsequ en t d eliveries.

II. INFANCY: BIRTH TO 15 MONTHS


A. Bonding of the parent to the infant
1. Bon d in g b etween th e caregiver an d th e in an t is en h an ced by physical contact between
th e two.
2. Bon d in g m ay b e ad versely a ected i :
a. Th e ch ild is o low birth weight or ill, leadin g to separation from the mother a ter delivery.
b. Th ere are p rob lem s in th e m oth er– ath er relation sh ip.

t a b l e 1.3 Postpartum Maternal Reactions

Maternal Reaction Incidence Onset of Symptoms Duration of Symptoms Characteristics


RISE USMLE NEPAL

Postpartum blues 33%–50% Within a few days Up to 2 wk after Exaggerated emotionality and
(“baby blues”) after delivery delivery tearfulness
Interacting well with friends and
family
Good grooming
Major depressive 5%–10% Within 4 wk after Up to 1 y without Feelings of hopelessness and
disorder delivery treatment; 3–6 wk helplessness
with treatment Lack of pleasure or interest in
usual activities
Poor self-care
May include psychotic
symptoms (“major
depressive disorder with
psychotic features”), e.g.,
hallucinations and delusions
(see Table 11.1)
Mother may harm infant
Brief psychotic 0.1%–0.2% Within 4 wk after Up to 1 mo Psychotic symptoms not better
disorder delivery accounted for by major
(postpartum depressive disorder with
onset) psychotic features
Mother may harm infant
4 BRS Behavioral Science

3. Wom en wh o are ed u cated an d p rep ared or ch ildbirth h ave sh orter labors, ewer m edical
com p lication s, less n eed or m ed ication , an d closer in itial in teraction s with th eir in an ts.

B. Attachment of the infant to the parent


1. Th e p rin cip al p sych ological task o in an cy is th e formation of an intimate attachment to th e
p rim ary caregiver, u su ally th e m oth er.
2. Toward th e en d o th e irst year o li e, separation from the primary caregiver leads to in itial
lou d p rotests rom th e in an t (typ ical “sep aration an xiety”).
3. With con tin u ed absen ce o th e m oth er, th e in an t is at risk or depression.
a. In an ts m ay exp erien ce dep ression even wh en th ey are livin g with th eir m oth ers i th e
m oth er is p h ysically an d em otion ally d istan t an d in sen sitive to th eir n eeds.
HELP OTHERS SO THAT GOD WILL HELP YOU.

b. Dep ressed in an ts m ay exh ib it poor health an d slowed physical growth.


c. Th e Diagn ostic an d Statistical Man u al of Men tal Disorders, Fifth Edition (DSM-5)
term s or distu rban ces in oth erwise typ ical ch ildren owin g to grossly p ath ological care
are reactive attachment disorder an d disinhibited social engagement disorder.
(1) Reactive attachment disorder: Ch ild ren are with drawn an d u n resp on sive.
(2) Disinhibited social engagement disorder: Ch ildren ap p roach an d attach in discrim i-
n ately to stran gers as th ou gh th e stran gers were am iliar to th em .

C. Studies of attachment
1. Harry Harlow d em on strated th at in an t m on keys reared in relative isolation by surrogate
artificial mothers d o n ot develop typ ical m atin g, m atern al, an d social beh avior as adu lts.
a. Males m ay b e m ore a ected th an em ales by su ch isolation .
b. Th e length of time of isolation is im p ortan t. You n g m on keys isolated or less th an
6 m on th s can b e reh ab ilitated by p layin g with typ ical you n g m on keys.
2. René Spitz docu m en ted th at ch ild ren with ou t p rop er m oth erin g (e.g., th ose in orp h an -
ages) sh ow severe developmental retardation, p oor h ealth , an d h igh er death rates (“hospi-
talism”) in sp ite o ad equ ate p h ysical care.
3. Partly b ecau se o su ch in d in gs, th e foster care system was establish ed or you n g ch ildren
in th e Un ited States wh o d o n ot h ave ad equ ate h om e situ ation s. Foster am ilies are th ose
th at h ave b een ap p roved an d u n d ed by th e state o residen ce to take care o a ch ild in
th eir h om es.

D. Characteristics of the infant


1. Reflexive behavior. At birth , th e typ ical in an t p ossesses sim p le re lexes su ch as th e sucking
re lex, startle re lex (Moro re lex), palmar grasp re lex, Babinski re lex, an d rooting re lex. All
o th ese re lexes disap p ear d u rin g th e irst year o li e (Table 1.4).
RISE USMLE NEPAL

2. Motor, social, verbal, an d cognitive development (Table 1.5)


a. Alth ou gh th ere is a re lexive sm ile p resen t at birth , th e social smile is on e o th e irst
m arkers o th e in an t’s resp on siven ess to an oth er in dividu al.
b. Cryin g an d with drawin g in th e p resen ce o an u n am iliar p erson (stranger anxiety) is
n orm al an d b egin s at ab ou t 7 m on th s o age.
(1) Th is b eh avior in d icates th at th e in an t h as develop ed a sp eci ic attach m en t to th e
m oth er an d is ab le to d istin gu ish h er rom a stran ger.

t a b l e 1.4 Reflexes Present at Birth and the Age at which They Disappear

Reflex Description Age of Disappearance

Palmar grasp The child’s fingers grasp objects placed in the palm 2 mo
Rooting and sucking The child’s head turns in the direction of a stroke on the cheek 3 mo
reflexes when seeking a nipple to suck
Startle (Moro) reflex When the child is startled, the arms and legs extend 4 mo
Babinski reflex Dorsiflexion of the largest toe when the plantar surface of the 12 mo
child’s foot is stroked
Tracking reflex The child visually follows a human face Continues
Chapter 1 The Beginning of Life: Pregnancy Through Preschool 5

t a b l e 1.5 Motor, Social, Verbal, and Cognitive Development of the Infant

Skill Area
Age (in Months) Motor Social Verbal and Cognitive

1–3 Lifts head when lying prone Smiles in response to a Coos or gurgles in response to
human face (the “social human attention
smile”)
4–6 Turns over (5 mo) Forms an attachment to Babbles (repeats single
Sits unassisted (6 mo) primary caregiver sounds over and over)
Reaches for objects Recognizes familiar people
Grasps with entire hand (“raking
HELP OTHERS SO THAT GOD WILL HELP YOU.

grasp”)
7–11 Crawls on hands and knees Shows stranger anxiety Imitates sounds
Pulls self up to stand Plays social games such Uses gestures
Transfers toys from hand to hand as peek-a-boo, waves Responds to own name
(10 mo) “bye-bye” Responds to simple
Picks up toys and food using instructions
“pincer” (thumb and
forefinger) grasp (10 mo)
12–15 Walks unassisted Shows separation anxiety Says first words
Shows object permanence

(2) In an ts exp osed to m an y caregivers are less likely to sh ow stran ger an xiety th an
th ose exp osed to ew caregivers.
c. At ab ou t 1 year, th e ch ild can m ain tain th e m en tal im age o an object or o th e m oth er
with ou t seein g it or h er (“object permanence ”).

E. Theories of development
1. Chess an d Thomas sh owed th at th ere are endogenous differences in th e temperaments o
in an ts th at rem ain qu ite stable or th e irst 25 years o li e. Th ese di eren ces in clu de such
ch aracteristics as reactivity to stim u li, resp on siven ess to p eop le, an d atten tion sp an .
a. Easy children are adap table to ch an ge, sh ow regu lar eatin g an d sleep in g p attern s, an d
h ave a p ositive m ood .
b. Difficult children sh ow traits op p osite to th ose o easy ch ild ren .
c. Slow-to-warm-up children sh ow traits o di icu lt ch ildren at irst bu t th en im p rove an d
adap t with in creased con tact with oth ers.
2. Sigmund Freud d escrib ed d evelop m en t in term s o th e p arts o th e b od y rom wh ich th e
m ost p leasu re is d erived at each stage o develop m en t (e.g., th e “oral stage” occu rs du rin g
RISE USMLE NEPAL

th e irst year o li e).


3. Erik Erikson describ ed d evelop m en t in term s o critical p eriods or th e ach ievem en t o
social goals; i a sp eci ic goal is n ot ach ieved at a sp eci ic age, th e in dividu al will h ave di -
icu lty ach ievin g th e goal in th e u tu re. For exam p le, in Erikson’s stage o basic trust versus
mistrust, ch ild ren m u st learn to tru st oth ers du rin g th e irst year o li e or th ey will h ave
trou ble orm in g close relation sh ip s as adu lts.
4. J ean Piaget described developm en t in term s o learn in g capabilities o the child at each age.
5. Margaret Mahler describ ed early d evelop m en t as a sequ en tial p rocess o sep aration o th e
ch ild rom th e m oth er or p rim ary caregiver.

III. THE TODDLER YEARS: 15 MONTHS–2½ YEARS


A. Attachment
1. Th e major theme o th e secon d year o li e is to separate from the mother or p rim ary care-
giver, a p rocess th at is com p lete by abou t age 3.
2. Th e todd ler h as n o u n d erstan din g o d eath an d sees th e death o a close am ily m em ber
as aban d on m en t or sep aration .
6 BRS Behavioral Science

t a b l e 1.6 Motor, Social, Verbal, and Cognitive Development of the Toddler and
Preschool Child

Skill Area
Age (Years) Motor Social Verbal and Cognitive
1.5 Throws a ball Moves away from and then Uses about 10 individual
Stacks three blocks returns to the mother for words
Climbs stairs one foot at a time reassurance (rapprochement) Says own name
Scribbles on paper
2 Kicks a ball Shows negativity (e.g., the favorite Uses about 250 words
Balances on one foot for 1 s word is “no”) Speaks in two-word
HELP OTHERS SO THAT GOD WILL HELP YOU.

Stacks six blocks Plays alongside but not with sentences and uses
Feeds self with spoon another child (“parallel play”: pronouns (e.g., “me do”)
2–4 y of age) Names body parts and objects
3 Rides a tricycle Has a sense of self as male or Uses about 900 words in
Undresses and partially dresses female (gender identity) speech
without help Usually achieves bowel and Understands about 3,500
Climbs stairs using alternate bladder control (problems words
feet such as encopresis [“soiling”] Identifies some colors
Stacks nine blocks and enuresis [“bed wetting”] Speaks in complete
Copies a circle cannot be diagnosed until 4 sentences (e.g., “I can do
and 5 y of age, respectively) it myself”)
Comfortably spends part of the Strangers can now
day away from mother understand her
4 Catches a ball with arms Begins to play cooperatively with Shows good verbal self-
Dresses independently, using other children expression (e.g., can tell
buttons and zippers Engages in role playing (e.g., “I’ll detailed stories)
Grooms self (e.g., brushes teeth) be the mommy, you be the Comprehends and uses
Hops on one foot daddy”) prepositions (e.g., under,
Draws a person May have imaginary companions above)
Copies a cross Curious about sex differences
(e.g., plays “doctor” with other
children)
Has nightmares and transient
phobias (e.g., of “monsters”)
5 Catches a ball with two hands Has romantic feelings about Shows further improvement
Draws a person in detail (e.g., the opposite sex parent (the in verbal and cognitive
with arms, hair, eyes) “oedipal phase”) at 4–5 y of skills
Skips using alternate feet age
Copies a square Overconcerned about physical
injury at 4–5 y of age
6 Ties shoelaces Begins to develop an internalized Begins to think logically (see
RISE USMLE NEPAL

Rides a two-wheeled bicycle moral sense of right and wrong Chapter 2)


Prints letters Begins to understand the finality Begins to read
Copies a triangle of death

3. Th ere is n o com p ellin g eviden ce th at daily sep aration rom workin g p aren ts in a good day
care settin g h as sh ort- or lon g-term n egative con sequ en ces or ch ildren . However, wh en
com p ared to ch ildren wh o stay at h om e with th eir m oth ers, th ose wh o h ave been in day
care sh ow m ore aggressiveness .

B. Motor, social, verbal, and cognitive characteristics of the toddler


See Table 1.6.

IV. THE PRESCHOOL CHILD: 3–6 YEARS


A. Attachment
1. A ter reach in g 3 years o age, a ch ild sh ou ld be able to sp en d a ew h ou rs away rom th e
m oth er in th e care o oth ers (e.g., in day care).
Chapter 1 The Beginning of Life: Pregnancy Through Preschool 7

2. A ch ild wh o can n ot do th is a ter age 3 is exp erien cin g separation anxiety disorder (see
Ch ap ter 15).
3. Presch ool ch ild ren m ay p erceive death as a punishment or bad beh avior. Th ey believe th at
death is tem p orary an d typ ically exp ect th at a d ead relative (or p et) will com e b ack to li e.

B. Characteristics
1. Th e ch ild’s vocabulary increases rap id ly. Th e 3-year-old ch ild can typ ically say abou t 900
words an d sp eaks in com p lete sen ten ces.
2. Toilet training typ ically occu rs at age 3 years. Delayed toilet train in g is m ost o ten related
to p h ysiological im m atu rity du e to gen etic actors, or exam p le, th e ath er was also a “bed
wetter” as a ch ild.
HELP OTHERS SO THAT GOD WILL HELP YOU.

3. Th e birth o a siblin g or oth er life stress , su ch as m ovin g or divorce, m ay resu lt in a ch ild’s


u se o regression, a d e en se m ech an ism (see Ch ap ter 6) in wh ich th e ch ild tem p orarily
beh aves in a “b aby-like” way (e.g., alth ou gh h e is toilet train ed, h e starts wettin g th e bed
again ). Regression o ten occu rs in typ ical ch ild ren as a reaction to li e stress.
4. Ch ild ren can d istin gu ish an tasy rom reality (e.g., th ey kn ow th at im agin ary rien d s are
n ot “real” p eop le), alth ou gh th e lin e between th em m ay still n ot be sh arp ly drawn .
5. Presch ool ch ild ren are typ ically active an d rarely sit still or lon g.
6. Oth er asp ects o m otor, social, verbal, an d cogn itive develop m en t o th e p resch ool ch ild
can be ou n d in Tab le 1.6.

C. Changes at 6 years of age


1. Th e ch ild begin s to u n derstan d th at death is final an d irreversible an d ears th at h is or h er
p aren ts will d ie an d leave. It is n ot u n til abou t age 9, h owever, th at th e ch ild u n derstan ds
th at h e or sh e also can d ie.
2. At th e en d o th e p resch ool years (ab ou t age 6), th e ch ild’s conscience (th e superego o
Freu d ) an d sen se o m orality b egin to develop.
3. A ter age 6, ch ildren can p u t th em selves in an oth er p erson’s p lace (empathy) an d beh ave
in a carin g an d sh arin g way toward oth ers.
4. Morality an d em p ath y in crease u rth er du rin g th e sch ool-age years (see Ch ap ter 2).
RISE USMLE NEPAL
Review Test

Directions: Each o th e n u m bered item s or in com p lete statem en ts in th is section is ollowed by


an swers or by com p letion s o th e statem en t. Select th e one lettered an swer or com p letion th at
is best in each case.
HELP OTHERS SO THAT GOD WILL HELP YOU.

1. Paren ts o a 13-m on th -old ch ild tell th e 4. An Am erican cou p le wou ld like to adop t
doctor th at th e ch ild sh ows n o in terest in a 10-m on th -old Rom an ian ch ild . However,
toilet train in g. Th ey also relate th at th e ch ild th ey are con cern ed becau se th e ch ild
sp eaks abou t 10 word s an d h as ju st started h as been in an orp h an age sin ce h e was
to walk u n assisted. Th e doctor sh ou ld sep arated rom h is birth m oth er 5 m on th s
(A) tell th e p aren ts th at th e ch ild’s h earin g ago. Th e orp h an age is clean an d well kep t
sh ou ld be ch ecked as soon as p ossible bu t h as a h igh sta tu rn over ratio. Wh ich o
(B) con tact ch ild p rotective services th e ollowin g ch aracteristics is th e cou p le
(C) reassu re th e p aren ts th at th e ch ild’s m ost likely to see in th e ch ild at th is tim e?
beh avior is typ ical or h er age (A) Lou d cryin g an d p rotests at th e loss o
(D) re er th e am ily to a p ediatric h is m oth er
gastroen terologist (B) In creased resp on siven ess to adu lts
(E) evalu ate th e ch ild or d elayed m otor (C) Typ ical develop m en t o m otor skills
develop m en t (D) Reactive attach m en t disorder
(E) Typ ical develop m en t o social skills
2. In a m ajor city h osp ital, th e h earin g o
all n ewborn s is evalu ated sh ortly a ter b irth . 5. Wh en a p h ysician con du cts a well-ch ild
Th e m ajor objective o th is h earin g loss ch eck u p on a typ ical 2-year-old girl, th e
screen in g is to ch ild is m ost likely to sh ow wh ich o th e
(A) determ in e th e n ecessity o u sin g coch lear ollowin g skills or ch aracteristics?
im p lan ts be ore th e age o 6 m on th s (A) Sp eaks in two-word sen ten ces
(B) determ in e th e n ecessity o sp eech (B) Is toilet train ed
th erapy b e ore th e age o 1 year (C) Can com ortably sp en d m ost o th e day
(C) diagn ose an d treat h earin g loss early in away rom h er m oth er
order to p reven t lan gu age develop m en t (D) Can ride a tricycle
delay (E) En gages in coop erative p lay
RISE USMLE NEPAL

(D) diagnose an d treat hearing loss early in


order to preven t m otor developm en t delay 6. Wh en a p h ysician con du cts a well-ch ild
(E) in crease th e cost-e ectiven ess o ch eck u p on a 3-year-old boy, h e f n ds th at
treatm en t or h earin g loss th e ch ild can ride a tricycle, copy a circle,
en gage in p arallel p lay with oth er ch ildren ,
3. Th e con cern ed p aren ts o a 5-year-old n am e som e o h is body p arts (e.g., n ose,
ch ild rep ort th at th e ch ild is still wettin g eyes) bu t n ot oth ers (e.g., h an d, f n ger),
th e bed . Th e ch ild is oth erwise d evelop in g an d h as abou t a 50-word vocabu lary. With
ap p rop riately or h is age an d p h ysical resp ect to m otor, social, an d cogn itive/
exam in ation is u n rem arkab le. Th e ch ild’s verbal skills, resp ectively, th e best
ath er was also a bed wetter u n til age 8 years. descrip tion o th is ch ild is
Th e m ost com m on cau se o en u resis in a (A) typ ical, typ ical, n eeds evalu ation
ch ild o th is age is (B) typ ical, typ ical, typ ical
(A) em otion al stress (C) n eeds evalu ation , typ ical, n eeds
(B) p h ysiological im m atu rity evalu ation
(C) sexu al ab u se (D) typ ical, n eeds evalu ation , n eeds
(D) u rin ary tract in ection evalu ation
(E) m ajor dep ression (E) typ ical, n eeds evalu ation , typ ical

8
Chapter 1 The Beginning of Life: Pregnancy Through Preschool 9

7. A m oth er brin gs h er 4-m on th -old ch ild to 11. A well-train ed, h igh ly qu alif ed
th e p ediatrician or a well-baby exam in ation . obstetrician h as a bu sy p ractice. Wh ich
Wh ich o th e ollowin g develop m en tal o the ollowin g is m ost likely to be tru e
sign p osts can th e doctor exp ect to be p resen t abou t p ostp artu m reaction s in th is doctor’s
in th is in an t i th e ch ild is develop in g p atien ts?
typically? (A) Postp artu m blu es will occu r in abou t
(A) Stran ger an xiety 10% o th e p atien ts.
(B) Social sm ile (B) Major dep ression will occu r in abou t
(C) Rap p roch em en t 25% o th e p atien ts.
(D) Core gen der iden tity (C) Brie p sych otic disorder will occu r in
(E) Ph obias abou t 8% o th e p atien ts.
HELP OTHERS SO THAT GOD WILL HELP YOU.

(D) Brie p sych otic disorder will u su ally last


8. Th e overall in an t m ortality rate in th e abou t 1 year.
Un ited States in 2010 was ap p roxim ately (E) Postp artu m blu es will u su ally last u p to
(A) 1 p er 1,000 live b irth s 2 weeks.
(B) 3 p er 1,000 live b irth s
(C) 6 p er 1,000 live b irth s 12. A wom an in th e 7th m on th o p regn an cy
(D) 11 p er 1,000 live birth s with h er th ird ch ild tells h er p h ysician sh e is
(E) 40 p er 1,000 live birth s worried th at sh e will exp erien ce d ep ression
a ter th e ch ild is born . Th e m ost im p ortan t
9. Th e m ost im p ortan t p sych ological task or th in g or th e doctor to say at th is tim e is
a ch ild b etween birth an d 15 m on th s is th e (A) “Do n ot worry, th ere are m an y e ective
develop m en t o m ed ication s or d ep ression .”
(A) th e ability to th in k logically (B) “Wom en o ten becom e m ore an xiou s
(B) sp eech toward th e en d o th eir p regn an cy.”
(C) stran ger an xiety (C) “Did you exp erien ce an y em otion al
(D) a con scien ce di icu lties a ter th e birth o you r oth er
(E) an in tim ate attach m en t to th e m oth er or ch ildren ?”
p rim ary caregiver (D) “Do you wan t to start takin g
an tid ep ressan t m ed ication n ow?”
10. Th e h u sban d o a 28-year-old wom an , (E) “Most wom en wh o worry ab ou t
wh o gave b irth to a h ealth y in an t 2 weeks dep ression n ever exp erien ce it.”
ago, rep orts th at h e ou n d h er sh akin g th e (F) “Som e dep ression is com m on a ter
in an t to stop it rom cryin g. Wh en th e ch ildb irth .”
d octor qu estion s th e wom an ab ou t th e
in cid en t, sh e says “I d id n ot realize it wou ld 13. Th e m oth er o a 3-year-old ch ild tells
b e so m u ch work.” Th e p atien t also rep orts th e doctor th at, alth ou gh sh e in stru cts th e
RISE USMLE NEPAL

th at sh e wakes u p at 5 a m every day an d ch ild to sit still at th e din n er table, th e ch ild


can n ot all back asleep an d h as very little can n ot seem to d o so or m ore th an 10
ap p etite. Th e n ext step in m an agem en t is or m in u tes at a tim e. Sh e squ irm s in h er seat
th e doctor to an d gets ou t o h er ch air. Th e ch ild’s m otor
an d verbal skills are ap p rop riate or h er age.
(A) assess th e p atien t or th ou gh ts o su icid e
Wh ich o th e ollowin g best f ts th is p ictu re?
(B) advise th e ath er to h ire a caregiver to
assist th e m oth er in carin g or th e ch ild (A) Sep aration an xiety disorder
(C) set u p an oth er ap p oin tm en t or th e (B) Typ ical beh avior
ollowin g week (C) Delayed develop m en t
(D) p rescribe an an tidep ressan t (D) Lack o basic tru st
(E) tell th e ath er th at th e m oth er is sh owin g (E) Atten tion de icit h yp eractivity disorder
evid en ce o th e “b aby blu es” (ADHD)
10 BRS Behavioral Science

14. A typ ical 8-m on th -old ch ild is b rou gh t to 17. Th e m oth er o a 1-m on th -old ch ild, h er
th e p ediatrician or h is m on th ly well-baby secon d, is con cern ed becau se th e baby cries
exam in ation . Th e ch ild is th e am ily’s f rst, every day rom 6 p m to 7 pm . Sh e tells th e
an d h e is cared or at h om e by h is m oth er. doctor th at, u n like h er f rst ch ild wh o was
Wh en th e doctor ap p roach es th e ch ild in h is always calm , n oth in g sh e does d u rin g th is
m oth er’s arm s, th e ch ild’s beh avior is m ost h ou r seem s to com ort th is baby. Ph ysical
likely to b e ch aracterized by exam in ation is u n rem arkable, an d th e
(A) with drawal rom th e doctor ch ild h as gain ed 2 p ou n d s sin ce b irth . With
(B) sm ilin g at th e d octor resp ect to th e m oth er, th e p h ysician sh ou ld
(C) in di eren ce to th e doctor (A) reassu re h er th at all ch ildren are di eren t
(D) an an ticip atory p ostu re toward th e an d th at som e cryin g is n orm al
HELP OTHERS SO THAT GOD WILL HELP YOU.

doctor (arm s h eld ou t to b e p icked u p ) (B) recom m en d th at sh e see a


(E) with drawal rom both th e doctor an d th e p sych oth erap ist
m oth er (C) p rescribe an an tidep ressan t
(D) recom m en d th at th e ath er care or th e
15. Wh ile sh e p reviou sly slep t in h er own ch ild wh en it is cryin g
bed, a ter h er p aren ts’ d ivorce, a 5-year- (E) re er h er to a p ediatrician sp ecializin g in
old girl begs to be allowed to sleep in h er “di icu lt” in an ts
m oth er’s bed every n igh t. Sh e says th at a
“robb er” is u n der h er bed. Sh e con tin u es 18. A 4-year-old boy survives a h ouse f re
to do well in kin dergarten an d to p lay with in wh ich his ath er was killed. The ch ild has
h er rien d s. Th e b est d escrip tion o th is girl’s n o visible in juries an d m edical evalu ation
behavior is is un rem arkable. Alth ough he has been told
(A) sep aration an xiety d isorder th at his ath er h as died, in the weeks a ter th e
(B) typ ical beh avior with regression f re, th e ch ild con tin ues to ask or h is ather.
(C) delayed develop m en t Th e best explan ation or th is boy’s behavior is
(D) lack o basic tru st (A) an acu te reaction to severe stress
(E) ADHD (B) a typ ical reaction or h is age
(C) delayed develop m en t
16. A 2-year-old girl wh o h as been in (D) re u sal to believe th e tru th
oster care sin ce b irth is very rien d ly an d (E) an u n d iagn osed h ead in ju ry
a ection ate with stran gers. Sh e p u ts h er
arm s ou t to th em to be p icked u p an d
th en “cu dd les u p” to th em . Th e oster Questions 19–24
m oth er states th at th e ch ild h as “beh avior
p roblem s” an d th en n otes th at sh e h as n ever For each develop m en tal m ileston e, select th e
elt “close” to th e ch ild . Th e m ost likely age at wh ich it com m on ly irst ap p ears.
RISE USMLE NEPAL

exp lan ation or th is ch ild’s b eh avior toward


stran gers is 19. Tran s ers toys rom on e h an d to th e
oth er.
(A) typ ical beh avior
(B) Rett’s d isord er (A) 0–3 m on th s
(C) reactive attach m en t d isord er (B) 4–6 m on th s
(D) disin h ib ited social en gagem en t (C) 7–11 m on th s
disord er (D) 12–15 m on th s
(E) m ild au tism sp ectru m disorder (E) 16–30 m on th s
Chapter 1 The Beginning of Life: Pregnancy Through Preschool 11

20. Tu rn s over. 23. Feeds sel with a sp oon .


(A) 0–3 m on th s (A) 0–3 m on th s
(B) 4–6 m on th s (B) 4–6 m on th s
(C) 7–11 m on th s (C) 7–11 m on th s
(D) 12–15 m on th s (D) 12–15 m on th s
(E) 16–30 m on th s (E) 16–30 m on th s

21. Sm iles in resp on se to a h u m an ace. 24. Wh en given a crayon , scribbles on p ap er


(A) 0–3 m on th s (A) 0–3 m on th s
(B) 4–6 m on th s (B) 4–6 m on th s
(C) 7–11 m on th s (C) 7–11 m on th s
HELP OTHERS SO THAT GOD WILL HELP YOU.

(D) 12–15 m on th s (D) 12–15 m on th s


(E) 16–30 m on th s (E) 16–30 m on th s

22. Resp on ds to own n am e.


(A) 0–3 m on th s
(B) 4–6 m on th s
(C) 7–11 m on th s
(D) 12–15 m on th s
(E) 16–30 m on th s
RISE USMLE NEPAL
An swers an d Exp lan ation s

Typical Board Question


The answer is A. Th e typ ical 2-year-old ch ild will see th e death o th e gran d ath er as aban don -
m en t, wh ile th e 4-year-old ch ild will see th e d eath as p u n ish m en t. It is n ot u n til a ter age 6 years
th at ch ild ren b egin to u n derstan d th at death is in al an d irreversible (see Ch ap ter 2).
HELP OTHERS SO THAT GOD WILL HELP YOU.

1. The answer is C. Th e p aren ts sh ou ld b e reassu red th at like th eir ch ild , 13-m on th -old
ch ild ren typ ically say on ly a ew word s an d are ju st startin g to walk. Ch ildren typ ically sh ow
n o in terest in n or can th ey be toilet train ed u n til th ey are at least 2½ –3 years o age.
2. The answer is C. Th e m ajor ob jective o h earin g loss screen in g in n ewborn s is or early
diagn osis an d treatm en t o h earin g loss in ord er to p reven t lan gu age develop m en t delay.
In older ch ild ren , evalu ation o h earin g loss is u se u l in determ in e th e n ecessity o u sin g
coch lear im p lan ts or sp eech th erapy. Hearin g loss is n ot sp eci ically associated with m otor
develop m en t delay.
3. The answer is B. Most ch ild ren are toilet train ed by th e age o 5 years. However, bed wettin g
in a 5-year-old wh o h as n ever been toilet train ed an d is oth erwise develop in g ap p rop riately
is m ost likely to be a resu lt o p h ysiological im m atu rity, p robably related to gen etic
actors, or exam ple, th e ath er was also a bed wetter. Em otion al stress, sexu al abu se, an d
dep ression are less likely to b e th e cau se o bed wettin g in a ch ild wh o h as n ever been toilet
train ed, alth ou gh th ey can lead to b ed wettin g in a p reviou sly toilet-train ed ch ild. Absen ce
o m ed ical in d in gs in dicates th at th is ch ild is u n likely to h ave a u rin ary tract in ection .
4. The answer is D. Th is ch ild is likely to sh ow reactive attach m en t disord er a ter th is
p rolon ged sep aration rom h is m oth er. Alth ou gh th e orp h an age is well kep t, it is u n likely
th e ch ild h as b een able to orm a stab le attach m en t to an oth er caretaker b ecau se o th e h igh
n u m ber o sta ch an ges. Lou d p rotests occu r in itially wh en th e m oth er leaves th e ch ild.
With h er con tin ued absen ce, th is ch ild exp erien ces oth er seriou s reaction s. Th ese reaction s
in clu d e d ep ression , d ecreased resp on siven ess to adu lts, an d de icits in th e develop m en t o
social an d m otor skills.
5. The answer is A. Two-year-old ch ild ren sp eak in two-word sen ten ces (e.g., “Me go”). Toilet
RISE USMLE NEPAL

train in g or th e ability to sp en d m ost o th e d ay away rom th e m oth er does n ot u su ally occu r


u n til age 3. Ch ildren en gage in coop erative p lay startin g at abou t age 4 an d can ride a th ree-
wh eeled b icycle at ab ou t age 3.
6. The answer is A. At th e age o 3 years, th e ch ild can ride a tricycle, copy a circle, an d en gage
in p arallel p lay (p lay alon gside bu t n ot coop eratively with oth er ch ildren ). However, 3-year-
old ch ild ren su ch as th is on e sh ou ld h ave a vocabu lary o abou t 900 words an d sp eak in
com p lete sen ten ces.
7. The answer is B. Th e social sm ile (sm ilin g in resp on se to seein g a h u m an ace) is on e o th e
irst develop m en tal m ileston es to ap p ear in th e in an t an d is p resen t by 1–2 m on th s o age.
Stran ger an xiety ( ear o u n am iliar p eop le) ap p ears at abou t 7 m on th s o age an d in dicates
th at th e in an t h as a sp eci ic attach m en t to th e m oth er. Rap p roch em en t (th e ten den cy to
ru n away rom th e m oth er an d th en ru n back or com ort an d reassu ran ce) ap p ears at abou t
18 m on th s o age. Core gen der iden tity (th e sen se o sel as m ale or em ale) is establish ed
between 2 an d 3 years o age. Tran sien t p h ob ias (irration al ears) occu r in typ ical ch ildren ,
ap p earin g m ost com m on ly at 4–5 years o age.

12
Chapter 1 The Beginning of Life: Pregnancy Through Preschool 13

8. The answer is C. In 2010, th e overall in an t m ortality rate in th e Un ited States was 6.14 p er
1,000 live birth s. Th is rate, wh ich is closely associated with socioecon om ic statu s, was at
least two tim es h igh er in A rican -Am erican in an ts th an in Wh ite in an ts.
9. The answer is E. Th e m ost im p ortan t p sych ological task o in an cy is th e develop m en t
o an in tim ate attach m en t to th e m oth er or p rim ary caregiver. Stran ger an xiety, wh ich
typ ically ap p ears at ab ou t 7 m on th s o age, dem on strates th at th e ch ild h as develop ed
th is attach m en t an d can d istin gu ish its m oth er rom oth ers. Sp eech , th e ab ility to th in k
logically, an d th e d evelop m en t o a con scien ce are skills th at are develop ed later du rin g
ch ildh ood.
10. The answer is A. Th is wom an is sh owin g evid en ce o a seriou s p ostp artu m reaction su ch
HELP OTHERS SO THAT GOD WILL HELP YOU.

as m ajor dep ression , n ot sim p ly th e “b aby b lu es.” Becau se sh e sh ows sign s o d ep ression ,
or exam p le, early m orn in g awaken in g an d lack o ap p etite, th e n ext step in m an agem en t
is to assess h er or th ou gh ts o su icide. Th e ch ild m u st also be p rotected. I sh e is su icidal
or likely to h arm th e ch ild , in p atien t treatm en t m ay b e in dicated. Ultim ately, assistan ce
with care o th e child m ay be h elp u l, bu t th e irst step is to p rotect th e p atien t an d th e
ch ild. Ju st settin g u p an oth er ap p oin tm en t or th e ollowin g week or p rescrib in g an
an tid ep ressan t will n ot p rotect eith er.
11. The answer is E. Postp artu m blu es m ay occu r in on e-th ird to on e-h al o n ew m oth ers an d
can last u p to 2 weeks. In terven tion in volves su p p ort an d p ractical h elp with th e ch ild.
Brie p sych otic d isord er is rare, occu rrin g in less th an 1% o n ew m oth ers an d lastin g u p to
1 m on th a ter ch ild birth . Postp artu m d ep ression occu rs in 5%–10% o n ew m oth ers an d is
treated p rim arily with an tid ep ressan t m edication .
12. The answer is C. “Did you exp erien ce an y em otion al di icu lties a ter th e birth o you r
oth er ch ildren ?” is an im p ortan t qu estion sin ce a p red ictor o p ostp artu m reaction s is
wh eth er or n ot they h ave occu rred be ore. Th is p atien t is p robably worried becau se sh e
h as h ad p reviou s p roblem s. Reassu rin g statem en ts, su ch as “Most wom en wh o worry
abou t d ep ression n ever exp erien ce it,” “Do n ot worry, th ere are m an y e ective m edication s
or dep ression ,” “Wom en o ten becom e m ore an xiou s toward th e en d o th eir p regn an cy,”
or “Som e dep ression is com m on a ter ch ildb irth ,” do n ot ad d ress th is p atien t’s realistic
con cern s.
13. The answer is B. It is typ ical or a 3-year-old ch ild to h ave di icu lty sittin g still or an y
len gth o tim e. By sch ool age, ch ild ren sh ou ld be able to sit still an d p ay atten tion or
lon ger p eriod s o tim e. Th u s, th is is n ot ADHD. Th ere is also n o eviden ce o delayed
develop m en t, lack o b asic tru st, or sep aration an xiety disorder.
RISE USMLE NEPAL

14. The answer is A. Stran ger an xiety (th e ten den cy to cry an d with draw in th e p resen ce o an
u n am iliar p erson ) develop s in typ ical in an ts at 7–9 m on th s o age. It does n ot in dicate
th at th e ch ild is develop m en tally d elayed , em otion ally distu rbed, or th at th e ch ild h as
been ab u sed b u t rath er th at th e ch ild can n ow d istin gu ish am iliar rom u n am iliar p eop le.
Stran ger an xiety is m ore com m on in ch ildren wh o are cared or by on ly on e p erson an d is
red u ced in th ose exp osed to m an y d i eren t caregivers.
15. The answer is B. Th e b est d escrip tion o th is girl’s b eh avior is typ ical. Her desire to sleep
with h er m oth er is a sign o regression , a de en se m ech an ism th at is com m on in typ ical
ch ildren u n der stress. Becau se sh e con tin u es to p lay well wh en away rom h er m oth er, th is
is n ot sep aration an xiety d isorder. Th ere is also n o eviden ce o delayed develop m en t, lack
o b asic tru st, or ADHD (see Ch ap ter 15).
16. The answer is D. Th e m ost likely d iagn osis or th is ch ild is d isin h ib ited social en gagem en t
disord er. Ch ild ren with th is d isord er orm in discrim in ate attach m en ts to stran gers becau se
th eir p rim ary attach m en t igu re, h ere th e oster m oth er, does n ot in teract n orm ally
with th e ch ild. Mild au tism sp ectru m disorder an d Rett’s disorder are ch aracterized by
decreased , n ot in creased, social in teraction .
14 BRS Behavioral Science

17. The answer is A. Th e p h ysician sh ou ld reassu re th e m oth er th at all ch ild ren are d i eren t
an d th at som e cryin g is n orm al. Th e ch ild’s ap p rop riate weigh t gain an d n egative m edical
in din gs in dicate th at th e ch ild is develop in g typ ically. On ce th e m oth er is reassu red, it will
n ot b e n ecessary to recom m en d a p sych oth erap ist, p rescribe an an tidep ressan t, re er h er
to a p ediatrician sp ecializin g in “di icu lt” in an ts, or recom m en din g th at th e ath er care or
th e ch ild wh en it is cryin g.
18. The answer is B. Th is 4-year-old ch ild is sh owin g a typ ical reaction or h is age. Ch ildren
u n d er th e age o 6 years d o n ot u n d erstan d th e in ality o death an d u lly exp ect dead
p eop le to com e back to li e. Th at is wh y, alth ou gh h e h as been told th at h is ath er h as died,
th is ch ild rep eatedly asks or h is ath er. Wh ile h e h as been severely stressed, h e is n eith er
sim p ly re u sin g to believe th e tru th n or sh owin g delayed develop m en t. Wh ile it is p ossible
HELP OTHERS SO THAT GOD WILL HELP YOU.

th at th is boy h as an u n diagn osed h ead in ju ry, a typ ical reaction is m ore likely.
19. The answer is C. Tran s errin g objects rom h an d to h an d com m on ly occu rs at abou t 10
m on th s o age.
20. The answer is B. In an ts can u su ally tu rn over at abou t 5 m on th s o age.
21. The answer is A. Ch ild ren b egin to sh ow social sm ilin g between 1 an d 2 m on th s o age.
22. The answer is C. Ch ildren begin to resp on d to th eir own n am es between 7 an d 11 m on th s
o age.
23. The answer is E. Ch ildren b egin to u se a u ten sil to eed th em selves at abou t 2 years o age.
24. The answer is E. Ch ildren b egin to m ake m arks (scrib b le) on p ap er at ab ou t 18 m on th s o
age.
RISE USMLE NEPAL
Sch ool Age, Adolescen ce,
Sp ecial Issu es o
c ha pte r
2 Develop m en t, an d
Adu lth ood
HELP OTHERS SO THAT GOD WILL HELP YOU.

Typical Board Question


Th e m oth er o a 12-year-old b oy com es h om e early rom work an d in ds h er son takin g a
sh ower with an oth er 12-year-old boy. Th e m oth er is very u p set an d im m ediately calls th e
p ediatrician . At th is tim e, th e doctor sh ou ld
(A) ask h er to com e in with h er son as soon as p ossib le
(B) ask h er to brin g h er son in an d th en let th e d octor sp eak to h im alon e
(C) reassu re th e m oth er th at th is beh avior is typ ical
(D) test th e b oy’s an d rogen levels
(E) tell th e m oth er th at th e b oy will p robably h ave a h om osexu al orien tation in
adu lth ood
(See “An sw ers an d Explan ation s” at th e en d of th e ch apter.)

I. SCHOOL AGE: 7–11 YEARS


A. Social characteristics. Th e sch ool-age ch ild :
1. Pre ers to in teract with children of the same gender; typ ically avoids an d is critical o th ose
RISE USMLE NEPAL

o th e op p osite sex.
2. Iden ti ies with th e p aren t o th e sam e gen der.
3. Has relation sh ip s with adults other than parents (e.g., teach ers, grou p lead ers).
4. Dem on strates little in terest in p sych osexu al issu es (sexu al eelin gs are laten t an d will
reap p ear at p u berty).
5. Has in tern alized a moral sense of right and wrong (conscience) an d u n derstan ds h ow to
ollow ru les (e.g., p layin g “ air”).
6. Gradu ally begin s to u n derstan d th at death is irreversible and final.
7. A ter ab ou t age 11, th e ch ild u n d erstan d s th at death is universal and inevitable .
8. Is typ ically in terviewed an d exam in ed by th e doctor with the mother present.

B. Cognitive characteristics. Th e sch ool-age ch ild :


1. Is industrious an d organ ized (e.g., gath ers collection s o ob jects).
2. Has th e cap acity or logical thought an d can determ in e th at objects h ave m ore th an on e
p rop erty (e.g., an object can b e b oth red an d m etal).

15
16 BRS Behavioral Science

3. Un derstan ds th e con cep ts o conservation and seriation; both are n ecessary or certain
typ es o learn in g.
a. Conservation in volves th e u n derstan din g th at a qu an tity o a su bstan ce rem ain s th e
sam e regardless o th e size o th e con tain er or sh ap e it is in (e.g., two con tain ers m ay
con tain th e sam e am ou n t o water, even th ou gh on e is a tall, th in tu be an d on e is a
sh ort, wid e bowl).
b. Seriation in volves th e ab ility to arran ge objects in order with resp ect to th eir sizes or
oth er qu alities.

C. Motor development. Th e typ ical sch ool-age ch ild en gages in com p lex m otor tasks (e.g., p lays
b aseb all, skip s rop e, rid es a two-wh eeled b icycle).
HELP OTHERS SO THAT GOD WILL HELP YOU.

II. ADOLESCENCE: 11–20 YEARS


A. Early adolescence (11–14 years of age)
1. Puberty occu rs in early adolescen ce an d is m arked by:
a. Th e develop m en t o secondary sex characteristics (Table 2.1) an d in creased skeletal
growth .
b. First menstruation (m en arch e) in girls, wh ich on average occu rs at 11–14 years o age.
c. First ejaculation in b oys, wh ich on average occu rs at 12–15 years o age.
d. Cognitive maturation an d formation of the personality.
e. Sex drives , wh ich are exp ressed th rou gh physical activity an d masturbation (daily m as-
tu rbation is typ ical).
f. Sexual practicing b eh avior with sam e- or op p osite-sex p eers.
2. Early adolescen ts sh ow stron g sen sitivity to th e op in ion s o p eers bu t are gen erally obedi-
en t an d u n likely to seriou sly ch allen ge p aren tal au th ority.
3. Alterations in expected patterns of development (e.g., acn e, obesity, late breast develop m en t
in girls, n ip p le en largem en t in boys [u su ally tem p orary bu t m ay con cern th e boy an d h is
p aren ts]) m ay lead to p sych ological d i icu lties.

B. Middle adolescence (15–17 years of age)


1. Characteristics
a. Th ere is great in terest in gender roles, body image, and popularity.
b. Heterosexu al or h om osexu al crushes (love or an u n attain able p erson su ch as a rock
star) are com m on .
RISE USMLE NEPAL

c. E orts to develop an identity by adop tin g cu rren t teen ash ion in cloth in g an d m u sic
an d p re eren ce or sp en din g tim e with p eers over am ily are typ ical, bu t m ay lead to
con lict with p aren ts.

t a b l e 2.1 Tanner Stages of Sexual Development


Stage Characteristics
1 Genitalia and associated structures are the same as in childhood; nipples (papillae) are slightly elevated
in girls
2 Scant, straight pubic hair, testes enlarge, scrotum develops texture; slight elevation of breast tissue in
girls
3 Pubic hair increases over the pubis and becomes curly, penis increases in length, and testes enlarge
4 Penis increases in width, glans develops, scrotal skin darkens; areola rises above the rest of the breast
in girls
5 Male and female genitalia are like adult; pubic hair now is also on the thighs, areola is no longer
elevated above the breast in girls
Chapter 2 School Age, Adolescence, Special Issues of Development, and Adulthood 17

2. Risk-taking behavior
a. Readin ess to ch allen ge p aren tal rules an d eelin gs o omnipotence m ay result in risk-taking
behavior (e.g., ailure to use con dom s, drivin g too ast, sm okin g cigarettes or m arijuan a).
b. Edu cation ab ou t obvious short-term benefits rath er th an re eren ces to lon g-term con se-
qu en ces o beh avior is m ore likely to decrease teenagers’ unwanted behavior. For exam -
p le, to discourage smoking cigarettes , tellin g teen agers th at th eir teeth will stay wh ite
i th ey d o n ot sm oke, or th at oth er teen s in d sm okin g d isgu stin g, will b e m ore h elp u l
th an tellin g th em th at th ey will avoid lu n g can cer in 30 years.

C. Late adolescence (18–20 years of age)


1. Development
a. Older ad olescen ts d evelop morals, ethics, self-control, an d a realistic ap p raisal o th eir
HELP OTHERS SO THAT GOD WILL HELP YOU.

own ab ilities; th ey becom e con cern ed with h u m an itarian issu es an d world p rob lem s.
b. Som e adolescen ts, b u t n ot all, develop th e ability or abstract reason in g (Piaget’s stage
of formal operations ).
2. In th e e ort to orm on e’s own iden tity, an identity crisis com m on ly develop s.
a. I th e id en tity crisis is n ot h an dled e ectively, adolescen ts m ay exp erien ce role confu-
sion in wh ich th ey do n ot kn ow wh ere th ey b elon g in th e world.
b. Exp erien cin g role con u sion , th e ad olescen t m ay d isp lay b eh avioral p roblem s su ch as
criminality or an interest in cults .

D. Teenage sexuality
1. In th e Un ited States, first sexual intercourse occu rs on average at 16 years o age; by 19
years o age, m ost m en an d wom en h ave h ad sexu al in tercou rse.
2. Fewer th an h al o all sexu ally active teen agers do not use contraceptives or reason s th at
in clu d e th e con viction th at th ey will n ot get p regn an t, lack o access to con tracep tives,
an d lack o ed u cation ab ou t wh ich m eth ods are m ost e ective.
3. Ph ysician s m ay cou n sel m in ors (p erson s u n der 18 years o age) an d p rovide th em with
con tracep tives with ou t p aren tal kn owledge or con sen t. Th ey m ay also p rovide to m in ors
treatm en t or sexu ally tran sm itted diseases, p roblem s associated with p regn an cy, an d
dru g an d alcoh ol ab u se (see Ch ap ter 23).
4. Becau se o th eir p oten tial sen sitivity, issu es in volvin g sexu ality an d dru g abu se, as well as
issu es con cern in g p h ysical ap p earan ce su ch as obesity, are typ ically discu ssed with teen -
agers without a parent present. Ph ysical exam in ation is d on e with ou t a p aren t p resen t b u t
with a chaperone p resen t.

E. Teenage pregnancy
1. Teen age p regn an cy is a social p roblem in th e Un ited States, alth ou gh th e birth rate and
RISE USMLE NEPAL

abortion rate in Am erican teen agers have been declining. In con trast, th e birth rate am on g
wom en 35–44 h as been in creasin g (Figu re 2.1).

200 200
p
25–29
u
o
r
100 100
g
e
20–24
g
30–34
a
50 50
d
e
i
f
i
c
e
35–39 15–19
p
s
n
i
40–44
n
10
e
10
m
o
w
5 5
0
0
0
,
1
FIGURE 2.1. Birth rates in the United States by selected age of mother:
r
e
Final 1990–2012 and preliminary 2013. (Reprinted from Hamilton BE,
p
e
Martin J A, Osterman MJ K, et al. Division of Vital Statistics. Births:
t
a
R
Preliminary data for 2013. Natl Vital Stat Rep. 2014;63(2).) Rates are 1 1
plotted on a logarithmic scale. 1990 1995 2000 2005 2010 2013
18 BRS Behavioral Science

2. Abortion is legal in th e Un ited States. However, in m an y states, m in ors m u st obtain p aren -


tal con sen t or abortion .
3. Factors p redisp osin g adolescen t girls to p regn an cy in clu de dep ression , p oor sch ool
ach ievem en t, an d h avin g d ivorced p aren ts.
4. Pregn an t teen agers are at h igh risk or obstetric complications becau se th ey are less likely
to get p ren atal care an d becau se th ey are p h ysically im m atu re.

III. SPECIAL ISSUES IN CHILD DEVELOPMENT


HELP OTHERS SO THAT GOD WILL HELP YOU.

A. Illness in childhood and adolescence. A ch ild’s reaction to illn ess is closely associated with
th e ch ild’s develop m en tal stage.
1. Du rin g th e toddler years (15 m on th s–2½ years), ill, h osp italized ch ildren fear separation
rom th e p aren t m ore th an th ey ear b od ily h arm or p ain .
2. Du rin g th e preschool years (2½ –6 years), th e ch ild’s greatest ear wh en h osp italized is o
bodily harm.
3. School-age children (7–11 years o age) cop e relatively well with h osp italization . Th u s, th is
is th e best age to perform elective surgery.
4. Ill adolescents m a y ch a llen ge th e a u th o rity o d o cto rs a n d n u rses a n d resist b ein g
d i eren t ro m p eers. Bo th o th ese a cto rs ca n resu lt in lack of adherence to medical
advice .
5. A ch ild with an ill sibling or p aren t m ay resp on d by actin g bad ly at sch ool or h om e (u se o
th e de en se m ech an ism o “acting out” [see Ch ap ter 6, Section II]).

B. Adoption
1. An adoptive parent is a p erson wh o volu n tarily becom es th e legal parent o a ch ild wh o is
n ot h is or h er gen etic o sp rin g.
2. Adop ted ch ild ren , p articu larly th ose adop ted a ter in an cy, m ay b e at in creased risk or
beh avioral p rob lem s in ch ild h ood an d ad olescen ce.
3. Ch ild ren should be told by th eir p aren ts th at th ey are ad op ted at the earliest age possible to
avoid th e ch an ce o oth ers tellin g th em irst.

C. Intellectual disability
1. Etiology
a. Th e m ost com m on gen etic cau ses o in tellectu al disability are Down’s syndrome and
fragile X syndrome .
RISE USMLE NEPAL

b. Oth er cau ses in clu de m etabolic actors a ectin g th e m oth er or etu s, p ren atal an d
p ostn atal infection (e.g., ru bella), an d maternal substance use ; m an y cases o in tellec-
tu al disability are o u n kn own etiology.
2. Mildly an d m oderately in tellectu ally disab led ch ildren an d adolescen ts com m on ly know
they are different (see Ch ap ter 8). Becau se o th is, th ey m ay b ecom e frustrated and socially
withdrawn. Th ey m ay h ave p oor sel -esteem becau se it is di icu lt or th em to com m u n i-
cate an d com p ete with p eers.
3. Th e Vineland Social Maturity Scale (see Ch ap ter 8) can b e u sed to evalu ate social skills an d
skills or d aily livin g in in tellectu ally d isabled an d oth er ch allen ged in dividu als.
4. Avoidance of pregnancy in adu lts with in tellectu al disabilities can becom e an issu e, p ar-
ticu larly in resid en tial social settin gs (e.g., su m m er cam p ). Long-acting, reversible contra-
ceptive methods su ch as su bcu tan eou s p rogesteron e im p lan ts can be p articu larly u se u l
to th ese in dividu als.
Chapter 2 School Age, Adolescence, Special Issues of Development, and Adulthood 19

IV. EARLY ADULTHOOD: 20–40 YEARS


A. Characteristics
1. At ab ou t 30 years of age , th ere is a period of reappraisal o on e’s li e.
2. Th e adu lt’s role in society is defined an d p h ysical develop m en t p eaks, an d th e adu lt
becom es in d ep en den t.

B. Responsibilities and relationships


1. The developm en t o an intimate (e.g., close, sexual) relationship with an oth er p erson occu rs.
2. Accord in g to Erikson, th is is th e stage o intimacy versus isolation; i th e in dividu al does n ot
HELP OTHERS SO THAT GOD WILL HELP YOU.

d evelop th e ability to su stain an in tim ate relation sh ip by th is stage o li e, h e or sh e m ay


exp erien ce em otion al isolation in th e u tu re.
3. By 30 years o age, m ost Am erican s are in a com m itted relation sh ip, or exam p le, m arriage
an d h ave ch ildren .
4. Du rin g th eir m iddle 30s, m an y wom en alter th eir li estyles by retu rn in g to work or sch ool
or by resuming their careers .

V. MIDDLE ADULTHOOD: 40–65 YEARS


A. Characteristics. Th e p erson in m id dle adu lth ood p ossesses m ore p ower an d au th ority th an
at oth er li e stages.

B. Responsibilities. Th e in d ivid u al eith er m ain tain s a con tin u ed sen se o p rodu ctivity or devel-
op s a sen se o em p tin ess (Erikson’s stage o gen erativity vs. stagn ation ).

C. Relationships
1. Man y m en in th eir m iddle 40s or early 50s exh ibit a midlife crisis . Th is m ay lead to:
a. A ch an ge in p ro ession or li estyle.
b. In id elity, sep aration , or d ivorce.
c. In creased u se o alcoh ol or oth er d ru gs.
d. Dep ression .
2. Midli e crisis is associated with an awareness of one’s own aging an d death an d severe or
u n exp ected li estyle ch an ges (e.g., death o a sp ou se, loss o a job, seriou s illn ess).
RISE USMLE NEPAL

D. Climacterium is th e ch an ge in p h ysiologic u n ction th at occu rs du rin g m id li e.


1. In men, decreased m u scle stren gth , p h ysical en du ran ce, an d sexu al p er orm an ce (see
Ch ap ter 18) occu r in m idli e.
2. In women, menopause occu rs.
a. Th e ovaries stop u n ction in g, an d m en stru ation stop s in th e late 40s or early 50s .
b. Absen ce o m en stru ation or 1 year de in es the end of menopause . To avoid u n wan ted
p regn an cy, con tracep tive m easu res sh ou ld be u sed u n til at least 1 year following the
last missed menstrual period.
c. Most wom en exp erien ce m en op au se with relatively ew p h ysical or p sych ological
p roblem s.
d. Vasom otor in stab ility, called hot flashes or flushes , is a com m on p h ysical p roblem seen
in wom en in all cou n tries an d cu ltu ral grou p s an d m ay con tin u e or years. Wh ile estro-
gen or estrogen / p rogesteron e rep lacem en t th erapy can relieve th is sym p tom , u se o
su ch th erapy h as d ecreased b ecau se it is associated with in creased risk o u terin e an d
breast can cer.
Review Test

Directions: Each o th e n u m b ered item s or in com p lete statem en ts in th is section is ollowed by


an swers or by com p letion s o th e statem en t. Select th e one lettered an swer or com p letion th at
is best in each case.
HELP OTHERS SO THAT GOD WILL HELP YOU.

1. A 15-year-old girl is brou gh t to th e doctor 4. A 52-year-old wom an in th e Un ited States


by h er m oth er becau se sh e is in sistin g h as a 52-year-old em ale rien d in Au stralia.
on gettin g a tattoo. Th e teen ager states Both are in good gen eral h ealth an d n eith er
th at sh e kn ows th ere is a risk o h u m an h as m en stru ated or abou t 1 year. Wh ich o
im m u n odef cien cy viru s (HIV) in ection bu t th e ollowin g sym p tom s are both wom en
wan ts to get th e tattoo an yway. Wh at is th e m ost likely to exp erien ce at th is tim e?
doctor’s best n ext step in m an agem en t? (A) Severe dep ression
(A) Say “I stron gly recom m en d th at you n ot (B) Severe an xiety
get th e tattoo” (C) Hot lash es
(B) Say “Let’s talk ab ou t th e p ros an d con s o (D) Fatigu e
gettin g a tattoo” (E) Leth argy
(C) Ask “I you kn ow th ere are risks, wh y d o
you wan t th e tattoo?” 5. In crease in p en is width , develop m en t
(D) Say “Tattoos are p erm an en t an d can o th e glan s, an d d arken in g o scrotal skin
rarely be com p letely rem oved” ch aracterize Tan n er stage
(E) Give th e teen a b roch u re d escribin g th e (A) 1
h ealth risks o tattoos (B) 2
(C) 3
2. A p h ysician discovers th at a 15-year-old (D) 4
p atien t is p regn an t. Wh ich o th e ollowin g (E) 5
actors is likely to h ave con tribu ted m ost to
h er risk o p regn an cy? 6. A m oth er tells th e p h ysician th at sh e
(A) Livin g in a ru ral area is con cern ed abou t h er son becau se h e
(B) Dep ressed m ood con sisten tly en gages in beh avior th at is
(C) In tact p aren tal u n it dan gerou s an d p oten tially li e th reaten in g.
(D) High ach ievem en t in sch ool Th e age o h er son is m ost likely to be
RISE USMLE NEPAL

(E) Havin g received in orm ation ab ou t (A) 11 years


con tracep tive m eth od s (B) 13 years
(C) 15 years
3. A 50-year-old m ale p atien t p resen ts or an (D) 18 years
in su ran ce p h ysical. Wh ich o th e ollowin g (E) 20 years
develop m en tal sign p osts is m ost likely to
ch aracterize th is m an ? 7. A p h ysician is con du ctin g a sch ool
p h ysical on a typ ical 10-year-old girl. Wh en
(A) Decreased alcoh ol u se
in terviewin g th e ch ild, th e p h ysician is
(B) Peak p h ysical develop m en t
m ost likely to f n d wh ich o th e ollowin g
(C) Possession o p ower an d au th ority
p sych ological ch aracteristics?
(D) Stron g resistan ce to ch an ges in social
relation sh ip s (A) Lack o con scien ce orm ation
(E) Stron g resistan ce to ch an ges in work (B) Poor cap acity or logical th ou gh t
relation sh ip s (C) Iden ti ication with h er ath er
(D) Relatively stron ger im p ortan ce o rien ds
over am ily wh en com p ared to ch ildren
o you n ger ages
(E) No p re eren ce with resp ect to th e sex o
p laym ates
20
Chapter 2 School Age, Adolescence, Special Issues of Development, and Adulthood 21

8. A ch ild’s p et h as recen tly died . Th e ch ild 12. A wom an an d her 15-year-old daughter
believes th at th e p et will soon com e b ack to p resen t to the physician’s o f ce together. The
li e. Th is ch ild is m ost likely to be o age m other asks the physician to f t her daughter
(A) 4 years or a diaphragm . The m ost appropriate action
(B) 6 years or the p hysician to take at this tim e is to
(C) 7 years (A) ollow th e m oth er’s wish es
(D) 9 years (B) ask th e m oth er wh y sh e wan ts a
(E) 11 years diap h ragm or h er d au gh ter
(C) recom m en d th at th e girl see a cou n selor
9. A 10-year-old girl with Down’s syn drom e (D) ask to sp eak to th e girl alon e
an d an IQ o 60 is b rou gh t to th e p h ysician’s (E) ask th e girl i sh e is sexu ally active
HELP OTHERS SO THAT GOD WILL HELP YOU.

o f ce or a sch ool p h ysical. Wh en th e doctor


in terviews th is girl, h e is m ost likely to f n d 13. A p h ysician is asked to evalu ate th e
th at sh e develop m en t o an 11-year-old girl. Wh ich
(A) h as good sel -esteem o the ollowin g m ileston es is u su ally n ot
(B) kn ows th at sh e is develop m en tally acquired u n til a ter th e age o 11 years?
delayed (A) Th e con cep t o seriation
(C) com m u n icates well with p eers (B) Th e con cep t o con servation
(D) com p etes su ccess u lly with p eers (C) Parallel p lay
(E) is socially ou tgoin g (D) Th e orm ation o a p erson al iden tity
(E) An u n d erstan d in g o th e con cep t o “ air
10. A 15-year-old boy tells h is p h ysician th at p lay”
h e h as b een sm okin g cigarettes or th e p ast
year. He relates th at h is rien d s sm oke an d 14. A girl tells h er m oth er th at sh e “h ates th e
h is ath er sm okes. Th e m ost likely reason boys becau se th ey are n oisy an d stu p id.” Th e
th at th is teen ager d oes n ot attem p t to stop age o th is girl is m ost likely to be
sm okin g is becau se (A) 4 years
(A) h e is dep ressed (B) 6 years
(B) h is ath er sm okes (C) 9 years
(C) h is p eers sm oke (D) 13 years
(D) h e does n ot kn ow th at sm okin g is (E) 15 years
h arm u l
(E) sm okin g is add ictive 15. At the lun ch table, a ch ild asks h is m oth er
to cut h is h ot d og u p in to th ree p ieces so th at
11. A orm erly ou tgoin g 10-year-old b oy h e can have three tim es as m uch to eat. Th e
b egin s to d o p oorly in sch ool a ter h is 6-year- age o th is ch ild is m ost likely to be
old broth er is diagn osed with leu kem ia. He (A) 4 years
RISE USMLE NEPAL

n ow p re ers to watch television alon e in h is (B) 6 years


room an d d oes n ot wan t to socialize with (C) 9 years
h is rien d s. His p aren ts are very stressed (D) 13 years
by carin g or th e you n ger ch ild b u t d o (E) 15 years
n ot ask th e old er ch ild or h elp. Th e m ost
ap prop riate su ggestion or th e doctor to 16. A 15-year-old overweigh t girl an d h er
m ake with resp ect to th e 10-year-old is to tell m oth er com e to see th e doctor or advice
th e p aren ts to abou t diet an d exercise. Th e m oth er states
(A) in sist th at h e take m ore resp on sibility or th at sh e d oes n ot kn ow wh y th e girl is
carin g or h is you n ger broth er overweigh t becau se sh e cooks th e sam e ood
(B) ign ore h is beh avior or her an d h er slim 16-year-old broth er. Th e
(C) rem ove th e television rom h is room doctor sh ou ld f rst
(D) p ay m ore atten tion to h im (A) talk to th e m oth er alon e
(E) tell h im n ot to worry, everyth in g will b e (B) talk to both th e teen s with th e m oth er
in e p resen t
(C) talk to th e girl with th e m oth er p resen t
(D) talk to th e m oth er, th e broth er, an d th e
girl togeth er
(E) talk to th e girl alon e
22 BRS Behavioral Science

17. A m ed ical stu den t on a su rgery rotation 19. A m oth er worried ly rep orts th at h er
is assign ed to stay with a 9-year-old girl wh o 7-year-old son is o ten dirty wh en h e com es
is waitin g to h ave su rgery to rep air a cle t in rom p layin g. Sh e n otes th at h e d igs in
p alate. Th e girl, wh o h as recen tly arrived th e dirt, wip es h is ace with h is d irty h an d s,
alon e rom Laos, does n ot sp eak En glish an d an d clim bs trees ou tside o th e h om e. Sh e
ap pears an xiou s. Th e h osp ital adm in istrator states th at sh e is worried th at h e will catch
h as requ ested a tran slator wh o h as n ot yet a disease or in ju re h im sel . Th e m oth er also
arrived. At th is tim e, th e m ost ap p rop riate rep orts th at sh e h ad a m eetin g with th e
action or th e m edical stu den t to take is to ch ild’s teach er wh o told h er th at th e ch ild
(A) sedate th e ch ild to d ecrease h er an xiety is doin g well in sch ool. Th e n ext step in
(B) give th e ch ild a toy to keep h er occu p ied m an agem en t is or th e doctor to
HELP OTHERS SO THAT GOD WILL HELP YOU.

(C) su ggest th at th e n u rse stay with th e ch ild (A) sp eak with th e ch ild’s teach er
so th at h e can review h er ch art (B) sp eak with th e ch ild
(D) look in th e ch ild’s ears with an otoscop e (C) say “He m u st be h ard to h an d le”
(E) listen to th e ch ild’s h eart with a (D) say “Tell m e m ore abou t you r con cern s
steth oscop e an d th en let th e ch ild try regardin g you r son ?”
u sin g th e steth oscop e to listen to h is (E) say “He is in e, d o n ot worry”
h eart

18. A p h ysician is sch edu led to see


8-year-old an d 15-year-old sisters or
rou tin e ch ecku p s. Th ey h ad con secu tive
ap poin tm en ts, bu t wh en th e doctor en ters
th e exam in in g room , th ey are b oth th ere
with th eir m oth er. Most ap p rop riately th e
doctor sh ou ld
(A) ask th e 15-year-old to leave an d talk to
th e 8-year-old with th e m oth er p resen t
an d th en , talk to th e 15-year-old alon e
(B) ask b oth girls to leave an d talk to th e
m oth er alon e an d th en , ask th e m oth er
to leave an d talk to th e two girls togeth er
(C) ask b oth girls to leave, talk to th e m oth er
alon e, an d th en ask th e m oth er to com e
back in an d talk to all th ree togeth er
(D) ask th e m oth er to leave, talk to both girls
togeth er an d th en talk to th e m oth er
RISE USMLE NEPAL

alon e
(E) ask th e m oth er an d th e old er girl to
leave, talk to th e you n ger ch ild alon e,
an d th en talk to th e old er girl alon e
An swers an d Exp lan ation s

Typical Board Question


The answer is C. Th e d octor sh ou ld reassu re th e m oth er th at sexu al p racticin g b eh avior with
sam e- or op p osite-sex p eers is com m on an d typ ical. Th ere ore, it is n ot n ecessary to talk to th e
boy or test h is an d rogen levels. Th e boy m ay h ave a h om osexu al or a h eterosexu al orien tation in
HELP OTHERS SO THAT GOD WILL HELP YOU.

adu lth ood . Eith er orien tation is typ ical.

1. The answer is B. Sayin g “Let’s talk abou t th e p ros an d con s o gettin g a tattoo” will
en cou rage th e girl to talk abou t h er m otivation or gettin g th e tattoo. Th e cu rren t risk o
gettin g th e tattoo or p roblem s with rem oval o th e tattoo in th e u tu re p robably are n ot as
im p ortan t at th is tim e as wh y sh e wan ts it so badly. Sayin g “I stron gly recom m en d th at you
n ot get th e tattoo” or criticizin g h er by sayin g “I you kn ow th ere are risks, wh y do you wan t
th e tattoo?” will n ot b e e ective. Ju st givin g h er a broch u re also will n ot be h elp u l; m ost
likely it will be ign ored.
2. The answer is B. Teen agers wh o becom e p regn an t requ en tly are dep ressed, com e rom
h om es wh ere th e p aren ts are d ivorced, h ave p roblem s in sch ool, an d m ay n ot kn ow abou t
e ective con tracep tive m eth od s. Stu d ies h ave n ot in dicated th at livin g in a ru ral area is
related to teen age p regn an cy.
3. The answer is C. Wh ile m idli e is associated with th e p ossession o p ower an d au th ority,
p h ysical abilities declin e. Th is tim e o li e is also associated with a m idli e crisis, wh ich m ay
in clu de in creased alcoh ol an d dru g u se as well as an in creased likelih ood o ch an ges in
social an d work relation sh ip s.
4. The answer is C. Th ese 52-year-old wom en in good gen eral h ealth are goin g th rou gh
m en op au se. Th e m ost com m on sym p tom o m en op au se occu rrin g cross-cu ltu rally is
h ot lash es, a p u rely p h ysiological p h en om en on . In m ost wom en , m en op au se is n ot
ch aracterized by p sych op ath ology su ch as severe dep ression or an xiety or p h ysical
sym p tom s like atigu e an d leth argy.
5. The answer is D. In crease in p en is width , develop m en t o th e glan s, an d darken in g o scrotal
skin ch aracterize Tan n er stage 4. Stage 1 is ch aracterized by sligh t elevation o th e p ap illae,
RISE USMLE NEPAL

an d stage 2 by th e p resen ce o scan t, straigh t p u bic h air, testes en largem en t, develop m en t


o textu re in scrotal skin , an d sligh t elevation o breast tissu e. In stage 3, p u bic h air in creases
over th e p u b is an d becom es cu rly, an d th e p en is in creases in len gth ; in stage 5, m ale an d
em ale gen italia are m u ch like th ose o adu lts.
6. The answer is C. Th e age o th is wom an’s son is m ost likely to be 15 years. Middle
adolescen ts (15–17 years) o ten ch allen ge p aren tal auth ority an d h ave eelin gs o
om n ip oten ce (e.g., n oth in g bad will h ap p en to th em becau se th ey are all p ower u l). You n ger
adolescen ts (11–14 years) are u n likely to ch allen ge p aren tal rules an d au th ority. Older
adolescen ts (18–20 years) h ave develop ed sel -con trol an d a m ore realistic p ictu re o th eir
own abilities.
7. The answer is D. Wh en com p ared to you n ger ages, p eers an d n on am ilial adu lts becom e
m ore im p ortan t to th e laten cy-age ch ild an d th e am ily becom es less im p ortan t. Ch ildren
7–11 years o age h ave th e cap acity or logical th ou gh t, h ave a con scien ce, iden ti y with th e
sam e-sex p aren t, an d sh ow a stron g p re eren ce or p laym ates o th eir own sex.
8. The answer is A. Presch ool ch ildren u su ally can n ot com p reh en d th e m ean in g o death an d
com m on ly believe th at th e dead p erson or p et will com e back to li e. Ch ildren over th e age
o 6 years com m on ly are aware o th e in ality o death (see Ch ap ter 1).
23
24 BRS Behavioral Science

9. The answer is B. Mild ly an d m od erately in tellectu ally disab led ch ild ren are aware th at th ey
h ave a d evelop m en tal d elay. Th ey o ten h ave low sel -esteem an d m ay becom e socially
with drawn . In p art, th ese p roblem s occu r becau se th ey h ave di icu lty com m u n icatin g
with an d com p etin g with p eers.
10. The answer is C. Peer p ressu re h as a m ajor in lu en ce on th e b eh avior o ad olescen ts wh o
ten d to do wh at oth er adolescen ts are doin g. Dep ression , th e sm okin g beh avior o th eir
p aren ts, an d th e ad d ictive qu ality o cigarettes h ave less o an in lu en ce. Most teen agers
h ave been ed u cated with resp ect to th e d an gers o sm okin g.
11. The answer is D. The doctor should rem ind the parents to pay m ore attention to the older
child. The child is likely to be righten ed by his younger sibling’s illness and the attitudes o
HELP OTHERS SO THAT GOD WILL HELP YOU.

his paren ts toward the youn ger child. School-age children such as this one m ay becom e
withdrawn or “act out” by showin g bad behavior when ear ul or depressed. While he can be
included in the care o his brother, it is not appropriate to insist that he take m ore responsibility
or him . Ign orin g his behavior or punishin g him can increase his ear and withdrawal. False
reassuran ce such as tellin g the child that everything will be ine is not appropriate.
12. The answer is D. Th e m ost ap p rop riate action or th e p h ysician to take at th is tim e is to ask
to sp eak to th e girl alon e. Th e p h ysician can th en ask th e girl abou t h er sexu al activity an d
p rovid e con tracep tives an d cou n selin g i sh e wish es, with ou t n oti ication or con sen t rom
th e m oth er. Th e m oth er’s wish es in th is circu m stan ce are n ot relevan t to th e p h ysician’s
action ; th e teen ager is th e p atien t.
13. The answer is D. Th e orm ation o a p erson al id en tity is u su ally ach ieved d u rin g th e
teen age years. Th e con cep ts o seriation an d con servation an d an u n derstan din g o th e
con cep t o “ air p lay” are gain ed du rin g th e sch ool-age years. Parallel p lay is u su ally seen
between ages 2 an d 4 years.
14. The answer is C. Laten cy-age ch ild ren (age 7–11 years) h ave little in terest in th ose o th e
op p osite sex an d o ten criticize or avoid th em . In con trast, you n ger ch ildren do n ot sh ow
stron g gen der p re eren ces or p laym ates, an d teen agers com m on ly seek th e com p an y o
op p osite-sex p eers.
15. The answer is A. Th is ch ild is m ost likely to b e 4 years o age. Presch ool ch ildren do n ot yet
u n d erstan d th e con cep t o con servation (i.e., th at th e qu an tity o a su bstan ce rem ain s th e
sam e regard less o th e sh ap e th at it is in ). Th u s, th is ch ild believes th at a h ot dog cu t in to
th ree p ieces h as m ore in it th an wh en it was in on ly on e p iece. Ch ildren u n derstan d th is
con cep t better as th ey ap p roach sch ool age.
16. The answer is E. As in Qu estion 12, th e p h ysician sh ou ld talk to th is 15-year-old girl alon e.
RISE USMLE NEPAL

In ad dition to sexu al an d d ru g u se issu es, th ose th at in volve body im age su ch as body


weigh t ideally sh ou ld be discu ssed with a teen ager alon e, with ou t oth er am ily m em bers
p resen t.
17. The answer is E. Th e b est th in g or th e m edical stu d en t to do at th is tim e is to in teract
with th e ch ild . Sin ce th ey d o n ot sp eak th e sam e lan gu age, in volvin g ch ildren o th is age
in an in teractive activity su ch as u sin g th e steth oscop e or drawin g p ictu res togeth er is th e
b est ch oice h ere. Neith er givin g th e ch ild a toy n or lookin g in h er ears is an in teractive
activity. Th e stu den t, n ot th e n u rse, is resp on sible or th e ch ild in th is in stan ce. Sedation is
in ap p rop riate at th is tim e; social activity is o ten e ective in decreasin g a p atien t’s an xiety.
18. The answer is A. Paren ts sh ou ld b e p resen t wh en a p h ysician sp eaks to a you n ger ch ild,
bu t teen agers u su ally sh ou ld b e in terviewed, p articu larly ab ou t sexu al issu es, with ou t
p aren ts p resen t. Th u s, th e d octor sh ou ld ask th e 15-year-old to leave an d talk to th e 8-year-
old with th e m oth er p resen t. Th en , th e d octor sh ou ld talk to th e 15-year-old alon e.
19. The answer is D. Alth ou gh th is boy is p robably sh owin g typical behavior or a 6-year-old,
the doctor n eeds to kn ow m ore abou t th is m other’s con cern s regardin g her son . Sin ce h e is
doin g well in sch ool; th ere is n o n eed to sp eak to th e teach er or th e ch ild. Sim p ly sayin g “he
m u st be h ard to h an dle” or “h e is in e, do n ot worry” will n ot address the m other’s con cern s.
Agin g, Death , an d
c ha pte r
3 Bereavem en t
HELP OTHERS SO THAT GOD WILL HELP YOU.

Typical Board Question


A orm erly well-groom ed 70-year-old p atien t h as ap p eared u n sh aven an d dish eveled sin ce
th e death o h is wi e 8 m on th s ago. He h as lost 20 p ou n ds, h as p ersisten t p roblem s sleep in g,
an d h as n o in terest in in teractin g with rien ds an d am ily. He also h as d i icu lty relatin g wh at
h e did th e p reviou s day or wh at h e ate or lu n ch today. Ph ysical exam in ation an d laboratory
tests are u n rem arkable. For th is p atien t, th e best recom m en dation o th e p h ysician is
(A) m ed ication or sleep
(B) evalu ation or m ajor d ep ression
(C) regu lar p h on e calls an d visits to “ch eck in” with th e doctor
(D) p sych oth erapy
(E) n eu rop sych ological evalu ation or Alzh eim er’s disease
(See “An sw ers an d Explan ation s” at th e en d of th e ch apter.)

I. AGING
A. Demographics
1. By 2020, m ore th an 15% o US p op u lation will be m ore th an 65 years o age.
2. Th e astest growin g segm en t o th e p op u lation is p eop le over age 85.
RISE USMLE NEPAL

3. Di eren ces in li e exp ectan cies by gen d er an d eth n icity (Figu re 3.1) h ave been decreasin g
over th e p ast ew years.
4. Gerontology, th e stu dy o agin g, an d geriatrics , th e care o agin g p eop le, h ave becom e
im p ortan t m edical ields.
a. Geriatrician s typ ically manage rather than cure th e ch ron ic illn ess o agin g su ch as
h yp erten sion , can cer, an d d iab etes.
b. A m ajor aim o geriatrics is to keep elderly p atien ts m obile an d active. Becau se fractures
(e.g., o th e h ip ) are m ore likely th an ch ron ic illn ess to cau se loss of mobility lead in g to
disab ility an d d eath in th e elderly, preventing falls an d p reven tion an d m an agem en t o
osteoporosis are im p ortan t oci in m an agem en t.
c. Preven tion an d m an agem en t o osteop orosis in clu des in creasin g weight-bearing exer-
cise an d in creasin g calcium an d vitamin D in th e diet. Medication s th at decrease bon e
resorp tion by b lockin g osteoclasts, or exam p le, alendronate sodium (Fosam ax), or
in crease bon e orm ation by stim u latin g osteoblasts, or exam p le, teriparatide (Forteo),
are also u se u l.

25
26 BRS Behavioral Science

85 Fe ma le 83.8
y)
81.0 Ma le 81.1
(
h
80 77.7 78.5
t
r
i
76.2 76.4
b
t
75
a
y
71.4
c
n
a
70
t
c
e
xp
65
e
e
f
i
L
60 FIGURE 3.1. Life expectancy at birth in years in the
All Africa n White His pa nic United States by selected characteristics in 2010.
Ame rica n Ame rica n
HELP OTHERS SO THAT GOD WILL HELP YOU.

(Data from National Center for Health Statistics,


Ethnicity 2013.)

B. Somatic and neurologic changes


1. Strength and physical health gradually decline. Th is declin e sh ows great variability bu t
com m on ly in clu des n ot on ly osteop orosis bu t also im p aired vision , h earin g, an d im m u n e
resp on ses; decreased m u scle m ass an d stren gth; in creased at dep osits; decreased ren al,
p u lm on ary, an d gastroin testin al u n ction ; redu ced bladder con trol; an d decreased
resp on siven ess to ch an ges in am bien t tem p eratu re.
2. Changes in the brain occu r with agin g.
a. Th ese ch an ges in clu de decreased brain weight, enlarged ventricles an d sulci, an d
decreased cerebral blood flow .
b. Amyloid (senile) plaques an d neurofibrillary tangles are p resen t in th e normally aging
brain b u t to a lesser exten t th an in n eu rocogn itive disorder du e to Alzh eim er’s disease,
h erein a ter Alzh eim er’s disease.
c. Neu roch em ical ch an ges th at occu r in agin g in clu de decreased availability of neu-
rotransmitters su ch as n orep in ep h rin e, dop am in e, γ-am in obu tyric acid, an d acetyl-
ch olin e; in creased availability of monoamine oxidase ; an d decreased responsiveness of
neurotransmitter receptors . Th ese ch an ges can be associated with p sych iatric sym p -
tom s su ch as depression an d anxiety (see below).

C. Cognitive changes
1. Alth ou gh learn in g sp eed m ay decrease, in th e absen ce o b rain d isease, intelligence
remains approximately the same th rou gh ou t li e.
2. Some memory problems m ay occu r in n orm al agin g (e.g., th e p atien t m ay orget th e n am e
RISE USMLE NEPAL

o a n ew acqu ain tan ce). However, th ese p rob lem s do not interfere with the patient’s func-
tioning or ab ility to live in dep en d en tly.

D. Psychological changes
1. In late adu lth ood, th ere is eith er a sen se o ego in tegrity (i.e., satis action an d p ride in
on e’s p ast accom p lish m en ts) or a sen se o d esp air an d worth lessn ess (Erikson’s stage of
ego integrity vs. despair). Most eld erly p eop le ach ieve ego in tegrity.
2. Psych op ath ology an d related p roblem s
a. Depression is th e m ost com m on p sych iatric disorder in th e elderly. Su icide is m ore
com m on in th e eld erly th an in th e gen eral p op u lation (see Tab le 12.2).
(1) Factors associated with dep ression in th e elderly in clu de loss o sp ou se, oth er am -
ily m em bers, an d rien ds; decreased social statu s; an d declin e o h ealth .
(2) Depression may mimic and thus be misdiagnosed as Alzheimer’s disease. Th is m is-
d iagn osed d isorder is re erred to as pseudodementia becau se it o ten is associated
with m em ory loss an d cogn itive p rob lem s (see Ch ap ter 14).
(3) Dep ression can be managed successfully u sin g su p p ortive p sych oth erapy in con -
ju n ction with p h arm acoth erapy or electrocon vu lsive th erapy (see Ch ap ter 15).
b. Sleep patterns change , resultin g in loss o sleep, poor sleep quality, or both (see Chapter 10).
Chapter 3 Aging, Death, and Bereavement 27

c. Anxiety an d ear u ln ess m ay be associated with realistic ear-in du cin g situ ation s (e.g.,
worries ab ou t d evelop in g a p h ysical illn ess or allin g an d breakin g a bon e).
d. Alcohol-related disorders are o ten u n iden ti ied bu t are p resen t in 10%–15% o th e geri-
atric p op u lation .
e. Psychoactive agents m ay p rodu ce di eren t e ects in th e elderly than in you n ger
p atien ts. For exam p le, u sin g an tih istam in es su ch as dip h en hydram in e as sleep agen ts
sh ou ld be avoided becau se th ey m ay cau se delirium (see Ch ap ter 14) in elderly p atien ts.
f. For a realistic p ictu re o th e u n ction in g level o elderly p atien ts, th e p h ysician sh ou ld
ideally evaluate patients in familiar surroundings , su ch as th eir own h om es.

E. Life expectancy and longevity


HELP OTHERS SO THAT GOD WILL HELP YOU.

1. Th e average life expectancy in th e United States is cu rren tly abou t 77 years . However, th is
igu re varies by gen d er an d eth n icity. Com p arin g th e th ree largest eth n ic grou p s, th e
longest-lived group is Hispanic American women an d th e shortest-lived group is African-
American men (Figu re 3.1).
2. Factors associated with longevity in clu de:
a. Fam ily h istory o lon gevity.
b. Con tin u ation o p h ysical an d occu p ation al activity.
c. Advan ced ed u cation .
d. Social su p p ort system s, in clu din g m arriage.

II. STAGES OF DYING AND DEATH


Accord in g to Dr. Elizabeth Kübler-Ross , th e p rocess o dyin g in volves five stages: denial, anger,
bargaining, depression, an d acceptance (DAng BaD Act). Th e stages u su ally occu r in th e ollowin g
order bu t also m ay be p resen t sim u ltan eou sly or in an oth er order.

A. Denial. Th e p atien t re u ses to believe th at h e or sh e is dyin g. (“Th e laboratory m ade an error.”)

B. Anger. Th e p atien t m ay b ecom e an gry at th e p h ysician an d h osp ital sta . (“It is you r au lt
th at I am d yin g. You sh ou ld h ave ch ecked on m e weekly.”) Ph ysician s m u st learn n ot to take
su ch com m en ts p erson ally (see Ch ap ter 21).

C. Bargaining. Th e p atien t m ay try to strike a b argain with God or som e h igh er b ein g. (“I will
RISE USMLE NEPAL

give h al o m y m on ey to ch arity i I can get rid o th is disease.”)

D. Depression. Th e p atien t becom es p reoccu p ied with d eath an d m ay b ecom e em otion ally
detach ed. (“I eel so distan t rom oth ers an d so h op eless.”) Som e p eop le becom e “stu ck” in
th is stage an d m ay be diagn osed with a com p licated grie reaction (see below).

E. Acceptance. Th e p atien t is calm an d accep ts h is or h er ate. (“I am read y to go n ow.”)

III. BEREAVEMENT (NORMAL GRIEF) VERSUS COMPLICATED


BEREAVEMENT (DEPRESSION)
A ter th e loss o a loved on e, th ere is a n orm al grie reaction . Th is reaction also occu rs with
oth er losses, su ch as loss o a b ody p art, or, or you n ger p eop le, with a m iscarriage or abortion .
A n orm al grie reaction m u st b e d istin gu ish ed rom a com p licated grie reaction , wh ich is o ten
p ath ologic (Table 3.1).
28 BRS Behavioral Science

t a b l e 3.1 Comparison between Normal Grief Reactions and Complicated Grief Reactions

Normal Grief Reaction (Bereavement) Complicated Grief Reaction (Depression)

Minor weight loss (e.g., <5 pounds) Significant weight loss (e.g., >5% of body weight)
Minor sleep disturbances Significant sleep disturbances
Mild guilty feelings Intense feelings of guilt and worthlessness
Illusions Hallucinations or delusions (see Chapter 11)
Attempts to return to work and social activities Resumes few, if any, work or social activities
Cries and expresses sadness Considers or attempts suicide
Severe symptoms resolve within 2 mo Severe symptoms persist for >2 mo
HELP OTHERS SO THAT GOD WILL HELP YOU.

Moderate symptoms subside within 1 y Moderate symptoms persist for >1 y


Management includes increased calls and visits to the Management includes antidepressants, antipsychotics,
physician, grief peer support groups, and short-acting electroconvulsive therapy, as well as increased contact
sleep agents, e.g., zolpidem (Ambien) for transient with the physician
problems with sleep
Adapted from Fadem B. Behavioral Science in Medicine. 2nd ed. Baltimore, MD: Lippincott Williams & Wilkins; 2012.

A. Characteristics of normal grief (bereavement)


1. Grie is ch aracterized in itially by shock an d denial.
2. In n orm al grie , th e b ereaved m ay exp erien ce an illusion (see Table 11.1) th at th e deceased
p erson is p h ysically p resen t.
3. Norm al grie gen erally subsides after 1–2 years , alth ou gh som e eatu res m ay con tin u e lon -
ger. Even a ter th ey h ave su b sid ed, sym p tom s m ay retu rn on h olidays or sp ecial occasion s
(th e “anniversary reaction”).
4. Th e mortality rate is h igh or close relatives (esp ecially widowed men) in th e irst year o
b ereavem en t.

B. Physician’s response to death


1. Th e m ajor responsibility of the physician is to give su p p ort to th e dyin g p atien t an d th e
p atien t’s am ily.
2. Gen erally, p h ysician s make the patient completely aware o th e d iagn osis an d p rogn osis.
However, a p h ysician sh ou ld ollow th e p atien t’s lead as to h ow m u ch h e or sh e wan ts to
kn ow abou t th e con dition . With th e p atien t’s p erm ission , th e physician may tell the family
th e diagn osis an d oth er d etails o th e illn ess (see Ch ap ter 23).
3. Ph ysician s o ten eel a sense of failure at n ot p reven tin g th e death o a p atien t. Th ey m ay
RISE USMLE NEPAL

d eal with th is sen se by becom in g emotionally detached rom th e p atien t. Su ch detach m en t


can p reclu de h elp in g th e p atien t an d am ily th rou gh th is im p ortan t tran sition .
Review Test

Directions: Each o th e n u m b ered item s or in com p lete statem en ts in th is section is ollowed by


an swers or by com p letion s o th e statem en t. Select th e one lettered an swer or com p letion th at
is best in each case.
HELP OTHERS SO THAT GOD WILL HELP YOU.

1. When question ed about her li estyle, a 4. A term in ally ill p atien t wh o u ses a
70-year-old wom an tells the doctor that she statem en t su ch as, “It is th e doctor’s
eats m ain ly sh an d chicken but en joys an ault th at I becam e ill; sh e didn’t do an
occasion al steak. She also n otes that she is electrocardiogram wh en I cam e or m y last
lactose in toleran t an d so avoids m ilk products o ce visit,” is m ost likely in wh ich stage o
but eats alm on ds, bean s, an d can n ed salm on dyin g, accordin g to Elizabeth Kü b ler-Ross?
with bon es daily. She also n otes that she drin ks (A) Den ial
on e cup o co ee an d on e glass o win e an d (B) An ger
sm okes on e cigarette daily. To help preven t (C) Bargain in g
osteoporosis, the m ost im portan t advice the (D) Dep ression
physician should give this patient is to (E) Accep tan ce
(A) stop drin kin g win e
(B) stop eatin g steak 5. A p h ysician con d u cts a p h ysical
(C) stop drin kin g co ee exam in ation on an active, in dep en den t
(D) start u sin g d airy p rodu cts d esp ite h er 75-year-old wom an . Wh ich o th e ollowin g
in toleran ce n din gs is m ost likely?
(E) stop sm okin g (A) In creased im m u n e resp on se
(B) In creased m u scle m ass
2. An 80-year-old wom an is bein g exam in ed (C) Decreased size o brain ven tricles
by a p h ysician or ad m ission to a n u rsin g (D) Decreased bladd er con trol
h om e. Th e wom an , wh o was brou gh t to (E) Severe m em ory p roblem s
th e doctor by h er son , seem s an xiou s an d
con u sed. Th e m ost e ective action or th e 6. Nin ety p ercen t o th e p atien ts in a
p h ysician to take at th is tim e is to p rim ary care p h ysician’s p ractice are over 65
(A) arran ge or im m ed iate ad m ission to a years o age. Wh en com p ared to th e gen eral
n u rsin g h om e p opu lation , th ese elderly p atien ts are
RISE USMLE NEPAL

(B) con d u ct a n eu rop sych ological evalu ation m ore likely to sh ow wh ich o th e ollowin g
(C) su ggest im m ed iate h osp italization p sych ological ch aracteristics?
(D) ask th e son i h e h as ob served ch an ges in (A) Lower likelih ood o su icide
th e p atien t’s beh avior (B) Less an xiety
(E) arran ge to exam in e th e wom an in h er (C) Lower in telligen ce
own h om e (D) Poorer sleep qu ality
(E) Less dep ression
3. Each year d u rin g th e rst week in May,
a 63-year-old wom an develop s ch est 7. Th e 78-year-old h u sban d o a 70-year-
discom ort an d a eelin g o oreb odin g. Her old wom an h as ju st died . I th is wom an
h u sb an d died 5 years ago d u rin g th e rst experien ces n orm al bereavem en t, wh ich o
week in May. Th is wom an’s exp erien ce is th e ollowin g resp on ses wou ld be exp ected?
best describ ed as
(A) In itial loss o ap p etite
(A) an atten tion -seekin g d evice (B) Feelin gs o worth lessn ess
(B) p ath ological grie (C) Th reats o su icide
(C) an an n iversary reaction (D) In ten se grie lastin g years a ter th e death
(D) m alin gerin g (E) Feelin gs o h op elessn ess
(E) dep ression

29
30 BRS Behavioral Science

8. A p h ysician h as ju st d iagn osed a case o 12. An 81-year-old p atien t com p lain s to


term in al p an creatic can cer in a 68-year-old h is doctor th at h e som etim es h as trou b le
m an . Wh ich o th e ollowin g statem en ts allin g asleep an d wou ld like to h ave “a p ill
regardin g th e reaction s an d b eh avior o th e to take” at th ose tim es. Ph ysical exam in ation
p h ysician is th e m ost tru e? is u n rem arkab le an d th e p atien t sh ows
(A) Sh e sh ou ld in orm th e am ily, bu t n ot th e n o eviden ce o p sych op ath ology. O th e
p atien t, abou t th e seriou s n atu re o th e ollowin g agen ts, wh ich sh ou ld be avoided in
illn ess. th is p atien t?
(B) Her in volvem en t with th e p atien t’s (A) Dip h en h ydram in e
am ily sh ou ld en d wh en h e dies. (B) Zalep lon
(C) Sh e sh ou ld p rovide stron g sedation or (C) Trazodon e
HELP OTHERS SO THAT GOD WILL HELP YOU.

am ily m em bers wh en th e p atien t dies (D) Zolp id em


u n til th e in itial sh ock o h is death wears (E) Ram elteon
o .
(D) Sh e will eel th at sh e h as ailed wh en th e 13. A 50-year-old wom an wh o is dyin g o
p atien t dies. can cer h as a 10-year-old son . Th e m oth er
(E) Sh e will eel closer an d closer to th e d oes n ot wan t th e ch ild to kn ow abou t
p atien t as h is death ap p roach es. h er illn ess or p rogn osis. Most correctly,
with resp ect to th e m oth er’s con d ition , th e
9. Th e average d i eren ce in li e exp ectan cy p h ysician sh ou ld
between Wh ite wom en an d A rican - (A) talk to th e m oth er an d en cou rage h er to
Am erican m en is ap p roxim ately tell h er son
(A) 3 years (B) talk to th e son alon e an d tell h im abou t
(B) 6 years h is m oth er’s illn ess
(C) 10 years (C) ollow th e m oth er’s wish es an d do n ot
(D) 15 years tell th e son
(E) 20 years (D) talk to both th e m oth er an d son togeth er
(E) in sist th at th e m oth er tell h er son
10. Six m on th s a ter th e d eath o a loved
on e, wh ich o th e ollowin g is m ost likely 14. A 70-year-old p atien t wh ose wi e died
to in dicate th at a p erson is exp erien cin g a 8 m on th s ago rep orts th at h e som etim es
com p licated grie reaction ? wakes u p an h ou r earlier th an u su al an d
(A) Lon gin g o ten cries wh en h e th in ks abou t h is wi e.
(B) Cryin g He also tells you th at on on e occasion , h e
(C) Den ial th at th e loved on e h as died b rief y ollowed a wom an down th e street
(D) Irritab ility wh o resem b led h is late wi e. Th e p atien t also
(E) Illu sion s relates th at h e h as rejoin ed h is bowlin g team
RISE USMLE NEPAL

an d en joys visits with h is gran dch ildren . For


11. An 80-year-old p atien t tells th e d octor th is p atien t, th e b est recom m en d ation o th e
th at sh e is con cern ed b ecau se sh e orgets p h ysician is
th e add resses o p eop le sh e h as ju st m et (A) m ed ication or sleep
an d takes lon ger th an in th e p ast to do th e (B) evalu ation or m ajor dep ression
Su n d ay crossword p u zzle. Sh e p lays card s (C) regu lar p h on e calls an d visits to “ch eck
regu larly with rien ds, is well groom ed , an d in” with th e doctor
sh ops an d cooks or h ersel . Th is p atien t is (D) p sych oth erapy
p robab ly (E) n eu rop sych ological evalu ation or
(A) sh owin g n orm al agin g Alzh eim er’s d isease
(B) sh owin g evid en ce o Alzh eim er’s d isease
(C) exp erien cin g d ep ression
(D) develop in g an an xiety d isorder
(E) u n ab le to live alon e
Chapter 3 Aging, Death, and Bereavement 31

15. An 85-year-old m an an d h is 80-year- (A) n ot to worry bu t in stead to con cen trate


old wi e are brou gh t to th e em ergen cy on h er own con dition
d ep artm en t a ter an au tom ob ile accid en t. (B) th at h er h u sban d h as died an d th en stay
Th e m an is d ead on arrival. Th e wom an is an d o er su p p ort
n ot seriou sly in ju red an d is con sciou s an d (C) th at h er son is on th e way an d th at th ey
alert. Th e cou p le’s son h as been called an d will discu ss everyth in g wh en th e son
is on h is way to th e h osp ital. Th e wom an arrives
asks th e p h ysician abou t h er h u sban d’s (D) th at h e will ch eck on h er h u sban d’s
con dition . Most correctly, th e p h ysician con d ition a ter sh e is treated or h er
sh ould tell h er in ju ries
(E) wh at h as h ap p en ed to h er bu t n ot wh at
HELP OTHERS SO THAT GOD WILL HELP YOU.

h as h ap p en ed to h er h u sban d
RISE USMLE NEPAL
An swers an d Exp lan ation s

Typical Board Question


The answer is B. Th is p atien t wh ose wi e died 8 m on th s ago sh ows evid en ce o a com p licated grie
reaction . He is sh owin g sign s o dep ression (e.g., p oor groom in g, sign i ican t weigh t loss, seri-
ou s sleep p roblem s, an d little in terest in in teractin g with rien ds an d am ily) (see Ch ap ter 12).
Psych oth erapy, wh ile h elp u l, will b e less u se u l th an an tidep ressan t m edication or th is p atien t.
HELP OTHERS SO THAT GOD WILL HELP YOU.

His sleep will im p rove as th e d ep ression im p roves. Elderly p atien ts exp erien cin g dep ression
o ten p resen t with m em ory p rob lem s th at m ay m im ic Alzh eim er’s disease (p seu dod em en tia).
Th e su dden on set o m em ory p roblem s (e.g., orgettin g wh at h e h as been eatin g) with th e con -
cu rren t loss o h is wi e in d icates th at th e p atien t is likely to b e exp erien cin g d ep ression rath er
th an Alzh eim er’s disease. Alth ou gh h e sh ou ld be closely ollowed, th ere is n o in dication at th is
tim e th at th is p atien t n eed s a n eu rop sych iatric evalu ation .

1. The answer is E. Cigarette sm okin g, even i m od erate, is associated with th e develop m en t


o osteop orosis. Alm on ds, bean s, an d can n ed salm on with bon es con tain calciu m an d th u s
can h elp com p en sate or th e absen ce o d ietary d airy p rod u cts. Moderate in take o steak,
co ee, an d alcoh ol as d escribed by th is p atien t are n ot associated with th e d evelop m en t or
worsen in g o osteop orosis.
2. The answer is E. Th e m ost e ective action or th e p h ysician to take at th is tim e is to exam in e
th e wom an in h er own h om e. An xiety or d ep ression at b ein g in an u n am iliar situ ation can
lead to th e an xiety an d con u sion th at th is p atien t sh ows. Im m ediate adm ission to a n u rsin g
h om e or h osp ital, or in terviewin g th e son are n ot ap p rop riate u n til a tru e p ictu re o th e
p atien t’s con d ition h as b een ob tain ed . A n eu rop sych ological evalu ation also m ay n ot be
h elp u l wh ile th is p atien t is sh owin g eviden ce o severe stress.
3. The answer is C. Th is wom an’s exp erien ce is b est described as an an n iversary reaction . In
th is reaction , th e b ereaved p erson exp erien ces m an y o th e eelin gs sh e exp erien ced wh en
h er h u sb an d died at sign i ican t tim es in su b sequ en t years. Th is is con sidered a n orm al
reaction , n ot p ath ological grie , an d is n ot associated with dep ression . It is also n ot a sign o
m alin gerin g or o seekin g atten tion .
RISE USMLE NEPAL

4. The answer is B. Du rin g th e an ger stage o dyin g, th e p atien t is likely to blam e th e


p h ysician .
5. The answer is D. O th e listed in d in gs, d ecreased bladd er con trol is th e m ost likely in din g
in th e exam in ation o an active, in d ep en den t 75-year-old wom an . In agin g, im m u n e
resp on ses an d m u scle m ass decrease an d b rain ven tricles in crease in size. Wh ile m ild
m em ory p roblem s m ay occu r, severe m em ory p roblem s do n ot occu r in n orm al agin g.
Severe m em ory p rob lem s th at in ter ere with n orm al u n ction in dicate th e develop m en t o a
dem en tia su ch as Alzh eim er’s d isease.
6. The answer is D. Sleep d istu rban ces, su ch as d ecreased delta (slow wave) sleep (see Ch ap ter
10) com m on ly occu r in th e elderly. Su icide an d dep ression are m ore com m on in th e elderly
th an in th e gen eral p op u lation . An xiety m ay arise easily d u e to ears o illn ess an d in ju ry.
In telligen ce d oes n ot decrease in typ ical elderly p eop le.
7. The answer is A. In itial loss o ap p etite is com m on in n orm al bereavem en t. Feelin gs
o worth lessn ess or h op elessn ess, th reats o su icide, an d an exten ded p eriod o grie
ch aracterize d ep ression rath er th an n orm al b ereavem en t.

32
Chapter 3 Aging, Death, and Bereavement 33

8. The answer is D. Ph ysician s o ten eel th at th ey h ave ailed wh en a p atien t dies. Rath er
th an becom in g closer, th is p h ysician m ay becom e em otion ally detach ed rom th e p atien t
in order to deal with h is im p en din g death . Heavy sedation is rarely in dicated as treatm en t
or th e bereaved becau se it m ay in ter ere with th e grievin g p rocess. Gen erally, p h ysician s
in orm p atien ts wh en th ey h ave a term in al illn ess an d p rovide an im p ortan t sou rce o
su p p ort or th e am ily be ore an d a ter th e p atien t’s death .
9. The answer is C. Th e di eren ce in li e exp ectan cy between Wh ite wom en (81.0 years)
an d A rican -Am erican m en (71.4 years) is ap p roxim ately 10 years. Th e di eren ce in li e
exp ectan cy by age an d sex is cu rren tly d ecreasin g.
10. The answer is C. Six m on th s a ter th e d eath o a loved on e, den yin g th at th e death h as
HELP OTHERS SO THAT GOD WILL HELP YOU.

actu ally occu rred su ggests a com p licated grie reaction . Norm ally, d en ial lasts u p to 24
h ou rs. Lon gin g, cryin g, irritability, an d illu sion s are all p art o a n orm al grie reaction .
11. The answer is A. Th is 80-year-old wom an is p robably sh owin g n orm al agin g, sin ce sh e
can u n ction well livin g alon e. Min or m em ory loss th at d oes n ot in ter ere with n orm al
u n ction in g su ch as sh e describes is typ ically seen in n orm ally agin g p eop le. Th ere is n o
evid en ce th at th is p atien t h as Alzh eim er’s d isease, d ep ression , or an an xiety d isord er.
12. The answer is A. Th e an tih istam in ergic agen t dip h en h ydram in e (Ben adryl) sh ou ld
be avoid ed in elderly p atien ts b ecau se it is likely to cau se sym p tom s o deliriu m .
Un ortu n ately, a n u m b er o over-th e-cou n ter sleep m edicin es su ch as Tylen ol PM con tain
dip h en h yd ram in e. Better ch oices or in som n ia in th e elderly in clu de n ewer sleep agen ts
su ch as zolp id em (Am bien ) an d ram elteon (Rozerem ). Trazodon e is a sedatin g tricyclic
an tid ep ressan t wh ich is also u se u l or occasion al p rob lem s allin g asleep in th e elderly.
13. The answer is A. It is u p to th e m oth er to d ecid e wh eth er, wh en , an d h ow to tell h er son
abou t h er illn ess. However, sch ool-age ch ildren are o ten aware wh en som eth in g seriou s
is goin g on with in th eir am ily an d can u n derstan d th e m ean in g o death (see Ch ap ter 2).
Th ere ore, wh ile it is n ot ap p rop riate or th e p h ysician to in sist th at th e p atien t tell h er son ,
th e p h ysician sh ou ld talk to th e m oth er an d en cou rage h er to talk to h er son abou t h er
term in al con dition . Th e p h ysician can also cou n sel th e p atien t on wh at to say to h er ch ild
abou t h er im m in en t death .
14. The answer is C. Th is p atien t, wh ose wi e died 8 m on th s ago, is sh owin g a n orm al grie
reaction . Alth ou gh h e som etim es wakes u p an h ou r earlier th an u su al an d cries wh en h e
th in ks abou t h is wi e, h e is attem p tin g to retu rn to h is li estyle by rejoin in g h is bowlin g
team an d visitin g with h is am ily. Th e illu sion o believin g h e sees an d th u s ollows a
wom an wh o resem bled h is late wi e is seen in a n orm al grie reaction . For a n orm al grie
RISE USMLE NEPAL

reaction , recom m en din g regu lar p h on e calls an d visits to “ch eck in” with th e doctor is
th e ap p rop riate in terven tion . Sleep m ed ication , an tidep ressan ts, p sych oth erapy, an d a
n eu rop sych ological evalu ation are n ot n ecessary or th is p atien t at th is tim e.
15. The answer is B. Th ere is n o in d ication th at th is eld erly wom an is im p aired m en tally or
p h ysically. Th ere ore, th e p h ysician sh ou ld tell h er th e tru th , th at is, th at h er h u sban d h as
died an d th en stay an d o er su p p ort. As with all adu lt p atien ts, eld erly p atien ts sh ou ld b e
told th e tru th . It is n ot n ecessary to wait or th e son to arrive, an d tellin g h er n ot to worry is
p atron izin g.
Gen etics, An atom y,
c ha pte r
4 an d Bioch em istry
o Beh avior
HELP OTHERS SO THAT GOD WILL HELP YOU.

Typical Board Question


Wh en a 70-year-old m an wh o h as h ad a stroke attem p ts to (a) divide a lin e in h al , (b) tu rn
sin gle lin es in to “Xs,” or (c) rep rod u ce a clock ace, h e does th e tasks like th is (see Figu re)
e ectively n eglectin g th e le t o th e drawin gs. Th e area(s) o th e brain m ost likely to be
a ected in th is p atien t is (are) th e
(A) righ t p arietal lob e (a )
(B) basal gan glia
(C) h ip p ocam p u s
(D) reticu lar system
(b)
(E) am ygd ala 12 1
(F) le t ron tal lobe 2
3
4
5
6
(See “An sw ers an d Explan ation s” at th e en d of th e ch apter.) (c) 11 10
9
87

I. THE GENETICS OF BEHAVIOR


A. Studies for examining the genetics of behavior
1. Family risk studies com p are h ow requ en tly a beh avioral disord er or trait occu rs in th e
relatives o th e a ected in d ivid u al (proband) with h ow requ en tly it occu rs in th e gen eral
RISE USMLE NEPAL

p op u lation .
2. Twin studies
a. Adoption studies u sin g monozygotic twins or dizygotic twins reared in th e sam e or in di -
eren t h om es are u sed to di eren tiate th e e ects o gen etic actors rom en viron m en tal
actors in th e occu rren ce o p sych iatric, su bstan ce abu se (e.g., alcoh olism ), an d n eu ro-
p sych iatric d isorders.
b. I th ere is a gen etic com p on en t to th e etiology, a disorder m ay be exp ected to h ave a
h igh er concordance rate in m on ozygotic twin s th an in d izygotic twin s (i.e., i con cor-
d an t, th e d isorder occu rs in b oth twin s).
3. It h as b een di icu lt to lin k p articu lar ch rom osom es with p sych iatric illn esses. However, in
a n u m ber o stu dies over years, su ch association s h ave been m ade.
a. Schizophrenia (Ch ap ter 11) h as b een associated with m arkers on ch rom osom es 1, 6, 7,
8, 13, 21, an d 22.
b. Bipolar disorder an d major depressive disorder (Ch ap ter 12) recen tly h ave been associ-
ated with m arkers on ch rom osom es 3, 5 an d 6.

B. Specific chromosomes h ave been associated with oth er disorders with beh avioral sym p tom s
(Table 4.1).

34
Chapter 4 Genetics, Anatomy, and Biochemistry of Behavior 35

t a b l e 4.1 Chromosomal Disorders with Behavioral Manifestations


Chromosome Disorder Behavioral Manifestations

1 Alzheimer’s disease Depression, anxiety, dementia (early onset)


4 Huntington’s disease Erratic behavior, psychiatric symptoms (e.g., depression,
psychosis), dementia
5 Sotos syndrome Intellectual impairment, phobias, hyperphagia
7 William’s syndrome Hypersociality, intellectual disability, behavioral problems,
hypotonia
8 Cohen’s syndrome Autistic behavior, intellectual disability
9 Dystonia musculorum deformans (DYT1) Depression, learning problems
HELP OTHERS SO THAT GOD WILL HELP YOU.

Tuberous sclerosis Seizures, cognitive impairment, autistic behavior


11 Acute intermittent porphyria Manic behavior, psychosis (see Chapter 5)
12 Phenylketonuria Attention deficit hyperactivity disorder (ADHD), intellectual
disability
13 Wilson’s disease Depression, personality changes, psychotic symptoms
14 Alzheimer’s disease Depression, anxiety, dementia (early onset)
15 Chromosome 15 inversion-duplication Seizures, autistic behavior, hypotonia
syndrome
Prader-Willi syndrome/Angelman Intellectual disability, rage, stubbornness, rigid thinking, and
syndrome self-injury
16 Tuberous sclerosis Seizures, cognitive impairment, autistic behavior
17 Neurofibromatosis-1 Cognitive impairment
Charcot-Marie-Tooth disease Peripheral neuropathy
Smith-Magenis syndrome Intellectual disability, impaired expressive language,
stereotyped behavior, clinging and dependency, seizures
18 Tourette’s disorder Dyscontrol of language and movements
19 Alzheimer’s disease (site of the APO Depression, anxiety, dementia (at the typical age of onset)
E4 gene)
21 Progressive myoclonic epilepsy Cognitive regression, aphasia, intellectual disability
Alzheimer’s disease (associated with Depression, anxiety, dementia (early onset)
Down’s syndrome)
22 Metachromatic leukodystrophy Personality changes, psychosis, dementia
Neurofibromatosis-2 Hearing impairment
DiGeorge/velocardiofacial syndrome Schizophrenia, bipolar disorder, psychomotor retardation,
language delay, ADHD, seizures
X Fragile X syndrome Autistic behavior, intellectual disability
Kallmann’s syndrome Anosmia, lack of sex drive, depression, anxiety, fatigue, insomnia
Lesch-Nyhan syndrome Self-mutilation and other bizarre behavior, intellectual disability
RISE USMLE NEPAL

Rett’s disorder Autistic behavior, hand-wringing, breathing abnormalities


Adapted from Fadem B, Monaco E. High Yield Brain and Behavior. Baltimore, MD: Lippincott Williams & Wilkins; 2007:27, with permission.

II. THE NEUROANATOMY OF BEHAVIOR


Th e h u m an n ervou s system con sists o th e central nervous system (CNS) an d th e peripheral ner-
vous system (PNS).

A. Th e CNS con tain s th e brain an d sp in al cord.


1. Th e cerebral cortex can be divided
a. Anatomically in to at least ou r sets o lobes: ron tal, tem p oral, p arietal, an d occip ital,
as well as th e lim b ic lobes (wh ich con tain m edial p arts o th e ron tal, tem p oral, an d
p arietal lobes an d in clu de th e h ip p ocam p u s, am ygdala, orn ix, sep tu m , p arts o th e
th alam u s, an d cin gu late gyru s an d related stru ctu res).
b. By arrangement o n eu ron layers or cytoarch itectu re.
c. Functionally in to m otor, sen sory, an d association areas.
36 BRS Behavioral Science

2. Th e cerebral h em isp h eres.


a. Th e h em isp h eres are connected by th e corp u s callosu m , an terior com m issu re, h ip p o-
cam p al com m issu re, an d h aben u lar com m issu re.
b. Th e u n ction s o th e h em isp h eres are lateralized.
(1) Th e right, or nondominant, hemisphere is associated p rim arily with perception; it is
also associated with spatial relations , body image , an d m u sical an d artistic ability.
(2) Th e left, or dominant, hemisphere is associated with language function in ab ou t
alm ost all righ t-h an ded p eop le an d m ost le t-h an ded p eop le.
c. Sex differences in cerebral lateralization . Women m ay h ave a larger corp u s callosu m
an d an terior com m issu re an d ap p ear to h ave better in terh em isp h eric com m u n ication
th an m en . Men m ay h ave b etter-develop ed righ t h em isp h eres an d ap p ear to be m ore
HELP OTHERS SO THAT GOD WILL HELP YOU.

adep t at sp atial tasks th an wom en .


3. Brain lesions cau sed by acciden t, d isease, su rgery, or oth er in su lt are associated with p ar-
ticu lar n eu rop sych iatric e ects (Table 4.2).
4. Memory systems
a. Exp licit or declarative memory in volves th e kn owledge o acts an d is retrieved consciously.
b. Im p licit or nondeclarative memory in volves in orm ation on h ow to p er orm an act an d
is recalled unconsciously.
c. Th e n eu roan atom y o th ese m em ory system s an d clin ical exam p les can be ou n d in
Table 4.3.

B. Th e PNS con tain s all sensory, motor, and autonomic ib ers ou tsid e o th e CNS, in clu d in g th e
spinal nerves, cranial nerves , an d peripheral ganglia .
1. Th e PNS ca rries sensory in orm a tio n to th e CNS a n d motor in orm a tio n awa y rom
th e CNS.

t a b l e 4.2 Neuropsychiatric Effects of Brain Lesions on Behavior


Location of Lesion Effects

Frontal lobes Mood changes (e.g., depression with dominant lesions, mood elevation with nondominant
lesions)
Difficulties with motivation, concentration, attention, orientation, and problem solving
(dorsolateral convexity lesions)
Difficulties with judgment, inhibitions, emotions, personality changes (orbitofrontal lesions)
Inability to speak fluently (i.e., Broca aphasia [dominant lesions])
Temporal lobes Impaired memory
Psychomotor seizures
RISE USMLE NEPAL

Changes in aggressive behavior


Inability to understand language (i.e., Wernicke’s aphasia [dominant lesions])
Limbic lobes Poor new learning; implicated specifically in Alzheimer’s disease
Hippocampus Klüver-Bucy syndrome (decreased aggression, increased sexual behavior, hyperorality)
Amygdala Decreased conditioned fear response
Problems recognizing the meaningfulness of facial and vocal expressions of anger in others
Parietal lobes Impaired processing of visual–spatial information (e.g., cannot copy a simple line drawing or
neglects the numbers on the left side when drawing a clock face [right-sided lesions])
Impaired processing of verbal information (e.g., cannot tell left from right, do simple math, name
fingers, or write [Gerstmann’s syndrome, dominant lesions])
Occipital lobes Visual hallucinations and illusions
Inability to identify camouflaged objects
Blindness
Hypothalamus Hunger leading to obesity (ventromedial nucleus damage), loss of appetite leading to weight loss
(lateral nucleus damage)
Effects on sexual activity and body temperature regulation
Reticular system Changes in sleep–wake mechanisms (e.g., decreased REM sleep)
Loss of consciousness
Basal ganglia Disorders of movement (e.g., Parkinson’s disease [substantia nigra], Huntington’s disease
[caudate and putamen], and Tourette’s syndrome [caudate])
Chapter 4 Genetics, Anatomy, and Biochemistry of Behavior 37

t a b l e 4.3 Memory Systems and Associated Neuroanatomy

Type of System Type of Memory Associated Anatomy Length of Recall Memory Used to Remember

Declarative Episodic Temporal lobes (medial), Long term Personally experienced


(explicit or anterior thalamic nuclei, events, e.g., what you
conscious) fornix, hippocampus, ate yesterday
mammillary bodies,
prefrontal cortex
Semantic Inferolateral temporal lobes Long term General knowledge about
the world, e.g., the
capital of New J ersey
Nondeclarative Procedural Cerebellum, basal ganglia, Long term Things you do
HELP OTHERS SO THAT GOD WILL HELP YOU.

(implicit or supplementary motor area automatically, e.g., how


unconscious) to tie your shoes
Working Prefrontal cortex, language Short term Recent information, e.g.,
and visual association the phone number just
areas obtained from a new
acquaintance

2. Th e au ton om ic n ervou s system , wh ich con sists o sympathetic an d parasympathetic divi-


sion s, in n ervates th e in tern al organ s.
3. Th e au ton om ic n ervou s system coord in ates em otion s with visceral resp on ses su ch as
h eart rate, blood p ressu re, an d p ep tic acid secretion .
4. Visceral resp on ses occu rrin g as a resu lt o psychological stress are in volved in th e devel-
op m en t an d exacerb ation o som e physical illnesses (see Ch ap ter 22).

III. NEUROTRANSMISSION
A. Synapses and neurotransmitters
1. In orm ation in th e n ervou s system is tran s erred across th e synaptic cleft (i.e., th e sp ace
b etween th e axon term in al o th e p resyn ap tic n eu ron an d th e den drite o th e p ostsyn ap -
tic n eu ron ).
2. Wh en th e p resyn ap tic n eu ron is stim u lated, a neurotransmitter is released, travels across
th e syn ap tic cle t, an d acts on recep tors on th e p ostsyn ap tic n eu ron . Neu rotran sm itters
are excitatory i th ey in crease th e ch an ces th at a n eu ron will ire an d inhibitory i th ey
decrease th ese ch an ces.
RISE USMLE NEPAL

B. Presynaptic and postsynaptic receptors are p rotein s p resen t in th e m em bran e o th e n eu -


ron th at can recogn ize sp eci ic n eu rotran sm itters.
1. Th e changeability o n u m ber or a in ity o recep tors or sp eci ic n eu rotran sm itters (neuro-
nal plasticity) can regu late th e resp on siven ess o n eu ron s.
2. Second messengers. Wh en stim u lated by n eu rotran sm itters, p ostsyn ap tic recep tors m ay
alter th e m etabolism o n eu ron s by th e u se o secon d m essen gers, wh ich in clu de cyclic
adenosine monophosphate (cAMP), lipids (e.g., diacylglycerol), Ca 2+, an d nitric oxide .

C. Classification of neurotransmitters. Biogenic amines (m on oam in es), amino acids , an d pep-


tides are th e th ree m ajor classes o n eu rotran sm itters.

D. Regulation of neurotransmitter activity


1. Th e con cen tration o n eu rotran sm itters in th e syn ap tic cle t is closely related to m ood
an d b eh avior. A n u m ber o m ech an ism s a ect th is con cen tration .
2. A ter release by th e p resyn ap tic n eu ron , n eu rotran sm itters are rem oved rom th e syn ap tic
cle t by m ech an ism s in clu d in g:
a. Reuptake by th e p resyn ap tic n eu ron .
b. Degradation by en zym es su ch as monoamine oxidase (MAO).
38 BRS Behavioral Science

t a b l e 4.4 Psychiatric Conditions and Associated Neurotransmitter Activity


Neuropsychiatric Condition Neurotransmitter Activity Increased (↑ ) or Decreased (↓ )

Depression Norepinephrine (↓ ), serotonin (↓ ), dopamine (↓)


Mania Dopamine (↑), g-aminobutyric acid (GABA) (↓)
Schizophrenia Dopamine (↑ ), serotonin (↑ ), glutamate (↑ or ↓ )
Anxiety GABA (↓ ), serotonin (↓ ), norepinephrine (↑ )
Alzheimer’s disease Acetylcholine (↓ ), glutamate (↑ )

3. Availability o sp eci ic n eu rotran sm itters is associated with com m on psychiatric con dition s
HELP OTHERS SO THAT GOD WILL HELP YOU.

(Table 4.4). Norm alization o n eu rotran sm itter availability by pharm acological agen ts is
associated with sym p tom im p rovem en t in th ese disorders (see Chap ter 16).

IV. BIOGENIC AMINES


A. Overview
1. Th e biogenic amines , or monoamines , in clu de catech olam in es, in dolam in es, eth yl am in es,
an d qu atern ary am in es.
2. Th e monoamine theory of mood disorder h yp oth esizes th at lowered monoamine activity
results in depression and elevated levels in mania .
3. Metabolites o the m onoam ines are o ten m easured in psychiatric research and diagnosis
because they are m ore easily m easured in body luids than the actual m onoam ines (Table 4.5).
4. Distrib u tion o d op am in ergic, n orad ren ergic, an d seroton ergic tracts in th e CNS can be
ou n d in Figu re 4.1.

B. Dopamine
1. Dop am in e, a catech olam in e, is in volved in th e p ath op h ysiology o schizophrenia an d other
psychotic disorders, Parkinson’s disease, mood disorders , th e con dition ed ear resp on se
(see Ch ap ter 7), an d th e “rewardin g” n atu re o certain d ru gs (see Ch ap ter 9).
2. Synthesis. Th e am in o acid tyrosin e is con verted to th e p recu rsor or dop am in e by th e
en zym e tyrosine hydroxylase .
3. Receptor subtypes. At least ive d op am in e recep tor su btyp es (D 1–D 5) h ave been iden ti ied;
th e m ajor site o action is D 2 or tradition al an tip sych otic agen ts an d D 1 an d D 4 as well as
D 2 or th e n ewer “atyp ical” an tip sych otic agen ts (see Ch ap ter 16).
RISE USMLE NEPAL

t a b l e 4.5 Metabolites of Monoamines and Associated Psychopathology

Increased (↑) or Decreased (↓) Concentration


of Metabolite in Blood Plasma, Cerebrospinal
Neurotransmitter Fluid, or Urine Associated Psychopathology

Dopamine (↑ ) HVA (homovanillic acid) Schizophrenia and other conditions involving


psychosis (see Chapters 9, 11, and 12)
(↓ ) HVA Parkinson’s disease
Treatment with antipsychotic agents
Depression
Norepinephrine (↑ ) VMA (vanillylmandelic acid) Adrenal medulla tumor (pheochromocytoma)
(↓ ) MHPG (3-methoxy-4-hydroxyphenylglycol) Severe depression and attempted suicide
Serotonin (↓ ) 5-HIAA (5-hydroxyindoleacetic acid) Severe depression and attempted suicide
Aggressiveness and violence
Impulsiveness
Tourette’s syndrome
Alcohol misuse
Bulimia
Chapter 4 Genetics, Anatomy, and Biochemistry of Behavior 39

Tube roinfundibula r tra ct


S tria tum
Nigros tria ta l
tra ct
Me s olimbic/
me s ocortica l
tra ct Fornix

Orbitofronta l Ve ntra l
corte x te gme nta l
a re a
S e pta l
nucle i
Nucle us S ubs ta ntia
a ccumbe ns nigra
HELP OTHERS SO THAT GOD WILL HELP YOU.

OH H H Amygda la
HO C C NH2 P ons Ce re be llum
H H
Dopa mine Me dulla
A

Ce rulocortica l
tra ct

Fornix

Locus
ce rule us
Amygda la

Hippoca mpus
OH
OH H
HO C C NH2
H H
Nore pine phrine
B

Ra phe cortica l
tra ct
RISE USMLE NEPAL

Fornix

Amygda la

Hippoca mpus
Ra phe nucle i
H H
HO
C C NH2
N H H
S e rotonin
C
FIGURE 4.1. Distribution of (A) dopaminergic, (B) noradrenergic, and (C) serotonergic tracts in the CNS.
40 BRS Behavioral Science

4. Dopaminergic tracts (Figure 4.1A)


a. Th e nigrostriatal tract is in volved in th e regu lation o m u scle ton e an d m ovem en t.
(1) Th is tract degenerates in Parkinson’s disease.
(2) Treatm en t with an tip sych otic d ru gs, wh ich block p ostsyn ap tic dop am in e recep tors
receivin g in p u t rom th e n igrostriatal tract, can resu lt in Parkin son -like sym p tom s.
b. Dop am in e acts on th e tuberoinfundibular tract to in h ib it th e secretion o p rolactin rom
th e an terior p itu itary.
(1) Blockad e o d op am in e recep tors by an tip sych otic dru gs p reven ts th e in h ibition o
p rolactin release an d resu lts in elevated prolactin levels.
(2) Th is elevation in tu rn resu lts in sym p tom s su ch as breast en largem en t, galactor-
rh ea, an d sexu al dys u n ction .
HELP OTHERS SO THAT GOD WILL HELP YOU.

c. Th e mesolimbic–mesocortical tract is associated with p sych otic disorders.


(1) Th is tract m ay h ave a role in th e exp ression o emotions sin ce it p rojects in to th e
lim bic system an d p re ron tal cortex.
(2) Hyp eractivity o th e m esolim bic tract is associated with the p ositive sym p tom s (e.g.,
h allu cin ation s) o sch izop h ren ia; h yp oactivity o the m esocortical tract is associ-
ated with th e n egative sym p tom s (e.g., ap ath y) o sch izop hren ia (see Ch ap ter 11).

C. Norepinephrine, a catech olam in e, p lays a role in mood, anxiety, arousal, learning, an d memory.
1. Synthesis
a. Like dop am in ergic n eu ron s, n oradren ergic n eu ron s syn th esize dop am in e.
b. Dop am in e β-h yd roxylase, p resen t in n oradren ergic n eu ron s, con verts th is dop am in e
to n orep in ep h rin e.
2. Localization. Most n oradren ergic n eu ron s (ap p roxim ately 10,000 p er h em isp h ere in th e
brain ) are located in th e locus ceruleus (Figure 4.1B).

D. Serotonin, an in dolam in e, p lays a role in mood, sleep, sexuality, an d impulse control. Elevation
o seroton in is associated with im p roved m ood an d sleep but decreased sexu al u n ction (p ar-
ticu larly delayed orgasm ). Very h igh levels are associated with psychotic symptoms (see Ch ap ter
11). Decreased seroton in is associated with poor im pulse con trol, dep ression , an d poor sleep.
1. Synthesis. The am ino acid tryptophan is converted to serotonin (also known as 5-hydroxytrypta-
mine [5-HT]) by the enzym e tryptophan hydroxylase as well as by an am in o acid decarboxylase.
2. Localization. Most seroton ergic cell bodies in th e brain are con tain ed in th e dorsal raphe
nucleus in the upper pons and lower midbrain (Figure 4.1C).
3. Antidepressants and serotonin. Heterocyclic an tidep ressan ts (HCAs ), selective seroton in
an d seroton in an d n orep in ep h rin e reu p take in h ib itors (SSRIs an d SNRIs ), an d m on o-
am in e oxidase in h ibitors (MAOIs ) u ltim ately in crease th e p resen ce o seroton in an d
RISE USMLE NEPAL

n orep in ep h rin e in th e syn ap tic cle t (Ch ap ter 16).


a. HCAs an d SNRIs block reu p take o seroton in an d n orep in ep h rin e, an d SSRIs su ch as
lu oxetin e (Prozac) selectively block reu p take o seroton in by th e p resyn ap tic n eu ron .
b. MAOIs p reven t th e d egrad ation o seroton in an d n orep in ep h rin e by MAO.

E. Histamine
1. Histam in e, an ethylamine , is a ected by p sych oactive dru gs.
2. Histam in e recep tor b lockade with dru gs su ch as an tip sych otics an d tricyclic an tidep res-
san ts is associated with com m on sid e e ects o th ese d ru gs su ch as sedation an d increased
appetite lead in g to weigh t gain .

F. Acetylcholine (Ach), a qu atern ary am in e, is th e tran sm itter u sed by nerve–skeleton–muscle


junctions .
1. Degeneration of cholinergic neurons is associated with Alzheimer’s disease, Down’s syn-
drome, an d movement and sleep disorders (e.g., d ecreased REM sleep, see Ch ap ter 10).
2. Cholinergic neurons syn th esize Ach rom acetyl coen zym e A an d ch olin e u sin g th e en zym e
choline acetyltransferase .
3. Th e nucleus basalis of Meynert is a brain area in volved in p rodu ction o Ach .
4. Acetylcholinesterase (Ach E) breaks Ach down in to ch olin e an d acetate.
Chapter 4 Genetics, Anatomy, and Biochemistry of Behavior 41

5. Blockin g th e action o AchE with d ru gs su ch as donepezil (Aricep t), rivastigmine (Exelon ),


an d galantamine (Rem in yl) m ay delay th e p rogression o Alzh eim er’s disease bu t can n ot
reverse th e u n ction alread y lost.
6. Blockade of muscarinic Ach receptors with dru gs su ch as an tip sych otics an d tricyclic an ti-
dep ressan ts resu lts in th e classic “an tich olin ergic” adverse e ects seen with th e u se o
th ese dru gs, in clu din g dry m ou th , blu rred vision , u rin ary h esitan cy, an d con stip ation .
Use o th ese agen ts can also resu lt in cen tral an tich olin ergic e ects su ch as con u sion an d
m em ory p rob lem s.
7. An tich olin ergic agen ts are com m on ly u sed to treat th e Parkin son -like sym p tom s cau sed
by an tip sych otic agen ts (see Section IV.B.4.a. above).
HELP OTHERS SO THAT GOD WILL HELP YOU.

V. AMINO ACID NEUROTRANSMITTERS


Th ese n eu rotran sm itters are in volved in m ost syn ap ses in th e brain an d in clu de glutamate,
γ-aminobutyric acid (GABA), and glycine .

A. Glutamate
1. Glu tam ate is an excitatory n eu rotran sm itter th at con tribu tes to th e p ath op h ysiology o
neurodegenerative illnesses su ch as Alzh eim er’s d isease an d sch izop h ren ia.
a. Th e m ech an ism o th is association in volves activation o th e glu tam ate recep tor
N-methyl-d -aspartate (NMDA) by su stain ed elevation o glu tam ate.
b. Su ch activation resu lts in calciu m ion s en terin g n eu ron s leadin g to n erve cell degen -
eration an d d eath th rou gh excitotoxicity.
c. Memantine (Nam en d a), an NMDA recep tor an tagon ist, u ltim ately blocks this influx of
calcium an d is in dicated or p atien ts with m oderate to severe Alzheimer’s d isease.

B. GABA
1. GABA is th e p rin cip al inhibitory n eu rotran sm itter in th e CNS. It is syn th esized rom glu ta-
m ate by th e en zym e glu tam ic acid d ecarboxylase, wh ich n eed s vitam in B6 (pyrid oxin e) as
a co actor.
2. GABA is closely in volved in th e action o an tian xiety agen ts su ch as benzodiazepines (e.g.,
d iazep am [Valiu m ]) an d barbiturates (e.g., secobarb ital [Secon al]). Ben zodiazep in es an d
b arb itu rates in crease th e a in ity o GABA or its GABAA-binding site , allowin g m ore ch lo-
rid e to en ter th e n eu ron . Th e ch lorid e-laden n eu ron s b ecom e h yp erp olarized an d in h ib-
ited , d ecreasin g n eu ron al irin g an d u ltim ately decreasin g an xiety. An ticon vu lsan ts also
RISE USMLE NEPAL

p oten tiate th e activity o GABA.

C. Glycine is an in h ibitory n eu rotran sm itter ou n d p rim arily in th e sp in al cord. Glycin e


works on its own an d as a regu lator o glu tam ate activity.

VI. NEUROPEPTIDES

A. Endogenous opioids such as enkephalins, endorphins, dynorphins, and endomorphins are p ro-
d u ced by th e b rain itsel . Th ey act to d ecrease p ain an d an xiety an d h ave a role in addiction
an d m ood.

B. Placebo effects (see Ch ap ter 25) m ay b e m ediated by th e en d ogen ou s op ioid system . For
exam p le, p rior treatm en t with an op ioid recep tor blocker su ch as n aloxon e can b lock p la-
ceb o e ects.
Review Test

Directions: Each o th e n u m bered item s or in com p lete statem en ts in th is section is ollowed by


an swers or by com p letion s o th e statem en t. Select th e one lettered an swer or com p letion th at
is best in each case.
HELP OTHERS SO THAT GOD WILL HELP YOU.

1. A 45-year-old m ale p atien t b ecom es 4. A 65-year-old em ale p atien t h as h ad a


dep ressed ollowin g a h ead in ju ry. Th e stroke a ectin g th e le t h em isp h ere o h er
area(s) o th e brain m ost likely to b e a ected brain . Wh ich o th e ollowin g u n ction s is
in th is p atien t is (are) th e m ost likely to b e a ected by th e stroke?
(A) righ t p arietal lob e (A) Percep tion
(B) basal gan glia (B) Mu sical ability
(C) h ip p ocam p u s (C) Sp atial relation s
(D) reticu lar system (D) Lan gu age
(E) am ygd ala (E) Artistic ab ility
(F) le t ron tal lobe
5. Wh ich o th e ollowin g two stru ctu ral
2. A 43-year-old m an p resen ts to th e en tities con n ect th e cerebral h em isp h eres?
em ergen cy d ep artm en t o a large h osp ital. (A) Basal gan glia an d an terior com m issu re
He is very an xiou s an d com p lain s o (B) An terior com m issure an d reticular system
abdom in al cram p s an d diarrh ea. Th e (C) Reticu lar system an d corp u s callosu m
p h ysician observes in ten se u sh in g o th e (D) Hip p ocam p al com m issu re an d corp u s
m an’s skin . In th is p atien t, a 24-h ou r u rin e callosu m
stud y is m ost likely to reveal elevated levels (E) Am ygdala an d h ab en u lar com m issu re
o
6. A 23-year-old p atien t sh ows side e ects
(A) acetylch olin e
su ch as sed ation , in creased ap p etite,
(B) 5-h ydroxyin d oleacetic acid
an d weigh t gain wh ile bein g treated with
(5-HIAA)
an tip sych otic m edication . O th e ollowin g,
(C) glycin e
th e m ech an ism m ost closely associated with
(D) van illylm an delic acid (VMA)
th ese e ects is
(E) h om ovan illic acid (HVA)
(A) blockad e o seroton in recep tors
RISE USMLE NEPAL

3. In a clin ical exp erim en t, a 48-year-old (B) blockad e o d op am in e recep tors


em ale p atien t with ch ron ic p ain wh o, (C) blockad e o n orep in ep h rin e recep tors
in th e p ast, h as resp on d ed to p laceb os (D) blockad e o h istam in e recep tors
is given n aloxon e. Sh ortly th erea ter th e (E) decreased availability o seroton in
p atien t is given an in ert su b stan ce th at sh e
b elieves is a p ain killer. A ter th e p a tien t 7. A 3-year-old girl wh o h ad been develop in g
receives th e in ert su b stan ce, h er p a in is typically sin ce birth begin s to with draw
m ost likely to socially an d th en stop s sp eakin g altogeth er.
Also, in stead o p u rp ose u l h an d m ovem en ts,
(A) in crease
th e ch ild h as begu n to sh ow rep etitive
(B) decrease
h an d wrin gin g beh avior. Th e ch rom osom e
(C) be u n ch an ged
m ost likely to be in volved in th is disorder is
(D) resp on d to lower doses o op ioids th an
ch rom osom e
p reviou sly
(E) ail to resp on d to op ioids in th e u tu re (A) 1
(B) 16
(C) 18
(D) 21
(E) X

42
Chapter 4 Genetics, Anatomy, and Biochemistry of Behavior 43

8. Th e m ajor n eu rotran sm itter im p licated in 13. A 55-year-old wom an was diagn osed
both Alzh eim er’s disease an d sch izop h ren ia is with sch izop h ren ia at th e age o 22. I th is
(A) seroton in diagn osis was ap p rop riate, th e volu m e o
(B) n orep in ep h rin e th e h ip p ocam p u s, th e size o th e cereb ral
(C) dop am in e ven tricles, an d glu cose u tilization in th e
(D) γ-am in obu tyric acid (GABA) ron tal cortex o th is p atien t are n ow m ost
(E) acetylch olin e (Ach ) likely to b e, resp ectively
(F) glu tam ate (A) in creased, in creased, in creased
(B) decreased , decreased , d ecreased
9. Th e m ajor n eu rotran sm itter in volved (C) decreased , decreased , in creased
in th e an tidep ressan t action o u oxetin e (D) decreased , in creased , decreased
HELP OTHERS SO THAT GOD WILL HELP YOU.

(Prozac) is (E) in creased, decreased, in creased


(A) seroton in
(B) n orep in ep h rin e 14. An 80-year-old em ale p atien t h as
(C) dop am in e a restin g trem or o h er le t h an d, little
(D) γ-am in obu tyric acid (GABA) exp ression in h er ace, an d p roblem s takin g
(E) acetylch olin e (Ach ) a f rst step wh en sh e h as been stan din g still.
(F) glu tam ate Th e area(s) o th e brain m ost likely to b e
a ected in th is p atien t is (are) th e
10. Th e n eu rotran sm itter m etabolized to (A) righ t p arietal lob e
5-HIAA (5-h yd roxyin doleacetic acid) is (B) basal gan glia
(A) seroton in (C) h ip p ocam p u s
(B) n orep in ep h rin e (D) reticu lar system
(C) dop am in e (E) am ygd ala
(D) γ-am in obu tyric acid (GABA) (F) le t ron tal lobe
(E) acetylch olin e (Ach )
(F) glu tam ate Questions 15 and 16

11. A 25-year-old m ale p atien t su stain s A 69-year-old orm er ban k p residen t can n ot
a seriou s h ead in ju ry in an au tom obile tell you th e n am e o th e cu rren t p resid en t
acciden t. He h ad b een aggressive an d an d h as d i icu lty id en ti yin g th e wom an sit-
assau ltive, b u t a ter th e accid en t, h e is tin g n ext to h im (h is wi e). He b egan h avin g
p lacid an d coop erative. He also m akes m em ory p roblem s 3 years ago.
in ap p rop riate su ggestive com m en ts to th e
n u rses an d m astu rb ates a great d eal. Th e 15. Atrop h y o wh ich area(s) o th e brain is
area(s) o th e brain m ost likely to be in ju red (are) m ost likely to b e seen in th is p atien t?
in th is p atien t is (are) th e (A) righ t p arietal lob e
RISE USMLE NEPAL

(A) righ t p arietal lob e (B) basal gan glia


(B) basal gan glia (C) h ip p ocam p u s
(C) h ip p ocam p u s (D) reticu lar system
(D) reticu lar system (E) am ygd ala
(E) am ygd ala (F) le t ron tal lobe
(F) le t ron tal lobe
16. Th e p ostm ortem brain biop sy o th is
12. A 35-year-old em ale p atien t rep orts p atien t is m ost likely to sh ow
th at sh e h as d i f cu lty sleep in g ever sin ce (A) in creased 3-m eth oxy-4-
sh e su stain ed a con cu ssion in a su bway h ydroxyp h en ylglycol (MHPG)
acciden t. Th e area(s) o th e brain m ost likely (B) righ t ron tal h yp ertrop h y
to be a ected in th is p atien t is (are) th e (C) decreased calciu m levels
(A) righ t p arietal lob e (D) decreased h om ovan illic acid (HVA)
(B) basal gan glia (E) dep osition o β-am yloid
(C) h ip p ocam p u s
(D) reticu lar system
(E) am ygd ala
(F) le t ron tal lobe
44 BRS Behavioral Science

17. A 28-year-old m ale p atien t is brou gh t to 20. A 24-year-old m an su stain s a h ead in ju ry


th e em ergen cy room a ter a f gh t in wh ich in an au tom obile accid en t. His ath er relates
h e attacked a m an wh o cu t in to h is lin e at th at p rior to th e accid en t, th e p atien t was
th e su p erm arket ch eckou t. In th e em ergen cy resp ect u l, m od est, con trolled , an d h ard
room , h e rem ain s assau ltive an d com b ative. workin g. In th e h osp ital, th e p atien t is ru d e
Th e bod y u ids o th is p atien t are m ost likely to th e n u rses an d aides, loses h is tem p er
to sh ow with th e sligh test p rovocation , an d re u ses
(A) in creased 3-m eth oxy-4- to wear a h osp ital gown or an yth in g else.
h ydroxyp h en ylglycol (MHPG) Th ese beh avioral ch an ges a ter th e accid en t
(B) decreased MHPG in d icate th at th e area o th e b rain m ost likely
(C) in creased 5-h yd roxyin doleacetic acid to h ave been in ju red in th is p atien t is th e
HELP OTHERS SO THAT GOD WILL HELP YOU.

(5-HIAA) (A) dorsolateral con vexity o th e ron tal lobe


(D) decreased 5-HIAA (B) h yp oth alam u s
(E) decreased h om ovan illic acid (HVA) (C) orb ito ron tal cortex
(D) reticu lar system
18. A 30-year-old wom an wh o is (E) am ygd ala
with d rawin g rom h eroin sh ows in ten se (F) n u cleu s basalis o Meyn ert
an xiety, in creased p u lse, elevated blood
p ressu re, an d a h an d trem or. Her sym p tom s 21. An alysis o th e b lood p lasm a o a
im p rove wh en sh e is given clon id in e, 45-year-old m ale p atien t sh ows in creased
an alp h a 2-adren ergic recep tor agon ist. con cen tration o h om ovan illic acid (HVA).
Th e area(s) o th e brain m ost likely to b e Th is elevation is m ost likely to be associated
in volved in th e im p rovem en t in th is p atien t’s with wh ich o th e ollowin g con d ition s?
sym p tom s is (are) th e (A) Parkin son’s disease
(A) righ t p arietal lob e (B) Dep ression
(B) basal gan glia (C) Bu lim ia
(C) locu s ceru leu s (D) Ph eoch rom ocytom a
(D) rap h e n u clei (E) Sch izop h ren ia
(E) am ygd ala
(F) su b stan tia n igra 22. Rem em b erin g th at sch ool closes early
b e ore Th an ksgivin g Day every year is an
19. A very an xiou s 25-year-old p atien t is exam p le o wh ich o th e ollowin g typ es o
exam in ed in th e em ergen cy room . Th ere is m em ory?
n o eviden ce o p h ysical illn ess. I it cou ld be (A) Sem an tic
m easu red, th e γ-am in obu tyric acid (GABA) (B) Ep isodic
activity in th e b rain o th is p atien t wou ld (C) Proced u ral
m ost likely be (D) Workin g
RISE USMLE NEPAL

(A) in creased
(B) decreased 23. A 55-year-old p atien t wh o is takin g
(C) u n ch an ged tiotrop iu m brom ide (Sp iriva) or ch ron ic
(D) h igh er th an th e activity o seroton in obstru ctive p u lm on ary disease h as m em ory
(E) h igh er th an th e activity o dop am in e p roblem s d u e to th e agen t’s action on wh ich
o the ollowin g recep tors?
(A) Adren ergic
(B) Ch olin ergic
(C) Dop am in ergic
(D) Histam in ergic
(E) Seroton ergic
Chapter 4 Genetics, Anatomy, and Biochemistry of Behavior 45

24. A 6-year-old ch ild sh ows seizu res, 26. A 30-year-old m an wh o h as h ad m an y


cogn itive d e ects, an d au tistic b eh avior. Th e n egative li e exp erien ces becom es u p set
ch ild also sh ows raised discolored areas on wh en h e sees p h otograp h s o h im sel taken
h er oreh ead ( oreh ead p laqu es). Wh ich o du rin g th ese tim es. Th e brain area m ost
th e ollowin g ch rom osom es is m ost likely likely to be activated by th ese p h otograp h s is
to be in volved in th e etiology o th is ch ild’s th e
sym p tom s? (A) dorsolateral con vexity o th e ron tal lobe
(A) 1 (B) h yp oth alam u s
(B) 16 (C) orb ito ron tal cortex
(C) 18 (D) reticu lar system
(D) 21 (E) am ygd ala
HELP OTHERS SO THAT GOD WILL HELP YOU.

(E) X (F) n u cleu s basalis o Meyn ert

25. A 72-year-old m an with Alzh eim er’s 27. Th e brain p ath way m ost closely
d isease is b ein g treated with m em an tin e. associated with th e disp lay o n egative
Wh at is believed to be th e basis o th e sym p tom s in sch izop h ren ia is th e
th erap eu tic action o m em an tin e on n eu ron s (A) m esocortical tract
in th e b rain ? (B) m esolim b ic tract
(A) To in h ibit th e action o (C) tu beroin u n dibu lar tract
acetylch olin esterase (D) ceru locortical tract
(B) To block th e in lu x o calciu m (E) rap h e cortical tract
(C) To in h ibit th e action o acetylch olin e
(D) To in crease th e in lu x o glu tam ate
(E) To acilitate th e in lu x o calciu m
RISE USMLE NEPAL
An swers an d Exp lan ation s

Typical Board Question


The answer is A. Dam age to th e righ t p arietal lob e can resu lt in im p aired visu al–sp atial p rocess-
in g. Th is can lead to p rob lem s copyin g sim p le drawin gs an d n eglect o th e le t side as seen in
th is p atien t.
HELP OTHERS SO THAT GOD WILL HELP YOU.

1. The answer is F. O th e listed brain areas, d ep ression is m ost likely to b e associated with
d am age to th e le t ron tal lobe.
2. The answer is D. A 24-h ou r u rin e stu dy is m ost likely to reveal elevated levels o VMA, a
m etab olite o n orep in ep h rin e. An xiety, ab dom in al cram p s an d d iarrh ea, an d skin lu sh in g
are sym p tom s o p h eoch rom ocytom a, a n orep in ep h rin e-secretin g adren al tu m or. Th is
p ictu re is n ot seen with elevated levels o oth er n eu rotran sm itter m etabolites.
3. The answer is C. Sin ce th e p lacebo respon se is based in part on activation o the
en dogen ou s op ioid system , it will be blocked by n aloxon e, an d this p atien t’s pain will be
u n ch an ged. Th is exp erim en t will n ot n ecessarily a ect her respon se to opioids in the uture.
4. The answer is D. Dom in an ce or lan gu age in both righ t-h an ded an d le t-h an ded p eop le is
u su ally in th e le t h em isp h ere o th e brain . Percep tion , m u sical ability, artistic ability, an d
sp atial relation s p rim arily are u n ction s o th e righ t sid e o th e brain .
5. The answer is D. Th e corp u s callosu m an d th e h ip p ocam p al, h aben u lar, an d an terior
com m issu res con n ect th e two h em isp h eres o th e b rain . Th e basal gan glia, reticu lar
system , an d am ygd ala do n ot h ave th is u n ction .
6. The answer is D. Sedation , in creased appetite, an d weight gain are side e ects o treatm en t
with certain an tipsychotic agen ts. The m echan ism m ost closely associated with these side
e ects is blockade o histam in e receptors sin ce these an tipsychotics are n ot speci ic or
dopam in e blockade. Blockade o dopam in e receptors by these antipsychotic m edications is
associated with side e ects such as parkinsonism -like sym ptom s an d elevated prolactin levels.
7. The answer is E. Th is 3-year-old girl is sh owin g sign s o Rett’s disorder th at is lin ked to th e X
ch rom osom e. Rett’s d isord er is ch aracterized by loss o social skills a ter a p eriod o typ ical
RISE USMLE NEPAL

un ction in g as well as h an d-wrin gin g an d breath in g abn orm alities (see also Ch ap ter 15).
8. The answer is F. Wh ile acetylch olin e (Ach ) is th e m ajor n eu rotran sm itter im p licated in
Alzh eim er’s disease, abn orm alities in glu tam ate are seen in both Alzh eim er’s disease an d
sch izop h ren ia.
9. The answer is A. Blockade o seroton in reu p take by p resyn ap tic n eu ron s is th e p rim ary
action o th e an tidep ressan t lu oxetin e.
10. The answer is A. Seroton in is m etabolized to 5-HIAA.
11. The answer is E. Th e p atien t is sh owin g evid en ce o th e Klü ver-Bu cy syn drom e, wh ich
in clu d es h yp ersexu ality an d docility an d is associated with dam age to th e am ygdala.
12. The answer is D. Sleep –arou sal m ech an ism s are a ected by dam age to th e reticu lar
system .
13. The answer is D. Alth ou gh n eu roim agin g can n ot be u sed to diagn ose p sych iatric disorders,
brain s o p atien ts with sch izop h ren ia su ch as th is wom an are likely to sh ow d ecreased
volu m e o lim b ic stru ctu res su ch as th e h ip p ocam p u s; in creased size o cerebral ven tricles
du e, in p art, to b rain sh rin kage; an d decreased glu cose u tilization in th e ron tal cortex.

46
Chapter 4 Genetics, Anatomy, and Biochemistry of Behavior 47

14. The answer is B. Th is 80-year-old em ale p atien t is sh owin g sign s o Parkin son’s disease
(e.g., a restin g trem or, little acial exp ression , an d p roblem s in itiatin g m ovem en t). Th is
disord er is associated with ab n orm alities o th e b asal gan glia.
15. The answers is C. 16. The answer is E. This patien t is sh owin g eviden ce o Alzh eim er’s
disease. O th e listed brain areas, th e m ajor on e im p licated in Alzheim er’s disease is th e
h ip p ocam p u s. Am yloid p laqu es are seen on brain biop sy o Alzheim er’s disease p atien ts.
17. The answer is D. Assau ltive, im p u lsive, aggressive beh avior like th at seen in th is
28-year-old m ale p atien t is associated with decreased levels o seroton in in th e
brain . Levels o 5-HIAA (5-h yd roxyin d oleacetic acid), th e m ajor m etabolite o
seroton in , h ave been sh own to b e decreased in th e body lu ids o violen t, aggressive,
HELP OTHERS SO THAT GOD WILL HELP YOU.

im p u lsive in dividu als as well as dep ressed in dividu als. MHPG (3-m eth oxy-4-
h ydroxyp h en ylglycol), a m etab olite o n orep in ep h rin e, is decreased in severe
dep ression , wh ile h om ovan illic acid (HVA), a m etabolite o d op am in e, is d ecreased in
Parkin son’s disease an d d ep ression .
18. The answer is C. Th e e ectiven ess o clon idin e in treatin g with drawal sym p tom s
associated with th e u se o op ioids an d sedatives is believed to be du e to its action on
alp h a 2-ad ren ergic recep tors, or exam p le, red u cin g th e irin g rate o n orad ren ergic
n eu ron s, m ost o wh ich are located in th e locu s ceru leu s.
19. The answer is B. g-Am in ob u tyric acid (GABA) is an in h ibitory am in o acid
n eu rotran sm itter in th e CNS. Th u s, th e activity o GABA in th e brain o th is an xiou s
p atien t is likely to be decreased. Decreased seroton in an d in creased dop am in e are also
in volved in an xiety (Table 4.4).
20. The answer is C. Beh avioral ch an ges su ch as decreased im p u lse con trol, p oor social
beh avior, an d lack o ch aracteristic m odesty in d icate th at th e area o th e b rain m ost
likely to h ave b een in ju red in th is p atien t is th e orbito ron tal cortex. Lesion s o th is
brain area resu lt in d isin h ib ition , in ap p rop riate beh avior, an d p oor ju d gm en t. In
con trast, lesion s o th e dorsolateral con vexity o th e ron tal lobe resu lt in decreased
execu tive u n ction in g (e.g., m otivation , con cen tration , an d atten tion ). Th e
h yp oth alam u s is associated with h om eostatic m ech an ism s an d th e reticu lar system
with con sciou sn ess an d sleep. Dam age to th e am ygdala resu lts in decreased, n ot
in creased, aggression . Th e n u cleu s basalis o Meyn ert is a site o Ach p rodu ction ; its
dam age cou ld resu lt in d e icits in in tellectu al u n ction in g.
21. The answer is E. In creased body lu id level o hom ovan illic acid (HVA), a m ajor
m etabolite o dopam in e, is seen in sch izoph ren ia. Decreased HVA is seen in Parkin son’s
RISE USMLE NEPAL

disease, depression , an d in m edicated schizophren ic patien ts. In creased van illylm an delic
acid (VMA), a m etabolite o n orep in ep h rin e, is seen in p heoch rom ocytom a. Decreased
body luid level o 5-HIAA, a m etabolite o seroton in , is seen in depression an d in bulim ia
(Table 4.5).
22. The answer is A. Rem em berin g th at sch ool closes early be ore Th an ksgivin g Day every
year is an exam p le o sem an tic m em ory. Sem an tic m em ory is a typ e o declarative
m em ory th at in volves rem em berin g gen eral kn owledge abou t th e world. Ep isodic
m em ory in volves rem em berin g p erson ally exp erien ced even ts, p rocedu ral m em ory
in volves rem em berin g th in gs on e does au tom atically, an d workin g m em ory in volves
rem em berin g recen t in orm ation .
23. The answer is B. Decreased availability o acetylch olin e by blockade o m u scarin ic
acetylch olin e recep tors (i.e., an tich olin ergic activity) in th e CNS is associated with
m em ory p rob lem s. Blockade o ad ren ergic, d op am in ergic, h istam in ergic, an d
seroton ergic recep tors are less likely to b e associated with m em ory p rob lem s.
24. The answer is B. Ch rom osom e 16 an d ch rom osom e 9 are b oth associated with
tu berou s sclerosis. Seizu res, cogn itive de ects, au tistic beh avior, an d oreh ead p laqu es
in th is 6-year-old ch ild are seen in th is disorder.
48 BRS Behavioral Science

25. The answer is B. Th e th erap eu tic action o m em an tin e in Alzh eim er’s d isease is b elieved to
be to d ecrease th e in lu x o glu tam ate, u ltim ately b lockin g th e in lu x o calciu m , wh ich can
lead to n erve cell degen eration an d d eath . In con trast to a grou p o dru gs also u sed to treat
Alzh eim er’s, th at is, th e acetylch olin esterase in h ib itors, m em an tin e d oes n ot directly a ect
acetylch olin e.
26. The answer is E. Th e am ygdala is an im p ortan t b rain area or th e evalu ation o sen sory
stim u li with em otion al sign i ican ce. Th u s, th e brain area m ost likely to be activated by
th ese p h otos is th e am ygdala.
27. The answer is A. Dop am in e h yp oactivity in th e m esocortical tract is associated with th e
n egative sym p tom s o sch izop h ren ia (an d see Ch ap ter 11). Dop am in e h yp eractivity in th e
HELP OTHERS SO THAT GOD WILL HELP YOU.

m esolim b ic tract is associated with th e p ositive sym p tom s o sch izop h ren ia. Dop am in e
acts on th e tu beroin u n d ibu lar tract to in h ibit th e secretion o p rolactin rom th e an terior
p itu itary. Th e cerulocortical tract is associated with th e action o n orep in ep h rin e, wh ile th e
rap h e cortical tract is associated with th e action o seroton in .
RISE USMLE NEPAL
Biological Assessm en t o
c ha pte r
5 Patien ts with Psych iatric
Sym p tom s
HELP OTHERS SO THAT GOD WILL HELP YOU.

Typical Board Question


A college-edu cated 72-year-old em ale p atien t h as scored 15 on th e Folstein Min i–Men tal
State Exam in ation . From th is score, th e p h ysician can con clu de th at th is p atien t p robably
(A) is sh owin g typ ical b eh avior
(B) can n ot calcu late sim p le su m s
(C) is cogn itively im p aired
(D) sh ou ld be p laced in an assisted livin g acility
(E) h as lower th an n orm al in telligen ce
(See “An sw ers an d Explan ation s” at th e en d of th e ch apter.)

I. OVERVIEW
Biological alteration s an d ab n orm alities can u n derlie p sych iatric sym p tom s an d in lu en ce th eir
occu rren ce. A variety o laboratory stu d ies are u sed clin ically to iden ti y su ch alteration s an d
abn orm alities in p atien ts.

II. MEASUREMENT OF BIOGENIC AMINES AND PSYCHOTROPIC


RISE USMLE NEPAL

DRUGS

A. Altered levels o b iogen ic am in es an d th eir m etabolites occu r in som e p sych iatric con dition s
(see Tab les 4.2 an d 4.3).

B. Plasm a levels o som e an tip sych otic an d an tidep ressan t agen ts are m easu red to evalu ate
patient compliance or to determ in e wh eth er therapeutic blood levels o th e agen t h ave been
reach ed.

C. Laboratory tests also are used to monitor patients for complications of pharmacotherapy.
1. Patien ts takin g certain m ood stabilizers, or exam p le, carbam azep in e (Tegretol), or an ti-
p sych otics, or exam p le, clozap in e (Clozaril), m u st be observed or blood abn orm alities
su ch as agranulocytosis (very low, e.g., <2,000, wh ite blood cell cou n t or gran u locyte cou n t
<1,000).

49
50 BRS Behavioral Science

2. Liver function tests are u sed in p atien ts b ein g treated with carbam azep in e an d valp roic
acid (m ood stabilizers).
3. Thyroid function an d kidney function tests sh ou ld be u sed in p atien ts wh o are bein g treated
with th e m ood stab ilizer lithium. Patien ts takin g lith iu m can develop hypothyroidism an d,
occasion ally, h yp erth yroid ism .
4. Lith iu m levels sh ou ld also be m on itored regu larly becau se o th e dru g’s narrow therapeu-
tic range (see Ch ap ter 16).

III. EVALUATING ENDOCRINE FUNCTION


HELP OTHERS SO THAT GOD WILL HELP YOU.

A. Dexamethasone suppression test (DST). In a n orm al p atien t with a n orm al h yp oth alam ic–
adren al–p itu itary axis, d exam eth ason e, a syn th etic glu cocorticoid, su p p resses th e secretion
o cortisol.
1. In con trast, ap p roxim ately on e-h al o th e p atien ts with m ajor dep ressive disorder h ave a
p ositive DST (i.e., th is su p p ression is lim ited or absen t).
2. Becau se p ositive in din gs are n ot sp eci ic an d n on su p p ression is seen in con dition s oth er
th an m ajor dep ressive disorder, th e DST h as lim ited clin ical u se u ln ess.

B. Thyroid u n ction tests are u sed to screen or hypothyroidism an d hyperthyroidism, wh ich can
m im ic d ep ression an d an xiety, resp ectively.
1. Ph ysical sym p tom s o h yp oth yroidism in clu de atigu e, weigh t gain , edem a, h air loss, an d
cold in toleran ce.
2. Ph ysical sym p tom s o h yp erth yroidism in clu de rap id h eartbeat (“p alp itation s”), lu sh in g,
ever, weigh t loss, an d diarrh ea.

C. Patien ts with d ep ression m ay h ave oth er endocrine irregularities , su ch as redu ced resp on se
to a ch allen ge with th yrotrop in -releasin g h orm on e, an d abn orm alities in growth h orm on e,
m elaton in , an d gon ad otrop in .

D. Psych iatric sym p tom s are associated with oth er en docrin e disorders, su ch as Addison’s dis-
ease (hypocortisolism), Cushing’s disease (hypercortisolism), an d en zym e d isord ers su ch as
acute intermittent porphyria.
1. Addison’s disease
a. Ph ysical sign s an d sym p tom s in clu de hyperpigmentation of the skin, p articu larly in skin
creases, low b lood p ressu re, p ain , ain tin g, h yp oglycem ia, diarrh ea, an d vom itin g.
b. Psych iatric sym p tom s in clu de atigu e, d ep ression , p sych osis, an d con u sion .
RISE USMLE NEPAL

2. Cushing’s disease
a. Ph ysical sign s an d sym p tom s in clu de rou n d “moon” face , bru isin g, p u rp le striae on th e
skin , sweatin g, acial h air, h yp erten sion , at on th e back o th e n eck (“bu alo h u m p”),
an d a p ositive DST.
b. Psych iatric sym p tom s in clu de elevated m ood, p sych osis, an xiety, an d dep ression .
3. Acute intermittent porphyria
a. Ph ysical sign s an d sym p tom s in clu d e abd om in al cram p s, diarrh ea an d vom itin g, sei-
zu res, cardiac arrh yth m ias, lu sh in g, an d purple/red discoloration of the urine du e to
elevated porphobilinogen.
b. Psych iatric sym p tom s in clu de p aran oid delusions an d h allu cin ation s as well as dep res-
sion an d an xiety.

IV. NEUROIMAGING AND ELECTROENCEPHALOGRAM STUDIES


Stru ctu ral brain abn orm alities an d EEG ch an ges m ay be associated with sp eci ic beh avioral dis-
orders (Tab le 5.1).
Chapter 5 Biological Assessment of Patients with Psychiatric Symptoms 51

t a b l e 5.1 Neuroimaging and Electroencephalography in the Biological Evaluation of


Psychiatric Patients

Specific Test or Measure Uses and Characteristics

Computed tomography (CT) Identifies anatomically based brain changes (e.g., enlarged brain
ventricles) seen in cognitive disorders, such as Alzheimer’s disease, as
well as in schizophrenia
Nuclear magnetic resonance imaging Identifies demyelinating disease (e.g., multiple sclerosis)
(NMRI) Shows the biochemical condition of neural tissues without exposing the
patient to ionizing radiation
Positron emission tomography (PET) or Localizes areas of the brain that are physiologically active during specific
functional MRI (fMRI) tasks by characterizing and measuring metabolism of glucose in neural
HELP OTHERS SO THAT GOD WILL HELP YOU.

tissue
Measures specific neurotransmitter receptors
Requires use of a cyclotron
Single photon emission tomography (SPECT) Obtains similar data to PET or fMRI but is more practical for clinical use
because it uses a standard gamma camera rather than a cyclotron
Electroencephalogram (EEG) Measures electrical activity in the cortex
Is useful in diagnosing epilepsy and in differentiating delirium (often
abnormal EEG) from dementia (often normal EEG)
Shows, in patients with schizophrenia, decreased alpha waves, increased
theta and delta waves, and epileptiform activity
Evoked EEG (evoked potentials) Measures electrical activity in the cortex in response to tactile, auditory, or
visual stimulation
Is used to evaluate vision and hearing loss in infants and brain responses
in comatose and suspected brain-dead patients

V. NEUROPSYCHOLOGICAL TESTS
A. Neu rop sych ological tests are d esign ed to assess gen eral in telligen ce, m em ory, reason in g,
orien tation , p ercep tu om otor p er orm an ce, lan gu age u n ction , atten tion , an d con cen tra-
tion in p atien ts with su sp ected n eu rologic p roblem s, su ch as dem en tia an d brain dam age
(Tab le 5.2).

B. In su ch p atien ts, th e Folstein Mini–Mental State Examination (Tab le 5.3) is d esign ed to ol-
low th e im p rovem en t or d eterioration in cogn itive u n ction , an d th e Glasgow Coma Scale
RISE USMLE NEPAL

(Tab le 5.4) is d esign ed to assess th e level o con sciou sn ess by ratin g p atien t resp on siven ess.

t a b l e 5.2 Neuropsychological Tests Used in Psychiatry

Test Used to Identify or Evaluate


Halstead-Reitan battery Brain lesion localization and effects
Luria-Nebraska neuropsychological battery Cerebral dominance
Types of brain dysfunction, such as dyslexia
Bender Visual Motor Gestalt Visual and motor ability through the reproduction of designs
Digit symbol substitution Dementia, depression, and age-related cognitive decline
Boston naming Dementia
Wisconsin card sort Executive function
Stroop color–word Directed attention
Folstein Mini–Mental State Examination Delirium and dementia (Table 5.3)
Glasgow Coma Scale Level of consciousness (Table 5.4)
52 BRS Behavioral Science

t a b l e 5.3 Folstein Mini–Mental State Examinationa

Skill Evaluated Sample Instructions to the Patient Maximum Score a

Orientation Tell me where you are and what day it is 10


Language Name the object that I am holding 8
Attention and concentration Subtract 7 from 100 and then continue to subtract 7s 5
Registration Repeat the names of these three objects 3
Recall After 5 min, recall the names of these three objects 3
Construction Copy this design 1
Maximum total score = 30; total score of 23 or higher suggests competence; total score of 18 or lower suggests incompetence (Applebaum, 2007,
a
HELP OTHERS SO THAT GOD WILL HELP YOU.

N Engl J Med, 357) (see Chapter 26).


Adapted from Fadem B. Behavioral Science in Medicine. 2nd ed. Lippincott Williams & Wilkins; 2012:64.

VI. OTHER TESTS


A. Drug-assisted interview
1. Adm in istration o a sedative , su ch as am obarbital sodiu m (“th e Am ytal in terview”), p rior
to th e clin ical in terview m ay be u se u l in determ in in g wh eth er organ ic p ath ology is
resp on sible or sym p tom atology in p atien ts wh o exh ibit certain p sych iatric disorders or
are malingering (see Ch ap ter 14).
2. Sed atives can relax patients with con dition s su ch as dissociative disorders, conversion dis-
order (see Ch ap ter 14), an d oth er d isord ers in volvin g h igh levels o an xiety an d mute psy-
chotic states (see Ch ap ter 11). Th is will allow p atien ts to exp ress th em selves coh eren tly
d u rin g th e in terview.

B. Sodium lactate administration. In traven ou s (IV) adm in istration o sodiu m lactate can provoke
panic attacks (see Chapter 13) in susceptible patients an d can th u s h elp to iden ti y in d ivid u als
with p an ic d isorder. In h alation o carb on dioxid e can p rod u ce th e sam e e ect.

C. Galvanic skin response (a com p on en t o th e “lie detector” test)


1. Th e electric resistan ce o skin (galvan ic skin resp on se) varies with th e p atien t’s p sych o-
logical state.
2. High er sweat glan d activity, seen with sympathetic nervous system arousal (e.g., wh en
lyin g), resu lts in decreased skin resistan ce an d a p ositive test. However, in n ocen t bu t an x-
RISE USMLE NEPAL

iou s p eop le m ay also h ave p ositive tests ( alse p ositives) an d gu ilty p eop le wh o are n ot
both ered by tellin g lies m ay h ave n egative tests ( alse n egatives).

t a b l e 5.4 Glasgow Coma Scale (GCS)a

Number of Points Best Eye-Opening Response (E) Best Verbal Response (V) Best Motor Response (M)

1 No eye opening No verbal response No motor response


2 Opens eyes in response to Makes incomprehensible Shows extension to painful
painful stimulus sounds stimulus
3 Opens eyes in response to a Speaks using inappropriate Shows flexion to painful
verbal command words stimulus
4 Opens eyes spontaneously Makes confused verbal response Withdraws from painful stimulus
5 — Is oriented and can converse Localizes a source of pain
6 — — Obeys commands
a
Maximum total score on the GCS = 15; lowest possible score = 3; a GSC score of 13–15 indicates mild, 9–12 indicates moderate, and <9 indicates
severe neurologic impairment. The reported score is commonly broken down into components (e.g., E2 V1 M3 = GCS 6).
Review Test

Directions: Each o th e n u m b ered item s or in com p lete statem en ts in th is section is ollowed by


an swers or by com p letion s o th e statem en t. Select th e one lettered an swer or com p letion th at
is best in each case.
HELP OTHERS SO THAT GOD WILL HELP YOU.

1. A 57-year-old m ale patien t wh o h as had 5. To determ in e wh eth er a 3-m on th -old


a stroke can n ot copy a design drawn by the in an t is able to h ear sou n ds, th e m ost
exam in er. Th e test that the exam in er is m ost ap p rop riate diagn ostic tech n iqu e is
likely to be usin g to evaluate this patien t is the (A) PET
(A) Ben der Visu al Motor Gestalt Test (B) CT
(B) Luria-Nebraska neuropsychological battery (C) am obarb ital sodiu m in terview
(C) Halstead -Reitan battery (D) EEG
(D) dexam eth ason e su p p ression test (DST) (E) evoked EEG
(E) electroen cep h alogram (EEG) (F) Glasgow Com a Scale
(G) Folstein Min i–Men tal State
2. A 36-year-old em ale p atien t p resen ts Exam in ation
to th e p h ysician com p lain in g o extrem e
atigu e an d dep ression . Ph ysical 6. A 27-year-old em ale p atien t sh ows
exam in ation reveals a d arken in g o h er skin , a su dd en loss o m otor u n ction b elow
p articu larly in th e creases o h er h an ds as th e waist th at can n ot be m edically exp lain ed.
well as d arken in g o th e b u ccal m u cosa. Th e To determ in e wh eth er p sych ological
m ost likely cau se o th is p ictu re is actors are resp on sible or th is sym p tom ,
(A) h yp ocortisolism th e m ost ap p rop riate diagn ostic
(B) h yp ercortisolism tech n iqu e is
(C) p h eoch rom ocytom a (A) PET
(D) h yp oth yroidism (B) CT
(E) h yp erth yroidism (C) am obarb ital sodiu m in terview
(D) EEG
3. A physician adm inisters sodium lactate (E) evoked EEG
intravenously to a 28-year-old wom an. Using (F) Glasgow Com a Scale
this technique, the physician is trying to provoke, (G) Folstein Min i–Men tal State
RISE USMLE NEPAL

and thus con rm , the patient’s diagnosis o Exam in ation


(A) con version d isorder
(B) deliriu m 7. To iden ti y an atom ical ch an ges in th e
(C) m alin gerin g brain o an 80-year-old em ale p atien t with
(D) p an ic disorder Alzh eim er’s d isease, th e m ost ap p rop riate
(E) m ajor dep ressive d isord er diagn ostic tech n iqu e is
(A) PET
4. To determ in e wh ich brain area is
(B) CT
p h ysiologically active wh en a 44-year-old
(C) am obarb ital sodiu m in terview
m ale p atien t is tran slatin g a p aragrap h rom
(D) electroen cep h alogram (EEG)
Sp an ish to En glish , th e m ost ap p rop riate
(E) evoked EEG
diagn ostic tech n iqu e is
(F) Glasgow Com a Scale
(A) p ositron em ission tom ograp h y (PET) (G) Folstein Min i–Men tal State
(B) com p u ted tom ograp h y (CT) Exam in ation
(C) am obarb ital sodiu m (Am ytal) in terview
(D) electroen cep h alogram (EEG)
(E) evoked EEG
(F) Glasgow Com a Scale
(G) Folstein Min i–Men tal State Exam in ation
53
54 BRS Behavioral Science

8. To di eren tiate deliriu m rom dem en tia 11. A 55-year-old m ale p atien t with n o
in a 75-year-old m ale p atien t, th e m ost h istory o p sych iatric illn ess is adm itted
ap p rop riate diagn ostic tech n iqu e is to th e h osp ital com p lain in g o in ten se
(A) PET abdom in al p ain . He states th at over th e p ast
(B) CT ew days h is wi e h as been givin g h im ood
(C) am obarb ital sodiu m in terview th at is p oison ed so th at sh e can kill h im an d
(D) EEG b e with an oth er m an . Th e wi e states th at
(E) evoked EEG sh e loves h er h u sban d an d wou ld n ever
(F) Glasgow Com a Scale h arm h im or leave h im . Wh en th e p atien t’s
(G) Folstein Min i–Men tal State Exam in ation u rin e is collected , it ap p ears p u rp lish red in
color. Urin e testin g is m ost likely to reveal an
HELP OTHERS SO THAT GOD WILL HELP YOU.

9. A 40-year-old wom an rep orts th at elevated level o


over th e p ast 6 m on th s sh e h as h ad little (A) glu cose
ap petite, sleep s p oorly, an d h as lost in terest (B) 5-h ydroxyin doleacetic acid (5-HIAA)
in her n orm al activities. Ph ysical exam is (C) p orp h obilin ogen
un rem arkab le. Wh ich o th e ollowin g is th e (D) cortisol
m ost likely laboratory n din g in th is wom an ? (E) van illylm an delic acid (VMA)
(A) Positive d exam eth ason e su p p ression test
(DST) 12. Fou r weeks a ter h e begin s to take a n ew
(B) Norm al growth h orm on e regu lation m edication , a 28-year-old m ale p sych iatric
(C) In creased 5-h yd roxyin doleacetic acid p atien t develop s ever an d sore th roat. He
(5-HIAA) levels rep orts eelin g tired, an d blood stu dies reveal
(D) Norm al m elaton in levels a wh ite blood cell (WBC) cou n t o less th an
(E) Hyp erth yroidism 2,000. Th is p atien t is m ost likely to b e takin g
wh ich o th e ollowin g agen ts?
10. A 50-year-old wom an with ou t a p reviou s (A) Am obarbital sodiu m
p sych iatric h istory rep orts th at over th e p ast (B) Clozap in e
ew m on th s sh e h as b egu n to exp erien ce (C) Lith iu m
in ten se an xiety an d h as lost 15 p ou n ds. Th e (D) Dexam eth ason e
p atien t also com p lain s o “f u sh in g an d (E) Sodiu m lactate
p alp itation s.” Wh ich o th e ollowin g is th e
m ost likely lab oratory n din g in th is wom an ?
(A) Positive DST
(B) Norm al growth h orm on e regu lation
(C) In creased 5-HIAA levels
(D) Norm al m elaton in levels
(E) Hyp erth yroidism
RISE USMLE NEPAL
An swers an d Exp lan ation s

Typical Board Question


The answer is C. Scores b elow 18 on th e Folstein Min i–Men tal State Exam in ation in dicate sig-
n i ican t cogn itive im p airm en t. Th is test does n ot evalu ate calcu latin g ability or in telligen ce.
Alth ou gh th e p atien t is im p aired, it is n ot clear wh at cau sed th e p roblem or wh eth er sh e n eed s
HELP OTHERS SO THAT GOD WILL HELP YOU.

to be p laced in an assisted livin g acility.

1. The answer is A. Th e Ben d er Visu al Motor Gestalt Test is u sed to evalu ate visu al an d
m otor ab ility by rep rod u ction o d esign s. Th e Lu ria-Neb raska n eu rop sych ological
b attery is u sed to d eterm in e cereb ral d om in an ce an d to id en ti y sp eci ic typ es o b rain
d ys u n ction , wh ile th e Halstead -Reitan b attery is u sed to d etect an d localize b rain lesion s
an d d eterm in e th eir e ects. Th e d exam eth ason e su p p ression test is u sed to p red ict
wh ich d ep ressed p atien ts will resp on d well to treatm en t with an tid ep ressan t agen ts or
electrocon vu lsive th erapy. Th e electroen cep h alogram (EEG), wh ich m easu res electrical
activity in th e cortex, is u se u l in d iagn osin g ep ilep sy an d in d i eren tiatin g d eliriu m rom
d em en tia.
2. The answer is A. Th is em ale p atien t is sh owin g eviden ce o h yp ocortisolism or Addison’s
d isease. Th is con dition is ch aracterized by darken in g o th e skin , p articu larly in p laces n ot
exp osed to th e su n su ch as skin creases an d in side th e m ou th . Th is darken in g is n ot seen
in h yp ercortisolism , p h eoch rom ocytom a, or h yp er- or h yp oth yroidism . Hyp ercortisolism ,
wh ich also m ay lead to dep ression an d an xiety, is ch aracterized by weigh t gain , “m oon -”
sh ap ed ace, an d skin striae. Dep ression , dry h air, an d weigh t gain ch aracterize
h yp oth yroid ism , wh ile an xiety, ever, weigh t loss, an d elevated h eart rate ch aracterize
h yp erth yroid ism . Patien ts with p h eoch rom ocytom a sh ow in ten se an xiety an d elevated VMA
in body lu ids (see Ch ap ter 4).
3. The answer is D. In traven ou s adm in istration o sodiu m lactate can h elp iden ti y in dividu als
with p an ic disorder sin ce it can p rovoke a p an ic attack in su ch p atien ts.
4. The answer is A. Positron em ission tom ograp h y (PET) localizes p h ysiologically active
b rain areas by m easu rin g glu cose m etab olism . Th u s, th is test can be u sed to determ in e
RISE USMLE NEPAL

wh ich brain area is bein g u sed du rin g a sp eci ic task (e.g., tran slatin g a p assage written in
Sp an ish ).
5. The answer is E. Th e au ditory evoked EEG can be u sed to assess wh eth er th is ch ild can h ear.
Evoked EEGs m easu re electrical activity in th e cortex in resp on se to sen sory stim u lation .
6. The answer is C. Th e am ob arb ital sod iu m (Am ytal) in terview is u sed to determ in e
wh eth er p sych ological actors are resp on sible or sym p tom s in th is p atien t wh o sh ows a
n on m ed ically exp lain ed loss o sen sory u n ction (con version disorder—see Ch ap ter 14).
7. The answer is B. Com p u ted tom ograp h y (CT) id en ti ies an atom ical b rain ch an ges, su ch
as en larged ven tricles. Th u s, alth ou gh n ot d iagn ostic, th is test can be u sed to id en ti y
an atom ical ch an ges in th e b rain , su ch as en larged ven tricles in a p atien t with su sp ected
Alzh eim er’s d isease.
8. The answer is D. Electroen cep h alogram (EEG) m easu res electrical activity in th e cortex an d
can b e u se u l in di eren tiatin g deliriu m (ab n orm al EEG) rom d em en tia (u su ally n orm al
EEG).

55
56 BRS Behavioral Science

9. The answer is A. Poor ap p etite, p oor sleep, an d lack o in terest in u su al activities


ch aracterize p atien ts wh o h ave m ajor d ep ression (see Ch ap ter 12). In th is dep ressed
wom an , th e dexam eth ason e su p p ression test is likely to be p ositive. A p ositive resu lt is
seen wh en th e syn th etic glu cocorticoid d exam eth ason e ails to su p p ress th e secretion
o cortisol as it wou ld in a p atien t with a n orm al m ood . Also, in d ep ression , th ere m ay
be ab n orm al growth h orm on e regu lation an d m elaton in levels, an d decreased 5-HIAA.
Hyp oth yroidism m ay be associated with d ep ression ; h yp erth yroid ism is m ore com m on ly
associated with th e sym p tom s o an xiety.
10. The answer is E. Th is wom an’s sym p tom s (e.g., an xiety, ever, weigh t loss, an d lu sh in g)
in d icate th at sh e h as h yp erth yroidism (also see th e an swer to Qu estion 9). Peop le
com m on ly d escribe th eir p ercep tion o a rap id h eartbeat as “p alp itation s.”
HELP OTHERS SO THAT GOD WILL HELP YOU.

11. The answer is C. Th is p atien t with abd om in al p ain , th e alse belie th at h is wi e is tryin g to
p oison h im (a delu sion , see Table 11.1), an d u rin e with a p u rp le-red color is m ost likely to
h ave acu te in term itten t p orp h yria, a d isord er th at is associated with p sych iatric sym p tom s
su ch as d elu sion s. Acu te in term itten t p orp h yria is a m etab olic d isorder in wh ich toxic
p orp h yrin s accu m u late in tissu e leadin g to h igh levels o p orp h obilin ogen in u rin e wh ich
colors it p u rp le-red. Pu rp le-red u rin e is n ot seen in th e seroton in syn d rom e (h igh 5-HIAA),
Cu sh in g’s disease (h igh cortisol), p h eoch rom ocytom a (h igh VMA), or diabetes (h igh
glu cose).
12. The answer is B. Agran u locytosis is seen p articu larly in p atien ts takin g clozap in e, an
an tip sych otic, or carb am azep in e, an an ticon vu lsan t th at is u sed to treat bip olar disorder
(see Ch ap ter 12). Lith iu m , am ob arb ital sod iu m , dexam eth ason e, an d sod iu m lactate are
n ot sp eci ically associated with agran u locytosis.
RISE USMLE NEPAL
Psych oan alytic
c ha pte r
6 Th eory an d De en se
Mech an ism s
HELP OTHERS SO THAT GOD WILL HELP YOU.

Typical Board Question


A 28-year-old em ale p atien t is hospitalized a ter m akin g a suicide attem pt because her doctor
did n ot resp on d to h er o er to rien d h im on Facebook. Wh en sh e is in terviewed, the patien t
states that all em ale doctors are good but all m ale doctors are in com peten t. The best exp lan a-
tion or th is statem en t by th e p atien t is
(A) p reju dice
(B) lack o basic tru st
(C) ch au vin ism
(D) sp littin g
(E) bias
(See “An sw ers an d Explan ation s” at th e en d of th e ch apter.)

I. OVERVIEW
Psych oan alytic th eory is b ased on Freu d’s con cep t th at beh avior is determ in ed by orces derived
rom u n con sciou s m en tal p rocesses. Psych oan alysis an d related th erap ies are p sych oth erap eu -
tic treatm en ts based on th is con cep t (see Ch ap ter 17).
RISE USMLE NEPAL

II. FREUD’S THEORIES OF THE MIND


To exp lain h is id eas, Freu d d evelop ed, early in h is career, th e top ograp h ic th eory o th e m in d
an d , later in h is career, th e stru ctu ral th eory.

A. Topographic theory of the mind. In th e top ograp h ic th eory, th e m in d con tain s th ree levels:
Th e u n con sciou s, p recon sciou s, an d con sciou s.
1. Th e unconscious mind con tain s rep ressed th ou gh ts an d eelin gs th at are n ot available to
th e con sciou s m in d, an d u ses p rim ary p rocess th in kin g.
a. Primary process is a typ e o th in kin g associated with p rim itive drives, wish u l illm en t,
an d p leasu re seekin g an d h as n o logic or con cep t o tim e. Prim ary p rocess th in kin g is
seen in you n g ch ildren an d p sych otic adu lts.
b. Dreams rep resen t grati ication o u n con sciou s in stin ctive im p u lses an d wish
u l illm en t.

57
58 BRS Behavioral Science

t a b l e 6.1 Freud’s Structural Theory of the Mind


Structural Topographic Level Age at Which It
Component of Operation Develops Characteristics

Id Unconscious Present at birth Contains instinctive sexual and aggressive drives


Controlled by primary process thinking
Not influenced by external reality
Ego Unconscious, Begins to develop Controls the expression of the id to adapt to the
preconscious, immediately after requirements of the external world primarily by the
and conscious birth use of defense mechanisms
Enables one to sustain satisfying interpersonal
relationships
HELP OTHERS SO THAT GOD WILL HELP YOU.

Through reality testing (i.e., constantly evaluating what


is valid and then adapting to that reality), enables one
to maintain a sense of reality about the body and the
external world
Superego Unconscious, Begins to develop at Associated with moral values and conscience
preconscious, about 6 y of age Controls the expression of the id
and conscious

2. Th e preconscious mind con tain s m em ories th at, wh ile n ot im m ediately available, can be
accessed easily.
3. Th e conscious mind con tain s th ou gh ts th at a p erson is cu rren tly aware o . It op erates in
close con ju n ction with th e p recon sciou s m in d b u t does n ot h ave access to th e u n con -
sciou s m in d . Th e con sciou s m in d u ses secon d ary p rocess th in kin g (logical, m atu re, tim e
orien ted) an d can delay grati ication .

B. Structural theory of the mind. In th e stru ctu ral th eory, th e m in d con tain s th ree p arts: th e id ,
th e ego, an d th e su p erego (Tab le 6.1).

III. DEFENSE MECHANISMS


A. Definition. De en se m ech an ism s are u n con sciou s m en tal tech n iqu es u sed by th e ego to keep
con licts ou t o th e con sciou s m in d, th u s decreasin g an xiety an d m ain tain in g a p erson’s
sen se o sa ety, equ ilibriu m , an d sel -esteem . Th ey can be u se u l in h elp in g p eop le deal with
d i icu lt li e situ ation s su ch as m ed ical illn ess, b u t, wh en u sed in excess, can becom e a bar-
RISE USMLE NEPAL

rier to seekin g care or adh erin g to treatm en t recom m en d ation s.

B. Specific defense mechanisms (Table 6.2)


1. Som e de en se m ech an ism s are immature (i.e., th ey are m an i estation s o ch ild-like or dis-
tu rb ed beh avior).
2. Mature defense mechanisms (e.g., altru ism , h u m or, su blim ation , an d su p p ression ), wh en
u sed in m od eration , d irectly h elp th e p atien t or oth ers.
3. Repression, p u sh in g u n accep table em otion s in to th e u n con sciou s, is th e basic defense
mechanism on wh ich all oth ers are based.

IV. TRANSFERENCE REACTIONS


A. Definition. Tran s eren ce an d cou n tertran s eren ce are unconscious mental attitudes b ased
on im p ortan t p ast p erson al relation sh ip s (e.g., with p aren ts). Th ese p h en om en a in crease
em otion ality an d m ay th u s alter ju dgm en t an d b eh avior in p atien ts’ relation sh ip s with th eir
doctors (tran s eren ce) an d doctors’ relation sh ip s with th eir p atien ts (cou n tertran s eren ce).
Chapter 6 Psychoanalytic Theory and Defense Mechanisms 59

t a b l e 6.2 Commonly Used Defense Mechanisms (Listed Alphabetically)


Defense Mechanism Explanation Example

Acting out Avoiding personally unacceptable A depressed 14-year-old girl with no history
emotions by behaving in an of conduct disorder has sexual encounters
attention-getting, often socially with multiple partners after her parents’
inappropriate manner divorce
Altruisma Assisting others to avoid negative A man with a poor self-image, who is a social
personal feelings worker during the week, donates every
other weekend to charity work
Denial Not accepting aspects of reality that A man who has a problem with alcohol insists
the person finds unbearable that he is only a social drinker
HELP OTHERS SO THAT GOD WILL HELP YOU.

Displacement Moving emotions from a personally A surgeon with unacknowledged anger


intolerable situation to one that is toward his sister is abrasive to the female
personally tolerable residents on his service
Dissociation Mentally separating part of one’s Although he was not injured, a teenager has
consciousness from real-life no memory of a car accident in which he
events or mentally distancing was driving and his girlfriend was killed
oneself from others
Humora Expressing personally uncomfortable A man who is concerned about his erectile
feelings without causing problems makes jokes about Viagra
emotional discomfort (sildenafil citrate)
Idealization Seeing others as more competent or A patient tells the doctor that he is not worried
powerful than they are because he is sure that the doctor will
always know what to do
Identification Unconsciously patterning one’s A man who was terrorized by his gym teacher
(introjection) behavior after that of someone as a child becomes a punitive, critical gym
more powerful (can be either teacher (identification with the aggressor)
positive or negative)
Intellectualization Using the mind’s higher functions to A sailor whose boat is about to sink calmly
avoid experiencing emotion explains the technical aspects of the hull
damage in great detail to the other crew
members
Isolation of affect Failing to experience the feelings Without showing any emotion, a woman tells
associated with a stressful her family the results of tests that indicate
life event, although logically her lung cancer has metastasized
understanding the significance of
the event
Projection Attributing one’s own personally A man with unconscious homosexual impulses
unacceptable feelings to others begins to believe that a male colleague is
Associated with paranoid symptoms attracted to him
RISE USMLE NEPAL

and prejudice
Rationalization Distorting one’s perception of an A man who loses an arm in an accident says
event so that its negative outcome the loss of his arm was good because it
seems reasonable kept him from getting in trouble with the
law
Reaction formation Adopting opposite attitudes to A woman who unconsciously is resentful
avoid personally unacceptable of the responsibilities of child rearing
emotions, i.e., unconscious overspends on expensive gifts and clothing
hypocrisy for her children
Regression Reverting to behavior patterns like A woman insists that her husband stay
those seen in someone of a overnight in the hospital with her before
younger age surgery
Splitting Categorizing people or situations into A patient tells the doctor that while all of
categories of either “fabulous” or the doctors in the group practice are
“dreadful” because of intolerance wonderful, all of the nurses and office help
of ambiguity are unfriendly and curt
Seen in patients with borderline
personality disorder
(continued)
60 BRS Behavioral Science

t a b l e 6.2 Commonly Used Defense Mechanisms (Listed Alphabetically) (continued)

Sublimationa Expressing a personally A man who got into fights as a teenager


unacceptable feeling (e.g., rage) becomes a professional prize fighter
in a socially useful way
Suppressiona Deliberately pushing personally A medical student taking a review course
unacceptable emotions out of for the United States Medical Licensing
conscious awareness (the only Examination mentally changes the subject
defense mechanism that includes when her mind wanders to the exam during
some aspect of consciousness) a lecture
Undoing Believing that one can magically A woman who stole money from a friend,
reverse past events caused by confesses to the theft, returns the money,
HELP OTHERS SO THAT GOD WILL HELP YOU.

“incorrect” behavior by now and then feels compelled to offer to drive


adopting “correct” behavior, the friend to and from work for a year
e.g., atonement, confession, or
penance. Seen in obsessive–
compulsive disorder
“Mature” defense mechanisms.
a

Adapted from Fadem B. Behavioral Science in Medicine. 2nd ed. Philadelphia, PA: Lippincott Williams & Wilkins; 2012:83.

B. Transference
1. In positive transference , th e p atien t h as con id en ce in th e d octor. I in ten se, th e p atien t
m ay overid ealize th e d octor or develop sexu al eelin gs toward th e doctor.
2. In negative transference , th e p atien t m ay b ecom e resen t u l or an gry toward th e d octor i
th e p atien t’s desires an d exp ectation s are n ot realized. Th is m ay lead to p oor adh eren ce
to m edical advice.

C. In countertransference, eelin gs abou t a p atien t wh o rem in d s th e d octor o a close rien d or


relative can in ter ere with th e doctor’s m edical ju dgm en t.
RISE USMLE NEPAL
Review Test

Directions: Each o th e n u m b ered item s or in com p lete statem en ts in th is section is ollowed by


an swers or by com p letion s o th e statem en t. Select th e one lettered an swer or com p letion th at
is best in each case.

1. Wh en a 27-year-old p atien t wh o h ad a 5. A p rim ary care p h ysician n otices th at


HELP OTHERS SO THAT GOD WILL HELP YOU.

con ten tious relation sh ip with his ather join s m an y o h er p atien ts u se statem en ts like “I
a n ew health in su ran ce plan , he m u st chan ge can’t stop sm okin g becau se I’ll gain weigh t”
rom h is p rim ary care p h ysician , a you n g or “wh en I’m sick, I on ly wan t to eat ju n k
m an , to a n ew p hysician , a m iddle-aged m an . ood.” Statem en ts like th ese
On h is rst visit to th e n ew doctor, th e p atien t (A) p rodu ce con lict in th e con sciou s m in d
seem s an n oyed with everyth in g th e doctor (B) are con sciou s m en tal tech n iqu es
says an d states, “You are an old m an with (C) in crease an xiety
old- ash ion ed ideas; you ju st wan t to con trol (D) are exam p les o th e u se o de en se
m y li e.” Th is p atien t’s beh avior is m ost m ech an ism s
closely related to wh ich o th e ollowin g? (E) decrease a p atien t’s sen se o sel -esteem
(A) Positive tran s eren ce
(B) Negative tran s eren ce 6. O th e ollowin g d e en se m ech an ism s,
(C) Cou n tertran s eren ce which is con sidered th e m ost m atu re?
(D) Dislike o th e d octor (A) Den ial
(E) Fear o th e doctor (B) Su blim ation
(C) Dissociation
2. A p h ysician becom es very an gry with a (D) Regression
p atien t wh en th e p atien t does n ot take h is (E) In tellectu alization
m ed ication . Th e p atien t rem in d s th e d octor
o h er rebelliou s son . Th is p h ysician’s in ten se 7. Wh en h avin g a m an ic ep isode, a 53-year-
reaction to th e p atien t’s b eh avior is m ost old p atien t with b ip olar d isord er sh ows
likely to be a resu lt o p rim ary p rocess th in kin g. Th is typ e o
(A) p ositive tran s eren ce th in kin g
(B) n egative tran s eren ce (A) is logical
(C) cou n tertran s eren ce (B) is closely attu n ed to tim e
RISE USMLE NEPAL

(D) dislike o th e p atien t (C) is associated with reality


(E) ear o th e p atien t (D) is accessible to th e con sciou s m in d
(E) is associated with p leasu re seekin g
3. Wh ich o th e ollowin g stru ctu res o th e
m in d work at least p artly on an u n con sciou s 8. Abou t 1 week a ter h er n al exam in ation
level? or a bioch em istry cou rse, a m edical
(A) Th e id on ly stu d en t’s kn owledge o th e d etails o th e
(B) Th e id an d th e ego on ly Krebs cycle is m ost likely to reside in h er
(C) Th e id, ego, an d su p erego (A) u n con sciou s m in d
(D) Th e ego an d su p erego on ly (B) p recon sciou s m in d
(E) Not th e id , ego, or su p erego (C) con sciou s m in d
(D) su p erego
4. Wh ich o th e ollowin g stru ctu res o th e (E) ego
m in d is (are) at least p artly develop ed in a
typ ical 4-year-old ch ild ?
(A) Th e id on ly
(B) Th e id an d th e ego on ly
(C) Th e id, ego, an d su p erego
(D) Th e ego an d su p erego on ly
(E) Not th e id , ego, or su p erego
61
62 BRS Behavioral Science

9. A 15-year-old steals rom am ily m em b ers 13. A 32-year-old m an wh o is u n con sciou sly
an d rien d s. Wh en n o on e is watch in g, h e attracted to h is wi e’s sister becom es
also tortu res th e am ily cat. Wh ich asp ect o extrem ely jealou s wh en ever h is wi e sp eaks
th e m in d is de cien t in th is teen ager? to an oth er m an .
(A) Th e u n con sciou s m in d
(B) Th e p recon sciou s m in d 14. A 45-year-old m an wh o is u n con sciou sly
(C) Th e con sciou s m in d a raid o f yin g rep eatedly states h is love o
(D) Th e su p erego airplan es.
(E) Th e ego
15. A 52-year-old m an receives a letter
rom his physician in orm ing him that his
HELP OTHERS SO THAT GOD WILL HELP YOU.

Questions 10–20 level o prostate-speci c antigen (PSA) was


abnorm ally high during his last visit. When
For th e in d ivid u al in each o th e n u m b ered the m an appears at his physician’s o ce or a
qu estion s, ch oose th e d e en se m ech an ism ollow-up visit, he com plains about a headache
th at h e or sh e is m ost likely to b e u sin g. but does not m ention or seem to rem em ber
An swers can b e u sed m ore th an on ce. receiving the letter about his PSA test.
(A) Regression
(B) Un doin g 16. A 34-year-old wom an relates th at sh e
(C) Den ial wakes u p u lly d ressed at least twice a week
(D) Ration alization b u t th en is tired all day. Sh e also n otes th at
(E) Projection sh e requ en tly receives p h on e calls rom
(F) Dissociation m en wh o say th ey m et h er in a bar bu t wh om
(G) Reaction orm ation sh e does n ot rem em ber m eetin g.
(H) In tellectu alization
(I) Su blim ation 17. A 35-year-old lawyer sch edu led or
(J ) Disp lacem en t su rgery th e n ext day in sists th at h er m oth er
(K) Su p p ression stay overn igh t in th e h osp ital with h er.
(L) Sp littin g

10. A 28-year-old m ed ical resid en t is 18. A wom an , wh ose p aren ts an d teach ers
assign ed to tell a p atien t th at h er illn ess is com p lain ed ab ou t h ow m essy sh e was as a
term in al. Prior to seein g th e p atien t, th e ch ild, grows u p to b ecom e a am ou s abstract
residen t con du cts exten sive library research p ain ter. Her tech n iqu e in volves th rowin g
on th e details an d statistics o len gth o p ain t an d sm all ob jects at large can vases an d
su rvival o p eop le with th is illn ess. Wh en h e th en u sin g h er n gers to m ix th e colors an d
sp eaks to th e p atien t, h e cites th e jou rn al textu res.
RISE USMLE NEPAL

articles th at h e h as read, in clu din g a detailed


exp lan ation o th e th eories o th e etiology 19. A m an wh o h as ju st received word th at
o her con dition . Later th at day, th e residen t h is ch ild h as been in an acciden t an d h as
tells th e atten d in g p h ysician th at th e p atien t b een taken to th e h osp ital calm ly arran ges
did n ot seem to u n d erstan d wh at h e told h er. or h is work to b e don e by a colleagu e b e ore
h e ru sh es to th e h osp ital.
11. A 40-year-old m an wh o is an gry at h is ill
wi e, bu t does n ot con sciou sly ackn owled ge 20. A 30-year-old wom an wh o was abu sed
th at an ger, sh ou ts at h is ch ild ren as soon as by her ath er th rou gh ou t h er ch ildh ood
h e retu rn s h om e rom work. m an ages h er h ostility toward h im by bakin g
cookies or h im .
12. A 26-year-old m ed ical stu den t wh o h as
u n con sciou s an gry, violen t eelin gs ch ooses 21. A p atien t wh o h as been d iagn osed with
to do a su rgery residen cy. obsessive–com p u lsive disorder tells th e
d octor th at h e h as to cou n t all th e ligh ts in
th e ceilin g b e ore h e can sit down to stu dy.
I h e d oes n ot cou n t th e ligh ts, h e becom es
an xiou s an d is u n able to stu dy.
An swers an d Exp lan ation s

Typical Board Question


The answer is D. Dividin g p eop le or situ ation s in to categories o good an d bad ch aracterizes th e
de en se m ech an ism o sp littin g. Sp littin g is com m on ly seen in p eop le with b ord erlin e p erson al-
ity d isord er. Th is d isord er also is ch aracterized by im p u lsive beh avior su ch as m akin g a su icide
HELP OTHERS SO THAT GOD WILL HELP YOU.

attem p t or a trivial reason (see Ch ap ter 14) like th at d em on strated in th is exam p le. Preju dice,
ch au vin ism , an d bias are m ore likely to b e related to cu ltu ral actors th an to th e u n con sciou s
u se o a de en se m ech an ism .

1. The answer is B. Th e p atien t wh o becom es very an gry at h is n ew doctor is sh owin g a


n egative tran s eren ce reaction . Th is em otion al dem on stration is likely to be a resu lt o
re-exp erien cin g n egative eelin gs ab ou t h is relation sh ip with h is m iddle-aged ath er in h is
relation sh ip with th e m id dle-aged m ale doctor. In n egative tran s eren ce, p atien ts becom e
resen t u l or an gry toward th e d octor i th eir desires an d exp ectation s are n ot realized. Th is
m ay lead to n on com p lian ce with m ed ical advice. In p ositive tran s eren ce, p atien ts h ave
a h igh level o con iden ce in th e d octor. Patien ts m ay also overidealize or develop sexu al
eelin gs toward th e doctor. Th is p atien t’s reaction to th e n ew doctor is less likely to be
related to dislike or ear o th e doctor.
2. The answer is C. Th e d octor wh o b ecom es very an gry at h er p atien t or n ot takin g h is
m edication is sh owin g a cou n tertran s eren ce reaction . Th is excessive sh ow o em otion is
a resu lt o re-exp erien cin g eelin gs ab ou t h er son’s beh avior in h er relation sh ip with th e
n on com p lian t p atien t. It is im p ortan t or th e doctor to iden ti y th is reaction becau se it can
in ter ere with h er m edical ju dgm en t (see also an swer to Qu estion 1). Th is doctor’s reaction
to th e p atien t is less likely to be related to dislike or ear o th e p atien t.
3. The answer is C. In Freu d’s stru ctu ral th eory, th e m in d is divided in to th e id, ego, an d
su p erego. Th e id op erates com p letely on an u n con sciou s level, wh ile th e ego an d su p erego
op erate p artly on an u n con sciou s an d p artly on p recon sciou s an d con sciou s levels.
4. The answer is B. Th e id is p resen t at b irth , th e ego begin s to develop im m ed iately a ter
b irth , an d th e su p erego b egin s to d evelop at abou t age 6 years.
RISE USMLE NEPAL

5. The answer is D. Statem en ts su ch as “I can’t stop sm okin g b ecau se I’ll gain weigh t” or
“wh en I’m sick, I on ly wan t to eat ju n k ood” are exam p les o th e de en se m ech an ism s
o ration alization an d regression , resp ectively. In ration alization , a p erson distorts h er
p ercep tion o an even t so th at its n egative ou tcom e seem s reason able, or exam p le, becau se
sh e eels u n able to stop sm okin g, th is p atien t claim s (an d so sh e reason ably eels) th at
gain in g weigh t is worse th an sm okin g, a li e-th reaten in g h abit. In regression , ill p atien ts
revert to beh avior p attern s like th ose seen in som eon e o a you n ger age (e.g., eatin g ju n k
ood, cryin g). De en se m ech an ism s su ch as th ese are u n con sciou s m en tal tech n iqu es th at
d ecrease an xiety an d h elp p eop le to m ain tain a sen se o equ ilibriu m an d sel -esteem .
6. The answer is B. Su blim ation , exp ressin g an u n accep tab le em otion in a socially accep table
way, is classi ied as a m atu re de en se m ech an ism . Den ial, dissociation , regression , an d
in tellectu alization are all classi ied as less m atu re de en se m ech an ism s.
7. The answer is E. Prim ary p rocess th in kin g is associated with p leasu re seekin g, disregards
logic an d reality, h as n o con cep t o tim e, an d is n ot accessible to th e con sciou s m in d.
Secon dary p rocess th in kin g is logical an d is associated with reality.

63
64 BRS Behavioral Science

8. The answer is B. Mem ory o th e details o th e Krebs cycle, wh ile n o lon ger in th e
ore ron t o th e m ed ical stu d en t’s m in d , can be recalled relatively easily 1 week a ter th e
exam in ation . Th is m em ory th ere ore resid es in th e p recon sciou s m in d. Th e u n con sciou s
m in d con tain s rep ressed th ou gh ts an d eelin gs, wh ich are n ot availab le to th e con sciou s
m in d . Th e con sciou s m in d con tain s th ou gh ts th at a p erson is cu rren tly aware o . Th e
id con tain s in stin ctive sexu al an d aggressive d rives an d is n ot in lu en ced by extern al
reality. Th e ego also con trols th e exp ression o th e id, su stain s satis yin g in terp erson al
relation sh ip s, an d, th rou gh reality testin g, m ain tain s a sen se o reality abou t th e body an d
th e extern al world (see also an swer to Qu estion 9).
9. The answer is D. Th e su p erego is associated with m oral valu es an d con scien ce, an d
con trols im p u lses o th e id. Th is teen ager wh o steals rom am ily m em b ers an d rien d s an d
HELP OTHERS SO THAT GOD WILL HELP YOU.

tortu res th e am ily cat is sh owin g de icien cies in h is su p erego. Ch ildren an d adolescen ts
u n d er age 18 years, wh o h ave p oor su p erego d evelop m en t, m ay be diagn osed with
con d u ct disord er (see Ch ap ter 15).
10. The answer is H. Th e residen t’s b eh avior in dealin g with th is p atien t re lects h is u se o th e
de en se m ech an ism o in tellectu alization . Th e resid en t h as u sed h is tech n ical kn owledge
to avoid exp erien cin g th e em otion associated with tellin g th e p atien t th at sh e is dyin g.
11. The answer is J . In disp lacem en t, th e m an’s p erson ally u n accep table an gry eelin gs toward
h is wi e are taken ou t on h is ch ild ren .
12. The answer is I. In su blim ation , th e su rgeon rerou tes h is u n con sciou s, u n accep table wish
or com m ittin g a violen t act to a socially accep table rou te (cu ttin g p eop le du rin g su rgery).
13. The answer is E. Usin g p rojection , th e h u sban d attribu tes h is own u n con sciou s,
u n accep table sexu al eelin gs toward an oth er wom an to h is wi e.
14. The answer is G. In reaction orm ation , th e m an d en ies h is u n con sciou s ear o lyin g an d
em b races th e op p osite id ea by statin g th at h e loves airp lan es.
15. The answer is C. Usin g den ial, th is p atien t h as seem in gly orgotten an asp ect o extern al
reality, th at is, th e letter abou t h is p roblem atic PSA test.
16. The answer is F. Th is p atien t wh o relates th at sh e wakes u p u lly dressed at least twice
a week an d receives p h on e calls rom m en wh om sh e does n ot rem em ber m eetin g is
exh ib itin g d issociative id en tity d isord er (m u ltip le p erson ality disorder). Dissociation ,
sep aratin g p art o on e’s con sciou sn ess rom real li e even ts, is th e de en se m ech an ism
u sed by in d ivid u als with th is d isord er. It is likely th at th is p atien t m et th e m en wh o h ave
h er p h on e n u m ber b u t d oes n ot rem em ber m eetin g th em becau se at th at tim e sh e was
RISE USMLE NEPAL

sh owin g an oth er p erson ality (see also Ch ap ter 14).


17. The answer is A. Regression , goin g b ack to a less m atu re way o b eh avin g, is th e d e en se
m ech an ism u sed by th is wom an sch ed u led or su rgery th e n ext d ay wh o in sists th at h er
m oth er stay overn igh t in th e h osp ital with h er.
18. The answer is I. Th e u se u l em p loym en t in h er abstract art o th is wom an’s “m essy”
ten d en cies is an exam p le o th e d e en se m ech an ism o su b lim ation .
19. The answer is K. Th is m an is u sin g th e p artly con sciou s de en se m ech an ism o su p p ression
d u rin g th e tim e th at h e is arran gin g or h is work to be don e by som eon e else be ore goin g
to th e h osp ital.
20. The answer is G. Th is wom an wh o bakes cookies or h er ab u sive ath er is m an agin g h er
h ostility toward h im by u sin g th e d e en se m ech an ism o reaction orm ation . In th is de en se
m ech an ism , a p erson adop ts beh avior th at is op p osite to the way sh e really eels, th at is,
th is wom an eels in ten se an ger toward h er ath er bu t sh ows carin g beh avior toward h im .
21. The answer is B. Th is p atien t with obsessive–com p u lsive disorder is u sin g th e de en se
m ech an ism o u n d oin g. Cou n tin g th e ligh ts rem oves or “u n does” th e stu den t’s an xiety,
wh ich is likely to be related to h is sch ool p er orm an ce.
c ha pte r
7 Learn in g Th eory
HELP OTHERS SO THAT GOD WILL HELP YOU.

Typical Board Question


Alth ou gh h e is scold ed by h is ath er or watch in g television wh en h e sh ou ld b e doin g h is
h om ework, a 9-year-old b oy in creases h is television watch in g. Th e ath er th en decides to
ign ore th e boy’s television -watch in g beh avior. With in a week, th e boy h as stop p ed watch -
in g television wh en h e sh ou ld b e doin g h om ework. Th e ath er’s in terven tion , wh ich led to
im p rovem en t in th e b oy’s “d oin g h is h om ework” beh avior, can best be described as
(A) p ositive rein orcem en t
(B) p u n ish m en t
(C) m odelin g
(D) sh ap in g
(E) extin ction
(See “An sw ers an d Explan ation s” at th e en d of th e ch apter.)

I. OVERVIEW
A. Learn in g is th e acqu isition o n ew beh avior p attern s.

B. Meth ods o learn in g in clu d e sim p le orm s, su ch as habituation an d sensitization, an d m ore


RISE USMLE NEPAL

com p lex typ es, in clu d in g classical conditioning an d operant conditioning.

C. Learn in g m eth ods are th e basis o behavioral treatment techniques , su ch as system atic desen -
sitization , aversive con dition in g, lood in g, bio eedback, token econ om y, an d cogn itive th er-
apy (see Ch ap ter 17).

II. HABITUATION AND SENSITIZATION


A. In habituation (also called desen sitization ), rep eated stim u lation resu lts in a decreased
resp on se (e.g., a ch ild wh o receives weekly allergy in jection s cries less an d less with each
in jection ).

B. In sensitization, rep eated stim u lation resu lts in an in creased resp on se (e.g., a ch ild wh o is
a raid o sp iders eels m ore an xiety each tim e h e en cou n ters a sp ider).

65
66 BRS Behavioral Science

III. CLASSICAL CONDITIONING


A. Principles. In classical con dition in g, a natural or reflexive response (behavior) is elicited by a
learned stimulus (a cue rom an in tern al or extern al even t). This type o learn in g is called asso-
ciative learning.
1. Th e hippocampus is p articu larly im p ortan t in associative learn in g.
2. Th e cerebellum p articip ates in classical con d ition in g, sp eci ically in association s in volv-
in g m otor skills.

B. Elements o classical con dition in g


HELP OTHERS SO THAT GOD WILL HELP YOU.

1. An unconditioned stimulus is som eth in g th at au tom atically, with ou t h avin g to be learn ed,
p rodu ces a resp on se (e.g., th e odor o ood).
2. An unconditioned response is a n atu ral, re lexive beh avior th at does n ot h ave to be learn ed
(e.g., salivation in resp on se to th e od or o ood).
3. A conditioned stimulus is som eth in g th at p rodu ces a resp on se ollowin g learn in g (e.g., th e
sou n d o th e lu n ch bell).
4. A conditioned response is a behavior that is learn ed by an association m ade between a con di-
tion ed stim ulus an d an un con dition ed stim ulus (e.g., salivation in response to the lunch bell).

C. Response acquisition, extinction, and stimulus generalization


1. In acquisition, the conditioned response (e.g., salivation in response to the lunch bell) is learned.
2. In extinction, th e con dition ed resp on se decreases i th e con dition ed stim u lu s (e.g., th e
sou n d o th e lu n ch b ell) is n ever again p aired with th e u n con dition ed stim u lu s (e.g., th e
od or o ood ).
3. In stimulus generalization, a n ew stim u lu s (e.g., a ch u rch bell) th at resem bles a con di-
tion ed stim u lu s (e.g., th e lu n ch b ell) cau ses a con dition ed resp on se (e.g., salivation ).

D. Aversive conditioning. An u n wan ted beh avior (e.g., settin g ires) is p aired with a p ain u l
or aversive stim u lu s (e.g., a p ain u l electric sh ock). An association is created between th e
u n wan ted beh avior ( ire-settin g) an d th e aversive stim u lu s (p ain ) an d th e ire-settin g ceases.

E. Learned helplessness
1. An an im al receives a series o p ain u l electric sh ocks rom wh ich it is unable to escape.
2. By classical con d ition in g, th e an im al learn s th at th ere is an association between an aver-
sive stim u lu s (e.g., p ain u l electric sh ock) an d th e in ability to escap e.
3. Su bsequ en tly, th e an im al m akes n o attem p t to escap e wh en sh ocked or wh en aced with
RISE USMLE NEPAL

an y n ew aversive stim u lu s; in stead , th e an im al becom es hopeless and apathetic.


4. Learn ed h elp lessn ess in an im als m ay b e a m odel system or depression (o ten ch aracter-
ized by h op elessn ess an d ap ath y) in h u m an s.
5. An tidep ressan t treatm en t in creases escap e attem p ts in an im al m odels.

F. Imprinting is th e ten d en cy o organ ism s to m ake an association with an d th en ollow th e irst


th in g th ey see a ter b irth or h atch in g (in b ird s).

IV. OPERANT CONDITIONING


A. Principles
1. Beh avior is determ in ed by its con sequ en ces or th e in dividu al. Th e con sequ en ce (rein -
orcem en t or p u n ish m en t) occu rs im m ediately ollowin g a beh avior.
2. In op eran t con dition in g, a beh avior th at is not part of the individual’s natural repertoire can
be learn ed th rou gh rein orcem en t.

B. Features
1. Th e likelih ood th at a behavior will occu r is increased by positive or negative reinforcement
an d decreased by punishment or extinction (Table 7.1).
Chapter 7 Learning Theory 67

t a b l e 7.1 Features of Operant Conditioning


Example: A mother would like her 8-year-old son to stop hitting his 6-year-old brother. She can achieve this goal by using
one of the following features of operant conditioning.

Feature Effect on Behavior Example Comments

Positive rein- Behavior is Child increases his kind Reward or reinforcement (praise) increases
forcement increased by behavior toward his desired behavior (kindness toward brother)
reward younger brother to get A reward can be praise or attention as well as a
praise from his mother tangible reward like money
Negative rein- Behavior is Child increases his kind Active avoidance of an aversive stimulus
forcement increased by behavior toward his (being scolded) increases desired behavior
HELP OTHERS SO THAT GOD WILL HELP YOU.

avoidance or younger brother to avoid (kindness toward brother)


escape being scolded
Punishment Behavior is Child decreases his hitting Delivery of scolding decreases unwanted
decreased by an behavior after his behavior (hitting brother) rapidly but not
aversive stimulus mother scolds him permanently
Extinction Behavior is Child stops his hitting Extinction is more effective than punishment for
eliminated by behavior when the long term reduction in unwanted behavior
nonreinforcement behavior is ignored There may be an initial increase in hitting
behavior before it disappears

a. Typ es o rein orcem en t in clu d e:


(1) Positive reinforcement (reward) is th e in trodu ction o a p ositive stim u lu s th at resu lts
in an in crease in th e rate o b eh avior.
(2) Negative reinforcement (escap e) is th e rem oval o an aversive stim u lu s th at also
resu lts in an in crease in th e rate o beh avior.
b. Punishment is th e in trod u ction o an aversive stim u lu s aim ed at redu cin g th e rate o an
u n wan ted beh avior.
2. Extinction in op eran t con dition in g is th e grad u al d isap p earan ce o a learn ed b eh avior
wh en rein orcem en t (reward) is with h eld.
a. Th e p attern , or schedule, of reinforcement a ects h ow qu ickly a beh avior is learn ed an d
h ow qu ickly a beh avior becom es extin ct wh en it is n ot rewarded (Tab le 7.2).

t a b l e 7.2 Schedules of Reinforcement


Schedule Reinforcement Example Effect on Behavior
RISE USMLE NEPAL

Continuous Presented after every A teenager receives a candy bar each Behavior (putting in a dollar to receive
response time she puts a dollar into a vending candy) is rapidly learned but disappears
machine. One time she puts a dollar rapidly (has little resistance to extinc
in and nothing comes out. She never tion) when not reinforced (no candy
buys candy from the machine again comes out)
Fixed ratio Presented after a A man is paid $10 for every five hats he Fast response rate (many hats are made
designated number makes. He makes as many hats as he quickly)
of responses can during his shift
Fixed Presented after a A student has an anatomy quiz every The response rate (studying) increases
interval designated amount Friday. He studies for 10 min on toward the end of each interval (1 wk)
of time Wednesday nights, and for 2 h on
Thursday nights
When graphed, the response rate forms
a scalloped curve
Variable Presented after a After a slot machine pays off $5 for a The behavior (playing the slot machine)
ratio random and unpre single quarter, a woman plays $50 continues (is highly resistant to
dictable number of in quarters despite the fact that she extinction) despite the fact that it is only
responses receives no further payoffs reinforced (winning money) after a large
but variable number of responses
Variable Presented after a ran After 5 min of fishing in a lake, a man The behavior (fishing) continues (is highly
interval dom and unpredict catches a large fish. He then spends resistant to extinction) despite the
able amount of time 4 h waiting for another bite fact that it is only reinforced (a fish is
caught) after varying time intervals
68 BRS Behavioral Science

b. Resistance to extinction is th e orce th at p reven ts th e beh avior rom disap p earin g wh en


a reward is with h eld.

C. Shaping and modeling


1. Shaping in volves reward in g closer an d closer ap p roxim ation s o th e wan ted beh avior
u n til th e correct b eh avior is ach ieved (e.g., a ch ild learn in g to write is p raised wh en sh e
m akes a letter, even th ou gh it is n ot orm ed p er ectly).
2. Modeling is a typ e o observation al learn in g (e.g., an in dividu al beh aves in a m an n er sim i-
lar to th at o som eon e sh e adm ires).
HELP OTHERS SO THAT GOD WILL HELP YOU.
RISE USMLE NEPAL
Review Test

Directions: Each o th e n u m b ered item s or in com p lete statem en ts in th is section is ollowed by


an swers or by com p letion s o th e statem en t. Select th e one lettered an swer or com p letion th at
is best in each case.
HELP OTHERS SO THAT GOD WILL HELP YOU.

1. A grad e sch ool p rin cip al h as 1 week to try 3. For th is scen ario, wh ich elem en t
ou t a n ew f re-alarm system or th e sch ool. rep resen ts th e u n con dition ed resp on se?
He decides to test th e system th ree tim es (A) Stom ach growlin g in resp on se to th e
du rin g th e week. Th e f rst tim e th e alarm is wh ite van
sou n d ed, all o th e stu d en ts leave th e sch ool (B) Stom ach growlin g in resp on se to p izza
with in 5 m in u tes. Th e secon d tim e, it takes (C) Th e wh ite van
th e stu den ts 15 m in u tes to leave th e sch ool. (D) Pairin g th e wh ite van with gettin g p izza
Th e th ird tim e th e alarm is sou n ded, th e (E) Pizza
stu d en ts ign ore it. Th e stu den ts’ resp on se to
th e f re alarm th e th ird tim e it is sou n ded is 4. For th is scen ario, wh ich elem en t
m ost likely to h ave b een learn ed by rep resen ts th e u n con dition ed stim u lu s?
(A) sen sitization (A) Stom ach growlin g in resp on se to th e
(B) h abitu ation wh ite van
(C) classical con d ition in g (B) Stom ach growlin g in resp on se to p izza
(D) ixed ratio rein orcem en t (C) Th e wh ite van
(E) con tin u ou s rein orcem en t (D) Pairin g th e wh ite van with gettin g p izza
(F) variab le ratio rein orcem en t (E) Pizza
(G) p u n ish m en t
5. For th is scen ario, wh ich elem en t
2. Wh en ever a 46-year-old m an visits h is rep resen ts th e con dition ed stim u lu s?
p h ysician , h is b lood p ressu re is elevated.
(A) Stom ach growlin g in resp on se to th e
When th e p atien t takes h is own blood
wh ite van
p ressu re at h om e, it is u su ally n orm al. Th e
(B) Stom ach growlin g in resp on se to p izza
doctor says th at wh ile oth er tests n eed to
(C) Th e wh ite van
be d on e, th e p atien t is p rob ab ly sh owin g
(D) Pairin g th e wh ite van with gettin g p izza
“wh ite-coat h yp erten sion .” For th is scen ario,
(E) Pizza
RISE USMLE NEPAL

th e p atien t’s blood p ressu re in th e doctor’s


o f ce rep resen ts
6. In th e p ast, a ch ild h as on occasion
(A) th e u n con dition ed stim u lu s received m on ey or clean in g h is room .
(B) th e u n con dition ed resp on se Desp ite th e act th at h e h as n ot received
(C) th e con dition ed stim u lu s m on ey or clean in g h is room or th e p ast
(D) th e con dition ed resp on se m on th , th e ch ild’s room -clean in g beh avior
con tin u es (is resistan t to extin ction ). Th is
Questions 3–5 ch ild’s room -clean in g beh avior was p robably
learn ed u sin g wh ich o th e ollowin g
For th e p ast year, p izza h as b een sold rom a m eth od s?
wh ite van ou tsid e a h igh sch ool. Th e teen age
(A) Con tin u ou s rein orcem en t
stu den ts com p lain th at th ey are o ten em bar-
(B) Fixed ratio rein orcem en t
rassed b ecau se th eir stom ach s b egin to growl
(C) Fixed in terval rein orcem en t
wh en ever th ey see an y wh ite veh icle, even on
(D) Variab le ratio rein orcem en t
weeken d s. Th e p rin cip al th en b an s th e van
(E) Pu n ish m en t
rom sellin g p izza n ear th e sch ool an d th e
stu den ts’ stom ach s stop growlin g at th e sigh t
o wh ite veh icles.

69
70 BRS Behavioral Science

7. A 10-year-old ch ild wh o likes an d looks 11. Alth ou gh a ath er sp an ks h is ch ild wh en


u p to h er p h ysician states th at sh e wan ts to sh e h its th e d og, th e ch ild con tin u es to h it
becom e a doctor wh en sh e grows u p. Th is th e dog. Th is ch ild’s h ittin g beh avior is m ost
beh avior by th e ch ild is an exam p le o likely to be a resu lt o
(A) stim u lu s gen eralization (A) p u n ish m en t
(B) m odelin g (B) n egative rein orcem en t
(C) sh ap in g (C) p ositive rein orcem en t
(D) im p rin tin g (D) sh ap in g
(E) learn ed h elp lessn ess (E) classical con d ition in g
(F) extin ction
8. A 4-year-old ch ild wh o h as received (G) sen sitization
HELP OTHERS SO THAT GOD WILL HELP YOU.

beatin gs in th e p ast, rom wh ich h e cou ld (H) h abitu ation


n ot escap e, ap p ears u n resp on sive an d n o
lon ger tries to escap e n ew beatin gs. Th is 12. A p atien t with diab etes in creases h er
beh avior by th e ch ild is an exam p le o tim e sp en t exercisin g in order to redu ce
(A) stim u lu s gen eralization th e n u m ber o in su lin in jection s sh e m u st
(B) m odelin g receive. Th e in creased exercisin g beh avior is
(C) sh ap in g m ost likely to b e a resu lt o
(D) im p rin tin g (A) p u n ish m en t
(E) learn ed h elp lessn ess (B) n egative rein orcem en t
(C) p ositive rein orcem en t
9. A 2-year-old ch ild is a raid o n u rses in (D) sh ap in g
white u n i orm s. Wh en h is gran dm oth er (E) classical con d ition in g
com es to visit h im wearin g a wh ite jacket, h e (F) extin ction
begin s to cry. Th is beh avior by th e ch ild is an (G) sen sitization
exam p le o (H) h abitu ation
(A) stim u lu s gen eralization
(B) m odelin g 13. A 44-year-old wom an h as u n dergon e
(C) sh ap in g th ree session s o ch em oth erapy in a h osp ital.
(D) im p rin tin g Each session h as resu lted in n au sea. Be ore
(E) learn ed h elp lessn ess th e ou rth session , th e p atien t becom es
n auseated wh en sh e en ters th e h osp ital
10. A ath er scold s h is ch ild wh en sh e h its lobby. Th is p atien t’s reaction is a resu lt o th e
th e dog. Th e ch ild stop s h ittin g th e dog. Th is typ e o learn in g best described as
ch an ge in th e ch ild’s b eh avior is m ost likely (A) p u n ish m en t
to be a resu lt o (B) n egative rein orcem en t
(C) p ositive rein orcem en t
RISE USMLE NEPAL

(A) p u n ish m en t
(B) n egative rein orcem en t (D) sh ap in g
(C) p ositive rein orcem en t (E) classical con d ition in g
(D) sh ap in g (F) extin ction
(E) classical con d ition in g (G) sen sitization
(F) extin ction (H) h abitu ation
(G) sen sitization
(H) h abitu ation
Chapter 7 Learning Theory 71

14. A 43-year-old wom an is h avin g d i f cu lty Questions 17–20


allin g asleep. Her p h ysician advises h er to
listen to a 30-m in u te tap e o ocean sou n ds A ch ild com es to th e clin ical lab oratory to
an d th en go th rou gh a series o relaxation h ave a blood sam p le drawn or th e irst tim e
exercises every n igh t p rior to goin g to sleep. an d h as a p ain u l exp erien ce. Th e n ext tim e
Two weeks later, th e p atien t rep orts th at th e ch ild retu rn s or th is p rocedu re, sh e
sh e alls asleep as soon as sh e h ears th e begin s to cry wh en sh e sm ells th e od or o
sou n d s on th e tap e, even with ou t doin g th e an tisep tic in th e clin ic h allway. For each clin i-
relaxation exercises. Fallin g asleep wh en cal scen ario, select th e d e in ition th at b est
sh e h ears th e tap e is m ost likely to be d u e to describ es it.
wh ich o th e ollowin g?
HELP OTHERS SO THAT GOD WILL HELP YOU.

(A) Pu n ish m en t 17. Th e p ain u l blood with drawal p rocedu re


(B) Negative rein orcem en t at th e ch ild’s in itial visit can b e called th e
(C) Positive rein orcem en t (A) u n con dition ed stim u lu s
(D) Sh ap in g (B) u n con dition ed resp on se
(E) Classical con d ition in g (C) con d ition ed stim u lu s
(F) Extin ction (D) con d ition ed resp on se
(G) Sen sitization
(H) Hab itu ation 18. Th e an tisep tic odor th at leads to cryin g
on th e ch ild’s retu rn visit to th e laboratory
Questions 15 and 16 can b e called th e
(A) u n con dition ed stim u lu s
A m oth er p icks u p h er 3-m on th -old baby (B) u n con dition ed resp on se
each tim e h e cries. Th e ch ild cries on m ore (C) con d ition ed stim u lu s
an d m ore occasion s each day an d th e m oth er (D) con d ition ed resp on se
p icks h im u p m ore requ en tly.
19. Th e ch ild’s cryin g u p on th e sm ell o
15. Th is ch ild h as learn ed to get p icked u p an tisep tic can b e called th e
m ain ly by th e p rocess o
(A) u n con dition ed stim u lu s
(A) p u n ish m en t (B) u n con dition ed resp on se
(B) n egative rein orcem en t (C) con d ition ed stim u lu s
(C) p ositive rein orcem en t (D) con d ition ed resp on se
(D) sh ap in g
(E) classical con d ition in g 20. Th e ch ild’s cryin g wh en th e blood
(F) extin ction sam p le is d rawn can be called th e
(G) sen sitization
(A) u n con dition ed stim u lu s
(H) h abitu ation
RISE USMLE NEPAL

(B) u n con dition ed resp on se


(C) con d ition ed stim u lu s
16. The m other has learn ed to pick up the
(D) con d ition ed resp on se
child m ore requen tly m ain ly by the process o
(A) p u n ish m en t
(B) n egative rein orcem en t
(C) p ositive rein orcem en t
(D) sh ap in g
(E) classical con d ition in g
(F) extin ction
(G) sen sitization
(H) h abitu ation
An swers an d Exp lan ation s

Typical Board Question


The answer is E. Extin ction is th e disap p earan ce o a learn ed b eh avior wh en rein orcem en t is
with h eld . Th e ath er’s in terven tion , wh ich led to im p rovem en t in th is boy’s beh avior, can best
be d escrib ed as extin ction . In itially, th e ch ild’s b ad beh avior in creased in order to gain a reward
HELP OTHERS SO THAT GOD WILL HELP YOU.

(atten tion , even i it was scold in g). Wh en th e ath er stop p ed rein orcin g th e ch ild’s bad beh avior
an d in stead ign ored it, th e beh avior u ltim ately disap p eared (b ecam e extin ct). In p ositive rein -
orcem en t b eh avior is in creased by a reward ; in p u n ish m en t beh avior is decreased by an aver-
sive stim u lu s. Mod elin g is a typ e o ob servation al learn in g. Sh ap in g in volves rewardin g closer
an d closer ap p roxim ation s o th e wan ted beh avior u n til th e correct beh avior is ach ieved.

1. The answer is B. Th e stu d en ts’ resp on se to th e ire alarm is m ost likely to h ave been
learn ed by h abituation , th at is, desen sitization . In th is orm o learn in g, con tin u ed
exp osu re to a stim u lu s (th e ire alarm , in th is exam p le) resu lts in a d ecreased resp on se
to th e stim u lu s. Th u s, wh ile th e stu den ts resp on d qu ickly to th e ire alarm at irst, with
rep eated sou n din gs o th e alarm , th ey u ltim ately ail to resp on d to it. I sen sitization
h ad occu rred, th e stu d en ts wou ld h ave resp on d ed m ore qu ickly with each exp osu re to
th e alarm . In classical con dition in g, a n atu ral resp on se is elicited by a learn ed stim u lu s.
In op eran t con dition in g, rein orcem en t is a con sequ en ce o a beh avior th at alters th e
likelih ood th at th e beh avior will occu r again . Pu n ish m en t is th e in trodu ction o an
aversive stim u lu s th at redu ces th e rate o an u n wan ted beh avior, wh ile extin ction is th e
disap p earan ce o a learn ed beh avior wh en rein orcem en t is with h eld .
2. The answer is D. Th e p atien t’s elevated b lood p ressu re in th e doctor’s o ice is th e
con d ition ed (learn ed) resp on se. Th is resp on se resu lts rom an association th at h as b een
m ade by classical con d ition in g b etween th e doctor an d / or h is wh ite coat (con d ition ed
stim u lu s) an d som eth in g n egative in th e p atien t’s p ast (u n con d ition ed stim u lu s), a
reaction com m on ly called “wh ite-coat h yp erten sion .” Th e cu e th at th is resp on se is learn ed
is th at th e p atien t’s blood p ressu re is relatively n orm al wh en taken at h om e.
3. The answer is B. 4. The answer is E. 5. The answer is C. Th e u n con d ition ed stim u lu s (p izza)
p rodu ces th e u n con d ition ed resp on se (stom ach growlin g in resp on se to p izza). Th e
RISE USMLE NEPAL

u n con dition ed resp on se is re lexive an d au tom atic an d does n ot h ave to be learn ed. Th e
u n con dition ed stim u lu s (p izza) is th e on ly elem en t h ere th at by itsel will elicit a n atu ral
GI re lex (stom ach growlin g). Th e wh ite van is an exam p le o th e con dition ed stim u lu s.
In th is scen ario, th e con dition ed or learn ed stim u lu s cau ses th e sam e resp on se as th e
u n con dition ed or u n learn ed stim u lu s on ly a ter it is p aired with p izza (stom ach growlin g
in resp on se to p izza).
6. The answer is D. Th is ch ild h as received m on ey on u n p redictable occasion s or clean in g
h is room . Beh avior learn ed in th is way (i.e., by variable ratio rein orcem en t) is very
resistan t to extin ction an d con tin u es even wh en it is n ot rewarded. Beh avior learn ed by
ixed sch edu les o rein orcem en t (ratio or in terval) is less resistan t to extin ction . Beh avior
learn ed by con tin u ou s rein orcem en t is least resistan t to extin ction . Pu n ish m en t is
aversive an d is aim ed at su p p ressin g an u n d esirable beh avior.
7. The answer is B. Th is b eh avior is an exam p le o m odelin g; th e ch ild wan ts to b ecom e
like th e doctor she adm ires. In stim u lu s gen eralization , a n ew stim u lu s th at resem bles a
con d ition ed stim u lu s cau ses a con d ition ed resp on se. Sh ap in g in volves rewardin g closer
an d closer ap p roxim ation s o th e wan ted beh avior u n til th e correct beh avior is ach ieved.
Im p rin tin g is th e ten den cy o organ ism s to m ake an association with an d th en ollow th e

72
Chapter 7 Learning Theory 73

irst th in g th ey see a ter birth or h atch in g. In learn ed h elp lessn ess, an association is m ade
between an aversive stim u lu s an d th e in ability to escap e.
8. The answer is E. Th is ch ild is sh owin g learn ed h elp lessn ess, in wh ich an association
is m ad e between an aversive stim u lu s (b eatin gs) an d th e in ability to escap e. A ter th e
b eatin gs, th is ch ild m akes n o attem p t to escap e bu t in stead becom es h op eless an d
ap ath etic wh en aced with an oth er beatin g. Learn ed h elp lessn ess m ay be a m odel system
or th e d evelop m en t o dep ression (see also an swer to Qu estion 7).
9. The answer is A. Th is b eh avior is an exam p le o stim u lu s gen eralization . In th is exam p le, it
occu rs wh en a n ew con dition ed stim u lu s (th e gran d m oth er’s wh ite jacket) th at resem bles
th e origin al con dition ed stim u lu s (th e n u rse’s wh ite u n i orm ) resu lts in th e con dition ed
HELP OTHERS SO THAT GOD WILL HELP YOU.

resp on se (cryin g wh en h e sees h is gran dm oth er) (see also an swer to Qu estion 7).
10. The answer is A. Becau se th e beh avior (h ittin g th e dog) decreased, th e scoldin g th at th is
ch ild received is p rob ably p u n ish m en t. Both n egative an d p ositive rein orcem en t in crease
beh avior. Sh ap in g in volves reward in g closer an d closer ap p roxim ation s o th e wan ted
beh avior u n til th e correct b eh avior is ach ieved. In classical con dition in g, a n atu ral or
re lexive resp on se (b eh avior) is elicited by a learn ed stim u lu s (a cu e rom an in tern al or
extern al even t). (See also an swers to Qu estion s 11–16).
11. The answer is C. Becau se th e b eh avior (h ittin g th e d og) is in creased , th e scold in g th at
th is ch ild received is p rob ab ly p ositive rein orcem en t. Both n egative an d p ositive
rein orcem en t in crease b eh avior. Th e reward or rein orcem en t or th is h ittin g beh avior is
m ost likely to b e in creased atten tion rom th e ath er. Pu n ish m en t decreases beh avior.
12. The answer is B. Becau se th e beh avior (exercise) is in creased to avoid som eth in g n egative
(in su lin in jection s), th is is an exam p le o n egative rein orcem en t.
13. The answer is E. Th is com m on clin ical p h en om en on is an exam p le o classical
con d ition in g. In th is exam p le, a wom an com es in to th e h osp ital or an in traven ou s
(IV) ch em oth erapy treatm en t (u n con d ition ed stim u lu s). Th e ch em oth erapy dru g is
toxic an d sh e b ecom es n au seated a ter th e treatm en t (u n con dition ed resp on se). Th e
ollowin g m on th , wh en sh e en ters th e h osp ital lob by (con d ition ed stim u lu s), sh e b ecom es
n au seated (con d ition ed resp on se). Th u s, th e h osp ital wh ere th e treatm en ts took p lace
(con d ition ed stim u lu s) h as b ecom e p aired with ch em oth erapy (th e u n con dition ed
stim u lu s), wh ich elicited n au sea. Now, n au sea (con dition ed resp on se) can be elicited by
en terin g th e h osp ital lobby (con dition ed stim u lu s), even th ou gh sh e h as n ot yet received
th e m ed ication . In op eran t con dition in g, beh avior is learn ed by its con sequ en ces.
Mod elin g is a typ e o ob servation al learn in g. Sh ap in g in volves rewardin g closer an d closer
ap p roxim ation s o th e wan ted b eh avior u n til th e correct beh avior is ach ieved. Extin ction is
RISE USMLE NEPAL

th e disap p earan ce o a learn ed beh avior wh en rein orcem en t is with h eld.


14. The answer is E. Via classical con dition in g, th e p atien t h as m ade an association between
th e sou n ds on th e tap e an d sleep in g, so sh e n ow alls asleep as soon as sh e h ears th e
sou n ds.
15. The answer is C. 16. The answer is B. In th is exam p le, th e ch ild’s cryin g beh avior in creases
as a resu lt o p ositive rein orcem en t, bein g p icked u p by h is m oth er each tim e h e cries. Th e
m oth er’s beh avior (p ickin g u p th e ch ild ) in creases as a resu lt o n egative rein orcem en t;
sh e p icks h im u p to avoid h earin g h im cry.
17. The answer is A. 18. The answer is C. 19. The answer is D. 20. The answer is B. Th e p ain u l
b lood with d rawal p roced u re is th e u n con d ition ed stim u lu s. Th e an tisep tic od or in th e
clin ic h as b ecom e associated with th e p ain u l p roced u re an d elicits th e sam e resp on se;
it is th ere ore th e con d ition ed stim u lu s. Th e con d ition ed resp on se, cryin g in resp on se
to th e sm ell o th e an tisep tic, h as b een learn ed . Becau se cryin g in resp on se to th e p ain
o an in jection is au tom atic an d d oes n ot h ave to b e learn ed , it is th e u n con d ition ed
resp on se.
Clin ical Assessm en t o
c ha pte r
8 Patien ts with Beh avioral
Sym p tom s
HELP OTHERS SO THAT GOD WILL HELP YOU.

Typical Board Question


A p h ysician exam in es a severely d ep ressed 75-year-old wom an . Th e wom an relates th at sh e
eels so low th at sh e can n ot en joy an yth in g in h er li e an d th at even win n in g th e state lottery
wou ld n ot m ake h er eel an y better. Th e best descrip tion o th is p atien t’s m ood is
(A) an h edon ic
(B) dysp h oric
(C) eu th ym ic
(D) labile
(E) eu p h oric
(See “An sw ers an d Explan ation s” at th e en d of th e ch apter.)

I. OVERVIEW OF PSYCHOLOGICAL TESTING


A. Types of tests
1. Psych ological tests are u sed to assess in telligen ce, ach ievem en t, p erson ality, an d
p sych op ath ology.
2. Th ese tests are classi ied by u n ction al area evalu ated.
RISE USMLE NEPAL

B. Individual versus group testing


1. Tests adm in istered to on e in dividu al at a tim e allow care u l observation an d evalu ation o
th at p articu lar p erson ; a test battery looks at u n ction in g o an in d ivid u al in a n u m b er o
d i eren t u n ction al areas.
2. Tests given to a grou p o p eop le sim u ltan eou sly h ave th e advan tages o e icien t adm in is-
tration , gradin g, an d statistical an alysis.

II. INTELLIGENCE TESTS


A. Intelligence and mental age
1. In telligen ce is de in ed as th e ability to u n derstan d abstract con cep ts; reason ; assim ilate,
recall, an alyze, an d organ ize in orm ation ; an d m eet th e sp ecial n eeds o n ew situ ation s.
2. Mental age (MA), as de in ed by Al red Bin et, re lects a p erson’s level o in tellectu al u n c-
tion in g. Chronological age (CA) is th e p erson’s actu al age in years.

74
Chapter 8 Clinical Assessment of Patients with Behavioral Symptoms 75

B. Intelligence quotient (IQ)


1. IQ is th e ratio o MA to CA m u ltip lied by 100: MA/CA × 100 = IQ. An IQ o 100 m ean s th at th e
p erson’s m en tal an d ch ron ological ages are equ ivalen t.
2. Th e h igh est CA u sed to d eterm in e IQ is 15 years.
3. IQ is d eterm in ed to a large exten t by gen etics. However, poor nutrition an d illness during
development can n egatively a ect IQ.
4. Th e resu lts o IQ tests are in lu en ced by a p erson’s cu ltu ral backgrou n d an d em otion al
resp on se to testin g situ ation s.
5. IQ is relatively stab le th rou gh ou t li e. In th e absen ce o brain p ath ology, an in dividu al’s IQ
is essentially the same in old age as in childhood.

C. Normal intelligence
HELP OTHERS SO THAT GOD WILL HELP YOU.

1. As stated above, an IQ o 100 m ean s th at th e MA an d CA are ap p roxim ately th e sam e.


Normal or average IQ is in the range of 90–109.
2. Th e stan dard deviation (see Ch ap ter 26) in IQ scores is 15. A p erson with an IQ th at is
m ore th an 2 stan d ard d eviation s below th e m ean (IQ 70) is u su ally con sid ered in tellectu -
ally d isab led (see Ch ap ter 2). Classifications of intellectual disability (th e overlap or gap in
categories is related to di eren ces in testin g in stru m en ts) are:
a. Mild (IQ 50–70).
b. Mod erate (IQ 35–55).
c. Severe (IQ 20–40).
d. Pro ou n d (IQ <20).
3. A score b etween 71 an d 84 in dicates borderline in tellectu al u n ction in g.
4. A p erson with an IQ m ore th an 2 stan dard deviation s above th e m ean (IQ >130) h as su p e-
rior in telligen ce.

D. The Wechsler intelligence tests and the Vineland Adaptive Behavior Scales
1. Th e Wech sler Ad u lt In telligen ce Scale—Fou rth Edition (WAIS-IV) is th e m ost com m on ly
u sed IQ test.
2. Th e WAIS-R h as ou r in dex scores: Verbal Comprehension Index (VCI), Working Memory
Index (WMI), Perceptual Reasoning Index (PRI), an d Processing Speed Index (PSI).
a. Th e VCI an d WMI togeth er m ake u p th e verbal IQ.
b. Th e PRI an d PSI togeth er m ake u p th e performance IQ.
c. Th e Full Scale IQ (FSIQ) is gen erated by all ou r in dex scores.
3. Th e Wech sler In telligen ce Scale or Ch ildren (WISC) is u sed to test in telligen ce in ch ild ren
6–16½ years o age.
4. Th e Wech sler Presch ool an d Prim ary Scale o In telligen ce (WPPSI) is u sed to test in tel-
ligen ce in ch ildren 4–6½ years o age.
RISE USMLE NEPAL

5. Th e Vineland Adaptive Behavior Scales are u sed to evalu ate skills or daily livin g (e.g.,
d ressin g, u sin g th e telep h on e) in p eop le with in tellectu al disability (see Ch ap ter 2) an d
oth er ch allen ges (e.g., th ose with im p aired vision or h earin g).

III. ACHIEVEMENT TESTS


A. Uses
1. Ach ievem en t tests evalu ate h ow well an in dividu al h as m astered specific subject areas ,
su ch as readin g an d m ath em atics.
2. Th ese tests are u sed or evalu ation an d career cou n selin g in sch ools an d in du stry.

B. Specific achievement tests


1. Ach ievem en t tests in clu de th e Sch olastic Ap titu de Test (SAT), Medical College Ad m ission
Test (MCAT), an d Un ited States Med ical Licen sin g Exam in ation (USMLE).
2. Th e Wid e Ran ge Ach ievem en t Test (WRAT), wh ich is o ten u sed clin ically, evalu ates arith -
m etic, readin g, an d sp ellin g skills.
3. Ach ievem en t tests o ten u sed by sch ool system s in clu de th e Cali orn ia, Iowa, Stan ord,
an d Peab od y Ach ievem en t Tests.
76 BRS Behavioral Science

t a b l e 8.1 Personality Tests

Name of Test Uses Characteristics Examples

Minnesota The most commonly used Objective test “I avoid most social
Multiphasic objective personality test Patients answer 567 true (T) or false (F) situations” (T or F)
Personality Useful for primary care questions about themselves “I often feel jealous”
Inventory physicians because no Clinical scales include depression, paranoia, (T or F)
(MMPI-2) training is required for schizophrenia, and illness anxiety disorder “I like being active”
administration and scoring Validity scales identify trying to look ill (T or F)
Evaluates attitude of the patient (“faking bad,” i.e., malingering) or trying to
toward taking the test look well (“faking good”)
Rorschach Test The most commonly used Projective test
HELP OTHERS SO THAT GOD WILL HELP YOU.

projective personality test Patients are asked to interpret 10 bilaterally


Used to identify thought symmetrical inkblot designs (e.g.,
disorders and defense “Describe what you see in this figure”)
mechanisms
Thematic Stories are used to evaluate Projective test
Apperception unconscious emotions and Patients are asked to create verbal
Test (TAT) conflicts scenarios based on 30 drawings depicting
ambiguous situations (e.g., “Using this
picture, make up a story that has a
beginning, a middle, and an end”)

Sentence Used to identify worries and Projective test “My mother …” “I


Completion problems using verbal Patients complete sentences started by the wish …” “Most
Test (SCT) associations examiner people …”
Original source of Rorschach illustration: Kleinmuntz B. Essentials of Abnormal Psychology. New York, NY: Harper & Row; 1974. Original source of TAT
illustration: Phares EJ. Clinical Psychology: Concepts, Methods, and Profession. 2nd ed. Homewood, IL: Dorsey; 1984. Both from Krebs D, Blackman R.
Psychology: A First Encounter. Harcourt Brace Jovanovich; 1988:632. Used with permission.

IV. PERSONALITY TESTS


A. Person ality tests are u sed to evalu ate p sych op ath ology an d p erson ality ch aracteristics
an d are categorized by wh eth er in orm ation is gath ered objectively or p rojectively.

B. Objective personality tests (e.g., th e Min n esota Mu ltip h asic Person ality In ven tory [MMPI]
an d th e Million Clin ical Mu ltiaxial In ven tory [MCMI]) are based on qu estion s th at are easily
scored an d statistically an alyzed.
RISE USMLE NEPAL

C. Projective personality tests (e.g., th e Rorsch ach Test, th e Th em atic Ap p ercep tion Test
[TAT], an d th e Sen ten ce Com p letion Test) req u ire th e su b ject to in terp ret th e q u estion s.
Resp on ses are assu m ed to b e b ased on th e su b ject’s m otivation al state an d d e en se
m ech an ism s.
Uses o som e o th ese p erson ality tests are described in Table 8.1.

V. PSYCHIATRIC EVALUATION OF THE PATIENT WITH EMOTIONAL


SYMPTOMS
A. Psychiatric history: Th e p atien t’s p sych iatric h istory is taken as p art o th e m edical h istory.
Th e p sych iatric h istory in clu des qu estion s ab ou t m en tal illn ess, dru g an d alcoh ol u se, sexu al
activity, cu rren t livin g situ ation , an d sou rces o stress.
Chapter 8 Clinical Assessment of Patients with Behavioral Symptoms 77

t a b l e 8.2 Variables Evaluated on the Mental Status Examination (MSE)


Variable Patient Example

General Presentation
Appearance A 40-year-old male patient looks older than his age but is well groomed. He
Behavior seems defensive when asked about his past experiences with drugs and
Attitude toward the interviewer denies that he has ever used them
Level of consciousness He has a Glasgow Coma Scale score of 15 (see Table 5.4)
Cognition
Orientation, memory, attention, A 55-year-old female patient is oriented to person, place, and time and shows
concentration; cognitive, spatial, and normal memory (cognitive ability), understanding of three-dimensional
abstraction abilities; and speech (volume, space (spatial ability), and can tell you how an apple and an orange are
HELP OTHERS SO THAT GOD WILL HELP YOU.

speed, and articulation) alike (abstraction ability). However, she speaks too quickly and is difficult
to understand
Mood and Affect
Described (mood) and demonstrated (affect) A 35-year-old male patient describes feeling “low” and shows less external
emotions expression of mood than expected (depressed with a restricted affect)
Match of emotions with current events
Thought
Form or process of thought A 40-year-old female patient tells you, in excessive detail (circumstantiality:
Thought content (e.g., delusion) problem in process of thought), that the Mafia is after her (a delusion: See
Table 11.1)
Perception
Illusion (see Table 11.1) A 12-year-old girl tells you that the clothes in her closet look like a person is in
Hallucination (see Table 11.1) there (an illusion). She then describes hearing voices (a hallucination)
J udgment and Insight A 38-year-old woman tells you that she would open a stamped letter found on
the sidewalk to see if it contained money. She also says that she knows
this would be dishonest (normal, insightful response)
Reliability A 55-year-old patient correctly provides the details of his previous illnesses
(a reliable patient)
Control of Aggressive and Sexual Impulses A 35-year-old man tells you that he often overreacts emotionally, although
there is little provocation (poor impulse control)

B. The mental status examination (MSE) and related instruments


1. Th e MSE is a stru ctu red in terview th at is u sed to evalu ate an in dividu al’s cu rren t state o
m en tal u n ction in g (Tab le 8.2).
2. Objective ratin g scales o dep ression th at are com m on ly u sed in clu de th e Hamilton,
Raskin, Zung, an d Beck scales.
a. In th e Ham ilton an d Raskin scales, an exam in er rates th e p atien t.
b. In th e Zu n g an d Beck scales (Table 8.3), th e p atien t rates h im sel (e.g., m easu res
RISE USMLE NEPAL

in clu de sadn ess, gu ilt, social with drawal, an d sel -blam e).
3. Term s u sed to describe p sych op h ysiological sym p tom s an d m ood in p atien ts with p sy-
ch iatric sym p tom s are listed in Tab le 8.4.

t a b l e 8.3 Items in the Beck Depression Inventory-II (BDI-II)

1. Sadness 12. Social withdrawal


2. Pessimism 13. Indecisiveness
3. Sense of failure 14. Negative body image
4. Dissatisfaction 15. Inability to work
5. Guilt 16. Insomnia
6. Expectation of punishment 17. Fatigability
7. Dislike of self 18. Loss of appetite
8. Self-blame 19. Loss of weight
9. Suicidal ideation 20. Preoccupation with health
10. Episodes of crying 21. Low level of sexual interest
11. Irritability
Each item can be scored from 0 to 3. Total scores of 30–63 indicate severe depression; scores of 5–9 indicate little or no depression.
78 BRS Behavioral Science

t a b l e 8.4 Glossary of Psychophysiological States


Psychophysiological State Symptom(s)
Mood
Euphoric Strong feelings of elation
Expansive Feelings of self-importance and generosity
Irritable Easily annoyed and quick to anger
Euthymic Normal mood, with no significant depression or elevation of mood
Dysphoric Subjectively unpleasant feeling
Anhedonic Inability to feel pleasure
Labile (mood swings) Alternates between euphoric and dysphoric moods
Affect
HELP OTHERS SO THAT GOD WILL HELP YOU.

Restricted Decreased display of emotional responses


Blunted Greatly decreased display of emotional responses
Flat No display of emotional responses
Labile Sudden alterations in emotional responses not related to environmental events
Fear and Anxiety
Fear Fright caused by real danger
Anxiety Fright caused by imagined danger
Free floating anxiety Fright not associated with any specific cause
Consciousness and Attention
Normal Alert, can follow commands, normal verbal responses
Clouding of consciousness Inability to respond normally to external events
Somnolence Abnormal sleepiness
Stupor Responds only to shouting, shaking, or uncomfortable prodding
Coma Total unresponsiveness
RISE USMLE NEPAL
Review Test

Directions: Each o th e n u m b ered item s or in com p lete statem en ts in th is section is ollowed by


an swers or by com p letion s o th e statem en t. Select th e one lettered an swer or com p letion th at
is best in each case.

1. A 12-year-old ch ild wh o is h avin g 5. A doctor is evalu atin g a 20-year-old


HELP OTHERS SO THAT GOD WILL HELP YOU.

di f cu lty in sch ool is given an in telligen ce em ale p atien t. Wh ich o th e ollowin g


test. Th e test determ in es th at th e ch ild ch aracteristics o th e p atien t is best
is u n ction in g m en tally at th e level o evalu ated u sin g th e Min n esota Mu ltip h asic
an 8-year-old ch ild. Wh at category o Person ality In ven tory-2 (MMPI-2)?
in tellectu al u n ction best describes th is (A) Skills or daily livin g
ch ild? (B) Dep ression
(A) Severely in tellectu ally d isabled (C) Kn owledge o gen eral in orm ation
(B) Mod erately in tellectu ally d isab led (D) Read in g com p reh en sion
(C) Mildly in tellectu ally disabled (E) In telligen ce
(D) Borderlin e
(E) Norm al or average 6. A 6-year-old ch ild is tested an d is ou n d
to have an IQ o 50. Th ere are n o sign if can t
2. A ch ild is tested an d ou n d to h ave m ed ical f n din gs. At th is tim e, it can b e
a m en tal age o 12 years. Th e ch ild’s exp ected th at th e ch ild is ab le to
ch ron ological age is 10 years. Wh at is th e IQ (A) iden ti y som e colors
o th is ch ild ? (B) ride a two-wh eeled bicycle
(A) 40 (C) u n derstan d th at death is p erm an en t
(B) 60 (D) copy a trian gle
(C) 80 (E) u tilize an in tern alized m oral sen se o
(D) 100 righ t an d wron g
(E) 120
7. For evalu atin g th e sel -care skills o a
3. A 29-year-old wom an tells th e d octor th at 22-year-old wom an with an IQ o 60 or
sh e o ten h ears th e voice o Abrah am Lin coln p lacem en t in a grou p h om e, wh at is th e m ost
sp eakin g d irectly to h er. Th is wom an is ap p rop riate test?
RISE USMLE NEPAL

sh owin g a disord er o (A) Th em atic Ap p ercep tion Test (TAT)


(A) p ercep tion (B) Min n esota Mu ltip h asic Person ality
(B) in sigh t In ven tory-2 (MMPI-2)
(C) ju dgm en t (C) Wech sler In telligen ce Scale or Ch ildren –
(D) m ood Revised (WISC-R)
(E) a ect (D) Rorsch ach Test
(E) Vin elan d Social Matu rity Scale
4. A ch ild is tested an d is ou n d to h ave an IQ (F) Wide Ran ge Ach ievem en t Test (WRAT)
o 90. Wh at category o in tellectu al u n ction (G) Beck Dep ression In ven tory-II (BDI-II)
best describ es th is ch ild ? (H) Raskin Dep ression Scale
(A) Severely in tellectu ally d isabled (I) Wiscon sin Card Sortin g Test
(B) Mod erately in tellectu ally d isab led
(C) Mildly in tellectu ally disabled
(D) Borderlin e
(E) Norm al or average

79
80 BRS Behavioral Science

8. For determ in in g, usin g bilaterally 11. Th e m ost ap p rop riate test or evalu atin g
sym m etrical in kblots, which de en se abstract reason in g an d p roblem solvin g in a
m echan ism s are used by a 25-year-old 54-year-old em ale p atien t is th e
wom an , what is the m ost appropriate test? (A) TAT
(A) TAT (B) MMPI-2
(B) MMPI-2 (C) WISC-R
(C) WISC-R (D) Rorsch ach Test
(D) Rorsch ach Test (E) Vin elan d Social Matu rity Scale
(E) Vin elan d Social Matu rity Scale (F) WRAT
(F) WRAT (G) BDI-II
(G) BDI-II (H) Raskin Dep ression Scale
HELP OTHERS SO THAT GOD WILL HELP YOU.

(H) Raskin Dep ression Scale (I) Wiscon sin Card Sortin g Test
(I) Wiscon sin Card Sortin g Test
12. A 24-year-old p atien t with sch izop h ren ia
9. For evalu atin g d ep ression in a 54-year-old tells th e p h ysician th at th e CIA is listen in g
m ale p atien t u sin g a sel -ratin g scale, wh at is to his telep h on e con versation s th rou gh h is
th e m ost ap p rop riate test? television set. Th is p atien t is d escribin g
(A) TAT (A) a h allu cin ation
(B) MMPI-2 (B) an illu sion
(C) WISC-R (C) clou d in g o con sciou sn ess
(D) Rorsch ach Test (D) blu n ted a ect
(E) Vin elan d Social Matu rity Scale (E) a d elu sion
(F) WRAT
(G) BDI-II
(H) Raskin Dep ression Scale
(I) Wiscon sin Card Sortin g Test

10. For evalu atin g, by a p rim ary care


p h ysician , illn ess an xiety d isord er in a
54-year-old m ale p atien t u sin g tru e/ alse
qu estion s, wh at is th e m ost ap p rop riate test?
(A) TAT
(B) MMPI-2
(C) WISC-R
(D) Rorsch ach Test
(E) Vin elan d Social Matu rity Scale
(F) WRAT
RISE USMLE NEPAL

(G) BDI-II
(H) Raskin Dep ression Scale
(I) Wiscon sin Card Sortin g Test
An swers an d Exp lan ation s

Typical Board Question


The answer is A. Th is severely d ep ressed 75-year-old wom an is sh owin g an h edon ia, th e in abil-
ity to eel p leasu re, a ch aracteristic o severe dep ression . Eu p h oric m ood is an elated m ood,
wh ile eu th ym ic m ood is a n orm al m ood, with n o sign i ican t dep ression or elevation . Dysp h oric
HELP OTHERS SO THAT GOD WILL HELP YOU.

m ood is a su bjectively u n p leasan t eelin g. Labile m oods (m ood swin gs) are alteration s between
elevated an d dysp h oric m ood s.

1. The answer is C. Usin g th e IQ orm u la, IQ = MA/ CA × 100, th e IQ o th is ch ild is 8 years


(m en tal age)/ 12 years (ch ron ological age) × 100, th at is, abou t 66 (IQ). An in d ivid u al with
an IQ o 66 is classi ied with m ild in tellectu al disability (IQ 50–70).
2. The answer is E. Usin g th e IQ orm u la, th e IQ o th e ch ild is 12/ 10 × 100 = 120.
3. The answer is A. Th is 29-year-old wom an wh o believes th at sh e h ears th e voice o
Abrah am Lin coln is sh owin g an au d itory h allu cin ation , wh ich is a d isord er o p ercep tion .
Disorders o ju d gm en t, in sigh t, m ood , an d a ect are oth er variables assessed on th e
Men tal Statu s Exam in ation (MSE).
4. The answer is E. An in divid u al with an IQ o 90 is classi ied as h avin g n orm al or average
in tellectu al u n ction (IQ 90–109).
5. The answer is B. Clin ical scales o th e Min n esota Mu ltip h asic Person ality In ven tory-2
(MMPI-2) evalu ate d ep ression as well as illn ess an xiety disorder, p aran oia, sch izop h ren ia,
an d oth er ch aracteristics. In telligen ce, in clu din g gen eral in orm ation an d read in g
com p reh en sion , can b e tested u sin g th e Wech sler Ad u lt In telligen ce Scale.
6. The a nswer is A. With an IQ o 50, the m en tal age o this 6-year-old child is 3 years. This is
calculated usin g th e IQ orm ula: IQ = MA/ CA × 100, that is, 50 = x/ 6 × 100, x = 3. Ch ildren who
are 3 years o age can iden ti y som e colors. However, the ability to ride a two-wheeled bicycle,
un derstan d th e m ean in g o death , copy a trian gle, or utilize an in tern alized m oral sen se o
right an d wron g do n ot develop un til about the m en tal age o 6 years (see Chap ter 1).
7. The answer is E. Th e Vin elan d Social Matu rity Scale is th e m ost ap p rop riate test or
RISE USMLE NEPAL

evalu atin g th e sel -care skills o th is wom an with in tellectu al disability or p lacem en t in a
grou p h om e.
8. The answer is D. Th e Rorsch ach Test, wh ich u tilizes b ilaterally sym m etrical in k blots, is th e
m ost ap p rop riate test to d eterm in e wh ich d e en se m ech an ism s are u sed by th is wom an .
9. The answer is G. For evalu atin g dep ression in th is p atien t u sin g a sel -ratin g scale,
th e m ost ap p rop riate test is th e Beck Dep ression In ven tory-II (BDI-II). In th e Raskin
Dep ression Scale, th e p atien t is rated by an exam in er.
10. The answer is B. Th e Min n esota Mu ltip h asic Person ality In ven tory-2 (MMPI-2) is th e m ost
ap p rop riate test or u se by a p rim ary care p h ysician to evalu ate dep ression in a p atien t
sin ce it is an ob jective test an d n o sp ecial train in g is requ ired or adm in istration an d
scorin g. Th e MMPI-2 u ses tru e/ alse qu estion s to evalu ate p erson ality ch aracteristics an d
p sych op ath ology. In con trast, in terp retation o p rojective p erson ality tests requ ires sp eci ic
train in g.

81
82 BRS Behavioral Science

11. The answer is I. Th e Wiscon sin Card Sortin g Test is th e m ost ap p rop riate test or evalu atin g
abstract reason in g an d p roblem solvin g in a p atien t. In th is test, a p atien t is asked to sort
128 resp on se cards th at vary in color, orm , an d n u m ber.
12. The answer is E. A alse b elie , in th is case th at th e CIA is listen in g to on e’s telep h on e
con versation s th rou gh th e television set, is an exam p le o a delu sion . A h allu cin ation is a
alse p ercep tion , an d an illu sion is a m isp ercep tion o reality (see also Table 11.1). Clou d in g
o con sciou sn ess is th e in ab ility to resp on d to extern al even ts, wh ile blu n ted a ect is a
d ecreased d isp lay o em otion al resp on ses.
HELP OTHERS SO THAT GOD WILL HELP YOU.
RISE USMLE NEPAL
Su bstan ce-Related
c ha pte r
9 Disorders
HELP OTHERS SO THAT GOD WILL HELP YOU.

Typical Board Question


A ter u n d ergoin g h is secon d d etoxi ication or h eroin addiction , a 30-year-old m an re u ses to
go in to a m eth adon e m ain ten an ce p rogram as h e did a ter h is irst detoxi ication . Th e m ost
likely reason th at th is p atien t wou ld re u se to be en rolled in a m eth adon e m ain ten an ce p ro-
gram at th is tim e is th at h e h as learn ed rom h is p reviou s exp erien ce th at m eth adon e
(A) cau ses severe dru g allergies
(B) h as sign i ican t side e ects
(C) h as to b e given m u ltip le tim es p er day
(D) m ain ten an ce p rogram s requ ire h im to wait in lin e
(E) is exp en sive
(See “An sw ers an d Explan ation s” at th e en d of th e ch apter.)

I. SUBSTANCE-RELATED DISORDERS: DEFINITIONS,


EPIDEMIOLOGY, AND DEMOGRAPHICS
A. Definitions
1. Su bstan ce-related disorders in clu de su bstan ce u se disorders an d su bstan ce-in du ced
RISE USMLE NEPAL

disord ers.
2. Substance-related disorders are m aladap tive p attern s o su bstan ce u se th at lead to
im p airm en t o occu p ation al, p h ysical, or social u n ction in g; th ey ran ge in severity as
m ild , m od erate, or severe.
3. Substance-induced disorders in clu de with drawal sym p tom s an d toleran ce.
a. Withdrawal is th e d evelop m en t o p h ysical or p sych ological sym p tom s a ter th e redu c-
tion or cessation o in take o a su bstan ce.
b. Tolerance is th e n eed or in creased am ou n ts o th e su bstan ce to ach ieve th e sam e p osi-
tive p sych ological e ect.
c. Cross-tolerance is th e develop m en t o toleran ce to on e su bstan ce as th e resu lt o u sin g
an oth er su bstan ce.

B. Epidemiology and demographics


1. Alcoh ol, m ariju an a, n on m ed ical u se o p rescrip tion agen ts (e.g., op ioids, sedatives),
cocain e, h allu cin ogen s, in h alan ts, an d h eroin are, accord in g to sel -rep orts, th e m ost
com m on ly u sed su bstan ces in th e Un ited States (Tab le 9.1).
2. Th e u se o illegal su b stan ces is m ore com m on am on g you n g adu lts (18–25 years of age )
an d is twice as common in males .

83
84 BRS Behavioral Science

t a b l e 9.1 Self-reported Substance Use in Last Month

Substance Number of People (in Millions)

Alcohol 136.9
Marijuana 19.3
Prescription agents (mainly nonmedical use of pain relievers) 6.5
Cocaine 1.5
Hallucinogen 1.3
Inhalants 0.5
Heroin 0.3
HELP OTHERS SO THAT GOD WILL HELP YOU.

Source: Substance Abuse and Mental Health Services Administration, 2013.

3. Most su b stan ces can b e classi ied categorically as stimulants, sedatives, opioids, or halluci-
nogens an d related agen ts.
4. Most su b stan ces can b e adm in istered by a n u m ber o rou tes. Rou tes th at p rovide qu ick
access to th e blood stream , an d h en ce th e brain , are o ten p re erred by u sers (e.g., sn i in g
in to th e n ose [“sn ortin g] an d sm okin g rath er th an in gestin g).

II. STIMULANTS
A. Overview
1. Stim u lan ts are central nervous system activators th at in clu de ca ein e, n icotin e, am p h et-
am in es, an d cocain e.
2. Th e e ects o u se an d with drawal o th ese su bstan ces can be ou n d in Table 9.2.

B. Caffeine is ou n d in co ee (125 m g/ cu p ), tea (65 m g/ cu p ), cola (40 m g/ cu p ), n on p rescrip tion


stim u lan ts, an d over-th e-cou n ter d iet agen ts.

C. Nicotine is a toxic su bstan ce p resen t in tobacco. Cigarette sm okin g decreases li e exp ectan cy
m ore th an th e u se o an y oth er su bstan ce. Sm okin g is in creasin g m ost in teen aged girls.

D. Amphetamines are u sed clin ically an d also are d ru gs o illegal u se.


RISE USMLE NEPAL

1. Th ey are m edically in dicated in th e m an agem en t o atten tion de icit h yp eractivity disor-


der (ADHD) (see Ch ap ter 15) an d narcolepsy (see Ch ap ter 10). Th ey are som etim es u sed
to treat depression in th e elderly an d term in ally ill, an d dep ression an d obesity in p atien ts
wh o d o n ot resp on d to oth er treatm en ts (see Ch ap ter 12).
2. Th e m ost com m on clin ically u sed am p h etam in es are dextroamphetamine (Dexed rin e),
methamphetamine (Desoxyn ), an d a related com p ou n d, methylphenidate (Ritalin ).
3. “Bath salts ” (m eth ylen edioxypyrovaleron e [MDPV]), “ecstasy” (m eth ylen edioxym eth -
am p h etam in e [MDMA]), an d “speed,” “ice ,” an d “base ” (m eth am p h etam in e) are street
n am es or am p h etam in e com p ou n d s.

E. Cocaine
1. “Crack” an d “freebase ” are ch eap, sm okable orm s o cocain e; in exp en sive, p u re orm
cocain e is snorted.
2. Hyperactivity an d growth retardation are seen in newborns o m oth ers wh o u sed cocain e
du rin g p regn an cy.
3. Tactile hallucinations o bu gs crawlin g on th e skin (i.e., formication) are seen with th e u se
o cocain e (“cocain e bu gs”).
Chapter 9 Substance-Related Disorders 85

t a b l e 9.2 Effects of Use and Withdrawal of Stimulant Agents

Substances Effects of Use Effects of Withdrawal

Psychological
Caffeine, nicotine Increased alertness and attention span Lethargy
Agitation and insomnia Mild depression of mood
Mild improvement in mood

Physical
Decreased appetite Increased appetite with slight weight gain
Increased blood pressure and heart rate Fatigue
(tachycardia) Headache
HELP OTHERS SO THAT GOD WILL HELP YOU.

Increased gastrointestinal activity


Psychological
Amphetamines, cocaine Significant elevation of mood (lasting only 1 h Significant depression of mood
with cocaine) Strong psychological craving (peaking a
Increased alertness and attention span few days after the last dose)
Aggressiveness, impaired judgment Irritability
Psychotic symptoms (e.g., paranoid delusions
with amphetamines and formication with
cocaine)
Agitation and insomnia
Physical
Loss of appetite and weight Hunger (particularly with amphetamines)
Pupil dilation Pupil constriction
Increased energy Fatigue
Tachycardia and other cardiovascular effects,
which can be life threatening
Seizures (particularly with cocaine)
Reddening (erythema) of the nose due to
“snorting” cocaine
Hypersexuality

F. Neurotransmitter associations
1. Stim u lan t dru gs work p rim arily by in creasin g th e availability o dopamine (DA) in th e
brain .
2. Stim u lan t u se stimulates the release of DA and blocks the reuptake of DA. Th ese action s
resu lt in in creased availability o th is n eu rotran sm itter in th e syn ap se.
3. In creased a vailab ility o DA in th e syn ap se is ap p aren tly in volved in th e m ood -
RISE USMLE NEPAL

elevatin g e ects o stim u lan ts an d op ioid s (th e “reward” system o th e b rain ). As in


sch izop h ren ia (see Ch ap ter 11), in creased DA availab ility m ay also resu lt in psychotic
symptoms .

III. SEDATIVES
A. Overview
1. Sed atives are central nervous system depressants th at in clu de alcoh ol, barbitu rates, an d
ben zod iazep in es.
2. Sed ative agen ts work p rim arily by increasing th e activity o th e in h ibitory n eu rotran sm it-
ter g-am in obu tyric acid (GABA).
3. Hosp italization o p atien ts or with drawal rom sedatives is p ru den t; th e with drawal syn -
drom e m ay in clu d e seizu res, p sych otic sym p tom s su ch as h allu cin ation s an d delu sion s,
an d card iovascu lar sym p tom s th at are life-threatening. Th e e ects o u se an d with drawal
o sed atives can b e ou n d in Tab le 9.3.
86 BRS Behavioral Science

t a b l e 9.3 Effects of Use and Withdrawal of Sedative Agents

Substances Effects of Use Effects of Withdrawal

Psychological
Alcohol, benzodiazepines, barbiturates Mild elevation of mood Mild depression of mood
Decreased anxiety Increased anxiety
Somnolence Insomnia
Behavioral disinhibition Psychotic symptoms (e.g., delusions and
formication)
Disorientation
Physical
HELP OTHERS SO THAT GOD WILL HELP YOU.

Sedation Tremor
Poor coordination Seizures
Respiratory depression Cardiovascular symptoms such as tachycardia
and hypertension

B. Alcohol
1. Acu te associated p rob lem s
a. Traffic accidents, homicide, suicide , an d rape are correlated with th e con cu rren t u se o
alcoh ol.
b. Child physical an d sexual abuse , dom estic p artn er abu se, an d elder abu se are also asso-
ciated with alcoh ol u se.
2. Ch ron ic associated p roblem s
a. Thiamine deficiency resu ltin g in Wernicke’s syndrome an d u ltim ately in Korsakoff’s syn-
drome (see Ch ap ter 14) is associated with lon g-term u se o alcoh ol.
b. Liver dysfunction, gastroin testin al p roblem s (e.g., u lcers), an d redu ced li e exp ectan cy
also are seen in h eavy u sers o alcoh ol.
c. Fetal alcohol syndrome (in clu d in g acial ab n orm alities, red u ced h eigh t an d weigh t, an d
in tellectu al disability) is seen in th e o sp rin g o wom en wh o drin k du rin g p regn an cy.
d. A ch ild h ood h istory o p roblem s su ch as ADHD an d conduct disorder (see Ch ap ter 15)
correlates with alcoh olism in th e adu lt.
3. Identification of alcoholism. Becau se in d ividu al wh o over-u se alcoh ol com m on ly u se
den ial as a de en se m ech an ism (see Ch ap ter 6), p ositive resp on ses to in direct qu eries
su ch as th ose in th e CAGE questions can h elp a p h ysician iden ti y a p erson wh o h as a
p roblem with alcoh ol. Th e CAGE qu estion s are: “Do you ever
a. try to Cu t down on you r d rin kin g?”
b. get An gry wh en som eon e com m en ts on you r drin kin g?”
RISE USMLE NEPAL

c. eel Gu ilty ab ou t you r drin kin g?”


d. take a drin k as an Eye-op en er in th e m orn in g?”
4. Intoxication
a. Legal in toxication is de in ed as 0.08%–0.15% blood alcohol concentration, dep en din g on
in dividu al state laws.
b. Com a occu rs at a blood alcoh ol con cen tration o 0.40%–0.50% in n on alcoh olics.
c. Psych otic sym p tom s (e.g., h allu cin ation s) m ay be seen in alcoh ol in toxication as well
as in with drawal (see below).
5. Delirium tremens (“the DTs”)
a. Alcohol withdrawal delirium (also called d eliriu m trem en s or “th e DTs”) m ay occu r d u r-
in g th e 1st week o with d rawal rom alcoh ol (m ost com m on ly on th e 3rd d ay o h osp i-
talization ). It u su ally occu rs in p atien ts wh o h ave been drin kin g h eavily or years.
b. Deliriu m trem en s is life threatening; th e m ortality rate is abou t 20%.

C. Barbiturates
1. Barbitu rates are u sed m ed ically as sleeping pills , sedatives, an tian xiety agen ts (tran qu il-
izers), an ticon vu lsan ts, an d an esth etics.
2. Frequ en tly u sed an d over-u sed b arb itu rates in clu de am obarbital, p en tobarbital, an d
secobarb ital.
Chapter 9 Substance-Related Disorders 87

3. Barbitu rates cau se resp iratory d ep ression an d h ave a low safety margin. As su ch , th ey are
very dan gerou s in overd ose.

D. Benzodiazepines
1. Ben zodiazep in es are u sed m edically as antianxiety agents , sedatives, m u scle relaxan ts,
an ticon vu lsan ts, an d an esth etics an d to treat alcohol withdrawal (p articu larly lon g-actin g
agen ts su ch as ch lordiazep oxide an d diazep am [see Ch ap ter 16]).
2. Ben zodiazep in es h ave a high safety margin u n less taken with an oth er sedative, su ch as
alcoh ol.
3. Flumazenil (Mazicon , Rom azicon ), a ben zodiazep in e recep tor an tagon ist, can reverse th e
e ects o ben zodiazep in es in cases o overdose.
HELP OTHERS SO THAT GOD WILL HELP YOU.

IV. OPIOIDS
A. Overview
1. Narcotics or op ioid dru gs in clu d e agents used medically as analgesics (e.g., m orp h in e) as
well as dru gs u sed illegally (e.g., h eroin ). Th e e ects o u se an d with drawal o op ioids can
b e ou n d in Table 9.4.
2. Com p ared to m edically u sed op ioids su ch as m orp h in e an d m eth adon e, illegal op ioids
su ch as heroin are m ore p oten t, cross th e blood–brain barrier m ore qu ickly, h ave a aster
on set o action , an d h ave more euphoric action.
3. In con trast to barbitu rate with drawal, wh ich m ay be atal, death from withdrawal of opioids
is rare u n less a seriou s p h ysical illn ess is p resen t.

B. Methadone and related agents


1. Methadone an d buprenorphine (Temgesic, Suboxone [wh en com b in ed with n aloxon e]) are
synthetic opioids u sed to treat h eroin ad d iction (see Tab le 9.5); th ey can also cau se p h ysi-
cal dep en den ce an d toleran ce.
2. Th ese legal op ioid s can b e su b stitu ted or illegal op ioids, su ch as h eroin , to p reven t with -
drawal sym p tom s.
3. Advantages o m eth ad on e an d b u p ren orp h in e over h eroin
a. Meth ad on e is d isp en sed by federal health authorities without charge to registered
addicts.
b. Buprenorphine is a n op ioid recep tor p artial a gon ist–an ta gon ist (m akin g it u n likely
to cau se resp iratory d ep ression ) th at ca n b lock b oth with d rawal sym p tom s an d ,
RISE USMLE NEPAL

wh en com b in ed with n a loxon e, th e eu p h oric action o h eroin . Bu p ren orp h in e can


n ow b e prescribed by physicians in private practice wh o com p lete a b rie train in g
p rogram .

t a b l e 9.4 Effects of Use and Withdrawal of Opioid Agents

Substances Effects of Use Effects of Withdrawal


Psychological
Heroin, methadone, other opioids Elevation of mood Depression of mood
Relaxation Anxiety
Somnolence Insomnia
Physical
Sedation Sweating, muscle aches, fever
Analgesia Rhinorrhea (running nose)
Respiratory depression (overdose may be fatal) Piloerection (goose bumps)
Constipation Yawning
Pupil constriction (miosis) Stomach cramps and diarrhea
Pupil dilation (mydriasis)
88 BRS Behavioral Science

t a b l e 9.5 Management (in Order of Utility, Highest to Lowest) of Use of Sedatives, Opioids,
Stimulants, and Hallucinogens and Related Agents

Category Immediate Management/Detoxification Extended Management/Maintenance

Minor Stimulants: Caffeine, Eliminate or taper from the diet Peer support group (e. g., “Smokenders”)
nicotine Analgesics to control headache due to Antidepressants (particularly bupropion
withdrawal [Zyban]) to prevent smoking
Support from family members or
nonsmoking physician
Hypnosis to prevent smoking
Nicotine-containing gum, patch, or nasal
spray (efficacy is equivocal)
HELP OTHERS SO THAT GOD WILL HELP YOU.

Stimulants: Amphetamines, Benzodiazepines to decrease agitation Education for initiation and maintenance
cocaine Antipsychotics to treat psychotic symptoms of abstinence
Medical and psychological support
Sedatives: Alcohol, Hospitalization Education for initiation and maintenance
benzodiazepines, Flumazenil (Mazicon) to reverse the effects of of abstinence
barbiturates benzodiazepines Specifically for alcohol: Alcoholics
Substitution of long-acting barbiturate (e.g., Anonymous (AA) or other peer support
phenobarbital) or benzodiazepine (e.g., group (12-step program), disulfiram
chlordiazepoxide [Librium] in decreasing (Antabuse), psychotherapy, behavior
doses); IV diazepam (Valium), lorazepam therapy, naloxone (Narcan), naltrexone
(Ativan), or phenobarbital if seizures occur (ReVia), acamprosate (Campral),
Specifically for alcohol: Thiamine (vitamin B1) topiramate (Topamax)
and restoration of nutritional state
Opioids: Heroin, methadone, Hospitalization and naloxone (Narcan) for Methadone or buprenorphine (Subutex)
opioids used medically overdose maintenance program
Clonidine (alpha 2-agonist) to stabilize Naltrexone or buprenorphine plus
the autonomic nervous system during naloxone (Suboxone) used
withdrawal prophylactically to block the effects of
Substitution of long-acting opioid (e.g., opioids
methadone) in decreasing doses to Narcotics Anonymous (NA) or other peer
decrease withdrawal symptoms support program
Hallucinogens and Related Calming or “talking down” the patient Education for initiation and maintenance
Agents: Marijuana, Benzodiazepines to decrease agitation of abstinence
hashish, LSD, PCP, Antipsychotics to treat psychotic symptoms
psilocybin, mescaline
PCP, phencyclidine; LSD, lysergic acid diethylamide.

c. Both agen ts can be taken orally. Th e in traven ou s m eth od o dru g u se em p loyed by


m an y h eroin ad dicts m ay in volve sh arin g con tam in ated n eedles, th u s con tribu tin g to
RISE USMLE NEPAL

AIDS an d h ep atitis B in ection .


d. Both agen ts h ave a longer duration of action.
e. Both agen ts cau se less euphoria and drowsiness , allowin g p eop le on m ain ten an ce regi-
m en s to keep th eir job s an d avoid th e crim in al activity th at is n ecessary to m ain tain a
costly h eroin h ab it.

V. HALLUCINOGENS AND RELATED AGENTS


A. Overview
1. Hallu cin ogen s an d related agen ts in clu de lysergic acid dieth ylam id e (LSD), p h en cyclidin e
(PCP or “angel dust”), can n abis (tetrah ydrocan n abin ol, m ariju an a, h ash ish ), p silocybin
( rom m u sh room s), m escalin e ( rom cactu s), an d ketam in e (“Special K”).
2. Hallu cin ogen s p rom ote altered states o con sciou sn ess, wh ich are u su ally p leasu rable
bu t can also b e righ ten in g (“bad trips ”).
3. In creased availability o serotonin is associated with th e e ects o som e o th ese agen ts
(e.g., LSD). Th e e ects o u se an d with d rawal o h allu cin ogen s an d related agen ts can be
ou n d in Table 9.6.
Chapter 9 Substance-Related Disorders 89

t a b l e 9.6 Effects of Use and Withdrawal of Hallucinogens and Related Agents

Substances Effects of Use Effects of Withdrawal

Psychological
Cannabis (marijuana, hashish), Altered perceptual states (auditory and visual Few, if any,
lysergic acid diethylamide (LSD), hallucinations, alterations of body image, distortions psychological
phencyclidine (PCP) or “angel dust,” of time and space) withdrawal symptoms
psilocybin, mescaline Elevation of mood
Impairment of memory (may be long term)
Reduced attention span
“Bad trips” (frightening perceptual states)
“Flashbacks” (a re-experience of the sensations
HELP OTHERS SO THAT GOD WILL HELP YOU.

associated with use in the absence of the drug even


months after the last dose)
Physical
Impairment of complex motor activity Few, if any, physical
Cardiovascular symptoms withdrawal symptoms
Sweating
Tremor
Nystagmus (PCP)

B. Marijuana
1. Tetrah yd rocan n ab in ol (THC) is th e p rim ary active com p ou n d ou n d in m ariju an a.
2. In low doses, m ariju an a increases appetite an d relaxation an d cau ses con ju n ctival
redden in g.
3. Ch ron ic u sers exp erien ce lung problems associated with sm okin g an d a decrease in m oti-
vation (“the amotivational syndrome ”) ch aracterized by lack o desire to work an d in creased
ap ath y.
4. Alth ou gh it is n ot legal in all o th e Un ited States, marijuana or m edical u se is p erm itted
in m ore th an 25 states an d Wash in gton , DC, p rim arily or treatin g glau com a an d can -
cer treatm en t–related n au sea an d vom itin g. Con trolled sale o m ariju an a or recreation al
p u rp oses is allowed in Colorado, Wash in gton State, Oregon , Alaska an d th e District o
Colu m bia an d will soon be legal in oth er states as well.

C. LSD and PCP


1. LSD is ingested an d PCP is smoked in a m ariju an a or oth er cigarette.
2. Wh ile LSD an d PCP both cau se altered p ercep tion , in con trast to LSD, episodes of violent
RISE USMLE NEPAL

behavior occu r with PCP use.


3. Em ergen cy dep artm en t in din gs or PCP in clu d e h yp erth erm ia an d nystagmus (vertical or
h orizon tal ab n orm al eye m ovem en ts).
4. PCP bin ds with N-m eth yl- d -asp artate (NMDA) recep tors o glutamate -gated ion ch an n els.
5. Con su m p tion o m ore th an 20 m g o PCP m ay cau se seizu res, com a, an d death .

VI. CLINICAL FEATURES OF SUBSTANCE-RELATED DISORDERS


A. Laboratory findings can o ten con irm su b stan ce u se (Table 9.7).

B. Emergency department findings. Ch an ges in th e p u p il o th e eye an d p resen ce or absen ce


o p sych otic sym p tom s can qu ickly n arrow th e search or th e su bstan ce resp on sible or
p atien ts’ sym p tom s in th e em ergen cy dep artm en t (Table 9.8).
90 BRS Behavioral Science

t a b l e 9.7 Laboratory Findings for Selected Drugs of Use


Length of Time After Use that
Category Elevated Levels in Body Fluids (e.g., Blood, Urine) Substance Can Be Detected
Stimulants Cotinine (nicotine metabolite) 1–2 d
Amphetamine 1–2 d
Benzoylecgonine (cocaine metabolite) 1–3 d in occasional users; 7–12 d
in heavy users
Sedatives Alcohol 7–12 h
g-Glutamyltransferase (GGT) 7–12 h
Specific barbiturate or benzodiazepine or its metabolite 1–3 d
Opioids Heroin 1–3 d
HELP OTHERS SO THAT GOD WILL HELP YOU.

Methadone 2–3 d
Hallucinogens and Cannabinoid metabolites 3–28 d
related agents Phencyclidine (PCP) 7–14 d in heavy users
Serum glutamic oxaloacetic transaminase (SGOT) (also called >7 d
aspartate transaminase [AST] or aspartate aminotransferase
[ASAT/AAT]) or creatine phosphokinase (CPK) (with PCP use)

VII. MANAGEMENT
A. Management of substance-related disorders ran ges rom ab stin en ce an d p eer su p p ort grou p s
to dru gs th at b lock p h ysical an d p sych ological with drawal sym p tom s.

B. Man agem en t o with d rawal sym p tom s in clu des im m ed iate treatm en t or detoxifica-
tion (“detox”) an d exten ded m an agem en t aim ed at p reven tin g relap se (“maintenance ”)
(Tab le 9.5).

C. Extended management programs for opioid addiction include


1. Methadone maintenance. Meth adon e, an op ioid, is disp en sed once a day to registered h er-
oin addicts by a federal government agency. Th ere is no cost to en rollees, bu t th ey m u st be
p resen t early in th e m orn in g (allowin g th em to m ain tain em p loym en t) an d, i th ey arrive
late, m ay h ave to wait in line or th eir d ose.
2. Buprenorphine maintenance. Physicians in private practice , wh o h ave taken a 1-day certi i-
cation cou rse, can p rescrib e b u p ren orp h in e (Su bu tex) or bu p ren orp h in e p lu s n aloxon e
(Su boxon e) to m ain tain h eroin ad d icts. Wh ile th ese drugs m ay be costly to th e p atien t,
RISE USMLE NEPAL

in con trast to m eth adon e m ain ten an ce, th ere is n o n eed or th e p atien t to register as an
add ict with th e ederal govern m en t n or are th ere lim itation s on tim e o dosin g.

D. Dual diagnosis or m en tally ill–ch em ically add icted (MICA) p atien ts requ ire treatm en t or
b oth su b stan ce-related d isorders an d th e com orbid p sych iatric illn ess (e.g., m ajor dep res-
sion ), o ten in a sp ecial u n it in th e h osp ital.

t a b l e 9.8 Quick Emergency Department Identification of the Used Substance

Emergency Department Observation Seen with Use of Seen with Withdrawal from
Pupil dilation Stimulants Opioids
Hallucinogens (e.g., LSD) Alcohol and other sedatives
Pupil constriction Opioids Stimulants
Psychotic symptoms (e.g., Stimulants Alcohol and other sedatives
hallucinations, delusions) Alcohol
Hallucinogens
Cardiovascular symptoms Stimulants Alcohol and other sedatives
LSD, lysergic acid diethylamide.
Review Test

Directions: Each o th e n u m b ered item s or in com p lete statem en ts in th is section is ollowed by


an swers or by com p letion s o th e statem en t. Select th e one lettered an swer or com p letion th at
is best in each case.
HELP OTHERS SO THAT GOD WILL HELP YOU.

Questions 1 and 2 4. A p h ysician is d oin g an em p loym en t


p h ysical on a 40-year-old m ale p atien t. Th e
p h ysician su sp ects th at th e p atien t h as a
A 29-year-old m an p resen ts to th e em ergen cy
p roblem with alcoh ol. Th e n ext step th at th e
dep artm en t com p lain in g o stom ach cram p s,
p h ysician sh ou ld take is to
agitation , severe m u scle ach es, an d diarrh ea.
Ph ysical exam in ation reveals th at th e p atien t (A) ch eck h is liver u n ction
is sweatin g; h as dilated p u p ils, a ever, an d a (B) ask h im i h e h as a p rob lem with alcoh ol
ru n n y n ose; an d sh ows goose b u m p s on h is (C) call h is p reviou s em p loyer or
skin . in orm ation
(D) ask h im th e CAGE qu estion s
1. O th e ollowin g, th e m ost likely cau se o (E) ch eck or th e stigm ata o alcoh olism (e.g.,
th is p ictu re is stria, broken blood vessels on th e n ose)
(A) alcoh ol u se 5. A 20-year-old em ale p atien t tells th e
(B) alcoh ol with drawal doctor th at sh e h as little in terest in goin g
(C) h eroin u se back to sch ool or in gettin g a job. Sh e also
(D) h eroin with drawal rep orts th at sh e o ten craves sn ack ood
(E) am p h etam in e with drawal an d h as gain ed over 10 p ou n ds in th e p ast
4 m on th s. Wh at su bstan ce is th is p atien t
2. O th e ollowin g, th e m ost e ective m ost likely to b e u sin g?
im m ediate treatm en t or relie o th is
p atien t’s sym p tom s is (A) Ph en cyclidin e (PCP)
(B) Lysergic acid d ieth ylam ide (LSD)
(A) n aloxon e (C) Mariju an a
(B) n altrexon e (D) Cocain e
(C) an an tip sych otic (E) Heroin
(D) a stim u lan t
RISE USMLE NEPAL

(E) clon idin e 6. A 22-year-old studen t tells the doctor that


he has been using “bath salts” nightly. Which o
3. Wh ich o th e ollowin g dru gs is (are), by the ollowing e ects o the drug is the student
sel -rep ort, th e m ost requ en tly u sed in th e m ost likely to experience when he is using?
Un ited States?
(A) In creased atigu e
(A) Hallu cin ogen s (B) Decreased p ain th resh old
(B) In h alan ts (C) In creased ap p etite
(C) Cocain e (D) Decreased ap p etite
(D) Heroin (E) Decreased lib id o
(E) Mariju an a
7. A p atien t h as b een u sin g h eroin or th e
p ast year. Wh ich o th e ollowin g is m ost
likely to ch aracterize th is p atien t?
(A) Age 40–50 years
(B) Fem ale gen der
(C) In som n ia wh en u sin g th e dru g
(D) An xiou s m ood wh en u sin g th e dru g
(E) Elevated m ood wh en u sin g th e dru g
91
92 BRS Behavioral Science

8. A p erson wh o u ses illegal d ru gs is m ost Questions 12 and 13


likely to be in wh ich o th e ollowin g age
grou p s?
A 40-year-old em ale p atien t wh o h as been
(A) 10–15 years takin g a ben zodiazep in e daily in h igh doses
(B) 15–18 years over th e p ast 5 years abru p tly stop s takin g th e
(C) 18–25 years dru g.
(D) 25–35 years
(E) 35–45 years 12. Wh en a p h ysician sees h er th e day a ter
h er last d ose, sh e is m ost likely to sh ow
9. A 60-year-old m an is b rou gh t to
(A) h yp ersom n ia
th e h osp ital a ter a all ou tside o a
(B) trem or
HELP OTHERS SO THAT GOD WILL HELP YOU.

n eigh borh ood bar. Radiologic stu dies


(C) leth argy
in dicate th at th e p atien t h as a ractu red h ip,
(D) resp iratory dep ression
an d su rgery is p er orm ed im m ed iately. Two
(E) sed ation
days later, th e p atien t b egin s to sh ow an
in ten se h an d trem or an d tach ycard ia. He
13. With ou t treatm en t or with drawal, wh at
tells th e doctor th at h e h as been “sh aky” ever
li e-th reaten in g sym p tom is th is wom an
sin ce h is ad m ission an d th at th e sh akin ess is
m ost likely to ace over su bsequ en t days?
gettin g worse. Th e p atien t states th at wh ile
h e eels righ ten ed, h e is com orted by th e (A) Seizu res
act th at th e n u rse is an old rien d (h e h as (B) Com a
n ever m et th e n u rse be ore). He also rep orts (C) Violen t beh avior
th at h e h as started to see sp iders crawlin g (D) Blood dyscrasia
on th e walls an d can eel th em crawlin g on (E) Severe h yp oten sion
h is arm s. Th e doctor n otes th at th e p atien t’s
sp eech seem s to b e d ri tin g rom on e su bject 14. A 24-year-old p atien t is exp erien cin g
to an oth er. O th e ollowin g, wh at is th e m ost in ten se h u n ger as well as tired n ess an d
likely cau se o th is p ictu re? h eadach e. Th is p atien t is m ost likely to be
with d rawin g rom wh ich o th e ollowin g
(A) Alcoh ol u se
su bstan ces?
(B) Alcoh ol with drawal
(C) Heroin u se (A) Alcoh ol
(D) Heroin with drawal (B) Am p h etam in es
(E) Am p h etam in e with d rawal (C) Ben zodiazep in es
(D) Ph en cyclidin e (PCP)
10. A p h ysician discovers th at h is 28-year- (E) Heroin
old em ale p atien t is u sin g cocain e. Wh ich o
th e ollowin g can th e d octor exp ect to see in 15. In th e Un ited States, th e grou p in wh ich
RISE USMLE NEPAL

th is p atien t? sm okin g cu rren tly sh ows th e largest in crease


is
(A) Severe p h ysical sign s o with d rawal
(B) Little p sych ological cravin g in (A) teen aged m ales
with drawal (B) m id dle-aged m ales
(C) Lon g-lastin g m ood elevation (C) teen aged em ales
(D) Delu sion s (D) m id dle-aged em ales
(E) Sedation with u se (E) eld erly em ales

11. A 20-year-old m an wh o h as b een 16. Wh at is th e m ajor m ech an ism o action


drin kin g eigh t cu p s o co ee a d ay or o cocain e on n eu rotran sm itter system s in
th e p ast week p resen ts in or a p h ysical th e brain ?
exam in ation . At th is tim e, th is m an is m ost (A) Blocks reu p take o d op am in e
likely to sh ow (B) Blocks release o d op am in e
(A) tach ycard ia (C) Blocks reu p take o seroton in
(B) decreased p eristalsis (D) Blocks release o seroton in
(C) weigh t gain (E) Blocks release o n orep in ep h rin e
(D) atigu e
(E) h eadach e
Chapter 9 Substance-Related Disorders 93

17. A ter 20 years o sm okin g, a 45-year-old 20. A 35-year-old m an is brou gh t to th e


em ale p atien t h as decid ed to qu it. O th e em ergen cy dep artm en t con u sed an d
ollowin g, wh ich is m ost likely to b e seen an xiou s. Th e m an rep orts th at som eon e is
as a resu lt o th is p atien t’s with drawal rom tryin g to kill h im bu t h e d oes n ot kn ow wh o
n icotin e? th e p erson is. In itial p h ysical exam in ation
(A) Weigh t gain reveals elevated h eart an d resp iration
(B) Eu p h oria rates. Wh ile in th e em ergen cy room , th e
(C) Excitability p atien t h as a seizu re an d th en develop s
(D) Deliriu m trem en s li e-th reaten in g card iovascu lar sym p tom s.
(E) Lon g-term abstin en ce Th e d ru g th at th is p atien t is m ost likely to b e
with d rawin g rom is
HELP OTHERS SO THAT GOD WILL HELP YOU.

18. A 43-year-old m an with a 5-year h istory (A) p h en cyclidin e (PCP)


o HIV tells h is p h ysician th at h e h as been (B) lysergic acid dieth ylam ide (LSD)
sm okin g m ariju an a a ew tim es a day to treat (C) h eroin
h is sym p tom s o n au sea an d lack o ap p etite. (D) secobarb ital
To ob tain th e m ariju an a, th e p atien t n otes (E) m ariju an a
th at, alth ou gh m ariju an a is illegal in h is
state, h e grows it in h is backyard. Th e 21. A 40-year-old wom an is brough t to th e
d octor’s best resp on se to th is p atien t’s em ergen cy room by a colleague. The p atien t
revelation is in itially shows clin ched sts an d in ten se
(A) “I am sorry bu t growin g or u sin g tooth grin din g. Sh ortly th erea ter, sh e begin s
m ariju an a is illegal an d I m u st n oti y th e to exp erien ce violen t seizu res. Her colleagu e
p olice” states th at over th e p ast 6 m on th s th e wom an
(B) “I h ave read ab ou t oth er p atien ts h as requ en tly been late or work an d h as
growin g m ariju an a” b een given warn in gs by th e b o ss b eca u se
(C) “Are you aware th at m ariju an a can cau se h er work is “n ot up to p ar.” The colleagu e
resp iratory p roblem s?” also states th at th e wom an’s m oth er died
(D) “Th ere are a n u m ber o m edication s th at yesterday an d th at th e p atien t h ad been with
I can p rescrib e to h elp alleviate you r h er m oth er 24 hours p er day or 3 days p rior
n au sea an d lack o ap p etite in p lace o to her death an d cam e to work directly rom
m ariju an a” th e h osp ital. O th e ollowin g, wh ich is th e
(E) “Do you th in k th at u sin g m ariju an a h as m ost likely cau se o th e p atien t’s sym p tom s?
n egative lon g-term e ects?” (A) Prim ary seizu re disord er
(B) Cerebral h em orrh age
19. A 24-year-old m an is brou gh t to th e (C) Malin gerin g
h osp ital com p lain in g o severe ch ills, (D) Com p licated grie reaction
stom ach cram p s, diarrh ea, an d ru n n y n ose (E) Alcoh ol with drawal
RISE USMLE NEPAL

(rh in orrh ea). Th e d octor d eterm in es th at


th e p atien t is with d rawin g rom h eroin an d
adm in isters clon idin e. In op ioid with drawal,
clon idin e im p roves sym p tom s by its action
as a
(A) beta 1-agon ist
(B) beta 2-agon ist
(C) alp h a 1-agon ist
(D) alp h a 2-agon ist
94 BRS Behavioral Science

Questions 22–26 24. Wh en a p h ysician exam in es a 17-year-


old h igh sch ool stu den t, sh e n otes th at
For th e p atien t in each n u m bered qu estion , h e h as eryth em a o th e n ose. Du rin g th e
select th e lettered dru g h e or sh e is m ost likely in terview, th e stu d en t seem s with drawn an d
to be u sin g. sad.
(A) Alcoh ol
25. A 28-year-old m an is h osp italized
(B) Secobarbital
a ter tryin g to ju m p rom th e roo o on e
(C) Cocain e
ap artm en t bu ildin g to an oth er. His rien ds
(D) Meth ylp h en id ate
relate th at p rior to th e ju m p, th e m an an grily
(E) Ca ein e
th reaten ed th em becau se th ey wou ld n ot
(F) Diazep am
HELP OTHERS SO THAT GOD WILL HELP YOU.

ju m p with h im .
(G) Heroin
(H) Mariju an a
26. A 22-year-old wom an is brou gh t to
(I) Nicotin e
th e em ergen cy room at 8 a m by h er rien d,
(J ) Ph en cyclidin e (PCP)
wh o states th at th e wom an has been actin g
(K) Lysergic acid d ieth ylam ide (LSD)
stran gely sin ce th e p reviou s even in g. Wh ile
lyin g on th e exam in in g table, th e p atien t
22. A 32-year-old m an is b rou gh t to a New
states th at sh e eels like sh e is f oatin g in th e
York City h osp ital. He ap p ears sedated b u t
air an d th e su n (th e overh ead electric ligh t) is
sh ows an elevated m ood . A b lood test reveals
big an d glarin g above h er.
th e p resen ce o HIV.

23. A 25-year-old m an is b rou gh t to


th e h osp ital a ter b ein g in volved in an
au tom obile acciden t in wh ich h e was drivin g
an d th e oth er d river was killed.
RISE USMLE NEPAL
An swers an d Exp lan ation s

Typical Board Question


The answer is D. Th e m ost likely reason th at th is p atien t, wh o h as h ad p rior exp erien ce in a m eth -
adon e m ain ten an ce p rogram , wou ld re u se to b e en rolled again is th at su ch p rogram s requ ire
waitin g in lin e each day to receive m eth ad on e. Meth adon e does n ot cau se severe dru g aller-
HELP OTHERS SO THAT GOD WILL HELP YOU.

gies n or does it h ave sign i ican t side e ects. It is given on ly on ce p er day an d is ree to p erson s
en rolled in m eth adon e m ain ten an ce p rogram s.

1. The answer is D. 2. The answer is E. Th e m ost likely cau se o th is p atien t’s sym p tom s o
sweatin g, m u scle ach es, stom ach cram p s, diarrh ea, ever, ru n n y n ose, goose bu m p s, an d
d ilated p u p ils is h eroin with drawal. Wh ile alcoh ol with drawal m ay be associated with
p u p il dilation , alcoh ol u se an d with drawal an d am p h etam in e with drawal are less likely to
cau se th is con stellation o sym p tom s. O th e ch oices given , th e m ost e ective im m ediate
treatm en t or h eroin with d rawal is clon idin e to stabilize th e au ton om ic n ervou s system .
Psych otic sym p tom s are u n com m on in op ioid with drawal, an d th is p atien t does n ot n eed
an an tip sych otic. Naloxon e an d n altrexon e as well as stim u lan ts will worsen rath er th an
am eliorate th e p atien t’s with d rawal sym p tom s.
3. The answer is E. Alm ost 20 m illion Am erican s rep ort th at th ey u se m ariju an a. In
con trast, 1.5 m illion , 1.3 m illion , 0.5 m illion , an d 0.3 m illion rep ort th at th ey u se cocain e,
h allu cin ogen s, in h alan ts, an d h eroin , resp ectively.
4. The answer is D. Th e n ext step in m an agem en t is or th e p h ysician to ask th is p atien t th e
CAGE qu estion s. Positive an swers to an y two o th ese qu estion s or to th e last on e alon e
in dicate th at h e h as a p roblem with alcoh ol. Patien ts with su ch p roblem s typ ically u se
d en ial as a de en se m ech an ism an d so rarely believe or adm it th at th ey h ave a p roblem
with alcoh ol. Liver u n ction p rob lem s or p resen ce o th e stigm ata o alcoh olism (e.g., stria,
b roken b lood vessels on th e n ose) do n ot n ecessarily in dicate th e p atien t cu rren tly h as a
p rob lem with alcoh ol. It is in ap p rop riate or th e doctor to call th e p reviou s em p loyer or
in orm ation .
5. The answer is C. Th e am otivation al syn drom e (e.g., lack o in terest in gettin g a job or goin g
RISE USMLE NEPAL

to sch ool) an d in creased ap p etite, p articu larly or sn ack oods, are ch aracteristically seen
in ch ron ic u sers o m ariju an a. Use o cocain e, h eroin , p h en cyclidin e (PCP), or lysergic
acid d ieth ylam ide (LSD) m ay cau se work-related p rob lem s b u t are less likely to in crease
ap p etite.
6. The answer is D. Like oth er stim u lan t dru gs, am p h etam in es su ch as “b ath salts” redu ce
ap p etite; u se can th u s resu lt in d ecreased bod y weigh t. Am p h etam in es also decrease
atigu e, in crease p ain th resh old, an d in crease libido.
7. The answer is E. Heroin u sers sh ow an elevated, relaxed m ood an d som n olen ce. Users are
m ost likely to b e you n g adu lt m ales.
8. The answer is C. Illegal d ru g u se is m ost com m on in p eop le 18–25 years o age.
9. The answer is B. Th e m ost likely cau se o trem or, tach ycardia, illu sion s (e.g., believin g th e
n u rse is an old rien d ), an d visu al an d tactile h allu cin ation s (e.g., orm ication —th e eelin g
o in sects crawlin g on th e skin ) in th is p atien t is alcoh ol with drawal, sin ce th e u se o alcoh ol
d u rin g th e p ast ew days o h osp italization is u n likely. His ractu red h ip m ay h ave been
su stain ed in th e all wh ile h e was in toxicated. Heroin u se an d h eroin an d am p h etam in e
with d rawal gen erally are n ot associated with p sych otic sym p tom s.

95
96 BRS Behavioral Science

10. The answer is D. Delu sion s an d oth er evid en ce o p sych osis are seen with th e u se o
cocain e. Th e in ten se eu p h oria p rodu ced by cocain e lasts on ly ab ou t 1 h ou r. Severe
p sych ological cravin g or th e d ru g p eaks 2–4 d ays a ter th e last dose, alth ou gh th ere m ay
be ew p h ysiologic sign s o with drawal. Cocain e in toxication is ch aracterized by agitation
an d irritability, n ot sedation .
11. The answer is A. Tach ycard ia, in creased p eristalsis, in creased en ergy, an d decreased
ap p etite are p h ysical e ects o stim u lan ts like ca ein e. Headach es are m ore likely to resu lt
rom with drawal rath er th an u se o stim u lan t dru gs.
12. The answer is B. 13. The answer is A. With drawal rom ben zodiazep in es is associated with
trem or, in som n ia, an d an xiety. Resp iratory d ep ression an d sedation are associated with th e
HELP OTHERS SO THAT GOD WILL HELP YOU.

u se o , n ot with d rawal rom , sedative dru gs. O th e ch oices in Qu estion 13, seizu res are th e
m ost com m on li e-th reaten in g sym p tom th at th is wom an will ace over th e n ext ew days.
14. The answer is B. Tiredn ess an d h eadach e are seen with with drawal rom stim u lan ts. Wh ile
in creased ap p etite can be seen in with drawal rom all stim u lan ts, th e m ost in ten se h u n ger
is seen with with drawal rom am p h etam in es.
15. The answer is C. In th e Un ited States, th e grou p in wh ich sm okin g cu rren tly sh ows th e
largest in crease is teen aged em ales.
16. The answer is A. Th e m ajor m ech an ism o action o cocain e on n eu ral system s is to block
th e reu p take o dop am in e, th ereby in creasin g its availability in brain syn ap ses. In creased
availab ility o d op am in e is in volved in th e “reward” system o th e brain an d th e eu p h oric
e ects o stim u lan ts.
17. The answer is A. Weigh t gain com m on ly occu rs ollowin g with drawal rom stim u lan ts su ch
as n icotin e. Mild d ep ression o m ood an d leth argy are also seen . Lon g-term ab stin en ce is
u n com m on in sm okers; m ost sm okers wh o qu it relap se with in 2 years. Deliriu m trem en s
occu r with with drawal rom sedatives su ch as alcoh ol.
18. The answer is D. Th e b est resp on se to th is p atien t’s revelation abou t growin g an d u sin g
m ariju an a is to recom m en d e ective bu t sa er su bstitu tes, or exam p le, p rescrip tion
m ed ication s to treat h is n au sea an d lack o ap p etite. It is n eith er ap p rop riate n or n ecessary
or a p h ysician to rep ort th e p atien t’s action s to th e p olice. Also, th is HIV-p ositive
p atien t is likely to be m ore con cern ed abou t eelin g ill in th e sh ort-term th an lon g-term
con sequ en ces o m ariju an a u se su ch as resp iratory p roblem s.
19. The answer is D. Clon id in e acts via in h ib ition o n oradren ergic activity th rou gh
p resyn ap tic stim u lation o alp h a 2-adren ergic n eu ron s.
RISE USMLE NEPAL

20. The answer is D. Th is 35-year-old p atien t is m ost likely to be with drawin g rom
secobarb ital, a barb itu rate. Barb itu rate with d rawal sym p tom s ap p ear abou t 12–20 h ou rs
a ter th e last dose an d in clu de an xiety, elevated h eart an d resp iration rates, p sych otic
sym p tom s (e.g., th e b elie th at som eon e is tryin g to kill h im ), con u sion , an d seizu res an d
can b e associated with li e-th reaten in g card iovascu lar sym p tom s. Th ere are ew p h ysical
with d rawal sym p tom s associated with m ariju an a, p h en cyclidin e (PCP), or lysergic acid
dieth ylam id e (LSD), an d th ose associated with h eroin are u n com ortable bu t rarely
p h ysically dan gerou s.
21. The answer is E. Th e violen t seizu res in th is 40-year-old wom an are m ost likely to be a sign
o alcoh ol with d rawal. Her laten ess an d p oor p er orm an ce at work over th e p ast 6 m on th s
are eviden ce th at sh e h as b een im p aired by alcoh ol. Sh e h as p robably becom e p h ysically
d ep en d en t on alcoh ol an d, becau se sh e b een with h er dyin g m oth er or at least 3 days,
h as n ot h ad th e op p ortu n ity to d rin k an d n ow is in with d rawal rom alcoh ol. It is m u ch
less likely th at th e seizu res are d u e to a p rim ary seizu re d isord er or cereb ral h em orrh age.
Th ere is n o reason or th is wom an to b e eign in g h er sym p tom s as wou ld be th e case in
m alin gerin g (see Ch ap ter 13), an d th e p h ysical in din gs su ggest an organ ic cau se rath er
th an a com p licated grie reaction (see Ch ap ter 3).
Chapter 9 Substance-Related Disorders 97

22. The answer is G. Th e p resen ce o HIV as well as sign s o sed ation an d eu p h oria in d icate
th at th is p atien t is an in traven ou s h eroin u ser.
23. The answer is A. Alcoh ol u se is com m on ly associated with au tom ob ile acciden ts.
24. The answer is C. Eryth em a o th e n ose is a resu lt o sn ortin g cocain e, an d dep ressed m ood
is seen in with drawal rom th e dru g.
25. The answer is J . Aggressiven ess an d p sych otic beh avior (ju m p in g rom on e roo top to
an oth er) in dicate th at th is p atien t h as u sed PCP.
26. The answer is K. Th is wom an , wh o h as b een actin g stran gely over a n u m ber o h ou rs an d
is exp erien cin g ou t-o -body exp erien ces (e.g., eelin gs o loatin g in th e air) an d illu sion s
HELP OTHERS SO THAT GOD WILL HELP YOU.

(e.g., m istakin g th e overh ead ligh t or th e su n ), h as p robab ly taken LSD. Th e p atien t’s lack
o aggression or agitation in d icates th at th e h allu cin ogen sh e h as u sed is less likely to h ave
been PCP.
RISE USMLE NEPAL
Norm al Sleep an d Sleep
c ha pte r
10 Disorders
HELP OTHERS SO THAT GOD WILL HELP YOU.

Typical Board Question


A 52-year-old overweigh t sin gle m an rep orts th at h e o ten wakes u p with a h eadach e. He
also n otes th at, d esp ite goin g to sleep at 10 p m an d wakin g at 7 a m , h e is tired all th e tim e an d
rep eated ly alls asleep du rin g th e d ay. Evalu ation o th e arterial blood o th is p atien t is m ost
likely to reveal
(A) resp iratory acidosis
(B) resp iratory alkalosis
(C) m etab olic alkalosis
(D) in creased PaO 2
(E) decreased PaCO 2
(See “An sw ers an d Explan ation s” at th e en d of th e ch apter.)

I. NORMAL AWAKE AND SLEEP STATES


A. Circadian cycle. In th e ab sen ce o ou tsid e in orm ation abou t ligh t an d d ark p eriods (i.e.,
zeitgebers), h u m an s sh ow a circadian cycle, in clu din g awake an d sleep in g states closer to
25 hours th an to 24 h ou rs in len gth .
RISE USMLE NEPAL

B. Awake state. Beta an d alp h a waves ch aracterize th e electroen cep h alogram (EEG) o th e
awake in dividu al (Table 10.1).
1. Beta waves over th e ron tal lobes are com m on ly seen with active mental concentration.
2. Alpha waves over th e occip ital an d p arietal lobes are seen wh en a p erson relaxes with
closed eyes.
3. Sleep latency (p eriod o tim e rom goin g to bed to allin g asleep ) is typ ically less th an
10 m in u tes.

C. Sleep state. Du rin g sleep, b rain waves sh ow d istin ctive ch an ges (Table 10.1).
1. Sleep is d ivid ed in to rap id eye m ovem en t (REM) sleep an d non-REM sleep. Non -REM sleep
con sists o stages 1, 2, 3, and 4.
2. Map p in g th e tran sition s rom on e stage o sleep to an oth er du rin g th e n igh t p rodu ces a
stru ctu re kn own as sleep architecture (Figu re 10.1).
a. Sleep arch itectu re ch an ges with age. Th e elderly o ten h ave p oor sleep qu ality becau se
agin g is associated with reduced REM sleep and delta sleep (stages 3–4 or slow wave)
an d in creased n igh ttim e awaken in gs, leadin g to poor sleep efficiency (p ercen t o tim e
actu ally sp en t sleep in g p er p ercen t o tim e tryin g to sleep ) (Table 10.2).

98
Chapter 10 Normal Sleep and Sleep Disorders 99

t a b l e 10.1 Electroencephalographic Tracings and Characteristics of the Awake State and


Sleep Stages

% Sleep Time in
Sleep Stage Associated EEG Pattern (Cycles/Second cps) Young Adults Characteristics

Awake Beta waves (14–30 cps) — Active mental concentration

— Relaxed with eyes closed


Alpha waves (8–13 cps)
HELP OTHERS SO THAT GOD WILL HELP YOU.

Stage 1 Theta waves (4–7 cps) 5% Lightest stage of sleep


characterized by peacefulness,
slowed pulse and respiration,
decreased blood pressure, and
episodic body movements
Stage 2 Sleep spindle (13–16 cps) and K-comple x 45% Largest percentage of sleep time;
bruxism (tooth grinding) may
occur

Stages 3 Delta (slow-wave sleep) waves (1–3 cps) 25% (decreases Deepest, most relaxed stage of
and 4 with age) sleep; sleep disorders, such
as night terrors, sleepwalking
(somnambulism), and bed-
wetting (enuresis), may occur

Rapid eye “Sawtooth,” beta, alpha, and theta waves 25% (decreases Dreaming; penile and clitoral
movement with age) erection; increased pulse,
(REM) respiration, and blood pressure;
REM
sleep absence of skeletal muscle
movement

b. Sedative agents , su ch as alcoh ol, barbitu rates, an d ben zodiazep in es, also are associ-
ated with reduced REM sleep and delta sleep.
c. Most delta sleep occu rs du rin g th e first half of the sleep cycle .
d. Longest REM periods occu r d u rin g th e second half of the sleep cycle .

3. During REM sleep, high levels of brain and cardiovascular activity occur.
RISE USMLE NEPAL

a. Average tim e to th e irst REM p eriod a ter allin g asleep (REM latency) is 90 minutes .
b. REM p eriods o 10–40 m in u tes each occu r abou t every 90 minutes th rou gh ou t th e n igh t.
c. A p erson wh o is dep rived o REM sleep on e n igh t (e.g., becau se o in adequ ate sleep,
rep eated awaken in gs, or sedative u se) h as in creased REM sleep th e n ext n igh t (REM
rebound).

Awa ke Wa king Wa king Wa king


REM a t e nd
REM REM REM of s le e p
s
1 cycle
e
g
a
2
t
s
p
3 De lta
e
e
De lta De lta
l
S
4

1 2 3 4 5 6 7
Hours of s le e p in a typica l young a dult
FIGURE 10.1. Sleep architecture in typical young adult. (Adapted from Wedding D. Behavior & Medicine. St. Louis, MO:
Mosby Year Book; 1995:416.)
100 BRS Behavioral Science

t a b l e 10.2 Summary of Characteristics of Sleep in Typical, Depressed, and Elderly People

Sleep Measure Typical Young Adult Depressed Young Adult Typical Elderly Adult

Sleep latency About 10 min >10 min >10 min


REM latency About 90 min About 45 min About 90 min
Sleep efficiency About 100% <100% <100%
Percentage delta About 25% <25% <25%
Percentage REM About 25% >25% <25%
REM, rapid eye movement.
HELP OTHERS SO THAT GOD WILL HELP YOU.

d. Exten ded REM d ep rivation or total sleep dep rivation m ay also resu lt in th e tran sien t
disp lay o psychopathology, u su ally an xiety or p sych otic sym p tom s.

D. Neurotransmitters are in volved in th e p rod u ction o sleep.


1. Increased levels o acetylcholine (ACh) in th e reticu lar orm ation increase both sleep effi-
ciency and REM sleep.
a. ACh levels, sleep e icien cy, an d REM sleep decrease in typ ical agin g as well as in
Alzh eim er’s d isease.
b. Patien ts takin g an tich olin ergic agen ts sh ow d ecreased REM sleep, wh ile p atien ts tak-
in g ch olin om im etic agen ts (e.g., p h ysostigm in e) sh ow in creased REM sleep.
2. Increased levels o dopamine decrease sleep efficiency. Treatm en t with an tip sych ot-
ics, wh ich b lock d op am in e recep tors, m ay im p rove sleep in p atien ts with p sych otic
sym p tom s.
3. Increased levels o norepinephrine decrease both sleep efficiency and REM sleep.
4. Increased levels o serotonin increase both sleep efficiency and delta sleep. Dam age to
th e dorsal rap h e n u clei, wh ich p rodu ce seroton in , decreases both o th ese m easu res.
Treatm en t with an tidep ressan ts, wh ich in crease seroton in availability, can im p rove sleep
e icien cy in d ep ressed p atien ts.

II. CLASSIFICATION OF SLEEP DISORDERS


Th e Diagn ostic an d Statistical Man u al of Men tal Disorders, Fifth Edition (DSM-5) classi ies sleep
disord ers in to sleep –wake d isord ers, b reath in g-related sleep disorders, an d p arasom n ias.
RISE USMLE NEPAL

A. Sleep–wake disorders in clu d e in som n ia, h yp ersom n olen ce, an d n arcolep sy an d are ch arac-
terized by p rob lem s in th e tim in g, qu ality, or am ou n t o sleep. Breathing-related sleep disor-
ders include sleep apnea, as well as circadian rhythm sleep disorder.

B. Parasomnias are ch aracterized by ab n orm alities in p h ysiology or in b eh avior associated with


sleep. Th ey in clu de bruxism (tooth grin d in g) an d sleepwalking, as well as sleep terror, REM
sleep behavior, an d nightm are disorders.

C. Th ese an d oth er sleep disorders are described in Table 10.3.

D. In som n ia, breath in g-related sleep disorder, an d n arcolep sy are described below.

III. INSOMNIA
A. In som n ia is difficulty falling asleep or staying asleep th at lasts for at least 1 month an d leads to
sleep in ess d u rin g th e d ay or cau ses p rob lem s u l illin g social or occu p ation al obligation s. It
is p resen t in at least 30% o th e p op u lation .
Chapter 10 Normal Sleep and Sleep Disorders 101

t a b l e 10.3 Other Sleep Disorders and Their Characteristics

Sleep Disorder Characteristics

Sleep terror disorder Repetitive experiences of fright in which a person screams in fear during sleep
(usually normal in children)
The person cannot be awakened
The person has no memory of having a dream
Occurs during delta sleep
Onset in adolescence may indicate temporal lobe epilepsy
Nightmare disorder Repetitive, frightening dreams that cause nighttime awakenings
The person usually can recall the nightmare
Occurs during REM sleep
HELP OTHERS SO THAT GOD WILL HELP YOU.

Sleepwalking disorder Repetitive walking around during sleep


No memory of the episode on awakening
Begins in childhood (usually 4–8 y of age)
Occurs during delta sleep
Circadian rhythm sleep disorder Inability to sleep at appropriate times
Delayed sleep phase type involves falling asleep and waking later than wanted
J et lag type lasts 2–7 d after a change in time zones
Shift work type (e.g., in physician training) can result in work errors
Nocturnal myoclonus Repetitive, abrupt muscular contractions in the legs from toes to hips
Causes nighttime awakenings
Treat with benzodiazepine, quinine, or antiparkinsonian, i.e., dopaminergic agent
(e.g., levodopa, ropinirole [Requip])
Restless legs syndrome Uncomfortable sensation in the legs necessitating frequent motion
Repetitive limb jerking during sleep
Causes difficulty falling asleep and nighttime awakenings
More common with aging, Parkinson’s disease, pregnancy, and kidney disease
Treat with antiparkinsonian agent, iron supplements, or magnesium supplements
Kleine-Levin syndrome and Recurrent periods of excessive sleepiness occurring almost daily for weeks to months
menstrual-associated Sleepiness is not relieved by daytime naps
syndrome (symptoms only in the Often accompanied by hyperphagia (overeating)
premenstruum) Kleine-Levin syndrome is more common in adolescent males
Sleep drunkenness Difficulty awakening fully after adequate sleep
Rare, must be differentiated from substance use or other sleep disorder
Associated with genetic factors
Bruxism Tooth grinding during sleep (stage 2)
Can lead to tooth damage and jaw pain
Treat with dental appliance worn at night or corrective orthodontia
REM sleep behavior disorder REM sleep without the typical skeletal muscle paralysis
While dreaming, patients can injure themselves or their sleeping partners
RISE USMLE NEPAL

Associated with Parkinson’s disease and Lewy body disease


Treat with antiparkinsonian agent, REM suppressor (e.g., benzodiazepin), or
anticonvulsant (e.g., carbamazepine)

B. Psychological causes o in som n ia in clu de th e m ood an d an xiety disorders.


1. Major depressive disorder (see Ch ap ter 12).
a. Ch aracteristics o th e sleep pattern in dep ression (Table 10.2):
(1) Lon g sleep laten cy.
(2) Rep eated n igh ttim e awaken in gs lead in g to p oor sleep e icien cy.
(3) Waking too early in th e m orn in g (term in al in som n ia) is th e m ost com m on sleep
ch aracteristic o dep ressed p atien ts.
b. Ch aracteristics o th e sleep stages in dep ression (Table 10.2):
(1) Short REM latency (ap p earan ce o REM with in abou t 45 m in u tes o allin g asleep ).
(2) Increased REM early in the sleep cycle an d decreased REM later in th e sleep cycle
(e.g., in th e early m orn in g h ou rs).
(3) Lon g irst REM p eriod an d increased total REM.
(4) Reduced delta sleep.
102 BRS Behavioral Science

2. Bipolar disorder. Manic or hypomanic p atien ts h ave trou ble allin g asleep an d sleep ewer
h ou rs.
3. Anxious p atien ts o ten h ave trou ble allin g asleep.

C. Physical causes o in som n ia:


1. Use of central nervous system (CNS) stimulants (e.g., ca ein e) is th e m ost com m on cau se o
in som n ia.
2. Withdrawal of agents with sedating action (e.g., alcoh ol, b en zodiazep in es) can resu lt in
wake u ln ess.
3. Medical conditions cau sin g p ain also resu lt in in som n ia, as d o en d ocrin e an d m etabolic
disord ers.
HELP OTHERS SO THAT GOD WILL HELP YOU.

IV. BREATHING-RELATED SLEEP DISORDER (SLEEP APNEA)


A. Patien ts with sleep ap n ea stop breathing or b rie in tervals. Low oxygen or h igh carbon diox-
ide level in th e blood repeatedly awakens the patient du rin g th e n igh t, resu ltin g in daytime
sleepiness an d respiratory acidosis (blood p H < 7.35).
1. In p atien ts with central sleep apnea (m ore com m on in th e eld erly), little or n o resp iratory
e ort occu rs, resu ltin g in less air reach in g th e lu n gs.
2. In p atien ts with obstructive sleep apnea , resp iratory e ort occu rs, bu t an airway obstru c-
tion p reven ts air rom reach in g th e lu n gs. Obstru ctive sleep ap n ea occu rs m ost o ten in
p eop le 40–60 years o age an d is m ore com m on in m en (8:1 m ale-to- em ale ratio) an d in
th e obese. Patien ts o ten snore .
3. Pickwickian syndrome is a related con dition in wh ich airway obstru ction resu lts in day-
tim e sleep in ess.

B. Sleep ap n ea occu rs in 1%–10% of the population an d is related to dep ression , m orn in g h ead-
ach es, an d pulmonary hypertension. It m ay also resu lt in sudden death du rin g sleep in th e
elderly an d in in an ts.

V. NARCOLEPSY
A. Patien ts with n arcolep sy h ave sleep attacks (i.e., all asleep su dd en ly d u rin g th e d ay) d esp ite
RISE USMLE NEPAL

h avin g a n orm al am ou n t o sleep at n igh t. Wh ile typ ical in am ou n t, th eir n igh ttim e sleep is
ch aracterized by decreased sleep latency, very short REM latency (<10 minutes), less total REM,
an d interrupted REM (sleep ragm en tation ).

B. Decreased REM sleep at n igh t leads to th e in tru sion o ch aracteristics o REM sleep (e.g.,
p aralysis, n igh tm ares) wh ile th e p atien t is awake resu ltin g in :
1. Hypnagogic or hypnopompic hallucinations. Th ese are stran ge p ercep tu al exp erien ces th at
occu r ju st as th e p atien t alls asleep or wakes u p, resp ectively, an d occu r in 20%–40% o
p atien ts.
2. Cataplexy. Th is is a su dd en p h ysical collap se cau sed by th e loss o all m u scle ton e a ter a
stron g em otion al stim u lu s (e.g., lau gh ter, ear) an d occu rs in 30%–70% o p atien ts.
3. Sleep paralysis. Th is is th e in ab ility to m ove th e body or a ew secon ds a ter wakin g.

C. Narcolep sy is u n com m on .
1. It occu rs m ost requ en tly in adolescents and young adults .
2. Th ere m ay be a genetic component.
3. Daytime naps allow th e p atien t to m ake u p som e lost REM sleep an d, as su ch , leave th e
p atien t eelin g re resh ed.
Chapter 10 Normal Sleep and Sleep Disorders 103

t a b l e 10.4 Management of the Major Sleep Disorders

Disorder Management (in Order of Highest to Lowest Utility)

Insomnia Avoidance of caffeine, especially before bedtime


Development of a series of behaviors associated with bedtime (i.e., “a sleep ritual”
“sleep hygiene”)
Maintaining a fixed sleeping and waking schedule (i.e., “sleep hygiene”)
Daily exercise (but not just before sleep)
Relaxation techniques
Psychoactive agents (i.e., limited use of sleep agents to establish an effective sleep pattern
and antidepressants or antipsychotics, if appropriate) (see Table 16.3)
Breathing-related sleep Weight loss (if overweight)
HELP OTHERS SO THAT GOD WILL HELP YOU.

disorder (obstructive Continuous positive airway pressure (CPAP) (a device with a mask applied to the face at
sleep apnea) night to gently move air into the lungs)
Breathing stimulant, e.g., medroxyprogesterone acetate, protriptyline (Vivactil), fluoxetine
(Prozac)
Surgery to enlarge the airway, e.g., uvulopalatoplasty
Tracheostomy (as a last resort)
Narcolepsy Stimulant agents (e.g., modafinil [Provigil]; methylphenidate [Ritalin], if cataplexy is present,
sodium oxybate [Xyrem] or an antidepressant may be added)
Scheduled daytime naps

VI. MANAGEMENT OF SLEEP DISORDERS


Th e m an agem en t o in som n ia, breath in g-related sleep disorder, an d n arcolep sy are described
in Table 10.4.
RISE USMLE NEPAL
Review Test

Directions: Each o th e n u m b ered item s or in com p lete statem en ts in th is section is ollowed by


an swers or by com p letion s o th e statem en t. Select th e one lettered an swer or com p letion th at
is best in each case.
HELP OTHERS SO THAT GOD WILL HELP YOU.

1. Th e p aren ts o a 5-year-old boy rep ort 4. Du rin g a sleep stu dy, a em ale p atien t’s
th at th e ch ild o ten scream s du rin g th e EEG sh ows p rim arily delta waves. Wh ich o
n igh t. Th ey are p articu larly con cern ed th e ollowin g is m ost likely to ch aracterize
becau se du rin g th ese d istu rb an ces, th e th is p atien t at th is tim e?
ch ild sits u p, op en s h is eyes, an d “looks (A) Clitoral erection
righ t th rou gh th em ,” an d th ey are u n able (B) Paralysis o skeletal m u scles
to awaken h im . Th e ch ild h as n o m em ory (C) Sleepwalkin g (som n am b u lism )
o th ese exp erien ces in th e m orn in g. (D) Nigh tm ares
Ph ysical exam in ation is u n rem arkable an d (E) In creased brain oxygen u se
th e ch ild is doin g well in kin dergarten .
Durin g th ese distu rban ces, th e ch ild’s 5. An 85-year-old p atien t rep orts th at h e
electroen cep h alogram is m ost likely to be sleep s p oorly. Sleep in th is p atien t is m ost
p rim arily ch aracterized by likely to be ch aracterized by in creased
(A) sawtooth waves (A) sleep e icien cy
(B) th eta waves (B) REM sleep
(C) K com p lexes (C) n igh ttim e awaken in gs
(D) delta waves (D) stage 3 sleep
(E) alp h a waves (E) stage 4 sleep

2. Du rin g a sleep stu dy, a p h ysician 6. A wom an rep orts th at m ost n igh ts
discovers th at a p atien t sh ows too little du rin g th e last year, sh e h as lain awake in
REM sleep du rin g th e n igh t. Th eoretically, bed or m ore th an 2 h ou rs b e ore sh e alls
to in crease REM sleep, th e p h ysician sh ou ld asleep. A ter th ese n igh ts, sh e is tired an d
give th e p atien t a m edication aim ed at orget u l an d m akes m istakes at work. O
in creasin g circu latin g levels o th e ollowin g, th e m ost e ective lon g-term
(A) seroton in treatm en t or th is wom an is
RISE USMLE NEPAL

(B) n orep in ep h rin e (A) con tin u ou s p ositive airway p ressu re


(C) acetylch olin e (CPAP)
(D) dop am in e (B) an an tip sych otic agen t
(E) h istam in e (C) a sedative agen t
(D) a stim u lan t agen t
3. Du rin g a sleep stu dy, a m ale p atien t’s (E) develop m en t o a “sleep ritu al”
electroen cep h alogram (EEG) sh ows
p rim arily sawtooth waves. Wh ich o th e Questions 7 and 8
ollowin g is m ost likely to ch aracterize th is
p atien t at th is tim e? A 22-year-old m edical stu den t wh o goes to
(A) Pen ile erection sleep at 11 pm an d wakes at 7 a m alls asleep in
(B) Movem en t o skeletal m u scles laboratory every day. He tells the doctor th at
(C) Decreased blood p ressu re h e sees stran ge im ages as h e is allin g asleep
(D) Decreased brain oxygen u se an d som etim es ju st as h e wakes u p. He h as
(E) Decreased p u lse h ad a ew m in or car acciden ts th at occu rred
becau se h e ell asleep wh ile drivin g.

104
Chapter 10 Normal Sleep and Sleep Disorders 105

7. O th e ollowin g th e b est f rst step in 13. Th eta waves are m ost ch aracteristic o
m an agem en t o th is stu den t’s p rob lem is wh at sleep stage?
(A) con tin u ou s p ositive airway p ressu re (A) Stage 1
(CPAP) (B) Stage 2
(B) an an tip sych otic agen t (C) Stages 3 an d 4
(C) a sed ative agen t (D) REM sleep
(D) a stim u lan t agen t
(E) develop m en t o a “sleep ritu al” 14. Wh at sleep stage takes u p th e largest
p ercen tage o sleep tim e in you n g adu lts?
8. Wh ich o th e ollowin g is th is stu den t (A) Stage 1
m ost likely to exp erien ce? (B) Stage 2
HELP OTHERS SO THAT GOD WILL HELP YOU.

(A) Lon g REM laten cy (C) Stages 3 an d 4


(B) Au ditory h allu cin ation s (D) REM sleep
(C) Tactile h allu cin ation s
(D) Delu sion s 15. Bed-wettin g is ch aracteristic o wh at
(E) Catap lexy sleep stage?
(A) Stage 1
Questions 9 and 10 (B) Stage 2
(C) Stages 3 an d 4
A p atien t rep orts th at h e is sleepy all d ay (D) REM sleep
desp ite h avin g 8 h ou rs o sleep each n igh t.
His wi e rep orts th at h is lou d sn orin g keep s 16. A 22-year-old stu den t in th e m iddle o
h er awake. f n als week tells h er d octor th at or th e last
2 weeks, sh e h as b een stu d yin g late in to th e
9. O th e ollowin g, th e b est f rst step in th e n ight an d h as started to h ave trou ble allin g
m an agem en t o th is p atien t is asleep. Wh at is th e doctor’s m ost ap p rop riate
(A) con tin u ou s p ositive airway p ressu re recom m en dation ?
(CPAP) (A) Exercise b e ore b ed tim e
(B) an an tip sych otic agen t (B) A large m eal be ore bedtim e
(C) a sed ative agen t (C) A glass o m ilk b e ore b ed tim e
(D) a stim u lan t agen t (D) A ixed wake-u p an d bed tim e
(E) develop m en t o a “sleep ritu al” sch ed u le
(E) A sh ort-actin g b en zodiazep in e at
10. O th e ollowin g, th is p atien t is m ost bedtim e
likely to b e
(A) dep ressed 17. A 45-year-old em ale p atien t rep orts
RISE USMLE NEPAL

(B) aged 25 years th at over th e last 3 m on th s, sh e h as lost


(C) overweigh t h er ap p etite an d in terest in h er u su al
(D) u sin g a stim u lan t agen t activities an d o ten eels th at li e is n ot
(E) with d rawin g rom a sed ative agen t worth livin g. Com p ared with typ ical
sleep, in th is p atien t, th e p ercen tage
11. Sawtooth waves are m ost ch aracteristic o REM sleep, p ercen tage o delta sleep,
o wh at sleep stage? an d sleep laten cy, resp ectively, are m ost
(A) Stage 1 likely to
(B) Stage 2 (A) in crease, decrease, decrease
(C) Stages 3 an d 4 (B) in crease, decrease, in crease
(D) REM sleep (C) decrease, stay th e sam e, in crease
(D) decrease, decrease, in crease
12. Sleep spin dles, K com p lexes, an d bruxism (E) in crease, in crease, in crease
are m ost characteristic o what sleep stage?
(A) Stage 1
(B) Stage 2
(C) Stages 3 an d 4
(D) REM sleep
106 BRS Behavioral Science

18. In a sleep laboratory, a wom an sh ows 21. A 32-year-old m an h as a 9 to 5 job in a


10% o sleep tim e in stage 1 sleep, 75% o law o f ce. Su n day n igh t th rou gh Th u rsday
sleep tim e in stage 2 sleep, 15% o sleep n igh t th e m an goes to b ed at 10 p m b u t is
tim e in REM sleep, n o delta sleep, an d six u n able to all asleep u n til abou t 2 a m . His
n igh ttim e awaken in gs. Th is sleep p attern alarm waken s h im at 6 a m an d h e eels tired
in d icates th at th is wom an all day. On Frid ay an d Satu rd ay n igh ts,
(A) h as n arcolep sy th e m an goes to bed at 2 a m , alls asleep
(B) h as a dep ressive illn ess qu ickly, sleep s u n til 10 a m , an d wakes eelin g
(C) is elderly re resh ed. Wh ich o th e ollowin g sleep
(D) h as an an xiety disorder disord ers b est m atch es th is p ictu re?
(E) h as n octu rn al m yoclon u s (A) Klein e-Levin syn drom e
HELP OTHERS SO THAT GOD WILL HELP YOU.

(B) Nigh tm are disorder


19. A 5-year-old ch ild o ten wakes d u rin g (C) Sleep terror disord er
th e n igh t, cryin g an d ear u l. Wh en h is (D) Sleep dru n ken n ess
p aren ts com e to h im , h e relates details o (E) Circadian rh yth m sleep disorder
dream s in volvin g righ ten in g creatu res an d (F) Noctu rn al m yoclon u s
situation s. Wh ich o th e ollowin g sleep (G) Restless legs syn d rom e
disord ers b est m atch es th is p ictu re? (H) Bru xism
(A) Klein e-Levin syn drom e
(B) Nigh tm are disorder 22. A 70-year-old m an can n ot all asleep
(C) Sleep terror disord er b ecau se o crawlin g eelin gs an d ach in g
(D) Sleep dru n ken n ess in his calves an d th igh s. He can su p p ress
(E) Circadian rh yth m sleep disorder th e u rge to m ove h is legs or a sh ort p eriod
(F) Noctu rn al m yoclon u s b u t th en m u st m ove th em . Wh ich o th e
(G) Restless legs syn d rom e ollowin g best f ts th is clin ical p ictu re?
(H) Bru xism (A) Klein e-Levin syn drom e
(B) Nigh tm are disorder
20. Th e m oth er o a 13-year-old b oy rep orts (C) Sleep terror disord er
th at h e h as “b ou ts” o overeatin g an d o (D) Sleep dru n ken n ess
oversleep in g, each lastin g a ew days to a ew (E) Circadian rh yth m sleep disorder
weeks. Wh ich o th e ollowin g sleep disorders (F) Noctu rn al m yoclon u s
best m atch es th is p ictu re? (G) Restless legs syn d rom e
(A) Klein e-Levin syn drom e (H) Bru xism
(B) Nigh tm are disorder
(C) Sleep terror disord er 23. Th e sleep o a p atien t wh o begin s takin g
(D) Sleep dru n ken n ess a m oderate dose o diazep am (Valiu m ) daily
(E) Circadian rh yth m sleep disorder is m ost likely to b e ch aracterized by wh ich o
RISE USMLE NEPAL

(F) Noctu rn al m yoclon u s th e ollowin g ch an ges?


(G) Restless legs syn d rom e (A) In creased stage 1 an d in creased stage 2
(H) Bru xism (B) In creased stage 1 an d decreased delta
(C) Decreased REM an d decreased d elta
(D) Decreased REM an d in creased d elta
(E) In creased REM an d decreased delta
Chapter 10 Normal Sleep and Sleep Disorders 107

24. A 21-year-old stu d en t wh o is p art o a 27. A 22-year-old stu den t rep orts th at h e
stu dy o circadian rh yth m s, sleep s in a cave alls asleep rep eatedly d u rin g th e d aytim e.
or 1 m on th with n o access to clocks or He also rep orts th at h e can n ot m ove or a
watch es. At th e en d o th e m on th , th e len gth ew m in u tes wh en h e f rst wakes u p in th e
o h er circad ian cycle is likely to be closest to m orn in g. I th e p atien t h as a sleep stu dy,
(A) 21 h ou rs wh ich o th e ollowin g sleep ch an ges is m ost
(B) 22 h ou rs likely to b e seen ?
(C) 23 h ou rs (A) In creased REM laten cy
(D) 24 h ou rs (B) Decreased REM laten cy
(E) 25 h ou rs (C) In creased Stage 2
(D) Decreased Stage 2
HELP OTHERS SO THAT GOD WILL HELP YOU.

25. Th e wi e o a 62-year-old m an tells th e (E) In creased sleep laten cy


d octor th at or th e p ast year, h er h u sb an d
h as p u n ch ed an d kicked h er rep eated ly 28. A ter bein g asleep or 6 h ou rs d u rin g a
d u rin g th e n igh t. Wh en sh e wakes h im sleep stu dy, a h ealth y 28-year-old wom an
d u rin g th ese ep isod es, h er h u sban d relates wakes u p to u rin ate. Wh ich o th e ollowin g
th at h e h as been h avin g a d ream in wh ich ch an ges is m ost likely to b e seen in h er EKG
h e is tryin g to escap e rom or f gh tin g with a at this tim e?
righ ten in g attacker. Over th e n ext ew years, (A) In creased h eart rate
th is m an is at in creased risk to develop (B) Decreased h eart rate
(A) Klein e-Levin syn drom e (C) In creased Q-T in terval
(B) sleep terror d isorder (D) Decreased Q-T in terval
(C) n octu rn al m yoclon u s (E) Atrial ib rillation
(D) Alzh eim er’s d isease
(E) Lewy body disease

26. A 33-year-old m an with m yasth en ia


gravis is takin g p h ysostigm in e or sym p tom
relie . Th e e ect o th is agen t on sleep
arch itectu re in th is p atien t is m ost likely to
b e in creased
(A) stage 1 sleep
(B) stage 2 sleep
(C) stage 3 sleep
(D) stage 4 sleep
(E) REM sleep
RISE USMLE NEPAL
An swers an d Exp lan ation s

Typical Board Question


The answer is A. Th is overweigh t m id d le-aged m ale p atien t is likely to h ave sleep ap n ea. Becau se
h e is sin gle an d sleep s alon e, h e does n ot realize th at h is n igh ttim e sleep is ch aracterized by
dep ressed b reath in g an d is o ten d isru p ted. Like oth er p atien ts with p u lm on ary d isord ers lead-
HELP OTHERS SO THAT GOD WILL HELP YOU.

in g to dep ressed breath in g, p atien ts with sleep ap n ea typ ically sh ow ch ron ic resp iratory acidosis
(in creased p artial p ressu re o carb on dioxid e [PaCO 2]). Resp iratory alkalosis (decreased PaCO 2)
resu lts rom h yp erven tilation as a resu lt o an xiety, h igh ever, or stim u lan t u se. Metabolic alka-
losis typ ically resu lts rom excessive vom itin g an d resu ltin g h yp okalem ia (see also an swers to
Qu estion s 9 an d 10).

1. The answer is D. Th is child dem on strates sleep terror disorder, wh ich is ch aracterized by
rep etitive occu rren ces o scream in g du rin g th e n igh t an d th e in ability to be awaken ed or to
rem em ber th ose exp erien ces in th e m orn in g. Sleep terrors typ ically occu r durin g delta sleep.
I th e ch ild were h avin g n igh tm ares, wh ich occu r in REM sleep, th e ch ild typ ically wou ld
awaken an d relate th e n atu re o h is righ ten in g dream s (see also an swer to Qu estion 19).
2. The answer is C. Acetylch olin e (Ach ) is in volved in both in creasin g REM sleep an d
in creasin g sleep e icien cy. In creased levels o dop am in e decrease sleep e icien cy.
In creased levels o n orep in ep h rin e decrease both sleep e icien cy an d REM sleep wh ile
in creased levels o seroton in in crease b oth sleep e icien cy an d delta (slow-wave) sleep.
3. The answer is A. Sawtooth waves ch aracterize REM sleep, wh ich is also associated with
p en ile erection ; dream in g; in creased p u lse, resp iration , an d blood p ressu re; an d p aralysis o
skeletal m u scles.
4. The answer is C. Delta waves ch aracterize sleep stages 3 an d 4 (slow-wave sleep ), wh ich is
also associated with som n am b u lism , n igh t terrors, ep isodic body m ovem en ts, an d en u resis.
Delta sleep is th e deep est, m ost relaxed stage o sleep. Clitoral erection , p aralysis o skeletal
m u scles, n igh tm ares, an d in creased b rain oxygen u se occu r d u rin g REM sleep.
5. The answer is C. Sleep in th e elderly is ch aracterized by in creased n igh ttim e awaken in gs,
decreased REM sleep, d ecreased d elta sleep, an d decreased sleep e icien cy.
RISE USMLE NEPAL

6. The answer is E. Th e m ost e ective lon g-term m an agem en t or th is wom an with in som n ia
is th e develop m en t o a series o beh aviors associated with bedtim e (i.e., a “sleep ritu al”).
By th e p rocess o classical con dition in g (see Ch ap ter 7), th e sleep ritu al th en becom es
associated with goin g to sleep. Sleep ritu als can in clu de th in gs like takin g a warm b ath ,
p u llin g d own th e b lin d s, an d listen in g to sooth in g m u sic. Con tin u ou s p ositive airway
p ressu re is u sed to treat sleep ap n ea; stim u lan t agen ts are u sed to treat n arcolep sy; an d
an tip sych otics are u sed to treat p sych otic sym p tom s. Sedative agen ts h ave a h igh p oten tial
or m isu se an d, becau se th ey ten d to redu ce REM an d delta sleep, th eir u se m ay resu lt in
sleep o p oorer qu ality.
7. The answer is D. 8. The answer is E. Th is m edical stu den t wh o alls asleep in laboratory
every day desp ite a n orm al am ou n t o sleep at n igh t p robably h as n arcolep sy. O th e listed
ch oices, th e m ost e ective m an agem en t or n arcolep sy is th e adm in istration o stim u lan t
agen ts su ch as m od a in il. Sedative agen ts are n ot u se u l or n arcolep sy. In n arcolep sy,
sh ort REM laten cy, sleep p aralysis, an d catap lexy occu r. Th e stu d en t’s stran ge p ercep tu al
exp erien ces as h e is allin g asleep an d wakin g u p are h yp n agogic an d h yp n op om p ic
h allu cin ation s, resp ectively.

108
Chapter 10 Normal Sleep and Sleep Disorders 109

9. The answer is A. 10. The answer is C. Th is m an wh o sn ores an d rep orts th at h e is sleepy all
d ay desp ite h avin g 8 h ou rs o sleep each n igh t p robably h as obstru ctive sleep ap n ea. O
th e listed ch oices, th e b est irst step in m an agem en t o th is p atien t is con tin u ou s p ositive
airway p ressu re (CPAP). Sin ce ob esity is associated with obstru ctive sleep ap n ea, oth er
su ggestion s or th is p atien t wou ld in clu de weigh t loss. Use o stim u lan ts an d with drawal
rom sed atives are associated with wake u ln ess rath er th an th e daytim e sleep in ess
seen h ere. Also, m ost sleep ap n ea p atien ts are m iddle aged (age 40–60 years). Alth ou gh
d ep ression an d an xiety are associated with sleep p roblem s, th is m an’s sn orin g in dicates
th at h is sleep p roblem is m ore likely to h ave a p h ysical basis.
11. The answer is D. Sawtooth waves are p rim arily seen in REM sleep.
HELP OTHERS SO THAT GOD WILL HELP YOU.

12. The answer is B. Sleep sp in d les, K com p lexes, an d b ru xism are p rim arily seen in stage 2
sleep.
13. The answer is A. Th eta waves are p rim arily seen in stage 1 sleep.
14. The answer is B. In you n g ad u lts, 45% o total sleep tim e is sp en t in stage 2 sleep. Five
p ercen t is sp en t in stage 1, 25% in REM, an d 25% in delta sleep.
15. The answer is C. Bed -wettin g occu rs p rim arily in stages 3 an d 4 (d elta) sleep.
16. The answer is D. Th e m ost ap p rop riate irst step in th e m an agem en t o th is 22-year-old
stu d en t wh o is h avin g tem p orary p roblem s with sleep du rin g in als week is to recom m en d
a ixed wake-u p an d bedtim e sch ed u le. Ben zodiazep in es are n ot ap p rop riate b ecau se o
th eir h igh p oten tial or m isu se an d p ossibility o cau sin g daytim e sedation in th is stu den t
du rin g exam in ation s. Th ese agen ts also decrease sleep qu ality by redu cin g REM an d
delta sleep. Exercise sh ou ld b e d on e early in th e day; i d on e b e ore bed tim e, it can b e
stim u latin g an d cau se wake u ln ess. A large m eal b e ore b ed tim e is m ore likely to in ter ere
with sleep th an to h elp sleep. Wh ile m an y p eop le believe th at m ilk h elp s in du ce sleep, th is
e ect h as n ever been sh own em p irically.
17. The answer is B. Th is wom an’s sym p tom s in dicate th at sh e is likely to be exp erien cin g a
m ajor dep ressive ep isod e (see Ch ap ter 12). Sleep in m ajor dep ression is associated with
in creased REM sleep, redu ced delta sleep, an d in creased sleep laten cy.
18. The answer is C. Th is sleep p attern in dicates th at th is wom an is eld erly. Sleep in eld erly
p atien ts is ch aracterized by in creased stage 1 an d stage 2 sleep, in creased n igh ttim e
awaken in gs, decreased REM sleep, an d m u ch redu ced or absen t d elta sleep.
19. The answer is B. Th is ch ild is exp erien cin g n igh tm are d isord er, wh ich occu rs d u rin g
REM sleep. In con trast to th e ch ild with sleep terror d isord er (see also an swer to
RISE USMLE NEPAL

Qu estion 1), th is ch ild wakes u p an d can relate th e n atu re o h is righ ten in g d ream s.
Klein e-Levin syn d rom e is u su ally seen in ad olescen ts an d in volves recu rren t p eriod s
o h yp ersom n ia an d h yp erp h agia, each lastin g d ays to weeks. In sleep d ru n ken n ess, a
p atien t can n ot com e u lly awake a ter sleep, an d in circad ian rh yth m sleep d isord er, th e
in d ivid u al sleep s an d wakes at in ap p rop riate tim es. Noctu rn al m yoclon u s (m u scu lar
con traction s in volvin g th e legs) an d restless legs syn d rom e (u n com ortab le sen sation
in th e legs) occu r m ore com m on ly in m id d le-aged an d eld erly p eop le. Bru xism is tooth
grin d in g d u rin g sleep.
20. The answer is A. Th e act th at th is p atien t is an ad olescen t, as well as th e recu rren t p eriod s
o h yp ersom n ia an d h yp erp h agia each lastin g or weeks to m on th s, in d icate th at th is
p atien t h as Klein e-Levin syn drom e (an d see also an swer to Qu estion 19).
21. The answer is E. Circad ian rh yth m sleep disorder in volves th e in ability to sleep at
ap p rop riate tim es. Th is m an sh ows th e d elayed sleep p h ase typ e o th is disorder, wh ich
is ch aracterized by allin g asleep an d wakin g later th an wan ted . Wh en th e m an is able to
ollow h is p re erred sleep sch edu le (e.g., on weeken d s), h e sleep s well an d wakes re resh ed
(see also an swer to Qu estion 19).
110 BRS Behavioral Science

22. The answer is G. In restless legs syn d rom e, th ere are crawlin g, ach in g eelin gs in th e legs
m akin g it n ecessary or th e p atien t to m ove th em an d cau sin g d i icu lty in allin g asleep
(see also an swer to Qu estion 19).
23. The answer is C. Decreased REM sleep an d decreased delta sleep ch aracterize th e sleep o
p atien ts su ch as th is on e, wh o are takin g sedatives su ch as diazep am (a ben zodiazep in e),
barb itu rates, or alcoh ol.
24. The answer is E. At th e en d o th e m on th , th e len gth o th is stu den t’s circadian cycle in th e
absen ce o cu es rom th e ou tside world is likely to be close to 25 h ou rs.
25. The answer is E. Dream in g typ ically occu rs du rin g REM sleep. Becau se typ ically th ere
is m u scle aton ia du rin g REM sleep, th is m an wh o is m ovin g wh ile dream in g is sh owin g
HELP OTHERS SO THAT GOD WILL HELP YOU.

sign s o REM sleep beh avior disorder. Th is disorder is associated with an in creased risk or
Parkin son’s disease an d Lewy body disease. Klein e-Levin syn drom e, sleep terror disorder,
n octu rn al m yoclon u s, an d Alzh eim er’s disease are n ot sp eci ically associated with REM
sleep b eh avior d isord er (see also an swer to Qu estion 19).
26. The answer is E. Ph ysostigm in e is a ch olin om im etic agen t u sed to treat m yasth en ia gravis.
Th e in crease in acetylch olin e resu ltin g rom treatm en t with th is agen t is m ost likely to
resu lt in in creased REM sleep in th is p atien t.
27. The answer is B. Th e sleep ch an ge m ost likely to be seen in th is stu d en t wh o is sh owin g
sym p tom s o n arcolep sy, th at is, excessive d aytim e sleep in ess an d sleep p aralysis, is
decreased REM laten cy. Alth ou gh in n arcolep sy th ere are n o sp eci ic ch an ges in Stage 2
sleep, sleep laten cy (th e tim e it takes to all asleep ) is typ ically d ecreased .
28. The answer is A. Th e m ost likely ch an ge seen in th e EKG o th is h ealth y you n g wom an is
in creased h eart rate. REM sleep, wh ich is ch aracterized by in creased h eart rate, occu rs
p rim arily in th e early m orn in g h ou rs (6 h ou rs a ter allin g asleep ) an d is th e stage o sleep
seen ju st b e ore wakin g (wakin g to u rin ate in th is qu estion ). Q-T in terval an d atrial rh yth m
ch an ges are n ot associated with wakin g d u rin g th e n igh t in h ealth y you n g adu lts.
RISE USMLE NEPAL
Sch izop h ren ia Sp ectru m
c ha pte r
11 an d Oth er Psych otic
Disorders
HELP OTHERS SO THAT GOD WILL HELP YOU.

Typical Board Question


A physician in ds th at 35-year-old hom eless m ale p atien t with schizop hren ia h as hyperkerato-
sis, in lam ed gum s, an d petechiae. Wh at is the m ost likely exp lan ation or th is clin ical picture?
(A) Th rom b ocytop en ia rom an tip sych otic m edication
(B) Occu lt in ection
(C) Low wh ite blood cell cou n t
(D) Vitam in de icien cy
(E) Medication -in d u ced vascu litis
(See “An sw ers an d Explan ation s” at th e en d of th e ch apter.)

I. SCHIZOPHRENIA
A. Overview
1. Sch izop h ren ia is a chronic, debilitating m en tal d isord er ch aracterized by p eriods o loss o
tou ch with reality (p sych osis); p ersisten t distu rban ces o th ou gh t, beh avior, ap p earan ce,
an d sp eech ; ab n orm al a ect; an d social with drawal.
2. Peak age of onset o sch izop h ren ia is 15–25 years for men and 25–35 years for women.
3. Sch izop h ren ia occu rs equally in men and women, all cultures, and all ethnic groups
RISE USMLE NEPAL

stu d ied .

B. Symptoms of schizophrenia can b e classi ied as positive or negative .


1. Positive symptoms are th in gs additional to expected behavior an d in clu d e delu sion s, h al-
lu cin ation s, agitation , an d talkativen ess.
2. Negative symptoms are th in gs missing from expected behavior an d in clu de lack o m otiva-
tion , social with d rawal, latten ed a ect, cogn itive distu rban ces, p oor groom in g, an d p oor
(i.e., im p overish ed ) sp eech con ten t.
3. Th ese sym p tom classi ication s can b e u se u l in p redictin g th e e ects o an tip sych otic
m ed ication (see Ch ap ter 16).
a. Positive symptoms resp on d well to m ost traditional an d atypical antipsychotic agents .
b. Negative symptoms resp on d b etter to atypical th an to tradition al an tip sych otics.
4. Patien ts with p red om in an tly n egative sym p tom s h ave m ore neuroanatomic (see below)
an d metabolic abnormalities (e.g., decreased cerebral m etabolism o glu cose) th an th ose
with p red om in an tly p ositive sym p tom s.

111
112 BRS Behavioral Science

t a b l e 11.1 Symptoms of Schizophrenia: Disorders of Perception, Thought Content, Thought


Process, and Form of Thought
Disorder of Symptom Definition Example

Perception Illusions Misperception of real external Interpreting the appearance of a


stimuli coat in a dark closet as a man
Hallucinations False sensory perception Hearing voices when alone in a room
Thought content Delusions False belief not shared by others The idea of being followed by the FBI
Ideas of reference False belief of being referred to The feeling of being discussed by
by others someone on television
Thought processes Impaired abstraction Problems discerning the When asked what brought her to
ability essential qualities of objects the emergency room, the patient
HELP OTHERS SO THAT GOD WILL HELP YOU.

or relationships says, “An ambulance”


Magical thinking Belief that thoughts affect the Knocking on wood to prevent
course of events something bad from happening
Form of thought Circumstantiality Inclusion of too much detail When asked about her health, the
patient explains everything that
she did since getting up that day
before getting to the subject of
her health
Loose associations Shift of ideas from one subject The patient begins to answer a
to another in an unrelated question about her health and then
way shifts to a statement about baseball
Neologisms Inventing new words The patient refers to her doctor as a
“medocrat”
Perseveration Repeating words or phrases The patient says, “I’m evil, I’m evil,
I’m evil”
Tangentiality Getting further away from the The patient begins to answer a
point as speaking continues question about her health and
ends up talking about her sister’s
abortion; she never gets back to
the subject of her health

C. Course. Sch izop h ren ia h as th ree phases: p rodrom al, active (i.e., p sych otic), an d residu al.
1. Prodromal sign s an d sym p tom s occu r p rior to th e irst p sych otic ep isode an d in clu de
avoidan ce o social activities; p h ysical com p lain ts; an d n ew in terest in religion , th e occu lt,
or p h ilosop h y.
2. In th e active phase , th e p atien t loses tou ch with reality. Disorders o p ercep tion , th ou gh t
con ten t, th ou gh t p rocesses, an d orm o th ou gh t (Table 11.1) occu r du rin g an acu te p sy-
RISE USMLE NEPAL

ch otic ep isode.
3. In th e residual phase (tim e p eriod b etween p sych otic ep isodes), th e p atien t is in tou ch
with reality b u t d oes n ot beh ave n orm ally.
a. Th is p h ase is ch aracterized by negative symptoms .
b. In th is p h ase, th e p atien t typ ically sh ows in tact m em ory cap acity; is oriented to p erson ,
p lace, an d tim e; an d h as a normal level of consciousness (e.g., is alert).
4. Active p h ase sym p tom s m u st be p resen t or at least 1 month an d active p h ase an d/
or resid u al p h ase sym p tom s m u st b e p resen t or at least 6 months or th e diagn osis o
sch izop h ren ia.

D. Prognosis
1. Sch izop h ren ia u su ally in volves rep eated p sych otic ep isodes an d a chronic, downhill
course over years. Th e illn ess o ten stabilizes in m idli e.
2. Suicide is common in p atien ts with sch izop h ren ia. More th an 50% attem p t su icide (o ten
du rin g p ostp sych otic d ep ression or wh en h avin g h allu cin ation s “com m an din g” th em to
h arm th em selves), an d 10% o th ose d ie in th e attem p t.
3. Th e prognosis is better, an d th e su icide risk is lower i th e p atien t is old er at on set o illn ess,
is m arried, h as social relation sh ip s, is em ale, h as a good em p loym en t h istory, h as m ood
sym p tom s, h as ew n egative sym p tom s, an d h as ew relap ses.
Chapter 11 Schizophrenia Spectrum and Other Psychotic Disorders 113

t a b l e 11.2 The Genetics of Schizophrenia


Group Approximate Occurrence

The general population 1%


Person who has one parent or sibling (or dizygotic twin) with schizophrenia 10%
Person who has two parents with schizophrenia 40%
Monozygotic twin of a person with schizophrenia 50%

E. Etiology. Wh ile th e etiology o sch izop h ren ia is n ot kn own , certain actors h ave been im p li-
cated in its d evelop m en t.
HELP OTHERS SO THAT GOD WILL HELP YOU.

1. Genetic factors
a. Sch izop h ren ia occu rs in 1% of the population. Person s with a close gen etic relation sh ip
to a p atien t with sch izop h ren ia are m ore likely th an th ose with a m ore distan t relation -
sh ip to develop th e disord er (Tab le 11.2).
b. Certain chrom osom al m arkers have been associated with schizophren ia (see Chapter 4).
2. Other factors
a. Th e season of birth is related to th e occu rren ce o sch izop h ren ia. More p eop le with
sch izop h ren ia are born during cold weather months (i.e., Jan u ary to Ap ril in th e n orth ern
h em isp h ere an d Ju ly to Sep tem ber in th e sou th ern h em isp h ere). On e p ossible exp la-
n ation or th is in din g is viral infection of the mother du rin g p regn an cy, sin ce su ch in ec-
tion s occu r season ally.
b. No social or environmental factor causes schizophrenia. However, becau se p atien ts with
sch izop h ren ia ten d to d ri t down th e socioecon om ic scale as a resu lt o th eir social
de icits (th e “downward drift” hypothesis ), th ey are o ten ou n d in lower socioecon om ic
grou p s (e.g., h om eless p eop le).

F. Neural pathology
1. Anatomy
a. Abnormalities of the frontal lobes , as evid en ced by d ecreased u se o glu cose in th e ron -
tal lobes on p ositron em ission tom ograp h y (PET) scan s are seen in th e brain s o p eop le
with sch izop h ren ia.
b. Lateral and third ventricle enlargement, abn orm al cerebral sym m etry, an d ch an ges in
brain den sity also m ay be p resen t.
c. Decreased volume of limbic structures (e.g., am ygd ala, h ip p ocam p u s) is also seen .
2. Neurotransmitter abnormalities (see also Table 4.3)
a. Th e dopamine hypothesis o sch izop h ren ia states th at th e p ositive sym p tom s resu lt
RISE USMLE NEPAL

rom excessive d op am in ergic activity (e.g., an excessive n u m ber o dop am in e recep -


tors, excessive con cen tration o d op am in e, h yp ersen sitivity o recep tors to dop am in e)
in th e lim bic system . As eviden ce or th is h yp oth esis, stim u lan t dru gs th at in crease
dop am in e availab ility (e.g., am p h etam in es an d cocain e) can cau se p sych otic sym p -
tom s (see Ch ap ter 9). Also, laboratory tests m ay sh ow elevated levels of homovanillic
acid (HVA), a m etabolite o dop am in e, in th e body lu ids o p atien ts with sch izop h re-
n ia. Th e n egative sym p tom s o sch izop h ren ia are b elieved to resu lt rom red u ced
d op am in ergic activity in th e ron tal cortex (see Ch ap ter 4).
b. Serotonin hyperactivity is im p licated in sch izop h ren ia becau se h allu cin ogen s th at
in crease seroton in con cen tration s cau se p sych otic sym p tom s, an d becau se som e
e ective an tip sych otics, su ch as clozap in e (see Ch ap ter 16), h ave an ti-seroton ergic-2A
(5-HT2A) activity.
c. Glutamate is im p licated in sch izop h ren ia; N-m eth yl- d -asp artate (NMDA) an tagon ists
(e.g., m em an tin e) are u se u l in treatin g som e o th e n eu rodegen erative sym p tom s
(e.g., loss o cogn itive abilities) in p atien ts with sch izoph ren ia.

G. Severity. Th e Diagnostic and Statistical Manual of Mental Disorders, Fifth Edition (DSM-5) n o
lon ger in cludes su btyp es o sch izop h ren ia bu t rath er distin gu ish es p atien ts by th e
severity o their sym p tom s (Table 11.3).
114 BRS Behavioral Science

t a b l e 11.3 DSM-5 Dimensions of Psychotic Symptom Severity in Schizophrenia (rated


over the past 7 days as 0 = not present; 1 = equivocal; 2 = present but mild;
3 = present and moderate; or 4 = present and severe)
Symptom Characteristics of a Severity Score of 4 (Present and Severe)

Hallucinations Severe pressure to respond to auditory hallucinations (voices) or is very upset by the voices
Delusions Severe pressure to act upon the delusions (false beliefs) or is very upset by the false beliefs
Disorganized speech Speech is almost impossible to follow
Abnormal psychomotor Severe abnormal or bizarre motor behavior or almost constant catatonia (stupor with lack of
behavior coherent speech)
Negative symptoms Severe decrease in facial expressivity, gestures, or self-initiated behavior
HELP OTHERS SO THAT GOD WILL HELP YOU.

H. Differential Diagnosis
1. Medical illnesses th at can cau se p sych otic sym p tom s, an d th us m im ic sch izoph ren ia (i.e.,
p sych otic disorder cau sed by a gen eral m edical con dition ), in clu de n eurologic in ection ,
n eoplasm , traum a, disease (e.g., Hun tin gton’s disease, m ultip le sclerosis), tem poral lobe
ep ilep sy, an d en docrin e disorders (e.g., Cu sh in g’s syn drom e, acute in term itten t p orp hyria).
2. Medications th at can cau se p sych otic sym p tom s in clu de an algesics, an tibiotics, an tich o-
lin ergics, an tih istam in es, an tin eop lastics, cardiac glycosides (e.g., digitalis), an d steroid
h orm on es.
3. Psychiatric illnesses oth er th an sch izop h ren ia th at m ay be associated with p sych otic
sym p tom s in clu de:
a. Oth er p sych otic d isord ers (see b elow).
b. Th e m an ic or dep ressive p h ase o bip olar I disorder, m ajor dep ressive disorder [see
Ch ap ter 12]).
c. Neu rocogn itive disord ers (e.g., deliriu m an d dem en tia [see Ch ap ter 14]).
d. Su bstan ce-related disorders (see Ch ap ter 9).
4. Schizotypal, paranoid, an d borderline personality disorders (see Ch ap ter 14) are n ot ch arac-
terized by ran k p sych otic sym p tom s bu t h ave oth er ch aracteristics o sch izop h ren ia (e.g.,
od d b eh avior, avoid an ce o social relation sh ip s).

I. Management
1. Pharmacologic management o sch izop h ren ia in clu des tradition al an tip sych otics (dop a-
m in e-2 [D 2]-recep tor an tagon ists) an d atyp ical an tip sych otic agen ts (see Ch ap ter 16).
Becau se o th eir better side e ect p ro iles, th e atyp ical agen ts are n ow irst-lin e treat-
RISE USMLE NEPAL

m en ts. Lon g-actin g in jectab le “d ep ot” orm s (e.g., h alop erid ol d ecan oate) o an tip sy-
ch otics are u se u l op tion s in p atien ts wh ose sym p tom s or social circu m stan ces lead to
n on com p lian ce with m edication .
2. Psychological management, in clu d in g in d ividu al, am ily, an d grou p p sych oth erapy (see
Ch ap ter 17), is u se u l to provide long-term support an d to h elp th e p atien t adh ere to th e
d ru g regim en . Also, b ecau se o p overty an d related actors, th ese p atien ts o ten h ave lim -
ited access to n u trition al ood s an d so m ay develop nutritional deficiencies wh ich u rth er
exacerb ate th eir clin ical con d ition .

II. OTHER PSYCHOTIC DISORDERS


A. Overview. Psych otic d isord ers are all ch aracterized at som e p oin t du rin g th eir cou rse by a
loss o tou ch with reality. However, th e oth er p sych otic disorders do not include all of the
criteria requ ired or th e diagn osis o sch izop h ren ia.
Chapter 11 Schizophrenia Spectrum and Other Psychotic Disorders 115

t a b l e 11.4 Schizophrenia and Other Psychotic Disorders


Disorder Characteristics Prognosis

Schizophrenia Psychotic and residual symptoms Lifelong social and occupational impairment
lasting at least 6 mo
Brief psychotic disorder Psychotic symptoms lasting >1 d 50%–80% recover completely
but <1 mo; often precipitating
psychosocial factors
Schizophreniform Psychotic and residual symptoms 33% recover completely
disorder lasting 1–6 mo
Schizoaffective disorder Symptoms of depression or mania as Lifelong social and occupational impairment
well as schizophrenia; presence of (somewhat higher overall level of functioning
HELP OTHERS SO THAT GOD WILL HELP YOU.

psychotic symptoms for at least 2 than in schizophrenia)


weeks without mood symptoms
Delusional disorder Fixed, persistent, delusional system 50% recover completely; many have relatively
(paranoid in the persecutory type normal social and occupational functioning
and romantic [often with a famous
person] in the erotomanic type);
few, if any, other thought disorders
Delusional disorder in Development of the same delusion in a 10%–40% recover completely when separated
partner of individual person in a close relationship (e.g., from the inducer
with delusional spouse, child) with someone with
disorder delusional disorder (the inducer)

B. Other psychotic disorders include (Table 11.4):


1. Brie p sych otic d isord er
2. Sch izop h ren i orm disorder
3. Sch izoa ective d isorder
4. Delu sion al d isord er
5. Delu sion al d isord er in p artn er o in d ivid u al with d elu sion al disord er
RISE USMLE NEPAL
Review Test

Directions: Each o th e n u m b ered item s or in com p lete statem en ts in th is section is ollowed by


an swers or by com p letion s o th e statem en t. Select th e one lettered an swer or com p letion th at
is best in each case.
HELP OTHERS SO THAT GOD WILL HELP YOU.

Questions 1–3 4. A 27-year-old p atien t with sch izop h ren ia


sh ows extrem e p sych om otor agitation to th e
p oin t o p h ysical exh au stion . At tim es, h e
A 26-year-old m edical stu d en t is b rou gh t to
h olds u n u su al, u n com ortable-lookin g body
th e em ergen cy dep artm en t by h er h u sban d.
p osition s. Th is p atien t is sh owin g wh ich o
Th e h u sban d tells th e doctor th at h is wi e
th e ollowin g sym p tom s o sch izop h ren ia?
h as sh own od d b eh avior ever sin ce ailin g
an exam 2 weeks ago. In p articu lar, sh e tells (A) Cataton ia
h im th at p eop le are tryin g to p oison h er. Th e (B) Hallu cin ation s
wom an h as n o p rior p sych iatric h istory, an d (C) Delu sion s
p h ysical exam in ation an d laboratory resu lts (D) Negative sym p tom s
are u n rem arkab le. (E) Ideas o re eren ce

1. Wh at is th e m ost ap p rop riate diagn osis


or th is p atien t at th is tim e? Questions 5 and 6
(A) Sch izop h ren ia
A 36-year-old p atien t with sch izop h ren ia tells
(B) Sch izoa ective d isorder
th e p h ysician th at th e govern m en t h as been
(C) Sch izop h ren i orm disorder
listen in g in on all o h is p h on e con versation s
(D) Brie p sych otic d isord er
or th e p ast year.
(E) Delu sion al d isord er
(F) Delu sion al d isord er in p artn er o 5. Th is p resen tation in dicates th at th e
in dividu al with delu sion al disorder p atien t is m ost likely to h ave wh ich o th e
(IPDD) ollowin g sym p tom s o sch izop h ren ia?
(G) Psych osis d u e to a gen eral m ed ical
con d ition (A) Hallu cin ation
(B) Delu sion
2. Th e p atien t’s belie th at p eop le are tryin g (C) Disorgan ized sp eech
RISE USMLE NEPAL

to poison h er is an exam p le o (D) Perseveration


(E) Magical th in kin g
(A) an illu sion
(B) a n eologism 6. Th e p atien t’s alse belie abou t th e
(C) a h allu cin ation govern m en t is an exam p le o a disorder o
(D) a d elu sion
(A) th ou gh t p rocesses
(E) an id ea o re eren ce
(B) th ou gh t con ten t
(C) orm o th ou gh t
3. An alysis o n eu rotran sm itter availab ility
(D) p ercep tion
in th e lim bic system o th is p atien t is m ost
(E) a ect
likely to reveal
(A) in creased dop am in e 7. Wh ich o th e ollowin g sym p tom s o
(B) decreased d op am in e sch izop h ren ia is likely to resp on d b est to
(C) in creased acetylch olin e an tip sych otic m edication ?
(D) decreased h istam in e
(A) Delu sion s
(E) decreased seroton in
(B) Flatten in g o a ect
(C) Poor sp eech con ten t
(D) Lack o m otivation
(E) Social with drawal
116
Chapter 11 Schizophrenia Spectrum and Other Psychotic Disorders 117

8. Wh en com p ared to tradition al 12. A 20-year-old m an rep orts th at h e


an tip sych otic m ed ication , atyp ical ju st ou n d ou t th at h is m oth er (wh om h e
an tip sych otic m ed ication is m ore likely to b e believed h ad died wh en h e was a ch ild)
h elp u l or wh ich o th e ollowin g sym p tom s? h as been in an in stitu tion or th e p ast 15
(A) Hallu cin ation s years with sch izop h ren ia. He asks wh at
(B) Delu sion s th e ch an ces are th at h e will develop
(C) Agitation sch izop h ren ia over th e cou rse o h is li e. Th e
(D) Overtalkativen ess m ost correct an swer is ap p roxim ately
(E) Social with drawal (A) 1%
(B) 5%
9. A 20-year-old wom an tells th e p h ysician (C) 10%
HELP OTHERS SO THAT GOD WILL HELP YOU.

th at som etim es sh e becom es righ ten ed (D) 50%


when h er room is dark becau se h er (E) 80%
com p u ter looks like a lion lu rkin g in th e
corn er. Th is is an exam p le o 13. A you n g m an rep orts th at h is 19-year-
(A) an illu sion old iden tical twin broth er h as ju st been
(B) a n eologism diagn osed with sch izop h ren ia an d wan ts
(C) a h allu cin ation to kn ow wh at th e likelih ood is th at h e will
(D) a d elu sion develop th is disorder. Th e m ost correct
(E) an id ea o re eren ce an swer is ap p roxim ately
(A) 1%
10. A 53-year-old h osp italized p atien t with (B) 5%
sch izop h ren ia tells th e p h ysician th at a (C) 10%
n ewscaster was talkin g ab ou t h er wh en h e (D) 50%
said on television , “A wom an was ou n d (E) 80%
sh opli tin g today.” Th is p atien t’s statem en t is
an exam p le o 14. Th e p ercen tage o p atien ts with
(A) an illu sion sch izop h ren ia wh o attem p t su icide is
(B) a n eologism ap proxim ately
(C) a h allu cin ation (A) 1%
(D) a d elu sion (B) 5%
(E) an id ea o re eren ce (C) 12%
(D) 50%
11. A 35-year-old m an wh o lives in a grou p (E) 80%
h om e says th at h is room m ates are spyin g on
h im by listen in g to h im th rou gh th e electrical 15. Wh ich o th e ollowin g is m ost closely
ou tlets. For th is reason , h e h as ch an ged associated with a good p rogn osis in
RISE USMLE NEPAL

room m ates a n u m b er o tim es over th e last sch izop h ren ia?


5 years. He d resses stran gely, is d irty with (A) You n ger age o on set
u n kem p t h air, an d seem s p reoccu p ied. He (B) Male gen der
rep orts th at h e h as trou b le p ayin g atten tion (C) Negative sym p tom s
to th e d octor’s qu estion s b ecau se “I am (D) Man y relap ses
listen in g to m y lead er givin g m e in stru ction s (E) Mood sym p tom s
in m y h ead .” Neu rop sych ological evalu ation
o this p atien t wh en h e is n ot h earin g voices, 16. Th e m ost com m on typ e o h allu cin ation
i.e., in th e residu al p h ase o th e illn ess, is seen in sch izop h ren ia is
m ost likely to reveal
(A) visu al
(A) severe m em ory im p airm en t (B) gu statory
(B) in ability to state h is n am e (C) au ditory
(C) severe in tellectu al disab ility (D) ol actory
(D) ron tal lobe dys u n ction (E) h yp n agogic
(E) lack o orien tation to p lace
118 BRS Behavioral Science

17. A 45-year-old m an with a 20-year 20. A 40-year-old attorn ey is con vin ced th at
h istory o severe dep ression an d p sych otic h is wi e is tryin g to kill h im . Wh en h e locks
sym p tom s h as h eld di eren t jobs, bu t n on e h im sel in th e basem en t an d re u ses to com e
o them or m ore th an 6 m on th s. He is ou t, th e p olice are called an d h e is taken to
su ccess u lly treated or h is severe d ep ressive th e em ergen cy room o th e local h osp ital.
sym p tom s, bu t h e rem ain s with drawn an d Th e wi e, wh o den ies h er h u sb an d’s ch arge,
odd an d exp resses th e belie th at h e h as n otes th at th e p atien t h as been sh owin g
been “ch osen” or a sp ecial m ission on earth . in creasin gly stran ge beh avior over th e p ast
Medical evalu ation is u n rem arkab le. Wh at 9 m on th s. An abn orm al gait is observed on
is th e m ost ap p rop riate d iagn osis or th is p h ysical exam in ation . Th e h istory reveals
p atien t? th at th e p atien t’s m oth er an d u n cle, wh o
HELP OTHERS SO THAT GOD WILL HELP YOU.

(A) Sch izop h ren ia h ad sh own sim ilar p sych iatric an d p h ysical
(B) Sch izoa ective d isorder sym p tom s, died in th eir early 50s a ter bein g
(C) Sch izop h ren i orm disorder in stitu tion alized in lon g-term care acilities
(D) Brie p sych otic d isord er or m an y years. Wh at is th e m ost ap p rop riate
(E) Delu sion al d isord er d iagn osis or th is p atien t?
(F) Delu sion al d isord er IPDD (A) Sch izop h ren ia
(G) Psych osis d u e to a gen eral m ed ical (B) Sch izoa ective disorder
con d ition (C) Sch izop h ren i orm disorder
(D) Brie p sych otic disorder
18. A 68-year-old p atien t tells th e p h ysician (E) Delu sion al d isord er
th at or th e last 7 years, h is n eigh bor h as (F) Delu sion al d isord er IPDD
been tryin g to get h im evicted rom h is (G) Psych osis du e to a gen eral m edical
ap artm en t by tellin g lies abou t h im to th e con d ition
lan dlord . Th e p atien t is m arried an d is
workin g u ll tim e in a job, wh ich h e h as 21. In a 50-year-old p atien t with
h eld or over 30 years. Medical evalu ation is sch izop h ren ia, th e size o th e cerebral
u n rem arkable. Wh at is th e m ost ap p rop riate ven tricles, glu cose u tilization in th e ron tal
diagn osis or th is p atien t? lobes, an d size o lim b ic stru ctu res are m ost
(A) Sch izop h ren ia likely to b e, resp ectively
(B) Sch izoa ective d isorder (A) in creased, decreased, decreased
(C) Sch izop h ren i orm disorder (B) in creased, decreased, in creased
(D) Brie p sych otic d isord er (C) in creased, in creased, decreased
(E) Delu sion al d isord er (D) decreased , decreased , d ecreased
(F) Delu sion al d isord er IPDD (E) decreased , in creased , decreased
(G) Psych osis d u e to a gen eral m ed ical (F) decreased , in creased , in creased
con d ition
RISE USMLE NEPAL

22. Over th e p ast week, a 30-year-old


19. A 60-year-old wom an wh ose h u sban d em ale p atien t with sch izop h ren ia h as n ot
believes (in th e absen ce o an y eviden ce) sp oken alth ou gh sh e occasion ally m akes
th at th eir h ou se is f lled with rad ioactive du st odd squ awkin g sou n ds. Sh e sh ows alm ost
worries ab ou t h er ab ility to clear th e h ou se n o acial exp ression bu t seem s extrem ely
o the du st wh en h e is h osp italized. Medical agitated an d h olds u n com ortable-lookin g
evalu ation is u n rem arkable. Wh at is th e m ost b ody p osition s. At tim es, sh e ap p ears to be
ap prop riate diagn osis or th is wom an ? listen in g to som e u n seen p erson . On th e
(A) Sch izop h ren ia d im en sion s o severity scale, th is p atien t will
(B) Sch izoa ective d isorder h ave a score closest to
(C) Sch izop h ren i orm disorder (A) 0
(D) Brie p sych otic d isord er (B) 4
(E) Delu sion al d isord er (C) 10
(F) Delu sion al d isord er IPDD (D) 12
(G) Psych osis d u e to a gen eral m ed ical (E) 18
con d ition
An swers an d Exp lan ation s

Typical Board Question


The answer is D. Th is p atien t wh o h as sch izop h ren ia is sh owin g eviden ce o a vitam in C de i-
cien cy. Vitam in C is ou n d m ain ly in resh ru it an d vegetables, an d de icien cy in vitam in C is
associated with h yp erkeratosis, in lam ed gu m s, an d p etech iae. In p art, becau se o p overty an d
HELP OTHERS SO THAT GOD WILL HELP YOU.

h om elessn ess, it is o ten a ch allen ge or p atien ts with severe m en tal illn esses to m ain tain th eir
n u trition al state. Th ere ore, p rovid in g p atien ts su ch as th is m an with n u trition al evalu ation ,
in orm ation an d , at tim es, d ietary su p p lem en ts can be an im p ortan t m edical in terven tion .

1. The answer is D. Th is p atien t is sh owin g evid en ce o b rie p sych otic disord er. Th is
d isorder is ch aracterized by p sych otic sym p tom s lastin g >1 d ay, bu t <1 m on th ; sh e h as
h ad sym p tom s or th e p ast 2 weeks. Also, th e stress o ailin g th e exam is likely to b e a
p recip itatin g p sych osocial actor in th is p atien t. Sch izoa ective d isord er is ch aracterized
by sym p tom s o m an ia an d / or d ep ression an d sch izop h ren ia, as well as p sych otic
sym p tom s th at occu r even in th e ab sen ce o m ood sym p tom s, an d li elon g social an d
occu p ation al im p airm en t. In sch izop h ren ia, p sych otic an d resid u al sym p tom s last at least
6 m on th s, an d th ere is li elon g social an d occu p ation al im p airm en t. Sch izop h ren i orm
d isorder is ch aracterized by p sych otic an d residu al sym p tom s lastin g 1–6 m on th s. In
d elu sion al disorder, wh ich o ten lasts or years, th ere is a ixed , delu sion al system ; ew, i
an y, oth er th ou gh t d isord ers; an d relatively n orm al social an d occu p ation al u n ction in g.
In delu sion al d isord er IPDD, a p erson d evelop s th e sam e delu sion as a p erson with
d elu sion al disorder with wh om th ey are in a close relation sh ip. Psych osis du e to a gen eral
m ed ical con d ition in volves p sych otic sym p tom s occu rrin g as a resu lt o p h ysical illn ess.
2. The answer is D. Believin g th at you are b ein g p oison ed is a delu sion , th at is, a alse belie .
A h allu cin ation is a alse p ercep tion ; an illu sion is a m isp ercep tion o real extern al stim u li;
an id ea o re eren ce is th e alse b elie o b ein g re erred to by oth ers; an d a n eologism is a
n ew, in ven ted word . All o th ese p h en om en a can be seen in p atien ts exh ibitin g p sych otic
sym p tom s n o m atter wh at th e cau se.
3. The answer is A. An alysis o n eu rotran sm itter availability in th e lim bic system o th is
p atien t with a p ositive p sych otic sym p tom (e.g., a delu sion ) is m ost likely to reveal
RISE USMLE NEPAL

in creased levels o dop am in e or seroton in . Acetylch olin e an d h istam in e are n ot so closely


in volved in th e p ath op h ysiology o p sych otic sym p tom s.
4. The answer is A. Th is p atien t wh o sh ows extrem e p sych om otor agitation an d u n u su al
u n com ortable-lookin g body p osition s is sh owin g cataton ia. Th e oth er listed sym p tom s
are n ot ch aracterized by p sych om otor agitation or h oldin g u n u su al body p osition s.
5. The answer is B. Th is p atien t’s b elie ab ou t th e govern m en t is a d elu sion . (see also an swer
to Qu estion 2).
6. The answer is B. A delusion is an exam ple o a disorder o thought content. Illusions and
hallucinations are disorders o perception, and loose associations and tangentiality are disorders
o orm o thought. Problem s with a ect are m ore likely to be seen in schizoa ective disorder.
7. The answer is A. Wh en com p ared to n egative sym p tom s (e.g., latten in g o a ect, p oor
sp eech con ten t, lack o m otivation , an d social with drawal), p ositive sym p tom s su ch as
delu sion s resp on d b etter to an tip sych otic m edication .
8. The answer is E. Social with d rawal is a n egative sym p tom o sch izop h ren ia. Negative
sym p tom s resp on d better to atyp ical an tip sych otic m edication th an to tradition al

119
120 BRS Behavioral Science

an tip sych otics. Hallu cin ation s, d elu sion s, agitation , an d overtalkativen ess are p ositive
sym p tom s o sch izop h ren ia.
9. The answer is A. An illusion is a m isperception o a real extern al stim ulus (e.g., a com puter
lookin g like a lion lurkin g in the corn er in a darken ed room ). A hallucin ation is a alse sen sory
p erception , an d a delusion is a alse belie n ot shared by others. An idea o re eren ce is the
alse belie o bein g re erred to by others, an d a n eologism is th e in ven tion o a n ew word.
10. The answer is E. An id ea o re eren ce is th e alse b elie o b ein g re erred to by oth ers (e.g., a
n ewscaster talkin g abou t th e p atien t on television ) (see also an swer to Qu estion 9).
11. The answer is D. Th is m an , wh o d resses stran gely, sh ows p oor groom in g, an d h as p aran oid
delu sion s an d au ditory h allu cin ation s over a p rolon ged p eriod, is m ost likely to h ave
HELP OTHERS SO THAT GOD WILL HELP YOU.

sch izop h ren ia. Neu rop sych ological evalu ation o a p atien t with sch izop h ren ia is m ost
likely to reveal ron tal lobe d ys u n ction . Peop le with sch izop h ren ia u su ally sh ow in tact
m em ory; orien tation to p erson , p lace, an d tim e; an d relatively typ ical.
12. The answer is C. Th e ch an ce th at th e son (or oth er irst-d egree relative) o a p erson with
sch izop h ren ia will develop th e d isord er over th e cou rse o h is li e is ap p roxim ately 10%.
13. The answer is D. Th e ch an ce th at th e iden tical twin o a p erson with sch izop h ren ia will
develop th e d isord er over th e cou rse o h is or h er li e is ap p roxim ately 50%.
14. The answer is D. Ap p roxim ately 50% o p atien ts with sch izop h ren ia attem p t su icide at
som e p oin t in th eir lives.
15. The answer is E. Mood sym p tom s are associated with a good p rogn osis in sch izop h ren ia. A
good p rogn osis is also associated with old er age o on set, ew n egative sym p tom s, em ale
gen d er, an d ew relap ses.
16. The answer is C. Au d itory h allu cin ation s are th e m ost com m on typ e o h allu cin ation s seen
in sch izop h ren ia.
17. The answer is B. Th is p atien t is sh owin g eviden ce o sch izoa ective disorder. Th is
disord er is ch aracterized by sym p tom s o a m ood disorder, as well as p sych otic sym p tom s
(th e delu sion th at h e h as b een “ch osen”) as well as an d li elon g social an d occu p ation al
im p airm en t (see also an swer to Qu estion 1). Sch izoa ective disorder is distin gu ish ed
rom b ip olar an d m ajor d ep ressive d isorder in th at th e p sych otic sym p tom s p ersist in th e
absen ce o m ood sym p tom s.
18. The answer is E. This p atien t is sh owin g eviden ce o delu sion al disorder, p ersecutory typ e.
In this disorder, there is a ixed, delusion al system (paran oid in the persecutory type); ew, i
an y, oth er thought disorders; an d relatively n orm al social an d occupation al un ction in g (e.g.,
RISE USMLE NEPAL

this p atien t is m arried an d has held a job or over 30 years) (see also an swer to Qu estion 1).
19. The answer is F. Th is p atien t is sh owin g eviden ce o delu sion al disorder IPDD. Sh e
h as develop ed th e sam e d elu sion th at h er h u sb an d h as (i.e., th eir h ou se is illed with
radioactive d u st). I sep arated or a p eriod o tim e rom h er h u sban d (th e in du cer), h er
p sych otic sym p tom s are likely to rem it (see also an swer to Qu estion 1).
20. The answer is G. Th is p atien t is sh owin g eviden ce o p sych osis du e to a gen eral m edical
con d ition . Th e abn orm al gait, age o th e p atien t, an d am ily h istory stron gly su ggest
Hu n tin gton’s d isease, wh ich o ten p resen ts with p sych iatric sym p tom s su ch as p sych osis
an d d ep ression (see also an swer to Qu estion 1).
21. The answer is A. In p atien ts with sch izop h ren ia, th e size o cerebral ven tricles, glu cose
u tilization in th e ron tal lobes, an d size o lim bic stru ctu res are m ost likely to be in creased,
decreased , an d decreased , resp ectively.
22. The answer is E. On th e d im en sion s o severity scale or sch izop h ren ia, th is p atien t will
h ave a score closest to 18. Sh e wou ld score 1 (equ ivocal) or delu sion s, 4 or h allu cin ation s
(listen in g to a n on existen t p erson ), 4 or disorgan ized sp eech (squ awkin g b u t n o clear
sp eech ), 4 or ab n orm al p sych om otor b eh avior (h oldin g odd p ostu res an d agitation ), an d
4 or n egative sym p tom s (n o acial exp ression or com m u n ication ).
Dep ressive Disorders
c ha pte r
12 an d Bip olar an d Related
Disorders
HELP OTHERS SO THAT GOD WILL HELP YOU.

Typical Board Question


A 35-year-old p h ysician tells h is in tern ist th at over th e p ast 3 m on th s h e h as lost in terest in
p layin g in th e h osp ital strin g qu artet, an activity h e orm erly en joyed. He also rep orts th at h e
com m on ly wakes u p a ew h ou rs b e ore h is alarm goes o an d can n ot all back to sleep an d
h as lost 15 p ou n ds with ou t dietin g. He states “m aybe m y am ily wou ld be better o with ou t
m e.” He says th at alth ou gh h e h as lots o ach es an d p ain s an d o ten eels tired , h e eels som e-
wh at better as th e day p rogresses. Ph ysical exam in ation an d laboratory stu dies are u n re-
m arkab le. Th e m ost ap p rop riate diagn osis or th is p atien t is
(A) p ersisten t d ep ressive disorder
(B) m ajor dep ressive d isord er
(C) m asked d ep ression
(D) illn ess an xiety disorder
(E) cycloth ym ic disord er
(F) m alin gerin g
(G) bip olar disord er
(See “An sw ers an d Explan ation s” at th e en d of th e ch apter.)

I. OVERVIEW
RISE USMLE NEPAL

A. Definitions
1. Th e d ep ressive an d b ip olar d isord ers are ch aracterized by a p rim ary disturbance in inter-
nal emotional state (mood), cau sin g su bjective distress an d p roblem s in social an d occu p a-
tion al u n ction in g.
2. Given the patient’s current social and occupational situation, h e or sh e em otion ally eels:
a. Som ewh at worse th an wou ld be exp ected (dysthymia ).
b. Very m u ch worse th an wou ld b e exp ected (depression).
c. Som ewh at better th an wou ld be exp ected (hypomania ).
d. Very m u ch better th an wou ld b e exp ected (mania ).
3. Th e Diagn ostic an d Statistical Man u al of Men tal Disorders, Fifth Edition (DSM-5) sep arates
wh at were p reviou sly kn own as m ood disorders in to bip olar disorders (bip olar I, bip o-
lar II, an d cycloth ym ic disorders) an d dep ressive disorders (m ajor dep ressive disorder,
p ersisten t dep ressive disord er, an d p rem en stru al dysp h oric disorder [m arked a ective
lability associated with m en ses]). Th ese disorders are de in ed in p art by th e len gth o th eir
ep isod es as ollows:
a. Major depressive disorder: On e or m ore ep isod es o d ep ression , each con tin u in g or at
least 2 weeks .

121
122 BRS Behavioral Science

b. Bipolar disorder: Ep isodes o both m an ia (con tin u in g or at least 1 week) an d dep res-
sion (bipolar I disorder) or b oth h yp om an ia (con tin u in g or at least 4 days ) an d dep res-
sion (bipolar II disorder).
c. Persistent depressive disorder: Dysth ym ia or dep ression con tin u in g over a 2-year p eriod
(1 year in ch ild ren ) with n o discrete ep isodes o illn ess.
d. Cyclothymic disorder: Hyp om an ia an d d ysth ym ia occu rrin g over a 2-year p eriod (1 year
in ch ild ren ) with n o d iscrete ep isodes o illn ess.
e. Depressive and bipolar disorder due to another medical condition and substance/medi-
cation-induced depressive and bipolar disorder can be con sidered secon dary m ood
disord ers.
HELP OTHERS SO THAT GOD WILL HELP YOU.

B. Epidemiology
1. Th ere are no differences in th e occu rren ce o dep ressive an d bip olar disorders associated
with eth n icity, ed u cation , m arital statu s, or in com e.
2. Lifetime prevalence o d ep ressive an d bip olar disorders
a. Major d ep ressive d isorder: 5%–12% or m en ; 10%–20% or wom en .
b. Bip olar d isord er: 1% overall; n o sex d i eren ce.
c. Persisten t d ep ressive disord er: 6% overall; u p to th ree tim es m ore com m on in wom en .
d. Cycloth ym ic d isord er: less th an 1% overall; n o sex di eren ce.

II. CLASSIFICATION OF DEPRESSIVE AND BIPOLAR DISORDERS


A. Major depressive disorder
1. Characteristics
a. SWAG is a m n em on ic device th at can qu ickly iden ti y dep ression an d di eren tiate it
rom n orm al sadn ess. I on e o th e ollowin g sym p tom s is p resen t, it is m ost likely th at
th e p atien t is dep ressed:
(1) S—Su icidality (h avin g a p lan or a m ean s o sel -d estru ction ).
(2) W—Weigh t loss (>5% o body weigh t).
(3) A—An h edon ia (loss o p leasu re or in terest in u su ally p leasu rable activities).
(4) G—Gu ilt ( eelin gs o resp on sib ility or n egative li e even ts wh en little or n on e
exists).
b. These and other symptoms o d ep ression are listed an d describ ed in Tab le 12.1.
2. Masked depression
a. As m an y as 50% o dep ressed p atien ts seem u n aware o or den y d ep ression an d th u s
RISE USMLE NEPAL

are said to h ave “masked depression.”


b. Patien ts with m asked dep ression o ten visit p rim ary care doctors com p lain in g o vague
physical symptoms .
c. In con trast to p atien ts wh o h ave som atic sym p tom disorders (p h ysical sym p tom s
resu ltin g rom p sych ological actors; see Ch ap ter 14), dep ressed p atien ts sh ow at least
on e SWAG sym p tom in ad d ition to th eir p h ysical com p lain ts.
3. Depression with seasonal pattern (DSP)
a. DSP is a su b typ e o m ajor dep ressive d isorder associated with th e win ter season an d
sh ort d ays. Wh ile less com m on , it can also be associated with th e su m m er season an d
lon g days.
b. DSP o ten is ch aracterized by atypical symptoms o dep ression (e.g., oversleep in g an d
overeatin g [p articu larly a cravin g or carboh ydrate-rich oods]) an d a h eavy eelin g in
th e lim bs (“leaden p aralysis”).
c. Patien ts with th e sh ort-day typ e o DSP m ay im p rove in resp on se to u ll-sp ectru m light
exposure with or with ou t an tidep ressan t m edication .
4. Premenstrual dysphoric disorder (PMDD)
a. Sym p tom s o m ood lab ility, dysth ym ia, an d an xiety occu rrin g in th e week preceding
menses an d m in im al or absen t in th e week p ostm en ses.
b. Sym p tom s were p resen t in m ost m en stru al cycles in th e p recedin g year.
Chapter 12 Depressive Disorders and Bipolar and Related Disorders 123

t a b l e 12.1 Signs and Symptoms of Depression and Mania


Depression Likelihood of Occurrence

SWAG (suicidality, weight loss, anhedonia, guilt) symptom ++++


Sadness, hopelessness, helplessness, low self-esteem ++++
Reduced energy and motivation ++++
Anxiety (is apprehensive about imagined dangers) ++++
Sleep problems (wakes frequently at night and too early in the morning) ++++
Cognitive problems (has difficulty with memory and concentration) +++
Change in physical activity (has psychomotor retardation or agitation) +++
Decreased or increased (in atypical depression) appetite for food and sex +++
Poor grooming ++
HELP OTHERS SO THAT GOD WILL HELP YOU.

Diurnal variation in symptoms (worse in the morning, better in the evening) ++


Suicidal ideation (has thoughts of killing oneself) ++
Suicide (takes one’s own life) +
Psychotic symptoms (has delusions of destruction and fatal illness) +
Mania Likelihood of Occurrence

Elevated mood (has strong feelings of happiness and physical well-being) ++++
Grandiosity and expansiveness (has feelings of self-importance) ++++
Irritability and impulsivity (is easily bothered and quick to anger) ++++
Disinhibition (shows uncharacteristic lack of modesty in dress or behavior) ++++
Assaultiveness (cannot control aggressive impulses; causes legal problems) ++++
Distractibility (cannot concentrate on relevant stimuli) ++++
Flight of ideas (thoughts move rapidly from one to the other) ++++
Pressured speech (seems compelled to speak quickly) ++++
Impaired judgment (provides unusual responses to hypothetical questions [e.g., says she ++++
would buy a blood bank if she inherited money])
Psychotic symptoms (has delusions of power and influence) +++
Approximate percentage of patients in which the sign or symptom is seen: +, <25%; ++, 50%; +++, 70%; ++++, >70%.

5. Suicide risk
a. Patien ts with dep ressive an d b ip olar disorders an d oth er p sych ological p roblem s are
at in creased risk for suicide .
b. Certain dem ograp h ic, p sych osocial, an d p h ysical actors a ect th is risk (Table 12.2).
c. Th e top ive risk actors or su icid e rom h igh er to lower risk are:
(1) Seriou s p rior su icide attem p t.
(2) Age older th an 45 years.
(3) Alcoh ol d ep en den ce.
RISE USMLE NEPAL

(4) History o rage an d violen t beh avior.


(5) Male sex.

B. Bipolar disorder
1. In bip olar disorder, th ere are ep isodes o both mania and depression (bipolar I disorder) or
both hypomania and depression (bipolar II disorder).
2. Th ere is n o sim p le m an ic d isord er becau se dep ressive sym p tom s even tu ally occu r.
Th ere ore, one episode of symptoms of mania (Table 12.1) alon e or h yp om an ia p lu s on e
ep isode o m ajor dep ression de in es bip olar disorder.
3. Psychotic symptoms , su ch as delu sion s, can occu r in dep ression (dep ression with p sy-
ch otic eatu res) as well as in m an ia.
a. In som e p atien ts (e.g., p oor p atien ts with low access to h ealth care), dep ressive or
bip olar d isord er with p sych otic sym p tom s can becom e severe en ou gh to be misdiag-
nosed as schizophrenia .
b. In con trast to sch izop h ren ia an d sch izoa ective disorder, in wh ich p atien ts are ch ron i-
cally im p aired , in d ep ressive an d bip olar disorders, th e p atien t’s m ood an d u n ction -
in g u su ally return to normal b etween ep isod es.
124 BRS Behavioral Science

C. Persistent depressive disorder (dysthymia) and cyclothymic disorder. In con trast to m ajor
dep ressive d isord er an d b ip olar disord er, resp ectively, p ersisten t dep ressive disorder an d
cycloth ym ic disorder are:
1. Non ep isodic.
2. Ch ron ic.
3. Rarely associated with p sych osis or su icid e.

III. ETIOLOGY
HELP OTHERS SO THAT GOD WILL HELP YOU.

A. Th e biologic etiology o d ep ressive an d bip olar disorders in clu des:


1. Altered neurotransmitter activity (see Ch ap ter 4).
2. A genetic component, stron gest in bip olar disorder (Table 12.3).
3. Physical illness an d related actors (Tab le 12.4).
4. Abn orm alities o th e lim b ic–h yp oth alam ic–p itu itary–adren al axis (see Ch ap ter 5).

t a b l e 12.2 Risk Factors for Suicide


Category Factor Increased Risk Decreased Risk

History Personal history Serious suicide attempt (about Suicidal gesture but not a serious
30% of people who attempt attempt
suicide try again and 10%
succeed)
<3 mo since previous attempt >3 mo since previous attempt
Possibility of rescue was remote Rescue was very likely
Family history Parent committed suicide No family history of suicide
Early loss of a parent through Intact family throughout childhood
divorce or death
Current, social, Psychiatric Depression Dysthymia or no depressive symptoms
psychological, symptoms Psychotic symptoms No psychotic symptoms
and physical Hopelessness Some hopefulness
factors Impulsiveness Thinks things out
Depth of Initial stages of recovery from The depth of severe depression;
depression deep depression; recovering patients rarely have the clarity of
patients may have enough thought or energy needed to plan
energy to commit suicide and commit suicide
Substance use Alcohol and drug dependence Little or no substance use
RISE USMLE NEPAL

Current intoxication Good health


Physical health Serious medical illness (e.g., No recent visit to a physician
cancer, AIDS)
Perception of serious illness
(most patients have visited a
physician in the 6 mo prior to
suicide)
Social relationships Divorced (particularly men) Married
Widowed Strong social support
Single, never married Has children
Lives alone Lives with others
Demographic factors Age Elderly (persons aged 65 years Children (up to age 15 y)
and older, especially men) Young adults (age 25–40 y)
Middle aged (over age 55 y in
women and age 45 y in men)
Adolescents (suicide is the third
leading cause of death in
those 15–24 y of age; rates
increase after neighborhood
suicide of a teen or when
media depict teenage
suicide)
Chapter 12 Depressive Disorders and Bipolar and Related Disorders 125

t a b l e 12.2 Risk Factors for Suicide (continued )


Category Factor Increased Risk Decreased Risk

Sex Male sex (men successfully Female sex (although women attempt
commit suicide three times suicide three times more often
more often than women) than men)
Occupation Professionals Nonprofessionals
Specific occupation Physicians (especially women
and psychiatrists)
Dentists and veterinarians
Police officers
Attorneys
HELP OTHERS SO THAT GOD WILL HELP YOU.

Musicians
Unemployed Employed
Race Caucasian Non-Caucasian
Religion Not religious Religious
J ewish Catholic
Protestant Muslim
Economic Economic recession or Strong economy
conditions depression
Lethality of attempt Plan and means A plan for suicide (e.g., decision No plan for suicide
to stockpile pills)
A means of committing suicide No means of suicide
(e.g., access to a gun)
Sudden appearance of
peacefulness in an agitated,
depressed patient (he has
reached an internal decision
to kill himself and is now
calm)
Method Shooting oneself Taking pills or poison
Crashing one’s vehicle Slashing one’s wrists
Hanging oneself
J umping from a high place

t a b l e 12.3 The Genetics of Bipolar Disorder


Group Approximate Occurrence (%)

General population 1
Person who has one parent or sibling (or dizygotic twin) with bipolar disorder 20
RISE USMLE NEPAL

Person who has two parents with bipolar disorder 60


Monozygotic twin of a person with bipolar disorder 75

t a b l e 12.4 Differential Diagnosis of Depression


Medical Conditions Psychiatric and Related Conditions

Cancer, particularly pancreatic and other gastrointestinal Schizophrenia (particularly after an acute psychotic
tumors episode)
Viral illness (e.g., pneumonia, influenza, acquired immune Adjustment disorder
deficiency syndrome [AIDS]) Anxiety disorder
Endocrinologic abnormality (e.g., hypothyroidism, diabetes, Normal reaction to a life loss, e.g., bereavement
Cushing’s syndrome) Somatic symptom disorder
Neurologic illness (e.g., Parkinson’s disease, multiple Eating disorder
sclerosis, Huntington’s disease, dementia, stroke Drug and alcohol use (particularly use of sedatives and
[particularly left frontal]) withdrawal from stimulants)
Nutritional deficiency (e.g., folic acid, B12) Prescription drug use (e.g., reserpine, steroids,
Renal or cardiopulmonary disease antihypertensives, antineoplastics)
126 BRS Behavioral Science

B. Th e psychosocial etiology o dep ression an d dysth ym ia can in clu de:


1. Loss of a parent in ch ildh ood.
2. Loss of a spouse or child in adu lth ood.
3. Loss of health.
4. Low self-esteem an d n egative in terp retation o li e even ts.
5. “Learned helplessness ” (i.e., b ecau se attem p ts to escap e bad situ ation s in th e p ast h ave
p roven u tile, th e p erson n ow eels h elp less) (see Ch ap ter 7).

C. Psych osocial actors are not directly involved in the etiology of mania or h yp om an ia.
HELP OTHERS SO THAT GOD WILL HELP YOU.

IV. MANAGEMENT
A. Overview
1. Dep ression is successfully managed in most patients .
2. On ly abou t 25% of patients with depression seek and receive treatment.
a. Patien ts d o n ot seek treatm en t in p art becau se Am erican s o ten b elieve th at m en tal
illn ess in dicates personal failure or weakness .
b. As in oth er illn esses, women are more likely than men to seek treatment.
3. Un treated ep isod es o d ep ression an d m an ia are usually self-limiting an d last ap p roxi-
m ately 6–12 m on th s an d 3 m on th s, resp ectively.
4. Th e most effective management o d ep ressive an d bip olar disorders is pharmacologic .

B. Pharmacologic management (see Ch ap ter 16)


1. Treatm en t or dep ression an d dysth ym ia in clu des antidepressant agents (e.g., h eterocy-
clics, selective seroton in an d selective seroton in an d n orep in ep h rin e reu p take in h ib itors
[SSRIs an d SNRIs], m on oam in e oxid ase in h ibitors [MAOIs], an d stim u lan ts).
2. Mood stabilizers
a. Lithium an d anticonvulsants su ch as carbam azep in e (Tegretol) an d divalp roex
(Dep akote) are u sed to m an age b ip olar disord er.
b. Mood stabilizers in d oses sim ilar to th ose u sed to m an age b ip olar d isorder are th e p ri-
m ary treatm en t or cyclothymic disorder.
c. Atypical antipsychotics su ch as olan zap in e (Zyp rexa) an d risp erid on e (Risp erd al).
d. Sedative agents su ch as lorazep am (Ativan ) are u sed to h elp m an age acu te m an ic ep i-
sod es b ecau se th ey im p rove sym p tom s qu ickly.
RISE USMLE NEPAL

C. Psychological management
1. Psych ological m an agem en t or dep ression an d dysth ym ia in clu des p sych oan alytic, in ter-
p erson al, am ily, b eh avioral, an d cogn itive th erap ies (see Ch ap ter 17).
2. Psychological management in conjunction with medication is more effective th an eith er typ e
o m an agem en t alon e.

D. Electroconvulsive therapy (ECT) (see Ch ap ter 16). Th e p rim ary in d ication or ECT is major
depressive disorder. It is u sed wh en :
1. Th e sym p tom s do not respond to antidepressant medications .
2. An tidep ressan ts are too dangerous or have intolerable side effects . Th u s, ECT m ay be p ar-
ticu larly u se u l or elderly patients .
3. Rapid resolution o sym p tom s is n ecessary (e.g., th e p atien t is acu tely suicidal or psychotic ).
Review Test

Directions: Each o th e n u m b ered item s or in com p lete statem en ts in th is section is ollowed by


an swers or by com p letion s o th e statem en t. Select th e one lettered an swer or com p letion th at
is best in each case.
HELP OTHERS SO THAT GOD WILL HELP YOU.

1. A 65-year-old wom an , wh o was diagn osed 4. Th is college stu den t h as two broth ers.
with advan ced lu n g can cer 3 m on th s ago, Th e rst is h is m on ozygotic twin ; th e secon d
h as lost 18 p ou n ds, wakes requ en tly du rin g is 2 years you n ger. Th e risks th at h is rst
th e n igh t, an d h as very little en ergy. Over th e an d secon d broth ers will develop bip olar
p ast m on th , sh e h as been p reoccu p ied with disord er are, resp ectively, ab ou t
eelin gs o gu ilt abou t “p eop le I h ave h u rt in (A) 75% an d 60%
m y li e” an d exp resses con cern th at sh e will (B) 75% an d 20%
die in p ain . Th e sign or sym p tom m ost likely (C) 60% an d 10%
to in dicate th at th is p atien t is exp erien cin g (D) 50% an d 10%
a m ajor d ep ressive ep isode rath er th an a (E) 10% an d 1%
n orm al reaction to li e-lim itin g illn ess is
(A) weigh t loss
(B) decreased en ergy Questions 5 and 6
(C) di icu lty sleep in g
(D) p reoccu p ation with eelin gs o gu ilt For th e p ast ew m on th s, a 28-year-old wom an
(E) con cern ab ou t d yin g in p ain has seem ed ull o en ergy an d op tim ism or
n o obviou s reason . Alth ou gh sh e gets on ly
about 6 h ours o sleep a n igh t, sh e h as been
Questions 2–4 very p roductive at work. Sh e is talkative an d
gregariou s an d relates th at sh e belon gs to
A 20-year-old m ale college stu den t is taken to ou r clu b s an d two di eren t sp orts team s. A
the em ergen cy departm en t by p olice because ew years previously, rien ds say sh e was o ten
he tried to en ter a state o ice buildin g to “h ave pessim istic an d seem ed tired an d “wash ed
a con eren ce with th e govern or” abou t con - ou t.” Du rin g th at p eriod, sh e con tin u ed to
ductin g a un d drive to “ in an ce m y cu re or work but did n ot seek out social activities an d
can cer.” Wh en p olice p reven t h im rom en ter- had little in terest in sex. Th ere is n o eviden ce
RISE USMLE NEPAL

in g the buildin g, he becom es irritable an d o a th ou gh t disorder an d th e p atien t den ies


hostile an d resists attem pts to restrain h im . su icidality or h op elessn ess. Ph ysical exam i-
n ation in clu din g body weigh t is n orm al.
2. Th e m ost ap p rop riate diagn osis or th is
p atien t is 5. Th is p atien t sh ows eviden ce o
(A) p ersisten t d ep ressive disorder (A) p ersisten t dep ressive disorder
(B) m ajor dep ressive d isord er (B) m ajor dep ressive disorder
(C) bip olar disord er (C) bip olar disord er
(D) illn ess an xiety disorder (D) illn ess an xiety disorder
(E) cycloth ym ic disord er (E) cycloth ym ic disorder

3. Th e m ost e ective lon g-term 6. Th e m ost e ective lon g-term


m an agem en t or th is p atien t is m an agem en t or th is p atien t is
(A) a h eterocyclic an tid ep ressan t (A) a h eterocyclic an tidep ressan t
(B) lith iu m (B) lith iu m
(C) electrocon vu lsive th erapy (C) electrocon vu lsive th erapy
(D) p sych oth erapy (D) p sych oth erapy
(E) a m on oam in e oxid ase in h ibitor (E) a m on oam in e oxid ase in h ibitor

127
128 BRS Behavioral Science

Questions 7 and 8 11. A 28-year-old m an p resen ts com p lain in g


o headach es an d a variety o oth er ach es
an d p ain s th at h ave been p resen t or
A 62-year-old wom an wh ose h u sban d d ied
th e p ast 6 m on th s. He d en ies th at h e is
6 m on th s ago tells h er p h ysician th at sh e
sad or h op eless. A ter a 4-week trial o
believes killin g h ersel wou ld en d h er su er-
an tidep ressan t m edication , th e p atien t’s
in g. Ph ysical exam in ation is u n rem arkable.
p h ysical com p lain ts h ave disap p eared. Th e
m ost ap p rop riate diagn osis or th is p atien t is
7. O th e ollowin g sign s an d sym p tom s,
wh ich is m ost likely to be seen in th is p atien t? (A) p ersisten t dep ressive disorder
(B) m ajor dep ressive disorder
(A) Weigh t gain
(C) m asked d ep ression
(B) Fligh t o ideas
HELP OTHERS SO THAT GOD WILL HELP YOU.

(D) illn ess an xiety disorder


(C) Au ditory h allu cin ation s
(E) cycloth ym ic disorder
(D) Feelin g better in th e m orn in g th an in th e
(F) m alin gerin g
even in g
(G) bip olar disord er
(E) Poor groom in g

8. An alysis o n eu rotran sm itter availab ility


in th e brain o th is p atien t is m ost likely to Questions 12–14
reveal
(A) in creased dop am in e A 65-year-old Cath olic m ale p atien t h as been
(B) decreased h istam in e abu sin g alcoh ol or th e p ast 15 years. His h is-
(C) in creased acetylch olin e tory reveals th at h is wi e recen tly asked h im
(D) decreased acetylch olin e or a sep aration .
(E) decreased seroton in
12. Wh ich o th e ollowin g ch aracteristics is
9. A 25-year-old m ale p atien t wh o is slow th is p atien t’s greatest risk actor or su icid e?
m ovin g an d h as a f at a ect is p u t on (A) Alcoh olism
f u oxetin e (Prozac). With in 2 weeks, th e (B) Male sex
p atien t is sh owin g greatly in creased activity (C) Marital sep aration
level, f igh t o ideas, an d p ressu red sp eech . (D) Religion
In th is p atien t, th e m edication h as (E) Age
(A) p recip itated a m an ic ep isode
(B) h ad a toxic e ect 13. Th is m an is at th e lowest risk or su icide
(C) h ad a delayed e ect i h e works as a
(D) in creased an xiety (A) m essen ger
(E) in creased dep ression (B) p olicem an
RISE USMLE NEPAL

(C) p h ysician
10. A 43-year-old sch ool teach er relates th at (D) lawyer
over th e p ast year h e h as o ten elt th at li e (E) den tist
is n ot worth livin g. He also n otes th at du rin g
th is tim e, h e h as n oticed a grad u al slowin g o 14. I th is p atien t tries to com m it su icide, th e
cogn ition , an d th e develop m en t o a gravelly m eth od m ost likely to ail is
voice, con stip ation , an d h air loss. Ph ysical
(A) sh ootin g h im sel with a gu n
exam in ation is n orm al, excep t or n oticeable
(B) crash in g h is car
dryn ess o th e skin an d a lag in th e relaxation
(C) slash in g h is wrists
p h ase o th e an kle jerk ref ex. At th is tim e,
(D) ju m p in g rom a h igh p lace
th e m ost likely diagn osis or th is p atien t is
(E) h an gin g h im sel
(A) m ajor dep ressive d isord er
(B) bip olar I d isorder 15. Th e p ercen tage o dep ressed p atien ts who
(C) bip olar II disorder seek treatm en t or their sym ptom s is about
(D) p ersisten t d ep ressive disorder
(A) 1%
(E) cycloth ym ic disord er
(B) 5%
(F) su b stan ce-in du ced d ep ressive d isord er
(C) 25%
(G) dep ressive d isord er du e to an oth er
(D) 50%
m ed ical con d ition
(E) 75%
Chapter 12 Depressive Disorders and Bipolar and Related Disorders 129

16. A 15-year-old girl is brou gh t to th e 18. A 30-year-old n an cial con su ltan t tells
em ergen cy room a ter in gestin g 20 h er doctor th at over th e p ast 5 years sh e h as
acetam in op h en tablets. Sh e tells th e elt “d own” m ost o th e tim e. Sh e relates th at
p h ysician th at sh e tried to com m it su icide wh en colleagu es ask h er to din n er or to a
b ecau se sh e was n ot adm itted to an h on ors get-togeth er sh e u su ally says “yes” bu t th en
En glish class. Th e girl is th e p residen t o rarely eels like goin g wh en th e tim e com es
h er grade in sch ool an d always tries to an d does n ot h ave a good tim e wh en sh e
b e p er ect. Th e m ost im p ortan t actor in does go. Th ere are n o sign i can t p h ysical
wh eth er th is girl tries to kill h ersel again is n din gs. Wh ile th e p atien t den ies su icidality,
(A) th at sh e is em ale sh e n otes th at sh e n ever eels really excited
(B) th e m eth od o th e su icide attem p t or hap py abou t an yth in g. Th e best diagn osis
HELP OTHERS SO THAT GOD WILL HELP YOU.

(C) th at sh e h as m ajor d ep ressive d isord er or th is p atien t at th is tim e is


(D) th at sh e tried to com m it su icide on ce (A) m ajor dep ressive disorder
(E) h er n eed to be p er ect (B) bip olar I d isorder
(C) bip olar II disorder
17. Wh en com p ared with a m an , th e ch an ces (D) p ersisten t dep ressive disorder
th at a wom an will develop m ajor d ep ressive (E) cycloth ym ic disorder
d isord er, p ersisten t d ep ressive d isorder, (F) su bstan ce-in du ced dep ressive disorder
or bip olar d isorder over th e cou rse o h er (G) dep ressive disorder du e to an oth er
li etim e are, resp ectively m ed ical con d ition
(A) h igh er, h igh er, equ al
19. A 45-year-old m an with bip olar disorder
(B) h igh er, h igh er, lower
tells h is d octor th at h e h as rem arried an d
(C) h igh er, equ al, h igh er
wou ld like to h ave a ch ild with h is n ew wi e.
(D) h igh er, h igh er, h igh er
He is con cern ed becau se th e 19-year-old
(E) equ al, h igh er, equ al
daugh ter th at h e h ad with h is rst wi e h as
(F) equ al, h igh er, lower
ju st been d iagn osed with b ip olar d isorder.
(G) equ al, equ al, equ al
Neith er o th e p atien t’s wives h as bip olar
disorder. Wh at is th e p robability th at th is
p atien t will h ave an oth er ch ild with bip olar
disorder?
(A) 1%
(B) 10%
(C) 20%
(D) 50%
(E) 70%
RISE USMLE NEPAL
An swers an d Exp lan ation s

Typical Board Question


The answer is B. Th is p atien t is m ost likely to h ave m ajor dep ressive disorder. Eviden ce or th is
is th at, wh ile th ere are n o p h ysical in din gs, h e h as lost in terest in h is u su al activities, wakes u p
too early in th e m orn in g, h as vagu e p h ysical sym p tom s, sh ows diu rn al variation in sym p tom s
HELP OTHERS SO THAT GOD WILL HELP YOU.

(worse in th e m orn in g), h as lost a sign i ican t am ou n t o weigh t, an d is sh owin g su icid al ideation
(e.g., “m aybe m y am ily wou ld be better o with ou t m e”). Also, h is sym p tom s h ave been p resen t
or a d iscrete, id en ti ied am ou n t o tim e. Persisten t dep ressive disorder in volves m ild-to-severe
dep ression m ost o th e tim e, occu rrin g over a 2-year p eriod with n o discrete ep isodes o ill-
n ess. Bip olar d isorder in volves ep isodes o both m an ia an d dep ression . Cycloth ym ic disorder
in volves ep isod es o h yp om an ia an d d ysth ym ia occu rrin g over a 2-year p eriod with n o discrete
ep isod es o illn ess. In illn ess an xiety d isorder (see Ch ap ter 13), p atien ts believe th at n orm al
bod y u n ction s or m in or illn esses are seriou s or li e th reaten in g. Peop le wh o are m alin gerin g
abricate sym p tom s or obviou s gain (e.g., to win a lawsu it) (see Ch ap ter 13).

1. The answer is D. Th e sign or sym p tom m ost likely to in dicate that this p atien t is experien cin g
a m ajor dep ressive ep isode rath er than a n orm al reaction to serious illn ess is her
p reoccupation with eelin gs o guilt. Such eelin gs are m ore characteristic o depression than
sadn ess abou t bein g very ill. Th e oth er sym p tom s that the patien t sh ows (e.g., weight loss,
decreased en ergy, an d sleep p roblem s) are ch aracteristic o p atien ts with advan ced can cer.
Fear o dyin g in pain is realistic an d com m on ly seen in patien ts with li e-lim itin g illn esses.
2. The answer is C. 3. The answer is B. 4. The answer is B. Th is p atien t is m ost likely to h ave
bip olar I disord er. Wh ile th is disord er in volves ep isodes o both m an ia an d dep ression ,
a sin gle ep isod e o m an ia de in es th e illn ess. Th e belie s th at on e is im p ortan t en ou gh
to d em an d a con eren ce with th e govern or an d cu re can cer are gran diose d elu sion s.
Sch izop h ren ic delu sion s are com m on ly p aran oid in n atu re. Irritability an d h ostility are also
com m on in a m an ic ep isode. O th e listed treatm en ts, th e on e m ost e ective or bip olar
disorder is lith iu m . Heterocyclic an tid ep ressan ts, electrocon vu lsive th erapy, m on oam in e
oxidase in h ib itors, an d p sych oth erapy are u sed p rim arily to m an age dep ression .
An tid ep ressan ts an d p sych oth erapy are u sed to m an age dysth ym ia. Th e ch an ces o th e
RISE USMLE NEPAL

m on ozygotic twin an d irst-d egree relative (e.g., broth er) o th is bip olar p atien t develop in g
th e d isord er are ab ou t 75% an d 20%, resp ectively.
5. The answer is E. 6. The answer is B. Th is p atien t sh ows eviden ce o cycloth ym ic disorder.
Th is d isord er in volves p eriods o b oth h yp om an ia (en ergy an d op tim ism ) an d dysth ym ia
(p essim ism an d eelin g “wash ed ou t”) occu rrin g over a 2-year p eriod with n o discrete
ep isodes o illn ess. O th e listed treatm en ts, th e on e m ost e ective or cycloth ym ic
disorder, as or b ip olar d isord er, is lith iu m . Heterocyclic an tidep ressan ts, electrocon vu lsive
th erapy, m on oam in e oxidase in h ibitors, an d p sych oth erapy are p rim arily u sed to m an age
dep ression . An tid ep ressan ts an d p sych oth erapy are u sed to m an age dysth ym ia.
7. The answer is E. 8. The answer is E. Th is wom an is showin g eviden ce o m ajor dep ression
(n ote: Su icidality is n ot ch aracteristic o a n orm al grie reaction ). Depressed p eop le typically
sh ow p oor groom in g. Sh e is also m ore likely to sh ow weight loss an d to eel better in the
even in g th an in th e m orn in g. Au ditory h allu cin ation s are com m on in schizop hren ia
bu t u n com m on in dep ression . Flight o ideas is characteristic o m an ia. An alysis o
n eurotran sm itter availability in th is p atien t is m ost likely to reveal decreased seroton in ,
com m on ly re lected in decreased plasm a levels o its m ajor m etabolite 5-HIAA. In creased
dopam ine is seen in schizophrenia, and decreased acetylcholine is seen in Alzheim er’s disease.

130
Chapter 12 Depressive Disorders and Bipolar and Related Disorders 131

9. The answer is A. In th is d ep ressed p atien t, th e an tid ep ressan t lu oxetin e h as p recip itated


a m an ic ep isode (i.e., greatly in creased activity level, ligh t o ideas, an d p ressu red sp eech ).
Th is in dicates th at th e p atien t h as b ip olar disorder rath er th an m ajor dep ressive disorder.
Th ere is n o evid en ce o in creased d ep ression , in creased an xiety, or a delayed or toxic e ect
in th is p atien t.
10. The answer is G. Th e m ost likely d iagn osis at th is tim e is dep ressive disorder du e to
an oth er m edical con dition . Th is p atien t is sh owin g sym p tom s o h yp oth yroid ism , or
exam p le, slowin g o cogn ition , gravelly voice, con stip ation , dry skin , h air loss, an d lag in
th e relaxation p h ase o th e an kle jerk re lex, in addition to sym p tom s o m ajor dep ression
(e.g., su icid ality). Dep ressive d isorders are diagn osed wh en th ere are n o m edical in din gs
to exp lain th e m ood sym p tom s. Major d ep ressive disorder in volves at least on e SWAG
HELP OTHERS SO THAT GOD WILL HELP YOU.

sym p tom m ost o th e tim e or a p eriod o at least 2 weeks. Persisten t dep ressive disorder
in volves m ild -to-severe dep ression m ost o th e tim e, occu rrin g over a 2-year p eriod with
n o d iscrete ep isodes o sym p tom s. Bip olar disorder in volves ep isodes o both m an ia an d
dep ression (b ip olar I) or h yp om an ia an d dep ression (bip olar II). Cycloth ym ic disorder
in volves ep isod es o h yp om an ia an d d ysth ym ia occu rrin g over a 2-year p eriod with n o
discrete ep isod es o sym p tom s.
11. The answer is C. Th is p atien t’s p h ysical com p lain ts (i.e., h ead ach es an d oth er ach es an d
p ain s) were relieved by an tidep ressan t m edication . Th is in dicates th at th ese sym p tom s
were m an i estation s o m asked (h id den ) dep ression rath er th an illn ess an xiety disord er.
Th ere is n o evid en ce in th is p atien t o b ip olar disorder, p ersisten t dep ressive disorder,
cycloth ym ic disord er, or m alin gerin g.
12. The answer is E. 13. The answer is A. 14. The answer is C. Alth ou gh m ale sex, alcoh ol ab u se,
an d m arital sep aration all are risk actors or su icid e, th e h igh est risk actor o th ose
m en tion ed is th is p atien t’s advan ced age. Cath olic religion is associated with a redu ced
risk o su icid e. Non p ro ession als are at a lower su icid e risk th an p ro ession als. Am on g
p ro ession als, th ose at th e h igh est risk or su icide are p olice o icers, p h ysician s, lawyers,
an d d en tists. Th e m eth od o su icid e m ost likely to ail is slash in g th e wrists or takin g p ills.
Sh ootin g, crash in g a car, ju m p in g rom a h igh p lace, an d h an gin g are m ore leth al m eth ods
o com m ittin g su icid e.
15. The answer is C. On ly ab ou t 25% o d ep ressed p atien ts seek treatm en t, alth ou gh
m an agem en t (an tidep ressan ts, p sych oth erapy, electrocon vu lsive th erapy) is e ective in
m ost d ep ressed p atien ts.
16. The answer is D. Th is girl sh ows a n u m ber o risk actors or dep ression an d attem p ted
su icid e, in clu d in g em ale sex an d h er excessive n eed to be p er ect. However, th e m ost
RISE USMLE NEPAL

im p ortan t actor in wh eth er sh e tries to kill h ersel again is th at sh e tried to com m it su icide
on ce. Takin g p ills su ch as asp irin or acetam in op h en is less leth al th an oth er m eth ods, b u t
you n g p eop le su ch as th is teen ager m ay n ot kn ow th is. Th u s, th is girl h as m ade a seriou s
su icid e attem p t. (See also an swers to Qu estion s 12–14.)
17. The answer is A. Wh en com p ared with a m an , a wom an is twice as likely to develop m ajor
dep ressive d isorder an d th ree tim es as likely to develop p ersisten t dep ressive disorder.
Bip olar d isord er an d cycloth ym ic disord er occu r equ ally in m en an d wom en .
18. The answer is D. Th is b est d iagn osis or th is p atien t is p ersisten t dep ressive d isord er.
Th e p atien t h as h ad a low m ood or years, sh e is n ever really h ap py or excited abou t wh at
sh ou ld be p leasan t exp erien ces. Becau se h er sym p tom s are ch ron ic an d n on ep isod ic, it is
less likely th at sh e h as m ajor d ep ressive disorder.
19. The answer is C. Th e likelih ood th at th is m an with bip olar disorder will h ave a ch ild with
bip olar disord er is abou t 20%. Th e act th at h is older ch ild h as b ip olar d isord er is n ot
relevan t to th e ch an ces th at h is n ext ch ild will h ave th e disorder.
Anxiety Disorders, Som atic
c ha pte r
13 Sym ptom Disorders, and
Related Conditions
HELP OTHERS SO THAT GOD WILL HELP YOU.

Typical Board Question


Sin ce h e was an in an t, an 8-year-old boy requ en tly witn essed h is ath er p h ysically abu sin g
h is m oth er. Th e m oth er rep orts th at, sin ce sh e d ivorced th e ath er 1 year ago, th e ch ild wakes
rep eatedly du rin g th e n igh t to go in to h is m oth er’s room to “ch eck on h er.” Becau se o th ese
rep eated awaken in gs, th e ch ild is tired du rin g th e day, o ten alls asleep in class, an d can -
n ot keep u p with h is sch oolwork. Th is ch ild is sh owin g eviden ce o wh ich o th e ollowin g
disord ers?
(A) Narcolep sy
(B) Gen eralized an xiety disorder
(C) Social an xiety disorder
(D) Posttrau m atic stress disorder (PTSD)
(E) Illn ess an xiety disorder
(See “An sw ers an d Explan ation s” at en d of ch apter.)

I. ANXIETY DISORDERS
A. Fear and anxiety
1. Fear is a n orm al reaction to a kn own , extern al sou rce o dan ger.
RISE USMLE NEPAL

2. In anxiety, th e in dividu al is righ ten ed bu t th e sou rce o th e dan ger is n ot kn own , n ot rec-
ogn ized, or inadequate to account for the symptoms .
3. Th e physiologic manifestations o an xiety are sim ilar to th ose o ear. Th ey in clu de:
a. Sh akin ess an d sweatin g.
b. Palp itation s (su b jective exp erien ce o tach ycardia).
c. Tin glin g in th e extrem ities an d n u m bn ess arou n d th e m ou th .
d. Dizzin ess an d syn cop e ( ain tin g).
e. Gastroin testin al an d u rin ary distu rban ces (e.g., diarrh ea an d u rin ary requ en cy).
f. Myd riasis (p u p il dilation )

B. Classification and occurrence of the anxiety disorders and related conditions


1. Th e Diagnostic and Statistical Manual of Mental Disorders, Fifth Edition (DSM-5) classi ica-
tion o an xiety disorders an d related con dition s in clu des:
a. Pan ic disorder.
b. Ph obias (sp eci ic p h obia, agorap h obia, an d social an xiety disorder).
c. Gen eralized an xiety disorder (GAD).
d. Sep aration an xiety disorder (see Ch ap ter 15).
e. Selective m u tism (see Ch ap ter 15).

132
Chapter 13 Anxiety Disorders, Somatic Symptom Disorders, and Related Conditions 133

2. Descrip tion s o th e irst th ree o th ese disorders as well as o obsessive–com p u lsive


disord ers an d trau m a- an d stress-related disorders can be ou n d in Table 13.1.
3. Th e an xiety disorders an d related con dition s are th e most commonly treated m en tal h ealth
p roblem s.

C. The organic basis of anxiety


1. Neu rotran sm itters in volved in th e d evelop m en t o an xiety in clu de n orep in ep h rin e
(in creased activity), seroton in (decreased activity), an d g-am in obu tyric acid (GABA)
(decreased activity) (see Ch ap ter 4).
2. Th e locus ceruleus (site o n oradren ergic n eu ron s), raphe nucleus (site o seroton ergic
n eu ron s), caudate nucleus (p articu larly in OCD), temporal cortex, an d frontal cortex are
HELP OTHERS SO THAT GOD WILL HELP YOU.

th e brain areas likely to be in volved in an xiety disorders.


3. Organ ic cau ses o sym p tom s o an xiety in clu de excessive caffeine intake , su b stan ce
u se, h yp erth yroid ism , vitam in B12 d e icien cy, h yp oglycem ia or h yp erglycem ia, cardiac
arrh yth m ia, an em ia, p u lm on ary disease, an d pheochromocytoma (adren al m edu llary
tu m or).
4. I th e etiology is p rim arily organ ic, th e diagn oses substance/medication-induced anxiety
disorder or anxiety disorder due to another medical condition m ay be ap p rop riate.

D. Management of the anxiety disorders


1. Antianxiety agents (see Ch ap ter 16), in clu d in g ben zod iazep in es, bu sp iron e, an d
β-b lockers, are u sed to treat th e sym p tom s o an xiety.
a. Benzodiazepines are ast-actin g an tian xiety agen ts.
(1) Becau se th ey carry a h igh risk o dep en den ce an d addiction , th ey are u su ally used
for only a limited amount of time to treat acu te an xiety sym p tom s.
(2) Becau se th ey work qu ickly, ben zodiazep in es, p articu larly alprazolam (Xanax), are
u sed or em ergen cy dep artm en t m an agem en t o panic attacks .
b. Buspirone (BuSpar) is a n on b en zodiazep in e an tian xiety agen t.
(1) Becau se o its low addiction potential, bu sp iron e is u se u l as long-term maintenance
th erapy or p atien ts with GAD.
(2) Becau se it takes up to 2 weeks to work, b u sp iron e h as little im m ed iate e ect on
an xiety sym p tom s.
c. Th e b -blockers , su ch as propranolol (Inderal), are u sed to con trol th e autonomic symp-
toms (e.g., tach ycardia) o an xiety, p articu larly or an xiety abou t p er orm in g in p u blic
or takin g an exam in ation .
2. Antidepressants (see Ch ap ter 16)
a. An tidep ressan ts, in clu d in g m on oam in e oxidase in h ibitors (MAOIs), tricyclics, an d
RISE USMLE NEPAL

esp ecially selective serotonin reuptake inhibitors (SSRIs), su ch as p aroxetin e (Paxil),


lu oxetin e (Prozac), an d sertralin e (Zolo t), are th e m ost e ective lon g-term (m ain te-
n an ce) th erapy or p an ic disord er an d OCD ( lu voxam in e [Lu vox] is on ly in dicated or
OCD) an d h ave sh own e icacy also in PTSD.
b. SSRIs (e.g., escitalop ram [Lexap ro]) an d th e selective serotonin and norepinephrine reup-
take inhibitors (SNRIs) ven la axin e (E exor) an d du loxetin e (Cym balta) are ap p roved to
treat GAD.
c. Paroxetin e, sertralin e, an d ven la axin e n ow also are in dicated in th e m an agem en t o
social anxiety disorder.
3. Psychological management (see also Ch ap ter 17)
a. Systematic desensitization an d cognitive–behavioral therapy are th e m ost e ective m an -
agem en t or sp eci ic p h obias an d are u se u l adju n cts to p h arm acoth erapy in oth er
an xiety disorders.
b. Beh avioral th erap ies, su ch as lood in g an d im p losion , also are u se u l in sp eci ic
c. Su p p ort grou p s (e.g., victim su rvivor grou p s) are p articu larly u se u l or ASD an d
PTSD.
t a b l e 13.1 DSM-5 Classification of the Anxiety Disorders, Obsessive–Compulsive and
Related Disorders, and Trauma- and Stress-Related Disorders
Anxiety Disorders: Panic Disorder

Episodic (about twice weekly) periods of intense anxiety (panic attacks)


Cardiac and respiratory symptoms and the conviction that one is about to die or lose one’s mind
Sudden onset of symptoms, increasing in intensity over a period of approximately 10 min, and lasting about 30 min (attacks
rarely follow a fixed pattern)
Attacks can be induced by administration of sodium lactate or CO2 (see Chapter 5)
Strong genetic component
More common in young women in their 20s
Anxiety Disorders: Phobias (Specific Phobia, Social Anxiety Disorder, and Agoraphobia)

In specific phobia, there is an irrational fear of certain things (e.g., elevators, snakes, or closed-in areas)
HELP OTHERS SO THAT GOD WILL HELP YOU.

In social anxiety disorder, there is an exaggerated fear of embarrassment in social situations (e.g., public speaking, eating in
public, using public restrooms)
In agoraphobia, there is an intense fear associated with being in open places or situations in which one cannot escape or
obtain help
Because of the fear, the person avoids the object or situation
Avoidance leads to social and occupational impairment
Anxiety Disorders: Generalized Anxiety Disorder

Persistent anxiety symptoms including hyperarousal and worrying lasting 6 mos or more
Gastrointestinal symptoms are common
Symptoms are not related to a specific person or situation (i.e., free-floating anxiety)
Commonly starts during the 20s
Obsessive–Compulsive Disorder (OCD) and Related Disorders: Body Dysmorphic Disorder, Hoarding Disorder, and
Hair-Pulling Disorder (Trichotillomania)

Recurring, intrusive feelings, thoughts, and images (obsessions) that cause anxiety
Anxiety is relieved in part by performing repetitive actions (compulsions)
A common obsession is avoidance of hand contamination and a compulsive need to wash the hands after touching things
Obsessive doubts lead to compulsive checking (e.g., of gas jets on the stove) and counting of objects, obsessive need
for symmetry leads to compulsive ordering and arranging, and obsessive concern about discarding valuables leads to
compulsive hoarding (hoarding disorder: A distinct diagnosis in DSM-5)
Patients usually have insight (i.e., they realize that these thoughts and behaviors are irrational and want to eliminate them)
Usually start in early adulthood but may begin in childhood
OCD is increased in first-degree relatives of Tourette disorder patients and both disorders also involve the caudate nucleus
Body dysmorphic disorder involves excessive focus on a minor or imagined physical defect; the symptoms are not accounted
for by anorexia nervosa
Hair-pulling disorder involves a strong need to pull out one’s own hair. It may also involve hair eating which can result in
RISE USMLE NEPAL

bezoars (hair balls) which can cause intestinal blockage


Trauma- and Stress-Related Disorders (Posttraumatic Stress Disorder [PTSD], Acute Stress Disorder [ASD], Adjustment
Disorder, Reactive Attachment Disorder, and Disinhibited Social Engagement Disorder)

Symptoms occurring after a catastrophic (life-threatening or potentially fatal event, e.g., war, house fire, serious accident,
rape, robbery) affecting the patient or the patient’s close relative or friend
Symptoms can be divided into four types:
(1) Reexperiencing (e.g., intrusive memories of the event [flashbacks] and nightmares)
(2) Hyperarousal (e.g., anxiety, increased startle response, impaired sleep, hypervigilance)
(3) Emotional numbing (e.g., difficulty connecting with others)
(4) Avoidance (e.g., survivor’s guilt, dissociation, and social withdrawal)
In PTSD, symptoms last for more than 1 mo (sometimes years) and may have a delayed onset
In ASD, symptoms last only between 2 d and 4 weeks
In adjustment disorder, there are emotional symptoms (e.g., anxiety, depression, or conduct problems) causing social, school,
or work impairment occurring within 3 mos and lasting less than 6 mos after a serious life event (e.g., divorce, bankruptcy,
changing residence) but do not meet full criteria for a mood or anxiety disorder
In adjustment disorder, symptoms can persist for more than 6 mos in the presence of a chronic stressor
Adjustment disorder is not diagnosed if the symptoms represent typical bereavement
Reactive attachment disorder and disinhibited social engagement disorder involve constellations of disturbances in social
relatedness due to abnormal rearing (e.g., being raised in an orphanage), see also Chapter 1
Chapter 13 Anxiety Disorders, Somatic Symptom Disorders, and Related Conditions 135

II. SOMATIC SYMPTOM AND RELATED DISORDERS

A. Characteristics and classification


1. Som atic sym p tom an d related disorders are ch aracterized by physical symptoms that cause
distress and disrupt daily life .
2. Th e p atien t th in ks th at th e sym p tom s h ave an organ ic cau se, bu t th e sym p tom s are
believed to h ave a psychological component an d th u s m ay be u n con sciou s exp ression s o
u n accep table eelin gs (see Ch ap ter 6).
3. Th e DSM-5 categories o som atic sym p tom an d related d isord ers an d th eir ch aracteristics
HELP OTHERS SO THAT GOD WILL HELP YOU.

are listed in Table 13.2.

B. Differential diagnosis
1. Th e m ost im p ortan t di eren tial diagn osis o som atic sym p tom an d related disorders is
unidentified organic disease .
2. Factitiou s disorder (see below), m alin gerin g ( akin g or eign in g illn ess), an d m asked
dep ression (see Ch ap ter 12) also m u st be exclu ded .

C. Management
1. E ective strategies or m an agin g p atien ts with som atic sym p tom an d related disorders
in clu de:
a. Form in g a good physician–patient relationship (e.g., sch ed u lin g regu lar m on th ly
ap p oin tm en ts, p rovidin g reassu ran ce).
b. Providin g a multidisciplinary approach in clu din g oth er m ed ical p ro ession als (e.g., p ain
m an agem en t, m en tal h ealth services).
c. Iden ti yin g an d decreasing the social difficulties in th e p atien t’s li e th at m ay in ten si y
th e sym p tom s.
2. Antianxiety an d antidepressant agents, hypnosis , an d behavioral relaxation therapy also
m ay be u se u l.

t a b l e 13.2 DSM-5 Classification of Somatic Symptom and Related Disorders

Classification (Previous
RISE USMLE NEPAL

DSM Terminology) Characteristics

Somatic symptom disorder One or more physical symptoms that disrupt daily life with excessive focus on the
symptoms
Being symptomatic for more than 6 mo
Illness anxiety disorder Exaggerated concern with health and illness lasting at least 6 mos in the absence of
somatic symptoms
Concern persists despite medical evaluation and reassurance
Care-seeking type goes to many different doctors seeking help (“doctor shopping”)
Conversion disorder Sudden, dramatic loss of sensory or motor function (e.g., blindness, paralysis), often
(functional neurological associated with a stressful life event
symptom disorder) Patients appear relatively unworried (“la belle indifférence”)
Somatic symptom disorder Intense acute or chronic pain not explained completely by physical disease and closely
with predominant pain associated with psychological stress
Onset usually in the 30s and 40s
136 BRS Behavioral Science

III. FACTITIOUS DISORDER, FACTITIOUS DISORDER IMPOSED ON


ANOTHER, AND MALINGERING
A. Characteristics
1. Wh ile in d ividu als with som atic sym p tom disorders tru ly believe th at th ey are ill, p atien ts
with actitiou s disorders an d m alin gerin g feign mental or physical illness , or actu ally
induce physical illness in th em selves or oth ers or p sych ological gain ( actitiou s disorder)
or tan gib le gain (m alin gerin g) (Tab le 13.3).
2. Patien ts with actitiou s d isord er o ten h ave worked in th e m edical ield (e.g., n u rses, tech -
HELP OTHERS SO THAT GOD WILL HELP YOU.

n ician s) an d kn ow h ow to p ersu asively sim u late an illn ess.


3. Malin gerin g is n ot a p sych iatric disorder.

B. Feigned symptoms m ost com m on ly in clu d e abd om in al p ain , ever (by h eatin g th e th erm om -
eter), blood in th e u rin e (by ad din g b lood rom a n eedlestick), in du ction o tach ycardia (by
d ru g ad m in istration ), skin lesion s (by in ju rin g easily reach ed areas), an d seizu res.

C. Wh en con ron ted by th e p h ysician with th e act th at n o organ ic cau se can b e ou n d,


p atien ts with actitiou s disorder or p atien ts wh o are m alin gerin g typ ically becom e angry
and abruptly leave the situation.

D. Factitious thyrotoxicosis (th yrotoxicosis actitia) can b e d iagn osed wh en an in d ivid u al


secretly u ses exogen ou s th yroid h orm on e in order to lose weigh t, even th ou gh th e p atien t
d oes n ot seek to gain atten tion or bein g ill.

t a b l e 13.3 Factitious Disorder, Factitious Disorder Imposed on Another, and Malingering


Disorder Characteristics

Factitious disorder Conscious simulation of physical or psychiatric illness


Aim is to gain attention for being “sick”
Undergoes unnecessary medical and surgical procedures
Has a “grid abdomen” (multiple crossed scars from repeated non-necessary
surgeries)
Factitious disorder imposed on another Conscious simulation of illness in another person, typically in a child by a
parent, to obtain attention from medical personnel
Is a form of child abuse (see Chapter 18) because the child undergoes
RISE USMLE NEPAL

unnecessary medical and surgical procedures


Must be reported to state child welfare authorities
Malingering Conscious simulation or exaggeration of physical or psychiatric illness for
financial (e.g., insurance settlement) or other obvious gain (e.g., avoiding
incarceration)
Avoids treatment by medical personnel
Health complaints cease as soon as the desired gain is obtained
Review Test

Directions: Each o th e n u m b ered item s or in com p lete statem en ts in th is section is ollowed by


an swers or by com p letion s o th e statem en t. Select th e one lettered an swer or com p letion th at
is best in each case.
HELP OTHERS SO THAT GOD WILL HELP YOU.

Questions 1–3 4. A 35-year-old wom an wh o was rap ed


5 years ago h as recu rren t vivid m em ories
o th e in cid en t accom p an ied by in ten se
A 23-year-old m edical stu d en t com es to th e
an xiety. Th ese m em ories requ en tly in tru de
em ergen cy room with elevated h eart rate,
du rin g h er daily activities, an d n igh tm ares
sweatin g, an d sh ortn ess o b reath . Th e stu -
abou t th e even t o ten wake h er. Her
den t is con vin ced th at sh e will soon su o-
sym p tom s in ten si ed wh en a coworker was
cate. Th e sym p tom s started su dd en ly d u rin g
raped 2 m on th s ago. O th e ollowin g, th e
a car rid e to sch ool. Th e stu d en t h as h ad ep i-
m ost e ective lon g-term m an agem en t or
sodes su ch as th is on at least th ree p reviou s
th is p atien t is
occasion s over th e p ast 2 weeks an d n ow is
a raid to leave th e h ou se even to go to sch ool. (A) an an tid ep ressan t
Sh e h as n o h istory o asth m a an d , oth er th an (B) a su p p ort grou p
an in creased p u lse rate, p h ysical in din gs are (C) a b en zod iazep in e
u n rem arkable. (D) a b u sp iron e
(E) a β-blocker
1. O th e ollowin g, th e m ost e ective
im m ediate treatm en t or th is p atien t is
(A) an an tid ep ressan t Questions 5 and 6
(B) a su p p ort grou p
(C) a b en zod iazep in e A 45-year-old wom an says th at sh e requ en tly
(D) bu sp iron e eels “n ervou s” an d o ten h as an “u p set stom -
(E) a β-blocker ach ,” wh ich in clu d es h eartb u rn , in digestion ,
an d diarrh ea. Sh e h as h ad th is p rob lem sin ce
2. O th e ollowin g, th e m ost e ective lon g- sh e was 25 years o age an d n otes th at oth er
term m an agem en t or th is p atien t is am ily m em bers also are “ten se an d n ervou s.”
RISE USMLE NEPAL

(A) an an tid ep ressan t


(B) a su p p ort grou p 5. Wh ich o th e ollowin g addition al sign s or
(C) a b en zod iazep in e sym p tom s is th is p atien t m ost likely to sh ow?
(D) bu sp iron e (A) Fligh t o ideas
(E) a β-blocker (B) Hallu cin ation s
(C) Tin glin g in th e extrem ities
3. Th e n eu ral m ech an ism m ost closely (D) Ideas o re eren ce
in volved in th e etiology o th is p atien t’s (E) Neologism s
sym p tom s is
(A) n u cleu s accu m ben s h yp osen sitivity 6. O th e ollowin g, th e m ost e ective lon g-
(B) ven tral tegm en tal h yp ersen sitivity term m an agem en t or th is p atien t is
(C) ven tral tegm en tal h yp osen sitivity (A) alp razolam (Xan ax)
(D) locu s ceru leu s h yp ersen sitivity (B) p sych oth erapy
(E) p erip h eral au ton om ic h yp ersen sitivity (C) p rop ran olol (In deral)
(D) bu sp iron e (Bu Sp ar)
(E) diazep am (Valiu m )

137
138 BRS Behavioral Science

7. A 39-year-old wom an claim s th at sh e 10. In th is situ ation , wh at is th e rst th in g


in jured h er h an d at work. Sh e asserts th at th e th e p h ysician sh ou ld d o?
p ain cau sed by h er in ju ry p reven ts h er rom (A) Take th e ch ild aside an d ask h im h ow h e
workin g. Sh e h as n o u rth er h an d p roblem s eels.
a ter sh e receives a $30,000 workers’ (B) Call a p ediatric p u lm on ologist to
com p en sation settlem en t. Th is clin ical determ in e th e cau se o th e dysp n ea.
p resen tation is an exam p le o (C) Call a p ediatric gastroen terologist to
(A) actitiou s disorder determ in e th e cau se o th e abd om in al
(B) con version d isorder p ain .
(C) actitiou s disorder im p osed on an oth er (D) Noti y th e ap p rop riate state social
(D) som atic sym p tom disorder service agen cy to rep ort th e p h ysician’s
HELP OTHERS SO THAT GOD WILL HELP YOU.

(E) p ain disorder su sp icion s.


(F) m alin gerin g (E) Wait u n til th e ch ild’s n ext visit be ore
takin g an y action .
8. Wh ich o th e ollowin g even ts is m ost
likely to resu lt in p osttrau m atic stress
disord er (PTSD)? Questions 11–18
(A) Divorce
(B) Ban kru p tcy For each o th e ollowin g cases, select th e dis-
(C) Diagn osis o diabetes m ellitu s order wh ich b est its th e clin ical p ictu re.
(D) Ch an gin g residen ce
(E) Robb ery at kn i ep oin t (A) Posttrau m atic stress disorder
(B) Illn ess an xiety disorder
(C) Obsessive–com p u lsive disorder
(D) Pan ic disorder
Questions 9 and 10
(E) Som atic sym p tom disorder
(F) Gen eralized an xiety disorder
A 39-year-old wom an takes h er 6-year-old (G) Body dysm orp h ic disorder
son to a p h ysician’s o ice. Sh e says th at th e (H) Con version disorder
ch ild o ten exp erien ces ep isod es o b reath - (I) Sp eci ic p h obia
in g p roblem s an d abdom in al p ain . Th e (J ) Social an xiety disorder
ch ild’s m ed ical record sh ows m an y o ice (K) Adju stm en t disord er
visits an d ou r abd om in al su rgical p roce- (L) Masked dep ression
du res, alth ou gh n o ab n orm alities were ever
ou n d. Ph ysical exam in ation an d laboratory 11. A 45-year-old wom an h as a 20-year
stu d ies are u n rem arkab le. Wh en th e doctor h istory o vagu e p h ysical com p lain ts
con ron ts th e m oth er with th e su sp icion th at in clu din g n au sea, p ain u l m en ses, an d loss
RISE USMLE NEPAL

sh e is abricatin g th e illn ess in th e ch ild , th e o eelin g in h er legs. Ph ysical exam in ation


m oth er an grily grab s th e ch ild an d leaves th e an d laboratory worku p are u n rem arkable.
o ice im m ed iately. Sh e says th at sh e h as always h ad p h ysical
p roblem s bu t h er doctors n ever seem to
9. Th is clin ical p resen tation is an exam p le o iden ti y th eir cau se.
(A) actitiou s disorder
(B) con version d isorder 12. Th ree m on th s a ter m ovin g, a teen ager
(C) actitiou s disorder im p osed on an oth er wh o was orm erly ou tgoin g an d a good
(D) som atic sym p tom disorder stu den t seem s sad, loses in terest in m akin g
(E) p ain disorder rien ds, an d begin s to do p oor work in
(F) m alin gerin g sch ool. His ap p etite is n orm al, an d th ere is
n o eviden ce o su icidal ideation .
Chapter 13 Anxiety Disorders, Somatic Symptom Disorders, and Related Conditions 139

13. A 29-year-old m an exp erien ces su dd en asked abou t th is, h e says th at h e takes a
right-sided h em ip aresis b u t ap p ears lon g tim e b ecau se h e eels com p elled to
u n con cern ed. He rep orts th at ju st b e ore th e wash h im sel in a certain m an n er an d h as
on set o weakn ess, h e saw h is girl rien d with to rep eat th e wh ole p rocess i h e m akes a
an oth er m an . Ph ysical exam in ation ails to m istake. He kn ows th at th is beh avior sou n ds
reveal evid en ce o a m ed ical p roblem . rid icu lou s an d th at it m akes h im late or
sch ool an d oth er activities, bu t h e can n ot
14. A 41-year-old m an says th at h e h as been seem to stop h im sel rom doin g it. Th ere are
“sickly” or m ost o h is li e. He h as seen n o sign i can t m ed ical n din gs.
m an y doctors bu t is an gry with m ost o th em
b ecau se th ey u ltim ately re erred h im or 20. A 22-year-old m an is brou gh t in to
HELP OTHERS SO THAT GOD WILL HELP YOU.

p sych ological h elp. He n ow ears th at h e h as th e em ergen cy room by th e p olice. Th e


stom ach can cer becau se h is stom ach m akes p olicem an tells th e p h ysician th at th e m an
n oises a ter h e eats. Ph ysical exam in ation is was cau gh t wh ile attem p tin g to rob a b an k.
u n rem arkable an d b ody weigh t is n orm al. Wh en th e p olice told h im to reeze an d drop
h is gu n , th e m an drop p ed to th e f oor an d
15. A 41-year-old m an says th at h e h as been cou ld n ot sp eak, bu t rem ain ed con sciou s.
“sickly” or th e p ast 3 m on th s. He ears Wh en th e doctor attem p ts to in terview
th at h e h as stom ach can cer. Th e p atien t h im , th e p atien t rep eated ly alls asleep. Th e
is u n sh aven an d ap p ears th in an d slowed h istory reveals th at th e p atien t’s broth er h as
d own . Ph ysical exam in ation , in clu d in g a n arcolep sy. Wh ich o th e ollowin g best ts
gastroin testin al worku p, is u n rem arkab le th is clin ical p ictu re?
excep t th at th ere is an u n exp lain ed loss o (A) A sleep d isorder
15 p ou n ds sin ce h is last visit 1 year ago. (B) A seizu re disord er
(C) A som atic sym p tom disord er
16. A 28-year-old wom an seeks acial (D) Malin gerin g
recon stru ctive su rgery or h er “saggin g” (E) An en docrin e disord er
eyelid s. Sh e rarely goes ou t in th e d aytim e (F) A actitiou s disord er
b ecau se sh e b elieves th at th is ch aracteristic
m akes h er look “like a gran dm oth er.” On 21. A 12-year-old boy is adm itted to the
p h ysical exam in ation , h er eyelid s ap p ear h ospital with a diagn osis o “pain o un kn own
com p letely n orm al. origin .” His paren ts tell the physician th at
the child has com p lain ed about p ain in
17. A 29-year-old m an is u p set b ecau se h e h is legs or about 1 m on th. Neurologic an d
m u st take a clien t to din n er in a restau ran t. orthop edic exam in ation s ail to iden ti y an y
Althou gh h e kn ows th e clien t well, h e is so p athology. The history reveals that the child
a raid o m akin g a m ess wh ile eatin g th at h e was h ospitalized on two p revious occasion s
RISE USMLE NEPAL

says h e is n ot h u n gry an d sip s rom a glass o or other pain sym p tom s or which n o cause
water in stead o orderin g a m eal. was oun d. A ter 4 days in the hosp ital, the
n urse reports that the ch ild sh ows little
18. A 29-year-old m an tells th e doctor th at eviden ce o pain an d seem s “rem arkably
h e h as b een so “n ervou s” an d u p set sin ce con ten t.” Sh e also rep orts th at sh e ou n d a
h is girl rien d broke u p with h im 1 m on th m edical textbook in the boy’s bedside table
ago th at h e h ad to qu it h is job an d stay at with a bookm ark in the section en titled
h om e. Th e m an h as n o h istory o m ed ical “skeletal pain o un kn own origin .” Which
or p sych iatric d isord ers, alth ou gh h is ath er o the ollowin g best describes sym ptom
h as a h istory o b ip olar d isord er, h is m oth er p roduction an d m otivation in th is case?
h as a h istory o alcoh olism , an d h is you n ger
(A) Sym p tom p rodu ction con sciou s,
b roth er was in reh ab or d ru g ad d iction th e
m otivation p rim arily con sciou s
p reviou s year.
(B) Sym p tom p rodu ction u n con sciou s,
m otivation p rim arily con sciou s
19. A 15-year-old b oy is b rou gh t to th e
(C) Sym p tom p rodu ction con sciou s,
d octor by h is m oth er or “stran ge b eh avior.”
m otivation p rim arily u n con sciou s
Sh e rep orts th at h er son is o ten late or
(D) Sym p tom p rodu ction u n con sciou s,
sch ool becau se h e sp en ds m ore th an an
m otivation p rim arily u n con sciou s
h ou r in th e sh ower every m orn in g. Wh en
140 BRS Behavioral Science

22. A 40-year-old m an tells h is p h ysician 24. A 35-year-old n u rse is brou gh t to th e


th at h e is o ten late or work b ecau se h e h as em ergen cy room a ter ain tin g ou tside o
d i cu lty wakin g u p on tim e. He attribu tes a p atien t’s room . Th e n u rse n otes th at sh e
th is p rob lem to th e act th at h e gets ou t o h as h ad ain tin g ep isod es b e ore an d th at
b ed rep eated ly d u rin g th e n igh t to rech eck sh e o ten eels weak an d sh aky. Laboratory
th e locks on th e doors an d to b e su re th e stu dies reveal h yp oglycem ia, very h igh
gas jets on th e stove are tu rn ed o . His in sulin level, an d su p p ressed p lasm a C
laten ess is exacerb ated by h is n eed to cou n t p ep tid e. Wh ich o th e ollowin g b est ts th is
all o th e tra c ligh ts alon g th e rou te. I h e clin ical p ictu re?
su sp ects th at h e m issed a ligh t, h e b ecom es (A) A sleep d isorder
qu ite an xiou s an d m u st th en go b ack an d (B) A seizu re disord er
HELP OTHERS SO THAT GOD WILL HELP YOU.

recou n t th em all. Ph ysical exam in ation an d (C) A som atic sym p tom disord er
lab oratory stu d ies are u n rem arkable. O (D) Malin gerin g
th e ollowin g, th e m ost e ective lon g-term (E) An en docrin e disord er
m an agem en t or th is p atien t is m ost likely (F) A actitiou s disord er
to b e
(A) an an tid ep ressan t 25. A h ealth y 45-year-old m an join s an
(B) an an tip sych otic on lin e illn ess su p p ort grou p, claim in g th at
(C) a b en zod iazep in e h e h as Stage 4 p rostate can cer. Su bsequ en tly,
(D) a b u sp iron e h e gets m an y e-m ails rom th e grou p
(E) a β-blocker m em bers exp ressin g em p ath y. He th en join s
an oth er on lin e su p p ort grou p claim in g to
23. Th e m oth er o a 4-year-old ch ild with h ave in op erable brain can cer. Som eon e
diabetes takes th e ch ild to th e p ed iatrician to in th e secon d su p p ort grou p recogn izes
“b e ch ecked” at least 3 tim es p er week. Sh e h im an d con ron ts h im with th e act th at
watch es th e ch ild at all tim es an d does n ot h e claim s to h ave two atal illn esses. In
let h im p lay ou tsid e. Sh e also m easu res an d resp on se, th e m an ap ologizes an d says h e
rem easu res h is ood p ortion s th ree tim es at d oes n ot kn ow wh y h e m akes u p stories
every m eal. Th e m oth er u n derstan ds th at abou t bein g sick. He th en resign s rom both
th is b eh avior is excessive b u t states th at su p p ort grou p s. However, a m on th later,
sh e is u n able to stop doin g it. Th e m ost h e join s an oth er su p p ort grou p claim in g
ap prop riate p h arm acological treatm en t or ( alsely) to h ave m u ltip le sclerosis. Th is m an
th is m oth er is is sh owin g eviden ce o
(A) diazep am (A) actitiou s disorder
(B) bu sp iron e (B) con version disorder
(C) clom ip ram in e (C) adju stm en t disord er
(D) h alop eridol (D) som atic sym p tom disorder
RISE USMLE NEPAL

(E) p rop ran olol (E) m alin gerin g


An swers an d Exp lan ation s

Typical Board Question


The answer is D. Th is 8-year-old ch ild is sh owin g eviden ce o PTSD. PTSD can be diagn osed in
close relatives (th e ch ild in th is case) o in dividu als wh o h ave aced li e-th reaten in g situ ation s.
Th ere is n o cu rren t eviden ce th at th is ch ild h as social an xiety disorder ( ear o em barrassm en t
in a p u blic situ ation ) or gen eralized an xiety disorder (at least 6 m on th s o an xiety sym p tom s
HELP OTHERS SO THAT GOD WILL HELP YOU.

with ou t a sp eci ic p recip itan t). Th e ch ild’s tiredn ess du rin g th e day is exp lain ed by h is requ en t
n igh ttim e awaken in gs—it is n ot d u e to n arcolep sy.

1. The answer is C. 2. The answer is A. 3. The answer is D. This patient is showin g eviden ce o pan ic
disorder with agoraphobia. Pan ic disorder is characterized by pan ic attacks, which in clude
in creased heart rate, dizzin ess, sweatin g, shortn ess o breath, an d ain tin g, an d the con viction
that one is about to die. This youn g wom an has also developed a ear o leavin g the house
(agoraphobia), which occurs in som e patien ts with pan ic disorder. While the m ost e ective
im m ediate treatm en t or this patien t is a ben zodiazepin e because it works quickly, the m ost
e ective lon g-term (m ain ten an ce) m an agem en t is an an tidepressan t, particularly a selective
seroton in reuptake in hibitor (SSRI) such as paroxetin e (Paxil). The n eural etiology m ost closely
in volved in pan ic disorder with agoraphobia is hypersen sitivity o the locus ceruleus.
4. The answer is B. Th is p atien t is m ost likely to h ave p osttrau m atic stress disorder (PTSD).
Th is d isorder, wh ich is ch aracterized by sym p tom s o an xiety an d in tru sive m em ories an d
n igh tm ares o a li e-th reaten in g even t su ch as rap e, can last or m an y years in ch ron ic orm
an d m ay h ave been in ten si ied in th is p atien t by reexp erien cin g h er own rap e th rou gh
th e rap e o h er coworker. Th e m ost e ective lon g-term m an agem en t or th is p atien t is a
su p p ort grou p, in th is case a rap e su rvivor’s grou p. Ph arm acologic treatm en t su ch as an
an tid ep ressan t is u se u l as an ad ju n ct to p sych ological m an agem en t in PTSD.
5. The answer is C. 6. The answer is D. Th is p atien t is m ost likely to have gen eralized an xiety
disorder (GAD). Th is disorder, wh ich in clu des ch ron ic an xiety an d, o ten , gastroin testin al
sym p tom s is m ore com m on in wom en an d o ten starts in th e 20s. Gen etic actors are seen in
th e observation th at oth er am ily m em bers have sim ilar p roblem s with an xiety. Addition al
sign s or sym p tom s o an xiety th at th is p atien t is likely to sh ow in clu de tin glin g in th e
RISE USMLE NEPAL

extrem ities an d n u m bn ess arou n d th e m ou th , o ten resu ltin g rom h yp erven tilation . Fligh t
o ideas, h allu cin ation s, ideas o re eren ce, an d n eologism s are p sychotic sym p tom s, wh ich
are n ot seen in th e an xiety disorders or th e som atic sym p tom d isorders. O th e ch oices,
th e m ost e ective lon g-term m an agem en t or th is p atien t is busp iron e because, un like th e
ben zodiazep in es alp razolam an d diazep am , it d oes n ot cau se dep en den ce or with drawal
sym p tom s with lon g-term u se. Th e an tidep ressan ts ven la axin e an d du loxetin e an d SSRIs
also are e ective or lon g-term m an agem en t o GAD. Psychoth erapy an d β-blockers can be
u sed as adjun cts to treat GAD but are n ot th e m ost e ective lon g-term treatm en ts.
7. The answer is F. Th is p resen tation is an exam p le o m alin gerin g, eign in g illn ess or obviou s
gain (th e $30,000 workers’ com p en sation settlem en t). Evid en ce or th is is th at th e wom an
h as n o u rth er h an d p rob lem s a ter sh e receives th e m on ey. In con version disorder, som atic
sym p tom disorder, actitiou s d isord er, an d actitiou s disord er im p osed on an oth er, th ere is
n o obviou s or m aterial gain related to th e sym p tom s.
8. The answer is E. Robbery at kn i ep oin t, a li e-threaten in g even t, is m ost likely to result in
p osttrau m atic stress disorder (PTSD). Wh ile li e even ts such as divorce, ban krup tcy, illn ess,
an d ch an gin g residen ce are stress ul, th ey are rarely li e threaten in g. Psychological sym ptom s
occu rrin g a ter such less severe even ts m ay resu lt in adju stm en t disorder, n ot PTSD.

141
142 BRS Behavioral Science

9. The answer is C. 10. The answer is D. Th is p resen tation is an exam p le o actitiou s disorder
im p osed on an oth er. Th e m oth er h as eign ed th e ch ild’s illn ess (ep isodes o breath in g
p roblem s an d abdom in al p ain ) or atten tion rom m edical p erson n el. Th is akin g h as
resu lted in ou r abdom in al su rgical procedu res in wh ich n o abn orm alities were ou n d.
Sin ce sh e kn ows sh e is lyin g, th e m oth er will becom e an gry an d lee wh en con ron ted
with th e tru th . Th e irst th in g th e p h ysician m u st do is to n oti y th e state social service
agen cy sin ce actitiou s d isorder im p osed on an oth er is a orm o ch ild ab u se. Waitin g u n til
th e ch ild’s n ext visit be ore actin g cou ld resu lt in th e ch ild’s u rth er in ju ry or even death .
Callin g in sp ecialists m ay b e ap p rop riate a ter th e p h ysician rep orts h is su sp icion s to
th e state. It is n ot ap p rop riate to take th e ch ild aside an d ask h im h ow h e really eels. He
p robably is n ot aware o h is m oth er’s beh avior.
HELP OTHERS SO THAT GOD WILL HELP YOU.

11. The answer is E. Th is wom an with a 20-year h istory o u n exp lain ed vagu e an d ch ron ic
p h ysical com p lain ts p rob ab ly h as som atic sym p tom d isorder. Th is can be d istin gu ish ed
rom illn ess an xiety d isorder, wh ich is an exaggerated worry abou t n orm al p h ysical
sen sation s an d m in or ailm en ts (see also an swers to Qu estion s 12–18).
12. The answer is K. Th is teen ager, wh o was orm erly ou tgoin g an d a good stu den t an d n ow
seem s sad, loses in terest in m akin g rien ds, an d b egin s to do p oor work in sch ool, p rob ably
h as adju stm en t disorder (with dep ressed m ood). It is likely th at h e is h avin g p roblem s
adju stin g to h is n ew sch ool. In con trast to ad ju stm en t d isord er, in m asked dep ression ,
th e sym p tom s are m ore severe an d o ten in clu de sign i ican t weigh t loss or su icidality (see
Ch ap ter 12).
13. The answer is H. Th is m an , wh o exp erien ces a su dd en n eu rological sym p tom triggered by
seein g h is girl rien d with an oth er m an , is sh owin g eviden ce o con version disorder. Th is
disord er is ch aracterized by an ap p aren t lack o con cern abou t th e sym p tom s (i.e., “la b elle
in di éren ce”).
14. The answer is B. Th is m an , wh o says th at h e h as b een “sickly” or m ost o h is li e an d ears
th at h e h as stom ach can cer, is sh owin g eviden ce o illn ess an xiety disorder exaggerated
con cern over n orm al p h ysical sen sation s (e.g., stom ach n oises) an d m in or ailm en ts. Th ere
are n o p h ysical in d in gs n or obviou s evid en ce o d ep ression in th is p atien t.
15. The answer is L. Th is m an p rob ably h as m asked d ep ression . In con trast to th e m an with
illn ess an xiety d isord er in th e p reviou s qu estion , evid en ce or d ep ression in th is p atien t
in clu des th e act th at, in ad d ition to th e som atic com p lain ts, h e sh ows sym p tom s o
d ep ression (e.g., h e is n ot groom ed , ap p ears slowed down [p sych om otor retardation ], an d
h as lost a sign i ican t am ou n t o weigh t).
RISE USMLE NEPAL

16. The answer is G. Th is wom an p robably h as b od y d ysm orp h ic d isord er, wh ich is
ch aracterized by overcon cern ab ou t a p h ysical eatu re (e.g., “saggin g” eyelids in th is case),
desp ite n orm al ap p earan ce.
17. The answer is J . Th is m an p robab ly h as social an xiety disord er. He is a raid o
em barrassin g h im sel in a p u blic situ ation (e.g., gettin g ood on h is ace wh ile eatin g
d in n er in ron t o oth ers in a restau ran t).
18. The answer is K. Th e m ost likely exp lan ation or th is clin ical p ictu re th at in clu d es
sym p tom s o an xiety th at begin a ter a li e stressor (e.g., a rom an tic break-u p ) is
adju stm en t disord er (with an xiety). Th e ab sen ce o a p reviou s h istory an d th e b rie
du ration in d icates th at th is is n ot an an xiety d isord er, an d th e act th at th e stressor was n ot
li e-th reaten in g ru les ou t PTSD an d ASD. Th e am ily h istory is n ot likely to be related to
th is p atien t’s sym p tom s in th is case.
19. The answer is C. Th is 15-year-old wh o m u st wash h im sel in a certain m an n er each day
is sh owin g eviden ce o OCD. OCD is a d isord er in wh ich on e is com p elled to en gage in
rep etitive n on p rodu ctive b eh avior th at, as in th is p atien t, im p airs u n ction (e.g., th e
p atien t is late or sch ool an d activities). Th e act th at th is teen ager h as in sigh t, th at is, h e
kn ows th at wh at h e is d oin g is “rid icu lou s,” also is ch aracteristic o OCD.
Chapter 13 Anxiety Disorders, Somatic Symptom Disorders, and Related Conditions 143

20. The answer is D. Wh en th ere is in an cial or oth er obviou s gain to be obtain ed rom an
illn ess, th e p ossibility th at th e p erson is m alin gerin g m u st be con sidered. In th is case, a
m an wh o h as com m itted a crim e is eign in g sym p tom s o n arcolep sy to avoid p rosecu tion .
Kn owledge o th e details o h is broth er’s illn ess h as tau gh t h im h ow to eign th e catap lexy
(su d den loss o m otor con trol) an d d aytim e sleep in ess associated with n arcolep sy (see
Ch ap ter 7).
21. The answer is C. Th is clin ical p resen tation is an exam p le o actitiou s d isord er (n ote: Most
p sych iatric d iagn oses d isorders can also be m ade in ch ildren ). In con trast to p atien ts with
som atic sym p tom disorders wh o really believe th at th ey are ill, p atien ts with actitiou s
disord er are con sciou s o th e act th at th ey are eign in g th eir illn ess. Pain is on e o th e m ost
com m on ly eign ed sym p tom s, an d th is p atien t’s n igh ttim e readin g is p rovidin g h im with
HELP OTHERS SO THAT GOD WILL HELP YOU.

sp eci ic kn owled ge o h ow to eign th e sym p tom s realistically. Alth ou gh h e is con sciou sly
p rodu cin g h is sym p tom s, th is boy is n ot receivin g tan gible ben e it or h is beh avior. Th u s,
in con trast to in dividu als wh o are con sciou sly eign in g illn ess or obviou s gain , th at is,
m alin gerin g (see also an swer to Qu estion 20), th e m otivation or th is p atien t’s p ain - akin g
beh avior is p rim arily u n con sciou s.
22. The answer is A. Th is m an’s rep eated ch eckin g an d cou n tin g beh avior in dicates th at
h e h as OCD. Th e m ost e ective lon g-term m an agem en t or OCD is an an tidep ressan t,
p articu larly a selective seroton in reu p take in h ibitor (SSRI) su ch as lu voxam in e (Lu vox) or
a h eterocyclic agen t su ch as clom ip ram in e. An tian xiety agen ts su ch as ben zodiazep in es
(e.g., diazep am ) an d bu sp iron e an d β-b lockers su ch as p rop ran olol are m ore com m on ly
u sed or th e m an agem en t o acu te or ch ron ic an xiety. An tip sych otic agen ts su ch as
h alop eridol m ay be u se u l as adju n cts bu t do n ot su bstitu te or SSRIs or clom ip ram in e in
OCD.
23. The answer is C. Th e n eed to ch eck an d rech eck th e ch ild’s p ortion s an d rep eatedly take
h im to th e doctor in dicates th at, as in Qu estion 22, th is p atien t is sh owin g sym p tom s o
OCD. Th e act th at sh e kn ows th at h er b eh avior is excessive (“in sigh t”) is typ ical o p atien ts
with OCD. As n oted in An swer 22, th e m ost e ective lon g-term m an agem en t or OCD is an
SSRI or a cyclic an tid ep ressan t su ch as clom ip ram in e.
24. The answer is F. Th e triad o h yp oglycem ia, very h igh in su lin level, an d su p p ressed
p lasm a C p ep tide in d icate th at th is n u rse h as sel -adm in istered in su lin , a situ ation kn own
as actitiou s h yp erin su lin ism . In h yp erin su lin ism du e to m edical cau ses, or exam p le,
in su lin om a (p an creatic β-cell tu m or), p lasm a C p ep tide is typ ically in creased, n ot
d ecreased . Factitiou s d isord er is m ore com m on in p eop le, like th is wom an , associated
with th e h ealth p ro ession s. Th ere is n o eviden ce in th is wom an o a sleep disorder, an xiety
RISE USMLE NEPAL

d isord er, som atic sym p tom d isorder, or en docrin e disorder su ch as diabetes. Becau se th ere
is n o obviou s or p ractical gain or th is wom an in bein g ill, m alin gerin g is u n likely.
25. The answer is A. Like th e wom an in th e p reviou s qu estion , th is h ealth y m an is sh owin g
evid en ce o actitiou s d isord er. Peop le with actitiou s disorder p u rp osely eign illn ess
(in con trast to con version d isord er an d som atic sym p tom disorder in wh ich sym p tom
p rodu ction is u n con sciou s) in order to be con sidered a sick p erson by oth ers (see also
an swer to Qu estion 21). Th ere is n o tan gib le gain to be ob tain ed rom assu m in g th e “sick
role” as th ere is in m alin gerin g. Th ere is n o eviden ce o a sp eci ic li e stressor in itiatin g
p sych ological sym p tom s as th ere wou ld be in adju stm en t disorder.
Neu rocogn itive,
c ha pte r
14 Person ality, Dissociative,
an d Eatin g Disorders
HELP OTHERS SO THAT GOD WILL HELP YOU.

Typical Board Question


A 16-year-old girl, wh o h as h ad typ e 1 d iab etes m ellitu s or th e p ast 3 years, is brou gh t to th e
p h ysician becau se o a 10-kg (22-p ou n ds) weigh t loss du rin g th e p ast 6 m on th s. Th e p atien t
rep orts th at sh e is eelin g in e, an d sh e does n ot th in k th at an yth in g is wron g. Sh e says th at
sh e is h ap py to h ave lost th e weigh t, an d sh e wou ld like to lose m ore weigh t. Sh e is 160 cm
(5 eet 3 in ch es) tall an d n ow weigh s 40 kg (87 p ou n d s); BMI is 16 kg/ m 2. Ph ysical exam i-
n ation sh ows n o oth er ab n orm alities. Lab oratory stu d ies sh ow a h em oglob in A1c o 8.4%;
6 m on th s ago, it was 5.8%. Wh ich o th e ollowin g p atien t b eh aviors m ost likely led to th is
teen ager’s weigh t loss?
(A) Restrictin g calorie con su m p tion
(B) Overu se o laxatives
(C) Decreasin g th e am ou n t o sel -ad m in istered in su lin
(D) Sel -in d u ced vom itin g a ter m eals
(E) Startin g an in ten se aerobic exercise p rogram
(See “An sw ers an d Explan ation s” at th e en d of th e ch apter.)

I. NEUROCOGNITIVE DISORDERS
RISE USMLE NEPAL

A. General characteristics
1. Neu rocogn itive disord ers (NCDs) in volve p roblem s in memory, orien tation , level o con-
sciousness , an d oth er in tellectu al u n ction s.
a. Th ese di icu lties are du e to abn orm alities in n eu ral ch em istry, stru ctu re, or p h ysiol-
ogy originating in the brain or secondary to systemic illness .
b. Patien ts with NCDs m ay also sh ow psychiatric symptoms (e.g., dep ression , an xiety,
h allu cin ation s, delu sion s, an d illu sion s; see Table 8.2), wh ich are secon dary to th e n eu -
rocogn itive p rob lem s.
c. Th e NCDs in clu de:
(1) Delirium.
(2) Major and mild NCD (dementia) du e to Alzheimer’s disease , ron totem p oral lobar
degen eration , Lewy b ody disease, vascu lar disease, trau m atic brain in ju ry, su b-
stan ce/ m edication -in d u ced, HIV in ection , p rion disease (e.g., Creu tz eld t-Jakob
disease), Parkin son’s d isease, or Hu n tin gton’s d isease.
(3) NCD due to another medical condition or multiple etiologies.
2. Ch aracteristics an d etiologies o m an y o th ese disorders can be ou n d in Table 14.1 an d
below.

144
Chapter 14 Neurocognitive, Personality, Dissociative, and Eating Disorders 145

t a b l e 14.1 Characteristics and Etiologies of Neurocognitive Disorders


Neurocognitive Disorder Due to
Characteristic Delirium Dementia Another Medical Condition

Hallmark Impaired consciousness Loss of memory and Loss of memory with few other
intellectual abilities cognitive problems
Etiology CNS disease (e.g., Huntington’s Alzheimer’s disease Thiamine deficiency due to long-term
or Parkinson’s disease) Vascular disease alcohol use, leading to destruction
of mediotemporal lobe structures
(e.g., mammillary bodies)
CNS trauma CNS trauma Temporal lobe trauma, vascular
disease, or infection (e.g., herpes
HELP OTHERS SO THAT GOD WILL HELP YOU.

simplex encephalitis)
CNS infection (e.g., meningitis, CNS infection (e.g., HIV or
HIV) Creutzfeldt-J akob
Systemic disease (e.g., hepatic, disease)
cardiovascular) NCD with Lewy bodies
High fever Pick’s disease
Substance use (frontotemporal
Substance withdrawal dementia)
Prescription drug overdose
(e.g., atropine)
Occurrence More common in children and More common in the More common in patients with a
the elderly elderly history of alcohol use
Most common etiology of Seen in about 20% of those
psychiatric symptoms in over age 85
medical and surgical hospital
units
Associated physical Acute medical illness No medical illness No medical illness
findings Autonomic dysfunction Little autonomic Little autonomic dysfunction
Abnormal EEG (fast wave dysfunction Normal EEG
activity or generalized Normal EEG
slowing)
Associated Impaired consciousness Normal consciousness Normal consciousness
psychological Illusions, delusions (often Psychotic symptoms Psychotic symptoms uncommon
findings paranoid) or hallucinations uncommon in early in early stages
(often visual and stages Depressed mood
disorganized) Depressed mood Little diurnal variability
“Sundowning” (symptoms much “Sundowning” Confabulation (untruths told to hide
worse at night) Personality changes memory loss)
Anxiety with psychomotor in early stages (in
agitation frontotemporal NCD)
Course Develops quickly Develops slowly Develops slowly
RISE USMLE NEPAL

Fluctuating course with lucid Progressive downhill Progressive downhill course if


intervals course alcohol use continues
Management and Removal of the underlying No effective treatment, No effective treatment, rarely
prognosis medical problem will allow rarely reversible reversible
the symptoms to resolve Pharmacotherapy and Pharmacotherapy and supportive
Provide a structured supportive therapy therapy to manage associated
environment to treat associated psychiatric symptoms
Increase orienting stimuli psychiatric symptoms Vitamin B1 for acute symptoms if
Delirium must be ruled out Acetylcholinesterase due to alcohol use
before dementia can be inhibitors and NMDA
diagnosed receptor antagonists
(for Alzheimer’s disease)
Antihypertensive or
anticlotting agent (for
vascular disease)
Provide a structured
environment
CNS, central nervous system; HIV, human immunodeficiency virus; EEG, electroencephalogram; NMDA, N-methyl-d -aspartate.
146 BRS Behavioral Science

B. Delirium
1. Deliriu m is a syn d rom e th at in clu d es confusion an d clouding of consciousness th at resu lt
rom cen tral n ervou s system im p airm en t.
2. It u su ally occu rs in th e cou rse o an acute medical illness su ch as en cep h alitis or m en in -
gitis bu t is also seen in dru g u se an d with drawal, p articu larly with drawal rom alcoh ol
(“deliriu m trem en s”).
3. It is com m on in surgical and coronary intensive care u n its an d in elderly debilitated patients .

C. Major and mild NCDs (dementia)


1. Dem en tia in volves th e gradual loss of intellectual abilities with ou t im p airm en t o
con sciou sn ess.
HELP OTHERS SO THAT GOD WILL HELP YOU.

2. Alzheimer’s disease is th e most common type o dem en tia (50%–65% o all dem en tias; th at
cau sed by vascular disease m akes u p 10%–15% o dem en tias). Th e term mild NCD is u sed
wh en th e in tellectu al de icits do n ot sign i ican tly in ter ere with th e p atien t’s u n ction in g.

D. Alzheimer’s disease
1. Diagnosis
a. Patien ts with Alzh eim er’s d isease sh ow a gradual loss of memory and intellectual abili-
ties . Th eir p sych iatric sym p tom s in clu d e in ability to con trol im p u lses an d lack o ju dg-
m en t as well as d ep ression an d an xiety.
b. Later in th e illn ess, sym p tom s in clu d e con u sion an d p sych osis th at p rogress to com a
an d death (usually within 8–10 years of diagnosis).
c. For p atien t m an agem en t an d progn osis, it is im p ortan t to m ake th e distin ction between
Alzheimer’s disease an d b oth pseudodementia (dep ression th at m im ics dem en tia) an d
beh avioral ch an ges associated with normal aging (Table 14.2).
2. Genetic associations in Alzh eim er’s d isease in clu d e:
a. Abn orm alities o chromosome 21 (Down’s syn drom e p atien ts u ltim ately develop
Alzh eim er’s disease).
b. Abn orm alities o chromosomes 1 and 14 (sites o th e p resen ilin 2 an d p resen ilin 1 gen es,
resp ectively) im p licated p articu larly in early onset Alzh eim er’s d isease (i.e., occu rrin g
b e ore th e age o 65).
c. Possession o at least on e copy o th e ap olip op rotein E4 (apoE4) gen e on chrom osom e 19.
d. Gen d er—th ere is a h igh er occu rren ce o Alzh eim er’s disease in women.

t a b l e 14.2 Memory Problems in the Elderly: A Comparison of Alzheimer’s Disease,


Pseudodementia, and Normal Aging
RISE USMLE NEPAL

Condition Etiology Clinical Example Major Manifestations Medical Interventions

Alzheimer’s Brain A 65-y-old former banker Severe memory loss Structured environment
disease dysfunction cannot remember to turn Other cognitive problems Acetylcholinesterase
off the gas jets on the Decrease in IQ inhibitors
stove nor can he name Disruption of normal life NMDA receptor
the object in his hand antagonists
(a comb) Ultimately, nursing home
placement
Pseudodementia Depression of A 65-y-old dentist cannot Moderate memory loss Antidepressants
(depression mood remember to pay her Other cognitive problems Electroconvulsive therapy
that mimics bills. She also appears No decrease in IQ (ECT)
dementia) to be physically “slowed Disruption of normal life Psychotherapy
down” (psychomotor
retardation) and very sad
Normal aging Minor changes A 65-y-old woman forgets Minor forgetfulness No medical intervention
in the normal new phone numbers and Reduction in the ability Practical and emotional
aging brain names but functions well to learn new things support from the
living on her own quickly physician
No decrease in IQ
No disruption of normal life
Chapter 14 Neurocognitive, Personality, Dissociative, and Eating Disorders 147

3. Neurophysiological factors in clu de:


a. Decreased activity o acetylch olin e (Ach ) an d redu ced brain levels o ch olin e acetyl-
tran s erase (i.e., th e en zym e n eeded to syn th esize Ach ; see Ch ap ter 4).
b. Abn orm al p rocessin g o am yloid p recu rsor p rotein .
c. Overstim u lation o th e N-m eth yl- d -asp artate (NMDA) recep tor by glutamate lead in g
to an in lu x o calciu m , n erve cell degen eration , an d cell death (see Ch ap ter 4,
Qu estion 25).
4. Gross anatomical brain changes in clu de:
a. Enlargement of brain ventricles.
b. Di u se atrop h y an d latten in g o brain su lci.
5. Microscopic anatomical brain changes in clu de:
HELP OTHERS SO THAT GOD WILL HELP YOU.

a. Amyloid plaques an d neurofibrillary tangles (also seen in oth er n eu rodegen erative


d iseases, Down’s syn drom e an d, to a lesser exten t, in n orm al agin g).
b. Loss o ch olin ergic n eu ron s in th e basal orebrain .
c. Neu ron al loss an d d egen eration in th e h ip p ocam p u s an d cortex.
6. Alzh eim er’s d isease h as a progressive, irreversible, downhill cou rse. Th e m ost e ective
in itial in terven tion s in volve providing a structured environment, in clu din g visu al-orien tin g
cu es. Su ch cu es in clu de labels over th e d oors o room s iden ti yin g th eir u n ction ; daily
p ostin g o th e d ay o th e week, d ate, an d year; daily written activity sch edu les; an d p racti-
cal sa ety m easu res (e.g., d iscon n ectin g th e stove, keep in g som e ligh ts on at n igh t).
7. Pharmacologic in terven tion s in clu d e:
a. Acetylcholinesterase inhibitors (e.g., tacrin e [Cogn ex], don ep ezil [Aricep t], rivastigm in e
[Exelon ], an d galan tam in e [Razadyn e]) to tem p orarily slow the progression o th e dis-
ease. However, th ese agen ts can n ot restore u n ction th at h as already been lost.
b. Mem an tin e (Nam en d a), an NMDA antagonist, d ecreases th e n eu rotoxic in lu x o glu -
tam ate an d calciu m , an d th u s slows deterioration in p atien ts with m oderate to severe
Alzh eim er’s disease.
c. Psychotropic agents are u sed to treat associated sym p tom s o an xiety, dep ression ,
or p sych osis. Sin ce antipsychotics are associated with increased mortality in eld erly
dem en ted p atien ts (p articu larly th ose with Lewy body disease, see later), they sh ou ld
be u sed with extrem e cau tion .

E. Other dementias (all may be major or mild NCDs)


1. Vascular NCD
a. It is cau sed by m u ltip le, sm all cereb ral in arction s u su ally resu ltin g rom card iovascu -
lar disorders su ch as h yp erten sion or ath erosclerosis.
b. In con trast to Alzh eim er’s d isease, vascu lar NCD h as a h igh er risk or m en an d is m ore
RISE USMLE NEPAL

likely to cau se m otor sym p tom s.


c. Th e p rim ary in terven tion is th e m an agem en t o th e cardiovascu lar disorder (e.g., an ti-
h yp erten sives, an ticoagu lan ts) to p reven t u rth er in arcts leadin g to deterioration in
n eu rocogn itive u n ction in g.
2. NCD with Lewy bodies
a. Grad u al, p rogressive loss o n eu rocogn itive abilities as well as h allu cin ation s (o ten
visu al) an d th e m otor ch aracteristics o Parkin son’s disease. Also associated with REM
sleep b eh avior d isord er (see Ch ap ter 10).
b. Path ology in clu d es am yloid p laqu es bu t, in con trast to Alzh eim er’s disease, ew n eu ro-
ibrillary tan gles.
c. Patien ts typ ically h ave adverse responses to antipsychotic medications .
3. NCD due to HIV infection
a. Dem en tia d u e to cortical atrop h y, in lam m ation , an d dem yelin ation resu ltin g
rom direct in ection o th e brain with HIV. Su p p ortive m easu res are th e p rim ary
m an agem en t.
b. Mu st be di eren tiated, in HIV p atien ts, rom deliriu m cau sed by cerebral lym p h om a or
op p ortu n istic b rain in ection . In con trast to dem en tia, su ch deliriu m is o ten reversible
with ch em oth erap eu tic or an tibiotic agen ts.
148 BRS Behavioral Science

4. Frontotemporal NCD (Pick’s disease). Progressive develop m en t o beh avioral ch an ges


(e.g., d isin h ibition an d ap ath y) an d / or lan gu age im p airm en t ollowed by n eu rocogn itive
d eclin e.
5. NCD due to prion disease (e.g., Creutzfeldt-J akob disease). Rap id p rogression (over several
m on th s) o p sych iatric sym p tom s su ch as an xiety, as well as atigu e, sleep an d ap p etite
distu rb an ces, with m otor eatu res (e.g., ataxia) ollowed by d em en tia an d resu ltin g in
death with in 1–2 years.

II. PERSONALITY DISORDERS


HELP OTHERS SO THAT GOD WILL HELP YOU.

A. Characteristics
1. In d ividu als with p erson ality d isorders (PDs) sh ow chronic, lifelong, rigid, unsuitable pat-
terns of relating to others th at cau se social an d occu p ation al di icu lties (e.g., ew rien ds,
job loss).
2. Person s with PDs gen erally are n ot aware th at th ey are th e cau se o th eir own p roblem s
(do not have “insight”), do n ot h ave ran k p sych otic sym p tom s, an d do not seek psychiatric
help.

B. Classification
1. PDs are categorized by th e Diagn ostic an d Statistical Man u al of Men tal Disorders, Fifth
Edition (DSM-5) in to clusters: A (p aran oid , sch izoid , sch izotyp al); B (h istrion ic, n arcissis-
tic, borderlin e, an d an tisocial); an d C (avoidan t, obsessive-com p u lsive, an d dep en den t).
Oth er categories are p erson ality ch an ge du e to an oth er m edical con dition , oth er sp eci-
ied PD, an d u n sp eci ied PD. Th e category oth er sp eci ied PD is u sed wh en several PDs are
p resen t bu t n on e reach criteria or a sp eci ic PD; u n sp eci ied PD is u sed wh en th e p erson
h as a PD b u t it is n ot in clu ded in an y o th e th ree clu sters (e.g., p assive–aggressive PD).
2. Each cluster has its own hallm ark characteristics an d genetic or fam ilial associations (e.g.,
relatives o people with PDs have a higher likelihood o having certain disorders) (Table 14.3).
3. PDs are typ ically irst observable du rin g adolescen ce an d m u st b e p resen t by early
adu lth ood . An tisocial PD can n ot b e d iagn osed u n til th e age o 18; p rior to th is age, th e
d iagn osis is con d u ct disord er (see Ch ap ter 15).

t a b l e 14.3 DSM-5 Classification and Characteristics of Personality Disorders


RISE USMLE NEPAL

Personality Disorder Characteristics

Cluster A
Hallmark Avoids social relationships and is “peculiar” but not psychotic
Genetic or familial association Psychotic illnesses
Paranoid Distrustful, suspicious, litigious
Attributes responsibility for own problems to others
Interprets motives of others as malevolent
Collects guns
Schizoid Long-standing pattern of voluntary social withdrawal
Detached, restricted emotions, lacks empathy, has no thought disorder
Schizotypal Peculiar appearance
Magical thinking (i.e., believing that one’s thoughts can affect the course of events)
Odd thought patterns and behavior without frank psychosis
Cluster B
Hallmark Dramatic, emotional, inconsistent
Genetic or familial association Depressive, bipolar, substance use, and somatic symptom disorders
Histrionic Theatrical, extroverted, emotional, sexually provocative, “life of the party”
Shallow, vain
In men, “Don J uan” dress and behavior
Cannot maintain intimate relationships
Chapter 14 Neurocognitive, Personality, Dissociative, and Eating Disorders 149

t a b l e 14.3 DSM-5 Classification and Characteristics of Personality Disorders (continued )

Personality Disorder Characteristics

Narcissistic Pompous, with a sense of special entitlement


Lacks empathy for others
Antisocial Refuses to conform to social norms and shows no concern for others
Associated with conduct disorder in childhood and criminal behavior in adulthood
(“psychopaths” or “sociopaths”)
Borderline Erratic, impulsive, unstable behavior, and mood
Feeling bored, alone, and “empty”
Suicide attempts for relatively trivial reasons
Self-mutilation (cutting or burning oneself)
HELP OTHERS SO THAT GOD WILL HELP YOU.

Often comorbid with depressive, bipolar, and eating disorders


Mini-psychotic episodes (i.e., brief periods of loss of contact with reality)
Cluster C
Hallmark Fearful, anxious
Genetic or familial association Anxiety disorders
Avoidant Overly sensitive to criticism or rejection
Obsessive–compulsive Feelings of inferiority, socially withdrawn
Perfectionistic, orderly, inflexible
Stubborn and indecisive
Ultimately inefficient
Dependent Allows other people to make decisions and assume responsibility for them
Poor self-confidence
May tolerate abuse by domestic partner because of fear of being deserted and alone
Unspecified
Passive–aggressive Procrastinates and is inefficient
Outwardly agreeable and compliant but inwardly angry and defiant

C. Management
1. For th ose wh o seek h elp, in divid u al an d grou p p sych oth erapy m ay be u se u l.
2. Ph arm acoth erapy also can be u sed to m an age sym p tom s su ch as dep ression an d an xiety
th at m ay be associated with th e PDs.

III. DISSOCIATIVE DISORDERS


RISE USMLE NEPAL

A. Characteristics
1. Th e d issociative d isord ers are ch aracterized by abru p t bu t tem p orary loss of memory
(amnesia) or identity, or by eelin gs o d etach m en t.
2. In con trast to th e NCDs in wh ich m em ory loss is cau sed by brain dys u n ction (see Section
I), th e m em ory p rob lem s in d issociative disorders are related to psychological factors su ch
as distu rb in g em otion al exp erien ces in th e p atien t’s recen t or rem ote p ast.

B. Classification and management


1. Th e DSM-5 categories o d issociative d isorders are listed in Table 14.4.
2. Man agem en t o th e d issociative d isord ers in clu d es hypnosis and drug-assisted inter-
views (see Ch ap ter 5) as well as lon g-term psychoanalytically oriented psychotherapy
(see Ch ap ter 17) to recover “lost” (rep ressed ) m em ories o d istu rb in g em otion al
exp erien ces.
150 BRS Behavioral Science

t a b l e 14.4 DSM-5 Classification and Characteristics of Dissociative Disorders

Classification Characteristics

Dissociative amnesia with or without Failure to remember important information about oneself after a stressful life
dissociative fugue event
Amnesia usually resolves in minutes or days but may last years
Fugue involves amnesia combined with sudden wandering from home after a
stressful life event
Fugue may also involve adoption of a different identity
Dissociative identity disorder (formerly At least two distinct personalities (“alters”) in an individual
multiple personality disorder) More common in women (particularly those sexually abused in childhood)
In a forensic (e.g., jail) setting, malingering and alcohol use must be considered
HELP OTHERS SO THAT GOD WILL HELP YOU.

and excluded
Depersonalization/derealization Recurrent, persistent feelings of detachment from one’s own body, the social
disorder situation (depersonalization), or the environment (derealization) when
stressed
Understanding that these perceptions are only feelings, that is, normal reality
testing
Identity disruption Dissociative symptom (e.g., trance-like state, memory loss) (1) in persons
exposed to intense coercive persuasion (e.g., brainwashing) or (2) indigenous
to particular locations or cultures (e.g., “Amok” in Indonesia)

IV. OBESITY AND EATING DISORDERS


A. Obesity
1. Overview
a. Obesity is d e in ed as bein g more than 20% over ideal weight on th e basis o com m on
h eigh t an d weigh t ch arts or h avin g a body mass index (BMI) (body weigh t in kg/ h eigh t
in m 2) of 30 or h igh er (Figu re 14.1).
b. At least 25% of adults are ob ese an d an in creasin g n u m ber o ch ildren are overweigh t (at
or ab ove th e 95th p ercen tile o BMI or age) in th e Un ited States.
c. Obesity is not an eating disorder. Genetic factors are m ost im p ortan t in obesity; adu lt
weigh t is closer to th at o b iologic rath er th an adop tive p aren ts.
RISE USMLE NEPAL

d. Obesity is m ore com m on in lower socioecon om ic grou p s an d is associated with


increased risk or card ioresp iratory, sleep, an d orth op edic p roblem s; h yp erten sion ;
an d d iab etes m ellitu s.
2. Man agem en t
a. Most weigh t loss ach ieved u sin g com m ercial dietin g an d weigh t loss p rogram s is
regained within a 5-year period.
b. Bariatric surgery (e.g., gastric byp ass, gastric ban din g) is in itially e ective bu t o lim ited
valu e or m ain tain in g lon g-term weigh t loss.
c. Ph arm acologic agen ts ap p roved by th e FDA or weigh t loss in clu de orlistat (Xenical,
Alli), a p an creatic lip ase in h ibitor th at lim its th e breakdown o dietary ats; phentermine
(Ionamin), a sym p ath om im etic am in e th at decreases ap p etite; an d lorcaserin (Belviq),
a selective 5-HT2C recep tor agon ist th at ap p aren tly leads to a eelin g o satiety, an d
p h en term in e com bin ed with th e an ticon vu lsan t topiramate (Qsymia ).
d. A com b in ation o sensible dieting and exercise is th e m ost e ective way to m ain tain
lon g-term weigh t loss.
Chapter 14 Neurocognitive, Personality, Dissociative, and Eating Disorders 151

WEIGHT HEIGHT
kg lb BODY cm in.
340
150
320
MAS S
140 INDEX 125
300 50
130 280 [WT/(HT)2 ] 130
120 70
260
110 240 60 135

100 220 55
50 140
95
90 200 Morbid obe s ity
85 190 40 (BMI ≥ 40) 145
80 180
170 150
75
HELP OTHERS SO THAT GOD WILL HELP YOU.

160 60
70
150 Ove rwe ight 30 Obe s e 155
65 (BMI 25–29.9) (BMI ≥ 30)
140
160
55 130 Norma l we ight
55 120 (BMI 20.0–24.9) 20 165 65

50 110 Cutoff for 170

45 100 a norexia 175


95 ne rvos a 70
90 (BMI < 17.0) 180
40
85
185
80 10
35
75 190 75
70
195
30 65
200
60 80
205
25 55
210
50 85
FIGURE 14.1. BMI. BMI is calculated by placing a straight edge between the
body-weight column (left) and the height column (right) and reading the BMI
from the point at which the straight edge crosses the BMI column.

B. Eating disorders: Anorexia nervosa, bulimia nervosa, and binge-eating disorder


1. In th e eatin g d isord ers, th e p atien t sh ows abn orm al beh avior associated with ood desp ite
normal appetite .
2. Th e subtypes of anorexia nervosa are th e restricting type (e.g., excessive dietin g) an d, in
50% o th e p atien ts, th e binge-eating purging type (e.g., excessive dietin g p lu s bin ge-eatin g
[con su m in g large qu an tities o h igh -calorie ood at on e tim e] an d com p en satory beh avior
su ch as p u rgin g [e.g., vom itin g, or m isu se o laxatives, diu retics, an d en em as]).
RISE USMLE NEPAL

3. In bulimia nervosa, th ere is bin ge-eatin g an d recu rren t com p en satory beh avior. Binge-
eating disorder in volves th e eatin g beh avior associated with bu lim ia with ou t recu rren t
com p en satory b eh avior.
4. I th ere is p u rgin g in eith er an orexia n ervosa or bu lim ia n ervosa, th ere are o ten elec-
trolyte abnormalities . Sp eci ic electrolyte ab n orm alities are related to th e typ e o p u rgin g
seen .
a. Low p otassiu m (h yp okalem ia), low sodiu m , an d h igh bicarbon ate (m etabolic alkalo-
sis) levels are seen with vomiting or diuretic abuse .
b. Low p otassiu m , h igh ch loride, an d low bicarbon ate levels (togeth er kn own as h yp er-
ch lorem ic m etab olic acidosis) are seen with laxative abuse .
5. Eatin g disorders are more common in women, in higher socioeconomic groups , an d in th e
United States (com p ared with oth er develop ed cou n tries).
6. Som e you n g p eop le with typ e 1 d iab etes regu larly redu ce or om it in su lin to con trol th eir
weigh t, a con dition th at h as b een called “diabulimia.”
6. Ph ysical an d p sych ological ch aracteristics an d m an agem en t o an orexia n ervosa an d
bu lim ia n ervosa can b e ou n d in Table 14.5.
152 BRS Behavioral Science

t a b l e 14.5 Physical and Psychological Characteristics and Management of Anorexia


Nervosa and Bulimia Nervosa

Management (in Order of


Disorder Physical Characteristics Psychological Characteristics Highest to Lowest Utility)

Anorexia nervosa Extreme weight loss (BMI Refusal to eat despite normal Hospitalization directed at
< 17.0) appetite because of an reinstating nutritional
Electrolyte disturbances overwhelming fear of being obese condition (starvation and
Hypercholesterolemia Belief that one is fat when very thin compensatory behavior
Mild anemia and leukopenia High interest in food-related such as purging can result
Lanugo (downy body hair on activities (e.g., cooking) in metabolic abnormalities
the trunk) Simulates eating [e.g., hypokalemia] leading
HELP OTHERS SO THAT GOD WILL HELP YOU.

Melanosis coli (darkened Lack of interest in sex to cardiac arrhythmia


area of the colon if there is Was a “perfect child” (e.g., good causing death)
laxative abuse) student) Family therapy (aimed
Osteoporosis Interfamily conflicts (e.g., patient’s particularly at normalizing
Cold intolerance problem draws attention away the mother–child
Syncope from parental marital problem relationship)
or an attempt to gain control to Group psychotherapy in an
separate from the mother) inpatient eating disorders
Excessive exercising program
(“hypergymnasia”)
Bulimia nervosa Relatively normal body weight Binge-eating (in secret) of Cognitive and behavioral
Esophageal varices caused by high-calorie foods, followed therapies
repeated vomiting by vomiting or other purging Average to high doses of
Tooth enamel erosion due to behavior to avoid weight gain antidepressants; fluoxetine
gastric acid in the mouth Depression is the only FDA-approved
Swelling or infection of the “Hypergymnasia” agent; bupropion is
parotid glands contraindicated because it
Metacarpal–phalangeal lowers seizure threshold
calluses (Russell’s sign) from Group psychotherapy in an
the teeth because the hand inpatient or outpatient
is used to induce gagging eating disorders program
Electrolyte disturbances
RISE USMLE NEPAL
Review Test

Directions: Each o th e n u m b ered item s or in com p lete statem en ts in th is section is ollowed by


an swers or by com p letion s o th e statem en t. Select th e one lettered an swer or com p letion th at
is best in each case.
HELP OTHERS SO THAT GOD WILL HELP YOU.

1. Th e m oth er o a 25-year-old m an , 3. Th e m oth er o an obese 12-year-old boy


wh o was diagn osed with AIDS 1 year ago, tells th e p h ysician th at th e “ch ild is n ot
rep orts th at h er son h ad been doin g well eatin g well.” Wh at is th e p h ysician’s b est
u n til th is m orn in g wh en sh e ob served h im resp on se to th e m oth er’s statem en t?
actin g stran gely. He was sittin g u p in b ed (A) Wh at do you m ean by “n ot eatin g well”?
an d ap p eared to b e p u n ch in g th e air an d (B) Th e ch ild looks like h e is eatin g well
grabbin g at in sects, alth ou gh n on e were en ou gh .
p resen t. Th e p atien t’s CD 4 cou n t is <100 (C) Th ere are a n u m ber o diets available
cells/ m m 3 an d h is tem p eratu re is 103°F. Th e th at are excellen t or ch ildren .
m oth er is con cern ed abou t th ese sym p tom s (D) In creased exercisin g m ay be th e an swer
b ecau se th e p atien t’s eld er b roth er h as to you r son’s weigh t p roblem .
sch izop h ren ia. Th is clin ical p ictu re is m ost (E) Diet p lu s exercise is th e m ost e ective
con sisten t with m an agem en t or obesity.
(A) AIDS dem en tia
(B) deliriu m cau sed by cryp tococcal 4. In Alzh eim er’s disease p atien ts, th e
m en in gitis m ajor e ect on n eu rotran sm itter system s
(C) sch izop h ren ia o tacrin e, d on ep ezil, rivastigm in e, an d
(D) brie p sych otic disorder galan tam in e is to
(E) n eu rocogn itive d isorder d u e to an oth er (A) in crease dop am in e availability
m ed ical con d ition (B) decrease dop am in e availab ility
(C) in crease Ach availability
2. Alth ou gh h e h as n o p rior p sych iatric (D) decrease Ach availab ility
h istory, a 45-year-old m an rep orts th at over (E) decrease seroton in availability
th e p ast ew weeks h e h as becom e very
an xiou s. His wi e tells th e doctor th at th e
p atien t also h as recen tly started to sh ow
Questions 5 and 6
RISE USMLE NEPAL

m em ory loss an d is beh avin g di eren tly th an


h e ever d id b e ore. Th e m an’s m em ory an d
beh avior declin e p recip itou sly over th e n ext A 78-year-old retired em ale p h ysician
year an d h e th en d ies. Th e au top sy reveals rep orts th at sh e h as been con u sed an d or-
n eu ro brillary tan gles in th e brain . Th e get u l over th e p ast 10 m on th s. Sh e also h as
m ost likely diagn osis or th is p atien t is m ajor di icu lty sleep in g, h er ap p etite is p oor, an d
n eu rocogn itive d isorder (NCD) du e to sh e h as lost 20 p ou n ds. Qu estion in g reveals
(A) ron totem p oral disease th at h er 18-year-old dog died 10 m on th s ago.
(B) Alzh eim er’s d isease
(C) p rion disease 5. At th is tim e, th e m ost ap p rop riate
(D) Parkin son’s disease diagn osis or th is p atien t is
(E) Hu n tin gton’s disease (A) deliriu m
(B) p seu dodem en tia
(C) Alzh eim er’s d isease
(D) dissociative am n esia with dissociative
u gu e
(E) n eu rocogn itive disorder du e to an oth er
m ed ical con d ition

153
154 BRS Behavioral Science

6. O th e ollowin g, th e m ost ap p rop riate 10. A 28-year-old stockbroker wh o is


in itial in terven tion or th is p atien t is m arried an d h as two ch ildren u su ally
(A) an tip sych otic m ed ication d resses con servatively. Sh e receives a letter
(B) p rovision o a stru ctu red en viron m en t con tain in g a recen t p h otograp h o h ersel in
(C) an tid ep ressan t m ed ication a skim py black leath er ou t t. Sh e does n ot
(D) don ep ezil rem em b er th e m an wh o sign ed th e letter, or
(E) reassu ran ce p osin g or th e p h otograp h . Th is wom an is
sh owin g eviden ce o
(A) dissociative am n esia
Questions 7 and 8 (B) dissociative am n esia with dissociative
u gu e
HELP OTHERS SO THAT GOD WILL HELP YOU.

(C) dissociative iden tity disorder


A 75-year-old m an is b rou gh t to th e em er- (D) dep erson alization / d erealization disord er
gen cy dep artm en t a ter bein g bu rn ed in a (E) sch izop h ren ia
h ou se ire. Th is is th e p atien t’s th ird em er-
gen cy visit in 2 m on th s. His oth er visits 11. A 30-year-old sin gle wom an wh o h as
occu rred a ter h e in h aled n atu ral gas wh en h e b een sm okin g th ree p acks o cigarettes a
le t th e stove on with ou t a lam e, an d b ecau se d ay or th e last 10 years asks th e p h ysician
h e ell down th e stairs a ter wan derin g ou t o to h elp h er stop sm okin g. Th e d octor asks
th e h ou se in th e m iddle o th e n igh t. Th ere th e p atien t wh y sh e sm okes so m u ch . Th e
is n o eviden ce o p h ysical illn ess an d n o h is- p atien t resp on ds, “I always eel very alon e
tory o su bstan ce u se. His wi e is distressed an d em p ty in side; I sm oke to ll m ysel u p.”
an d b egs th e doctor to let h er h u sban d com e Th e p atien t reveals th at sh e som etim es cu ts
h om e. th e skin on h er arm s with a kn i e in order
to “ eel som eth in g.” Sh e also n otes th at
7. Th is p atien t is sh owin g eviden ce o wh en sh e is u p set, sh e o ten u ses cocain e
(A) deliriu m an d h as sex with m en wh om sh e does n ot
(B) p seu dodem en tia kn ow well. A ter th ese ep isodes sh e typ ically
(C) Alzh eim er’s d isease eels even m ore alon e an d em p ty. Wh ich
(D) dissociative u gu e o the ollowin g is th e m ost ch aracteristic
(E) n eu rocogn itive d isorder d u e to an oth er d e en se m ech an ism u sed by p eop le with th is
m ed ical con d ition wom an’s p erson ality ch aracteristics?
(A) Disp lacem en t
8. O th e ollowin g, th e m ost ap p rop riate (B) In tellectu alization
in itial in terven tion or th is p atien t is (C) Den ial
(A) an tip sych otic m ed ication (D) Reaction orm ation
(B) p rovision o a stru ctu red en viron m en t (E) Sp littin g
RISE USMLE NEPAL

(C) an tid ep ressan t m ed ication


(D) don ep ezil 12. Th e elderly p aren ts o a 45-year-old
(E) reassu ran ce p atien t with m ild in tellectu al d isability
tell th e p h ysician th at th eir son b egan to
9. A 43-year-old wom an says th at wh en sh e h ave di cu lty iden ti yin g am iliar objects
is un der stress, sh e o ten eels as i sh e is an d p eop le abou t 6 m on th s ago. Ph ysical
“ou tside o h ersel ” an d is watch in g h er li e exam in ation reveals th at th e p atien t is
as th ou gh it were a p lay. Sh e kn ows th at th is sh ort in statu re, h as a p rotru din g ton gu e,
p ercep tion is on ly a eelin g an d th at sh e is f at acies, h yp oton ia, an d a th ick n eck. Th e
really livin g h er li e. Th is wom an is sh owin g gen etic abn orm ality resp on sib le or th is
evid en ce o clin ical p ictu re is m ost likely to be associated
(A) dissociative am n esia with ch rom osom e
(B) dissociative am n esia with d issociative (A) 1
u gu e (B) 4
(C) dissociative iden tity d isorder (C) 14
(D) dep erson alization / d erealization disord er (D) 19
(E) sch izop h ren ia (E) 21
Chapter 14 Neurocognitive, Personality, Dissociative, and Eating Disorders 155

13. An 18-year-old stu d en t wh o is abou t 10 16. Wh ich o th e ollowin g is m ost likely to


p ou n d s overweigh t tells h er p h ysician th at ch aracterize th is wom an ?
sh e h as decid ed to go on a low carb oh yd rate (A) Lack o in terest in p rep arin g ood
d iet th at sh e read ab ou t in a b ook. Sh e says (B) Em b arrassm en t abou t h er ap p earan ce
th at th e b ook gu aran tees th at p eop le wh o (C) Lack o ap p etite
ollow th e diet will lose at least 25 p ou n d s in (D) Con lict with h er m oth er
3 weeks. Th e doctor’s best statem en t to th e (E) Poor sch ool p er orm an ce
p atien t at th is tim e is
(A) Th at is n on sen se, you can’t lose th at 17. Wh ich o th e ollowin g disorders is th is
m u ch weigh t in on ly 3 weeks. p atien t at th e h igh est risk or in th e u tu re?
(B) You m ay lose th e weigh t bu t you will en d (A) Derm atitis
HELP OTHERS SO THAT GOD WILL HELP YOU.

u p gain in g b ack even m ore weigh t. (B) Osteoarth ritis


(C) Please tell m e m ore abou t th e book th at (C) Osteop orosis
you read . (D) Pan creatic can cer
(D) You m ay b e sh owin g sign s o an eatin g (E) Biliary atresia
disord er.
(E) Man y p eop le in you r age grou p h ave 18. A p h ysician wou ld like to p rescrib e
eatin g d isorders. an an tidep ressan t to treat h er 24-year-old
m ale p atien t wh o h as bu lim ia. Wh ich o th e
14. Two d ays a ter, a 23-year-old m an is ollowin g agen ts sh ou ld be avoided in th is
rescu ed rom a b u rn in g bu ild in g, h e h as p atien t?
n o m em ory o th e re or o th e ew h ou rs
(A) Desip ram in e
b e ore or a ter it. Ph ysical exam in ation is
(B) Flu oxetin e
u n rem arkable. Th e m ost likely exp lan ation
(C) Bu p rop ion
or th is clin ical p ictu re is
(D) Tran ylcyp rom in e
(A) p osttrau m atic stress disorder (E) Paroxetin e
(B) dissociative am n esia
(C) adju stm en t disord er 19. A 20-year-old m an states th at h e is
(D) early on set Alzh eim er’s d isease u n com ortab le arou n d wom en . He says th at
(E) su b arach n oid h em orrh age h e gets an xiou s wh en h e is with a wom an
an d “ju st does n ot kn ow wh at to say to h er.”
15. A p h ysician con d u cts a yearly p h ysical Th e p atien t, a h igh sch ool grad u ate, rep orts
exam in ation on a typ ical 85-year-old th at h e h as a ew m ale rien ds with wh om
p atien t. Wh ich o th e ollowin g m en tal h e “h an gs ou t” an d is d oin g well in h is job
ch aracteristics is th e doctor m ost likely to in con stru ction . Th is clin ical p ictu re is m ost
see in th is p atien t? con sisten t with wh ich o th e ollowin g?
(A) Im p aired con sciou sn ess
RISE USMLE NEPAL

(A) Sch izoid p erson ality disorder


(B) Abn orm al level o arou sal (B) Sch izotyp al p erson ality disorder
(C) Min or orget u ln ess (C) Avoidan t p erson ality disorder
(D) Psych osis (D) Mild au tism sp ectru m disorder
(E) Dep ression (E) Norm al sh yn ess

Questions 16 and 17

A 21-year-old ballet dan cer, wh o is 5 eet


7 in ch es tall an d weigh s 95 p ou n ds (BMI =
14.5) tells th e doctor th at sh e n eeds to lose
an oth er 15 p ou n d s to p u rsu e a career in
dan ce. Her m ood ap p ears good . Fin d in gs
on p h ysical exam in ation are u n rem arkab le
excep t or excessive growth o d own y bod y
h air. Sh e rep orts th at sh e h as n ot m en stru -
ated in m ore th an 3 years.
156 BRS Behavioral Science

20. A 75-year-old m an with a 5-year h istory 22. A 20-year-old em ale college stu den t tells
o Alzh eim er’s disease h as recen tly becom e th e doctor th at b ecau se sh e was a raid to
disorien ted wh en th e ligh ts are tu rn ed o b e alon e, sh e tried to com m it su icide a ter a
at n igh t. He wan ders abou t th e ap artm en t m an with wh om sh e h ad two dates did n ot
at n igh t an d h is wi e is con cern ed th at h e call h er again . A ter th e in terview, sh e tells
will in ju re h im sel wh ile sh e is sleep in g. Th e h im th at all o th e oth er doctors sh e h as seen
Folstein Min i–Men tal State Exam sh ows th at were terrible an d th at h e is th e on ly doctor
th e p atien t is d isorien ted regard in g tim e wh o h as ever u n derstood h er p roblem s.
an d p lace an d h as p oor sh ort-term m em ory.
Ph ysical exam in ation is u n rem arkab le 23. Wh en ever a 28-year-old wom an
an d th e p atien t is n ot cu rren tly takin g an y p resen ts to th e d octor’s o ce, sh e b rin gs
HELP OTHERS SO THAT GOD WILL HELP YOU.

m edication . Wh at is th e m ost ap p rop riate gi ts or th e recep tion ist an d th e n u rses.


rst recom m en d ation or th e m an agem en t Wh en sh e h ears th at on e o th e n u rses h as
o this p atien t? taken an oth er job, sh e b egin s to sob lou d ly.
(A) Ask th e wi e to in crease h om e n igh ttim e Wh en th e d octor sees h er, sh e rep orts th at
ligh tin g. sh e is so warm th at sh e m u st h ave “a ever
(B) Prescrib e don ep ezil or th e p atien t. o at least 106°F.”
(C) Prescrib e h alop eridol or th e p atien t.
(D) Prescrib e m eth ylp h en idate or th e wi e 24. Two weeks a ter a 50-year-old, overweigh t,
so th at sh e can stay alert d u rin g th e h yperten sive wom an agreed to start an
n igh t. exercise program , she gain ed 4 poun ds. She
(E) Recom m en d th at th e p atien t b e p u t in rep orts th at she h as n ot exercised yet becau se
m ech an ical restrain ts at bed tim e. “th e gym was too crowded.”

25. Th e p aren ts o a 26-year-old wom an say


Questions 21–28 th at th ey are con cern ed ab ou t h er b ecau se
sh e h as n o rien d s an d sp en d s m ost o h er
For each p atien t below ch oose th e m ost tim e h ikin g in th e wood s an d workin g on
ap p rop riate p erson ality disord er h er com p u ter. Th e d octor exam in es h er an d
(A) Borderlin e p erson ality disorder n d s th at sh e is con ten t with h er solitary li e
(B) Histrion ic p erson ality disorder an d h as n o evid en ce o a th ou gh t d isord er.
(C) Obsessive–com p u lsive p erson ality
disord er 26. A 22-year-old m edical studen t is un able
(D) Avoid an t p erson ality disord er to stop studyin g un til she has m em orized the
(E) An tisocial p erson ality d isord er en tire set o n otes or each o h er cou rses.
(F) Dep en den t p erson ality disord er Makin g com prehen sive lists o all the subjects
RISE USMLE NEPAL

(G) Dissociative id en tity disord er she m ust study also takes up her study tim e.
(H) Paran oid p erson ality d isord er Because o th is, sh e is con stan tly beh in d in
(I) Passive–aggressive p erson ality d isord er h er work an d in dan ger o ailin g her courses.
(J ) Narcissistic p erson ality disord er
(K) Sch izotyp al p erson ality disorder 27. A 40-year-old p atien t with b ru ises on h is
(L) Sch izoid p erson ality disorder arm s, n eck, an d b ack tells th e d octor th at
h is lover o ten b erates h im an d p h ysically
21. A 38-year-old m an asks h is d octor to ab u ses h im . He b egs th e d octor n ot to
re er h im to a p h ysician wh o atten d ed a top - in ter ere b ecau se h e is a raid th at th e m an
rated m ed ical sch ool. He says th at h e kn ows will d esert h im an d th at h e will b e alon e.
th e doctor will n ot b e o en d ed becau se sh e
will u n d erstan d th at h e is “b etter” th an h er
28. A 20-year-old em ale college stu den t wh o
oth er p atien ts.
was u n ab le to an swer a teach er’s qu estion in
class drop s ou t o sch ool th e n ext d ay.
An swers an d Exp lan ation s

Typical Board Question


The answer is C. Overu se o laxatives, restrictin g calorie con su m p tion , sel -in du ced vom itin g, an d
startin g an in ten se aerobic exercise p rogram can all lead to weigh t loss. However, th e decrease
th is d iab etic p atien t sh ows in h em oglobin A1c over th e p ast 6 m on th s in dicates th at sh e h as
HELP OTHERS SO THAT GOD WILL HELP YOU.

decreased h er d osage o sel -ad m in istered in su lin in order to lose weigh t.

1. The answer is B. Th is clin ical p ictu re th at in clu des th e su dden on set o a p sych iatric
sym p tom (i.e., visu al h allu cin ation s) coin cidin g with th e on set o a h igh ever in a relatively
recen tly d iagn osed (1 year) AIDS p atien t is m ost con sisten t with deliriu m cau sed by an
op p ortu n istic in ection o th e brain su ch as cryp tococcal m en in gitis. Psych otic illn esses
su ch as sch izop h ren ia, b rie p sych otic d isorder, or AIDS dem en tia can n ot be diagn osed i
th e sym p tom s (as in th is p atien t) can be exp lain ed by an acu te m edical illn ess. Also, AIDS
d em en tia occu rs in th e late stages o th e d isease an d wou ld be ch aracterized p rim arily by
grad u ally worsen in g cogn itive u n ction in g (e.g., m em ory loss) as well as m otor sym p tom s.
Neu rocogn itive disord er du e to an oth er m edical con dition is o ten associated with a h istory
o alcoh olism an d h as a gradu al, p rogressive down h ill cou rse.
2. The answer is C. Becau se th is p atien t died with in 1 year o sh owin g sym p tom s, th is clin ical
p ictu re is m ost con sisten t with a p rion disease su ch as Creu tz eldt-Jakob disease. Th e NCDs
d u e to ron totem p oral d isease, Alzh eim er’s disease, Parkin son’s disease, an d Hu n tin gton’s
d isease typ ically p rogress over m an y years to death . Neu ro ibrillary tan gles are ou n d in
a n u m b er o n eu rodegen erative d iseases as well as n orm al agin g an d are n ot sp eci ic to
Alzh eim er’s d isease.
3. The answer is A. Th e p h ysician’s b est resp on se to th e m oth er’s statem en t is to get m ore
in orm ation , or exam p le, “Wh at d o you m ean by n ot eatin g well?” Recom m en din g ch an ges
in d iet or exercise or com m en tin g on th e ch ild’s ap p earan ce are n ot ap p rop riate u n til you
in d ou t m ore abou t th e m oth er’s p ercep tion o th e p roblem .
4. The answer is C. Low con cen tration o acetylch olin e is associated with th e sym p tom s
o Alzh eim er’s disease. Tacrin e, don ep ezil, rivastigm in e, an d galan tam in e are
RISE USMLE NEPAL

acetylch olin esterase in h ib itors (i.e., th ey b lock th e b reakd own o Ach , in creasin g its
availability). Th ese agen ts can th u s b e e ective in slowin g down th e p rogression o th e
illn ess. Th ey do n ot restore th e u n ction th e p atien t h as already lost.
5. The answer is B. 6. The answer is C. Th e b est exp lan ation or th is p atien t’s sym p tom s is
p seu dod em en tia—dep ression th at m im ics dem en tia. In th e elderly, dep ression is o ten
associated with cogn itive p roblem s as well as sleep an d eatin g p rob lem s. Evid en ce or
d ep ression is p rovid ed by th e act th at th is p atien t’s sym p tom s began with th e loss o an
im p ortan t relation sh ip (i.e., th e death o h er dog). Deliriu m an d dem en tia are cau sed by
p h ysiological ab n orm alities in th e brain . Dissociative am n esia with dissociative u gu e
in volves wan derin g away rom h om e, an d n eu rocogn itive disorder du e to an oth er m edical
con dition is associated with a h istory o alcoh olism . Th e m ost e ective in terven tion or
th is dep ressed p atien t is an tid ep ressan t m edication . Wh en th e m edication relieves th e
d ep ressive sym p tom s, h er m em ory will im p rove. An tip sych otic m edication , p rovision o
a stru ctu red en viron m en t, acetylch olin esterase in h ibitors su ch as d on ep ezil, an d sim p le
reassu ran ce are n ot ap p rop riate or th is p atien t.

157
158 BRS Behavioral Science

7. The answer is C. 8. The answer is B. Th is p atien t is sh owin g eviden ce o Alzheim er’s disease.
He is h avin g accid en ts becau se h e is orget u l (e.g., orgettin g to tu rn o th e gas jet), an d
wan ders ou t o th e h ou se becau se he does n ot kn ow wh ich is th e closet or bath room door
an d wh ich is th e ou tsid e d oor. Th ere is n o evid en ce o a m ed ical cau se or h is sym p tom s,
as th ere wou ld b e in d eliriu m . Th ere is n o eviden ce o d ep ression , as in p seu dod em en tia,
or o a h istory o alcoh ol u se, as in n eu rocogn itive disorder du e to an oth er m edical
con d ition . Th e m ost e ective in itial in terven tion or th is p atien t is p rovision o a
stru ctu red en viron m en t (e.g., givin g th e p atien t visu al cu es or orien tation [labelin g doors
or u n ction ]) an d takin g p ractical m easu res (e.g., rem ovin g th e gas stove). Don ep ezil can
th en be u sed to slow th e p rogression o h is illn ess. Oth er m edication s an d reassu ran ce
m ay b e u se u l or sym p tom s su ch as p sych osis, dep ression , an d an xiety, bu t will h ave little
HELP OTHERS SO THAT GOD WILL HELP YOU.

e ect on th e p atien t’s orget u l an d p oten tially dan gerou s beh avior.
9. The answer is D. Th is wom an , wh o eels as i sh e is “ou tsid e o h ersel ,” watch in g h er li e as
th ou gh it were a p lay, is sh owin g eviden ce o d ep erson alization / derealization disorder, a
p ersisten t eelin g o detach m en t rom on e’s own body or th e social situ ation . In con trast to
p sych otic d isorders su ch as sch izop h ren ia (see Ch ap ter 11), th is wom an is aware th at th is
p ercep tion is on ly a eelin g an d th at sh e is really livin g h er li e.
10. The answer is C. Th is stockbroker is sh owin g eviden ce o dissociative iden tity disorder. Sh e
does n ot rem em ber th e m an wh o sign ed th e letter or p osin g or th e p h otograp h becau se
th ese even ts occu rred wh en sh e was sh owin g an oth er p erson ality. Dissociative am n esia
in volves a ailu re to rem em ber im p ortan t in orm ation abou t on esel , an d dissociative
am n esia with dissociative u gu e is am n esia com bin ed with su dden wan derin g rom h om e
an d takin g on a di eren t id en tity. Dep erson alization / d erealization d isorder is a p ersisten t
eelin g o detach m en t rom on e’s own body, th e social situ ation , or th e en viron m en t
(derealization ) (an d see also an swer to Qu estion 9).
11. The answer is E. Th is wom an , wh o h as always elt em p ty an d alon e (n ot m erely lon ely),
sh ows eviden ce o borderlin e p erson ality disorder. Borderlin e p atien ts typ ically u se
sp littin g (see Ch ap ter 6) as a d e en se m ech an ism . Sel -in ju riou s beh avior an d im p u lsive
b eh avior (e.g., dru g u se, sex with m u ltip le p artn ers) also are ch aracteristic o p eop le with
th is p erson ality disorder.
12. The answer is E. Th is p atien t with m ild in tellectu al d isability an d associated p h ysical
in din gs p rob ably h as Down’s syn drom e, wh ich is associated with ch rom osom e 21. Down’s
syn drom e p atien ts o ten develop Alzh eim er’s disease in m iddle age, wh ich exp lain s th e
m em ory loss th at th is p atien t disp lays.
13. The answer is C. Th e doctor’s best statem en t to th e p atien t at th is tim e is, “Please tell m e
RISE USMLE NEPAL

m ore ab ou t th e book th at you read.” It is im p ortan t to get as m u ch in orm ation as p ossible


rom th e p atien t be ore decidin g on a cou rse o action .
14. The answer is B. Th e m ost likely exp lan ation or th is clin ical p ictu re, th at is, h avin g n o
m em ory o a trau m atic even t with n o p h ysical in din gs, is dissociative am n esia. In PTSD
an d in adju stm en t disord er, th ere is n o ran k m em ory loss. Su barach n oid h em orrh age,
a h em orrh age in th e sp ace b etween th e arach n oid sp ace an d th e p ia m ater, typ ically
p resen ts with a “th u n d erclap” h eadach e, vom itin g, or oth er n eu rologic sym p tom s.
15. The answer is C. Th is typ ical 85-year-old p atien t is likely to show m in or orget uln ess, such
as orgettin g n ew n am es an d p hon e n um bers. Im paired con sciousn ess, p sych osis, an d
abn orm al level o arou sal are seen in deliriu m , wh ich is associated with a variety o p h ysical
illn esses. As in you n ger p eop le, in th e elderly dep ression is an illn ess (see Chapter 12), n ot a
n atu ral con sequ en ce o typ ical agin g.
Chapter 14 Neurocognitive, Personality, Dissociative, and Eating Disorders 159

16. The answer is D. 17. The answer is C. Th is wom an is already u n derweigh t yet wan ts to lose
m ore weigh t, an d sh e h as develop ed lan u go (growth o down y body h air) an d am en orrh ea
(absen ce o m en ses). Th ese in d in gs in d icate th at sh e h as an orexia n ervosa. Sin ce dan cers
an d gym n asts o ten m u st be sm all an d slim , th ese activities are closely associated with
th e develop m en t o an orexia n ervosa. An orexia is also ch aracterized by am ily con licts,
p articu larly with th e m oth er; n orm al ap p etite; h igh in terest in ood an d cookin g; low
sexu al in terest; good sch ool p er orm an ce; an d excessive exercisin g. Patien ts wh o h ave
an orexia n ervosa or an exten d ed p eriod (5 years in th is you n g wom an ) are at h igh risk or
osteop orosis.
18. The answer is C. Bu p rop ion is con train dicated in eatin g disorder p atien ts wh o also p u rge
becau se it can lower th e seizu re th resh old. Th e on ly an tidep ressan t th at is FDA ap p roved
HELP OTHERS SO THAT GOD WILL HELP YOU.

or p atien ts with b u lim ia n ervosa is lu oxetin e.


19. The answer is E. Th is clin ical p ictu re is m ost con sisten t with n orm al sh yn ess. Alth ou gh
th is 20-year-old p atien t is som ewh at an xiou s arou n d wom en , th e act th at h e h as rien ds
an d is doin g well in h is job m akes it u n likely th at h e h as a p erson ality disorder or au tism
sp ectru m disorder (see Ch ap ter 15).
20. The answer is A. Th e m ost ap p rop riate irst recom m en d ation or th e m an agem en t o
th is p atien t is to ask th e wi e to in crease h om e n igh ttim e ligh tin g. Ligh tin g will im p rove
th e p atien t’s ability to n egotiate th e ap artm en t at n igh t an d so redu ce h is n octu rn al
disorien tation . Keep in g th e wi e awake is n ot p ractical or p ositive or h er an d m ech an ical
restrain ts sh ou ld b e avoided i p ossib le (see also an swers to Qu estion s 7 an d 8).
21. The answer is J . Th is 38-year-old m an , wh o asks to be re erred to a p h ysician wh o
atten ded a top -rated m edical sch ool becau se h e is “better” th an oth er p atien ts, is sh owin g
evid en ce o n arcissistic p erson ality d isorder (see also an swers to Qu estion s 22–28).
22. The answer is A. Th is 20-year-old college stu den t, wh o m ade a su icide attem p t a ter a
relatively trivial relation sh ip b roke u p an d wh o u ses sp littin g as a de en se m ech an ism (e.g.,
all o th e oth er doctors sh e h as seen were terrib le an d th is d octor is p er ect), is sh owin g
evid en ce o borderlin e p erson ality d isorder.
23. The answer is B. Th is 28-year-old wom an wh o brin gs gi ts or th e recep tion ist an d
th e n u rses b ecau se sh e n eeds everyon e’s atten tion is sh owin g eviden ce o h istrion ic
p erson ality disorder. Patien ts with th is p erson ality disorder ten d to exaggerate th eir
p h ysical sym p tom s or d ram atic e ect (e.g., “a ever o at least 106°F”).
24. The answer is I. Th is 50-year-old wom an , wh o agreed to start an exercise p rogram an d
th en m akes weak excu ses or h er ailu re to ollow th e p rogram , is sh owin g evid en ce o
RISE USMLE NEPAL

p assive–aggressive p erson ality d isorder. Sh e did n ot really wan t to ollow th e doctor’s


exercise p rogram (was in wardly de ian t) bu t agreed to do it (was ou twardly com p lian t).
25. The answer is L. Th is 26-year-old wom an , wh o sh ows n o eviden ce o a th ou gh t disorder,
h as n o rien ds, an d sp en d s m ost o h er tim e at solitary p u rsu its, is sh owin g eviden ce o
sch izoid p erson ality disorder. Patien ts with sch izoid p erson ality disorder are typ ically
con ten t with th eir solitary li estyle.
26. The answer is C. Th is m edical stu d en t, wh o m u st con stan tly m ake lists an d review an d
m em orize h er n otes, is sh owin g eviden ce o obsessive–com p u lsive p erson ality disorder.
Th is beh avior is ultim ately in e icien t an d h as resu lted in h er academ ic p roblem s.
27. The answer is F. Th is abu sed m an is sh owin g eviden ce o dep en den t p erson ality disorder.
He tolerates h is p artn er’s ab u se b ecau se o h is overridin g ear o bein g deserted by h is
lover, b ein g alon e, an d h avin g to m ake h is own decision s.
28. The answer is D. Th is 20-year-old em ale college stu den t sh ows eviden ce o avoidan t
p erson ality disorder. Sh e is so overwh elm ed by wh at sh e p erceives as criticism an d
rejection th at sh e drop s ou t o sch ool rath er th an ace h er teach er an d classm ates again .
Psych iatric Disorders in
c ha pte r
15 Ch ildren
HELP OTHERS SO THAT GOD WILL HELP YOU.

Typical Board Question


A 2-year-old girl wh o h as reach ed all o h er d evelop m en tal m ileston es at th e typ ical ages can -
n ot seem to p ay atten tion to a task or m ore th an 15 m in u tes at a tim e in n u rsery sch ool. Sh e
o ten gets ou t o h er seat to walk arou n d th e room or to p lay on th e loor. Th e girl p lays with
th e oth er ch ildren bu t re u ses to sh are h er toys with th em . Ph ysical exam in ation is u n rem ark-
able. Th e b est exp lan ation or th is ch ild’s beh avior is
(A) typ ical age-ap p rop riate beh avior
(B) atten tion -de icit/ h yp eractivity disord er (ADHD)
(C) au tism sp ectru m d isorder (ASD)
(D) op p osition al d e ian t d isord er
(E) Rett’s syn d rom e
(See “An sw ers an d Explan ation s” at en d of ch apter.)

I. AUTISM SPECTRUM AND RELATED DISORDERS


A. Overview
1. Au tism sp ectru m disorder an d related disorders are ch aracterized by th e failure to acquire
or th e early loss o social skills an d di icu lties with language , resu ltin g in li elon g p rob-
RISE USMLE NEPAL

lem s in social and occupational u n ction in g.


2. Th ese disorders are not reversible . Man agem en t in volves behavioral therapy to in crease
social an d com m u n icative skills, d ecrease beh avior p roblem s (e.g., sel -in ju ry), an d
im p rove sel -care skills, as well as su p p ortive th erapy an d cou n selin g to paren ts (see
Ch ap ter 17).

B. Autism spectrum disorder (ASD)


1. Ch aracteristics o ASD in clu de:
a. Sign i ican t p roblem s with communication (d esp ite n orm al h earin g).
b. Sign i ican t p roblem s orm in g social relationships (in clu din g th ose with caregivers).
c. Restricted range of interests ; do n ot p lay im agin ative gam es or p oin t at objects o
in terest.
d. Repetitive, purposeless behavior (e.g., sp in n in g, sel -in ju ry).
e. Below-n orm al in telligen ce in 25%–75% o ch ildren with ASD.
f. Unusual abilities in som e ch ildren (e.g., excep tion al m em ory or calcu lation skills).
Th ese are re erred to as savant skills .

160
Chapter 15 Psychiatric Disorders in Children 161

2. ASD is classi ied in th e DSM-5 by severity level.


a. Level 1: Requ irin g su p p ort.
b. Level 2: Requ irin g su bstan tial su p p ort.
c. Level 3: Requ irin g very su bstan tial su p p ort.
d. In Levels 2 an d 3, th ere are m arked de icits in verbal an d n on verbal com m u n ication .
e. In Level 1, th e in d ivid u al can sp eak in u ll sen ten ces an d com m u n icate, alth ou gh con -
versation al lan gu age skills are o ten im p aired .
3. Occurrence of ASD
a. It occu rs in alm ost 1% o ch ild ren an d ad u lts.
b. It b egin s in early childhood.
c. Th e disorder is ou r tim es m ore com m on in boys .
HELP OTHERS SO THAT GOD WILL HELP YOU.

4. Abn orm alities th at give clu es or th e neurobiological etiology (n o p sych ological cau ses
h ave been iden ti ied ) o ASD in clu de:
a. Cerebral dysfunction; 25% o p atien ts develop seizures .
b. A h istory o perinatal complications .
c. A genetic component (e.g., h igh er con cordan ce rate in m on ozygotic th an in dizygotic
twin s).
d. Eviden ce o total brain as well as amygdala overgrowth du rin g th e irst ew years o li e.
e. Abn orm alities in th e h ip p ocam p u s, ewer Pu rkin je cells in th e cereb ellu m .
f. Less circu latin g oxytocin an d dysregu lation o serotonin syn th esis.

C. Related disorders
1. Rett’s syndrome in volves:
a. Dim in ish ed social, verbal, an d cognitive development a ter u p to 4 years o n orm al
u n ction in g.
b. Occu rren ce on ly in girls (Rett’s syn d rom e is X lin ked, sp eci ically Xq28, an d a ected
m ales typ ically die b e ore birth ).
c. Stereotyp ed, hand-wringing m ovem en ts; ataxia.
d. Breathing p roblem s.
e. In tellectu al disability.
f. Motor p rob lem s later in th e illn ess.
2. Selective mutism in volves:
a. Sp eakin g in som e social situ ation s (e.g., at h om e) bu t n ot in oth ers (e.g., at sch ool).
b. More com m on occu rren ce in girls .
c. Whispering or com m u n icatin g on ly with hand gestures .
d. Selective m u tism m u st be distin gu ish ed rom typ ical sh yn ess.
RISE USMLE NEPAL

II. ATTENTION-DEFICIT/HYPERACTIVITY DISORDER.


OPPOSITIONAL DEFIANT DISORDER AND CONDUCT DISORDER
A. Overview
1. Atten tion -de icit/ h yp eractivity d isord er (ADHD ), con du ct disorder, an d op p osition al
d e ian t d isord er are ch aracterized by beh avior th at is inappropriate or th e age o th e ch ild
an d cau ses di icu lties in social relation sh ip s an d sch ool p er orm an ce.
2. Th ere is no frank intellectual disability.
3. Th ese disorders are n ot u n com m on an d are seen m ore o ten in boys .
4. Differential diagnosis in clu des depressive an d anxiety disorders .
5. I th e beh avioral abn orm alities occur only in one setting (e.g., on ly at h om e or on ly at
sch ool), th ese disorders are n ot d iagn osed , rath er, relation sh ip p rob lem s (e.g., with eith er
p aren ts or teach ers) m u st be exp lored.
6. Ch aracteristics an d p rogn oses o th ese disord ers can b e ou n d in Table 15.1.
162 BRS Behavioral Science

t a b l e 15.1 Characteristics and Prognosis of Attention-Deficit/Hyperactivity Disorder,


Conduct Disorder, and Oppositional Defiant Disorder

Attention-Deficit/Hyperactivity
Disorder (ADHD) Conduct Disorder Oppositional Defiant Disorder

Characteristics (must be present in at least two settings, e.g., at home and at school)
Hyperactivity Behavior that grossly violates social Behavior that, while defiant, negative,
Inattention norms (e.g., torturing animals, and noncompliant, does not grossly
Impulsivity stealing, truancy, fire setting) violate social norms (e.g., anger,
Carelessness argumentativeness, resentment
Propensity for accidents toward authority figures)
History of excessive crying, high
HELP OTHERS SO THAT GOD WILL HELP YOU.

sensitivity to stimuli, and irregular


sleep patterns in infancy
Symptoms present before age 12
Prognosis
Hyperactivity is the first symptom to Can begin in childhood (ages 6–10) or Gradual onset, usually before age 8
disappear as the child reaches adolescence (no symptoms prior
adolescence to age 10)
Risk for conduct disorder and Risk for criminal behavior, antisocial A significant number of cases progress
oppositional defiant disorder personality disorder, substance use to conduct disorder
disorders, and depressive disorders
in adulthood
Most children show remission by Most children show remission by Most children show remission by
adulthood adulthood adulthood

B. Etiology
1. Genetic factors are in volved. Relatives o ch ildren with con du ct disorder an d ADHD h ave
an in creased in cid en ce o th ese disord ers an d o antisocial personality disorder an d sub-
stance use disorders .
2. Alth ou gh evid en ce o seriou s stru ctu ral p rob lem s in th e brain is n ot p resen t, ch ildren
with con d u ct disorder an d ADHD m ay h ave minor brain dysfunction.
3. Su bstan ce u se d isord ers, seriou s p aren tal discord, an d dep ressive disorders are seen in
som e p aren ts o ch ild ren with th ese d isord ers; becau se o th eir p roblem atic beh avior,
th ese ch ildren are also more likely to be abused by p aren ts or caretakers.
4. Th ere is no scientific basis or claim s o an association between ADHD an d eith er im p rop er
diet (e.g., excessive su gar in take) or ood allergy (e.g., arti icial colors or lavors).
RISE USMLE NEPAL

C. Management
1. Ph arm acologic treatm en t or ADHD con sists o u se o cen tral n ervou s system (CNS)
stimulants in clu din g m eth ylp h en idate (Ritalin , Con certa), dextroam p h etam in e su l ate
(Dexed rin e), a com bin ation o am p h etam in e an d dextroam p h etam in e (Ad d erall), an d
dexm eth ylp h en id ate (Focalin ). Atomoxetine (Strattera) is a norepinephrine reuptake inhibi-
tor, also in dicated or ADHD.
a. For ADHD, CNS stim u lan ts ap p aren tly h elp reduce activity level and increase attention
span an d th e ab ility to con cen trate; an tid ep ressan ts also m ay b e u se u l.
b. Sin ce stim u lan t dru gs decrease appetite (see Ch ap ter 9), th ey m ay in h ibit growth an d
lead to failure to gain weight; both growth an d weigh t u su ally retu rn to n orm al on ce th e
ch ild stop s takin g th e m edication .
2. Family therapy is th e m ost e ective m an agem en t or con du ct disorder an d op p osition al
de ian t disord er (see Ch ap ter 17).
Chapter 15 Psychiatric Disorders in Children 163

III. OTHER DISORDERS OF CHILDHOOD


A. Tourette’s disorder
1. Tou rette’s disorder is ch aracterized by involuntary movements and vocalizations (tics) th at
m ay in clu de th e in volu n tary u se o p ro an ity (cop rolalia). Wh ile th ese beh aviors can be
con trolled brie ly, th ey m u st u ltim ately b e exp ressed .
2. Th e disorder, wh ich is lifelong and chronic , b egin s b e ore age 18. It u su ally starts with a
m otor tic (e.g., acial grim acin g) th at ap p ears between ages 7 an d 8.
3. Th e disorder is th ree tim es more common in males an d h as a strong genetic component.
4. Th ere is a gen etic relation sh ip between Tou rette’s disorder an d both ADHD an d obsessive–
HELP OTHERS SO THAT GOD WILL HELP YOU.

compulsive disorder (see Ch ap ter 13).


5. Wh ile th e m an i estation s are beh avioral, th e etiology o Tou rette’s d isord er is neurologic . It
is b elieved to in volve dys u n ction al regu lation o dopamine in th e caudate nucleus an d is
com m on ly m an aged with typ ical antipsychotic agents (e.g., h alop erid ol, p im ozid e) as well
as atyp ical agen ts, (e.g., risp eridon e). In m ilder cases, alp h a 2-adren ergic agon ists agen ts
su ch as clonidine also are h elp u l.

B. Separation anxiety disorder


1. O ten in correctly called sch ool p h ob ia, b ecau se th e ch ild re u ses to go to sch ool, th is d is-
order is ch aracterized by an overwhelming fear of loss of a major attachment figure , p articu -
larly th e m oth er.
2. Th e ch ild o ten com p lain s o physical symptoms (e.g., stom ach p ain or h eadach e) to avoid
goin g to sch ool an d leavin g th e m oth er.
3. Th e m ost e ective m an agem en t o a ch ild with th is disorder is to h ave a p aren t accom -
p an y th e ch ild to sch ool an d th en , wh en th e ch ild is m ore com ortable, gradu ally decrease
th e p aren t’s tim e sp en t at sch ool.
4. In dividu als with a h istory o sep aration an xiety disorder in ch ildh ood are at greater risk
or an xiety disorders in adu lth ood, p articu larly agoraphobia.

C. Elimination disorders: Enuresis and encopresis


1. Typ ically, m ost ch ild ren are b owel an d bladder train ed by age 3 years.
2. Th e elim in ation disorders encopresis (soilin g) an d enuresis (wettin g) are n ot diagn osed
u n til a ter age 4 years an d 5 years, resp ectively.
3. A ter m ed ical actors (e.g., u rin ary tract in ection ) are ru led ou t, th e m ost com m on cau se
o en u resis is physiological immaturity (see Ch ap ter 1).
4. Management o n igh ttim e en u resis (in ord er o u tility) in clu des:
RISE USMLE NEPAL

a. Restricting fluids a ter din n er.


b. Use o a bell and pad ap p aratu s. A p ad th at can sen se m oistu re is p laced u n d er th e
ch ild at n igh t. I th e p ad becom es wet, a b u zzer goes o wh ich waken s th e ch ild . By
n egative rein orcem en t (see Ch ap ter 7), th e ch ild even tu ally wakes be ore wettin g at
n igh t.
c. Use o a p h arm acologic agen t su ch as desmopressin acetate (a syn th etic an alog o
an tid iu retic h orm on e) or a cyclic an tidep ressan t su ch as imipramine at b ed tim e. Both
agen ts redu ce n octu rn al u rin e ou tp u t; desm op ressin is p re erred becau se it h as ewer
adverse e ects.
Review Test

Directions: Each o th e n u m b ered item s or in com p lete statem en ts in th is section is ollowed by


an swers or by com p letion s o th e statem en t. Select th e one lettered an swer or com p letion th at
is best in each case.
HELP OTHERS SO THAT GOD WILL HELP YOU.

Questions 1 and 2 Questions 4–7

Sin ce th e age o 8, a 15-year-old girl with n or- A 9-year-old boy with norm al intelligence re-
m al in telligen ce an d social skills h as sh own a quently gets into trouble at school because he
n u m b er o rep etitive m otor m ovem en ts. Sh e blurts out answers, interrupts the teacher, dis-
recen tly h as begu n to h ave ou tbu rsts in wh ich turbs the other students, and cannot seem to sit
sh e cu rses an d sh rieks. Wh en asked i sh e can still in class. He also requently injures him sel
con trol th e vocalization s an d m ovem en ts sh e during play and rarely sits through an entire m eal
says, “For a sh ort tim e on ly; it is like h oldin g at hom e. His siblings say that he is “a real pest.”
you r breath —even tu ally you h ave to let it However, the child does his schoolwork well and
ou t.” Med ical evalu ation is u n rem arkab le. behaves well when he is alone with his tutor.

1. Th is ch ild is sh owin g eviden ce o 4. Th e best exp lan ation or th is ch ild’s


(A) au tism sp ectru m d isorder (ASD) beh avior is
(B) Rett’s syn d rom e (A) op p osition al d e ian t d isord er
(C) atten tion -de icit/ h yp eractivity disord er (B) ADHD
(ADHD) (C) social d i icu lties in th e am ily
(D) Tou rette’s disorder (D) con d u ct d isord er
(E) selective m u tism (E) typ ical age-ap p rop riate beh avior

2. Th e m ost e ective m an agem en t o th e 5. Which o the ollowin g is m ost closely


u n wan ted vocalization s an d m ovem en ts is in volved in the etiology o this child’s problem ?
(A) an an tip sych otic (A) Food allergy
(B) an an tid ep ressan t (B) Im p rop er diet
(C) am ily th erapy (C) Neu rologic dys u n ction
(D) a stim u lan t (D) Excessive p u n ish m en t
RISE USMLE NEPAL

(E) in d ivid u al p sych oth erapy (E) Excessive len ien cy

3. A 4-year-old ch ild wh o h as n ever sp oken 6. O th e ollowin g, th e m ost e ective


volun tarily shows n o in terest in or con n ection m an agem en t or th is ch ild is
to his p aren ts, oth er adu lts, or oth er ch ildren . (A) an an tip sych otic
Medical exam in ation an d otological testin g (B) an an tid ep ressan t
are u n rem arkable. Th e ch ild’s m oth er tells (C) am ily th erapy
the doctor th at he p ersisten tly tu rn s on th e (D) a stim u lan t
taps to watch th e water run n in g an d th at h e (E) in dividu al p sych oth erapy
scream s an d struggles ercely when she tries
to dress h im . Which o th e ollowin g disorders 7. Th is boy is at a h igh er risk th an oth er
best ts this clin ical picture? ch ildren to d evelop wh ich o th e ollowin g
(A) ASD disord ers?
(B) Rett’s syn d rom e (A) Tou rette’s disorder
(C) ADHD (B) Sep aration an xiety disorder
(D) Tou rette’s disorder (C) Bip olar d isord er
(E) Selective m u tism (D) Con du ct disorder
(E) Sch izop h ren ia
164
Chapter 15 Psychiatric Disorders in Children 165

8. A ter startin g rst grade, a 7-year-old b oy Questions 10 and 11


o ten com p lain s o eelin g ill an d re u ses
to go to sch ool. Th ere are n o m edical Con cern ed p a ren ts o a 7-yea r-old b oy
n din gs. At h om e, th e ch ild is ap p rop riately b rin g th eir ch ild to th e p ed ia tricia n or
in teractive with h is p aren ts an d, wh en eva lu a tio n . Th ey n o te th a t ever sin ce h e
rien ds visit, h e p lays well with th em . At rst wa s an in a n t, th eir so n h a s n ever wa n ted
h is p aren ts let h im stay at h om e, bu t th ey to b e h eld , cries wh en ever h e is b a th ed , a n d
are b ecom in g in creasin gly con cern ed th at b ecom es very u p set wh en h is d a ily rou tin e
h e is allin g beh in d in h is sch oolwork. Th e is ch a n ged in an y way. Alth ou gh th e ch ild
p aren ts wan t to h ire a h om e tu tor or th e ca n n o t yet rea d , h is p aren ts rem a rk th at
ch ild. Wh at is th e p ed iatrician’s n ext step in h e ca n id en ti y th e sta te o o rigin o a n y
HELP OTHERS SO THAT GOD WILL HELP YOU.

m an agem en t? ca r licen se p late a n d a lm o st exclu sively


(A) Advise th e p aren ts to go to sch ool with p la ys with rep licas o car licen se p la tes. Th e
th e ch ild an d, over days, grad u ally ch ild’s sp ea ks in co m p lete sen ten ces a n d
decrease th e tim e th ey sp en d th ere. h a s a go od vo cab u la ry, b u t h is b eh avior
(B) Advise th e p aren ts to allow th e ch ild to seem s od d a n d h e d oes n ot m a ke eye con -
stay at h om e u n til h e in d icates th at h e is ta ct wh en sp oken to. Med ica l evalu a tio n is
com ortable sep aratin g rom th e p aren ts. u n rem a rka b le.
(C) Advise th e p aren ts to ign ore th e sch ool
re u sal, brin g th e ch ild to sch ool, an d tell 10. As an adolescen t, th is boy is likely to h ave
h im wh at tim e th ey will p ick h im u p. th e m ost di cu lty in wh ich o th e ollowin g
(D) Reassu re th e p aren ts th at h irin g a h om e areas?
tu tor or th e cu rren t sch ool year is b est (A) Payin g atten tion in sch ool
or th e ch ild. (B) Con cen tratin g on relevan t stim u li
(E) Prescrib e an an tian xiety agen t or th e (C) Carin g or p ets
ch ild to b e given on ly on sch ool d ays. (D) Makin g rien ds
(E) Con trollin g h is activity level
9. A 9-year-old b oy with n orm al in telligen ce
h as a h istory o gh tin g with oth er ch ildren 11. Th e m ajor ch aracteristic th at su ggests
an d catch in g an d tortu rin g b irds, squ irrels, th at th is ch ild h as Level 1 rath er th an Level 3
an d rabb its. Wh en asked wh y h e en gages au tism sp ectru m disorder is th at th is ch ild
in th is beh avior, h e says, “It’s ju st u n .” does not sh ow
Develop m en tal h istory an d m ed ical
(A) restricted in terests
exam in ation are u n rem arkable. Th e b est
(B) sp ecial abilities
exp lan ation o th is ch ild’s b eh avior is
(C) ocu s on keep in g u p rou tin es
(A) op p osition al d e ian t d isord er (D) p roblem s in p eer relation sh ip s
(B) ADHD (E) lan gu age delay
RISE USMLE NEPAL

(C) social d i icu lties in th e am ily


(D) con d u ct d isord er 12. Th e p aren ts an d teach er o a 7-year-
(E) typ ical age-ap p rop riate beh avior old boy n ote th at h e requ en tly sh ru gs
h is sh ou ld ers. O ten h e b lin ks h is eyes
excessively an d, at oth er tim es, sh ou ts ou t
word s or n o reason . In adu lth ood, th is ch ild
is at risk to develop wh ich o th e ollowin g
con d ition s?
(A) A seizu re disord er
(B) Obsessive–com p u lsive disorder
(C) Con du ct disorder
(D) Sch izop h ren ia
(E) ASD
166 BRS Behavioral Science

13. Th e p aren ts o a 10-year-old ch ild rep ort 14. Th e p aren ts o an 8-year-old boy rep ort
th at th e ch ild is still wettin g th e b ed . Th e th at h is beh avior at h om e is p rob lem atic. He
ch ild is very u p set ab ou t th is b ecau se h e re u ses to d o h is ch ores an d o ten gh ts with
wou ld like to go away to su m m er cam p b u t is h is 6-year-old broth er an d 11-year-old sister.
a raid th at h e will wet th e bed th ere as well. His teach ers rep ort th at h e is well beh aved
Ph ysical exam in ation is u n rem arkab le an d at sch ool, is workin g at th e exp ected level,
th e ch ild is oth erwise d evelop in g typ ically an d gets alon g well with th e oth er ch ildren .
or h is age. Beh avioral in terven tion s su ch as Medical exam in ation is u n rem arkable. Th e
lim itin g f u ids b e ore b ed an d th e b ell an d m ost likely exp lan ation or th is p ictu re is
p ad ap p aratu s h ave n ot been e ective. At (A) op p osition al d e ian t d isord er
th is tim e, wh ich o th e ollowin g is th e b est (B) atten tion -de icit/ h yp eractivity disord er
HELP OTHERS SO THAT GOD WILL HELP YOU.

ch oice or p h arm acologic m an agem en t o (C) social d i icu lties in th e am ily


en uresis in th is ch ild? (D) con d u ct d isord er
(A) Im ip ram in e (E) typ ical age-ap p rop riate beh avior
(B) Diazep am
(C) Desm op ressin acetate
(D) Acetam in op h en
(E) Asp irin
RISE USMLE NEPAL
An swers an d Exp lan ation s

Typical Board Question


The answer is A. Th is 2-year-old ch ild is sh owin g typ ical beh avior or h er age. Two-year-old ch il-
dren can n ot b e exp ected to p ay atten tion or m ore th an a ew m in u tes at a tim e, do n ot yet
p lay coop eratively with oth er ch ild ren , an d com m on ly are relu ctan t to sh are th eir toys (see also
HELP OTHERS SO THAT GOD WILL HELP YOU.

Ch ap ter 1). ADHD, con d u ct d isord er, an d op p osition al d e ian t disord er can be d i icu lt to dis-
tin gu ish rom n orm ative beh avior in ch ild ren u n der age 4 years.

1. The answer is D. 2. The answer is A. Th is girl is m ost likely to h ave Tou rette’s disorder, a
ch ron ic n eu rologic con d ition with beh avioral m an i estation s su ch as u n wan ted m otor
activity an d vocalization s. Th e vocalization s an d m otor tics can be con trolled on ly brie ly
an d th en th ey m u st be exp ressed . ASD an d Rett’s syn drom e are develop m en tal disorders
o ch ildh ood th at are ch aracterized by abn orm al social in teraction an d sp eech . ADHD
in volves n orm al develop m en t o sp eech an d social in teraction bu t di icu lty p ayin g
atten tion or sittin g still. Selective m u tism in volves volu n tary absen ce or decrease in
sp eakin g in social situ ation s. Th e m ost e ective m an agem en t or Tou rette’s disorder is
an tip sych otic m ed ication , su ch as h alop eridol. Th ere is n o eviden ce th at an tidep ressan ts
or stim u lan ts are h elp u l or con trol o m otor or vocal tics. Psych oth erapy can h elp p atien ts
with Tou rette’s disorder deal with the social p roblem s th eir disorder m ay cau se, bu t is n ot
th e m ost e ective m an agem en t or th e sym p tom s o th e d isorder.
3. The answer is A. Th is ch ild , wh o h as n ever sp oken volu n tarily an d wh o sh ows n o in terest
in or con n ection to h is p aren ts, oth er adu lts, or oth er ch ildren desp ite n orm al h earin g,
p robably h as ASD. He tu rn s on th e tap to watch th e water ru n n in g becau se, as with m an y
ch ild ren with ASD, rep etitive m otion calm s h im . An y ch an ge in h is en viron m en t, su ch as
b ein g d ressed , lead s to in ten se d iscom ort, stru gglin g, an d scream in g (see also an swer to
Qu estion 1).
4. The answer is B. 5. The answer is C. 6. The answer is D. 7. The answer is D. Th is 9-year-
old boy wh o gets in to trou ble at sch ool becau se h e distu rbs th e teach er an d th e oth er
stu d en ts, h as b eh avioral di icu lties at h om e an d with sib lin gs, an d can n ot seem to sit still
is sh owin g eviden ce o ADHD (see also an swer to Qu estion 1). Ch ildren with ADHD can
RISE USMLE NEPAL

o ten learn well wh en th ere are ew distraction s (e.g., alon e with a tu tor). Ch ildren with
con du ct d isorder sh ow b eh avior th at violates social n orm s (e.g., stealin g). In con trast,
ch ild ren with ADHD h ave trou ble con trollin g th eir beh avior bu t do n ot in ten tion ally cau se
h arm . Ch ildren with op p osition al de ian t disorder h ave p roblem s dealin g with au th ority
igu res bu t n ot with oth er ch ildren or an im als. ADHD is believed to resu lt rom n eu rologic
d ys u n ction . Alth ou gh an ecdotal evid en ce h as been p u t orward, scien ti ic stu dies h ave
n ot revealed an association b etween ADHD an d eith er im p rop er diet (e.g., excessive su gar
in take) or ood allergy (e.g., to arti icial colors or lavors). Th e disorder also is n ot a resu lt
o p aren tin g style (e.g., excessive p u n ish m en t or len ien cy). However, in part becau se o
th eir di icu lt beh avior, ch ild ren with ADHD are m ore likely to b e p h ysically ab u sed by
p aren ts. Th e m ost e ective m an agem en t or ch ildren with ADHD is u se o cen tral n ervou s
system stim u lan ts in clu d in g m eth ylp h en id ate (Ritalin ), an d d extroam p h etam in e su l ate
(Dexedrin e). Lith iu m is u sed to treat b ip olar disord er, an tidep ressan ts are u sed p rim arily
to treat dep ression , an d sedatives are u sed p rim arily to treat an xiety. Wh ile p sych oth erapy
m ay h elp th e p aren ts an d ch ild deal with th e b eh avioral sym p tom s, it is n ot th e m ost
e ective m an agem en t sin ce th e disord er is based on n eu rologic dys u n ction . Ch ildren with
ADHD are at h igh er risk th an oth er ch ild ren or op p osition al d e ian t d isord er an d con d u ct
d isorder.

167
168 BRS Behavioral Science

8. The answer is A. Th is ch ild is sh owin g evid en ce o sep aration an xiety d isorder. By th e age
o 3 to 4 years ch ild ren sh ou ld be able to sp en d som e tim e away rom p aren ts in a sch ool
settin g. Th e p ed iatrician’s b est recom m en dation is or th e p aren ts to go to sch ool with th e
ch ild an d , over days, grad u ally d ecrease th e tim e th ey sp en d th ere. Allowin g th e ch ild to
stay at h om e or h irin g a h om e tu tor will ju st in crease th e ch ild’s d i icu lty sep aratin g rom
h is p aren ts. Ph arm acologic th erapy is n ot th e irst ch oice in th e m an agem en t o th is ch ild.
9. The answer is D. Th is ch ild is sh owin g evid en ce o con d u ct disord er. Ch ild ren with th is
disord er h ave little or n o con cern or oth ers or or an im als (e.g., th is ch ild in ds tortu rin g
an im als “ u n”) (see also an swer to Qu estion 4).
10. The answer is D. 11. The answer is E. Th is ch ild wh o d oes n ot wan t to b e h eld , cries wh en
HELP OTHERS SO THAT GOD WILL HELP YOU.

h is en viron m en t is ch an ged (e.g., wh en bath ed ), an d does n ot m ake eye con tact is likely to
h ave ASD. Ch ildren with ASD h ave great d i icu lty with in terp erson al relation s. Problem s
with atten tion an d con cen tration are m ore ch aracteristic o ADHD. Ch ild ren with con du ct
d isord er ten d to h ave p oor sel -con trol an d to b reak societal ru les. Hyp eractivity m ay
b e p resen t b u t is n ot sp eci ically associated with ASD. Th e m ajor ch aracteristic th at
d i eren tiates Level 1 rom Level 3 au tism sp ectru m disorder is th at in th e latter bu t n ot in
th e orm er, th ere is d evelop m en tal lan gu age d elay an d th is ch ild sh ows relatively n orm al
lan gu age develop m en t, restricted or u n u su al in terests (h ere, in ten se ocu s on state licen se
p lates), sp ecial ab ilities, ocu s on keep in g u p rou tin es an d p roblem s in p eer relation sh ip s
are ch aracteristic o all levels o au tism sp ectru m disorder.
12. The answer is B. Th is ch ild wh o sh ows evid en ce o Tou rette’s disorder is at risk to develop
ob sessive–com p u lsive d isord er (OCD) in ad u lth ood. Both disorders in volve dys u n ction o
th e cau date n u cleu s. Seizu re disorders, con du ct disorder, sch izop h ren ia, an d ASD are n ot
p articu larly associated with Tou rette’s disorder (see also an swer to Qu estion 1).
13. The answer is C. Th e best ch oice or th e p h arm acologic m an agem en t o b edwettin g in an
older ch ild su ch as th is is d esm op ressin acetate. Im ip ram in e is also u se u l in m an agin g
en u resis b u t h as m ore sid e e ects. Diazep am (a b en zod iazep in e), u sed to treat an xiety,
an d acetam in op h en an d asp irin , u sed in th e m an agem en t o m in or p ain , are n ot u se u l in
m an agin g en u resis.
14. The answer is C. Th e m ost likely exp lan ation or wh y th is ch ild m isbeh aves at h om e bu t
n ot at sch ool is that th ere are social di icu lties in th e am ily, or exam p le, p roblem s in th e
relation sh ip between th e m oth er an d ath er. In con trast, ch ildren with con du ct disorder
sh ow b eh avior th at violates social n orm s (e.g., stealin g), ch ildren with ADHD h ave trou ble
con trollin g th eir beh avior, an d ch ild ren with op p osition al de ian t disorder h ave p roblem s
dealin g with au th ority igu res. In th ese d isorders, beh avioral d i icu lties typ ically are
RISE USMLE NEPAL

p resen t both at h om e an d at sch ool.


Biologic Th erap ies:
c ha pte r
16 Psych op h arm acology
HELP OTHERS SO THAT GOD WILL HELP YOU.

Typical Board Question


A 74-year-old m an with b ip olar disord er wh o h as been takin g lith iu m com ortably or years
rep orts bein g trem u lou s an d n au seou s over th e p ast 2 weeks. He also rep orts th irstin ess,
drowsin ess, an d m u scle weakn ess. Alth ou gh h e h as n ot in creased h is d osage o lith iu m ,
h is lith iu m level, wh ich h as con sisten tly ran ged rom 1.0 to 1.2 m Eq/ L, is n ow 2.4 m Eq/ L.
Ju st p rior to th e start o h is sym p tom s 2 weeks ago, h is p h ysician in creased th e dose o h is
blood p ressu re m edication (p rop ran olol) an d d iab etes m ed ication s (m et orm in , in su lin ) an d
started h im on an an tib iotic (am oxicillin ). At th e sam e tim e, th e p atien t in creased h is over-
th e-cou n ter p ain m edication (ibu p ro en ) du e to an exacerbation o h is osteoarth ritis. Wh ich
o th e ollowin g m ed ication s is m ost likely to cau se h is in creased lith iu m level?
(A) p rop ran olol
(B) in su lin
(C) m et orm in
(D) am oxicillin
(E) ibu p ro en
(See “An sw ers an d Explan ation s” at th e en d of th e ch apter.)

I. OVERVIEW
RISE USMLE NEPAL

A. Neurotransmitter abnormalities are in volved in th e etiology o m an y p sych iatric illn esses (e.g.,
p sych otic d isord ers, dep ressive an d bip olar disorders, an xiety disorders) (see Ch ap ter 4).

B. Alth ou gh n orm alization o n eu rotran sm itter levels by pharmacologic agents can am eliorate
m an y o th e sym p tom s, th ese agen ts do not cure p sych iatric disorders.

C. Psych op h arm acologic agen ts m ay also b e u se u l in th e treatment of symptoms of certain


medical conditions (e.g., gastroin testin al p roblem s, p ain , seizu res).

II. ANTIPSYCHOTIC AGENTS


A. Overview
1. An tip sych otic agen ts ( orm erly called n eu rolep tics or m ajor tran qu ilizers) are u sed in th e
treatm en t o schizophrenia as well as in th e m an agem en t o p sych otic sym p tom s associ-
ated with oth er p sych iatric an d p h ysical disorders.

169
170 BRS Behavioral Science

2. An tip sych otics are also u sed m ed ically to treat n au sea, h iccu p s, in ten se an xiety an d agita-
tion , an d Tou rette’s disorder.
3. Alth ou gh an tip sych otics com m on ly are taken daily by m ou th , n on adh eren t p atien ts can
be treated with lon g-actin g “depot” orm s, su ch as haloperidol decanoate or fluphenazine
decanoate ad m in istered in tram u scu larly every 2–4 weeks.
4. An an tip sych otic agen t can be classi ied as traditional (i.e., typical) or atypical dep en din g
on its m ode o action an d side e ect p ro ile.

B. Traditional an tip sych otic agen ts


1. Trad ition al an tip sych otic agen ts act p rim arily by blocking central dopamine-2 (D2) recep tors.
2. Alth ou gh n egative sym p tom s o sch izop h ren ia, su ch as with drawal, m ay im p rove with
HELP OTHERS SO THAT GOD WILL HELP YOU.

con tin u ed treatm en t, trad ition al an tip sych otic agen ts are most effective for positive symp-
toms , su ch as h allu cin ation s an d delu sion s (see Ch ap ter 11).
3. Adverse effects o typ ical an tip sych otics (Tab le 16.1)
a. Low-potency agen ts (e.g., ch lorp rom azin e [Th orazin e], th iorid azin e [Mellaril]) are
associated p rim arily with nonneurologic adverse e ects. Becau se th ere are better
ch oices (e.g., atyp ical agen ts), low-p oten cy agen ts are n ow rarely u sed .
b. High-potency agen ts (e.g., h alop eridol [Haldol], tri lu op erazin e [Stelazin e], lu p h en -
azin e [Prolixin ], p erp h en azin e [Trila on ], th ioth ixen e [Navan e], an d m olin d on e
[Moban ]) are associated p rim arily with neurologic adverse e ects.
c. Agen ts related to an tip sychotics su ch as th e dop am in e recep tor an tagon ist metoclo-
pramide (Reglan ) which is used to reduce n au sea an d vom itin g in m edical p atien ts, can
have sim ilar adverse e ects, or exam p le, akathisia an d extrapyram idal sym ptom s (EPS).

C. Atypical an tip sych otic agen ts (e.g., arip ip razole [Abili y], clozap in e [Clozaril], olan zap in e
[Zyp rexa], risp eridon e [Risp erdal], qu etiap in e [Seroqu el], zip rasidon e [Geodon ], asen ap in e
[Sap h ris], ilop eridon e [Fan ap t], lu rasidon e [Latuda], an d p alip eridon e [In vega])
1. In con trast to tradition al an tip sych otic agen ts, a m ajor m ech an ism o action o atyp i-
cal an tip sych otics ap p ears to be on serotonergic systems . Th ey also a ect dop am in ergic
recep tors in addition to D 2 (e.g., D 1, D 3, an d D 4).

t a b l e 16.1 Adverse Effects of Typical Antipsychotic Agents

System Adverse Effects

Nonneurologic Adverse Effects—More Common with Traditional, Low-potency Agents


Circulatory Orthostatic (postural) hypotension
RISE USMLE NEPAL

Electrocardiogram abnormalities (e.g., prolongation of QT and PR intervals)


Endocrine Increase in prolactin level resulting in gynecomastia (breast enlargement), galactorrhea, erectile
dysfunction, amenorrhea, and decreased libido
Hematologic Leukopenia, agranulocytosis
Hepatic J aundice, elevated liver enzyme levels
Dermatologic Skin eruptions, photosensitivity, and blue-gray skin discoloration
Ophthalmologic Irreversible retinal pigmentation (particularly thioridazine)
Anticholinergic Peripheral effects: dry mouth, constipation, urinary retention, and blurred vision
Central effects: agitation and disorientation
Antihistaminergic Weight gain and sedation
Neurologic Adverse Effects—More Common with Traditional, High-potency Agents
Extrapyramidal Pseudoparkinsonism (muscle rigidity, shuffling gait, resting tremor, mask-like facial expression)
Akathisia (subjective feeling of motor restlessness)
Acute dystonia (prolonged muscular spasms); more common in men under age 40
Treat with anticholinergic (e.g., benztropine) or antihistaminergic (e.g., diphenhydramine) agent
Other Tardive dyskinesia (abnormal writhing movements of the tongue, face, and body; more common
in women and after at least 6 months of treatment); to treat, substitute atypical antipsychotic
agent. Is rarely reversible
Neuroleptic malignant syndrome (see Table 16.4 for symptoms and treatment) dystonia, apathy;
more common in men and early in treatment; mortality rate about 20%
Decreased seizure threshold
Chapter 16 Biologic Therapies: Psychopharmacology 171

t a b l e 16.2 Adverse Effects of Atypical Antipsychotic Agents a

Adverse
Effect Aripiprazole Clozapine Olanzapine Risperidone Quetiapine Ziprasidone Asenapine Iloperidone Lurasidone Paliperidone

MBS ± ++++ +++ ++ +++ + ± + ± ±


EPS + ± + +++ ± ± + ± ++ ++
Prolactin ± ± ± +++ ± ± ± ± ± ++
elevation
Sedation ± ++++ +++ ++ +++ ++ ++ ++ ++ ±
Increased QT + ++ ++ ++ ++ ++++ ++ ++++ ± ++
interval
HELP OTHERS SO THAT GOD WILL HELP YOU.

EPS, extrapyramidal symptoms; MBS, metabolic syndrome (weight gain, diabetes mellitus); ±, few if any; +, mild; ++, moderate; +++, severe;
++++, very severe.
a
The assistance of Meredith Brandon, MD, in compiling this table is gratefully appreciated.

2. Som e o th e atyp ical an tip sych otics are also in dicated to treat bipolar disorder.
3. Advantages o atyp ical agen ts over tradition al agen ts
a. Atyp ical agen ts, p articu larly clozap in e, m ay be more effective wh en u sed to treat th e
negative , ch ron ic, an d re ractory symptoms o sch izop h ren ia (see Ch ap ter 11).
b. Th ey are less likely to cause adverse neurological symptoms an d dyston ias (Table 16.1)
an d so are n ow th e first-line agen ts or treatin g ch ron ic p sych iatric disorders su ch as
sch izop h ren ia.
4. Disadvantages o atyp ical agen ts
a. Atyp ical agen ts m ay in crease th e likelih ood o blood dyscrasias su ch as agranulocytosis
(very low gran u locyte cou n t lead in g to severe in ection s), with clozapine as th e m ost
p roblem atic agen t.
b. Th ey m ay also in crease th e likelih ood o seizures , m etabolic ab n orm alities lead in g to
weight gain, an tich olin ergic side e ects, an d p an creatitis.
c. Som e atyp ical agen ts h ave m ore adverse e ects th an oth ers. Table 16.2 p rovides
th e ad verse e ects or d i eren t atyp ical agen ts with resp ect to m etabolic syn drom e
in clu d in g weight gain and type 2 diabetes, EPS and prolactin elevation, sedation, an d car-
d iovascu lar e ects su ch as prolongation of the QT interval.

III. ANTIDEPRESSANT AGENTS


A. Overview
RISE USMLE NEPAL

1. Heterocyclic antidepressants (HCAs), selective serotonin reuptake inhibitors (SSRIs), selec-


tive serotonin and norepinephrine reuptake inhibitors (SNRIs), monoamine oxidase inhibitors
(MAOIs), an d atypical antidepressants are u sed to treat dep ression . Th ese agen ts also h ave
oth er clin ical u ses (Table 16.3).
2. All an tid ep ressan ts are believed to in crease th e availability o seroton in an d/ or n orep i-
n ep h rin e in th e syn ap se via in h ib ition o reu p take m ech an ism s (e.g., HCAs, SSRIs, SNRIs)
or blockad e o MAO (e.g., MAOIs), wh ich u ltim ately lead s to down-regulation of postsynap-
tic receptors an d im p rovem en t in m ood (see Ch ap ter 4).
3. All an tidep ressan ts take ab ou t 3–6 weeks to work an d all h ave equal efficacy.
4. Wh ile h eterocyclics were on ce th e m ain stay o m an agem en t, becau se o th eir m ore p osi-
tive side e ect p ro ile, SSRIs (e.g., lu oxetin e [Prozac]) are n ow u sed as first-line agents .
5. An tidep ressan t agen ts do not elevate mood in nondepressed people an d have no addiction
potential. Th ey can , h owever, precipitate manic episodes in p oten tially bip olar p atien ts.
6. Stimulants , su ch as m eth ylp h en idate or dextroam p h etam in e, also m ay be u se u l in treat-
in g d ep ression . Th ey work qu ickly an d th u s m ay h elp to im p rove m ood in term in ally ill
or elderly p atien ts. Th ey are also u se u l in p atien ts with dep ression re ractory to oth er
treatm en t an d in th ose at risk or th e develop m en t o adverse e ects o oth er agen ts or
d ep ression . Disad van tages in clu d e th eir addiction p oten tial.
172 BRS Behavioral Science

t a b l e 16.3 Antidepressant Agents (Grouped Alphabetically by Category)

Agent (Current or Former FDA Indicated Clinical Uses in Addition


Brand Name) Effects to Depression

Heterocyclic Agents (HCAs)


Amitriptyline (Elavil) Sedating Depression with insomnia
Anticholinergic Chronic pain
Prolonged QT interval
Clomipramine (Anafranil) Most serotonin specific of the HCAs Obsessive–compulsive disorder (OCD)
Doxepin (Adapin, Sinequan) Sedating, antihistaminergic Peptic ulcer disease
Anticholinergic Pruritis, i.e., itching
Imipramine (Tofranil) Orthostatic hypotension Panic disorder with agoraphobia
HELP OTHERS SO THAT GOD WILL HELP YOU.

Prolonged QT interval Enuresis


Eating disorders
Maprotiline (Ludiomil) May cause seizures Anxiety with depressive features
Nortriptyline (Aventyl, Least likely of the HCAs to cause Depression in the elderly
Pamelor) orthostatic hypotension Pruritus
Patients with cardiac diseases
Protriptyline (Vivactil) Stimulating ADHD
Narcolepsy
Selective Serotonin Reuptake Inhibitors (SSRIs)
Citalopram (Celexa) More cardiotoxic than other SSRIs OCD (paroxetine, sertraline, fluoxetine)
Low cytochrome P-450 effects Panic disorder (paroxetine, sertraline, fluoxetine)
Escitalopram (Lexapro) Most serotonin specific of the SSRIs Chronic pain
Low cytochrome P-450 effects Paraphilias
Fewer side effects than citalopram Generalized anxiety disorder (paroxetine and
Rapid symptom relief escitalopram)
Fluoxetine (Prozac, Sarafem) May cause agitation and insomnia Social anxiety disorder (paroxetine,
initially sertraline)
Sexual dysfunction Premenstrual dysphoria (Sarafem, sertraline)
May uniquely cause some weight loss PTSD (paroxetine, sertraline)
Fluvoxamine (Luvox) Currently indicated only for OCD Bulimia nervosa (fluoxetine)
Paroxetine (Paxil, Brisdelle) Most sedating SSRI Premature ejaculation
Most anticholinergic SSRIs Body dysmorphic disorder
Sexual dysfunction Hot flashes (Brisdelle)
Sertraline (Zoloft) Most likely of the SSRIs to cause
gastrointestinal disturbances (e.g.,
diarrhea)
Vortioxetine (Brintellix) Sexual dysfunction Indicated only for major depressive disorder
(vortioxetine)
Selective Serotonin and Norepinephrine Reuptake Inhibitors (SNRIs)
Duloxetine (Cymbalta) Rapid symptom relief Generalized anxiety disorder
Few sexual side effects Social anxiety disorder
RISE USMLE NEPAL

Venlafaxine (Effexor) and Rapid symptom relief Panic disorder


desvenlafaxine (Pristiq) Few sexual side effects Chronic pain (duloxetine)
Low cytochrome P-450 effects
Increased diastolic blood pressure at
higher doses
Levomilnacipran (Fetzima) Once daily dosing Indicated only for major depressive disorder
(levomilnacipran)
Monoamine Oxidase Inhibitors (MAOIs)
Isocarboxazid (Marplan) Hyperadrenergic crisis precipitated Geriatric depression
Tranylcypromine (Parnate) by ingestion of pressor amines Atypical depression
Phenelzine (Nardil) in tyramine-containing foods or Pain disorders
sympathomimetic agents Eating disorders
Panic disorder
Selegiline (Emsam: Orthostatic hypotension Social anxiety disorder
transdermal patch)
Chapter 16 Biologic Therapies: Psychopharmacology 173

t a b l e 16.3 Antidepressant Agents (Grouped Alphabetically by Category) (continued )

Agent (Current or Former Clinical Uses in Addition to Depression (FDA


Brand Name) Effects Indications)

Other Antidepressants
Amoxapine (Asendin) Antidopaminergic effects (e.g., Depression with psychotic features
parkinsonian symptoms,
galactorrhea, sexual dysfunction)
Most dangerous in overdose
Bupropion (Wellbutrin, Zyban, Insomnia Smoking cessation (Zyban)
Contrave) Seizures: avoid in eating disorder Major depressive disorder with seasonal pattern
patients who purge SSRI-induced sexual dysfunction
HELP OTHERS SO THAT GOD WILL HELP YOU.

Sweating Adult ADHD


Few adverse sexual effects Obesity (Contrave)
Mirtazapine (Remeron) Increased appetite Insomnia
Trazodone (Desyrel, Oleptro) Sedation Insomnia
Priapism
Hypotension
Vilazodone (Viibryd) Few adverse sexual effects Indicated only for major depressive disorder

7. Thyroid hormones can be u sed also in th e m an agem en t o m ood disord ers.


a. Levoth yroxin e (Syn th roid) is a syn th etic orm o th yroxin e (T4), wh ich h as m ood stab i-
lizin g e ects in p atien ts with bip olar d isorder.
b. Lioth yron in e (Cytom el) is a syn th etic orm o T4’s m etabolically active orm triiodoth y-
ron in e (T3), wh ich can au gm en t th e e ects o an tid ep ressan ts.

B. Heterocyclic Agents (HCAs)


1. HCAs block reuptake of norepinephrine an d serotonin at th e syn ap se. Som e also b lock reu p -
take o dop am in e.
a. Th ese agen ts also block m u scarin ic acetylch olin e recep tors, resu ltin g in anticholiner-
gic effects (e.g., dry m ou th , b lu rred vision , u rin e reten tion , con stip ation ); th ey are con -
train dicated in p atien ts with glaucoma .
b. Histam in e recep tors also are blocked by HCAs, resu ltin g in an tih istam in ergic e ects
(e.g., weight gain an d sedation).
2. Other adverse e ects in clu de cardiovascular e ects, such as orthostatic hypoten sion an d
QT prolon gation , an d n eurologic e ects, such as trem or, as well as weight gain , an d sexual
dys un ction .
3. Heterocyclics are dangerous in overdose.
RISE USMLE NEPAL

C. SSRIs and SNRIs


1. SSRIs selectively block the reuptake of serotonin only; SNRIs block the reuptake of both sero-
tonin and norepinephrine.
2. SSRIs an d SNRIs h ave little e ect on acetylch olin e or h istam in e system s.
3. Becau se o th is selectivity, SSRIs an d SNRIs cau se fewer side effects an d are safer in over-
dose , in th e elderly, an d in pregnancy th an h eterocyclics or MAOIs.
4. SNRIs m ay work more quickly (e.g., in 2–3 weeks) an d cau se ewer sexu al side e ects th an
SSRIs.

D. MAOIs
1. MAOIs in h ibit th e breakdown o n eu rotran sm itters by m on oam in e oxidase A (MAOA) in
th e brain in an irreversible reaction .
2. Th ese agen ts m ay be p articu larly u se u l in th e m an agem en t o atypical depression (see
Ch ap ter 12) an d treatm en t resistan ce to oth er agen ts.
3. A m ajor drawback o u sin g MAOIs is a p oten tially fatal reaction wh en th ey are taken in
con ju n ction with certain oods or m edication s. Th is reaction occu rs b ecau se
a. MAO metabolizes tyramine, a pressor, in th e gastroin testin al tract.
174 BRS Behavioral Science

b. I MAO is in h ibited, in gestion o tyramine-rich foods (e.g., aged ch eese, beer, win e,
broad b ean s, b ee or ch icken liver, an d sm oked or p ickled m eats or ish ) can in crease
tyram in e levels.
c. In crease in tyram in e can cau se elevated blood p ressure, sweatin g, h eadache, an d vom -
itin g (i.e., the noradrenergic or hypertensive crisis ), wh ich in turn can lead to stroke an d
death.
d. Use o sympathomimetic drugs (e.g., ep h edrin e, m ethylphen idate [Ritalin ], phen yleph-
rin e [Neo-Syn ephrin e], pseudoephedrin e [Suda ed]) can have the sam e e ect an d m ust
be avoided.
4. Oth er adverse e ects o MAOIs are sim ilar to th ose o th e h eterocyclics, in clu din g dan ger
in overd ose.
HELP OTHERS SO THAT GOD WILL HELP YOU.

5. The serotonin syndrome


a. MAOIs an d SSRIs or HCAs u sed togeth er as well as MAOIs u sed alon g with seroton ergic
an algesics su ch as meperidine (Dem erol) or tramadol (Ultram ) can cau se a p oten tially
atal d ru g–dru g in teraction , th e seroton in syn drom e.
b. Th is syn drom e is ch aracterized by high fever, au ton om ic in stability, h eadach e, sei-
zu res, deliriu m , n au sea, diarrh ea, vom itin g, an d m yoclon u s (m u scle sp asm s).
c. To avoid th is reaction , th e recom m en d ed washout period or an SSRI or an HCA b e ore
startin g an MAOI is 5 weeks an d 2 weeks, resp ectively.
d. See Table 16.4 or a comparison of three life-threatening syndromes (each ollowed
with a m n em on ic) associated with u se o p sych oactive m edication s: Th e n eu rolep tic
m align an t syn drom e (FALTER), seroton in syn d rom e (FADEM), an d h yp erten sive crisis
(ETHICS).

t a b l e 16.4 Comparison of Neuroleptic Malignant Syndrome, Serotonin Syndrome, and


Hypertensive Crisis

Neurotransmitter
Syndrome Basis Causative Agents Symptoms Lab Findings Management

Neuroleptic Dopamine (DA) Antipsychotics (e.g., “FALTER” Increased Stop agent


malignant decrease haloperidol) 1. Fever 1. Creatinine Supportive
syndrome Antiemetics (e.g., 2. Autonomic instability phosphokinase treatment and
metoclopromide) 3. Leukocytosis (CPK) DA agonist (e.g.,
4. Tremor 2. Liver enzymes bromocriptine)
5. Rigidity “lead pipe” 3. White and muscle
6. Mental status blood cells relaxant (e.g.,
(leukocytosis) dantrolene)
RISE USMLE NEPAL

changes
4. Myoglobin in
7. Bradykinesia
urine
8. Slow onset
Serotonin Serotonin (5-HT) SSRI + MAOI (e.g., “FADEM” No change in Stop agent
syndrome increase phenelzine) 1. Fever CPK or liver Supportive treatment
SSRI + serotonergic 2. Autonomic instability enzymes and antiserotonin
pain medication 3. Diarrhea agent (e.g.,
(e.g., meperidine, 4. Myoclonus cyproheptadine)
tramadol) 5. Mental status
SSRI + cough changes
suppressant (e.g., 6. Hyperkinesia
dextromethorphan)
7. Rapid onset
Hypertensive Norepinephrine MAOI + tyramine (in “ETHICS” Increased blood Stop agent
crisis (NE) increase food) 1. Elevated blood urea nitrogen Supportive treatment
MAOI + stimulant pressure (BUN) and hypotensive
(e.g., 2. Tyramine agent (e.g., IV
pseudoephedrine) 3. Headache phentolamine)
4. Chest pain
5. Seizures

SSRI, selective serotonin reuptake inhibitor; MAOI, monoamine oxidase inhibitor.


Chapter 16 Biologic Therapies: Psychopharmacology 175

IV. MOOD STABILIZERS


A. Lithium (carb on ate an d citrate)
1. Lith iu m is a m ood stabilizer u sed to prevent both th e m an ic an d dep ressive p h ases o
b ip olar disord er.
2. It m ay b e u sed also to increase the effectiveness of antidepressant agents in dep ressive ill-
n ess an d to control aggressive behavior (see Ch ap ter 20).
3. Adverse effects o ch ron ic u se o lith iu m in clu d e:
a. Hyp oth yroidism .
b. Trem or.
HELP OTHERS SO THAT GOD WILL HELP YOU.

c. Ren al dys u n ction lead in g to diab etes in sip id u s.


d. Cardiac con du ction p roblem s.
e. Gastric distress.
f. Mild cogn itive im p airm en t.
4. Lith iu m takes 2–3 weeks to work. An tip sych otics or ben zodiazep in es (BZs) rath er th an
lith iu m are th ere ore th e in itial treatm en t or p sych otic sym p tom s in an acu te m an ic
ep isod e.
5. Becau se o p oten tial toxicity, b lood levels o lith iu m m u st be m ain tain ed at 0.8–1.2 mEqL.
Agen ts th at can in crease lith iu m levels an d sh ou ld th ere ore be avoided in p atien ts takin g
lith iu m in clu de n on steroid al an ti-in lam m atory dru gs (NSAIDs), diu retics (e.g., h ydro-
ch loroth iazid e), an d an gioten sin -con vertin g en zym e (ACE) in h ibitors (e.g., cap top ril).
6. Con gen ital ab n orm alities (p articu larly o th e cardiovascu lar system , e.g., Ebstein’s anom-
aly) (see Table 16.6).

B. Anticonvulsants: carbamazepine (Tegretol), oxcarbazepine (Trileptal), valproic acid (Depakene,


Depakote), and others
1. An ticon vu lsan ts are also u sed to m an age b ip olar d isorder, p articu larly rapid cycling bipo-
lar disorder (i.e., m ore th an ou r ep isodes an n u ally).
2. Carbamazepine m ay b e associated with severe adverse e ects, su ch as aplastic anemia an d
agranulocytosis .
3. Valproic acid m ay b e p articu larly u se u l or treatin g bip olar sym p tom s resu ltin g rom cog-
n itive disorders (see Ch ap ter 14) an d or p rop h ylaxis o m igrain e h eadach es.
4. Adverse e ects o valp roic acid in clu d e gastroin testin al an d liver p roblem s, con gen ital
n eu ral tu be de ects (e.g., spina bifida ), an d alop ecia (h air loss).
5. Other anticonvulsant agents th at ap p ear to h ave m ood-stab ilizin g e ects in clu de lam otrig-
in e (Lam ictal), gabap en tin (Neu ron tin ), top iram ate (Top am ax), an d tiagabin e (Gab itril).
RISE USMLE NEPAL

C. Atypical antipsychotics are also in dicated in th e m an agem en t o b ip olar disord er.

V. ANTIANXIETY AGENTS
A. Benzodia zepines (BZs)
1. BZs activate b in d in g sites on th e g-aminobutyric acid A (GABAA) receptor, th ereby decreas-
in g n eu ron al an d m u scle cell irin g.
2. Th ese agen ts h ave a sh ort, in term ed iate, or lon g on set an d du ration o action an d m ay be
u sed to treat disorders oth er th an an xiety disorders (Table 16.5).
3. Th eir ch aracteristics o action are related to th eir clin ical in dication s an d th eir p oten tial
or addiction ; or exam p le, sh ort-actin g agen ts are good h yp n otics (sleep in du cers) bu t
h ave a h igh er p oten tial or ad diction th an lon ger-actin g agen ts.
4. BZs com m on ly cau se sedation bu t h ave ew oth er adverse e ects in adu lts.
5. Tolerance an d dependence m ay occu r with ch ron ic u se o th ese agen ts an d with drawal
sym p tom s can b e li e-th reaten in g.
176 BRS Behavioral Science

t a b l e 16.5 Antianxiety Agents (Listed Alphabetically by Duration of Action within Category)

Agent (Brand Name) Duration of Action Clinical Uses in Addition to Anxiety

Benzodiazepines
Clorazepate (Tranxene) Short Adjunct in management of partial seizures
Triazolam (Halcion) Short Insomnia
Alprazolam (Xanax) Intermediate Depression, panic disorder, social anxiety disorder
Lorazepam (Ativan) Intermediate Psychotic agitation, alcohol withdrawal, acute control of
seizures
Temazepam (Restoril) Intermediate Insomnia
Chlordiazepoxide (Librium) Long Alcohol withdrawal (particularly for agitation)
Clonazepam (Klonopin) Long Seizures, mania, social anxiety disorder, panic disorder,
HELP OTHERS SO THAT GOD WILL HELP YOU.

obsessive–compulsive disorder
Diazepam (Valium) Long Muscle relaxation, analgesia, seizures, alcohol withdrawal
(particularly for seizures)
Flurazepam (Dalmane) Long Insomnia
Nonbenzodiazepines
Ramelteon (Rozerem) Short Indicated only for insomnia
Zaleplon (Sonata) Short Indicated only for insomnia
Zolpidem (Ambien) Short (longer-acting “CR” Indicated only for insomnia
version available)
Eszopiclone (Lunesta) Intermediate Indicated only for insomnia
Buspirone (BuSpar) Very long Anxiety in the elderly, generalized anxiety disorder

6. Flumazenil (Mazicon , Rom azicon ) is a BZ recep tor an tagon ist th at can reverse th e e ects
o BZs in cases o overd ose or wh en BZs (e.g., m idazolam [Versed]) are u sed or sedation
d u rin g m ed ical or su rgical p roced u res.

B. Nonbenzodiazepines
1. Buspirone (Bu Sp ar), an azasp irodecan edion e, is n ot related to th e BZs.
a. In con trast to BZs, bu sp iron e is n on sedatin g an d is not associated with dependence,
addiction, or withdrawal p roblem s.
b. It is u sed p rim arily to treat con d ition s cau sin g ch ron ic an xiety, in wh ich BZ d ep en -
den ce can becom e a p rob lem (e.g., generalized anxiety disorder) (see Ch ap ter 13).
c. Bu sp iron e takes up to 2 weeks to work an d m ay n ot be accep table to p atien ts wh o are
accu stom ed to takin g th e ast-actin g BZs or th eir sym p tom s.
2. Zolpidem (Am bien ), zaleplon (Son ata), eszopiclone (Lu n esta), an d ramelteon (Rozerem ) are
sh ort-actin g agen ts u sed p rim arily to treat in som n ia (see Ch ap ter 10). Like th e BZs, th e
irst th ree o th ese agen ts act on th e GABAA recep tor. In con trast, ram elteon is a selective
RISE USMLE NEPAL

melatonin agonist.
3. Antihypertensives in clu din g β-blockers (block both α1- an d β2-adren ergic recep tors) su ch as
p rop ran olol (In deral) an d α2-ad ren ergic recep tor an tagon ists su ch as clon id in e (Catap res)
d ecrease au ton om ic h yp erarou sal an d are u sed to treat sym p tom s o an xiety (e.g., tach y-
cardia), p articu larly in p atien ts with social an xiety su ch as ear o p u blic sp eakin g.

VI. PSYCHOACTIVE MEDICATIONS IN PREGNANCY


A. Wh en wom en o ch ild bearin g age h ave p sych iatric sym p tom s, th ere are o ten qu estion s as to
wh eth er to ad m in ister p sych oactive m ed ication s du rin g p regn an cy.

B. Th e p oten tial ben e its o m an y p sych oactive agen ts m ay warran t th eir u se du rin g p regn an cy.
Table 16.6 lists th e Food an d Dru g Adm in istration (FDA) p regn an cy categories or p sych oac-
tive agen ts. For exam p le, m ost an tid ep ressan ts an d an tip sych otics are in FDA p regn an cy
category C. In con trast, b ecau se BZs m ay cau se breath in g di icu lties, “ lop py” m u scles, an d
oth er ad verse e ects in n ewb orn s o m oth ers wh o u se th em d u rin g p regn an cy, th ese agen ts
are in FDA p regn an cy categories D and X.
Chapter 16 Biologic Therapies: Psychopharmacology 177

t a b l e 16.6 Psychoactive Agents by FDA Pregnancy Category (Listed Alphabetically within


Type of Agent)

Pregnancy
Category Antipsychotics Antidepressants Mood Stabilizers Antianxiety Agents

A None None None None


B Clozapine Bupropion None Buspirone
Lurasidone Maprotiline Zolpidem
C Aripiprazole Amitriptyline Lamotrigine Eszopiclone
Asenapine Amoxapine Zaleplon
Chlorpromazine Citalopram
Fluphenazine Clomipramine
HELP OTHERS SO THAT GOD WILL HELP YOU.

Haloperidol Desipramine
Iloperidone Doxepin
Loxapine Duloxetine
Olanzapine Escitalopram
Paliperidone Fluoxetine
Perphenazine Fluvoxamine
Pimozide Imipramine
Quetiapine Mirtazapine
Risperidone Nortriptyline
Thioridazine Protriptyline
Thiothixene Sertraline
Trifluoperazine Trazadone
Ziprasidone Venlafaxine
D None Paroxetine Carbamazepine Alprazolam
Lithium Chlordiazepoxide
Valproic acid Clonazepam
Clorazepate
Diazepam
Lorazepam
Oxazepam
X None None None Flurazepam
Temazepam
Triazolam
Midazolam
Category A. No risk to the fetus in animal or human studies.
Category B. No or slight risk to the fetus in animal studies, no risk seen in human studies.
Category C. Risk to the fetus in animals; no adequate studies in humans. Potential benefit may warrant use in pregnancy.
Category D. Risk to the fetus in humans. Potential benefit may warrant use in pregnancy.
Category X. Human fetal risk outweighs potential benefits.
RISE USMLE NEPAL

VII. ELECTROCONVULSIVE THERAPY AND RELATED THERAPIES


A. Uses of electroc onvulsive therapy (ECT)
1. ECT p rovides rapid, effective, safe treatm en t or som e p sych iatric disorders.
a. It is m ost com m on ly u sed to treat major depressive disorder th at is refractory to
antidepressants .
b. ECT m ay be in dicated also or seriou s dep ression wh en rap id sym p tom resolu tion is
im p erative becau se o psychotic symptoms or suicide risk (see Ch ap ter 12).
c. ECT is p articu larly u se u l or treatin g depression in the elderly becau se it m ay be sa er
th an lon g-term u se o an tidep ressan t agen ts. It also m ay be used to treat dep ression
du rin g p regn an cy.
2. Th e m ech an ism o action o ECT is n ot kn own bu t m ay be related to alteration o n eu -
rotran sm itter u n ction in a m an n er sim ilar to th at o treatm en t with p sych oactive agen ts.

B. Administration
1. ECT in volves th e in d u ction o a generalized seizure , lastin g 25–60 secon ds, by passing an
electric current across the brain.
178 BRS Behavioral Science

2. Prior to seizu re in du ction , th e p atien t is premedicated (e.g., with atrop in e), th en adm in -
istered a sh ort-actin g gen eral an esth esia (e.g., m eth ohexital) an d a m u scle relaxan t (e.g.,
su ccin ylch olin e) to p reven t in ju ry d u rin g th e seizu re.
3. Im p rovem en t in m ood typ ically begins after a few ECT treatments . A m axim u m resp on se to
ECT is u su ally seen a ter 5–10 treatm en ts given over a 2- to 3-week p eriod.

C. Problems associated with ECT


1. Th e m ajor adverse e ects o ECT are memory problems .
2. Increased intracranial pressure or recen t (with in 2 weeks) myocardial infarction is a relative
contraindication or ECT.
3. Th e m ortality rate associated with ECT is very low an d is com p arable to th at associated
HELP OTHERS SO THAT GOD WILL HELP YOU.

with th e in du ction o gen eral an esth esia.

D. Other somatic therapies


Transcranial magnetic stimulation (TMS) is a th erapy in wh ich an electric cu rren t is ap p lied to
th e scalp to gen erate a m agn etic ield abou t 2 cm deep th at stim u lates cortical in tern eu ron s
lyin g p arallel to th e b rain su r ace. TMS h as b een ap p roved by th e FDA an d ap p ears to be
u se u l in som e p atien ts with m ajor dep ressive d isorder an d obsessive–com p u lsive disorder.
RISE USMLE NEPAL
Review Test

Directions: Each o th e n u m b ered item s or in com p lete statem en ts in th is section is ollowed by


an swers or by com p letion s o th e statem en t. Select th e one lettered an swer or com p letion th at
is best in each case.
HELP OTHERS SO THAT GOD WILL HELP YOU.

Questions 1 and 2 4. Th e side e ect described in Qu estion 3 is


best treated in itially
A 22-year-old m an with sch izop h ren ia wh o (A) by ch an gin g to a low-p oten cy or atyp ical
h as been takin g an an tip sych otic or th e p ast an tip sych otic agen t
3 m on th s rep orts th at recen tly h e h as exp e- (B) with an an tian xiety agen t
rien ced an u n com ortable sen sation in h is (C) with an an tidep ressan t agen t
arm s an d legs du rin g th e d ay an d m u st con - (D) with an an ticon vu lsan t
stan tly m ove th em . Becau se o th is, h e can sit (E) by stop p in g th e an tip sych otic agen t
still or on ly a ew m in u tes at a tim e.

1. Th is m edication side e ect is best Questions 5 and 6


describ ed as
(A) restless legs syn drom e A 25-year-old p atien t wh o h as taken h alop er-
(B) n eu rolep tic m align an t syn drom e idol or th e p ast 2 m on th s is brou gh t to th e
(C) akath isia h osp ital with a tem p eratu re o 104°F, blood
(D) tard ive dyskin esia p ressu re o 190/ 110, an d m u scu lar rigidity.
(E) acu te d yston ia
(F) p seu dop arkin son ism 5. Th ese sign s in dicate th at th e p atien t h as
an an tip sych otic m edication side e ect
2. Th e an tip sych otic agen t th at th is p atien t kn own as
is m ost likely to be takin g is
(A) restless legs syn drom e
(A) risp eridon e (B) n eu rolep tic m align an t syn drom e
(B) th ioridazin e (C) akath isia
(C) olan zap in e (D) tardive dyskin esia
(D) h alop eridol (E) acu te d yston ia
(E) clozap in e (F) p seu dop arkin son ism
RISE USMLE NEPAL

Questions 3 and 4 6. Th e side e ect described in Qu estion 5 is


best treated in itially
A 54-year-old wom an with sch izop h ren ia (A) by ch an gin g to a low-p oten cy
wh o h as been takin g a h igh -p oten cy an tip sy- an tip sych otic agen t
ch otic agen t or th e p ast 5 years h as b egu n to (B) with an an tian xiety agen t
sh ow in volu n tary ch ewin g an d lip -sm ackin g (C) with an an tidep ressan t agen t
m ovem en ts. (D) with an an ticon vu lsan t
(E) by stop p in g th e an tip sych otic agen t
3. Th is sign in dicates th at th e p atien t is
exp erien cin g a sid e e ect o an tip sych otic
m ed ication kn own as
(A) restless legs syn drom e
(B) n eu rolep tic m align an t syn drom e
(C) akath isia
(D) tard ive dyskin esia
(E) acu te d yston ia
(F) p seu dop arkin son ism

179
180 BRS Behavioral Science

7. A 40-year-old m an p resen ts to th e 11. A 40-year-old bu sin essm an wh o h as


em ergen cy d ep artm en t with a 4-day h istory b een a p h ysician’s p atien t or th e p ast 5
o vom itin g an d d iarrh ea. Th e p atien t years asks h er or a m edication to h elp h im
h as elevated blood p ressu re an d bod y sleep on an overn igh t igh t to Au stralia. Th e
tem p eratu re as well as m yclon u s. Blood an d p atien t’s gen eral h ealth is good, an d h e h as
u rin e tests are with in n orm al lim its, an d n o h istory o a su b stan ce u se d isord er. O th e
th ere is n o eviden ce o in ection . Th e p atien t, ollowin g, th e best agen t or th is u se is
who h as been on 40 m g/ daily o u oxetin e (A) zalep lon
or years, h as recen tly started to take a n ew (B) lu razep am
m ed ication or back p ain . Wh ich o th e (C) clon azep am
ollowin g p ain m edication s is m ost likely to (D) bu sp iron e
HELP OTHERS SO THAT GOD WILL HELP YOU.

h ave been p rescribed or th is p atien t? (E) ch lordiazep oxid e


(A) Oxycod on e (F) bu p rop ion
(B) Hydrocod on e
(C) Gabap en tin 12. A 57-year-old m ale p atien t with a h istory
(D) Tram adol o alcoh ol u se disorder h as decided to
(E) Ibu p ro en stop drin kin g. O th e ollowin g, th e agen t
m ost com m on ly u sed to treat an xiety an d
8. A 30-year-old wom an tells th e p h ysician agitation associated with th e in itial stages o
th at sh e m u st drive th e rou te sh e takes h om e alcoh ol with drawal is
rom work each day at least th ree tim es to (A) zalep lon
be su re th at sh e did n ot h it an an im al in th e (B) lu razep am
road. O th e ollowin g, th e m ost ap p rop riate (C) clon azep am
lon g-term p h arm acological m an agem en t or (D) bu sp iron e
th is p atien t is (E) ch lordiazep oxid e
(A) a h igh -p oten cy an tip sych otic agen t (F) bu p rop ion
(B) an an tich olin ergic agen t
(C) an an tian xiety agen t 13. Wh ich o th e ollowin g p sych otrop ic
(D) an an tid ep ressan t agen t agen ts sh ou ld be avoided in a 30-year-old
(E) lith iu m m ale p atien t with a h istory o su bstan ce use
disorder?
9. A 45-year-old wom an p resen ts with th e (A) Diazep am
sym p tom s o a m ajor dep ressive ep isode. (B) Halop eridol
The p atien t h as n ever p reviou sly taken an (C) Flu oxetin e
an tidep ressan t. Her p h ysician decides to (D) Bu sp iron e
prescribe u oxetin e (Prozac). Th e m ost likely (E) Lith iu m
reason or th is ch oice is th at, wh en com p ared
RISE USMLE NEPAL

to a h eterocyclic an tid ep ressan t, u oxetin e 14. An 80-year-old m an is brought to the


(A) is m ore e ective em ergency room by his wi e. The m an, who
(B) works aster has a history o depression and suicidal
(C) h as ewer side e ects behavior, re uses to eat and states that li e is
(D) is less likely to cau se addiction n ot worth living anym ore. The m edical exam is
(E) is lon ger lastin g unrem arkable. Consultations with his prim ary
care physician and a consulting psychiatrist
10. Th e best ch oice o an tian xiety agen t or a reveal that the patient has n ot respon ded
40-year-old wom an with gen eralized an xiety to at least three di erent antidepressan t
disord er an d a h istory o ben zod iazep in e m edications that he has taken in adequate
addiction is doses and or adequate tim e periods in the past
(A) zolp idem 2 years. The m ost appropriate next step in the
(B) lu razep am m anagem ent o this patient is to recom m end
(C) clon azep am (A) diazep am
(D) bu sp iron e (B) electrocon vu lsive th erapy (ECT)
(E) ch lordiazep oxid e (C) p sych oth erapy
(F) bu p rop ion (D) bu sp iron e
(E) lith iu m
Chapter 16 Biologic Therapies: Psychopharmacology 181

15. A 30-year-old m an with sch izop h ren ia 19. Wh at is th e an tidep ressan t agen t
h as been very with d rawn an d ap ath etic or m ost likely to cau se p ersisten t erection s
m ore th an 10 years. He n ow is takin g an (p riap ism ) in a 40-year-old m ale p atien t?
an tip sych otic agen t th at is h elp in g h im to (A) Ven la axin e
b e m ore ou tgoin g an d sociable. However, (B) Tran ylcyp rom in e
th e p atien t is exp erien cin g seizu res an d (C) Trazodon e
agran u locytosis. Th e an tip sych otic agen t (D) Doxep in
th at th is p atien t is m ost likely to be takin g is (E) Am oxap in e
(A) risp eridon e (F) Flu oxetin e
(B) th ioridazin e (G) Nortrip tylin e
(C) olan zap in e (H) Im ip ram in e
HELP OTHERS SO THAT GOD WILL HELP YOU.

(D) h alop eridol


(E) clozap in e 20. Wh ich o th e ollowin g an tidep ressan t
agen ts is m ost likely to cau se gyn ecom astia
Questions 16 and 17 an d p arkin son ian sym p tom s in a 45-year-old
m ale p atien t?
A 30-year-old p atien t is b rou gh t to th e em er- (A) Ven la axin e
gen cy dep artm en t a ter bein g ou n d ru n n in g (B) Tran ylcyp rom in e
down th e street n aked . He is sp eakin g very (C) Trazodon e
qu ickly an d tells th e p h ysician th at h e h as (D) Doxep in
ju st given h is cloth in g an d all o h is m on ey (E) Am oxap in e
to a h om eless m an . He states th at God sp oke (F) Flu oxetin e
to h im an d told h im to do th is. th e m edical (G) Nortrip tylin e
exam is u n rem arkab le. His h istory reveals (H) Im ip ram in e
th at h e is a p racticin g attorn ey wh o is m ar-
ried with th ree ch ildren . 21. Wh at is th e m ost ap p rop riate
an tidep ressan t agen t or rap id relie o th e
16. Th e m ost e ective im m ed iate sym p tom s o dep ression in a 25-year-old
m an agem en t or th is p atien t is wom an ?
(A) lith iu m (A) Ven la axin e
(B) lu oxetin e (B) Tran ylcyp rom in e
(C) am itrip tylin e (C) Trazodon e
(D) bu sp iron e (D) Doxep in
(E) h alop eridol (E) Am oxap in e
(F) Flu oxetin e
17. Th e m ost e ective lon g-term (G) Nortrip tylin e
m an agem en t or th is p atien t is (H) Im ip ram in e
RISE USMLE NEPAL

(A) lith iu m
(B) lu oxetin e 22. Wh ich o th e ollowin g an tidep ressan t
(C) am itrip tylin e agen ts is m ost likely to cau se extrem e
(D) bu sp iron e sedation ?
(E) h alop eridol (A) Ven la axin e
(B) Tran ylcyp rom in e
18. Wh at is th e m ost ap p rop riate agen t or (C) Trazodon e
a d octor to recom m en d or a 34-year-old , (D) Doxep in
overweigh t, dep ressed p atien t wh o n eed s (E) Am oxap in e
to take an an tid ep ressan t bu t is a raid o (F) Flu oxetin e
gain in g weigh t? (G) Nortrip tylin e
(A) Ven la axin e (H) Im ip ram in e
(B) Tran ylcyp rom in e
(C) Trazodon e
(D) Doxep in
(E) Am oxap in e
(F) Flu oxetin e
(G) Nortrip tylin e
(H) Im ip ram in e
182 BRS Behavioral Science

23. A 30-year-old wom an wh o is takin g 27. A resid en t is called to assess an on cology


an an tip sych otic m ed ication rep orts th at p atien t on a gen eral m ed ical oor wh o h as
sh e h as been exp erien cin g u id disch arge d evelop ed a m u scle sp asm cau sin g h er n eck
rom th e n ip p les. Wh ich o th e ollowin g to twist u n con trollably to th e le t. Th e in tern
h orm on es is m ost likely to be resp on sib le or evalu ates th e p atien t’s list o m edication s
th is p rob lem ? an d con clu des th at h er n ew sym p tom s are
(A) Progesteron e p robab ly d u e to wh ich o th e ollowin g?
(B) Testosteron e (A) Asp irin
(C) Prolactin (B) Digoxin
(D) Estrogen (C) Eryth rom ycin
(E) Cortisol (D) Flu oxetin e
HELP OTHERS SO THAT GOD WILL HELP YOU.

(E) Metoclop ram id e


24. A 35-year-old m an wh o h as b een takin g
h alop eridol or th e last year d evelop s a 28. A 23-year-old m an with a sch izop h ren ia
restin g trem or, m ask-like acial exp ression , h as been m ain tain ed on a low dose o
an d d i f cu lty in itiatin g b od y m ovem en ts. h alop eridol or th e p ast 3 years. For th e
A ter redu cin g th e h alop eridol dose, th e n ext p ast m on th , h is sym p tom s o p aran oia an d
step th e p h ysician sh ou ld take to relieve au ditory h allu cin ation s h ave been m ore
th ese sym p tom s is to give th e p atien t p rom in en t, an d h is p sych iatrist decides
(A) a h igh -p oten cy an tip sych otic agen t to in crease h is d ose o h alop erid ol. Th e
(B) an an tich olin ergic agen t p atien t’s m oth er calls th e p sych iatrist,
(C) an an tian xiety agen t con cern ed th at h e seem s slower th an u su al
(D) an an tid ep ressan t agen t an d h as a f n e restin g trem or o h is u p p er
(E) lith iu m extrem ities. Fu rth erm ore, th e p atien t
com p lain s th at h e eels “sti .” Th e rest o
25. A 45-year-old m an p resen ts in th e th e m ed ical exam is u n rem arkable. Th is
em ergen cy room with sin u s tach ycard ia clin ical p ictu re su ggests th at th e p atien t is
(112/ b p m ), atten in g o T waves an d exp erien cin g wh ich o th e ollowin g?
p rolon ged QT in terval. Th e p atien t tells (A) Ben ign essen tial trem or
th e p h ysician th at h e is takin g “n erve p ills.” (B) An tip sych otic-in du ced p arkin son ism
Wh ich o th e ollowin g m ed ication s is th is (C) Neu rolep tic m align an t syn d rom e
p atien t m ost likely to b e takin g? (D) Parkin son’s disease
(A) Bu p rop ion (E) Tardive dyskin esia
(B) Flu oxetin e
(C) Lorazep am
(D) Valp roic acid Questions 29 and 30
(E) Im ip ram in e
RISE USMLE NEPAL

A 32-year-old wom an an d h er h u sban d tell


26. A 45-year-old wom an with sch izop h ren ia th e p h ysician th at th ey wan t to h ave a baby,
h as been takin g an atyp ical an tip sych otic or bu t th ey are con cern ed ab ou t th e im p act
th e p ast year. Sin ce startin g th e m edication , th is will h ave on th e wi e’s em otion al state.
sh e h as gain ed 35 p ou n d s, h as develop ed Sh e h as h ad th ree m ajor dep ressive ep i-
diabetes m ellitu s, an d sh ows a p rolon ged QT sodes with in th e p ast 10 years th at h ave been
in terval. Becau se o th ese m ed ication sid e treated su ccess u lly with an tidep ressan t
e ects, h er p h ysician wou ld like to switch m ed ication . Th e wom an is con cern ed th at
h er to a di eren t atyp ical agen t. O th e sh e m ay b ecom e d ep ressed wh ile p regn an t
ollowin g atyp ical agen ts, wh ich is likely to an d will be u n able to take an y m ed ication
be th e b est ch oice or th is p atien t? becau se o th e p regn an cy.
(A) Qu etiap in e
(B) Zip rasid on e
(C) Arip ip razole
(D) Clozap in e
(E) Olan zap in e
Chapter 16 Biologic Therapies: Psychopharmacology 183

29. Wh ich o th e ollowin g is th e m ost 31. Wh at is th e p rim ary m ech an ism o action
ap prop riate resp on se by th e p h ysician ? o the dru g o ch oice to treat bu lim ia in a
(A) “Dep ression associated with p regn an cy 29-year-old wom an ?
is u n related to m ajor dep ression ; (A) Ap p etite su p p ression
th ere ore, you are n ot at greater risk or (B) An tiem etic
bein g dep ressed wh ile p regn an t.” (C) Norep in ep h rin e reu p take in h ib ition
(B) “Th e risk or dep ression is greatest (D) Seroton in reu p take in h ibition
a ter d elivery an d dep ression d u rin g (E) Dop am in e an tagon ism
p regn an cy can o ten be sa ely treated.”
(C) “Th e risk or dep ression is greatest 32. A 26-year-old m an p resen ts to th e
du rin g p regn an cy, bu t ECT is qu ite sa e.” em ergen cy dep artm en t with elevated blood
HELP OTHERS SO THAT GOD WILL HELP YOU.

(D) “Sin ce you h ave b een sym p tom - ree p ressu re, sweatin g, h eadach e, an d vom itin g.
or th e p ast year, you sh ou ld n ot b e at His com p an ion tells th e p h ysician th at th e
greatest risk or dep ression th an th e p atien t becam e ill at a p arty wh ere h e ate
n orm al p op u lation .” p izza an d d ran k alcoh olic p u n ch . Th e dru g
(E) “We will n eed to ollow you closely sin ce th at th is p atien t is m ost likely to be takin g is
th e su icide rate is h igh er or wom en wh o (A) lu oxetin e
are p regn an t th an or wom en wh o are (B) lith iu m
n ot p regn an t.” (C) n ortrip tylin e
(D) tran ylcyp rom in e
30. I th e doctor decides to p rescribe an (E) h alop eridol
an tidep ressan t or th is p atien t du rin g
p regn an cy, wh ich o th e ollowin g SSRIs
sh ou ld b e avoid ed?
(A) Citalop ram
(B) Escitalop ram
(C) Flu oxetin e
(D) Paroxetin e
(E) Sertralin e
RISE USMLE NEPAL
An swers an d Exp lan ation s

Typical Board Question


The answer is E. Com b in ation s o lith iu m an d h igh doses o n on steroidal an ti-in lam m atory
agen ts su ch as ibu p ro en can lead to in creased seru m lith iu m levels, p articu larly in elderly
p atien ts su ch as th is 74-year-old m an . Sym p tom s o lith iu m toxicity in clu de trem u lou sn ess,
HELP OTHERS SO THAT GOD WILL HELP YOU.

n au sea, th irstin ess, d rowsin ess, an d m u scle weakn ess. Lith iu m levels also m ay be in creased
with th e u se o diu retics or ACE in h ibitors bu t are n ot sp eci ically in creased by th e u se o p ro-
p ran olol, in su lin , m et orm in , or am oxicillin .

1. The answer is C. 2. The answer is D. Th e sym p tom th at th is p atien t d escrib es is akath isia,
a su b jective, u n com ortab le eelin g o m otor restlessn ess related to u se o som e
an tip sych otics. Restless legs syn drom e also in volves u n com ortable sen sation s in th e legs,
bu t it is a sleep disord er (see Ch ap ter 10), wh ich cau ses di icu lty allin g an d stayin g asleep.
Oth er an tip sych otic sid e e ects in clu de n eu rolep tic m align an t syn drom e (h igh ever,
sweatin g, in creased p u lse an d blood p ressu re, an d m u scu lar rigidity), p seu dop arkin son ism
(m u scle rigid ity, sh u lin g gait, restin g trem or, an d m ask-like acial exp ression ), an d tardive
dyskin esia (in volu n tary m ovem en ts in clu din g ch ewin g an d lip -sm ackin g). High -p oten cy
an tip sych otics, su ch as h alop eridol, are m ore likely to cau se th ese n eu rologic side e ects
th an low-p oten cy agen ts, su ch as th iorid azin e, or atyp ical agen ts, su ch as risp erid on e,
olan zap in e, an d clozap in e.
3. The answer is D. 4. The answer is A. Th ese in volu n tary ch ewin g an d lip -sm ackin g
m ovem en ts in dicate th at th e p atien t h as develop ed tardive dyskin esia, a seriou s an d
rarely reversib le sid e e ect o treatm en t with an tip sych otic m edication (see also an swer to
Qu estion 1). Tardive dyskin esia u su ally occu rs a ter at least 6 m on th s o startin g a h igh -
p oten cy an tip sych otic an d is b est treated by ch an gin g to a low-p oten cy or atyp ical agen t;
stop p in g th e an tip sych otic m ed ication will exacerbate th e sym p tom s.
5. The answer is B. 6. The answer is E. High body tem p eratu re an d blood p ressu re an d
m u scu lar rigidity in d icate th at th e p atien t h as d evelop ed an an tip sych otic m edication
sid e e ect kn own as n eu rolep tic m align an t syn drom e (see also an swer to Qu estion 1).
Neu rolep tic m align an t syn drom e is seen m ost com m on ly with h igh -p oten cy an tip sych otic
RISE USMLE NEPAL

treatm en t an d is b est relieved by stop p in g th e an tip sych otic m edication , p rovidin g m edical
su p p ort, an d adm in isterin g d an trolen e, a m u scle relaxan t. A ter recoverin g rom th is li e-
th reaten in g con dition , th e p atien t can b e switch ed to an atyp ical agen t sin ce th ey are less
likely th an h igh -p oten cy agen ts su ch as h alop eridol to cau se th is dan gerou s side e ect.
7. The answer is D. Com bin ation s o tram adol (Ultram ) a seroton ergic an algesic with SSRIs such
as luoxetin e can lead to th e sym p tom s th is p atien t shows, that is, the seroton in syn drom e.
Opioids such as oxycodon e an d h ydrocodon e an d m ood stabilizers su ch as gabap en tin or
ibu p ro en are un likely to p roduce this syn drom e wh en com bin ed with an SSRI.
8. The answer is D. Th e m ost e ective p h arm acological treatm en t or th is p atien t wh o h as
ob sessive–com p u lsive d isorder is an an tidep ressan t, p articu larly a selective seroton in
reu p take in h ibitor (see Ch ap ter 13). An tip sych otics, an tian xiety agen ts, an d lith iu m are n ot
as ap p rop riate as an an tid ep ressan t or th is p atien t.
9. The answer is C. Th e d octor d ecid es to give th is p atien t lu oxetin e becau se, wh en
com p ared to a h eterocyclic an tidep ressan t, SSRIs su ch as lu oxetin e h ave ewer side e ects.
Heterocyclics an d SSRIs h ave equ al e icacy, equ ivalen t sp eed o action , an d equ ivalen t
len gth o action . Neith er SSRIs n or h eterocyclics are likely to lead to addiction .

184
Chapter 16 Biologic Therapies: Psychopharmacology 185

10. The answer is D. Th e b est ch oice o an tian xiety agen t or a 40-year-old p atien t
with gen eralized an xiety disorder an d a h istory o BZ addiction is bu sp iron e, a
n on ben zodiazep in e with very low addiction p oten tial. Ben zodiazep in es su ch as
lu razep am , clon azep am , an d ch lordiazep oxide h ave h igh er addiction p oten tial th an
bu sp iron e. Bu p rop ion is an an tid ep ressan t, wh ich is also u sed or sm okin g cessation .
Zolp id em is a n on ben zod iazep in e sleep agen t.
11. The answer is A. Zalep lon , a n on ben zodiazep in e sleep agen t, is th e best ch oice to aid
sleep on an overn igh t ligh t. Ben zod iazep in es h ave h igh er add iction p oten tial th an agen ts
su ch as zalep lon . Bu sp iron e h as little ad d iction p oten tial b u t d oes n ot cau se sed ation
an d, in an y case, takes weeks to work. Bu p rop ion is an an tidep ressan t agen t an d is
n on sed atin g.
HELP OTHERS SO THAT GOD WILL HELP YOU.

12. The answer is E. Becau se it is lon g actin g an d h as relatively low addiction p oten tial or a
BZ, ch lordiazep oxide is th e an tian xiety agen t m ost com m on ly u sed to treat th e an xiety an d
agitation associated with th e in itial stages o alcoh ol with drawal.
13. The answer is A. O th e listed agen ts, BZs su ch as d iazep am are m ost likely to cau se
add iction . An tip sych otics su ch as h alop eridol, an tidep ressan ts su ch as lu oxetin e, m ood
stab ilizers su ch as lith iu m , an d n on b en zod iazep in es su ch as b u sp iron e (see also an swer to
Qu estion 10) h ave little or n o ad d iction p oten tial.
14. The answer is B. Th e m ost ap p rop riate n ext step is to recom m en d a cou rse o
electrocon vu lsive th erapy (ECT) or th is elderly, severely dep ressed p atien t. ECT is a
sa e, ast, e ective treatm en t or m ajor d ep ression . Diazep am , lith iu m , bu sp iron e, an d
p sych oth erapy will n ot be e ective as ECT in relievin g th is p atien t’s su icidal dep ression
qu ickly.
15. The answer is E. Th e an tip sych otic agen t th at th is p atien t is m ost likely to be takin g
is clozap in e. Like oth er atyp ical agen ts, clozap in e is m ore e ective again st n egative
sym p tom s (e.g., with d rawal) th an trad ition al agen ts su ch as h alop eridol. However,
clozap in e is also is m ore likely to cau se seizu res an d agran u locytosis th an trad ition al
agen ts or oth er atyp icals, su ch as risp eridon e an d olan zap in e.
16. The answer is E. 17. The answer is A. Th is p atien t’s good em p loym en t an d relation sh ip
h istory su ggest th at h is p sych otic sym p tom s are an acu te m an i estation o a m an ic
ep isode. Wh ile th e m ost e ective im m ediate treatm en t or th is p atien t is a ast-actin g,
h igh -p oten cy an tip sych otic agen t, su ch as h alop eridol, to con trol h is h allu cin ation s an d
d elu sion s, lith iu m , wh ich takes 2–3 weeks to work, wou ld be m ore e ective or lon g-term
m ain ten an ce. Flu oxetin e, am itrip tylin e, an d bu sp iron e are less ap p rop riate p rim ary
RISE USMLE NEPAL

treatm en ts or th is bip olar p atien t.


18. The answer is F. In con trast to m ost an tidep ressan t agen ts, wh ich are associated with
weigh t gain , lu oxetin e (Prozac) is associated with som e weigh t loss. Th u s, it is th e m ost
ap p rop riate an tidep ressan t agen t or a p atien t wh o is ear u l o gain in g weigh t.
19. The answer is C. Trazodon e is th e agen t m ost likely to cau se p riap ism in th is p atien t.
20. The answer is E. Am oxap in e h as an tid op am in ergic action an d , th u s, is th e agen t m ost
likely to cau se gyn ecom astia as well as p arkin son ian sym p tom s in th is p atien t.
21. The answer is A. SNRIs m ay work m ore q u ickly (e.g., in 2-3 weeks) th an oth er
an tid ep ressan ts an d , as su ch , ven la axin e is a good ch oice or rap id relie o d ep ressive
sym p tom s in th is you n g wom an .
22. The answer is C. Trazadon e n ot on ly can cau se p riap ism (see also an swer to Qu estion 19)
bu t also is h igh ly sedatin g. It is th u s o ten u sed in p atien ts wh o h ave dep ression with
in som n ia.
23. The answer is C. Prolactin is th e h orm on e resp on sible or galactorrh ea, lu id disch arge
rom th e n ip p les. Galactorrh ea is m ore com m on with th e u se o low-p oten cy an tip sych otic
agen ts.
186 BRS Behavioral Science

24. The answer is B. Th is p atien t is sh owin g eviden ce o p seu dop arkin son ism , a n eu rologic
sid e e ect cau sed by excessive blockad e o p ostsyn ap tic dop am in e recep tors du rin g
treatm en t with h igh -p oten cy an tip sych otics, su ch as h alop eridol. Becau se dop am in e
n orm ally su p p resses acetylch olin e activity, givin g th e p atien t an an tich olin ergic agen t
(e.g., ben ztrop in e) will serve to in crease d op am in ergic activity an d relieve th e p atien t’s
sym p tom s. An tian xiety agen ts su ch as b en zod iazep in es can b e u sed as ad ju n cts
to an tich olin ergics, bu t an tidep ressan ts an d lith iu m are n ot e ective or reversin g
p arkin son ian sym p tom s cau sed by an tip sych otics.
25. The answer is E. TCAs su ch as im ip ram in e cau se sin u s tach ycardia, lat T waves, p rolon ged
QT in terval, an d dep ressed ST segm en ts. Bu p rop ion , lu oxetin e, lorazep am , an d valp roic
acid are less likely to cau se th ese cardiovascu lar e ects.
HELP OTHERS SO THAT GOD WILL HELP YOU.

26. The answer is C. Becau se o h er weigh t gain , typ e 2 diabetes, an d cardiovascu lar p roblem ,
th e best ch oice o atyp ical an tip sych otic agen t or th is p atien t n ow is arip ip razole.
Clozap in e an d olan zap in e carry h igh risk an d zip rasid on e an d arip ip razole carry low risk
or weigh t gain an d diabetes. However, zip rasidon e p rolon gs th e QT in terval an d so sh ou ld
be avoid ed in th is p atien t.
27. The answer is E. Metoclop ram id e (Reglan ), a gastric m otility agen t an d an tiem etic, is o ten
u sed to con trol n au sea an d vom itin g in can cer p atien ts receivin g ch em oth erapy. It h as
an tidop am in ergic p rop erties an d can cau se acu te dyston ic reaction s su ch as are occu rrin g
in th is p atien t. Man agem en t in clu des stop p in g th e m etoclop ram id e an d p rovid in g an
an tich olin ergic agen t, su ch as ben ztrop in e, or an an tih istam in e, su ch as dip h en h ydram in e,
b oth o wh ich are u su ally given in in tram u scu lar orm or im m ediate e ect. Asp irin ,
d igoxin , eryth rom ycin , an d lu oxetin e are u n likely to cau se dyston ic reaction s.
28. The answer is B. Th is p atien t wh o is slowed down an d h as a in e restin g trem or o
h is u p p er extrem ities an d sti n ess is sh owin g eviden ce o an tip sych otic-in du ced
p arkin son ism , o ten a sid e e ect o h igh d oses o h igh -p oten cy an tip sych otics su ch
as h alop erid ol. Ben ign essen tial trem or an d Parkin son’s disease are n ot related to
an tip sych otic m ed ication . Alth ou gh th ey can b oth be side e ects o h alop eridol treatm en t,
n eu rolep tic m align an t syn d rom e an d tard ive d yskin esia are ch aracterized by h igh ever
an d ab n orm al ton gu e an d acial m ovem en ts, resp ectively.
29. The answer is B. 30. The answer is D. The m ost app rop riate resp on se or th e p hysician
is to tell the patien t that the risk or depression is greater a ter than be ore delivery an d
that depression durin g pregn an cy can o ten be sa ely treated. Most an tidepressan ts are in
pregn an cy category C but two, bup rop ion an d m aprotilin e, are in category B. Discussin g
di eren tial suicide rates is n ot a h elp ul in terven tion . In an y case, the su icide rate is lower
RISE USMLE NEPAL

or wom en who are pregn an t than or those who are n ot pregn an t. While ECT is quite sa e in
pregn an cy, p sychop h arm acology is less in vasive an d usually p re erred. I the doctor decides
to p rescribe an SSRI or th is patien t durin g p regn an cy, paroxetin e (A category D agen t)
sh ould be avoided.
31. The answer is D. Th e d ru g o ch oice in th e treatm en t o bu lim ia is an SSRI su ch as
lu oxetin e. Th e action o SSRIs is seroton in reu p take in h ibition .
32. The answer is D. Th is p atien t wh o b ecam e ill at a p izza p arty is m ost likely to b e takin g
tran ylcyp rom in e, a m on oam in e oxidase in h ibitor (MAOI). Th ese agen ts can cau se a
h yp erten sive crisis i certain oods (e.g., aged ch eese, sm oked m eats, beer, an d win e) are
in gested. A p atien t wh o eats in an u n am iliar p lace (e.g., a p arty) m ay u n wittin gly in gest
orbidden oods. Th is p atien t ate p izza th at p robably con tain ed aged Parm esan ch eese
an d d ran k p u n ch th at p rob ab ly con tain ed red win e. Th is resu lted in a h yp erten sive crisis
(e.g., elevated b lood p ressu re, sweatin g, h eadach e, an d vom itin g). Flu oxetin e, lith iu m ,
n ortrip tylin e, an d h alop eridol do n ot in teract n egatively with ood.
c ha pte r
17 Psych ological Th erap ies
HELP OTHERS SO THAT GOD WILL HELP YOU.

Typical Board Question


Each tim e sh e com bs h er h air, a 20-year-old wom an with Level 3 au tism sp ectru m disorder
receives a cou p on th at can be exch an ged or dessert in th e ca eteria. Her groom in g beh avior
su b sequ en tly im p roves. Wh ich o th e ollowin g p sych ological m an agem en t tech n iqu es does
th is exam p le illu strate?
(A) Im p losion
(B) Bio eed b ack
(C) Aversive con dition in g
(D) Token econ om y
(E) Floodin g
(F) System atic desen sitization
(G) Cogn itive/ b eh avioral th erapy
(See “An sw ers an d Explan ation s” at th e en d of th e ch apter.)

I. PSYCHOANALYSIS AND RELATED THERAPIES


A. Overview
1. Psychoan alysis an d related therapies (e.g., psychoan alytically oriented psychotherapy, brie
RISE USMLE NEPAL

dyn am ic psychotherapy) are psychotherapeutic treatm ents based on Freud’s concepts o the
unconscious mind, defense mechanisms , an d transference reactions (see Chapter 6).
2. Th e cen tral strategy o th ese th erap ies is to uncover experiences th at are repressed in the
unconscious mind an d in tegrate th em in to th e p erson’s con sciou s m in d an d p erson ality.
B. Techniques used to recover repressed experiences include:
1. Free association
a. In classic p sych oan alysis, th e p erson lies on a couch in a reclin ed p osition acin g away
rom th e th erap ist an d says wh atever com es to m in d ( ree association ).
b. In th erap ies related to classic p sych oan alysis, th e p erson sits in a ch air an d talks wh ile
acin g th e th erap ist.
2. Interpretation of dreams is u sed to exam in e u n con sciou s con licts an d im p u lses.
3. Analysis of transference reactions (i.e., th e p erson’s u n con sciou s resp on ses to th e th erap ist)
is u sed to exam in e im p ortan t p ast relation sh ip s (see Ch ap ter 6).

C. Peop le wh o are ap p rop riate or u sin g p sych oan alysis an d related th erap ies sh ou ld h ave th e
ollowin g ch aracteristics:
1. Are you n ger th an 40 years o age.
2. Are in telligen t an d n ot p sych otic.
187
188 BRS Behavioral Science

3. Have good relation sh ip s with oth ers (e.g., n o eviden ce o an tisocial or borderlin e p erson -
ality disorder).
4. Have a stab le li e situ ation (e.g., n ot be in th e m idst o divorce).
5. Have th e tim e an d m on ey to sp en d on treatm en t.

D. In psychoanalysis , p eop le receive treatm en t four to five times weekly for 3–4 years ; related
th erap ies are brie er an d m ore direct (e.g., brie dyn am ic p sych oth erapy is lim ited to
12–40 weekly session s).

II. BEHAVIORAL THERAPIES


HELP OTHERS SO THAT GOD WILL HELP YOU.

A. Behavioral therapies are based on learning theory (see Chapter 7), that is, sym ptom s are relieved
by u n learn in g m aladap tive beh avior p attern s an d alterin g n egative thin kin g pattern s.

B. In con trast to p sych oan alysis an d related th erap ies, th e p erson’s h istory an d unconscious
conflicts are irrelevant an d th u s are n ot exam in ed in beh avioral th erap ies.
C. Ch aracteristics o sp eci ic b eh avioral th erap ies (e.g., system atic desen sitization , aversive
con d ition in g, lood in g an d im p losion , token econ om y, bio eedback, an d cogn itive/ beh av-
ioral th erapy) can be ou n d in Table 17.1.

t a b l e 17.1 Behavioral Therapies: Uses and Strategies

Most Common Use Strategy

Systematic Desensitization
Management of phobias (irrational In the past, through the process of classical conditioning (see Chapter 7), the
fears; see Chapter 13) person associated an innocuous object with a fear-provoking stimulus until the
innocuous object became frightening
In the present, increasing doses of the fear-provoking stimulus are paired with a
relaxing stimulus to induce a relaxation response
Because one cannot simultaneously be fearful and relaxed (reciprocal inhibition), the
person shows less anxiety when exposed to the fear-provoking stimulus in the future
Aversive Conditioning
Management of paraphilias Classical conditioning is used to pair a maladaptive but pleasurable stimulus with an
(e.g., pedophilia) or addictions aversive or painful stimulus (e.g., a shock) so that the two become associated.
(e.g., smoking) The person ultimately stops engaging in the maladaptive behavior because it
automatically provokes an unpleasant response
Flooding and Implosion
RISE USMLE NEPAL

Management of phobias The person is exposed to an actual (flooding) or imagined (implosion) overwhelming
dose of the feared stimulus.
Through the process of habituation (see Chapter 7), the person becomes
accustomed to the stimulus and is no longer afraid
Token Economy
To increase positive behavior in a person Through the process of operant conditioning (see Chapter 7), desirable behavior
who is disorganized (e.g., psychotic) (e.g., shaving, hair combing) is reinforced by a reward or positive reinforcement
or has severe autism spectrum (e.g., the token)
disorder or intellectual disability The person increases the desirable behavior to gain the reward
Biofeedback
To manage hypertension, Raynaud’s Through the process of operant conditioning, the person is given ongoing
disease, migraine and tension physiologic information (e.g., blood pressure measurement), which acts as
headaches, chronic pain, fecal reinforcement (e.g., when blood pressure drops)
incontinence, and temporomandibular The person uses this information along with relaxation techniques to mentally control
joint pain visceral changes (e.g., heart rate, blood pressure, smooth muscle tone)
Cognitive/Behavioral Therapy (CBT)
To manage mild to moderate depression, Weekly, for 15–25 wk, the person is helped to identify distorted, negative thoughts
somatic symptom disorders, eating about him- or herself
disorders The person replaces these negative thoughts with positive, self-assuring thoughts,
and symptoms improve
Dialectal behavioral therapy is a form of CBT which is particularly useful for
borderline personality disorder
Chapter 17 Psychological Therapies 189

III. OTHER THERAPIES


Other th erap ies in clu de grou p, am ily, m arital/ cou p les, sup p ortive an d in terp erson al therapy, as
well as stress m an agem en t tech n iqu es. Sp eci ic u ses o th ese th erap ies can be oun d in Table 17.2.

A. Group therapy
1. Groups with therapists
a. Grou p s o u p to ab ou t eigh t p eop le with a com m on p roblem or n egative li e exp eri-
en ce u su ally m eet weekly or 1–2 h ou rs; sh arin g th e th erap ist redu ces cost.
HELP OTHERS SO THAT GOD WILL HELP YOU.

b. Mem bers o th e grou p p rovide th e op p ortu n ity to exp ress eelin gs as well as feedback,
support, an d friendship to each oth er.
c. The therapist has little input. He or sh e acilitates an d observes th e m em bers’ in terp er-
son al in teraction s.
2. Leaderless groups
a. In a leaderless grou p, no one person is in authority.
b. Mem bers o th e grou p p rovid e each oth er with support and practical help or a sh ared
p roblem (e.g., alcoh olism , loss o a loved on e, a sp eci ic illn ess).
c. Twelve-step grou p s like Narcotics Anonymous (NA) an d Overeaters Anonymous (OA) are
b ased on th e Alcoholics Anonymous (AA) leaderless grou p m odel (see Ch ap ter 9).

B. Family therapy
1. Family systems theory
a. Fam ily th erapy is based on th e am ily system s idea th at p sych op ath ology in on e am ily
m em b er (i.e., th e id en ti ied p atien t) re lects dysfunction of the entire family system.
b. Becau se all m em bers o th e am ily cau se beh avioral ch an ges in oth er m em bers,
the family ( not the identified patient) is really the patient.
c. Strategies o am ily th erapy in clu d e id en ti yin g dyads (i.e., su b system s b etween two
am ily m em bers), triangles (i.e., d ys u n ction al allian ces b etween two am ily m em bers
again st a th ird m em ber), an d boundaries (i.e., b arriers between su b system s) th at m ay
be too rigid or too p erm eab le.
2. Specific techniques are u sed in am ily th erapy.
a. Mutual accommodation is en cou raged. Th is is a p rocess in wh ich am ily m em bers work
toward m eetin g each oth er’s n eeds.
b. Normalizing boundaries b etween su bsystem s an d red u cin g th e likelih ood o trian gles is
en cou raged .
RISE USMLE NEPAL

c. Redefining “blame” (i.e., en cou ragin g am ily m em bers to recon sider th eir own resp on -
sib ility or p roblem s) is an oth er im p ortan t tech n iqu e.

t a b l e 17.2 Uses of Group, Family, Marital/Couples, Supportive, Interpersonal, and Stress


Management Therapies
Type of Therapy Targeted Population

Group therapy People with a common problem (e.g., rape victims)


People with personality disorders or other interpersonal problems
People who have trouble interacting with therapists as authority figures in individual therapy
Family therapy Children with behavioral problems
Families in conflict
People with eating or substance use disorders
Marital/couples therapy Domestic partners with communication or psychosexual problems
Domestic partners with differences in values
Supportive therapy People who are experiencing a life crisis
People with mental illnesses who are dealing with ordinary life situations
Interpersonal therapy People with emotional difficulties owing to problems with interpersonal skills
Stress management People with anxiety disorders or stress-related somatic symptoms
190 BRS Behavioral Science

C. Supportive and interpersonal therapy


1. Su p p ortive th erapy is aim ed n ot at in sigh t in to p roblem s, bu t rath er at h elp in g p eop le eel
p rotected an d su p p orted d u rin g li e crises (e.g., seriou s illn ess o a loved on e). For p eop le
with chronic mental illnesses , su p p ortive th erapy m ay be u sed over m an y years alon g with
m edication .
2. Based on th e id ea th at p sych iatric p rob lem s su ch as an xiety an d dep ression are based
on d i icu lties in dealin g with oth ers, in terp erson al th erapy aim s to d evelop interpersonal
skills in 12–16 weekly session s.
HELP OTHERS SO THAT GOD WILL HELP YOU.
RISE USMLE NEPAL
Review Test

Directions: Each o th e n u m b ered item s or in com p lete statem en ts in th is section is ollowed by


an swers or by com p letion s o th e statem en t. Select th e one lettered an swer or com p letion th at
is best in each case.

1. A 30-year-old m an who is a raid to ride 4. Ten arth ritis p atien ts m eet on ce p er week
HELP OTHERS SO THAT GOD WILL HELP YOU.

in an elevator is p u t in to a relaxed state to talk with each oth er an d to in orm each


an d th en sh own a f lm o p eople en terin g oth er o n ew d evices an d services to h elp
elevators in a h igh -rise bu ildin g. Th is m eth od disabled p eop le with everyday tasks. Th is
o m an agem en t is based p rim arily on type o th erapy is best described as
(A) recip rocal in h ibition (A) grou p th erapy
(B) classical con d ition in g (B) leaderless grou p th erapy
(C) aversive con d ition in g (C) brie dyn am ic p sych oth erapy
(D) op eran t con d ition in g (D) am ily th erapy
(E) stim u lu s gen eralization (E) su p p ortive th erapy

2. A 28-year-old wom an join s 10 oth er 5. A 50-year-old m ale h yp erten sive p atien t is


wom en wh o h ave b een ab u sed by th eir given on goin g blood p ressu re readin gs as h e
h u sban ds. Th e wom en m eet weekly an d u ses m en tal relaxation tech n iqu es to try to
are led by a p sych oth erap ist wh o is train ed lower h is blood p ressu re. A ter ou r session s
in dom estic violen ce issu es. Th is typ e o u sin g th is tech n iqu e, h is blood p ressu re
th erapy is best described as is lower. Th is m eth od o blood p ressu re
(A) grou p th erapy redu ction is based p rim arily on
(B) leaderless grou p th erapy (A) recip rocal in h ibition
(C) brie d yn am ic p sych oth erapy (B) classical con d ition in g
(D) am ily th erapy (C) aversive con dition in g
(E) su p p ortive th erapy (D) op eran t con d ition in g
(E) stim u lu s gen eralization
3. A 9-year-old b oy wh o is an gry an d
resen t u l toward adu lts (op p osition al def an t 6. A 35-year-old m an wh o is a raid o h eigh ts
disord er; see Ch ap ter 15) m eets with a is in stru cted to stan d in th e observation
RISE USMLE NEPAL

th erap ist or 2 h ou rs each week, alon g with tower o th e Em p ire State Bu ildin g an d look
h is p aren ts an d h is sister. A ter 6 m on th s, th e down rom th e win d ow u n til h e is n o lon ger
boy’s op p osition al beh avior toward ad u lts a raid. A ter th ree visits to th e tower each
h as im p roved. Th is typ e o th erapy is best lastin g 1 h ou r, th e m an is n o lon ger a raid o
describ ed as h eigh ts. Wh ich o th e ollowin g m an agem en t
(A) grou p th erapy tech n iqu es does th is exam p le illu strate?
(B) leaderless grou p th erapy (A) Im p losion
(C) brie d yn am ic p sych oth erapy (B) Bio eed b ack
(D) am ily th erapy (C) Aversive con dition in g
(E) su p p ortive th erapy (D) Token econ om y
(E) Floodin g
(F) System atic desen sitization
(G) Cogn itive/ beh avioral th erapy

191
192 BRS Behavioral Science

7. A m an wh o is a raid to d rive is told to 10. A 5-year-old ch ild, wh o at age 2 years


im agin e drivin g a car rom th e n orth ern m ost was p layin g with a large d og wh en a ceilin g
bord er to th e sou th ern m ost b ord er o th e tile ell on h er h ead, is n ow so a raid o dogs
State o New Jersey. Even tu ally, h e is able to th at sh e re u ses to go to th e p ark b ecau se
drive with ou t ear. Wh ich o th e ollowin g d ogs are th ere. Medical exam in ation is
m an agem en t tech n iqu es does th is exam p le u n rem arkable, an d th e ch ild’s m otor, social,
illustrate? an d cogn itive develop m en t are typ ical
(A) Im p losion or h er age. To m an age th e ch ild’s ear
(B) Bio eed b ack o dogs, th e p h ysician f rst recom m en ds
(C) Aversive con dition in g th at h er ath er carry a sm all toy dog very
(D) Token econ om y gradu ally toward h er wh ile sh e is listen in g
HELP OTHERS SO THAT GOD WILL HELP YOU.

(E) Floodin g to h er avorite CD. Wh ich o th e ollowin g


(F) System atic desen sitization p sych ological th erap ies does th is exam p le
(G) Cogn itive/ b eh avioral th erapy illu strate?
(A) Im p losion
8. A 30-year-old dep ressed m an is told to (B) Bio eed b ack
rep lace each sel -dep recatin g th ou gh t with a (C) Aversive con dition in g
m en tal im age o victory an d p raise. Over th e (D) Token econ om y
n ext ew m on th s, h is dep ression gradu ally (E) Floodin g
li ts. Wh ich o th e ollowin g m an agem en t (F) System atic desen sitization
tech n iqu es does th is exam p le illu strate? (G) Cogn itive/ beh avioral th erapy
(A) Im p losion
(B) Bio eed b ack 11. Th e m ajor reason th at p atien ts wh o
(C) Aversive con dition in g could ben ef t rom p sych oan alytically
(D) Token econ om y orien ted p sych oth erapy do n ot receive it is
(E) Floodin g th at th ey o ten
(F) System atic desen sitization (A) do n ot wan t to reveal th eir h istories to
(G) Cogn itive/ b eh avioral th erapy stran gers
(B) do n ot wan t to reveal th eir p erson al
9. A 42-year-old m an with sexu al in terest p roblem s to stran gers
in ch ildren (p edop h ilia) is given an electric (C) believe th at it is exp en sive an d tim e
sh ock each tim e h e is sh own a vid eotap e con su m in g
o ch ild ren . Later, h e eels ten se arou n d (D) h ave little in terest in exp lorin g th eir
ch ildren an d avoid s th em . Wh ich o th e ch ildh ood s
ollowin g m an agem en t tech n iqu es does th is (E) do n ot eel com ortab le in th e
exam p le illu strate? th erap eu tic settin g
(A) Im p losion
RISE USMLE NEPAL

(B) Bio eed b ack


(C) Aversive con dition in g
(D) Token econ om y
(E) Floodin g
(F) System atic desen sitization
(G) Cogn itive/ b eh avioral th erapy
An swers an d Exp lan ation s

Typical Board Question


The answer is D. Th e m an agem en t tech n iqu e described h ere is token econ om y. In token econ -
om y, th e d esired b eh avior (e.g., groom in g) is rein orced by a token (e.g., a cou p on th at can be
exch an ged or d essert) an d th e p erson in creases h er beh avior to gain th e reward (e.g., d essert).
HELP OTHERS SO THAT GOD WILL HELP YOU.

In system atic desen sitization , in creasin g doses o a righ ten in g stim u lu s are p aired with a relax-
in g stim u lu s to p rovoke a relaxation resp on se in situ ation s in volvin g th e righ ten in g stim u lu s.
Floodin g is a m an agem en t tech n iqu e or p h obias in wh ich a p erson is exp osed to an overwh elm -
in g dose o th e eared stim u lu s or situ ation u n til h e or sh e is n o lon ger ear u l. In im p losion , a
p erson is exp osed to an im agin ed , rath er th an actu al, overwh elm in g dose o a eared stim u lu s
or situ ation . In bio eed back, a p erson is given on goin g p h ysiologic in orm ation , wh ich acts as
rein orcem en t. In aversive con d ition in g, a m aladap tive bu t p leasu rable stim u lu s is p aired with
a p ain u l stim u lu s so th at th e two b ecom e associated an d th e m alad ap tive b eh avior d isap p ears.
In cogn itive/ b eh avioral th erapy, a p erson is h elp ed to iden ti y distorted, n egative th ou gh ts an d
to rep lace th em with p ositive, sel -assu rin g th ou gh ts.

1. The answer is B. Th is m eth od o m an agem en t, system atic d esen sitization , is b ased on


classical con dition in g. Th e ilm o p eop le en terin g elevators in a h igh -rise bu ildin g is p aired
with relaxation . A ter con tin u ed p airin g o elevators an d relaxation , elevators will n o lon ger
in du ce ear. Later on in treatm en t, th e p erson will be en cou raged to look in to a real elevator
an d in ally to rid e in on e (an d see TBQ an d Qu estion 10).
2. The answer is A. Th is typ e o th erapy is best described as group therapy, a m an agem en t
tech n iqu e in wh ich p eop le with a com m on problem (e.g., victim s o abuse) get togeth er with
a psych otherapist. In leaderless groups, there is n o therapist or other person in authority;
m em bers o th e grou p p rovide each oth er with supp ort an d p ractical help or a shared
p roblem . Brie dyn am ic p sych oth erapy is a orm o psychoan alytically orien ted therapy in
wh ich a p erson works with a th erap ist to gain in sigh t in to th e cau se o h is or h er p roblem s. In
su p p ortive th erapy, a th erap ist h elp s a p erson eel p rotected an d su p p orted du rin g li e crises.
3. The answer is D. Th is typ e o th erapy, in wh ich a ch ild with a beh avior p roblem an d h is
am ily m eet with a th erap ist, is best described as am ily th erapy. Fam ily th erapy is based on
RISE USMLE NEPAL

th e idea th at p sych op ath ology in on e am ily m em ber (e.g., a ch ild) re lects dys u n ction o
th e en tire am ily system .
4. The answer is B. Th is typ e o th erapy, in wh ich patien ts with a particular illn ess (e.g., arthritis)
m eet or com m un ication an d practical help, is best described as leaderless group therapy.
5. The answer is D. The techn ique described here (i.e., bio eedback) is based prim arily on operant
condition ing (see Chapter 7 or a discussion o classical conditioning, stim ulus gen eralization ,
and operan t condition ing). Reciprocal in hibition is the m echan ism that prevents one rom
eelin g two opposin g em otion s at the sam e tim e (e.g., relaxation and ear) an d is associated
with system atic desen sitization . In aversive con ditionin g, classical conditioning is used to pair
a m aladaptive but pleasurable stim ulus with an aversive or pain ul stim ulus so that the two
becom e associated an d the person stops en gaging in the m aladaptive behavior.
6. The answer is E. Th e m an agem en t tech n iqu e d escrib ed h ere is lood in g, a treatm en t
tech n iqu e or p h obias. In loodin g, a p erson is exp osed to an overwh elm in g dose o th e
eared stim u lu s or situ ation —in th is case, h eigh ts—u n til h e or sh e is n o lon ger a raid.
7. The answer is A. The m an agem en t techn ique described here is im plosion, a m anagem ent
techn ique related to looding (see also answer to Question 6) in which the person is instructed to
im agine exten sive exposure to a eared stim ulus (driving a car) until he or she is no longer a raid.
193
194 BRS Behavioral Science

8. The answer is G. Th e m an agem en t tech n iqu e d escrib ed h ere is cogn itive/ beh avioral
th erapy, a sh ort-term beh avioral m an agem en t tech n iqu e in wh ich th e p erson is in stru cted
to rep lace each n egative th ou gh t with a p ositive m en tal im age.
9. The answer is C. Th e m an agem en t tech n iqu e described h ere is aversive con dition in g, in
wh ich a m aladap tive b u t p leasu rab le stim u lu s ( or th is m an , sexu al in terest in ch ildren ) is
p aired with p ain u l stim u lu s (e.g., a sh ock) so th at th e two becom e associated. Th e p erson
n ow associates sexu al in terest in ch ild ren with p ain an d stop s th is m aladap tive beh avior.
10. The answer is F. Th e m an agem en t tech n iqu e d escribed h ere is system atic d esen sitization .
In th is exam p le, th e ch ild m ad e an erron eou s n egative association between dogs an d p ain
wh en sh e was in ju red in th e p resen ce o th e dog. In system atic desen sitization , in creasin g
HELP OTHERS SO THAT GOD WILL HELP YOU.

doses o th e righ ten in g stim u lu s (e.g., d ogs) are p aired with a relaxin g stim u lu s (e.g.,
th e avorite CD) to p rovoke a relaxation resp on se in situ ation s in volvin g th e righ ten in g
stim u lu s. Later in treatm en t, th is ch ild will rem ain in a relaxed state wh en sh e is exp osed to
a livin g d og.
11. The answer is C. Th e m ajor reason th at p atien ts wh o cou ld ben e it rom p sych oan alytically
orien ted p sych oth erapy do n ot receive it is th at th ey o ten believe it is exp en sive an d
tim e con su m in g. Less com m on ly, p eop le d o n ot wan t to reveal th eir h istories an d
p erson al p roblem s to stran gers, are n ot in terested in exp lorin g th eir ch ildh oods, or eel
u n com ortable in th e th erap eu tic settin g.
RISE USMLE NEPAL
Th e Fam ily, Cu ltu re,
c ha pte r
18 an d Illn ess
HELP OTHERS SO THAT GOD WILL HELP YOU.

Typical Board Question


Th e d au gh ter of a 65-year-old Vietn am ese wom an brin gs h er m oth er in for m an agem en t
of a seriou s m edical con d ition . Th e old er wom an , wh o lives with h er dau gh ter, is alert an d
orien ted. Th e m an agem en t regim en is qu ite com p lex, an d th e older wom an does n ot sp eak
En glish . To best con vey th e n eed ed in form ation to th is p atien t, th e p h ysician sh ou ld
(A) write th e in stru ction s down in En glish to be tran slated for th e p atien t later
(B) exp lain th e in stru ction s to th e dau gh ter an d h ave h er m on itor th e p atien t’s
treatm en t
(C) call in a p rofession al tran slator to exp lain th e in stru ction s to th e p atien t
(D) ask th e d au gh ter to tran slate th e in stru ction s to th e p atien t
(E) refer th e p atien t to a doctor wh o sp eaks Vietn am ese
(See “An sw ers an d Explan ation s” at th e en d of th e ch apter.)

I. OVERVIEW OF THE FAMILY


A. Definition
1. A grou p of p eop le related by blood, adoption, marriage or p erson al ch oice is a fam ily.
2. Th e in terp erson al relation sh ip s in fam ilies p lay a sign ifican t role in th e h ealth of fam ily
RISE USMLE NEPAL

m em b ers.

B. Types of families
1. Th e traditional nuclear family in clu des a m oth er, a fath er, an d dep en den t ch ildren (i.e.,
u n der age 18) livin g togeth er in on e h ou seh old.
2. Oth er typ es of fam ilies in clu de coh abitin g h eterosexu al fam ilies an d gay-p aren t fam ilies
an d sin gle-p aren t fam ilies.
3. Th e extended family in clu d es fam ily m em bers, su ch as gran d p aren ts, au n ts, u n cles, an d
cou sin s, wh o live ou tsid e th e h ou seh old.

II. DEMOGRAPHICS AND CURRENT TRENDS


A. Marriage and children
1. In th e Un ited States, th e average age of first marriage is abou t 27 years for women
an d 29 years for men an d h as been in creasin g steadily sin ce th e 1970s.
2. Abou t 50% of adu lts are m arried.

195
196 BRS Behavioral Science

3. A good m arriage is an im p ortan t p red ictor of h ealth . Married p eop le are mentally an d
physically healthier an d h ave h igh er self-esteem th an u n m arried p eop le.
4. Ap p roxim ately 60% of children live in fam ilies with two working parents ; on ly abou t 20% of
children live in th e “traditional family,” in wh ich th e fath er works ou tsid e of th e h om e an d
th e m oth er is a fu ll-tim e h om em aker.
5. Raisin g ch ild ren is exp en sive. Th e total cost of raisin g a ch ild to age 17 in th e Un ited
States is cu rren tly m ore th an $250,000. Postsecon d ary ed u cation greatly in creases th is
figu re.

B. Divorce and single-parent families


1. About 40% of marriages in the United States end in divorce. Divorce rates overall are decreas-
HELP OTHERS SO THAT GOD WILL HELP YOU.

in g, bu t th ey are decreasin g m ore am on g college-educated th an am on g h igh -sch ool–


ed u cated p eop le.
a. Factors associated with divorce in clu d e young age at m arriage, sh ort cou rtsh ip, lack of
fam ily su p p ort, p rem arital p regn an cy, d ivorce in th e fam ily, differen ces in religion or
socioecon om ic backgrou n d, an d seriou s illn ess or death of a ch ild .
b. Physicians have a higher divorce rate th an p eop le in oth er occu p ation s. Mu ch of th is
d ifferen ce m ay be a resu lt of th e lifestyle an d stresses associated with a career in
m edicin e.
2. Single-parent families
a. Sin gle-p aren t fam ilies often h ave lower incomes an d less social support an d, th erefore,
face in creased ch an ces of p h ysical an d m en tal illn ess.
b. Wh ile m an y u n m arried m oth ers belon g to low socioecon om ic grou p s, th e fastest
growin g p op u lation of sin gle m oth ers is educated professional women.
c. Most sin gle-p aren t fam ilies are headed by women.
3. Children in single-parent families
a. Th e p ercen tage of ch ild ren livin g in sin gle-p aren t fam ilies varies by eth n ic grou p
(Tab le 18.1).
b. Ch ild ren in sin gle-p aren t fam ilies are at increased risk for failu re in sch ool, d ep ression ,
d ru g ab u se, su icide, crim in al activity, an d divorce.
c. Even if th e noncustodial parent d oes n ot p rovide fin an cial su p p ort, ch ildren wh o con -
tin u e to h ave regular contact with th at p aren t h ave fewer of th ese p roblem s th an th ose
wh o h ave n o con tact.
4. Child custody
a. After d ivorce, th e typ es of ch ild cu stod y th at m ay b e gran ted by th e cou rts in clu d e
join t, sp lit, an d sole cu stod y; fathers are increasingly being granted join t or sole
cu stod y.
RISE USMLE NEPAL

b. In sole custody, th e ch ild lives with on e p aren t wh ile th e oth er h as visitation righ ts.
In th e p ast, sole cu stody was th e m ost com m on typ e of cu stody arran gem en t after
divorce.
c. In joint residential custody, wh ich h as becom e m ore p op u lar, th e ch ild sp en ds som e
tim e livin g with each p aren t.
d. In split custody, each p aren t h as cu stody of at least on e ch ild.

t a b l e 18.1 Percentage of White American African American, and Hispanic American


Children in Different Living Arrangements in the United States in 2014
Ethnic Group Both Parents (%) Mother Only (%) Father Only (%) Neither Parent (%)

White American 77.3 15.5 4.3 3.0


African American 41.3 48.8 4.2 5.9
Hispanic American (Latino) 64.9 27.5 3.2 4.4
All children 68.7 23.6 3.9 3.8
Chapter 18 The Family, Culture, and Illness 197

III. CULTURE IN THE UNITED STATES


A. Characteristics
1. Th ere are at least 316 million people in th e Un ited States. Th e p op u lation is m ade u p of
m an y minority subcultures as well as a large, white middle class , wh ich is th e m ajor cu ltu ral
in flu en ce.
2. Alth ou gh m an y su b cu ltu res h ave form ed th e Am erican cu ltu re, th e cu ltu re seem s to h ave
certain ch aracteristics of its own .
a. Financial and personal independence are valu ed at all ages an d esp ecially in th e elderly.
Twen ty-eigh t p ercen t of elderly Americans (18% of m en an d 34% of wom en ) sp en d
HELP OTHERS SO THAT GOD WILL HELP YOU.

th eir last years living alone . Ab ou t 68% of th e elderly live with sp ou ses or oth er fam ily
m em b ers; on ly ab ou t 5% of th ose 65+ an d 15% of th ose 85+ live in nursing homes .
b. Em p h asis is p laced on personal hygiene an d clean lin ess.
c. Th e nuclear family with few ch ild ren is valu ed.

B. Culture and illness


1. Wh ile eth n ic grou p s are n ot h om ogen eou s (i.e., th eir m em bers h ave differen t back-
grou n d s an d d ifferen t reason s for em igratin g), grou p s often h ave ch aracteristic ways of
dealin g with illn ess.
2. Alth ou gh th e m ajor p sych iatric disorders su ch as sch izop h ren ia an d dep ression are seen
to abou t th e sam e exten t in all cu ltu res, th e typ e of beh avior considered abnormal may
differ con siderably by cu ltu re.
3. While differences in presentation of sym ptom s m ay be the result of the individual characteris-
tics of a patien t, they m ay also be related to the characteristics of the particular ethn ic group.
4. A p atien t’s belief system h as m u ch to do with adh eren ce an d resp on se to m an agem en t.
Ph ysician s m u st h ave resp ect for an d work in th e con text of su ch beliefs in order to h elp
p atien ts. For exam p le:
a. In certain eth n ic grou p s, it is b elieved th at illn ess can be cu red by eatin g certain foods .
Th erefore, if n ot con train d icated m edically, th e doctor sh ou ld attem p t to m ake avail-
able th e food th e ill p atien t believes can h elp h im or h er.
b. Th e idea th at an outside influence (e.g., a h ex or a cu rse im p osed by th e an ger of an
acqu ain tan ce or relative) can cau se illn ess is seen in som e eth n ic grou p s. Th e d octor
sh ou ld n ot d ism iss th e p atien t’s b elief, b u t rath er sh ou ld ask th e p atien t wh o can h elp
to rem ove th e cu rse an d in volve th at p erson in th e m an agem en t p lan .
c. Peop le m ay seek h ealth care from folk or religious healers (e.g., ch am an es, cu ran de-
ros, an d espiritistas am on g Latin os). Man agem en t p rovid ed by th ese h ealers in clu d es
RISE USMLE NEPAL

herbal medicines an d sp ecific ch an ges in diet. Th e p h ysician sh ou ld n ot disp arage th e


u se of folk m edicin e bu t rath er in clu de it, if p ossible, in th e m an agem en t p lan .
d. A b elief in communication with spirits of the dead is seen in som e cu ltu res (e.g., Latin o).
Becau se th ey are sh ared by m em b ers of a cu ltu ral grou p, su ch b eliefs are n ot delu -
sion al (see Ch ap ter 11).

C. Acculturative stress
1. Th e con cep t of accu ltu rative stress is u sed as an altern ative to th e term “cu ltu re sh ock.” It
is an emotional response , in volvin g psychiatric symptoms as well as in creased su scep tibility
to disease, wh ich is related to geograp h ic relocation an d th e n eed to adap t to u n fam iliar
social an d cu ltu ral su rrou n d in gs. Accu ltu rative stress is redu ced wh en grou p s of im m i-
gran ts of a p articu lar cu ltu re live in th e sam e geograp h ic area.
2. Young immigrant men ap p ear to b e at higher risk for accu ltu rative stress, in clu din g sym p -
tom s su ch as p aran oia an d d ep ression , th an oth er sex an d age grou p s. Th is is tru e in p art
b ecau se:
a. You n g m en lose the most status on leavin g th eir cu ltu re of origin .
b. Un like oth ers in th e grou p wh o can stay at h om e am on g fam iliar p eop le, you n g m en
often m u st get ou t in to th e n ew cu ltu re an d earn a livin g.
198 BRS Behavioral Science

D. Ethnic disparities in health care


1. Racial an d eth n ic m in orities in th e Un ited States often face obstacles in obtain in g qu ality
m en tal an d p h ysical h ealth care.
2. Eth n ic d isp arities in h ealth care resu lt in p art from :
a. Economic factor, for exam p le, th e average in com e of African Am erican fam ilies, is on ly
abou t 60% th at of Wh ite Am erican fam ilies.
b. Decreased physical access to h ealth care.
c. Doctor–p atien t communication d ifficu lties.
d. Overt b ias an d n egative racial stereotypes h eld by som e p h ysician s.
e. Relative scarcity of m in ority p h ysician s.
HELP OTHERS SO THAT GOD WILL HELP YOU.

IV. AMERICAN SUBCULTURES


A. African Americans
1. African Am erican s m ake u p abou t 13.2% of th e total p op u lation in th e Un ited States.
2. Com p ared to Wh ite Am erican s, African Americans h ave:
a. Shorter life expectancies (see Figu re 3.1).
b. Higher rates of h yp erten sion , h eart disease, stroke, in fan t m ortality (see also Table 1.1),
ob esity, asth m a, tu bercu losis, d iabetes, p rostate can cer, an d AIDS.
c. Higher death rates from h eart disease an d from m ost form s of can cer.
3. Religion an d strong extended family networks p lay a m ajor role in social an d p erson al su p -
p ort am on g m an y African Am erican s. Th is m ay in p art exp lain wh y th e overall suicide rate
is lower am on g African Am erican s th an am on g Wh ite Am erican s.
4. Wh ile th e overall su icide rate is lower, su icide in African Am erican teen agers, on ce u n com -
m on , h as in creased in th e last 20 years. It is n ow th e th ird leadin g cau se of death in th is
grou p, with h om icide th e leadin g cau se an d acciden ts secon d. In Wh ite Am erican teen ag-
ers, accid en ts are th e lead in g cau se of d eath , with su icide secon d, an d h om icid e th ird .

B. Hispanic/Latino Americans
1. Overview
a. With abou t 17.1% of th e p op u lation , Hisp an ic Am erican s (m ain ly in clu din g p eop le
from Sp an ish -sp eakin g region s of Latin Am erica, i.e., Latin os) are n ow th e largest
minority group in th e Un ited States.
b. As a grou p, Latin os p lace great valu e on th e n u clear fam ily an d on nuclear families with
many children.
RISE USMLE NEPAL

c. Respect for the elderly is im p ortan t. You n ger p eop le are exp ected to care for elderly
fam ily m em bers, to protect elderly relatives from negative medical diagnoses , an d, often ,
to m ake m ed ical d ecision s con cern in g th e care of elderly relatives.
d. Am on g som e Latin os, “hot” and “cold” influences are believed to relate to illn ess.
e. Latin o wom en are less likely to get m am m ogram s an d m ore likely to h ave cervical can -
cer th an are Wh ite Am erican or African Am erican wom en .
2. Two-th irds of all Latin os, esp ecially th ose in th e Sou th west, are of Mexican origin .
3. Th e secon d largest grou p (9.4%) of Latin os is of Puerto Rican origin . Most live in th e
North eastern states.
4. More th an 3.7% of Latin os are of Cuban origin an d live p rim arily in th e Sou th east, esp e-
cially in Florida.
5. Alth ou gh th e exp lan ation is elu sive, as a grou p, Latin os h ave lon ger life exp ectan cies th an
African Am erican s or Wh ite Am erican s (see Figu re 3.1).

C. Asian Americans
1. Abou t 5.3% of th e US p op u lation is Asian Am erican . Th e largest grou p s are Chinese,
Filipino, an d Asian Indian.
2. Oth er Asian Am erican grou p s in clu de th e Vietnamese, Korean, an d J apanese .
Chapter 18 The Family, Culture, and Illness 199

3. Alth ou gh m an y grou p s are assimilated, eth n ic differen ces m ay still resu lt in d ifferen t
resp on ses to illn ess am on g Asian Am erican grou p s.
4. Ch aracteristics of th ese cu ltu res in clu de th e followin g:
a. As in Latin o cu ltu res, ad u lt Asian Am erican ch ild ren show strong respect for an d are
expected to care for their elderly parents , p rotect elderly relatives from n egative m edical
d iagn oses, an d m ake m ed ical d ecision s abou t elderly relatives’ care.
b. Patien ts m ay exp ress em otion al p ain as p h ysical illn ess.
c. In som e Asian Am erican grou p s, th e abdominal–thoracic area , rath er th an th e b rain , is
often th ou gh t to be th e spiritual core of th e p erson . Th u s, th e con cep t of brain death
an d resu ltin g organ tran sp lan t are gen erally n ot well accep ted.
d. Folk remedies in clu de coining (a coin is ru bbed on th e affected area p ressin g a m edi-
HELP OTHERS SO THAT GOD WILL HELP YOU.

cated oil in to th e skin ); in ju ries occu rrin g as a resu lt of u se of su ch rem ed ies m ay b e


m istaken by m edical p erson n el for abu se (see Ch ap ter 20).
e. Certain disorders, for exam p le, gastric cancer, are m ore com m on in Asian s livin g in
th eir n ative cou n tries th an in Asian s livin g in th e Un ited States. For exam p le, rates of
gastric can cer are fou r tim es h igh er in Jap an ese p eop le livin g in Jap an th an in Jap an ese
Am erican s livin g in Los An geles. Cu ltu re-related d ietary factors (e.g., m ore n itrate-rich
foods in Jap an ) m ay h elp exp lain th is differen ce.

D. Native Americans: American Indians and Eskimos


1. Makin g u p between 1% and 2% of th e p op u lation , Native Am erican s (in clu din g th ose with
m ixed h eritage) are covered by a p rogram of m edical care u n der th e direction of th e Indian
Health Service of th e fed eral govern m en t.
2. Th e d istin ction between m en tal an d p h ysical illn ess m ay be blu rred; en gagin g in forbid-
den beh avior an d witch craft are th ou gh t to resu lt in illn ess.
3. In gen eral, Native Am erican s h ave low in com es an d high rates of alcoholism an d suicide,
p articu larly am on g teen agers.

E. Americans of Middle Eastern/North African descent


1. Peop le of Middle Eastern or North African origin (abou t 0.4% of th e p op u lation ), wh o
sp eak dialects of th e Arabic language (e.g., Leban on , Egyp t, Syria, Palestin e, Morocco, Iraq,
Jord an , an d Yem en ), are often referred to as Arab s. Oth er Mid d le Eastern grou p s in clu d e
p eop le from Sau d i Arabia, Ku wait, Bah rain , Om an , Qatar, Iran , Afgh an istan , an d Pakistan .
2. Som e Middle Eastern p eop le are Ch ristian (e.g., Cop tic Ch ristian ) or Jewish ; m ost follow
th e Muslim religion .
3. Peop le wh o follow th e Mu slim religion value female modesty. Fem ale p atien ts m ay wish
to rem ain as covered as p ossib le in th e exam in in g room (e.g., h ead an d face covered by a
RISE USMLE NEPAL

scarf). Th ey often p refer to h ave a fem ale p h ysician or, if exam in ed by a m ale p h ysician ,
m ay wish to h ave th eir h u sban d or m oth er p resen t. Ph ysician s sh ou ld m ake every effort
to h on or su ch wish es.

F. Non-Hispanic White Americans (62.6% of the US population)


1. Anglo-Americans are th ose origin atin g in En glish -sp eakin g Eu rop ean cou n tries, m ostly
from Irelan d.
2. An glo-Am erican s in gen eral are less em otion al, m ore stoic, an d less vocal abou t p ain an d
illn ess th an m em bers of grou p s of Mediterranean origin (e.g., Jewish , Greek, an d Italian
p eop le).
3. Th erefore, An glo-Am erican s m ay b ecom e very ill before seekin g treatm en t, wh ile p eop le
of Med iterran ean origin m ay b e con sid ered com p lain ers an d ign ored wh en th ey are, in
fact, qu ite ill.
Review Test

Directions: Each of th e n u m b ered item s or in com p lete statem en ts in th is section is followed by


an swers or by com p letion s of th e statem en t. Select th e one lettered an swer or com p letion th at
is best in each case.
HELP OTHERS SO THAT GOD WILL HELP YOU.

1. A 24-year-old m arried Mu slim wom an , 3. Sim ilarly, th e p rin cip al’s best gu ess abou t
who is exp erien cin g severe p elvic p ain , th e p ercen tage of African Am erican stu den ts
is brou gh t to th e em ergen cy room by h er who live with ju st th eir m oth ers is
h u sban d. Wh en in stru cted to disrobe an d (A) <10%
p u t on a h osp ital gown , sh e refu ses u n less (B) 15%–25%
sh e can be assu red th at sh e will be seen by (C) 45%–50%
a fem ale p h ysician . Th e m ost ap p rop riate (D) 50%–60%
statem en t th e m ale em ergen cy room (E) 75%–85%
p h ysician can m ake at th is tim e is
(A) “I will try to locate a fem ale p h ysician bu t 4. Th e p rin cip al’s best gu ess abou t th e
if I can n ot do so, I m u st exam in e you .” p ercen tage of stu den ts wh o are Native
(B) “I am a b oard -certified p h ysician an d Am erican is
am as qu alified as a fem ale doctor to (A) <10%
exam in e an d treat you .” (B) 15%–25%
(C) “I will try to locate a fem ale p h ysician ; (C) 45%–50%
if I can n ot do so, h ow can I h elp you (D) 55%–65%
b e m ore com fortable with m e as you r (E) 75%–85%
d octor?”
(D) “I can n ot h elp you if you will n ot 5. A large exten d ed fam ily im m igrates to th e
coop erate.” Un ited States. Th e p erson in th e fam ily wh o
(E) “Severe p elvic p ain is som etim es life is at h igh est risk for p sych iatric sym p tom s
th reaten in g. I m u st exam in e you after th e m ove is th e
im m ediately.”
(A) 84-year-old great-gran dfath er
(B) 28-year-old u n cle
Questions 2–4
(C) 36-year-old au n t
(D) 10-year-old sister
RISE USMLE NEPAL

An elem en tary sch ool in Texas in clu d es ch il-


(E) 55-year-old gran dm oth er
dren from m an y cu ltu res. In fact, th e p rin -
cip al h as discovered th at th e p op u lation of
6. A 12-year-old ch ild is told to write a
th e sch ool directly m irrors th at of th e US
rep ort abou t h is “n u clear fam ily.” To do
p op u lation .
th is task correctly, th e rep ort m u st con tain
in form ation on h is
2. If th e sch ool’s p rin cip al is tryin g to
estim ate h ow m an y of th e sch ool’s stu den ts (A) 84-year-old great-gran dfath er
live in a “trad ition al” fam ily situ ation , h er (B) 28-year-old u n cle
best gu ess is (C) 36-year-old au n t
(D) 10-year-old sister
(A) <10%
(E) 55-year-old gran dm oth er
(B) 15%–25%
(C) 45%–50%
(D) 55%–65%
(E) 75%–85%

200
Chapter 18 The Family, Culture, and Illness 201

Questions 7 and 8 11. A 26-year-old wom an and a 29-year-old


m an get m arried after a 2-year engagem ent.
In th e Un ited States, in d ep en d en ce is valu ed They are both Episcopalian an d are both from
at all ages. However, m an y elderly p eop le m iddle-class fam ilies. Both sets of their parents
requ ire care by oth ers wh en th ey becom e are divorced. Which of the following factors
in cap acitated. Nu rsin g h om e care is on e puts this couple at highest risk for divorce?
op tion for su ch care. (A) Th eir ages
(B) Th e len gth of th eir en gagem en t
7. Wh at p ercen tage of elderly Am erican s (C) Th eir p aren ts’ m arital h istories
sp en d th eir last years livin g in a n u rsin g (D) Th eir socioecon om ic backgrou n ds
h om e? (E) Th eir religiou s backgrou n ds
HELP OTHERS SO THAT GOD WILL HELP YOU.

(A) <10%
(B) 15%–25% 12. A 40-year-old Japan ese wom an born
(C) 45%–50% an d livin g in Tokyo has been experien cin g
(D) 55%–65% gastric bleedin g an d, after an en doscopy, is
(E) 75%–85% diagn osed with gastric can cer. Up on h earin g
th is n ews, th e wom an’s iden tical twin sister,
8. Wh ich of th ese p atien ts is m ost likely to who has been livin g in the Un ited States for
sp en d th e last years of h er life in a n u rsin g 20 years an d works out at the gym th ree tim es
h om e? p er week, has an en doscopy. Th e results of
(A) An 80-year-old An glo-Am erican wom an th e sister’s test are un rem arkable, an d n o
(B) An 80-year-old Pu erto Rican Am erican eviden ce of gastric can cer is fou n d. The
wom an m ost likely exp lan ation for this differen ce in
(C) An 80-year-old Jap an ese Am erican disease between th e twin s is th at
wom an (A) gastric can cer is u n related to gen etic
(D) An 80-year-old Mexican Am erican factors
wom an (B) en viron m en tal factors are likely to p lay a
(E) An 80-year-old Vietn am ese Am erican role in th e develop m en t of gastric can cer
wom an (C) testin g tech n iqu es for gastric can cer are
better in Jap an th an in th e Un ited States
9. A p h ysician h as two 56-year-old m ale (D) exercise can p reven t gastric can cer
p atien ts. On e of th em is African Am erican (E) a h igh n itrate d iet is p robab ly
an d on e is Wh ite Am erican . Statistically, resp on sib le for gastric can cer
th e African Am erican p atien t h as a lower
likelih ood of 13. A 40-year-old Mexican Am erican m an
(A) stroke wh o h as been d iagn osed with h yp erten sion
(B) asth m a tells th e p h ysician th at a h ealer, u sed by
RISE USMLE NEPAL

(C) h yp erten sion m an y m em bers of h is com m u n ity, told h im


(D) su icid e th at eatin g corn every day will lower h is
(E) p rostate can cer blood p ressu re. He exp lain s th at th e h ealer
told h im h yp erten sion is a “h ot” illn ess an d
10. Wh ich of th e followin g livin g situ ation s corn is a “cold” food. If eatin g corn p oses n o
is likely to b e least com m on in th e Un ited dan ger to th is p atien t, wh at is th e doctor’s
States? m ost ap p rop riate n ext statem en t?
(A) A 34-year-old m ed ical resid en t livin g (A) “Th ere is n o m edical eviden ce th at corn
with h is p aren ts is h elp fu l for lowerin g b lood p ressu re.”
(B) A 46-year-old divorced wom an livin g (B) “I can n ot treat you u n til you stop goin g
with h er 10-year-old son to th e h ealer.”
(C) A 46-year-old sin gle m an livin g alon e (C) “Is th e h ealer train ed in m od ern
(D) A 46-year-old m an livin g with h is wife m edicin e?”
an d ch ildren (D) “Th ere are m edical treatm en ts for h igh
(E) An 85-year-old wom an livin g with blood p ressu re th at you can u se alon g
relatives with eatin g corn .”
(E) “Try th e corn for a m on th an d if you r
blood p ressu re is still h igh , I will give you
m edication to lower it.”
202 BRS Behavioral Science

14. A 70-yea r-o ld La tin o wo m a n , wh o se (A) “Do you believe th at you r h u sban d is still
h u sb a n d d ied 4 m o n th s a go, ca lm ly tells alive?”
h er p h ysicia n th a t sh e a n d h er h u sb a n d (B) “Do oth er p eop le in th e Latin o
still co m m u n ica te with ea ch o th er. Th e com m u n ity b elieve th at th e livin g
p atien t sh ows n o evid en ce o f a th o u gh t an d th e dead com m u n icate with each
d isord er, a n d h er p h ysica l exa m in a tio n oth er?”
is u n rem a rka b le. Wh ich of th e followin g (C) “I wou ld like you to take a m edication
is th e m o st a p p ro p ria te q u estio n o r called risp erid on e for th e n ext few
sta tem e n t fro m th e p h ysicia n a t th is m on th s.”
tim e? (D) “Most p eop le d o n ot th in k th at th ey can
com m u n icate with th e dead .”
HELP OTHERS SO THAT GOD WILL HELP YOU.

(E) “How do you feel wh en you r h u sban d


com m u n icates with you ?”
RISE USMLE NEPAL
An swers an d Exp lan ation s

Typical Board Question


The answer is C. Wh en th e m a n agem en t regim en is co m p lex a n d th e p a tien t d o es n o t sp ea k
En glish , th e p h ysicia n’s b est ch o ice is to ca ll in a p ro fessio n al tra n slator so th a t h e ca n exp la in
th e in stru ctio n s d irectly to h is p a tien t (in th is ca se, th e eld erly wo m a n ). Com m u n ica tin g
HELP OTHERS SO THAT GOD WILL HELP YOU.

a s d irectly a s p o ssib le with th e p a tien t is p a rticu larly im p orta n t in cu ltu res in wh ich ad u lt
ch ild ren m a y p rotect an eld erly relative from a n egative m ed ica l d ia gn osis (e.g., Asia n a n d
Hisp an ic cu ltu res). Th u s, in tran sla tin g th e in form ation , or m on itorin g th e treatm en t, th e
d a u gh ter m ay n o t rela y th e co m p lete p ictu re to th e eld erly p atien t. Writin g th e in stru ction s
d own in En glish to b e tra n sla ted la ter is n ot a p p rop ria te b ecau se it is u n certa in h ow a n d
wh en th e tra n slation will b e d on e. Sin ce th e d octor ca n ca ll in a tran sla tor, th ere is n o rea son
to refer th e p a tien t to a n oth er d o cto r. In a n y ca se, referra ls sh o u ld b e m a d e o n ly fo r m ed ical
rea son s.

1. The answer is C. Mu slim wom en often p refer to h ave a fem ale p h ysician , p articu larly for
gyn ecological or ob stetrical p roblem s. In th is case, th e p h ysician sh ou ld try to h on or th e
p atien t’s wish es. If th is is n ot p ossible, th e p atien t sh ou ld be con su lted for altern ative
accep table strategies, for exam p le, sh e m ay su ggest h avin g h er h u sb an d or oth er fam ily
m em ber (e.g., h er m oth er) p resen t wh en sh e is exam in ed by th e m ale p h ysician . Tryin g to
im p ress th e p atien t with on e’s creden tials, or frigh ten h er in to adh eren ce are n ot ap p rop ri-
ate or u sefu l strategies (see also Ch ap ter 21).
2. The answer is B. 3. The answer is C. 4. The answer is A. Ap p roxim ately 20% of Am erican
ch ild ren live in a “trad ition al” fam ily situ ation (th e m oth er stays h om e an d th e fath er
works). Ap p roxim ately 49% of African Am erican ch ildren live with ju st th eir m other. Native
Am erican s m ake u p b etween 1% an d 2% of all Am erican s.
5. The answer is B. You n g im m igran t m en , su ch as th e 28-year-old u n cle, are at h igh er risk
for p sych iatric sym p tom s wh en en terin g a n ew cu ltu re th an are an y oth er gen der or age
grou p. Th is is b ecau se th ey lose th e m ost statu s on leavin g th eir old cu ltu re an d becau se,
u n like oth er grou p s th at can stay at h om e am on g th eir fam ilies, you n g m en often m u st get
ou t in to th e n ew cu ltu re to work an d m ake a livin g.
RISE USMLE NEPAL

6. The answer is D. Th e “n u clear fam ily” con sists of p aren ts an d dep en den t ch ildren (e.g., th e
b oy’s sister) livin g in on e h ou seh old . Th e great-gran dfath er, u n cle, au n t, an d gran dm oth er
u su ally are p art of th e “exten ded fam ily.”
7. The answer is A. 8. The answer is A. Ab ou t 5% of elderly Am erican s sp en d th eir last years
livin g in a n u rsin g h om e. Elderly Asian Am erican an d Hisp an ic Am erican p eop le are m ore
likely th an An glo-Am erican s to be cared for by th eir adu lt ch ildren rath er th an in a n u rsin g
h om e settin g.
9. The answer is D. Statistically, a m id d le-aged African Am erican p atien t h as a lower likeli-
h ood of su icid e th an a Wh ite Am erican p atien t of th e sam e age. However, wh en com p ared
to Wh ite Am erican p atien ts, African Am erican p atien ts h ave a h igh er likelih ood of stroke,
asth m a, h yp erten sion , an d p rostate can cer as well as h eart d isease, tu b ercu losis, d iab etes,
an d AIDS.
10. The answer is A. In th e Un ited States, it is relatively u n com m on to see a self-su p p ortin g
ad u lt, su ch as th e 34-year-old m ed ical residen t, livin g with h is p aren ts. A 46-year-old m an
livin g with h is wife an d ch ild ren is a com m on livin g situ ation in th e Un ited States; th e

203
204 BRS Behavioral Science

d ivorce rate is h igh , b u t m ost p eop le in th eir 40s are m arried, n ot sin gle or divorced. It is
also relatively com m on to see an 85-year-old wom an livin g with fam ily m em bers.
11. The answer is C. Of th e listed factors, th eir p aren ts’ h istories of divorce are a risk factor for
divorce for th is cou p le. Teen age m arriages, sh ort cou rtsh ip, an d d ifferen ces in socioeco-
n om ic an d religiou s b ackgrou n ds also p u t cou p les at risk for divorce.
12. The answer is B. Th e m ost likely exp lan ation for th is d ifferen ce b etween th e twin s (i.e.,
th e on e in Jap an h as an d th e on e in th e Un ited States h as n ot been diagn osed with gastric
can cer) is th at en viron m en tal factors are likely to p lay a role in th e develop m en t of gastric
can cer. If on ly gen etic factors were in volved , b oth wom en wou ld be likely to h ave th e dis-
ease. A diet h igh in n itrates su ch as th at eaten in Jap an is a risk factor for gastric can cer, b u t
HELP OTHERS SO THAT GOD WILL HELP YOU.

it is n ot clear th at th is is th e on ly en viron m en tal factor to wh ich th e two wom en are differ-


en tially exp osed. Th ere is n o reason to b elieve th at testin g tech n iqu es for gastric can cer are
differen t in Jap an an d in th e Un ited States or th at exercise can p reven t gastric can cer.
13. The answer is D. As lon g as th e treatm en t will n ot h arm th e p atien t, th e p h ysician sh ou ld
try to work in con ju n ction with th e h ealer. Sin ce in th is case th e folk rem edy is in n ocu ou s,
th e p atien t can con tin u e u sin g it alon g with tradition al m edical m an agem en t (e.g., an an ti-
h yp erten sive agen t). Th e p h ysician sh ou ld n ot try to sep arate th e p atien t from h is cu ltu ral
beliefs by refu sin g to treat h im u n til h e stop s u sin g th e folk h ealer, qu estion in g th e h ealer’s
train in g in m odern m edicin e, or dou btin g th e valu e of th e recom m en ded rem edy. It cou ld
be d an gerou s to delay th e p atien t’s treatm en t for a m on th to p rove to h im th at eatin g corn
will n ot h elp h is con dition .
14. The answer is B. Th is Latin o p atien t wh o rep orts th at sh e com m u n icates with h er d ead
h u sban d is p robably n ot exp erien cin g a delu sion (i.e., a false belief n ot sh ared by oth ers
[see Table 11.1]). Rath er, sh e is m ost likely to be rep ortin g a cu ltu ral p h en om en on based on
th e belief, in som e Latin o cu ltu res, th at th e lin e between th e dead an d th e livin g is blu rred.
As fu rth er eviden ce th at th is is n ot a d elu sion th e p atien t sh ows n o eviden ce of a th ou gh t
disord er. Th u s, sh e d oes n ot n eed to take an an tip sych otic su ch as risp eridon e. Th ere is
n o evid en ce th at sh e eith er b elieves h er h u sb an d is alive or th at sh e is distu rbed by th ese
exp erien ces.
RISE USMLE NEPAL
c ha pte r
19 Sexu ality
HELP OTHERS SO THAT GOD WILL HELP YOU.

Typical Board Question


A 17-year-old m ale is b rou gh t to th e d octor by h is p aren ts wh o exp ress con cern abou t h is
sexu al in terests an d b eh avior. Wh en in terviewed alon e, th e p atien t n otes th at h e h ad sex with
an oth er you n g m an on on e occasion bu t states th at h e is also attracted to wom en . He tells
th e doctor th at on ce, at a p arty, h e wore wom en’s cloth in g an d elt sexu ally arou sed. Th e m ost
ap p rop riate descrip tion / d iagn osis or th is you n g m an is
(A) oth er sp eci ied p arap h ilic disord er
(B) gen der dysp h oria
(C) typ ical beh avior
(D) h om osexu ality
(E) tran svestic disord er
(See “An sw ers an d Explan ation s” at th e en d of th e ch apter.)

I. SEXUAL DEVELOPMENT
A. Prenatal physical sexual development
1. Di eren tiation o th e gonads is d ep en den t on th e p resen ce or ab sen ce o th e Y chromo-
some , wh ich con tain s th e testis-determ in in g actor gen e.
RISE USMLE NEPAL

2. Th e an drogen ic secretion s o th e etal testes d irect th e di eren tiation o male in tern al an d


extern al gen italia.
a. In the absence of androgens du rin g p ren atal li e, in tern al an d extern al genitalia are
female .
b. In androgen insensitivity syndrome ( orm erly testicu lar em in ization ), desp ite an XY
genotype an d testes th at secrete an d rogen , a gen etic de ect p reven ts th e body cells rom
resp on din g to an d rogen , resu ltin g in a em ale p h en otyp e. At p u b erty, th e d escen d in g
testes m ay ap p ear as lab ial or in gu in al m asses.
c. In th e p resen ce o excessive adren al an drogen secretion p ren atally (congenital virilizing
adrenal hyperplasia), th e gen italia o a gen etic em ale are m ascu lin ized an d th e ch ild
m ay b e visu ally id en ti ied in itially as m ale.

B. Prenatal psychological sexual development


1. Di eren tial exp osu re to gon ad al h orm on es du rin g p ren atal li e also resu lts in gender dif-
ferences in certain brain areas (e.g., th e h yp oth alam u s, an terior com m issu re, corp u s cal-
losu m , an d th alam u s).

205
206 BRS Behavioral Science

t a b l e 19.1 Gender Identity, Gender Role, and Sexual Orientation

Term Definition Presumed Etiology Comments

Gender identity Sense of self as being male or Differential exposure to May not agree with physiological
female prenatal sex hormones sex (i.e., gender dysphoria)
Gender role Expression of one’s gender Societal pressure to May not agree with gender identity
identity in society conform to sexual norms or physiological sex (e.g., choice
of opposite gender’s clothing)
Sexual Persistent and unchanging Differential exposure to Homosexuality and bisexuality are
orientation preference for people of the prenatal sex hormones considered normal variants of
same sex (homosexual), the Genetic influences sexual expression
opposite sex (heterosexual),
HELP OTHERS SO THAT GOD WILL HELP YOU.

or no preference (bisexual)
for love and sexual
expression

2. Gender identity, gender role , an d sexual orientation (Table 19.1) also m ay be a ected by
p ren atal exp osu re to gon adal h orm on es.
a. In d ividu als with gender dysphoria (tran ssexu al or tran sgen der in dividu als) h ave a p er-
vasive p sych ological eelin g o bein g born in to th e body o th e wron g sex desp ite a
b od y orm typ ical o th eir p h ysiological sex.
b. Sch ool-age ch ildren with gen der d ysp h oria p re er to dress like an d h ave p laym ates o
th e op p osite sex. Sin ce gender identity is permanent, th e m ost e ective m an agem en t o
th is situ ation is to h elp p aren ts accep t th e ch ild as h e or sh e is.
c. In adu lth ood, th ese in dividu als com m on ly take th e horm on es o their p re erred gen der
an d h ave p lastic su rgery p rocedu res in order to better assu m e th at gen der role; som e
m ay also seek su rgery to ch an ge th eir gen ital sex.

II. THE BIOLOGY OF SEXUALITY IN ADULTS


In adu lts, alteration s in circu latin g levels o gon adal h orm on es (estrogen , p rogesteron e, an d
testosteron e) can a ect sexu al in terest an d exp ression .

A. Hormones and behavior in women


1. Becau se estrogen is on ly m in im ally in volved in libido, menopause (i.e., cessation of ovarian
RISE USMLE NEPAL

estrogen production) an d aging do not reduce sex drive i a wom an’s gen eral h ealth is good
(see Ch ap ter 2).
2. Testosterone is secreted by th e adren al glan ds (as well as th e ovaries an d testes) th rou gh -
ou t adu lt li e an d is believed to p lay an im p ortan t role in sex drive in both men and women.

B. Hormones and behavior in men


1. Testosteron e levels in m en gen erally are higher than necessary to m ain tain n orm al sexu al
u n ction in g; low testosteron e levels are less likely th an relation sh ip p roblem s, age, alco-
h ol u se, or u n id en ti ied illn ess to cau se sexu al d ys u n ction .
2. Psych ological an d p h ysical stress may decrease testosterone levels.
3. Medical treatm en t with estrogens, progesterone , or antiandrogens (e.g., to treat p rostate
can cer) can decrease testosteron e availability via h yp oth alam ic eedb ack m ech an ism s,
resu ltin g in decreased sexual interest and behavior.

C. Homosexuality (i.e., gay or lesbian sexual orientation; see Table 19.1)


1. Etiology
a. Th e etiology o h om osexu ality is b elieved to be related to alterations in levels of pre-
natal sex hormones (e.g., in creased an d rogen s in em ales an d decreased an drogen s
in m ales) resu ltin g in an atom ic ch an ges in som e h yp oth alam ic n u clei; sex h orm on e
Chapter 19 Sexuality 207

t a b l e 19.2 Characteristics of the Stages of Sexual Response Cycles in Men and Women

Men Women Both Men and Women

Excitement Stage
Penile erection Clitoral erection Increased pulse, blood pressure, and
Labial swelling respiration
Vaginal lubrication Nipple erection
Tenting effect (rising of the uterus in
the pelvic cavity)
Plateau Stage
Increased size and upward Contraction of the outer third of the Further increase in pulse, blood pressure,
HELP OTHERS SO THAT GOD WILL HELP YOU.

movement of the testes vagina, forming the orgasmic and respiration


Secretion of a few drops of platform (enlargement of the upper Flushing of the chest and face (the “sex
sperm-containing fluid third of the vagina) flush”)
Orgasm Stage
Forcible expulsion of seminal Contractions of the uterus and vagina Contractions of the anal sphincter
fluid Further increase in pulse, blood pressure,
and respiration
Resolution Stage
Refractory, or resting, period Little or no refractory period Muscle relaxation
(length varies by age and Return of the sexual, muscular, and
physical condition) when cardiovascular systems to the
restimulation is not possible prestimulated state over 10–15 min

levels in adu lth ood are in distin gu ish able rom th ose o h eterosexu al p eop le o th e
sam e b iological sex.
b. Evid en ce or in volvem en t o genetic factors in clu des m arkers on th e X ch rom osom e
an d h igh er con cord an ce rate in m on ozygotic th an in dizygotic twin s.
c. Social factors , su ch as early sexu al exp erien ces, are not associated with th e etiology o
h om osexu ality.
d. Hom osexu ality is a n orm al varian t o sexu al exp ression . Becau se it is n ot a dys u n ction ,
no treatment is n eed ed . Peop le wh o are u n com ortab le with th eir sexu al orien tation
m ay b en e it rom p sych ological in terven tion to h elp th em becom e m ore com ortable.
2. Occurrence
a. By m ost estim ates, at least 5%–10% of the population h as an exclu sively h om osexu al
sexu al orien tation ; m an y m ore p eop le h ave h ad at least on e sexu al en cou n ter leadin g
to arou sal with a p erson o th e sam e sex.
b. Th ere are no significant ethnic differences in th e occu rren ce o h om osexu ality.
RISE USMLE NEPAL

c. Man y p eop le with gay an d lesb ian sexu al orien tation s h ave exp erien ced h eterosexu al
sex an d h ave h ad ch ild ren .

D. The sexual response cycle


1. Masters an d Joh n son devised a four-stage model or sexu al resp on se in both m en an d
wom en , in clu d in g th e excitement, plateau, orgasm, an d resolution stages (Tab le 19.2).
2. Sexu al d ys u n ction in volve a d i icu lty with on e or m ore asp ects o th e sexu al resp on se
cycle.

III. SEXUAL DYSFUNCTION


A. Characteristics
1. Sexu al d ys u n ction can resu lt rom b iological, p sych ological, or in terp erson al cau ses or
rom a com bin ation o cau ses.
a. Biological causes in clu d e an u n iden ti ied gen eral m ed ical con d ition (e.g., diabe-
tes can cau se erectile disorder; pelvic adhesions can cau se p elvic p ain ), side effects
208 BRS Behavioral Science

of medication (e.g., selective seroton in reu p take in h ibitors [SSRIs] can cau se delayed
orgasm ), substance use (e.g., alcoh ol u se can cau se erectile d isorder), an d hormonal or
neurotransmitter alterations .
b. Psychological causes in clude curren t relation ship problem s, stress, depression , and an xi-
ety (e.g., guilt, per orm an ce pressure). In m en with erectile disorder, the presen ce o m orn -
in g erection s, erection s durin g m asturbation, or erection s durin g rapid eye m ovem en t
(REM) sleep suggests a psychological rather than a physical cause o erectile disorder.
2. Dys u n ction s m ay always h ave b een p resen t (lifelong), or, m ore com m on ly, th ey occu r
a ter an in terval wh en u n ction h as been typ ical (acquired).

B. Diagnostic and Statistical Manual of Mental Disorders, Fifth Edition (DSM-5) classifications o
sexu al d ys u n ction s
HELP OTHERS SO THAT GOD WILL HELP YOU.

1. Th e sexu al desire an d arou sal disorders are male hypoactive sexual desire disorder, female
sexual interest/arousal disorder, an d erectile disorder (d isorders o th e excitem en t an d p la-
teau p h ases).
2. Th e orgasm ic disorders are delayed ejaculation, premature ejaculation, an d female orgas-
mic disorder.
3. Genitopelvic pain/penetration disorder is diagn osed wh en th e sym p tom s are n ot cau sed by
a gen eral m ed ical con dition .
4. Table 19.3 sh ows ch aracteristics o th e sexu al dys u n ction s.

C. Management
1. Th e p h ysician m u st understand th e p atien t’s sexu al p rob lem be ore p roceed in g with treat-
m en t (e.g., clari y wh at a p atien t m ean s wh en h e says, “I h ave a p roblem with sex.”).
2. Th e p h ysician sh ou ld not assume anything abou t a p atien t’s sexu ality (e.g., a m iddle-aged
m arried m ale p atien t m ay b e h avin g an extram arital h om osexu al relation sh ip ).
3. Th ere is a growin g ten den cy or p h ysician s to manage the sexual problems of heterosexual
and homosexual patients rath er th an to re er th ese p atien ts to sex th erap ists.
4. Man agem en t o sexu al p rob lem s m ay b e b eh avioral, m edical, or su rgical.
5. Behavioral management techniques
a. In sensate-focus exercises (u sed to m an age sexu al d esire, arou sal, an d orgasm ic disor-
d ers), th e in divid u al’s awaren ess o tou ch , sigh t, sm ell, an d sou n d stim u li are in creased

t a b l e 19.3 Characteristics of the DSM-5 Sexual Dysfunctions

Disorder Characteristics

Male hypoactive sexual desire disorder Decreased interest in sexual activity


RISE USMLE NEPAL

Female sexual interest/arousal disorder Inability to maintain vaginal lubrication until the sex act is completed, despite
adequate physical stimulation
Erectile disorder (ED) Lifelong (rare): Has never had an erection sufficient for penetration
Acquired (the most common male sexual disorder): Is currently unable to maintain
erections despite normal erections in the past
Situational (common): Has difficulty maintaining erections in some sexual
situations but not in others
Female orgasmic disorder Lifelong: Has never had an orgasm
Acquired: Is currently unable to achieve orgasm despite adequate genital
stimulation and normal orgasms in the past
Premature ejaculation Ejaculation before the man would like it to occur
Plateau phase of the sexual response cycle is short or absent
Is usually accompanied by anxiety
Is the second most common male sexual disorder after ED
Genitopelvic pain–penetration disorder Persistent (at least 6 mo) pain occurs in association with sexual intercourse
without pelvic pathology
Can also be caused by pelvic pathology, e.g., pelvic inflammatory disease (PID)
due to chlamydial infection (most common) or gonorrhea (most serious)
Is diagnosed only in women
Painful spasms occur in the outer third of the vagina, which make intercourse or
pelvic examination difficult
Chapter 19 Sexuality 209

du rin g sexu al activity, an d p sych ological p ressu re to ach ieve an erection or orgasm is
decreased .
b. In th e “squeeze“ technique , wh ich is u sed to m an age premature ejaculation, th e m an is
tau gh t to iden ti y th e sen sation th at occu rs ju st be ore th e em ission o sem en . At th is
m om en t, th e m an asks h is p artn er to exert p ressu re on th e coron al ridge o th e glan s
on b oth sid es o th e p en is u n til th e erection su b sides, th ereby d elayin g ejacu lation .
c. Relaxation techniques, hypnosis , an d systematic desensitization (see Ch ap ter 17) are
u sed to redu ce an xiety associated with sexu al p er orm an ce.
d. Masturbation m ay b e recom m en ded to h elp th e p erson learn wh at stim u li are m ost
e ective or ach ievin g arou sal an d orgasm .
6. Medical and surgical management
a. Becau se th ey delay orgasm , SSRIs (e.g., lu oxetin e) can b e u sed to m an age premature
HELP OTHERS SO THAT GOD WILL HELP YOU.

ejaculation.
b. Systemic administration of opioid antagonists (e.g., n altrexon e) an d vasodilators (e.g.,
yoh im b in e) h ave b een u sed to m an age erectile disorder.
c. In erectile disorder, sildenafil citrate (Viagra) an d related agen ts:
(1) Act by in h ib itin g th e en zym e (p h osp h od iesterase typ e 5 [PDE5]) th at destroys
cyclic gu an osin e m on op h osp h ate (cGMP), a vasodilator secreted in th e p en is with
sexu al stim u lation . Th is in h ib ition lead s to slowin g o th e d egrad ation o cGMP
an d p ersisten ce o th e erection .
(2) Have side e ects th at in clu de b lu e-colored vision .
(3) Are con train d icated in m en wh o take n itrates.
(4) In clu de n ewer PDE5 in h ibitors with greater p oten cy an d selectivity th an silden a il,
or exam p le, vardenafil (Levitra, Nu viva) an d tadalafil (Cialis).
d. Intracorporeal injection of vasodilators (e.g., p ap averin e, p h en tolam in e) an d im p lan ta-
tion o prosthetic devices are also u sed to m an age erectile disorder.
e. Apomorphine hydrochloride (Up rim a) in creases sexu al in terest an d erectile u n ction by
in creasin g d op am in e availability in th e brain . It is dissolved su blin gu ally, an d its side
e ects in clu d e p ostu ral h yp oten sion an d syn cop e ( ain tin g).

IV. PARAPHILIAS AND PARAPHILIC DISORDERS


A. Definition. Parap h ilias in volve th e p re eren tial u se o unusual objects o sexu al desire or
en gagem en t in unusual sexual activity (Table 19.4). To it DSM-5 criteria or a p arap h ilic disor-
der, th e b eh avior m u st con tin u e over a p eriod o at least 6 m on th s an d cau se im p airm en t in
occu p ation al or social u n ction in g.
RISE USMLE NEPAL

t a b l e 19.4 Paraphilic Disorders

Paraphilic Disorder The Preferential Means of Obtaining Sexual Pleasure Is By

Exhibitionistic Revealing one’s genitals to unsuspecting women so that they will be shocked
Fetishistic Contact with inanimate objects (e.g., women’s shoes, rubber sheets)
Frotteuristic Rubbing the penis against a clothed woman who is not consenting and not aware
(e.g., on a crowded train)
Necrophilistic Engaging in sexual activity with corpses
Pedophilic Engaging in fantasies or actual behaviors with children under age 14 y, of the opposite
or same sex; person is at least age 16 y and ≥ 5 y older than the child; is the most
common paraphilia
Sexual masochism Receiving physical pain or humiliation
Sexual sadism Giving physical pain or humiliation
Transvestic Wearing women’s clothing, particularly underclothing (exclusive to heterosexual men)
Voyeuristic Secretly watching other people (often by using binoculars or cameras) undressing or
engaging in sexual activity
Other specified, e.g., zoophilia Preferentially engaging in sexual activity with animals
210 BRS Behavioral Science

B. Occurrence and management


1. Parap h ilias occu r almost exclusively in men.
2. Pharmacologic management in clu des antiandrogens , female sex hormones, an d SSRIs or
p arap h ilias th at are ch aracterized by h yp ersexu ality.

V. ILLNESS, INJ URY, AND SEXUALITY


A. Heart disease and myocardial infarction (MI)
1. Men wh o h ave a h istory o MI o ten h ave erectile disorder. Both m en an d wom en wh o
HELP OTHERS SO THAT GOD WILL HELP YOU.

h ave a h istory o MI m ay h ave decreased libido becau se o th e side e ects o card iac m ed i-
cation s an d th e fear that sexual activity will cause another heart attack.
2. Gen erally, i exercise th at raises th e h eart rate to 110–130 bpm (e.g., exertion equ al to clim b-
in g two ligh ts o stairs) can be tolerated with ou t severe sh ortn ess o breath or ch est p ain ,
sexu al activity can be resu m ed a ter a cardiac even t su ch as a m yocardial in arction .
3. Sexual positions th at p rodu ce th e least exertion in th e p atien t (e.g., th e partner in the supe-
rior position) are th e sa est a ter MI.

B. Diabetes
1. On e-qu arter to on e-h al o diabetic m en (m ore com m on ly older p atien ts) h ave erectile
disorder. Orgasm an d ejacu lation are less likely to be a ected.
2. Th e m ajor cau ses o erectile d isorder in m en with diabetes are vascular changes an d dia-
betic neuropathy cau sed by dam age to blood vessels an d n erve tissu e in th e p en is as a
resu lt o hyperglycemia .
a. Erectile p rob lem s gen erally occu r several years a ter diabetes is diagn osed bu t may be
the first symptom o th e d isease.
b. Poor metabolic control o d iab etes is related to in creased in ciden ce o sexu al p roblem s.
c. Sild en a il citrate an d related agen ts o ten are e ective in diabetes-related erectile
disord ers.
d. Alth ou gh p h ysiologic cau ses are m ost im p ortan t, psychological factors su ch as ear o
sexu al “ ailu re” also m ay in lu en ce erectile p roblem s associated with diabetes.

C. Spinal cord injury


1. Sp in al cord in ju ries in men cau se erectile an d orgasm ic dys u n ction , retrograde ejacu la-
tion (in to th e bladd er), redu ced testosteron e levels, an d decreased ertility.
2. Sp in al cord in ju ries in women cau se p rob lem s with vagin al lu brication , p elvic vasocon -
RISE USMLE NEPAL

gestion , an d orgasm . Fertility is u su ally n ot adversely a ected.

VI. AGING AND SEXUALITY


A. Physical changes. Alteration s in sexu al u n ction in g n orm ally occu r with th e agin g p rocess.
1. In men, th ese ch an ges in clu de slower erection, diminished intensity of ejaculation, longer
refractory period, an d need for more direct stimulation.
2. In women, th ese ch an ges in clu de vaginal thinning, shortening of vaginal length, an d vaginal
dryness .
3. Horm on e rep lacem en t th erapy, wh ich can reverse th ese vagin al ch an ges, is u sed less re-
qu en tly n ow th an in th e p ast (see Ch ap ter 2). However, local ap p lication o a m oistu rizin g
agen t to th e vagin a to acilitate in tercou rse can be h elp u l.

B. Sexual interest and activity


1. In sp ite o p h ysical ch an ges, societal attitu des, an d loss o th e sexu al p artn er becau se o
illn ess or death , sexual interest usually does not change significantly with increasing age .
2. Con tin u ed sexu al activity is associated with good h ealth . Prolon ged abstin en ce rom sex
leads to aster p h ysical atrop h y o th e gen ital organ s in old age (“use it or lose it”).
Chapter 19 Sexuality 211

t a b l e 19.5 The Effects of Some Prescription Drugs on Sexuality

Associated Neurotransmitter (↑ [Increased]


Effect Drug Type (Representative Agent) or ↓ [Decreased]) Availability

Reduced libido Antidepressant (fluoxetine) ↑Serotonin


Antihypertensive (propranolol) ↓Norepinephrine β
Antihypertensive (methyldopa) ↑Central norepinephrine α
Increased libido Antiparkinsonian (levodopa [l -dopa]) ↑Dopamine
Erectile disorder Antihypertensive (propranolol) ↓Norepinephrine β
Antihypertensive (methyldopa) ↑Central norepinephrine α
Antidepressant (fluoxetine) ↑Serotonin
HELP OTHERS SO THAT GOD WILL HELP YOU.

Antipsychotic (thioridazine) ↓Dopamine


Vaginal dryness Antihistamine (diphenhydramine) ↓Histamine
Anticholinergic (atropine) ↓Acetylcholine
Inhibited orgasm Antidepressant (fluoxetine) ↑Serotonin
(in men and women)
Priapism Antidepressant (trazodone) ↑Serotonin
(persistent erection)
Inhibited ejaculation Antidepressant (fluoxetine) ↑Serotonin
Antipsychotic (thioridazine) ↓Dopamine

VII. DRUGS AND SEXUALITY


A. Prescription drugs can a ect libido, erection , orgasm , ejacu lation , an d oth er sexu al u n c-
tion s, o ten as a resu lt o th eir e ects on n eu rotran sm itter system s (Tab le 19.5).

B. Prescription drugs that lead to decreased sexual function include


1. Antihypertensives , p articu larly α-adren ergic agon ists (e.g., m eth yldop a) an d β-adren ergic
blockers (e.g., p rop ran olol); th e ewest sexu al p roblem s are ou n d with th e u se o
an gioten sin -con vertin g en zym e (ACE) in h ibitors (e.g., cap top ril).
2. Antidepressants , p articu larly SSRIs, sin ce seroton in m ay dep ress sexu al in terest an d delay
orgasm .
3. Antipsychotics , p articu larly dop am in e-2 (D 2) recep tor blockers
a. Dop am in e m ay en h an ce sexu ality; its blockade m ay decrease sexu al u n ction in g.
b. Prolactin levels increase as a result o dopam ine blockade; this m ay in turn depress sexuality.
RISE USMLE NEPAL

C. Drugs of abuse
1. Alcohol and marijuana in crease sexu ality in th e sh ort term by decreasin g p sych ological
in h ib ition s.
a. With lon g-term u se, alcohol may cause liver dysfunction, resu ltin g in in creased estrogen
availab ility an d sexu al d ys u n ction in m en .
b. Ch ron ic u se o marijuana may reduce testosterone levels in m en an d pituitary gonadotro-
pin levels in wom en .
2. Amphetamines an d cocaine in crease sexu ality by stim u latin g dop am in ergic system s.
3. Heroin an d, to a lesser exten t, methadone are associated with su p p ressed libido, retarded
ejacu lation , an d ailu re to ejacu late.

VIII. THE HUMAN IMMUNODEFICIENCY VIRUS AND SEXUALITY


A. Occurrence of human immunodeficiency virus (HIV)
1. More th an 33 million people in the world are in ected with HIV.
a. Most HIV-in ected p eop le live in A rica, Asia, an d Eastern Eu rop e.
212 BRS Behavioral Science

t a b l e 19.6 Route of Contact and Risk of Contracting HIV

Approximate Risk of Contracting


Infection Route HIV per 10,000 Exposures
Sexual Activity with an HIV-Infected Person
Receptive anal intercourse 138
Insertive anal intercourse 11
Receptive vaginal intercourse 8
Insertive vaginal intercourse 4
Direct Contact with Blood of an HIV-Infected Person
Transfusion 9,250
HELP OTHERS SO THAT GOD WILL HELP YOU.

Needle sharing 63
Needlestick 23
HIV-Positive Mother to Fetus
Mother not taking antiretrovirals or other preventative measure 2,500 (1 in 4)
Mother taking antiretrovirals, having a C-section, and not breast-feeding 200 (1 in 50)

b. Fewer th an 1 m illion in ected p eop le live in North Am erica.


c. Fewer th an 2 m illion in ected p eop le live in Latin Am erica an d th e Caribbean ; abou t
0.5 m illion live in Western an d Cen tral Eu rop e.
2. Th ere is a sex difference in th e HIV viral load an d th e sym p tom s o AIDS; a wom an with
th e sam e HIV viral load as a m an is likely to d evelop AIDS soon er th an th e m an .

B. Transmission of HIV
1. Becau se o th e likelih ood o tissu e tearin g leadin g to con tact with th e blood su p p ly, anal
intercourse is th e sexu al beh avior th at is riskiest or tran sm ittin g HIV (Table 19.6).
2. Patien ts wh o are HIV p ositive m u st p rotect th eir sexu al p artn ers rom in ection . I th ey ail
to do so (e.g., do n ot u se a con dom ) an d th e p h ysician h as kn owledge o su ch ailu re, th e
p h ysician m u st en su re th at th e th reaten ed p artn er is in orm ed (see Ch ap ter 23).
3. Pren atal treatm en t with antiretroviral agents su ch as zidovu din e (AZT), lam ivu din e (3TC),
an d / or n evirap in e (NTP) can red u ce th e risk o tran sm ission o HIV rom m oth er to etu s.
However, even i th ey are at h igh -risk, p regn an t wom en can n ot be com p elled to be tested
or treated (see Ch ap ter 23).
RISE USMLE NEPAL
Review Test

Directions: Each o th e n u m b ered item s or in com p lete statem en ts in th is section is ollowed by


an swers or by com p letion s o th e statem en t. Select th e one lettered an swer or com p letion th at
is best in each case.
HELP OTHERS SO THAT GOD WILL HELP YOU.

1. A 45-year-old p h ysician states th at h e h as 4. A 17-year-old wom an p resen ts to th e


been livin g with an oth er m an in a stab le, doctor becau se she h as n ever m en struated
sexu al, love relation sh ip or th e p ast 10 years. an d because sh e has discovered labial
Th is p h ysician is m ost likely to h ave a h istory m asses. In itial exam in ation reveals a tall, th in
in adolescen ce o em ale with n orm al extern al gen italia an d
(A) sedu ction by an old er m an breast develop m en t. A p elvic exam in ation is
(B) m en tal illn ess n ot p er orm ed. Th ere are n o Barr bodies in
(C) sexu al an tasies ab ou t m en th e buccal sm ear. The best way to describe
(D) ch oosin g to sp en d tim e alon e th is p atien t is th at sh e is exp erien cin g
(E) wan tin g sex ch an ge su rgery (A) con gen ital virilizin g adren al h yp erp lasia
(B) an d rogen in sen sitivity syn d rom e
2. A 32-year-old m an wh o was recen tly (C) gen der dysp h oria
diagn osed with HIV states th at h e is su re h e (D) tran svestic etish ism
got th e in ection rom a sexu al en cou n ter h e (E) a lesb ian sexu al orien tation
h ad wh ile h e was on vacation . Wh ich o th e
ollowin g is th e m ost ap p rop riate qu estion 5. A 35-year-old m an m u st wear wom en’s
to ask th e p atien t in order to determ in e h is h igh h eels an d lin gerie to becom e arou sed
sexu al orien tation ? when ever h e h as sexu al in tercou rse with a
(A) “Wou ld you d escrib e you rsel as wom an . He den ies h avin g sexu al in terest in
h om osexu al?” m en . Th e b est way to describ e th is p atien t is
(B) “Are you m ain ly gay or m ain ly straigh t?” th at h e is exp erien cin g
(C) “Are you exclu sively gay or exclu sively (A) con gen ital virilizin g adren al h yp erp lasia
straigh t?” (B) an d rogen in sen sitivity syn d rom e
(D) “Do you p re er to h ave sex with m en , (C) gen der dysp h oria
wom en , or both m en an d wom en ?” (D) tran svestic etish ism
(E) “Do you th in k you are h eterosexu al, (E) a gay sexu al orien tation
RISE USMLE NEPAL

bisexu al or h om osexu al?”


6. Th e best estim ate o th e occu rren ce o
3. A 29-year-old wom an says th at sh e h as h om osexu ality in m en is
always elt as i sh e were “a m an in th e (A) 0.5%–1%
body o a wom an .” Ph ysical an d p elvic (B) 2%–3%
exam in ation s are n orm al. Sh e is sexu ally (C) 5%–10%
attracted to h eterosexu al wom en an d wan ts (D) 20%–25%
to wear m en’s cloth es, take m ale h orm on es, (E) 30%–35%
an d u n dergo a m astectom y an d su rgical sex
reversal so th at sh e can live as a m an . Th e 7. A 50-year-old m an shows breast
best way to describe th is p atien t is th at sh e is en largem en t a ter years o usin g a substan ce.
exp erien cin g The substan ce he is m ost likely to have used is
(A) con gen ital virilizin g adren al h yp erp lasia (A) alcoh ol
(B) an d rogen in sen sitivity syn drom e (B) m ariju an a
(C) gen der dysp h oria (C) h eroin
(D) tran svestic etish ism (D) am p h etam in e
(E) a lesbian sexu al orien tation (E) am yl n itrite

213
214 BRS Behavioral Science

Questions 8 and 9 12. A 65-year-old m arried cou p le com p lain s


to th e d octor th at th eir sex li e is n ot wh at it
A 34-year-old m an h as b een takin g lu oxetin e u sed to b e. Wh ich o th e ollowin g p rob lem s
or th e treatm en t o dep ression or th e p ast is th e cou p le m ost likely to rep ort?
4 m on th s. His m ood is n ow n orm al, bu t h e (A) Prem atu re ejacu lation
rep orts th at h e is h avin g sexu al p roblem s. (B) Vagin al d ryn ess
(C) Sh orter re ractory p eriod
8. Wh ich o th e ollowin g sexu al (D) Decreased sexu al in terest
dys u n ction s is th is m an m ost likely to (E) Pain –p en etration disorder
rep ort?
(A) Li elon g erectile disorder 13. A 32-year-old m an com p lain s th at h e
HELP OTHERS SO THAT GOD WILL HELP YOU.

(B) Acqu ired erectile d isord er h as n o p roblem with erection , bu t th at h e


(C) Prem atu re ejacu lation u su ally h as an orgasm an d ejacu lates b e ore
(D) Delayed orgasm h e ach ieves vagin al p en etration . Th is m an’s
(E) Pain –p en etration d isord er com p lain t
(A) is u n com m on
9. Th e n eu rotran sm itter alteration m ost (B) is associated with dep ression
likely to be associated with th is m an’s sexu al (C) is associated with an absen t excitem en t
di cu lty is p h ase
(A) in creased dop am in e (D) can be e ectively m an aged with
(B) decreased d op am in e in ten sive p sych oth erapy
(C) in creased seroton in (E) can be e ectively m an aged with th e
(D) decreased seroton in squ eeze tech n iqu e
(E) decreased n orep in ep h rin e
Questions 14 and 15
10. A 30-year-old m ale p atien t wh o is HIV
p ositive asks th e doctor wh at typ e o sexu al A 62-year-old p atien t tells th e p h ysician th at
behavior p oses th e m ost risk or tran sm ittin g h e is h avin g di icu lty m ain tain in g an erec-
HIV to h is p artn er. Th e d octor’s b est tion wh en h e h as in tercou rse with h is wi e.
resp on se is
(A) an al in tercou rse 14. Wh ich o th e ollowin g illn esses is
(B) oral–p en ile con tact m ost likely to be associated with th is m an’s
(C) oral–vu lva con tact p roblem ?
(D) vagin al in tercou rse (A) Alzh eim er’s d isease
(E) kissin g (B) Un treated h yp erten sion
(C) Un treated diabetes
11. A h u sb an d an d wi e in th eir m id-30s (D) Myocardial in arction
RISE USMLE NEPAL

state th at th ey are h avin g sexu al p roblem s. (E) Un treated sch izop h ren ia
Du rin g th e in terview, th e d octor d iscovers
th at wh ile th eir sex li e h ad b een good , th e 15. Th e p h ysician recom m en ds th at th e
last tim e th ey tried to h ave in tercou rse (4 p atien t take silden a l citrate (Viagra) p rior to
weeks p reviou sly), th e h u sban d cou ld n ot h avin g in tercou rse. Th e m ajor action o th is
m ain tain an erection . Wh ich o th e ollowin g agen t in th e m an agem en t o erectile disorder
agen ts is m ost likely to h ave cau sed th is is to
sexual p roblem ? (A) in crease th e con cen tration o cGMP
(A) Cocain e (B) decrease th e con cen tration o cGMP
(B) Prop ran olol (C) in crease th e degradation o cGMP
(C) Levod op a (l -dop a) (D) in crease th e con cen tration o
(D) Am yl n itrite p h osp h odiesterase 5 (PDE5)
(E) Dextroam p h etam in e (E) decrease th e degradation o
p rostaglan din E
Chapter 19 Sexuality 215

16. A 25-year-old m an m astu rbates by 20. Wh ich stage o th e sexu al resp on se


ru bbin g again st u n su sp ectin g wom en in cycle sh ows th e greatest di eren ce in len gth
crowd ed bu ses. Th is m an is sh owin g wh ich between m en an d wom en ?
o the ollowin g p arap h ilias? (A) Excitem en t
(A) Fetish ism (B) Plateau
(B) Exh ibition ism (C) Orgasm
(C) Frotteu rism (D) Resolu tion
(D) Voyeu rism
21. Uterin e con traction s m ain ly occu r in
(E) Sexu al m asoch ism
wh ich stage o th e sexu al resp on se cycle?
17. A 55-year-old m arried p atien t com p lain s (A) Excitem en t
(B) Plateau
HELP OTHERS SO THAT GOD WILL HELP YOU.

o erectile p roblem s. Th e m an h as
Parkin son’s disease th at is well con trolled (C) Orgasm
with l -d op a, an d h e is able to work u ll tim e. (D) Resolu tion
He also h as a p art-tim e job th at cau ses h im
22. Th e m ost com m on cau se o p elvic
to work late m ost even in gs. Th e p atien t also
in f am m atory d isease (PID) in wom en is
relates th at h e d rin ks two m artin is each
even in g, sm okes two p acks o cigarettes a (A) gon orrh ea
d ay, an d som etim es u ses cocain e. His sexu al (B) syp h ilis
d ys u n ction is m ost likely to be associated (C) trich om on iasis
with h is (D) ch lam ydia
(E) can d id iasis
(A) work sch edu le
(B) cocain e u se 23. A m an an d wom an in their m id-20s who
(C) l -dop a u se have been m arried or 3 years present to a
(D) alcoh ol drin kin g physician or evaluation o in ertility. During
(E) cigarette sm okin g the interview, the wi e states, “I cannot
understand why I cannot get pregnant. We
18. Th e ten tin g e ect is m ost likely to begin have had sexual relations two to three tim es
in wh ich stage o th e sexu al resp on se cycle per week or the past year.” What is the m ost
an d in m en on ly, wom en on ly, or both m en appropriate next step or the physician to take?
an d wom en ? (A) Re er th e cou p le or m arital cou n selin g.
(A) Excitem en t p h ase: m en on ly (B) Per orm a gyn ecological exam on the wi e.
(B) Plateau p h ase: m en on ly (C) Per orm a testicular exam on the husban d.
(C) Orgasm p h ase: m en on ly (D) Su ggest th at th e h u sban d u n dergo a
(D) Excitem en t p h ase: m en an d wom en ertility evalu ation .
(E) Plateau p h ase: m en an d wom en (E) Su ggest th at th e wi e u n dergo a ertility
(F) Orgasm p h ase: m en an d wom en evalu ation .
RISE USMLE NEPAL

(G) Excitem en t p h ase: wom en on ly (F) Ask th e cou p le wh at th ey m ean by


(H) Plateau p h ase: wom en on ly “sexu al relation s.”
(I) Orgasm p h ase: wom en on ly
24. Con cern ed paren ts tell the pediatrician
19. Th e sex f u sh is m ost likely to begin in that last week their 6-year-old daughter
wh ich stage o th e sexu al resp on se cycle an d walked in on them while they were havin g
in m en on ly, wom en on ly, or b oth m en an d in tercourse. This week, they ound their
wom en ? daughter with an other 6-year-old girl playin g
(A) Excitem en t p h ase: m en on ly the m ale role in what looked like a sim ulation
(B) Plateau p h ase: m en on ly o sexual in tercourse. Physical exam is n orm al.
(C) Orgasm p h ase: m en on ly The n ext step or the doctor to take is to
(D) Excitem en t p h ase: m en an d wom en (A) reassu re th e p aren ts th at th is beh avior is
(E) Plateau p h ase: m en an d wom en typ ical in ch ildren o th is age
(F) Orgasm p h ase: m en an d wom en (B) tell th e p aren ts th at th e ch ild is likely
(G) Excitem en t p h ase: wom en on ly to h ave a lesbian sexu al orien tation in
(H) Plateau p h ase: wom en on ly adu lth ood
(I) Orgasm p h ase: wom en on ly (C) re er th e am ily to a ch ild p sych iatrist
(D) evalu ate th e ch ild’s sex h orm on e levels
(E) advise th e p aren ts th at th e ch ild
p robably h as been sexu ally abu sed
216 BRS Behavioral Science

25. Worried p aren ts tell th eir d octor th at (C) Reassu re th e p aren ts th at cross-gen der
th eir 8-year-old son on ly wan ts to p lay with beh avior su ch as th is is com m on an d will
girls, likes to d ress u p like a girl, an d in sists disap p ear in tim e.
on urin atin g sittin g down . He also says th at (D) In orm th e p aren ts th at it is likely
boys are d irty an d th at girls h ave b etter stu th at th e ch ild will h ave a h om osexu al
an d th at h e wan ts to be called by a girl’s orien tation .
n am e. Wh at is th e m ost ap p rop riate action (E) Teach th e p aren ts th at it is OK or th e
by the p h ysician at th is tim e? ch ild to h ave th ese in terests an d h elp
(A) Tell the paren ts to give the child a tim e-out th em accep t th e ch ild as h e is.
whenever they see him playin g with girls. (F) Tell th e p aren ts th at th ey sh ou ld
(B) Tell th e p aren ts to give th e ch ild on ly con sid er sex reassign m en t su rgery or
HELP OTHERS SO THAT GOD WILL HELP YOU.

m ascu lin e toys su ch as tru cks an d action th e ch ild.


igu res to p lay with .
RISE USMLE NEPAL
An swers an d Exp lan ation s

Typical Board Question


The answer is C. Th is 17-year-old m ale is sh owin g typ ical beh avior. Th ere is n o eviden ce o tran s-
vestic or oth er p arap h ilic d isorder sin ce h e is n ot socially or legally im p aired by h is occasion al
cross-d ressin g b eh avior. He d oes n ot exp ress d iscom ort with h is biological sex an d so does n ot
HELP OTHERS SO THAT GOD WILL HELP YOU.

it criteria or gen der dysp h oria. He m ay or m ay n ot h ave a h om osexu al orien tation ; in eith er
case, h om osexu ality is a n orm al varian t o sexu al exp ression .

1. The answer is C. Like oth er m en with a h om osexu al sexu al orien tation , th is p h ysician is
likely to h ave a h istory o sexu al an tasies ab ou t m en (h eterosexu al m en com m on ly h ave
a h istory o sexu al an tasies ab ou t wom en ). Hom osexu ality is a n orm al varian t o sexu al
exp ression an d is b iologically b ased (an d see TBQ). Th ere is n o evid en ce th at it is asso-
ciated with a h istory in adolescen ce o sed u ction by an old er m an , m en tal illn ess, or a
p re eren ce or bein g alon e. Wh ile p eop le with gen der dysp h oria ( eelin g o bein g b orn in to
th e wron g b od y) m ay seek sex reassign m en t su rgery, in h om osexu ality, th ere is n o d esire
to ch an ge b iological sex.
2. The answer is D. Th e m ost ap p rop riate qu estion to ask th is p atien t is a straigh t orward
on e, or exam p le, “Do you p re er to h ave sex with m en , wom en , or both m en an d wom en ?”
Usin g descrip tors su ch as h om osexu al, h eterosexu al, gay, an d straigh t is less likely to
clari y th e p atien t’s sexu al orien tation an d beh avior.
3. The answer is C. Th is p atien t, wh o h as always elt as i sh e were “a m an in th e body o a
wom an” in th e p resen ce o a typ ical em ale body, h as gen der dysp h oria. Fem ales with con -
gen ital virilizin g ad ren al h yp erp lasia h ave m ascu lin ized gen italia, an d tran svestic etish -
ists are always m ale. Peop le with an drogen in sen sitivity syn drom e are gen etic m ales with
em ale b odies (with wh ich th ey are con ten t); th ey m ore com m on ly h ave sexu al in terest in
m en . Lesb ian wom en h ave sexu al in terest in wom en bu t h ave a em ale gen der iden tity an d
n o desire to ch an ge th eir p h ysical sex. (See also an swers to Qu estion s 4 an d 5.)
4. The answer is B. Th is p atien t, wh o h as a em ale p h en otyp e d esp ite a m ale gen otyp e (e.g.,
n o Barr bodies in th e bu ccal sm ear), h as an drogen in sen sitivity syn drom e. In th is gen etic
RISE USMLE NEPAL

de ect, b ody cells d o n ot resp on d to th e an d rogen b ein g p rodu ced by th e testes, resu lt-
in g in ailu re o p h ysical m ascu lin ization p ren atally. Th e m asses n oted by th e p atien t are
p robably testes, wh ich h ave descen ded in to th e labia. Peop le with an drogen in sen sitiv-
ity syn drom e are gen erally h eterosexu al with resp ect to p h en otyp ic sex (i.e., wom en with
sexu al in terest in m en ). (See also an swers to Qu estion s 3 an d 5.)
5. The answer is D. Th is p atien t, wh o m u st wear wom en’s cloth es to b ecom e sexu ally
arou sed, is sh owin g tran svestic etish ism . (See also an swers to Qu estion s 3 an d 4.)
6. The answer is C. Th e b est estim ate o th e occu rren ce o h om osexu ality in m en is 5%–10%.
7. The answer is A. Th e su b stan ce th at th is 50-year-old m an with breast en largem en t is m ost
likely to h ave u sed is alcoh ol. Lon g-term u se o alcoh ol dam ages th e liver, resu ltin g in
accu m u lation o estrogen s an d em in ization o th e b ody. Mariju an a, h eroin , am p h etam in e,
an d am yl n itrite are m u ch less likely to cau se estrogen accu m u lation .
8. The answer is D. 9. The answer is C. Wh ile th ey m ay be associated with loss o libido an d
erectile disorder, lu oxetin e an d oth er selective seroton in reu p take in h ibitors (SSRIs) are
m ore likely to cau se d elayed or ab sen t orgasm (orgasm ic disorder). Th at is wh y th e SSRIs
are u se u l in m an agin g p rem atu re ejacu lation . Pain –p en etration disorder is n ot associated

217
218 BRS Behavioral Science

sp eci ically with SSRI treatm en t. Th e n eu rotran sm itter alteration m ost likely to be asso-
ciated with d elayed or absen t orgasm is in creased seroton in resu ltin g rom treatm en t
with lu oxetin e. In creased d op am in e ten d s to in crease sexu al in terest an d p er orm an ce.
Decreased dop am in e, decreased seroton in , an d decreased n orep in ep h rin e are less likely to
b e associated with delayed orgasm th an is in creased seroton in .
10. The answer is A. Becau se tissu e tears p rovid in g access to th e blood su p p ly are m ore likely
to occu r in an al in tercou rse, th is is th e typ e o sexu al beh avior th at p oses th e m ost risk
or tran sm ittin g HIV. Wh ile it is p ossib le to tran sm it HIV by oth er sexu al beh aviors (e.g.,
ellatio [oral–p en ile con tact], cu n n ilin gu s [oral–vu lval con tact], vagin al in tercou rse, an d
kissin g), su ch tran sm ission is m u ch less likely th an with an al in tercou rse.
11. The answer is B. O th e listed agen ts, th e on e m ost likely to h ave cau sed erectile disorder
HELP OTHERS SO THAT GOD WILL HELP YOU.

is p rop ran olol, an an tih yp erten sive m edication (β-b locker). Cocain e, am p h etam in es, an d
l -dop a ten d to in crease sexu al in terest an d p er orm an ce by elevatin g dop am in e availabil-
ity. Am yl n itrite (a vasod ilator) is u sed to en h an ce th e sen sation o orgasm .
12. The answer is B. Th is 65-year-old m arried cou p le is m ost likely to be h avin g sexu al p rob-
lem s b ecau se o vagin al d ryn ess du e to lack o estrogen a ter m en op au se. Agin g is also
ch aracterized by a lon ger re ractory p eriod an d delayed ejacu lation in m en an d decreased
in ten sity o orgasm in m en an d wom en . Alth ou gh sexu al beh avior m ay decrease with agin g
becau se o th ese p rob lem s, sexu al in terest rem ain s abou t th e sam e. Pain –p en etration dis-
order is n ot p articu larly associated with agin g.
13. The answer is E. Th is m an is d escrib in g p rem atu re ejacu lation , a com m on sexu al d ys u n c-
tion , wh ich o ten can b e e ectively m an aged with th e squ eeze tech n iqu e (n ot p sych o-
th erapy). Prem atu re ejacu lation is associated with an absen t p lateau p h ase o th e sexu al
resp on se cycle an d is n ot sp eci ically associated with dep ression .
14. The answer is C. 15. The answer is A. Un treated diabetes is m ost likely to be associated with
erectile disorder. Alth ou gh th e m edication s u sed to m an age th ese con dition s are associ-
ated with erectile d isorder, u n treated cardiac p roblem s, h yp erten sion , an d sch izop h ren ia
are n ot associated with erectile disorder. Alzh eim er’s disease is n ot associated with erectile
d isorder. In act, sexu al exp ression m ay be th e last orm o com m un ication in a coup le
wh ere on e p artn er h as Alzh eim er’s disease. Silden a il citrate (Viagra) works by in creasin g
th e con cen tration o cGMP, a vasodilator, in th e p en is, wh ich cau ses erection to p ersist.
16. The answer is C. Th is m an wh o m astu rbates by ru bbin g again st wom en in crowded bu ses
is exh ibitin g rotteu rism . Exh ibition ism in volves a sexu al p re eren ce or revealin g on e’s
gen itals to u n susp ectin g p erson s so th at th ey will be sh ocked. Fetish ism is a sexu al p re er-
en ce or in an im ate ob jects. Sexu al m asoch ism is a p re eren ce or receivin g p h ysical p ain or
RISE USMLE NEPAL

h u m iliation . Voyeu rism is a p re eren ce or secretly watch in g p eop le u n dressin g or en gag-


in g in sexu al activity.
17. The answer is D. Th is p atien t’s erectile p rob lem s are m ost likely to be associated with h is
alcoh ol d rin kin g. Cigarette sm okin g is less likely th an alcoh ol to a ect sexu al u n ction .
l -Dop a an d cocain e ten d to in crease rath er th an decrease sexu al in terest an d p er orm an ce
by elevatin g d op am in e availability. Th e m an’s work sch edu le, wh ile stress u l, is less likely
th an alcoh ol to a ect h is sexu al u n ction in g.
18. The answer is G. Th e ten tin g e ect, elevation o th e u teru s in th e p elvic cavity, begin s du r-
in g th e excitem en t p h ase o th e sexu al resp on se cycle in wom en .
19. The answer is E. Th e sex lu sh irst ap p ears du rin g th e p lateau p h ase o th e sexu al resp on se
cycle in both m en an d wom en .
20. The answer is D. Resolu tion sh ows th e greatest di eren ce in len gth between m en an d
wom en . Men h ave a restin g (re ractory) p eriod a ter orgasm wh en restim u lation is n ot p os-
sib le. Wom en are less likely th an m en to h ave a re ractory p eriod.
21. The answer is C. Uterin e con traction s occu r m ain ly du rin g th e orgasm p h ase o th e sexu al
resp on se cycle.
Chapter 19 Sexuality 219

22. The answer is D. Th e m ost com m on cau se o p elvic in lam m atory disease (PID) in wom en
is in ection with ch lam yd ial; it m ay accou n t or as m an y as 50% o th e cases. Oth er sexu -
ally tran sm itted d iseases can also cau se PID, bu t are less com m on in th e p op u lation th an
ch lam ydial in ection .
23. The answer is F. Th e m ost ap p rop riate n ext step or th e p h ysician to take is to clari y wh at
th e cou p le m ean s by “sexu al relation s.” Sexu ally in exp erien ced p eop le m ay n ot kn ow th at
som e orm s o sexu al exp ression (e.g., ellatio, in tercou rse with ou t ejacu lation ) can n ot
resu lt in p regn an cy. It is in ap p rop riate to con d u ct p h ysical or lab oratory exam in ation s or
th e cau se o in ertility u n til th e p h ysician is assu red th at th e cou p le is h avin g sexu al in ter-
cou rse in volvin g vagin al p en etration with ejacu lation .
HELP OTHERS SO THAT GOD WILL HELP YOU.

24. The answer is A. Th e n ext step or th e d octor to take is to reassu re th e p aren ts th at cu riosity
abou t sexu al b eh avior is typ ical or ch ild ren o th is age. Alth ou gh th e p aren ts sh ou ld sp eak
to th e ch ild to be su re th at sh e h as n ot been sexu ally ab u sed by an ad u lt, h er b eh avior
with th e oth er girl can b e exp lain ed as an attem p t to rep licate th e b eh avior o h er p aren ts.
Sin ce sexu al orien tation is p rim arily b iological, th ere is n o reason to believe th at observin g
sexu al beh avior or p layin g th e m ale role in a sexu al gam e will resu lt in th e ch ild’s h avin g
a lesbian sexu al orien tation . Sin ce p h ysical exam in ation is u n rem arkable, th e ch ild’s sex
h orm on e levels are likely to b e n orm al.
25. The answer is E. Th is 8-year-old boy, wh o h as adop ted th e p lay, dress, an d social p re er-
en ces typ ical o a sch ool-age girl, is sh owin g eviden ce o gen der d ysp h oria. Gen d er id en tity
is related to early exp osu re o th e brain to sex h orm on es an d is u n ch an geable. Th e m ost
e ective strategy in d ealin g with p aren ts o ch ildren with th is disorder is to teach th em th at
it is alrigh t or th e ch ild to h ave th ese in terests an d h elp th em accep t th e ch ild as h e or sh e
is. Presen tin g on ly m ascu lin e toys or p reven tin g u se o em in in e toys will n ot be e ective
in ch an gin g th is ch ild’s beh avior (see also an swer to Qu estion 3 above). Wh en th e ch ild
is an adu lt, h e can decide wh eth er or n ot to p u rsu e sex reassign m en t su rgery. Un like th is
ch ild, h om osexu al in divid u als are com ortable with th eir biological sex an d p re er to h ave
rom an tic an d sexu al relation sh ip s with p eop le o th eir own sex.
RISE USMLE NEPAL
c ha pte r
20 Aggression an d Abu se
HELP OTHERS SO THAT GOD WILL HELP YOU.

Typical Board Question


At 10 o’clock on a Tu esd ay even in g, a su p erm arket em p loyee in ds an 8-year-old boy alon e
in th e store. Th e ch ild looks sad an d is u n kem p t, an d h e h as a 5-cm cu t on h is h an d. Variou s
p ackages o ood are ou n d h idden in h is cloth es. Th e ch ild states th at h is p aren ts do n ot care
th at h e goes ou t alon e. Th e su p erm arket m an ager brin gs th e ch ild to th e n earest h osp ital
em ergen cy room (ER). A ter assessin g an d m an agin g th e ch ild’s in ju ry, wh ich o th e ollowin g
is th e m ost ap p rop riate n ext step or th e ER p h ysician to take?
(A) Con tact th e p rin cip al o th e ch ild’s sch ool
(B) Rep ort th e case to th e h osp ital eth ics com m ittee th e n ext d ay
(C) Rep ort th e case to th e state ch ild p rotective service agen cy
(D) Evalu ate th e ch ild or con d u ct disord er
(E) Con tact th e ch ild’s p aren ts
(See “An sw ers an d Explan ation s” at th e en d of th e ch apter.)

I. AGGRESSION
A. Social determinants of aggression
1. Factors associated with in creased aggression in clu de p overty, ru stration , p h ysical p ain ,
RISE USMLE NEPAL

an d exp osu re to aggression in th e m ed ia (e.g., violence on television).


2. Ch ild ren at risk or sh owin g aggressive beh avior in adu lth ood h ave requ en tly m oved an d
ch an ged sch ools rep eatedly, h ave b een physically an d/ or sexually abused, mistreat animals
an d you n ger or weaker ch ildren , an d can n ot de er grati ication . Th eir p aren ts requ en tly
d isp lay crim in al b eh avior an d u se dru gs an d alcoh ol.
3. Hom icide occu rs m ore o ten in low socioeconomic populations , an d its in ciden ce is
in creasin g. At least h al o th e h om icides resu lt rom guns .
4. In A rican -Am erican an d wh ite males 15–24 years of age , h om icid e is th e leading an d
second leading cause of death, resp ectively; acciden ts are th e secon d an d irst leadin g
cau se o d eath in A rican Am erican an d wh ite m ales, resp ectively, in th is age grou p.

B. Biological determinants of aggression


1. Hormones
a. Androgens are closely associated with aggression . In m ost an im al sp ecies an d h u m an
societies, m ales are m ore aggressive th an em ales; homicide in volvin g stran gers is
committed alm ost exclu sively by men.
b. Androgenic or anabolic steroids , o ten taken by bodybu ilders to in crease m u scle m ass,
can resu lt in high levels of aggression an d even p sych osis. Dep ression requ en tly occu rs
in with drawal rom th ese h orm on es.
220
Chapter 20 Aggression and Abuse 221

c. Estrogen, progesterone , an d antiandrogens th ere ore can b e u se u l in treatin g m ale sex


o en d ers (see Ch ap ter 19).
2. Substances of abuse and their effects on aggression
a. Low doses o alcohol an d barbiturates in h ibit aggression , wh ile h igh doses acilitate it.
b. Wh ile in toxicated h eroin u sers sh ow little aggression , in creased aggression is associ-
ated with th e u se o cocaine, amphetamines , an d phencyclidine (PCP).
3. Neural bases of aggression
a. Serotonin an d γ-am in obu tyric acid (GABA) in h ib it aggression , an d d op am in e an d n or-
ep in ep h rin e acilitate it; low levels o th e seroton in m etabolite 5-h ydroxyin doleacetic
acid (5-HIAA) are seen in p eop le wh o sh ow im p u lsive aggression (see Ch ap ter 4).
b. Drugs u sed to treat in ap p rop riate aggressiven ess in clu de an tidep ressan ts, ben zodi-
HELP OTHERS SO THAT GOD WILL HELP YOU.

azep in es, an tip sych otics (p articu larly atyp ical agen ts), an d m ood stabilizers (e.g.,
lith iu m ).
c. Abnormalities of the brain (e.g., abn orm al activity in th e amygdala an d p repyri orm area
an d p sych om otor an d tem p oral lobe ep ilep sy) an d lesion s o th e tem p oral lobes, ron -
tal lob es, an d h yp oth alam u s are associated with in creased aggression .
d. Violent people o ten h ave a h istory o head injury an d/ or sh ow abn orm al electroen -
cep h alogram (EEG) readin gs.

II. ABUSE AND NEGLECT OF CHILDREN, THE ELDERLY, AND


IMPAIRED PERSONS
A. Overview
1. Typ es o ch ild (p erson s u n der age 18) an d eld er (p erson s aged 65 an d over) ab u se an d
abu se o m en tally or p h ysically im p aired p erson s in clu d e physical abuse, emotional or
physical neglect, an d sexual abuse . Th e elderly m ay also be exp loited or m on etary gain .
2. Abuse-related injuries m u st be di eren tiated rom in ju ries obtain ed du rin g n orm al activ-
ity. Exam p les o accidental (i.e., n on abu se) injuries in ch ild ren in clu d e b ru ises an d scrap es
on bon y p rom in en ces (e.g., ch in , oreh ead, kn ees, elbows) or, in th e elderly, bru isin g on
exten sor su r aces o th e lim bs.
3. Occu rren ce o abu se an d ch aracteristics an d sign s th at in dicate n eglect an d abu se are
sh own in Table 20.1.

B. Sequelae of child abuse


1. Ch ild ren wh o are b ein g abu sed o ten seem sad, sh ow behavioral changes (e.g., are n o lon -
RISE USMLE NEPAL

ger ou tgoin g an d rien d ly), an d do poorly in school.


2. Adu lts wh o were abu sed as ch ildren are m ore likely to:
a. Have dissociative disorders (e.g., dissociative identity disorder) an d borderline personal-
ity disorder (see Ch ap ter 14).
b. Have posttraumatic stress disorder an d oth er an xiety disorders (see Ch ap ter 13).
c. Have depression an d substance-related disorders (see Ch ap ters 12 an d 9, resp ectively).
d. Abuse th eir own ch ildren .

C. Sexual abuse of children


1. Signs
a. Sexually transmitted diseases (STDs) in ch ild ren are sign s o sexu al abu se; ch ild ren do
n ot con tract STDs th rou gh casu al con tact with an in ected p erson or with th eir bed-
cloth es, towels, or toilet seats.
b. Genital or anal trauma is also a sign o sexu al abu se.
c. You n g ch ild ren h ave on ly a vagu e kn owledge abou t sexu al activities; sp eci ic knowl-
edge about sexual acts (e.g., ellatio) in a you n g ch ild o ten in dicates th at th e ch ild h as
b een sexu ally ab u sed.
d. Recu rren t urinary tract infections an d excessive initiation of sexual activity with rien d s
or with you n ger ch ild ren also are sign s o sexu al abu se.
222 BRS Behavioral Science

t a b l e 20.1 Physical Abuse of Children and Elders

Category Features of Child Physical Abuse Features of Elder Physical Abuse

Occurrence
Annual occurrence At least 1 million cases are reported At least 1 million cases are reported
Most cases are not reported Most cases are not reported
Most likely abuser The closest family member (e.g., the mother) The closest family member (e.g., spouse,
daughter, son, or other relative) with
whom the person lives (and who is often
supported financially by the elder)
Characteristics of the Abused and the Abuser
HELP OTHERS SO THAT GOD WILL HELP YOU.

Characteristics of Hyperactivity or mild physical handicap; child is Some degree of worsening cognitive
the abused perceived as slow or different impairment (e.g., Alzheimer’s disease)
Premature, low-birth-weight infant Physical dependence on others
Colicky or “fussy” infant Does not report the abuse but instead says
Physical resemblance to the abuser’s absent, that he fell and injured himself
rejecting, or abusive partner Incontinence
In one-third of cases, victims are younger than 5 y
of age; in one-fourth of cases, victims are 5–9
y of age
Characteristics of Substance-related disorder Substance-related disorder
the abuser Poverty Poverty
Social isolation Social isolation
Delays seeking treatment for the victim Delays seeking treatment for the victim
Personal history of abuse by caretaker or spouse
Signs of Abuse
Neglect Poor personal care and hygiene (e.g., diaper rash, Poor personal care and hygiene (e.g.,
dirty hair) urine odor in incontinent person), lack
of medication or health aids such as
eyeglasses, or dentures
Lack of needed nutrition Lack of needed nutrition
Bruises Particularly in the areas not likely to be injured Often on the inner (flexor) surfaces of the
during normal play, such as buttocks or lower arms from being grabbed
back, or not over bony prominences
Belt or belt-buckle marks
Fractures and Fractures at different stages of healing Fractures at different stages of healing
burns Spiral fractures caused by twisting the limbs Spiral fractures caused by twisting the
limbs
Cigarette and other burns Cigarette and other burns
Wrist or ankle rope burns caused by tying to a bed Wrist or ankle rope burns caused by tying
or chair to a bed or chair
Burns on the feet or buttocks caused by immersion
in hot water
RISE USMLE NEPAL

Other signs Internal abdominal injuries (e.g., ruptured spleen) Internal abdominal injuries (e.g., ruptured
“Shaken baby” syndrome (i.e., retinal detachment spleen)
or hemorrhage and subdural hematoma caused Evidence of depleted personal finances
by shaking the infant to stop it from crying) (the elder’s money was spent by the
Injuries of the mouth caused by forced feeding abuser and other family members)
Injuries of the mouth caused by forced
feeding

2. Occurrence
a. An estim ated 500,000 Am erican ch ildren are sexu ally abu sed p er year.
b. Most sexu ally ab u sed ch ild ren are 8–13 years of age , an d 25% are you n ger th an
8 years old.
c. Ap p roxim ately 20% o wom en an d 5%–10% o m en rep ort sexu al abu se at som e tim e
du rin g th eir ch ild h ood an d ad olescen ce.
3. Characteristics of the sexual abuser
a. Seven ty to n in ety p ercen t o sexu al ab u sers are known to the child, an d 90% o th ese are
m en . Abou t 50% o th ese m en are relatives (e.g., u n cle, ath er, step ath er), an d 50% are
am ily acqu ain tan ces (e.g., m oth er’s boy rien d , n eigh b or).
Chapter 20 Aggression and Abuse 223

b. Alcohol an d drugs are com m on ly u sed by th e abu ser.


c. Th e abu ser typ ically h as marital problems an d no appropriate alternate sexual partner;
occasion ally, h e h as p edop h ilic disorder (i.e., h e p re ers ch ildren to ap p rop riate sexu al
p artn ers) (see Ch ap ter 19).

III. PHYSICAL AND SEXUAL ABUSE OF DOMESTIC PARTNERS


A. Occurrence
1. Dom estic p artn ers are couples who live together an d sh are livin g exp en ses an d h ou seh old
HELP OTHERS SO THAT GOD WILL HELP YOU.

resp on sib ilities.


a. Abu se occu rs wh en on e p artn er h as power (e.g., p h ysical, in an cial, p sych ological)
over th e oth er.
b. Dom estic abu se occu rs between h eterosexu al or h om osexu al m arried cou p les (sp ou -
sal abu se) b u t also between u n m arried cou p les, ad u lt sib lin gs, or cou p les in oth er
dom estic arran gem en ts.
2. Domestic abuse is a com m on reason wom en com e to a h osp ital em ergen cy room . Th e
abu se m ay be p h ysical or sexu al, an d th e abu ser is alm ost always m ale.
3. Th e abu sed p erson may not report to the police or leave the abuser b ecau se h e or sh e h as
n owh ere to go an d becau se th e abu ser h as threatened to harm the abused i h e or sh e
rep orts or leaves h im . (In act, a wom an h as a greatly in creased risk o b ein g killed by h er
abu sive p artn er i sh e leaves.)

B. Evidence of domestic abuse


1. Th e victim com m on ly h as bruises (e.g., blacken ed eyes) an d broken bon es.
2. In pregnant women (wh o h ave a h igh er risk o bein g abu sed), th e injuries are o ten in th e
“baby zone” (i.e., th e b reasts an d ab dom en ).
3. An irrational explanation o h ow th e in ju ry occu rred , delay in seekin g treatm en t, an d
ap p earan ce o sadness in th e victim are oth er in dication s o dom estic abu se.

C. The cycle of abuse includes three phases


1. Buildup of tension in th e abu ser.
2. Abu sive b eh avior (b atterin g).
3. Apologetic an d loving behavior by th e ab u ser toward th e victim .

D. Characteristics o d o m estic a b u sers a n d th eir ab u sed p artn ers can b e ou n d in


RISE USMLE NEPAL

Ta b le 20.2.

t a b l e 20.2 Physical and Sexual Abuse of Domestic Partners

Characteristics of the Abuser


• Is almost always male
• Often uses alcohol or drugs
• Is impulsive and angry
• Has a low tolerance for frustration
• Has threatened to harm the abused if he or she reports or leaves him
• Shows apologetic and loving behavior after the abuse
• Has low self-esteem
Characteristics of the Abused
• Can be male or female
• Has been raised in a home in which there was domestic abuse
• Is financially or emotionally dependent on the abuser
• Blames him- or herself for the abuse
• May neither report to the police nor leave the abuser
• Has low self-esteem
224 BRS Behavioral Science

IV. THE ROLE OF THE PHYSICIAN IN SUSPECTED CHILD, ELDER,


AND DOMESTIC PARTNER ABUSE
A. Abuse of “protected persons” (i.e., children, the elderly, those with physical or mental
impairment)
1. Accord in g to th e law in every state, physicians must report su sp ected physical or sexual
abuse of a child or elderly person, or o an adu lt wh o ap p ears to be p h ysically or mentally
impaired (i.e., protected persons), to th e ap p rop riate am ily social service agen cy (e.g., state
ch ild p rotective service or state adu lt p rotective service) before or in conjunction with treat-
HELP OTHERS SO THAT GOD WILL HELP YOU.

ment o th e p atien t.
2. Th e p h ysician is not required to tell the suspected abuser o th e p rotected p erson th at h e or
sh e su sp ects abu se.
3. Th e p h ysician does not need family consent to h osp italize th e p rotected p erson or p rotec-
tion or treatm en t.
4. Even i there was no in ten tion to in jure, i a cultural remedy such as “coining” (see Chapter 20)
in jures a protected person , such in jury also must be reported to the appropriate agen cy.

B. Domestic partner abuse


1. Direct reporting by the physician of domestic partner abuse is not appropriate becau se th e
victim is u su ally a com p eten t ad u lt between th e ages o 18 an d 64.
2. A p h ysician wh o su sp ects domestic partner abuse sh ou ld :
a. Docu m en t th e abu se.
b. En su re th e cu rren t sa ety o th e abu sed p erson .
c. Develop an emergency escape plan or th e abu sed p erson .
d. Provide emotional support to th e ab u sed p erson .
e. Re er th e ab u sed p erson to an appropriate shelter or program.
f. En cou rage th e abu sed p erson to rep ort th e case to law en orcem en t o icials.

V. SEXUAL AGGRESSION: RAPE AND RELATED CRIMES


A. Definitions. Rape is a crim e o violen ce, n ot o p assion , an d is kn own legally as “sexu al
assau lt” or “aggravated sexu al assau lt.”
1. Rap e in volves sexual contact without consent.
RISE USMLE NEPAL

2. Vagin al p en etration by a p en is, in ger, or oth er object m ay occu r.


3. Erection an d ejacu lation do n ot h ave to occu r.
4. Sodomy is de in ed as th e in sertion o th e p en is in to th e oral or anal orifice . Th e victim m ay
b e m ale or em ale.

B. Legal considerations
1. Becau se rapists may use condoms to avoid con tractin g HIV or to avoid DNA iden ti ication ,
or b ecau se th ey m ay h ave d i icu lty with erection or ejacu lation , sem en m ay n ot be p res-
en t in th e vagin a or an u s o a rap e victim .
2. A victim is not required to prove that she resisted the rapist or h im to be con victed. A rap ist
can b e con victed even th ou gh th e victim asks h im to u se a con dom or oth er orm o sexu al
p rotection .
3. Certain in orm ation ab ou t th e victim (e.g., p reviou s sexu al activity, “sedu ctive” cloth in g
worn at th e tim e o th e attack) is gen erally n ot adm issible as eviden ce in rap e trials.
4. Husbands can be prosecuted or orcin g th eir wives to h ave in tercou rse. It is illegal to orce
an yon e to en gage in sexu al activity.
5. Even i a wom an con sen ts to go on a d ate with a m an an d con sen ts to sexu al activity n ot
in volvin g in tercou rse, a m an can b e p rosecu ted or rap e (“date rape ”).
6. Con sen sual sex m ay be con sidered rape (“statutory rape ”) i the victim is younger than 16 or
18 years old (depen din g on state law) or is older than this but is physically or mentally impaired.
Chapter 20 Aggression and Abuse 225

C. Characteristics of the rapist and victim


1. The rapist
a. Rapists are u su ally you n ger th an 25 years o age.
b. Th ey are u su ally th e same race as th e victim .
c. Th ey are u su ally known to the victim.
d. Th ey o ten u se alcohol.
2. The victim
a. Rap e victim s are m ost typ ically between 16 and 24 years o age.
b. Rap e m ost com m on ly occu rs inside the victim’s home .
c. Vaginal injuries may be absent, p articu larly in p arou s wom en (th ose wh o h ave h ad
ch ildren ).
HELP OTHERS SO THAT GOD WILL HELP YOU.

D. The sequelae of rape


1. For a variety o reason s, in clu din g sh am e, ear o retaliation , an d th e di icu lties in volved
in su bstan tiatin g rap e ch arges, only 25% of all rapes are reported to the police .
2. Oth ers m ay com m on ly blame the victim in rap e cases.
3. Th e len gth o th e em otion al recovery p eriod a ter rap e varies bu t is com m on ly at least
1 year. Posttraumatic stress disorder som etim es occu rs a ter rap e (see Ch ap ter 14).
4. Th e m ost e ective typ e o cou n selin g is grou p th erapy with oth er rap e victim s.

E. The role of the physician in rape cases


1. Immediately after the rape , th e p h ysician sh ou ld:
a. Take th e p atien t’s h istory in a su p p ortive m an n er an d n ot qu estion th e p atien t’s verac-
ity or ju dgm en t.
b. Per orm a gen eral physical examination an d con d u ct laboratory tests (e.g., cu ltu res or
sexu ally tran sm itted d iseases rom th e vagin a, an u s, an d p h aryn x; test or p resen ce o
sem en ).
c. Prescrib e p rop h ylactic antibiotics an d p ostcoital contraceptive measures (e.g., levo-
n orgestrel [“Plan B”]) i ap p rop riate.
d. Encourage th e p atien t to n oti y th e p olice. Th e doctor is n ot requ ired to n oti y th e
p olice i th e wom an is a com p eten t ad u lt.
2. Up to 6 weeks after the rape
a. Discu ss with th e p atien t th e emotional an d physical sequelae o th e rap e (e.g., su icidal
th ou gh ts, vagin al b leed in g) an d , i n eed ed , re er h er or lon g-term cou n selin g or a su p -
p ort grou p.
b. Do a pregnancy test an d rep eat oth er laboratory tests i ap p rop riate.
RISE USMLE NEPAL
Review Test

Directions: Each o th e n u m b ered item s or in com p lete statem en ts in th is section is ollowed by


an swers or by com p letion s o th e statem en t. Select th e one lettered an swer or com p letion th at
is best in each case.
HELP OTHERS SO THAT GOD WILL HELP YOU.

1. A ter h is p aren ts die, th e am ily o a 3. At a p ostp artu m exam in ation 3 weeks


30-year-old p atien t with Down’s syn d rom e a ter a n orm al delivery, a p h ysician n otices
decid e th at h e will live with an d b e cared th at th e p atien t h as a tear in th e vagin al
or by h is 27-year-old n ep h ew. Wh en th e orif ce. Wh en asked , th e p atien t tells th e
n ep h ew brin gs th e p atien t to th e doctor or doctor th at h er h u sb an d orced h er to
a yearly ch ecku p, th e d octor n otices th at th e resu m e sexu al in tercou rse 6 days a ter givin g
p atien t is m aln ou rish ed, m alodorou s, an d birth even th ou gh sh e told h im “No.” Wh at is
u n kem p t. Th e n ep h ew re u ses to leave th e th e m ost ap p rop riate th in g or th e doctor to
doctor alon e with th e p atien t. Wh ich o th e say to th e p atien t at th is tim e?
ollowin g is th e m ost ap p rop riate n ext step (A) “Have you been abu sed like th is in th e
or th e doctor to take? p ast?”
(A) Con ron t th e n ep h ew abou t n eglectin g (B) “Tearin g o th e vagin al ori ice is a n orm al
th e p atien t com p lication o in tercou rse a ter vagin al
(B) Call th e state agen cy or am ily delivery.”
p rotective services a ter th e n ep h ew (C) “I am sorry or you r exp erien ce, n o on e
gives con sen t deserves th at kin d o treatm en t.”
(C) Call th e state agen cy or am ily (D) “I m u st call th e p olice an d rep ort th at
p rotective services with ou t n oti yin g th e you h ave been sexu ally abu sed.”
n ep h ew (E) “Please tell m e m ore abou t you r
(D) Sp eak to th e n ep h ew to discu ss h ow to relation sh ip with you r h u sban d.”
deal with beh avioral issu es in p atien ts
with Down’s syn drom e 4. A 3-m on th-old in an t is brought to the
(E) Sp eak to th e n ep h ew to clari y wh y th e em ergency departm ent unconscious. While no
p atien t is bein g n eglected external injuries are seen, physical exam ination
reveals a subdural hem atom a an d retin al
2. A 3-m on th -old ch ild is b rou gh t to th e ER. hem orrhages. The parents tell the physician
RISE USMLE NEPAL

Th e ch ild is op py an d di f cu lt to arou se. that the child ell o his changing table the
Laboratory resu lts in clu de decreased previous day. A ter stabilizin g the child, the
glucose, in creased in su lin , an d decreased em ergency departm ent physician should
p lasm a C p ep tid e. Wh at is m ost likely to h ave (A) con tact th e state ch ild p rotective service
cau sed th is p ictu re? agen cy
(A) Exogen ou s in su lin (B) qu estion th e p aren ts to determ in e i th ey
(B) In su lin om a h ave abu sed th e ch ild
(C) Pan creatitis (C) in orm th e p aren ts th at h e su sp ects th at
(D) Adren al tu m or th ey h ave abu sed th e ch ild
(E) Nu trition al de icien cy (D) ob tain th e p aren ts’ p erm ission to
h osp italize th e ch ild
(E) ob tain th e p aren ts’ p erm ission to call a
p ediatric n eu rologist

226
Chapter 20 Aggression and Abuse 227

5. Wh ich o th e ollowin g in ju ries in a 8. A 33-year-old sin gle wom an wh o h as a


4-year-old ch ild is m ost likely to be th e resu lt 4-year-old ch ild p resen ts to th e em ergen cy
o p h ysical ab u se? room an d rep orts th at sh e was rap ed by a
(A) Cu t on th e ch in m an sh e was on a d ate with 2 d ays ago. Th e
(B) Bilateral b ru ises on th e kn ees p h ysical exam in ation sh ows n o p h ysical
(C) Scrap ed oreh ead evid en ce o rap e (e.g., n o in ju ries, n o
(D) Cu t on th e elbow sem en ). Sh e ap p ears an xiou s, dish eveled,
(E) Ru p tu red sp leen an d “sp acey.” It is m ost likely th at th is
wom an
6. A 40-year-old wom an p resen ts to th e (A) is delu sion al
em ergen cy room with bru ises on h er righ t (B) is m alin gerin g
HELP OTHERS SO THAT GOD WILL HELP YOU.

ch eek an d a d eep laceration above h er (C) h as illn ess an xiety disorder


righ t eye. Th e wom an , wh o n otes th at sh e (D) h as con version disorder
h as h ad “a p roblem with alcoh ol” or m ore (E) h as been rap ed an d th e rap ist u sed a
th an 10 years, states th at h er h u sb an d h it con d om
h er becau se sh e did n ot h ave din n er on th e
table wh en h e cam e h om e rom work. A ter 9. A 7-year-old ch ild an d h er m oth er both
treatin g h er in ju ries, th e m ost ap p rop riate h ave ch lam ydia. Th e ch ild’s in ection is m ost
qu estion or th e p h ysician to ask is likely to be th e resu lt o
(A) “Wou ld you d escrib e you rsel as an (A) sleep in g in th e sam e bed as th e m oth er
alcoh olic?” (B) sexu al ab u se
(B) “Wh y d o you th in k you r h u sban d ab u ses (C) m astu rb ation
you ?” (D) u sin g th e m oth er’s towel
(C) “Do you th in k it is sa e or you to retu rn (E) bath in g in th e m oth er’s b ath tu b
h om e to you r h u sb an d?”
(D) “Wou ld you like to talk ab ou t you r 10. On at least th ree occasion s, a 10-year-
p roblem with alcoh ol?” old boy is ou n d takin g lu n ch m on ey rom
(E) “Did you r ath er ab u se you r m oth er?” th e oth er ch ildren in h is class. Th e b oy is
(F) “Do you th in k you r drin kin g h as h ad a u n derweigh t, h as dirty cloth es an d h air, an d
n egative e ect on you r m arriage?” lives in a m otel room with h is sin gle m oth er
(G) “Wou ld you like som e in orm ation on an d ou r siblin gs. Th is p ictu re m ost closely
Alcoh olics An on ym ou s?” su ggests
(A) atten tion de icit h yp eractivity disorder
7. An 18-year-old in tellectu ally disab led (ADHD)
wom an wh o h as an IQ o 50 agrees to h ave (B) Tou rette’s disorder
sexu al in tercou rse with th e 18-year-old (C) con d u ct d isord er
p residen t o th e h igh sch ool sen ior class. (D) op p osition al d e ian t d isord er
RISE USMLE NEPAL

Sexu al in tercou rse between th ese two p eop le (E) ch ild n eglect
is best described as
(A) con sen su al sex
(B) statu tory rap e
(C) sodom y
(D) ch ild ab u se
(E) sexu al ab u se
228 BRS Behavioral Science

11. A m oth er brin gs h er 9-year-old d au gh ter 13. Th e m ost ap p rop riate n ext step or th e
to th e p h ysician wh o h as been carin g or p h ysician to take is to
th e am ily or th e p ast 10 years. Th e m oth er (A) con tact th e state ch ild p rotective service
rep orts th at over th e p ast 2 weeks, th e agen cy
ch ild h as b een u rin atin g requ en tly an d (B) ask th e m oth er’s p erm ission to con su lt
com p lain in g o p ain on u rin ation . Sh e n otes with a ch ild p sych iatrist
th at 2 m on th s ago, th e ch ild sh owed th e (C) qu estion th e ath er abou t th e ch ild’s
sam e sym p tom s. Th e p h ysician ob serves rem ark
th at, wh ile orm erly rien d ly an d ou tgoin g, (D) qu estion th e ch ild u rth er to determ in e i
th e ch ild n ow seem s sad an d does n ot m ake sh e is tellin g th e tru th
eye con tact with h im . Th e m oth er also states (E) con tact a ch ild p sych iatrist to determ in e
HELP OTHERS SO THAT GOD WILL HELP YOU.

th at sin ce sh e rem arried 5 m on th s ago, i th e ch ild is tellin g th e tru th


th e ch ild h as b een d oin g p oorly in sch ool. (F) con tact a p ed iatric gyn ecologist to
Th e m ost likely exp lan ation or th is clin ical determ in e i sexu al ab u se h as occu rred
p ictu re is th at th e ch ild
(A) is an gry th at h er m oth er rem arried 14. In evalu atin g th e risk o leavin g th is ch ild
(B) is com p lain in g to gain atten tion rom with h er p aren ts, wh ich o th e ollowin g is
h er m oth er m ost closely associated with an in creased
(C) is bein g sexu ally abu sed by th e m oth er’s risk th at th e ch ild will b e ab u sed again ?
n ew h u sban d (A) Th e ch ild h as a qu iet, p assive p erson ality.
(D) is com p lain in g to avoid sch ool (B) Th e p aren ts are in volved in m arital
(E) is com p lain in g to exp lain h er sch ool th erapy.
p roblem s (C) Th e p aren ts are in tellectu ally disabled.
(D) Th ere is a h istory o abu se in th e p aren ts’
12. A 93-year-old m ild ly dem en ted wom an , own ch ild h ood s.
wh o is occasion ally in con tin en t, lives (E) Th e ath er is em p loyed in law
with h er d au gh ter. Sh e atten d s a d ay care en orcem en t.
p rogram rom 9 a m to 1 pm . From 1 pm to 4
pm , a n eigh bor (wh o h as an alcoh olic son 15. Th e con cern ed m oth er o an 11-year-
an d an u n em p loyed son ) takes care o th e old boy brin gs h er son to th e doctor a ter
elderly wom an . Th e wom an is brou gh t d iscoverin g h im in bed with an d sexu ally
to th e em ergen cy room by h er dau gh ter on dlin g h is 4-year-old m ale cou sin . Th e
with in ju ries th at su ggest p h ysical ab u se. m oth er n otes th at at a recen t m eetin g, th e
Th e p erson m ost likely to h ave abu sed th is 11-year-old’s teach er rep orted th at, wh ile
wom an is h e h ad b een doin g well in sch ool an d with
(A) a d ay care p rogram worker rien ds, h is grades recen tly h ave been
(B) th e n eigh bor’s alcoh olic son slackin g an d h e h as stop p ed socializin g
RISE USMLE NEPAL

(C) th e n eigh bor with th e oth er ch ild ren in class. Ph ysical


(D) th e elderly wom an’s dau gh ter exam in ation is u n rem arkable. Wh ich o th e
(E) th e n eigh b or’s u n em p loyed son ollowin g sh ou ld th e d octor con sid er f rst to
exp lain th e boy’s p roblem atic beh avior?
Questions 13 and 14 (A) He h as con du ct disorder.
(B) He h as been sexu ally abu sed by an adu lt.
A 4-year-old girl tells th e p h ysician th at h er (C) He is sh owin g n orm al p readolescen t
ath er, a law en orcem en t o icer, asked h er to beh avior.
tou ch h is p en is. Ph ysical exam in ation o th e (D) He h as op p osition al de ian t disorder.
ch ild is u n rem arkab le. (E) He is sh owin g th e em ergen ce o a
h om osexu al orien tation .
An swers an d Exp lan ation s

Typical Board Question


The answer is C. An 8 -year-old ch ild sh ou ld n ot be ou t alon e at n igh t, an d i on e is ou n d alon e, it
is likely th at th e p aren ts are n egligen t. Th ere ore, th e case sh ou ld be rep orted to th e state ch ild
p rotective services im m ed iately. Wh ile ch ildren with con du ct disorder sh ow beh avior th at vio-
HELP OTHERS SO THAT GOD WILL HELP YOU.

lates social n orm s (e.g., stealin g), th ere is eviden ce ( ood p ackages secreted in h is cloth in g, th in
ap p earan ce) th at th is ch ild is stealin g ood b ecau se h e is h u n gry. Con tactin g th e p rin cip al o th e
ch ild’s sch ool or rep ortin g th e case to th e h osp ital eth ics com m ittee th e n ext d ay is n ot goin g to
p rotect th e ch ild. Sin ce th ey are ap p aren tly are n ot aware or do n ot care th at th e ch ild is m issin g,
th e p aren ts will b e con tacted by th e ch ild p rotective agen cy wh ile th e ch ild is bein g cared or in
th e h osp ital (see also Qu estion 10).

1. The answer is C. Beca u se th is p a tien t with an in tellectu al d isab ility ap p ears to h ave
b een n eglected , th e d octor m u st p rotect th e p atien t by im m ed iately n oti yin g th e state
a gen cy or a m ily p rotective services. Sh e d o es n ot h a ve to tell th e n ep h ew o r get h is
con sen t to d o th is (a n d see Qu estion 7). Con ron tin g th e n ep h ew ab ou t n eglectin g th e
p a tien t, cla ri yin g wh y th e p a tien t is b ein g n eglected , or sp eakin g to th e n ep h ew ab ou t
h ow to d ea l with b eh aviora l issu es in p a tien ts with Down’s syn d rom e will n ot p rotect th e
p a tien t a t th is tim e.
2. The answer is A. Th is 3-m on th -old ch ild wh o is di icu lt to arou se sh ows eviden ce, or
exam p le, decreased glu cose, in creased in su lin , an d decreased p lasm a C p ep tide, o h av-
in g received exogen ou s in su lin . It is likely th at th e m oth er h as given th e ch ild in su lin in
order to get atten tion rom m edical p erson n el, a kn own m an i estation o actitiou s disorder
im p osed on an oth er (see Ch ap ter 13) an d a orm o ch ild abu se. In in su lin om a, th ere wou ld
b e in creased rath er th an decreased p lasm a C p ep tide. Th ere is n o eviden ce o p an creatitis,
ad ren al tu m or, or n u trition al de icien cy in th is ch ild .
3. The answer is E. No on e h as th e righ t to orce an oth er p erson to h ave sexu al activity, an d
tearin g o th e vagin al ori ice, wh ile n ot a n orm al com p lication o in tercou rse a ter vagin al
d elivery, can occu r i in tercou rse is resu m ed too soon a ter ch ildbirth . However, becau se
th is wom an is a com p eten t adu lt, i sh e wan ts to ile ch arges again st h er h u sb an d , sh e m u st
RISE USMLE NEPAL

rep ort th e abu se to th e p olice h ersel . Certain ly, n o on e deserves th at kin d o treatm en t, bu t
in order to get m ore in orm ation abou t th e p atien t’s m arital relation sh ip, th e m ost ap p ro-
p riate th in g or th e doctor to say to th e p atien t at th is tim e is “Please tell m e m ore abou t
you r relation sh ip with you r h u sb an d .”
4. The answer is A. A ter stabilizin g th e in an t, th e em ergen cy dep artm en t p h ysician sh ou ld
con tact th e state ch ild p rotective service agen cy to rep ort su sp ected ch ild abu se. Su bdu ral
h em atom a, retin al h em orrh age, an d retin al detach m en t are sign s o th e “sh aken baby”
syn d rom e, a orm o ch ild ab u se in wh ich an adu lt sh akes a ch ild to stop its cryin g. Th e
sh aken ch ild m ay h ave n o extern al in ju ries. Ch ild abu sers, su ch as th ese p aren ts, com m on ly
d elay seekin g treatm en t an d m ake u p som e exp lan ation or th e in ju ries su ch as “th e ch ild
ell.” Th e p h ysician m u st rep ort an y su sp icion o abu se to th e ap p rop riate au th ority, b u t
d oes n ot h ave to qu estion th e p aren ts or in orm th em o th is su sp icion . Sim ilarly, wh en a
p h ysician su sp ects ch ild p h ysical or sexu al abu se, h e or sh e d oes n ot n eed a p aren t’s p er-
m ission to exam in e, h osp italize, or treat th e ch ild or to con su lt with a sp ecialist.
5. The answer is E. An in tern al in ju ry, su ch as a ru p tu red sp leen , is m ost likely to be th e resu lt
o abu se in a 4-year-old ch ild. Ch in , kn ee, oreh ead, an d elbow in ju ries are m ore likely to
h ave b een obtain ed d u rin g n orm al p lay.

229
230 BRS Behavioral Science

6. The answer is C. Th e m ost im p ortan t th in g or th e p h ysician to d o is to en su re th e


sa ety o th is ab u sed p atien t. Th ere ore, q u estion in g th e wom an ab ou t wh eth er it is sa e
or h er to retu rn h om e sh ou ld b e th e in itia l in terven tion . Im p lyin g th at h er d rin kin g
(e.g., “Do you th in k you r d rin kin g h as h ad a n egative e ect on you r m arria ge?” “Wou ld
you d escrib e you rsel as an a lcoh olic?” or “Wh y d o you th in k th at you r h u sb a n d a b u ses
you ?”) or h er a th er’s b eh avior (e.g., “Did you r a th er ab u se you r m oth er?”) is related
to th e a b u se seem s to b lam e th e victim a n d is n ot ap p rop ria te. Trea tm en t (e.g., “Wou ld
you like to talk ab ou t you r p rob lem with alcoh ol?” or “Wou ld you like som e in orm a -
tion on Alcoh olics An on ym ou s?”) can wa it u n til th e im m ed iate p rob lem , en su rin g h er
sa ety, is ad d ressed .
7. The answer is B. Even th ou gh b oth are legally o adu lt age, sexu al in tercou rse between
HELP OTHERS SO THAT GOD WILL HELP YOU.

th is p erson with an in tellectu al disability an d a n on im p aired p erson is best described as


statu tory rap e. Becau se th e wom an h as im p aired m en tal u n ction in g (i.e., a m en tal age
o 7.5 years, p er Ch ap ter 2), sh e m ay n ot u lly u n derstan d th e m ean in g o h er con sen t in
th is con text. Con sen su al sex im p lies th at both p eop le h ave th e ability to decide to in teract.
Sodom y is oral–p en ile or an al–p en ile con tact. Ch ild abu se an d sexu al abu se are n ot th e
best id en ti iers or th e b eh avior d escrib ed h ere.
8. The answer is E. It is m ost likely th at th is wom an h as b een rap ed by th e m an sh e wen t
ou t with (“d ate rap e”). Becau se th ere is n o sem en , th e rap ist m ay h ave u sed a con d om .
Parou s wom en su ch as th is p atien t m ay sh ow n o p h ysical sign s o rap e. Rap e victim s
m ay ap p ear an xiou s, d ish eveled , an d “sp acey” (e.g., u se o d issociation as a d e en se
m ech an ism ). Patien ts rarely lie to d octors. Th ere is n o in d ication th at th is wom an is
lyin g or obviou s gain (m alin gerin g), is d elu sion al, or h as illn ess an xiety d isord er or
con version d isord er.
9. The answer is B. Th e ch ild’s ch lam yd ial in ection is m ost likely to be th e resu lt o sexu al
ab u se. Sexu ally tran sm itted d iseases are rarely i ever con tracted by m astu rbation or by
sleep in g in th e sam e b ed , u sin g th e sam e towel, or b ath in g in th e sam e b ath tu b as an
in ected p erson .
10. The answer is E. Th e m ost likely exp lan ation or th is ch ild’s stealin g beh avior is th at h e is
bein g n eglected an d is takin g m on ey to b u y ood . Evid en ce or n eglect in clu d es th e act
th at h e is u n d erweigh t, h as d irty cloth es an d h air, an d lives in a crowded situ ation . It is less
likely th at h e h as on e o th e b eh avioral d isru p tive disorders, an d th ere is n o eviden ce o
ADHD or th e tics o Tou rette’s d isord er (see also th e TBQ).
11. The answer is C. Th e m ost likely exp lan ation or th is clin ical p ictu re is th at th is 9-year-old
girl is bein g sexually abu sed by h er m oth er’s n ew h u sban d. Sign s o sexu al abu se in clu de
RISE USMLE NEPAL

u rin ary tract in ection s an d b eh avioral ch an ges, or exam p le, sadn ess an d with drawal,
as well as sch ool p rob lem s. It is m u ch less likely th at th e ch ild’s sign s an d sym p tom s are
becau se o an ger at h er m oth er, or rep resen t an attem p t to gain atten tion or avoid sch ool.
Rath er, th e abu se m ay h elp exp lain th e ch ild’s recen t di icu lties in sch ool.
12. The answer is D. A close relative wh o cares or th e p erson (e.g., th e dau gh ter) is m ost likely
to h ave ab u sed th is eld erly d em en ted wom an . Alth ou gh n o excu se or abu se, th is m ay be
a resu lt, in p art, o th e stresses associated with carin g or a d em en ted, in con tin en t elderly
p erson . Un related p eop le su ch as caretakers (even i alcoh olic or u n em p loyed) are m u ch
less likely th an a close relative to abu se an elderly p erson .
13. The answer is A. 14. The answer is D. Wh en a ch ild o an y age rep orts in ap p rop riate sexu al
tou ch in g, th e p h ysician m u st con tact th e state ch ild p rotective service agen cy. Th is exam -
p le d em on strates th at a ch ild m ay sh ow n o p h ysical sign s o sexu al ab u se. Th e p h ysician
m u st assu m e th at p atien ts (even you n g on es) are tellin g th e tru th . Th e p h ysician does n ot
n eed to talk to th e ch ild u rth er, con su lt a ch ild p sych iatrist, con tact a p ediatric gyn ecolo-
gist, or talk to th e ath er to con irm th e story. Th e state agen cy will h an d le th ese m atters. In
evalu atin g th e risk o leavin g abu sed ch ildren with th eir p aren ts, a h istory o abu se in th e
p aren ts’ own ch ild h oods is associated with an in creased risk th at th e ch ild will b e ab u sed
Chapter 20 Aggression and Abuse 231

again . In telligen ce o th e p aren ts, em p loym en t o th e p aren ts in law en orcem en t, an d


wh eth er th e p aren ts are in volved in m arital th erapy are n ot sp eci ically associated with th e
risk o ch ild abu se.
15. The answer is B. Wh ile h eterosexual an d hom osexual play between children o about the
sam e age is typ ical an d n orm al (see Ch ap ters 2 an d 19), sexual play between children who
vary in age (n o m atter what their sex) is o con cern . In this case, the behavior by the older
ch ild suggests that recen tly h e has been sexually abused by an adult an d is replicatin g what
has been don e to h im with h is 4-year-old cou sin . His slackin g grades an d u n sociable behav-
ior give urther eviden ce that the 11-year-old boy has been abused. Con duct disorder an d
op position al de ian t disorder are u n likely sin ce the child has n o previous history o behavior
problem s.
HELP OTHERS SO THAT GOD WILL HELP YOU.
RISE USMLE NEPAL
Th e Ph ysician –Patien t
c ha pte r
21 Relation sh ip
HELP OTHERS SO THAT GOD WILL HELP YOU.

Typical Board Question


A p h ysician advises a 50-year-old m an wh o sm okes 2 p acks o cigarettes a day th at h e n eeds
to stop sm okin g. In resp on se, th e p atien t tells th e doctor th at h e h ad an u n cle wh o sm oked
all o h is li e an d died at age 95 o n atu ral cau ses. Accord in g to th e “Stages o Ch an ge” m od el,
in wh ich stage o ch an ge is th is p atien t m ost likely to be?
(A) Precon tem p lation
(B) Con tem p lation
(C) Prep aration
(D) Action
(E) Main ten an ce
(See “An sw ers an d Explan ation s” at th e en d of th e ch apter.)

I. MEDICAL PRACTICE
A. Seeking medical care
1. Patien ts’ b eh avior wh en ill an d th eir exp ectation s o p h ysician s are in lu en ced by th eir
culture (see Ch ap ter 18), p reviou s exp erien ces with m edical care, p h ysical an d m en tal
con d ition s, personality styles (n ot n ecessarily p erson ality disorders; see Table 14.3 an d
RISE USMLE NEPAL

Table 21.1), an d coping skills .


2. On ly abou t one-third of Americans with symptoms seek medical care ; m ost p eop le con ten d
with illn esses at h om e with over-th e-cou n ter m edication s an d h om e m an agem en t.

B. Seeking psychiatric care


1. In th e Un ited States, th ere is a stigma to h avin g a p sych iatric illn ess. Psych iatric sym p tom s
are con sid ered by m an y Am erican s to in dicate a moral weakness or a lack of self-control.
Becau se o th is stigm a, m an y p atien ts ail to seek h elp.
2. It is im p ortan t or p atien ts to seek h elp sin ce th ere is a stron g correlation between psycho-
logical illness an d physical illness . Morbidity rates an d m ortality rates are m u ch h igh er in
p atien ts wh o n eed p sych iatric atten tion .

C. The “sick role”


1. A p erson assu m es a p articu lar role in society an d certain beh avioral p attern s wh en h e or
sh e is ill (th e “sick role,” d escrib ed by T. Parson s). Th e sick role in clu des exemption from
usual responsibilities an d exp ectation o care by oth ers, as well as workin g toward b ecom -
in g h ealth y an d coop eratin g with h ealth care p erson n el in gettin g well.

232
Chapter 21 The Physician–Patient Relationship 233

t a b l e 21.1 Patient Personality Style and Behavioral Characteristics during Illness

Personality Style Behavioral Characteristics during Illness

Paranoid Blames the physician for the fact that he or she is ill
Schizoid Becomes even more withdrawn during illness
Schizotypal Bizarre behavior may mask serious illness
Histrionic May be dramatic, emotionally changeable, and approach the physician in an
inappropriate sexual fashion during illness
Narcissistic Has a perfect self-image, which is threatened by illness and may refuse needed
treatment, which can alter his or her appearance
Antisocial May self-write or alter prescriptions and lie to the physician
HELP OTHERS SO THAT GOD WILL HELP YOU.

Borderline Idealizes the physician at first, may make gestures of or attempt self-harm when ill
Avoidant Interprets physician health suggestions as criticisms, fears rejection by the doctor, is
overly sensitive to a perceived lack of attention or caring
Obsessive–compulsive Fears loss of control and may in turn become more controlling during illness
Dependent Becomes more needy during illness and wants the physician to make all decisions
and assume all responsibility
Passive–aggressive Asks for help but then does not adhere to the physician’s advice

2. Critics o th e sick role th eory argu e th at it applies only to middle-class p atien ts with acu te
p h ysical illn ess, em p h asizes th e p ower o th e p h ysician , an d u n dervalu es th e in dividu al’s
social su p p ort n etwork in gettin g well.

D. Telling patients the truth


1. In th e Un ited States, adu lt p atien ts gen erally are told the complete truth abou t th e diagn osis,
the m an agem en t an d its side e ects, an d the p rogn osis o their illn ess. Falsely reassuring
or patronizing statements in respon se to patien t question s (e.g., “Do n ot worry, we will take
good care o you” or “You can get p regn an t again” [a ter a m iscarriage]) are n ot ap p rop riate.
2. In orm ation abou t th e illn ess m u st be given directly to the adult patient an d not relayed
to the patient through relatives. Parents decide i , h ow, an d wh en su ch in orm ation will b e
given to an ill child.
a. With th e patient’s permission, th e p h ysician can tell relatives th is in orm ation in con -
ju n ction with , or a ter, tellin g th e p atien t.
b. Relievin g th e ears o close relatives o a seriou sly ill p atien t can bolster th e su p p ort
system an d th u s h elp th e p atien t.

E. Special situations
RISE USMLE NEPAL

1. Patien ts m ay be afraid to ask questions abou t issu es th at are embarrassing (e.g., sexu al p rob-
lem s) or fear provoking (e.g., laboratory results). A physician should n ot try to guess what is
troublin g a p atien t; it is th e p h ysician’s respon sibility to ask about such issues in an op en -
en ded ash ion (see Section III.B.2.b.) an d address them truth ully an d ully with the patien t.
2. Ph ysician s h ave th e p rim ary resp on sibility or dealin g with adherence issues (see Section
II b elow), as well as with angry, seductive , or complaining behavior by th eir p atien ts
(Tab le 21.2). Re errals to oth er p h ysician s sh ou ld be reserved on ly or m edical an d p sy-
ch iatric p roblem s ou tside o th e treatin g p h ysician’s ran ge o exp ertise.

II. ADHERENCE
A. Patient characteristics associated with adherence
1. Adh eren ce re ers to th e exten t to wh ich a p atien t ollows th e recom m en d ation s o th e
p h ysician , su ch as takin g m edication s on sch edu le, h avin g a n eeded m edical test or su rgi-
cal p rocedu re, an d ollowin g d irection s or ch an ges in li estyle, su ch as d iet or exercise.
234 BRS Behavioral Science

t a b l e 21.2 Do’s and Do Not’s for Answering USMLE Questions Involving Common Problems
in the Physician–Patient Relationship

Problem Do Do Not

Angry patient Do acknowledge the patient’s anger. Do not take the patient’s anger personally (the
patient is probably fearful about becoming
dependent as well as of being ill).
Complaining patient: Do encourage the patient to speak to the Do not intervene in the patient’s relationship with
about another doctor other physician directly if the patient another physician unless there is a medical
complains about a relationship with reason to do so.
another physician.
Complaining patient: Do speak to your own office staff if the Do not blame the patient for problems with you or
HELP OTHERS SO THAT GOD WILL HELP YOU.

about you or your staff patient has a complaint about one your office staff.
of them.
Crying patient Do acknowledge the patient’s sadness Do not rush the patient or use patronizing statements
and quietly wait for the patient to such as “do not worry” to comfort the patient.
speak. Do not say “I understand.” The patient makes that
judgment.
Nonadherent patient: Do examine the patient’s willingness to Do not attempt to frighten the patient into adhering
needs to improve change his or her health-threatening (e.g., showing graphic photographs of untreated
health behavior behavior (e.g., smoking); if he or she illness).
is not willing, you must address that
issue first.
Nonadherent patient: Do identify the real reason (e.g., fear) for Do not refer the patient to another physician.
needs a test or the patient’s refusal to adhere to or to
treatment (e.g., consent to a needed intervention and
mammogram) address it.
Seductive patient Do call in a chaperone when you are Do not refuse to see the patient.
with the patient. Do not refer the patient to another physician.
Do gather information using direct rather Do not fail to act if the patient crosses a social
than open-ended questions. boundary.
Do set limits on the behavior that you will
tolerate.
Suicidal patient Do assess the seriousness of the threat. Do not assume that the threat is not serious.
Do suggest that the patient remain in Do not release a hospitalized patient who is a threat
the hospital voluntarily if the threat is to himself or herself (patients who are a threat
serious. to self or others can be held involuntarily [see
Chapter 23]).
RISE USMLE NEPAL

2. Patien ts n eed to recogn ize th at th eir b eh avior or con d ition (e.g., ob esity) is p rob lem atic
b e ore th ey are m otivated to ch an ge or seek m ed ical care. Th e “Stages of Change” m od el
re ers to th e p oin t at wh ich th is recogn ition an d read in ess to ch an ge occu rs (Tab le 21.3).
3. Patien ts’ unconscious tran s eren ce reaction s to th eir p h ysician s, wh ich are b ased in ch ild-
h ood p aren t–ch ild relation sh ip s, can in crease or decrease adh eren ce (see Ch ap ter 6).
4. On ly abou t one-third of patients adhere fully to management recommendations , on e-th ird
adh ere som e o th e tim e, an d on e-th ird do n ot adh ere to su ch recom m en dation s.

B. Factors that increase and decrease adherence


1. Adh eren ce is not related to patient intelligence, education, sex, religion , race, socioeco-
n om ic statu s, or m arital statu s.
2. Adh eren ce is m ost closely related to how well the patient likes the doctor. Th e stren gth
o th e doctor–p atien t relation sh ip is also th e m ost im p ortan t actor in wh eth er or n ot
p atien ts su e th eir doctors wh en a m edical error or om ission is m ade or wh en th ere is a
p oor ou tcom e (see Ch ap ter 23).
3. Som e actors associated with adh eren ce are listed in Table 21.4.
Chapter 21 The Physician–Patient Relationship 235

t a b l e 21.3 The Stages of Change Model

Name of Stage
Stage # (Readiness for Change) Patient Quote Patient Characteristics Physician Strategies

Stage 1 Precontemplation (not “My father was also a big Fails to recognize or Elicit the patient’s feelings
ready for change) heavy guy—he was a denies that there is about the problem and
longshoreman and was a problem explain the risks of the
hardly ever sick.” unwanted behavior.
Stage 2 Contemplation (getting “Maybe his weight had Is ambivalent about Weigh the pros and cons of
ready for change) something to do with my making the change making the change and
father’s death, but I don’t identify things that may be
think so.” reducing the likelihood of
HELP OTHERS SO THAT GOD WILL HELP YOU.

change.
Stage 3 Preparation (ready for “I need to lose weight; I will Trying small Prepare a plan of action for
change) try to cut back on my improvements the patient; identify social
eating.” support systems.
Stage 4 Action (makes “I ate my last pizza last Makes the needed Acknowledge the achievement.
change) night.” change
Stage 5 Maintenance “It’s really hard not to eat Continues the changed Develop strategies to manage
(continues change) too much when the behavior temptation and reward
family gets together but I success.
am doing it.”
Stage 6 Relapse (goes back on “I have been eating all the Feel guilt, anger, and Identify the factors that led
change) wrong foods all week. I disappointment to the relapse, and help
am so angry at myself.” the patient to “get back on
track.”

III. THE CLINICAL INTERVIEW


A. Communication skills
1. Patien t ad h eren ce with m ed ical ad vice, detection o both p h ysical an d p sych ological
p roblem s, an d p atien t satis action with th e p h ysician are im p roved by good p h ysician –
p atien t com m u n ication .
2. On e o th e m ost im p ortan t skills or a p h ysician to h ave is how to interview patients .
a. Th e physical setting or th e in terview sh ou ld be as p rivate as p ossible. Ideally, th ere
sh ou ld b e no desk or other obstacle between th e p h ysician an d p atien t, an d th e p artici-
p an ts sh ou ld in teract at eye level (e.g., b oth seated ).
RISE USMLE NEPAL

b. Du rin g th e in terview, th e p h ysician m u st irst establish trust in an d rapport with th e


p atien t an d th en gath er p h ysical, p sych ological, an d social in orm ation to iden ti y th e
p atien t’s p roblem .
c. Fin ally, th e p h ysician sh ou ld try to edu cate th e p atien t abou t th e illn ess an d m otivate
th e p atien t to adh ere to m an agem en t recom m en dation s.
d. Th e p h ysician sh ou ld obtain backu p (e.g., h osp ital secu rity) i it ap p ears th at a p atien t
is dangerous or threatening.
3. Th e in terview serves to ob tain th e p atien t’s psychiatric history, in clu din g in orm ation
abou t p rior m en tal p roblem s, d ru g an d alcoh ol u se, sexu al activity, cu rren t livin g situ a-
tion , an d sou rces o stress.
4. Wh en in terviewin g young children, th e p h ysician sh ou ld
a. First establish rap p ort by in teractin g with th e ch ild in a n on m edical way, or exam p le,
drawin g p ictu res.
b. Use d irect rath er th an op en -en d ed qu estion s, or exam p le, “Wh at is you r sister’s
n am e?” rath er th an “Tell m e ab ou t you r am ily.”
c. Ask qu estion s in th e th ird p erson , or exam p le, “Wh y do you th in k th at th e little boy in
th is p ictu re is sad?”
236 BRS Behavioral Science

t a b l e 21.4 Factors Associated with Adherence to Medical Advice

Factors Associated with Factors Associated with


Increased Adherence Decreased Adherence Comments

Good physician–patient Poor physician–patient Liking the physician is the most important factor in
relationship relationship adherence; it is even more important than the physician’s
technical skill.
Physicians perceived as unapproachable have low
adherence from patients.
Patient feels ill and usual Patient experiences few In asymptomatic illnesses, such as hypertension, only about
activities are disrupted by symptoms and little half of the patients initially adhere to management.
the illness disruption of usual Many asymptomatic patients who initially adhered have
HELP OTHERS SO THAT GOD WILL HELP YOU.

activities stopped adhering within 1 y of diagnosis.


Short time spent in the Long time spent in the waiting Patients kept waiting get angry and then fail to adhere.
waiting room room
Belief that the benefits Belief that financial and time The “Health Belief Model” of health care
of care outweigh its costs of care outweigh its
financial and time costs benefits
Written diagnosis and Verbal diagnosis and Patients often forget what is said during a visit to the
instructions for instructions for physician because they are anxious.
management management Asking the patient to repeat your verbal instructions can
improve understanding and thus increase adherence.
Acute illness Chronic illness Chronically ill people see physicians more often but are
more critical of them than acutely ill people.
Recommending only one Recommending multiple To increase adherence, ask the patient which change they
behavioral change at behavioral changes at would like to start with and then ask the patient to make
a time once this change (e.g., stop smoking) this month and make
another change (e.g., start dieting) next month.
Recommending too many changes at once will reduce the
likelihood that the patient will make any changes.
Simple management Complex management Adherence is higher with medications that require once
schedule schedule daily dosing, preferably with a meal.
Patients are more likely to forget to take medications
requiring frequent or between-meal dosing.
Older physician Younger physician Usually, young physician age is only an issue for patients in
the initial stages of management.
Peer support Little peer support Membership in a group of people with a similar problem
(e.g., smoking [see Chapter 9]) can increase adherence.
RISE USMLE NEPAL

B. Specific interviewing techniques


1. Direct questions. Direct qu estion s are u sed to elicit sp eci ic in orm ation qu ickly rom a
p atien t in an emergency situ ation (e.g., “Have you been sh ot?”) or wh en th e p atien t is
sedu ctive or overly talkative.
2. Open-ended questions
a. Alth ou gh direct qu estion s can elicit in orm ation qu ickly, op en -en d ed typ es o qu es-
tion s are m ore likely to aid in obtain in g in orm ation abou t th e p atien t an d n ot close o
p oten tial areas o p ertin en t in orm ation .
b. Usin g op en -en d ed qu estion s (e.g., “Wh at brin gs you in today?”), th e in terviewer gives
little stru ctu re to th e p atien t an d encourages the patient to speak freely.
3. Table 21.5 lists aim s o th e clin ical in terview an d gives exam p les o som e sp eci ic in ter-
viewin g tech n iqu es.
Chapter 21 The Physician–Patient Relationship 237

t a b l e 21.5 Aims of the Clinical Interview and Specific Interviewing Techniques


Aim Technique Specific Use Example

To establish Support and To express the physician’s interest, “You must have really been frightened when
rapport empathy understanding, and concern for you realized you were going to fall.”
the patient
Validation To give value and credence to the “Many people would feel the same way if
patient’s feelings they had been injured as you were.”
To maximize Facilitation To encourage the patient to elaborate “Please tell me more about what happened
information on an answer; can be a verbal after your fall.”
gathering question or body language, such
as a quizzical expression
HELP OTHERS SO THAT GOD WILL HELP YOU.

Reflection To encourage elaboration of the “You said that your pain increased after
answer by repeating part of the lifting the package?”
patient’s previous response
Silence To increase the patient’s Waiting silently for the patient to speak.
responsiveness
To clarify Confrontation To call the patient’s attention to “You say that you are not worried about
information inconsistencies in his or her tomorrow’s surgery, but you seem really
responses or body language upset to me.”
Recapitulation To sum up all of the information “Let’s go over what you told me. You fell last
obtained during the interview night and hurt your side. Your husband
to ensure that the physician called 911. The paramedics came but the
understands the information pain got worse until they gave you a shot
provided by the patient in the emergency room. Have I gotten it
right?”
RISE USMLE NEPAL
Review Test

Directions: Each o th e n u m b ered item s or in com p lete statem en ts in th is section is ollowed by


an swers or by com p letion s o th e statem en t. Select th e one lettered an swer or com p letion th at
is best in each case.
HELP OTHERS SO THAT GOD WILL HELP YOU.

1. A 31-year-old wom an wh o h as b een in 3. A 9-year-old girl wh o h as a term in al illn ess


a seriou s car accid en t re u ses to go on th e asks th e p h ysician , “Am I goin g to d ie?” Th e
am bu lan ce to th e h osp ital u n til h er h u sb an d ch ild’s p aren ts p reviou sly told th e p h ysician
arrives at th e scen e so th at h e can go with th at th ey do n ot wan t th e ch ild to kn ow h er
h er. Sh e n otes th at sh e h as called h im an d diagn osis or p rogn osis. Th e p h ysician’s b est
th at h e is on th e way. Her in ju ries are seriou s resp on se to th e ch ild’s qu estion is to say
bu t n ot li e th reaten in g. Most ap p rop riately, (A) “Do n ot worry, you will be in e.”
th e am bu lan ce team sh ou ld (B) “Yes, you will die o th is illn ess.”
(A) wait or th e h u sban d (C) “Tell m e wh at you r p aren ts h ave told you
(B) take th e p atien t to th e h osp ital again st abou t you r illn ess.”
h er will (D) “You r p aren ts do n ot wan t you to kn ow
(C) wait u n til th e p atien t loses abou t you r illn ess so I can n ot tell you .”
con sciou sn ess an d th en take h er to th e (E) “Man y ch ild ren with th is kin d o illn ess
h osp ital live a lon g tim e.”
(D) tell th e p atien t th at th ey can n ot wait
(E) advise th e p atien t th at sh e can b e 4. A 40-year-old m ale p atien t tells h is
p rosecu ted i sh e re u ses treatm en t p h ysician th at h e sm okes at least two p acks
o cigarettes a day b u t wou ld like to stop.
2. Over th e p ast 4 m on th s, a 16-year- Which o th e ollowin g is th e m ost e ective
old m ale h igh sch ool stu d en t h as been statem en t or qu estion th e p h ysician can u se
com p lain in g o leth argy, n au sea, an d to en cou rage th e p atien t to give u p sm okin g?
dizzin ess, wh ich h ave cau sed h im to m iss (A) “You m ust stop sm okin g because it causes
a sign i can t n u m b er o sch ool d ays. His lun g can cer an d m an y other illn esses.”
p aren ts brou gh t h im to th e p rim ary care (B) “Wh y does an in telligen t p erson like you
p h ysician or evalu ation . Ph ysical exam con tin u e to sm oke?”
an d th e resu lts o lab oratory testin g are (C) “Do you h ave an y relatives wh o died o
RISE USMLE NEPAL

u n rem arkable. Wh at is th e p h ysician’s m ost lu n g can cer?”


ap p rop riate n ext step in m an agem en t? (D) “I would like to show you a picture o what
(A) Do a toxicology screen or su bstan ce u se. lun gs look like a ter a li etim e o sm okin g.”
(B) Ask th e p atien t’s p aren ts to ob serve h im (E) “Please tell m e h ow I can h elp you to
closely over th e n ext ew weeks. stop sm okin g.”
(C) Recom m en d th at th e p atien t ch an ge
5. A 50-year-old p atien t with diabetes
sch ools as soon as p ossib le.
tells h is p h ysician th at h e an d h is wi e are
(D) Talk to the patien t in private an d say,
h avin g p roblem s in bed. Th e p h ysician’s best
“Please tell m e what you thin k is goin g on”
resp on se to th e p atien t is to say
(E) Recom m en d th at th e p atien t take an
an tid ep ressan t. (A) “Do n ot worry; sexu al p roblem s are
com m on in d iab etes.”
(B) “Please tell m e wh at you m ean by
‘p roblem s in bed.’”
(C) “I th in k th at you an d you r wi e sh ou ld
see a sex th erap ist.”
(D) “I will give you a p rescrip tion or Viagra.”
(E) “We n eed to do som e lab oratory tests to
determ in e wh at is cau sin g th e p rob lem .”
238
Chapter 21 The Physician–Patient Relationship 239

6. A 50-year-old , p oorly groom ed wom an 9. Wh en a p h ysician p rescribed f u oxetin e


h as m on th ly ap p oin tm en ts with a (Prozac) or a 35-year-old m ale p atien t,
card iologist. Th e p atien t, wh o requ en tly sh e exp lain ed th e m ajor side e ects o th e
com p lain s abou t th e o ce an d sta du rin g dru g. Fou r m on th s later, th e p atien t asks h er
th ese visits, tells th e card iologist th at on th is wheth er f u oxetin e h as an y side e ects. Th e
day, th e o ce recep tion ist (wh o is well liked p h ysician’s best resp on se is to say
by p atien ts an d sta ) was u n rien dly to h er. (A) “Th e side e ects are n ervou sn ess,
Th e p h ysician’s best resp on se is to in som n ia, an d sexu al dys u n ction .”
(A) n ot com m en t an d p roceed with th e (B) “I will h ave th e n u rse go over th e side
exam in ation e ects with you again .”
(B) ap ologize to th e p atien t an d o er to (C) “Please tell m e abou t wh at you h ave
HELP OTHERS SO THAT GOD WILL HELP YOU.

sp eak to th e recep tion ist been exp erien cin g wh ile takin g Prozac.”
(C) re er the patien t or psychiatric evaluation (D) “Wou ld you like m e to ch eck or p ossible
(D) ask a m em b er o th e o ice sta to sid e e ects?”
resch edu le th e p atien t’s ap p oin tm en t or (E) “The side e ects are m in or; do n ot worry.”
an oth er d ay
(E) in orm th e p atien t th at everyon e else
likes th e recep tion ist Questions 10 and 11

7. A 45-year-old m an , wh o was p reviou sly A 38-year-old salesm an , who previously had a


a su ccess u l b u sin essm an an d d evoted m yocardial in arction , presen ts to the physi-
h u sban d an d ath er, n ow n eglects h is work cian’s o ice or a routin e visit. On seein g the
an d am ily. At h is last visit, h e con d ed to th e physician , he an grily exclaim s, “What is the
p h ysician th at h e d rin ks a large am ou n t o m atter with th is p lace? I can n ever in d a p ark-
alcoh ol d aily. His wi e tells th e p h ysician th at in g space aroun d here an d everyon e is so dis-
h is drin kin g is ru in in g th e am ily. Wh ich o organ ized!” He then in sists on m akin g a phon e
th e ollowin g is th e m ost e ective qu estion call while the physician waits to exam in e him .
or in itiatin g a discu ssion with th e p atien t
abou t th e e ects o alcoh ol on h is am ily? 10. Most ap p rop riately, th e p h ysician sh ou ld
(A) “Do you kn ow th at m ost p atien ts wh o n ow say
drin k as m u ch as you d o even tu ally lose (A) “I can n ot exam in e you u n til you calm
th eir am ilies?” down .”
(B) “Do you eel gu ilty ab ou t wh at you r (B) “Are you always th is an gry?”
drin kin g is d oin g to you r ch ild ren ?” (C) “You seem u p set.”
(C) “Do you realize th e dam age th at you r (D) “Wou ld you like a re erral to an oth er
drin kin g is d oin g to you r m arriage?” p h ysician ?”
(D) “Wh at d o you th in k is th e im p act o you r (E) “I will resch edu le you r ap p oin tm en t or
RISE USMLE NEPAL

drin kin g on you r am ily?” an oth er d ay.”


(E) “You r wi e says you r d rin kin g is ru in in g
you r am ily. Do you agree?” 11. Th e p erson ality typ e th at best describes
th is p atien t is
8. In a hospital em ergency room , a 43-year-old
(A) h istrion ic
m ale patient shouts “I want a real doctor, not
(B) sch izoid
som e jerk in training” and then throws his urine
(C) obsessive–com p u lsive
sam ple at the resident who is exam ining him .
(D) p assive–aggressive
The rst action the resident should take is to
(E) dep en d en t
(A) alert h osp ital secu rity
(B) ask an atten d in g p h ysician to take over
th e case
(C) dem an d th at th e p atien t stop sh ou tin g
an d th rowin g th in gs
(D) say to th e p atien t “I see you are u p set,
wh at can I do?”
(E) ask th e p atien t wh y h e is u p set
(F) ign ore th e beh avior an d con tin u e to
exam in e th e p atien t
240 BRS Behavioral Science

Questions 12 and 13 16. On the day he is to receive the results o a


lun g biopsy, a patien t tells the physician that
A 28-year-old wom an p resen ts to a p h ysi- he eels n e. However, the physician n otices
cian’s o ice wearin g a low-cu t blou se. Wh en that the patien t is pale, sweaty, an d shaky.
th e p h ysician begin s to in terview h er, th e Which o the ollowin g is the m ost appropriate
wom an p u ts h er h an d on h is arm an d asks statem en t or the physician to m ake?
h im i h e is m arried. (A) “Tell m e again abou t th e p ain in you r
ch est.”
12. Th e p h ysician’s m ost ap p rop riate (B) “How do you eel?”
behavior at th is tim e is to (C) “You’ll be in e.”
(A) re u se to exam in e th e p atien t at th is tim e (D) “You look righ ten ed.”
HELP OTHERS SO THAT GOD WILL HELP YOU.

bu t give h er an oth er ap p oin tm en t (E) “Are you u p set abou t bein g in th e


(B) call in a ch ap eron e or th e in terview an d h osp ital?”
exam in ation
(C) u se on ly op en -en ded qu estion s in 17. Patien ts are m ost likely to ad h ere to
in terviewin g th e p atien t m edical advice or wh ich o th e ollowin g
(D) re er th e p atien t to a em ale p h ysician reason s?
(E) ask th e p atien t abou t h er p erson al li e (A) Th e illn ess h as ew sym p tom s.
(B) Th e p atien t likes th e p h ysician .
13. Th e p erson ality typ e th at best describes (C) Th e p h ysician is you n g.
th is p atien t is (D) Th e illn ess is ch ron ic.
(A) h istrion ic (E) Th e m an agem en t sch edu le is com p lex.
(B) sch izoid
(C) ob sessive–com p u lsive 18. A 50-year-old A rican Am erican m ale
(D) p assive–aggressive p atien t wh o is well-ed u cated h as a h ern iated
(E) dep en d en t d isc. Th e ch aracteristic o th is p atien t m ost
likely to in crease h is ad h eren ce to th e
14. A 46-year-old m an p resen ts to th e m an agem en t p lan is h is
em ergen cy dep artm en t com p lain in g o ch est (A) race
p ain . Wh ich o th e ollowin g statem en ts will (B) socioecon om ic statu s
elicit th e m ost in orm ation rom th is p atien t? (C) back p ain
(A) “Poin t to th e area o p ain in you r ch est.” (D) ed u cation
(B) “Tell m e ab ou t th e p ain in you r ch est.” (E) gen der
(C) “Tell m e abou t th e p ain .”
(D) “Have you b een to a p h ysician with in th e 19. Th e “sick role” as described by Parson s
p ast 6 m on th s?” (A) ap p lies m ain ly to low socioecon om ic
(E) “Is th ere a h istory o h eart d isease in
RISE USMLE NEPAL

grou p s
you r am ily?” (B) overvalu es p eop le’s social su p p ort
n etworks
15. A 50-year-old wom an p resen ts with a (C) in clu des lack o coop eration with h ealth
com p lain t o gastric distress. Sh e seem s care workers
agitated an d says th at sh e is a raid sh e h as (D) in clu des exem p tion rom u su al
cirrh osis o th e liver bu t th en stop s sp eakin g. resp on sibilities
Wh ich o th e ollowin g will b est en cou rage (E) ap p lies m ain ly to ch ron ic illn ess
th is p atien t to con tin u e sp eakin g?
(A) “Please go on .”
(B) “How m u ch liqu or d o you d rin k?”
(C) “Do you d rin k?”
(D) “Wh y d id you wait so lon g to com e in ?”
(E) “Th ere are m an y ways to treat
alcoh olism .”
Chapter 21 The Physician–Patient Relationship 241

Questions 20 and 21 23. Th e p aren ts o a critically ill 6-year-old


p atien t tell th e p h ysician th at wh en th e ch ild
A 34-year-old ath er o 4 ch ild ren sm okes becam e ill, h is 14-year-old broth er started to
2 p acks o cigarettes a d ay b u t b elieves th at behave badly in sch ool an d at h om e. At th is
sm okin g does n ot cau se h im h arm . In stead , tim e, th e you n ger ch ild’s p h ysician sh ou ld
h e b elieves th at sm okin g p reven ts h im rom (A) re er th e teen ager to an adolescen t
gettin g colds. p sych ologist
(B) ask to sp eak to th e teen ager alon e as
20. To en cou rage th is p atien t to sm oke less, soon as p ossib le
th e p h ysician sh ou ld rst (C) ask to sp eak to th e teen ager wh en th e
(A) recom m en d a sm okin g cessation you n ger ch ild is ou t o dan ger
HELP OTHERS SO THAT GOD WILL HELP YOU.

su p p ort grou p (D) tell th e p aren ts th at th e p atien t is th e


(B) recom m en d a n icotin e p atch you n ger ch ild, n ot th e teen ager
(C) sh ow h im p h otograp h s o th e lu n gs o (E) tell th e p aren ts to con cen trate on th e
p atien ts with lu n g can cer you n ger ch ild
(D) determ in e h ow willin g h e is to stop
sm okin g 24. Du rin g a ollow-u p visit a ter a
(E) tell h im h is ch ildren will be ath erless i m astectom y, a 39-year-old, well-groom ed,
h e con tin u es sm okin g m arried wom an seem s sad an d tells h er
su rgeon th at sh e is em barrassed to u n dress
21. Alth ou gh th e p atien t h as agreed to try to in ron t o h er h u sban d . Most ap p rop riately,
stop sm okin g, h e h as n ot m ade an y p rogress th e su rgeon sh ou ld n ow say
in 2 m on th s. Th e m ost ap p rop riate th in g or (A) “You sh ou ld n ot be em barrassed, you
th e p h ysician to say at th is tim e is still look good .”
(A) “It can be really h ard or p eop le to stop (B) “Th ere are a n u m ber o breast
sm okin g.” recon stru ction p rocedu res th at can
(B) “How m u ch are you sm okin g n ow?” im p rove you r ap p earan ce.”
(C) “Have you b een ollowin g m y (C) “Th e m ost im p ortan t th in g is n ot h ow
in stru ction s?” you look, bu t th at we cau gh t th e disease
(D) “I can n ot h elp you i you d o n ot listen to in tim e.”
m e.” (D) “Please tell m e h ow th e su rgery h as
(E) “I recom m en d th at you start takin g a ected you r relation sh ip with you r
bu p rop ion (Zyb an ).” h u sban d.”
(E) “It is n ot so bad, you still h ave on e
22. A p atien t wh ose ath er d ied o p rostate breast.”
can cer says th at h e can n ot take a p rostate-
sp eci c an tigen test becau se “th e n eedle will 25. A 30-year-old wom an visitin g a p h ysician
RISE USMLE NEPAL

leave a m ark.” Th e m ost ap p rop riate n ext or th e rst tim e sits silen tly in th e waitin g
step or th e p h ysician to take is to room with h er sts clen ch ed . Wh en asked to
(A) sp eak to th e p atien t’s wi e an d ask h er to by a n u rse, sh e re u ses to ll ou t a p erson al
con vin ce h im to h ave th e test data orm . In th e exam in in g room , sh e
(B) reassu re th e p atien t th at th e n eedle m ark says to th e doctor, “Let’s get th is over with .”
will ade with tim e Th e m ost ap p rop riate statem en t or th e
(C) sh ow th e p atien t p h otograp h s o p atien ts p h ysician to m ake at th is tim e is
with u n treated p rostate can cer (A) “I can n ot exam in e you u n til you ill ou t
(D) reassu re th e p atien t th at wh atever th e th e p erson al data orm .”
ou tcom e o th e test, h e can b e cu red (B) “Please tell m e wh y you re u sed to ill ou t
(E) ask th e p atien t to describ e h is eelin gs th e p erson al data orm .”
abou t h is ath er’s illn ess (C) “You seem to be qu ite ten se.”
(D) “Are you righ ten ed?”
(E) “Do n ot worry, I will take good care o
you .”
242 BRS Behavioral Science

26. A leth argic, 19-m on th -old Mexican 28. A 16-year-old girl h as a ch ron ic disorder
Am erican boy with a tem p eratu re o 102°F th at occasion ally requ ires an op ioid
is brou gh t to th e em ergen cy room by h is an algesic. Sh e calls th e p h ysician wh en h er
m oth er. Th e p h ysician n ds th at th e ch ild is p rescrip tion ru n s ou t 2 d ays p rior to h er
dehyd rated . Wh en th e ch ild re u ses to d rin k n al exam s. Sh e lives 2 h ou rs away rom th e
water, th e d octor o ers th e ch ild a ru it- p h ysician . Th e p atien t h as access to a local
f avored ice p op. Wh en th e ch ild takes it, th e m edical clin ic th at ren ews th e p rescrip tion
m oth er becom es p an icky an d takes th e ice wh en n eeded , bu t sh e ch ecked with th em
p op away. Sh e states th at in h er cu ltu re, on e an d th ey stated th at th eir wait tim e or a visit
n ever gives ood to a ch ild with a ever. Wh at is 3 days. Th e p h ysician sh ou ld
is th e p h ysician’s n ext step ? (A) con tact th e p h arm acy with in stru ction s
HELP OTHERS SO THAT GOD WILL HELP YOU.

(A) Exp lain th at th e ch ild n eeds reh ydration to re ill th e p rescrip tion
an d is m ore likely to eat an ice p op th an (B) recom m en d an over-th e-cou n ter
drin k water. m ed ication to treat th e p ain
(B) Follow th e m oth er’s wish es an d start an (C) con tact th e m edical clin ic an d requ est
IV to rep lace lu id s. th at th ey see th e p atien t im m ediately
(C) Call in a con su ltan t to con vin ce th e (D) ask a rien d wh o p ractices n ear th e
m oth er to allow th e ch ild to eat th e ice p atien t to p rescribe th e dru g
p op. (E) drive to th e p atien t’s h om e with a ready
(D) Exp lain to th e m oth er th at you are a p rescrip tion
licen sed p h ysician an d kn ow wh at is best
or th e ch ild. 29. A 39-year-old wom an goes to h er
(E) Exp lain to th e m oth er th at th e ch ild can p h ysician a ter discoverin g a breast m ass
die o d eh ydration . d u rin g sel -exam in ation . Two m on th s
(F) Elicit a su ggestion rom th e m oth er earlier, at h er yearly p h ysical, th e sam e
abou t h ow to b est get lu id s in to th e p h ysician h ad told th e p atien t th at all
ch ild th at its in with h er b elie s. n din gs were n orm al. Th e p atien t sch edu les
a m am m ogram an d, learn in g th at th e m ass
27. A 38-year-old p atien t asks h er p rim ary is su sp iciou s or b reast can cer, begin s to
care d octor, Dr. 1, or a re erral b ecau se won d er i th e d octor m issed n din g th e
sh e is m ovin g to a d i eren t city. Dr. 1 re ers lu m p 2 m on th s ago. Th is p atien t is m ost
th e p atien t to Dr. 2, an old m ed ical sch ool likely to le a m alp ractice su it again st th e
rien d , in th e n ew city. Wh en th e p atien t p h ysician i
goes to Dr. 2, h e n otices th at th e p atien t (A) th e biop sy in dicates th at sh e h as breast
seem s d ep ressed an d an xiou s, so h e re ers can cer
h er to Dr. 3, wh o is a p sych iatrist. Dr. 3 will (B) sh e b elieves th at sh e can get a sign i ican t
b e ou t o town or a wh ile so h e re ers th e in an cial settlem en t rom th e doctor’s
RISE USMLE NEPAL

p atien t to Dr. 4. Dr. 4 h as n o tim e to see th e in su ran ce com p an y


p atien t so h e re ers h er to Dr. 5. Eth ically, (C) sh e h as p oor com m u n ication with th e
wh ich step in th e re erral sequ en ce was doctor
least ap p rop riate? (D) a am ily m em ber in sists th at sh e su e th e
(A) Dr. 1 to Dr. 2 doctor
(B) Dr. 2 to Dr. 3 (E) sh e learn s th at th e can cer h as
(C) Dr. 3 to Dr. 4 m etastasized
(D) Dr. 4 to Dr. 5
Chapter 21 The Physician–Patient Relationship 243

30. A 22-year-old m an with sch izop h ren ia 32. A wom an an d h er 15-year-old dau gh ter
is brou gh t to th e em ergen cy dep artm en t p resen t to a p h ysician’s o ce togeth er. Wh en
rom an in p atien t p sych iatric acility a h al th e p h ysician asks wh at b rin gs th em in , th e
h ou r a ter acciden tally cu ttin g h im sel wh ile m oth er states: “I wan t you to t m y dau gh ter
slicin g bread. Th e p atien t h as a 5-cm cu t on or a d iap h ragm .” Th e m ost ap p rop riate
h is h an d , wh ich requ ires su tu rin g. He re u ses action or th e p h ysician to take at th is tim e
treatm en t or th e cu t an d states “I kn ow I is to
h ave a th ou gh t p rob lem I take m ed ication (A) ollow th e m oth er’s wish es an d it th e girl
or, b u t I n ever kn ow wh o to tru st. I wan t to or a diap h ragm
wait u n til m y p aren ts get h ere. I kn ow m y (B) ask th e m oth er wh y sh e wan ts a
h an d can get in ected i I wait too lon g, b u t diap h ragm or h er d au gh ter
HELP OTHERS SO THAT GOD WILL HELP YOU.

I don’t th in k a ew h ou rs are goin g to m ake (C) recom m en d th at th e girl see a sex


a big di eren ce.” Wh at is th e best cou rse o ed u cation cou n selor
action or th e doctor to take at th is tim e? (D) ask th e m oth er to leave an d sp eak to th e
(A) Do n ot treat an d determ in e i th e p aren ts girl alon e
are on th e way. (E) ask th e girl i th ere is som eth in g sh e
(B) Do n ot treat u n til a cou rt order is wan ts to say in p rivate
ob tain ed .
(C) Do n ot treat u n til obtain in g a p sych iatric
evalu ation .
(D) Treat becau se th e p atien t adm its h e h as
a p sych iatric disord er.
(E) Treat becau se th e p atien t h as been
p laced in a p sych iatric acility.
(F) Treat becau se th e p atien t sh ows sign s o
p aran oia an d is th u s in com p eten t.

31. A ter a sch ool b u s accid en t, a 6-year-


old boy with an in ju red leg is brou gh t to
th e em ergen cy room . Th e ch ild is awake
an d alert bu t is carryin g n o iden ti yin g
in orm ation . Th e p aram ed ics rep ort th at th e
ch ild h as n ot sp oken sin ce th e acciden t. Th e
m ost ap p rop riate rst step by th e p h ysician
is to
(A) get crayon s an d p ap er an d ask th e ch ild
to draw p ictu res with h er
RISE USMLE NEPAL

(B) say to th e ch ild “I am very sorry abou t


wh at h ap p en ed”
(C) look th e ch ild solem n ly in th e eye an d
ask h im ab ou t th e acciden t
(D) say “I you will n ot talk to m e I can n ot
locate you r m oth er”
(E) leave th e ch ild alon e so th at h e can rest
An swers an d Exp lan ation s

Typical Board Question


The answer is A. Th is 50-year-old sm oker is in the stage o chan ge kn own as precon tem plation . In
this stage, the person does n ot recogn ize or is in den ial about the n eed to stop sm okin g. His com -
m en t about the un cle who sm oked all o his li e an d died at age 95 o n atural causes dem on strates
HELP OTHERS SO THAT GOD WILL HELP YOU.

his re usal to ackn owledge the problem . In the con tem plation stage, the patien t is thin kin g about
it but is am bivalen t about m akin g the n eeded chan ge. In the preparation stage, the patien t m akes
sm all im p rovem en ts in the behavior. In the action stage, the patien t m akes the n eeded chan ge in
behavior, an d in the m ain ten an ce stage, th e patien t con tin ues the chan ged behavior. In the relapse
stage, th e patien t eels guilt, an ger, an d disapp oin tm en t at reen gagin g in the un wan ted behavior.

1. The answer is A. Wh erever p ossib le, a p atien t’s wish es con cern in g h ealth care sh ou ld be
resp ected. Sin ce th is p atien t’s in ju ries are n ot li e-th reaten in g an d h er h u sban d is on th e
way, m ost ap p rop riately, th e am bu lan ce team sh ou ld wait or th e h u sban d. Takin g h er to
th e h osp ital again st h er will, waitin g u n til sh e loses con sciou sn ess an d th en takin g h er to
th e h osp ital, or tellin g h er th at th ey can n ot wait are n ot correct. Ad visin g th e p atien t th at
sh e can be p rosecu ted i sh e re u ses treatm en t is n ot tru th u l.
2. The answer is D. The n ext step in m an agem en t is to talk to th is teen age p atien t in private an d
say, “Tell m e what you thin k is goin g on” Because n othin g abn orm al is oun d on p hysical exam
an d the results o laboratory testin g are un rem arkable, doin g a toxicology screen or talkin g
to p aren ts is n ot app rop riate. Makin g a recom m en dation , or exam p le, chan gin g schools or
takin g an tidepressan ts be ore in din g out m ore about the problem is n ot app ropriate.
3. The answer is C. While in orm ation about an illn ess is given directly to an adult patien t,
paren ts decide i , how, an d when such in orm ation will be given to an ill child. In this situation ,
the physician should in d out what the child kn ows about her illn ess by askin g her what her
paren ts have told her. False reassuran ce is as in appropriate or children as it is or adults.
4. The answer is E. Th e m ost e ective statem en t or qu estion th e p h ysician can u se to h elp
th e p atien t stop sm okin g is, “Please tell m e h ow I can h elp you to stop sm okin g.” Tryin g to
righ ten th e p atien t in to adh eren ce (e.g., tellin g h im it will cau se lu n g can cer, sh owin g h im
RISE USMLE NEPAL

p ictu res o lu n gs exp osed to cigarette sm oke, or askin g abou t relatives wh o died o lu n g
can cer) is less likely to b e e ective.
5. The answer is B. Th e p h ysician’s b est resp on se is to id en ti y th e sp eci ic p rob lem by askin g
th e p atien t wh at h e m ean s by “p rob lem s in b ed .” Th e p atien t’s p rob lem m u st b e id en ti ied
be ore testin g, treatm en t, or reassu ran ce is given .
6. The answer is B. Th e p h ysician’s b est resp on se is to ap ologize to th e p atien t an d o er
to sp eak to th e recep tion ist. Th e p h ysician is resp on sib le or d ealin g with illn ess-related
em otion al n eeds an d p roblem s o p atien ts an d sh ou ld n ot blam e th e p atien t, n o m atter
h ow u n p leasan t sh e is abou t p roblem s in teractin g with th e o ice sta . Th ere is n o reason to
re er th is p atien t or p sych iatric evalu ation .
7. The answer is D. Th e m ost e ective qu estion is th e m ost op en -en ded on e, or exam p le,
“Wh at do you th in k is th e im p act o you r drin kin g on you r am ily?” Qu estion s with im p lied
ju dgm en t su ch as “Do you kn ow m ost p atien ts wh o drin k as m u ch as you do lose th eir
am ilies?,” “Do you eel gu ilty ab ou t wh at you are doin g to you r ch ildren ?,” “Do you realize
th e d am age th at you r u se o alcoh ol is d oin g to you r m arriage?,” or “You r wi e says you r
drin kin g is ru in in g you r am ily” can cau se th e p atien t to becom e de en sive an d/ or an gry
an d, as su ch , are n ot likely to be h elp u l.

244
Chapter 21 The Physician–Patient Relationship 245

8. The answer is A. Patien ts wh o th row th in gs h ave lost th eir sel -con trol an d are th ere ore in
dan ger. Th e m ost im p ortan t th in g or th e residen t to d o in d ealin g with th is an gry p atien t
is to en su re both h er own an d h er p atien t’s sa ety. Th ere ore, sh e sh ou ld im m ediately alert
h osp ital secu rity. Ackn owledgin g th e p atien t’s an ger or askin g th e p atien t wh y h e is u p set
are step s th at can be taken a ter everyon e’s sa ety, in clu din g th at o th e p atien t, is en su red.
Dem an d in g th at an ou t-o -con trol p atien t stop sh ou tin g an d th rowin g th in gs is rarely
e ective.
9. The answer is C. Th e p h ysician’s b est resp on se is to say, “Please tell m e abou t wh at you
h ave b een exp erien cin g wh ile takin g Prozac,” an op en -en ded qu estion m ean t to en cou rage
th e p atien t to sp eak reely. It is likely th at th e p atien t is h avin g sexu al side e ects, com m on
with lu oxetin e, an d is u n com ortable abou t discu ssin g th em . It is n ot ap p rop riate to ju st
HELP OTHERS SO THAT GOD WILL HELP YOU.

rep eat th e p ossible side e ects, reassu re th e p atien t, or h ave th e n u rse do th e p h ysician’s
work by talkin g to th e p atien t.
10. The answer is C. 11. The answer is C. Be ore exam in in g th is p atien t, th e p h ysician sh ou ld
ackn owledge h is an ger by sayin g, “You seem u p set.” Wh ile directed at th e p h ysician via
th e p arkin g p roblem , th e p atien t’s an ger is m ore likely to be related to h is an xiety abou t
h avin g a seriou s illn ess. Treatin g h im in a ch ild -like way (e.g., tellin g h im th at h e can n ot
b e exam in ed u n til h e calm s d own ) will u rth er an ger h im . Th e p h ysician is resp on sible or
d ealin g with illn ess-related em otion al n eeds an d p roblem s o p atien ts. Th ere is n o reason
to re er th is p atien t to an oth er p h ysician . Th e p erson ality typ e th at best describ es th is
p atien t is ob sessive–com p u lsive. Patien ts with obsessive–com p u lsive p erson ality typ es ear
loss o con trol an d m ay in tu rn b ecom e con trollin g (e.g., h avin g th e p h ysician wait wh ile
h e m akes a p h on e call) du rin g illn ess (Tab le 21.1).
12. The answer is B. 13. The answer is A. Th e p h ysician’s m ost ap p rop riate beh avior is to call in
a ch ap eron e wh en d ealin g with th is sed u ctive p atien t. Re u sin g to treat h er, askin g abou t
h er p erson al li e, or re errin g h er to an oth er p h ysician is n ot ap p rop riate. For sed u ctive
p atien ts, closed -en ded qu estion s th at lim it resp on siven ess are o ten m ore ap p rop riate
th an op en -en d ed qu estion s. Th e p erson ality typ e th at best describ es th is p atien t is
h istrion ic. Histrion ic p atien ts are d ram atic an d, like th is p atien t, m ay beh ave in a sexu ally
in ap p rop riate ash ion d u rin g illn ess (Table 21.1).
14. The answer is C. Th e m ost op en -en d ed o th ese qu estion s, “Tell m e abou t th e p ain ,” gives
little stru ctu re to th e p atien t an d can th ere ore elicit th e m ost in orm ation .
15. The answer is A. Th e in terview tech n iqu e kn own as acilitation is u sed by th e in terviewer
to en cou rage th e p atien t to elaborate on an an swer. Th e p h rase, “Please go on ,” is a
acilitative statem en t.
RISE USMLE NEPAL

16. The answer is D. Th e p h ysician’s statem en t, “You look righ ten ed,” dem on strates th e
in terviewin g tech n iqu e o con ron tation , wh ich calls th e p atien t’s atten tion to th e
in con sisten cy in h is resp on se an d h is body lan gu age an d h elp s h im to exp ress h is ears.
17. The answer is B. Patien ts are m ost likely to adh ere to m edical advice becau se th ey like
th e p h ysician . Adh eren ce is also associated with sym p tom atic illn esses, older p h ysician s,
acu te illn esses, an d sim p le m an agem en t sch ed u les.
18. The answer is C. Th e act th at h e is exp erien cin g p ain is m ost likely to in crease th is
p atien t’s adh eren ce to th e m an agem en t p lan . Th ere is n o clear association between
adh eren ce an d race, socioecon om ic statu s, ed u cation , or gen d er.
19. The answer is D. Th e “sick role” ap p lies m ain ly to m idd le-class p atien ts with acu te p h ysical
illn esses. It in clu des th e exp ectation o care by oth ers, lack o resp on sibility or becom in g
ill, an d exem p tion rom on e’s u su al resp on sibilities. It u n dervalu es social su p p ort
n etworks.
20. The answer is D. 21. The answer is A. In order to get this patient to sm oke less, the physician
should irst determ in e how willin g he is to stop sm oking. A support group or m edication such
246 BRS Behavioral Science

as bupropion is only use ul or m otivated patients. This patient is not m otivated. In act, he
believes that sm okin g helps him avoid colds. Scaring patients about the consequences o their
behavior is n ot appropriate or e ective in gain in g adherence. The best thing or the physician
to say a ter the patien t has tried but n ot succeeded at stopping sm oking is a statem ent that
ackn owledges the di iculty o the task the patien t aces. Thus, the interview technique
o validation , or exam ple, “It can be really hard or people to stop sm oking,” is the m ost
appropriate statem en t to m ake at this tim e. Criticizing the patient’s behavior or threatening
to abandon the patient is not appropriate. A orm er sm okers’ support group can be a use ul
adjun ct to the physician’s program , but ackn owledging the di iculty o the task is m ore
im portan t at this tim e.
22. The answer is E. Th e p h ysician’s m ost ap p rop riate beh avior with th is p atien t wh o re u ses a
HELP OTHERS SO THAT GOD WILL HELP YOU.

n eed ed test is to determ in e th e b asis o h is re u sal—p robably h is eelin gs abou t h is ath er’s
atal illn ess. Th e reason h e re u ses to h ave th e test p robably h as little to do with th e m ark
it will leave. Tellin g h im th at h e can b e cu red is p atron izin g, in ap p rop riate, an d p ossibly
u n tru e. Sp eakin g to h is wi e also is n ot ap p rop riate; p h ysician s sh ou ld deal d irectly with
p atien ts wh en ever p ossible. Tryin g to righ ten p atien ts abou t th e con sequ en ces o th eir
b eh avior is n ot ap p rop riate or e ective in gain in g adh eren ce.
23. The answer is B. Th e you n ger ch ild’s p h ysician sh ou ld sp eak to th e teen ager alon e as
soon as p ossib le to p rovid e in orm ation an d relieve h is ears. Th is teen ager is likely to be
righ ten ed abou t h is siblin g’s illn ess an d th e ch an ged beh avior o h is p aren ts. Adolescen ts
o ten “act ou t” wh en ear u l or d ep ressed (see Ch ap ter 6). It is th e p h ysician’s role to d eal
with p rob lem s in th e 6-year-old p atien t’s su p p ort system to redu ce stress an d th u s h elp in
recovery. Th ere is u su ally n o n eed to re er am ily m em bers to m en tal h ealth p ro ession als.
Waitin g u n til th e you n ger ch ild is ou t o dan ger will n eedlessly p rolon g th e older ch ild’s
p roblem an d u rth er stress th e am ily.
24. The answer is D. Be ore o erin g su ggestion s (e.g., “Th ere are a n u m ber o breast
recon stru ction p rocedu res th at can im p rove you r ap p earan ce”), th e p h ysician sh ou ld
try to ad dress a con cern th at m an y p atien ts h ave a ter u n d ergoin g d is igu rin g su rgery
su ch as th is, or exam p le, em barrassm en t abou t u n dressin g in ron t o h er h u sban d. Th e
p h ysician sh ou ld also avoid alsely reassu rin g or p atron izin g statem en ts su ch as, “You still
look good ,” “You still h ave on e breast,” or “Th e m ost im p ortan t th in g is th at we cau gh t th e
d isease in tim e.”
25. The answer is C. Th e m ost ap p rop riate statem en t or th e p h ysician to m ake at th is tim e
to th is wom an is to ackn owledge wh at sh e seem s to be eelin g by sayin g, “You seem to be
qu ite ten se,” sin ce sh e seem s m ore ten se an d an gry th an righ ten ed. Askin g h er wh y sh e
re u sed to ill ou t a p erson al data orm or in sistin g th at sh e do so is likely to m ake h er m ore
RISE USMLE NEPAL

ten se an d an gry. Falsely reassu rin g statem en ts (see also Qu estion 24) su ch as “Th ere is
n oth in g to worry ab ou t” are p atron izin g as well as n on p rodu ctive.
26. The answer is F. I p ossible, a p h ysician sh ou ld try to work with in a p atien t’s cu ltu ral belie
system . Th u s, th is p h ysician’s n ext step in d ealin g with th is case in volvin g a deh ydrated
tod d ler is to ask th e m oth er to su ggest a m ean s o gettin g lu id in to th e ch ild th at its in
with h er cu ltu ral belie system . Startin g an IV is n ot n ecessary b ecau se th e ch ild seem s
ready to take th e ice p op by m ou th . Callin g in a con su ltan t, statin g th at you kn ow wh at
is best, or warn in g o th e worst p ossible ou tcom e will n ot oster adh eren ce or a good
p h ysician –p atien t relation sh ip.
27. The answer is B. Th e step in th e re erral sequ en ce, wh ich was least ap p rop riate, is wh en
Dr. 2 re erred th e p atien t to Dr. 3 becau se th e p atien t seem ed dep ressed an d an xiou s.
Doctor 2 sh ou ld h ave care u lly in vestigated th e p ossible m edical/ p h arm acologic cau ses o
th e p atien t’s beh avioral sym p tom s be ore decidin g on a cou rse o action . A p rim ary care
p h ysician is exp ected to address su ch beh avioral sym p tom s an d, becau se th e p atien t is
dep ressed, evalu ate su icide risk. Re errals can be in dicated wh en p atien ts ask or a re erral
(e.g., i th ey are m ovin g ou t o town ) or i th e p h ysician will n ot be available (e.g., h e or sh e
h as a u ll sch edu le).
Chapter 21 The Physician–Patient Relationship 247

28. The answer is C. Th e m ost ap p rop riate action or th e p h ysician to take is to con tact th e
local m edical clin ic, exp lain th e situ ation , an d ask th em to see th e p atien t im m ediately.
Wh ile it is n ot requ ired th at th e p h ysician drive to th e p atien t’s h om e, th e p atien t m u st
be evalu ated by a p h ysician b e ore an op ioid p rescrip tion is re illed. So, con tactin g th e
p h arm acy is n ot ap p rop riate. Patien ts with p ain severe en ou gh to requ ire op ioids are
u n likely to resp on d to over-th e-cou n ter p ain m edication .
29. The answer is C. Th is p atien t is m ost likely to ile a m alp ractice su it i sh e h as p oor
com m u n ication with th e doctor. Th e severity o th e illn ess, th e p ossible in an cial rewards
o a lawsu it, an d p ressu re rom am ily m em bers to su e are u n likely to lead to a lawsu it
wh en a p atien t likes th e doctor.
HELP OTHERS SO THAT GOD WILL HELP YOU.

30. The answer is A. Do n ot treat an d determ in e i th e p aren ts are on th e way. Th is p atien t


ap p aren tly u n d erstan d s th e risk o waitin g bu t h as elected to wait or h is p aren ts. Un less
a p atien t is obviou sly in com p eten t becau se o cu rren t p sych otic or su icidal th in kin g, or in
im m in en t d an ger, a doctor sh ou ld ollow th e p atien t’s wish es. Havin g a p sych iatric illn ess,
bein g in a p sych iatric acility, or h avin g p aran oid th ou gh ts does n ot m ake th is p atien t
in com p eten t to m ake h ealth care decision s or h im sel (an d see Ch ap ter 23).
31. The answer is A. Th e p h ysician sh ou ld irst p lay with th e ch ild to bu ild rap p ort. A ter th e
ch ild is m ore com ortab le, th e d octor can ask h im d irect (n ot op en -en d ed ) qu estion s ab ou t
th e acciden t. Expressin g sym p ath y or coercin g th e ch ild to talk will be stress u l as well as
in e ective.
32. The answer is D. For issu es in volvin g sexu ality, th e p h ysician sh ou ld irst ask to sp eak to
th e teen ager alon e. Th e teen sh ou ld n ot be qu estion ed in ron t o h er m oth er abou t h er
(th e girl’s) n eed or sp eakin g p rivately with th e p h ysician . Th e p h ysician does n ot h ave
to ollow th e m other’s wish es or ask th e m oth er qu estion s. Th e dau gh ter is th e p atien t.
Re errin g th e teen is n ot ap p rop riate; th e p h ysician can deal with th is situ ation .
RISE USMLE NEPAL
c ha pte r
22 Psych osom atic Medicin e
HELP OTHERS SO THAT GOD WILL HELP YOU.

Typical Board Question


A 68-year-old p hysician with n o h istory o p sych iatric problem s reports that over the past
3 m on ths she has been havin g di iculty sleepin g through the n ight an d has lost her appetite or
ood. She states that i she had been a better doctor, som e o her patien ts would n ot have died,
an d sh e exp resses stron g n egative eelin gs abou t n ot spen din g m ore tim e with her children
when they were youn g. The p atien t also rep orts th at she has lost 25 poun ds sin ce the previous
year without dietin g. The m ost appropriate n ext step in the m an agem en t o this patien t is
(A) an an tid ep ressan t
(B) an an tip sych otic
(C) p sych oth erapy
(D) to reassu re h er th at wh at sh e is eelin g is p art o th e typ ical agin g p rocess
(E) to sch ed u le a m edical evalu ation
(See “An sw ers an d Explan ation s” at th e en d of th e ch apter.)

I. STRESS AND HEALTH


A. Psychological factors affecting health. Psych ological actors m ay in itiate or exacerbate sym p -
tom s o m ed ical d isord ers (psychosomatic symptoms ) in volvin g alm ost all body system s.
RISE USMLE NEPAL

Th ese actors in clu de:


1. Poor health behavior (e.g., sm okin g, ailu re to exercise).
2. Maladaptive personality style (e.g., obsessive–com pulsive person ality type) (see Chapter 21).
3. Chronic or acute life stress cau sed by em otion al (e.g., dep ression ), social (e.g., divorce), or
econ om ic (e.g., job loss) di icu lties.

B. Mechanisms of the physiologic effects of stress


1. Acu te or ch ron ic li e stress leads to activation of the autonomic nervous system, wh ich in
tu rn a ects card iovascu lar an d resp iratory system s.
2. Stress also leads to altered levels of neurotransmitters (e.g., seroton in , n orep in ep h rin e),
wh ich resu lt in ch an ges in m ood an d b eh avior.
3. Stress can in crease th e release of adrenocorticotropic hormone (ACTH), wh ich leads to th e
release o cortisol, u ltim ately resu ltin g in depression of the immune system as m easu red
by d ecreased lym p h ocyte resp on se to m itogen s an d an tigen s an d im p aired u n ction o
n atu ral killer cells.

248
Chapter 22 Psychosomatic Medicine 249

t a b l e 22.1 Magnitude of Stress Associated with Selected Life Events According to the
Holmes and Rahe Social Readjustment Scale
Relative Stressfulness Life Event (Exact Point Value of Stressor)

Very high Death of a spouse (100)


Divorce (73)
Marital separation (65)
Death of a close family member (63)
High Major personal loss of health because of illness or injury (53)
Marriage (50)
J ob loss (47)
Retirement (45)
HELP OTHERS SO THAT GOD WILL HELP YOU.

Major loss of health of a close family member (44)


Birth or adoption of a child (39)
Moderate Assuming major debt (e.g., taking out a mortgage) (31)
Promotion or demotion at work (29)
Child leaving home (29)
Low Changing residence (20)
Vacation (15)
Major holiday (12)

C. Stressful life events. High levels o stress in a p atien t’s li e m ay be related to an increased like-
lihood of medical an d psychiatric illness .
1. Th e Social Readjustment Rating Scale (SRRS) by Holmes and Rahe (wh ich also in clu d es
“p ositive” even ts like h olid ays) ran ks th e e ects o li e even ts (Table 22.1). Even ts with th e
h igh est scores requ ire p eop le to m ake th e m ost social readju stm en t in th eir lives.
2. Th e n eed or social readju stm en t is directly correlated with in creased risk o m edical an d
p sych iatric illn ess; 80% o p atien ts with an SRRS score of 300 points in a given year becam e
ill du rin g th e n ext year.

D. Other psychosomatic relationships


1. Medical conditions n ot u n com m on ly p resen t with p sych iatric sym p tom s.
a. Con dition s th at p resen t with depression or anxiety in clu d e neurologic illnesses (e.g.,
d em en tia), neoplasms (p articu larly p an creatic or oth er gastroin testin al can cers), endo-
crine and enzyme disturbances (e.g., h yp oth yroidism , d iabetes) (see Ch ap ter 5), an d
viral illnesses (e.g., AIDS) (see Table 12.4), blood abn orm alities, h yp oglycem ia, an d
adren al glan d tu m or (p h eoch rom ocytom a).
RISE USMLE NEPAL

b. Conditions that presen t with personality changes or psychotic symptoms include neu-
rological illn esses (e.g., tem poral lobe epilepsy), tertiary syphilis , Wilson’s disease, acute
intermittent porphyria (see Chapter 5), connective tissue disorders (e.g., system ic lupus ery-
thematosus), Huntington’s disease, AIDS (see later), Cushing’s disease, and m ultiple sclerosis.
2. Nonpsychotropic medications can produce psychiatric symptoms su ch as con u sion (e.g.,
an tiasth m atics), an xiety an d agitation (e.g., an tip arkin son agen ts), dep ression (e.g., an ti-
h yp erten sives), sedation (e.g., an tih istam in es), an d p sych otic sym p tom s (e.g., an algesics,
an tibiotics, an tih istam in es, steroid h orm on es).
3. Med ical con d ition s su ch as diabetes an d m ed ication s su ch as an tih yp erten sives also
com m on ly p rod u ce sexual symptoms su ch as erectile d ys u n ction (see Ch ap ter 19).
Th ese sym p tom s in tu rn can lead to d ep ression or oth er p sych iatric d i icu lties in
p atien ts.
4. Vitamin and mineral toxicity. Alth ou gh th ere is little empirical evidence , m an y Am erican s
b elieve th at vitam in an d m in eral su p p lem en ts en h an ce cogn itive an d em otion al u n c-
tion in g. Becau se o th ese belie s, Am erican s are likely to take excessive am ou n ts o
vitam in s an d m in erals leadin g to beh avioral sym p tom s related to vitam in an d m in eral
toxicity (Tab le 22.2) rath er th an to de icien cy.
250 BRS Behavioral Science

t a b l e 22.2 Selected Vitamins and Minerals: Neuropsychological Symptoms of Toxicity


and Their Management
Vitamin or Mineral Symptoms of Toxicity Management of Toxicity

Vitamin A Increased intracranial pressure leading Limit further use of supplements and
to headache, altered mental status, foods containing vitamin A
neurological deficits
Vitamin B6 (pyridoxine) Depression, peripheral neuropathy Limit further use of supplements and
foods containing vitamin B6
Vitamin B12 (cobalamin) Tingling and burning sensations in the Limit further use of supplements and
extremities foods containing vitamin B12
Vitamin D (pyridoxine) Confusion, apathy, poor appetite, thirst IV hydration and corticosteroids or
HELP OTHERS SO THAT GOD WILL HELP YOU.

bisphosphonate to reduce serum


calcium levels
Iron (found in prenatal vitamins) Confusion, seizures, sedation, vomiting Give deferoxamine mesylate
“coffee grounds”
Lead (found in some paints) Depression, cognitive deficits, Give ethylenediaminetetraacetic acid
hyperactivity, aggression (EDTA)
Copper Inappropriate or psychotic behavior Give d -penicillamine
Zinc (found in denture creams) Nerve damage leading to feelings of Give EDTA
burning, numbness, and weakness

II. PSYCHOLOGICAL STRESS IN SPECIFIC PATIENT POPULATIONS


A. Overview
1. Not u n com m on ly, m edical an d su rgical p atien ts h ave con cu rren t p sych ological p rob-
lem s. Th ese d i icu lties cau se p sych ological stress, wh ich can exacerbate th e p atien t’s
p h ysical disorder.
2. Usu ally, th e treatin g p h ysician h an d les th ese p rob lem s by h elp in g to organize the patient’s
social support systems an d by u sin g sp eci ic p sych oactive m edication s.
3. For severe p sych iatric p rob lem s (e.g., p sych otic sym p tom s) in h osp italized p atien ts,
consultation–liaison (CL) psychiatrists m ay b e n eed ed.

B. Hospitalized patients
1. Com m on p sych ological com p lain ts in h osp italized p atien ts in clu de an xiety, sleep disor-
ders, an d d isorien tation , o ten as a resu lt o delirium (see Ch ap ter 14) an d depression.
RISE USMLE NEPAL

2. Patien ts wh o are at th e greatest risk or su ch p roblem s in clu de th ose u n dergoin g su rgery,


or ren al d ialysis, or th ose bein g treated in th e in ten sive care u n it (ICU) or coron ary care
u n it (CCU) (e.g., “ICU p sych osis”); in all grou p s, elderly patients are at in creased risk.
3. Patien ts u n dergoin g su rgery wh o are at greatest p sych ological an d m edical risk are th ose
wh o believe that they will not survive su rgery as well as th ose wh o do not admit that they are
worried b e ore su rgery.
4. Psych ological an d m ed ical risk can be red u ced by enhancing sensory an d social input (e.g.,
p lacin g th e p atien t’s bed n ear a win d ow, en cou ragin g th e p atien t to talk), p rovidin g in or-
m ation on wh at th e p atien t can exp ect du rin g an d a ter a p rocedu re, an d allowin g th e
p atien t to con trol th e en viron m en t (e.g., ligh tin g, p ain m edication ) as m u ch as p ossib le.

C. Patients undergoing renal dialysis


1. Patien ts on ren al dialysis are at in creased risk or p sych ological p roblem s (e.g., dep res-
sion , su icid e, an d sexu al d ys u n ction ) in p art b ecau se th eir lives depend on other people
an d on technology.
2. Psych ological an d m ed ical risk can b e reduced th rou gh th e u se o in-home dialysis units ,
wh ich cau se less disru p tion o th e p atien t’s li e.
Chapter 22 Psychosomatic Medicine 251

D. Patients with sensory deficits


1. Patien ts with sen sory d e icits su ch as blindness or deafness are also at in creased p sych o-
logical risk in p art becau se th ey can becom e m ore easily disorien ted wh en ill.
2. En cou ragin g an d accom m od atin g su ch p atien ts in th e u se o th eir su p p ort tech n ology or
strategy, or exam p le, th eir seein g-eye dog, can in crease a p atien t’s sen se o con trol an d
th u s redu ce h is or h er stress du rin g illn ess.

III. PATIENTS WITH CHRONIC PAIN


HELP OTHERS SO THAT GOD WILL HELP YOU.

A. Psychosocial factors
1. Chron ic pain (pain lastin g at least 6 m on ths) is a commonly encountered complaint o patien ts.
It m ay be associated with p hysical actors, psychological actors, or a com bin ation o both.
a. Decreased tolerance for pain is associated with d ep ression , an xiety, an d li e stress in
adu lth ood an d p h ysical an d sexu al ab u se in ch ild h ood.
b. Pain tolerance can b e in creased th rou gh b io eedb ack, p h ysical th erapy, h yp n osis, p sy-
ch oth erapy, m editation , an d relaxation train in g.
2. Ch ron ic p ain o ten lead s to a loss of independence , wh ich can lead to dep ression . Practical
su ggestion s or sel -care in addition to p h arm acologic m an agem en t can be h elp u l or
su ch p atien ts.
3. Dep ression m ay p red isp ose a p erson to develop ch ron ic p ain . More com m on ly, chronic
pain results in depression.
4. Peop le wh o exp erien ce p ain a ter a m edical or su rgical p rocedu re h ave a higher risk of
morbidity and mortality an d a slower recovery rom th e p roced u re.
5. Religiou s, cu ltu ral, an d eth n ic actors m ay in lu en ce th e p atien t’s exp ression o p ain an d
th e resp on ses o th e patient’s support systems to th e p ain (see Ch ap ter 18).

B. Managing pain
1. Pain relief in p ain cau sed by p h ysical illn ess is best ach ieved by analgesics (e.g., op ioids),
u sin g patient-controlled analgesia (PCA) or n erve-blockin g su rgical p rocedu res.
2. Antidepressants , are u se u l in th e m an agem en t o ch ron ic p ain .
a. An tidep ressan ts are m ost u se u l or p atien ts with arthritis, facial pain, an d headache .
b. Th eir an algesic e ect m ay be th e resu lt o stimulation of efferent inhibitory pain pathways .
c. Alth ou gh th ey h ave d irect an algesic e ects, an tidep ressan ts m ay also decrease p ain
in directly by improving mood symptoms .
3. Accord in g to th e gate control theory, th e p ercep tion o p ain can be blocked by electric
RISE USMLE NEPAL

stim u lation o large-d iam eter a eren t n erves. Som e p atien ts are h elp ed by th is treatm en t.
4. Patien ts with p ain cau sed by p h ysical illn ess also b en e it rom behavioral, cognitive , an d
other psychological therapies (see Ch ap ter 17), by n eed in g less p ain m edication , b ecom -
in g m ore active, an d sh owin g in creased attem p ts to retu rn to a n orm al li estyle.

C. Programs of pain management


1. Scheduled administration of an analgesic be ore th e p atien t requ ests it (e.g., every 3 h ou rs)
an d PCA are m ore e ective th an m ed ication ad m in istered wh en th e p atien t requ ests it
(on dem an d). Sch ed u led ad m in istration sep arates th e exp erien ce o p ain rom th e receip t
o m ed ication .
2. Man y patients with chronic pain are undermedicated becau se th e p h ysician ears th at th e
p atien t will b ecom e ad dicted to op ioid s. In act, p ain p atien ts are at higher risk for depres-
sion th an th ey are or d ru g ad d iction .

D. Pain in children
1. Ch ild ren eel p ain an d rem em b er p ain as m u ch as adu lts d o.
2. Becau se ch ild ren are a raid o in jection s, th e m ost u se u l ways o ad m in isterin g p ain m ed-
ication s to th em are orally (e.g., a en tan yl “lollip op”), transdermally (e.g., a skin cream to
p reven t p ain rom in jection s or sp in al tap s), or, in older ch ildren an d adolescen ts, via PCA.
252 BRS Behavioral Science

IV. PATIENTS WITH ACQUIRED IMMUNE DEFICIENCY SYNDROME


A. Psychological stressors. Acqu ired im m u n e d e icien cy syn drom e (AIDS) an d HIV-p ositive
p atien ts m u st d eal with p articu lar p sych ological stressors n ot seen togeth er in oth er
disord ers.
1. Th ese stressors in clu de h avin g a life-limiting illness, eelin g guilty ab ou t h ow th ey con -
tracted th e illn ess (e.g., sex with m u ltip le p artn ers, in traven ou s d ru g u se) an d ab ou t
p ossib ly in ectin g oth ers, an d b ein g m et with fear of contagion rom m edical p erson n el,
am ily, an d rien ds.
2. HIV-positive p atien ts wh o h ave a homosexual orientation m ay be com p elled (becau se o
HELP OTHERS SO THAT GOD WILL HELP YOU.

th eir illn ess) to “come out” (i.e., reveal th eir sexu al orien tation ) to oth ers.
3. Medical an d p sych ological counseling can redu ce m edical an d p sych ological risk or HIV-
p ositive p atien ts.
4. It is im p ortan t to n ote th at p sych iatric sym p tom s su ch as dep ression or p sych osis in AIDS
p atien ts m ay also resu lt rom infection of the brain with HIV or with an op p ortu n istic in ec-
tion su ch as cryp tococcal m en in gitis (see Ch ap ter 14) or cerebral lym p h om a.

B. Contagion
1. I th ey com p ly with accep ted m eth ods o in ection con trol, HIV-positive physicians do not
risk transmitting the virus to their patients .
2. Few health care workers h ave con tracted HIV rom p atien ts. Th e m ain risk o tran sm ission
is th rou gh acciden tal con tam in ation rom n eedles an d oth er sh arp s, alth ou gh th is risk is
very low (see Table 19.6).
3. Un d er certain circu m stan ces, p h ysician s can iden ti y th eir HIV-p ositive p atien ts to th ose
th ey p u t at im m in en t risk (e.g., sexu al p artn ers) (see Ch ap ter 23).
RISE USMLE NEPAL
Review Test

Directions: Each o th e n u m b ered item s or in com p lete statem en ts in th is section is ollowed by


an swers or by com p letion s o th e statem en t. Select th e one lettered an swer or com p letion th at
is best in each case.
HELP OTHERS SO THAT GOD WILL HELP YOU.

1. A 45-year-old m an h as a rou tin e yearly 3. Psych ological stress en gen dered by th is


p h ysical exam in ation . Wh ile takin g p atien t’s p ain is m ost likely to resu lt in
th e h istory, th e doctor determ in es th at in creased
du rin g th e last year, th e p atien t took ou t (A) lym p h ocyte resp on se to m itogen s
a su bstan tial m ortgage an d m oved to a (B) release o adren ocorticotrop ic h orm on e
n ew h ou se. Du rin g th e m ove, h e ell an d (ACTH)
su stain ed a h ead in ju ry, wh ich requ ired (C) u n ction o n atu ral killer cells
h osp italization . Wh ile recu p eratin g, h e (D) u n ction o th e im m u n e system
an d h is wi e o 10 years wen t on a 2-week (E) cortisol su p p ression
vacation . He recovered com p letely, b u t
a ter th e vacation , th e cou p le sep arated. 4. In th e Un ited States, th e n u m ber o
Accordin g to th e Holm es an d Rah e scale, p atien ts con rm ed to h ave con tracted HIV
which o th ese social exp erien ces p u ts th is rom th eir p h ysician s is
m an at th e h igh est risk or p h ysical illn ess in
(A) ewer th an 50
th e n ext year?
(B) between 51 an d 100
(A) Exp erien cin g m arital sep aration (C) between 101 an d 200
(B) Takin g ou t a large m ortgage (D) between 201 an d 300
(C) Ch an gin g residen ce (E) m ore th an 300
(D) Su stain in g a seriou s in ju ry
(E) Goin g on vacation Questions 5 and 6

Questions 2 and 3 A 65-year-old m ale p atien t is sch edu led or


card iac su rgery. A ter th e su rgery, h e will be
A 35-year-old wom an with a h ern iated d isc in th e in ten sive care u n it (ICU) or abou t
h as h ad b ack p ain or th e p ast 2 years. To h elp 24 h ou rs an d will requ ire a m ech an ical
con trol h er p ain , sh e takes an op ioid -b ased ven tilator.
RISE USMLE NEPAL

m ed ication daily.
5. To redu ce th is p atien t’s likelih ood o
2. Wh ich o th e ollowin g is m ost likely to be p sych ological p roblem s in th e ICU, th e
true abou t th is p atien t? p h ysician sh ou ld
(A) Sh e is at h igh risk or dru g addiction . (A) lim it visits rom h is am ily
(B) Psych o lo gica l th era p ies will n o t (B) redu ce h is exp osu re to am bien t ligh t
b en e it h er. (C) exp lain th e n eed or an d u n ction o th e
(C) Her exp ression o p ain is related m ech an ical ven tilator
exclu sively to th e exten t o h er p ain . (D) discou rage com m u n ication b etween th e
(D) Sh e is at h igh risk or dep ression . p atien t an d th e sta
(E) Sh e is receivin g too m u ch p ain (E) h ave th e n u rses con trol th e p atien t’s
m ed ication . ligh tin g level

253
254 BRS Behavioral Science

6. Du rin g h is stay in th e ICU a ter su rgery, 9. A 40-year-old wom an com es to th e


th is p atien t is m ost likely to exp erien ce doctor with th e com p lain t th at sh e is
which o th e ollowin g disorders? tired all th e tim e. Sh e also n otes th at h er
(A) Pan ic disorder sp eakin g voice is ch an gin g an d seem s lower
(B) Obsessive–com p u lsive disord er th an be ore. Th yroid u n ction tests reveal
(C) Illn ess an xiety disorder in creased th yroid-stim u latin g h orm on e
(D) Som atic sym p tom disorder (TSH) an d d ecreased T3 an d ree T4. Ph ysical
(E) Deliriu m exam in ation reveals delayed relaxation
p h ase o th e m u scle stren gth ref ex, an d
7. A 25-year-old wom an with n o p sych iatric a bon e scan reveals eviden ce o early
h istory h as a rap id h eart rate an d eelin gs o osteop orosis. Th is p atien t is m ost likely to
HELP OTHERS SO THAT GOD WILL HELP YOU.

an xiety, wh ich h ave been p resen t or th e p ast also sh ow wh ich o th e ollowin g p sych iatric
3 m on th s. Th e p atien t h as lost 15 p ou n ds sym p tom s?
an d rep orts th at sh e d oes n ot sleep well. (A) Hallu cin ation s an d d elu sion s
Ph ysical exam in ation reveals exop h th alm os (B) Dep ression
(bulgin g eyes) an d a n eck m ass. Th e m ost (C) Obsession s an d com p u lsion s
ap p rop riate n ext step in th e m an agem en t o (D) Pan ic attacks
th is p atien t is (E) Som atic sym p tom disorder
(A) an an tid ep ressan t
10. A 30-year-old blin d p atien t with a
(B) an an tip sych otic
seein g-eye dog com es to a p h ysician or th e
(C) a b en zod iazep in e
rst tim e. Wh en it is tim e or th e p atien t to
(D) p sych oth erapy
go in to th e exam in in g room , th e p h ysician
(E) a m edical evalu ation
sh ould m ost ap p rop riately
8. Up on retu rn in g rom a week-lon g (A) allow th e p atien t to b e led in to th e
vacation in th e Carib b ean , a 25-year-old exam in ation room by th e d og
A rican -Am erican wom an with n o h istory (B) take th e p atien t by th e arm an d lead h er
o p sych iatric sym p tom s seem s agitated in to th e exam in in g room with th e dog
an d an xiou s. Wh en sh e tells h er sister ollowin g
th at a television n ewscaster is p u blically (C) ask a m em b er o th e o ice sta to care
discu ssin g h er beh avior, th e sister b rin gs h er or th e dog du rin g th e exam in ation
to th e em ergen cy dep artm en t. Th e p atien t (D) tell th e p atien t th at th e dog will h ave
su bsequ en tly is ad m itted to th e h osp ital to stay in th e waitin g room du rin g th e
with ever, atigu e, join t p ain , an d a rash exam in ation or reason s o san itation
across th e b rid ge o h er n ose. Hem atologic (E) m ake an oth er ap p oin tm en t or th e
n din gs in clu de m ild an em ia an d p resen ce p atien t wh en sh e can com e in with ou t
o an tin u clear an tib odies (ANA). O th e th e dog
RISE USMLE NEPAL

ollowin g, th e m ost likely exp lan ation o th e


p atien t’s beh avioral sym p tom s is
(A) dissociative am n esia with d issociative
u gu e
(B) gen eralized an xiety disorder
(C) brie p sych otic disorder
(D) system ic lu p u s eryth em atosu s (SLE)
(E) som atic sym p tom disorder
Chapter 22 Psychosomatic Medicine 255

11. A 45-year-old wom an with rh eu m atoid 12. A ter a 5-year-old ch ild takes h is m oth er’s
arthritis calls h er p h ysician on a Mon day p ren atal vitam in s, h e th rows u p m aterial
m orn in g becau se sh e can n ot tu rn on th e th at looks like co ee grou n ds an d th en loses
b ath tu b au cet b ecau se o th e p ain in h er con sciou sn ess. In th e em ergen cy room , th e
h an d s an d wrists. Sh e is tear u l an d says, “My p h ysician’s n ext step in p h arm acological
h u sban d h as alread y le t or work an d m y m an agem en t is to give th is ch ild
h an d s h u rt too m u ch to tu rn th e water on . (A) d -p en icillam in e
Now I can’t even take a b ath .” Wh ich o th e (B) eth ylen ed iam in etetraacetic acid (EDTA)
ollowin g is th e d octor’s m ost ap p rop riate (C) bisp h osp h on ate
resp on se? (D) de eroxam in e m esylate
(A) “I sym p ath ize with you . Un ortu n ately, (E) lu m azen il
HELP OTHERS SO THAT GOD WILL HELP YOU.

it looks like you r on ly op tion is to wait


u n til you r h u sb an d com es h om e.”
(B) “Peop le with rh eu m atoid arth ritis o ten
eel th at th eir in dep en den ce h as been
lost becau se o th eir p ain . Perh ap s u sin g
a tool like a wren ch with a lon g h an d le
will h elp.”
(C) “Com e to th e o ice righ t away an d I will
give you a cortison e sh ot.”
(D) “I kn ow th is m u st b e d i icu lt or you ,
p erh ap s you can call a p lu m b er.”
(E) “Man y p eop le with rh eu m atoid arth ritis
h ave di icu lty tu rn in g au cets. Perh ap s
you wou ld b e in terested in join in g an
arth ritis su p p ort grou p.”
RISE USMLE NEPAL
An swers an d Exp lan ation s

Typical Board Question


The answer is E. Th e m ost ap p rop riate n ext step in th e m an agem en t o th is p atien t, wh o h as
n o p reviou s h istory o p sych iatric illn ess, is to sch edu le a m edical evalu ation . Th is p atien t h as
sym p tom s o d ep ression in clu din g sleep p rob lem s, in ap p rop riate gu ilt, su icidal ideation , an d
HELP OTHERS SO THAT GOD WILL HELP YOU.

sign i ican t weigh t loss. Dep ression is n ot typ ically seen in n orm al agin g. Becau se p an creatic can -
cer an d oth er gastroin testin al can cers as well as diabetes an d h yp oth yroidism n ot u n com m on ly
p resen t with d ep ression in th e eld erly, th is p atien t sh ou ld be evalu ated or su ch con dition s p rior
to treatin g h er dep ression . Psych oth erapy, an tid ep ressan ts, an d an tip sych otics do n ot address
th is p atien t’s u n derlyin g p h ysical p roblem (see also Qu estion 7).

1. The answer is A. Accord in g to th e Holm es an d Rah e scale, m arital sep aration p u ts th is m an


at th e h igh est risk or p h ysical illn ess th is year. Th e oth er even ts in th is m an’s li e in decreas-
in g order o stress u ln ess are seriou s h ead in ju ry, large m ortgage, ch an gin g residen ce, an d
goin g on vacation .
2. The answer is D. Th is ch ron ic p ain p atien t is at h igh risk or d ep ression b u t at relatively
lower risk or d ru g ad d iction . Pain p atien ts ten d to b e u n d erm ed icated ; it is m ore likely
th at th is p atien t is receivin g too little rath er th an too m u ch p ain m ed ication . Psych ological
th erap ies can b e o sign i ican t b en e it to ch ron ic p ain p atien ts. Th is p atien t’s exp ression o
p ain is related n ot on ly to th e exten t o h er p ain bu t also to religiou s, cu ltu ral, an d eth n ic
actors.
3. The answer is B. Psych ological stress en gen dered by th is p atien t’s p ain is likely to resu lt in
in creased release o adren ocorticotrop ic h orm on e (ACTH) an d cortisol. Th is, in tu rn , resu lts
in decreased u n ction o th e im m u n e system as re lected in decreased lym p h ocyte resp on se
to m itogen s an d u n ction o n atu ral killer cells.
4. The answer is A. In th e Un ited States, n o p h ysician -to-p atien t tran sm ission o HIV h as been
con irm ed.
5. The answer is C. 6. The answer is E. To redu ce th is p atien t’s likelih ood o p sych ological p rob -
lem s in th e in ten sive care u n it (ICU), th e p h ysician sh ou ld exp lain the n eed or an d u n ction
RISE USMLE NEPAL

o th e m ech an ical ven tilator an d an y oth er m ech an ical sup p ort th at h e will n eed. Th e p h ysi-
cian sh ou ld also en cou rage visits rom am ily an d com m u n ication between p atien t an d sta .
Th e p atien t sh ou ld also be en cou raged to con trol asp ects o h is en viron m en t (e.g., ligh tin g
level). Outside stim u li (e.g., ligh t) should be in creased rather th an decreased, or exam p le,
p lacin g th e p atien t’s bed n ear a win dow. Becau se o th e disorien tin g n ature o the ICU,
deliriu m is com m on ly seen in ICU p atien ts. Pan ic disorder, obsessive–com p ulsive disorder,
illn ess an xiety disorder, an d som atization disorder are n o m ore com m on in ICU p atien ts
th an in the gen eral p op u lation .
7. The answer is E. Th e m ost ap p rop riate n ext step in th e m an agem en t o th is an xiou s
p atien t is a m edical evalu ation , p articu larly or th yroid u n ction . Sh e h as sym p tom s o
an overactive th yroid , in clu d in g a n eck m ass (en larged th yroid glan d ), exop h th alm os an d
weigh t loss. Peop le with th yroid h yp eractivity m ay also p resen t with an xiety (see Ch ap ter
13) an d in som n ia. Psych oth erapy, an tid ep ressan ts, an d b en zod iazep in es can m an age th e
associated sym p tom s b u t d o n ot ad d ress th is p atien t’s u n d erlyin g p h ysical con d ition .

256
Chapter 22 Psychosomatic Medicine 257

8. The answer is D. Th ese sign s (b u tter ly rash , ever), sym p tom s ( atigu e, join t p ain ), an d
laboratory test resu lts (m ild an em ia an d p resen ce o ANA) su ggest th at th is p atien t h as
system ic lu p u s eryth em atosu s (SLE), a con n ective tissu e disorder. SLE is m ore com m on in
A rican -Am erican wom en o rep rod u ctive age an d is exacerb ated by exp osu re to th e su n
(su ch as th is p atien t exp erien ced on vacation ). Person ality ch an ges an d p sych otic sym p -
tom s, su ch as the n otion th at p eop le on television are re errin g to h er (an idea o re eren ce;
see Tab le 11.1), also occu r in con n ective tissu e disorders su ch as SLE. Dissociative u gu e,
gen eralized an xiety d isorder, b rie p sych otic disorder, an d som atic sym p tom disorder are
n ot diagn osed wh en a m edical illn ess exp lain s th e beh avioral an d p h ysical sym p tom s.
9. The answer is B. In creased TSH, decreased T3 an d ree T4 as well as tired n ess, coarsen in g
o th e voice, an d osteop orosis in d icate th at th is p atien t has h yp oth yroidism . Patien ts with
HELP OTHERS SO THAT GOD WILL HELP YOU.

h yp oth yroidism n ot u n com m on ly p resen t with beh avioral sym p tom s su ch as th ose seen
in m ajor d ep ressive disord er. An xiety sym p tom s su ch as th ose seen in p an ic disorder an d
OCD are m ore closely associated with h yp erth yroid ism . Som atic sym p tom d isord er is
diagn osed wh en p h ysical in din gs do n ot ad equ ately exp lain th e p atien t’s sym p tom s. In
th is p atien t, th e p h ysical in din gs are sign i ican t.
10. The answer is A. Patien ts wh o u se a h elp er an im al sh ou ld be p erm itted to u se th e h elp
o th e an im al in as m an y situ ation s as p ossib le. Th ere ore, wh en it is tim e or th is blin d
p atien t to go in to th e exam in in g room , th e p h ysician sh ou ld allow th e p atien t to be led in to
th e exam in ation room by h er gu id e d og. Seein g-eye dogs are train ed to take th eir m asters
to m an y p laces an d are com m on ly allowed wh ere p et an im als are n ot, or exam p le, p u blic
bu ild in gs an d tran sp ortation . Th ere is n o reason to believe th at th e dog will in crease th e
risk o in ection or th e p atien t.
11. The answer is B. Th e doctor’s m ost ap p rop riate resp on se to th is p atien t with p ain cau sed
by rh eu m atoid arth ritis is to recogn ize th at sh e is u p set b ecau se h er in d ep en den ce h as
been lost. A p ractical su ggestion su ch as u sin g a lon g-h an d led tool is h elp u l. Tellin g h er to
wait u n til h er h usban d com es h om e or callin g a p lu m ber can in ten si y h er eelin gs o h elp -
lessn ess. Recom m en din g an arth ritis su p p ort grou p m ay be u se u l in th e lon g term bu t will
n ot h elp h er with th e cu rren t p roblem . A cortison e sh ot m igh t or m igh t n ot be h elp u l bu t
is again n ot ap p rop riate to deal with th e im m ediate p roblem .
12. The answer is D. Pren atal vitam in s con tain iron an d th is ch ild h as taken an iron overdose.
Th e p h ysician’s n ext step in m an agem en t o an iron overdose is to give de eroxam in e
m esylate. d -Pen icillam in e, eth ylen ed iam in etetraacetic acid (EDTA), bisp h osp h on ate, an d
lu m azen il are u sed to m an age overdoses o cop p er, lead, vitam in D, an d ben zodiazep in es,
resp ectively.
RISE USMLE NEPAL
Legal an d Eth ical
c ha pte r
23 Issu es in Medicin e
HELP OTHERS SO THAT GOD WILL HELP YOU.

Typical Board Question


A con cern ed m oth er p resen ts to th e p h ysician with h er 10-year-old son . Sh e tells th e p h ysi-
cian th at th e ch ild’s biological ath er was recen tly d iagn osed with Hu n tin gton’s disease (HD).
Sh e th en asks th e p h ysician to order a gen etic test or HD or h er son . Eth ically, sh ou ld th e
doctor ord er th e test?
(A) Yes, becau se th e m oth er h as th e righ t to kn ow abou t h er ch ild’s u tu re risk o seriou s
illn ess.
(B) Yes, because the m oth er will be able to em otion ally p repare th e ch ild i he is p ositive.
(C) No, becau se wh en h e is an ad u lt, th e ch ild can d ecid e wh eth er or n ot to b e tested .
(D) No, b ecau se th e ch ild is likely to b ecom e d ep ressed i h e is p ositive.
(E) No, becau se th e ch ild’s ath er h as n ot also con sen ted to th e testin g.
(See “An sw ers an d Explan ation s” at th e en d of th e ch apter.)

I. LEGAL COMPETENCE
A. Definition
1. To be legally competent to m ake h ealth care decision s, a p atien t m u st u n derstan d th e
risks, benefits , an d likely ou tcom e o su ch decision s.
RISE USMLE NEPAL

2. All adults (p erson s 18 years o age an d old er) are assu m ed to be legally com p eten t to m ake
h ealth care d ecision s or th em selves.

B. Minors
1. Minors (p erson s you n ger th an 18 years o age) u su ally are not con sidered legally com p eten t.
2. Emancipated minors are p eop le aged 14–17 years wh o are con sidered legally com p eten t
adu lts an d can give con sen t or th eir own m edical care.
3. To be con sidered an em an cip ated m in or, an in dividu al m u st m eet at least on e o th e ol-
lowin g criteria.
a. Be self-supporting.
b. Be in th e military.
c. Be married.
d. Have a child or wh om h e or sh e cares.
C. Questions of decision-making capacity and competence
1. I an adult’s com petence is in question (e.g., a person with an intellectual disability or dem en-
tia), physicians in volved in the case can evaluate an d testi y the capacity o the patient to m ake
a curren t health care decision. However, on ly a judge (with in put rom the patien t’s am ily or
physicians) can m ake the legal determination o com petence or uture health care decisions.

258
Chapter 23 Legal and Ethical Issues in Medicine 259

2. I a p erson is ou n d to be incompetent, a legal guardian will be ap p oin ted by th e cou rt to


m ake d ecision s or th at p erson . Th e legal gu ardian m ay or m ay n ot be a am ily m em ber.
3. A p erson may meet the legal standard or com p eten ce to accep t or re u se m ed ical m an age-
m en t even if she is mentally ill, h as an in tellectu al disability, or is in com p eten t in oth er
areas o h er li e (e.g., with in an ces).
4. The Folstein Mini–Mental State Examination (MMSE) (see Table 5.3) correlates to som e
exten t with clin ician s’ evalu ation o capacity. Wh ile a total score o 23 or h igh er su ggests
com p eten ce an d a total score o 18 or lower su ggests in com p eten ce, th e MMSE score
alon e can n ot be u sed to m ake a d eterm in ation o legal com p eten ce.
HELP OTHERS SO THAT GOD WILL HELP YOU.

II. INFORMED CONSENT


A. Overview. With th e excep tion o li e-th reaten in g em ergen cies (e.g., wh en th e tim e taken to
obtain con sen t wou ld u rth er en dan ger th e p atien t), a p h ysician m u st in orm an d obtain
con sen t (verbal or n on verbal) rom a com p eten t adu lt p atien t be ore p roceedin g with any
medical or surgical test or treatment.
1. Alth ou gh a sign atu re m ay n ot be requ ired or m in or m edical p roced u res, p atien ts u su ally
sign a document of consent or m ajor m ed ical p roced u res or or su rgery.
2. Other hospital personnel (e.g., n u rses) u su ally cannot ob tain in orm ed con sen t.

B. Components of informed consent


1. Be ore p atien ts can give con sen t to b e treated by a p h ysician , th ey m u st be in orm ed o
an d u n derstan d th e health implications o th eir d iagn osis.
2. Patien ts m u st also b e in orm ed o th e health risks and benefits o treatm en t and the alterna-
tives to treatm en t.
3. Patien ts m u st kn ow th e likely outcome if they do not consent to th e treatm en t.
4. Th ey m u st also be in orm ed th at th ey can withdraw consent for treatment at any time b e ore
th e p rocedu re.
5. Ph ysician s m u st also obtain in orm ed con sen t be ore en terin g a p atien t in a research
study. However, i a p atien t’s con d ition worsen s du rin g th e stu dy as a resu lt o lack o
treatm en t, p lacebo treatm en t, or exp osu re to exp erim en tal treatm en t, th e p atien t m u st
be taken ou t o th e stu dy an d given th e stan dard treatm en t or h is or h er con dition .

C. Special situations
1. Competent patients h ave th e right to refuse to consent to a n eeded test or p rocedu re or reli-
RISE USMLE NEPAL

giou s or oth er reason s, even i th eir h ealth will su er or death will resu lt rom su ch re u sal.
2. Alth ou gh m edical or su rgical in terven tion m ay be n ecessary to p rotect th e h ealth or li e o
th e etu s, a competent pregnant woman has the right to refuse su ch in terven tion (e.g., cesar-
ean section ) even i th e etu s will d ie or be seriou sly in ju red with ou t th e in terven tion .
3. Wh ile all o th e m edical in din gs are gen erally p rovided to a p atien t, a p h ysician can
delay tellin g th e p atien t th e diagn osis i th e p h ysician believes th at su ch kn owledge will
adversely affect the patient’s health (e.g., a coron ary p atien t) or u n til th e p atien t in dicates
th at h e or sh e is read y to receive th e n ews.
4. Th e opinions of family members , wh ile h elp u l or in orm ation ab ou t th e p atien t’s state
o m in d, can n ot dictate wh at in orm ation th e p h ysician tells th e p atien t. At the patient’s
request, am ily m em bers m ay be p resen t wh en th e p h ysician p rovides th e diagn osis.

D. Unexpected findings
1. I an unexpected finding d u rin g su rgery n ecessitates a nonemergency p roced u re or wh ich
th e p atien t h as n ot given con sen t (e.g., biop sy o an u n su sp ected ovarian m align an cy
ou n d du rin g a tu b al ligation ), th e p atien t m u st be given th e op portu n ity to p rovide
in orm ed con sen t b e ore th e ad d ition al p rocedu re can b e p er orm ed.
2. In an emergency in wh ich it is im p ossible to obtain con sen t with ou t u rth er en dan gerin g
th e p atien t (e.g., a “h ot” ap p en dix is ou n d du rin g a tu bal ligation ), th e p rocedu re can be
don e with ou t obtain in g con sen t.
260 BRS Behavioral Science

E. Treatment of minors
1. On ly th e parent or legal guardian can give con sen t or su rgical or m edical treatm en t o a
m in or (p erson s u n d er age 18 years).
2. Paren tal con sen t is not required in th e treatm en t o m in ors in emergency situ ation s
(e.g., wh en th e p aren t or gu ardian can n ot b e located an d a d elay in treatm en t can p oten -
tially h arm th e ch ild ). Paren tal con sen t also is n ot requ ired in th e treatm en t o m in ors
ages 14 to 17 years or:
a. Man agem en t o sexu ally tran sm itted diseases (STDs ).
b. Prescrip tion o contraceptives .
c. Medical care du rin g pregnancy.
d. Man agem en t issu es associated with drug or alcohol use .
HELP OTHERS SO THAT GOD WILL HELP YOU.

3. Most states requ ire p aren tal n oti ication or con sen t wh en a m in or seeks an abortion.
4. A court order can be obtain ed rom a ju dge (with in h ou rs i n ecessary) i a m in or h as a li e-
threaten in g illn ess or acciden t an d the parent or guardian refuses to consent to an established
(but n ot an experim en tal) m edical or surgical in terven tion or religious or other reason s.
5. Becau se th e likelih ood o a p oor ou tcom e is in evitable or extrem ely h igh , noninitiation of
resuscitation after delivery is u su ally ap p rop riate or in an ts b orn before the 23rd gesta-
tional week, at birth weigh t less than 400 g, or with anencephaly, or trisomy 13 or 18.
6. Testin g or genetic disorders
a. I th e disord er h as a p ediatric on set an d p reven tive th erapy or treatm en t is available
(e.g., cystic ib rosis), gen etic testin g sh ou ld be o ered or even requ ired.
b. I th ere are no preventive therapies or treatm en ts or th e disorder an d it h as a pediatric
onset (e.g., Tay-Sach s d isease), parents should have the discretion as to wh eth er or n ot to
do gen etic testin g.
c. Becau se th e ch ild can m ake th e decision to be tested or n ot wh en h e or sh e is an adu lt,
gen etic testin g sh ou ld gen erally not be done :
(1) I th e disorder h as an adult onset an d th ere are n o p reven tative th erap ies (e.g.,
Hu n tin gton’s d isease).
(2) To determ in e wh eth er a p rep u bescen t ch ild is a carrier of a genetic disorder th at
will a ect h is or h er o sp rin g (e.g., ragile X syn drom e).
d. I gen etic testin g reveals in orm ation (e.g., issues of paternity) u n related to th e p resen ce
or ab sen ce o th e gen etic d isord er, it is n ot n ecessary or th e p h ysician to divu lge su ch
in orm ation to an yon e.

III. CONFIDENTIALITY
RISE USMLE NEPAL

A. Alth ou gh p h ysician s are expected ethically to maintain patient confidentiality, th ey are n ot


requ ired to do so i :
1. Th eir p atien t is su sp ected o child or elder abuse .
2. Th eir p atien t h as a sign i ican t risk of suicide .
3. Th eir p atien t p oses a seriou s threat to another person.
4. Th eir p atien t p oses a risk to public safety (e.g., an im p aired d river).
B. In terven tion by th e p h ysician i th e patient poses a threat to an oth er p erson .
1. Th e p h ysician m u st irst ascertain th e credibility o th e th reat or d an ger.
2. I th e th reat or dan ger is credible, th e p h ysician m u st notify th e ap p rop riate law
en orcem en t o icials or social service agen cy an d warn th e in ten ded victim (th e
Taraso d ecision ).

IV. REPORTABLE ILLNESSES


A. Most states requ ire p h ysician s to report certain in ectiou s illn esses to th eir state health depart-
ments (rep ortable illn esses). State h ealth d ep artm en ts rep ort th ese illn esses to th e ederal
Cen ters or Disease Con trol an d Preven tion (CDC) or statistical p u rp oses.
Chapter 23 Legal and Ethical Issues in Medicine 261

B. Specific illnesses
1. “B A SSSMMART Clam or Chicken or you’re Gone.” In m ost states, h ep atitis B an d A,
Salm on ellosis, Sh igellosis, Syp h ilis, Measles, Mu m p s, AIDS, Ru bella, Tu bercu losis,
Ch lam yd ia, Ch icken p ox, an d Gon orrh ea are rep ortable.
2. STDs th at are rep ortable in m ost states in clu de AIDS, HIV-p ositive statu s is n ot rep ortable
in all states; gen ital h erp es is n ot rep ortable in m ost states.
3. Quarantine, th e seclu sion o in divid u als rom society du rin g the in ectiou s p eriod, is lim -
ited to illn esses su ch as cholera, diphtheria, tuberculosis, plague, small pox, viral hemorrhagic
fevers (e.g., Ebola and Marburg), and severe acute respiratory syndromes. Diseases su ch as
m easles, m u m p s, ru bella, an d ch icken p ox are n ot gen erally quaran tain able illn esses.
4. In ection with hepatitis A is related to exp osu re to infected feces as a resu lt o :
HELP OTHERS SO THAT GOD WILL HELP YOU.

a. Poor access to clean drinking water. Hep atitis A is less com m on in th e Un ited States,
Can ada, Western Eu rop e, Au stralia, an d Jap an th an in cou n tries with p oorer p u blic
san itation su ch as Mexico an d In d ia.
b. Anal sexual contact. Hep atitis A is m ore com m on in m en wh o h ave u n p rotected an al
sex with m en .

V. ETHICAL ISSUES INVOLVING HIV INFECTION


A. HIV-positive doctors. Ph ysician s are not required to inform eith er p atien ts or th e m edical estab-
lish m en t ab ou t th eir own or an oth er p h ysician’s HIV-p ositive statu s sin ce, i th e p h ysician
ollows accep ted p roced u res or in ection con trol, h e or sh e does n ot p ose a risk to p atien ts
(see Ch ap ter 22).

B. HIV-positive patients
1. Eth ically an d legally, a p h ysician cannot refuse to treat HIV-p ositive p atien ts becau se o
ear o in ection .
2. A pregnant patient at h igh risk or HIV in ection cannot be tested or treated or th e viru s
(e.g., with zid ovu d in e [AZT] an d / or n evirap in e [Viram u n e]) against her will, even i th e
etu s cou ld b e ad versely a ected by su ch re u sal. A ter th e ch ild is born , however, th e
m oth er can n ot re u se to allow th e ch ild to be tested an d treated or th e viru s.
3. I a health care provider is exp osed to th e body lu ids o a p atien t wh o m ay p oten tially be
in ected with HIV (e.g., a n u rse is stu ck with a n eedle wh ile ob tain in g b lood rom a p atien t
wh ose HIV statu s is u n kn own ), it is accep table to test the patient for HIV infection even i
th e p atien t re u ses to con sen t to th e test.
RISE USMLE NEPAL

4. Physician s are not required to maintain confidentiality when an HIV-p ositive patien t puts an
iden ti ied person at risk by en gagin g in un protected sex (see Section III.B above).

VI. INVOLUNTARYAND VOLUNTARYPSYCHIATRIC


HOSPITALIZATION
A. Un der certain circu m stan ces, p atien ts in p sych iatric emergency situations wh o will n ot or
can n ot agree to be h osp italized m ay be h osp italized again st th eir will or with ou t con sen t
(in volu n tary h osp italization ) with certi ication by one or two physicians. Su ch p atien ts m ay
b e h osp italized or u p to 90 days (d ep en d in g on state law) be ore a cou rt h earin g.

B. Even i a p sych iatric p atien t chooses voluntarily to be hospitalized, h e or sh e m ay be requ ired


to wait 24–48 h ou rs b e ore b ein g p erm itted to sign ou t again st m edical advice.

C. Patients , wh o are con in ed to m en tal h ealth acilities, wh eth er volu n tarily or in volu n tarily,
h ave th e right to receive treatment an d to refuse treatment (e.g., m edication , electrocon vu lsive
th erapy). Patien ts wh o are actively p sych otic or su icidal, h owever, gen erally can n ot re u se
treatm en t aim ed at stabilizin g th eir con dition .
262 BRS Behavioral Science

VII. ADVANCE DIRECTIVES


A. Overview
1. Advan ce directives are in stru ction s given by p atien ts in anticipation of the need for a medical
decision. A du rable p ower o attorn ey an d a livin g will are exam ples o advan ce directives.
a. A durable power of attorney is a directive in which a com p eten t p erson designates another
person (e.g., sp ou se, rien d) as h is or h er legal represen tative (i.e., h ealth care p roxy) to
m ake decision s abou t h is or h er h ealth care wh en h e or sh e can n o lon ger do so.
b. A living will is a written document or oral statement in wh ich a com p eten t p erson gives
direction s or h is or h er u tu re h ealth care i h e or sh e becom es in com p eten t to m ake
HELP OTHERS SO THAT GOD WILL HELP YOU.

decision s wh en h e or sh e n eed s care.


c. Th e most recent information abou t th e p atien t’s wish es is th e m ost relevan t advan ce
directive.
2. Health care acilities that receive Medicare paym ents (m ost hospitals and nursing hom es) are
required to ask patients whether they have advan ce directives and, i necessary, help patien ts to
prepare them . They m ust also in orm patients o their right to refuse treatm en t or resuscitation .

B. Persistent vegetative state (PVS). A p erson in a PVS m ay appear to be awake with eyes open
bu t is n ot aware o oth ers or o th e en viron m en t an d is n ot exp ected to ever recover brain
u n ction . Wh eth er to m ain tain a PVS p atien t on li e su p p ort is dep en den t u p on h is or h er
advan ce directives or on th e d ecision s o su rrogates.

C. Surrogates
1. I an in com p eten t p atien t does n ot h ave an advance directive , p eop le wh o kn ow th e
p atien t, or exam p le, am ily m em bers or rien ds (surrogates ), m u st d eterm in e what the
patient would have done if he or she were competent (th e su b stitu ted ju dgm en t stan d ard ).
Th e personal wishes of surrogates are irrelevant to th e m ed ical decision .
2. Th e priority order in wh ich am ily m em bers m ake th is determ in ation is th e (1) spouse, (2)
adult children, (3) parents, (4) siblings , an d, in ally, (5) other relatives . I th ere is a con lict
am on g am ily m em b ers at th e sam e p riority level an d discu ssion am on g am ily m em bers
does n ot settle th e issu e, th e eth ics com m ittee o th e h osp ital m ay m ake th e d ecision . For
in tractab le disagreem en t, legal in terven tion (e.g., by a ju dge) m ay be n ecessary.
3. Even i a h ealth care p roxy or su rrogate h as been m akin g decision s or an in com p eten t
p atien t, i th e p atien t regains function (com p eten ce) even brie ly or in term itten tly, he or she
regain s the right durin g those periods to m ake decision s about his or her own health care.
RISE USMLE NEPAL

VIII. DEATH AND EUTHANASIA


A. Legal standard of death
1. In th e Un ited States, th e legal stan dard o death (wh en a p erson’s h eart is beatin g) is irre-
versible cessation of all functions of the entire brain, in clu din g th e brain stem . Th is stan -
dard d i ers am on g states b u t com m on ly in volves absence of:
a. Resp on se to extern al even ts or p ain u l stim u li.
b. Sp on tan eou s resp iration .
c. Cep h alic re lexes (e.g., p u p illary, corn eal, p h aryn geal).
d. Electrical p oten tials o cereb ral origin over 2 m V rom sym m etrically p laced electrodes
m ore th an 10 cm ap art.
e. Cerebral blood low or m ore th an 30 m in u tes.
2. Physicians certify th e cause of death (e.g., n atu ral, su icide, acciden t) an d sign th e death
certi icate.
3. I th e p atien t is dead accordin g to th e legal stan dard, th e p h ysician is au th orized to remove
life support. A cou rt ord er or relative’s p erm ission is n ot n ecessary.
Chapter 23 Legal and Ethical Issues in Medicine 263

B. Organ donation
1. A p atien t’s organs cannot be harvested after death u n less th e p atien t (or p aren t i th e p atien t
is a m in or) h as sign ed a d ocu m en t (e.g., an organ d on or card) or in orm ed su rrogates o
h is or h er wish to d on ate.
2. A minor (bu t n ot an adu lt) can be compelled to donate tissu e (e.g., b on e m arrow, skin ) to a
close relative i h e or sh e:
a. Is th e on ly ap p rop riate sou rce.
b. Will n ot be h arm ed seriou sly by th e don ation .

C. Euthanasia. Accord in g to m edical codes o eth ics (e.g., th ose o th e Am erican Med ical
Association an d m edical sp ecialty organ ization s), euthanasia (d irectly killin g a p atien t or
com p assion ate reason s) is a criminal act an d is never appropriate.
HELP OTHERS SO THAT GOD WILL HELP YOU.

1. Physician-assisted suicide (providin g a m eans or a patient to com m it suicide or com pas-


sion ate reason s) is legal in som e states but is not generally an indictable o ense in other states
as lon g as the physician does n ot actually per orm the killing (e.g., the patient injects him sel ).
2. There is no ethical distinction between withholding an d withdrawing li e-sustain in g treatm en t.
a. I requ ested by a com p eten t p atien t, ood, water, an d m edical care can be with h eld
rom a p atien t wh o h as no reasonable prospect of recovery.
b. I a com p eten t p atien t requests removal of artificial life support (e.g., ven tilator su p -
p ort), it is both legal and ethical or a p h ysician to com p ly with th is requ est. Su ch an
action by th e p h ysician is n ot con sidered eu th an asia or assisted su icide.

IX. MEDICAL MALPRACTICE


A. Overview
1. Medical m alp ractice occu rs wh en h arm com es to a p atien t as a resu lt o th e action or
in action o a p h ysician . Th e elem en ts o m alp ractice (the 4 Ds ) are:
a. Dereliction, or n egligen ce (i.e., deviation rom n orm al stan dards o care), o a
b. Duty (i.e., th ere is an establish ed p h ysician –p atien t relation sh ip ) th at cau ses
c. Damages (i.e., in ju ry)
d. Directly to th e p atien t (i.e., th e dam ages were cau sed by th e n egligen ce, n ot by an oth er
actor).
2. Surgeons (in cludin g obstetrician s) an d anesthesiologists are the specialists m ost likely to be
sued or m alp ractice. Psych iatrists an d am ily practition ers are the least likely to be sued.
3. Malp ractice is a tort, or civil wrong, n ot a crim e. A in din g or th e p lain ti (th e p atien t)
resu lts in a in an cial award to th e p atien t rom th e d e en dan t p h ysician or h is or h er in su r-
RISE USMLE NEPAL

an ce carrier, n ot a jail term or loss o licen se.


4. Th e n u m ber o m alp ractice claim s h as in creased over th e years. Th is in crease is m ain ly a
resu lt o a breakdown o th e tradition al physician–patient relationship becau se o :
a. Technological advances in m edicin e, wh ich redu ce p erson al con tact with th e p h ysician .
b. Limits on time for personal interaction an d p h ysician au ton om y, p artly as a resu lt o th e
growth o m an aged care.

B. Damages. Th e p atien t m ay be award ed com p en satory d am ages on ly, or b oth com p en satory
an d p u n itive dam ages.
1. Compensatory damages are given to reimburse th e p atien t or m ed ical b ills or lost salary
an d to com p en sate th e p atien t or p ain an d su erin g.
2. Punitive damages are awarded to th e p atien t to punish the physician an d set an exam p le
or th e m ed ical com m u n ity. Pu n itive dam ages are rare an d are awarded on ly in cases o
wan ton carelessn ess or gross n egligen ce (e.g., a dru n k su rgeon cu ts a vital n erve).

C. Relationships with patients


1. Sexu al relation sh ip s with cu rren t or orm er p atien ts are inappropriate an d are prohibited
by th e eth ical stan dards o m ost sp ecialty boards.
2. Patien ts wh o claim th at th ey h ad a sexu al relation sh ip with a p h ysician m ay file an ethics
complaint or a medical malpractice complaint, or b oth .
264 BRS Behavioral Science

3. Ph ysician s sh ou ld avoid treating family members , close friends, or employees sin ce p erson al
eelin gs can in ter ere with p ro ession al objectivity, an d am iliarity m ay lim it qu estion s or
p h ysical exam in ation s o a sen sitive n atu re.
4. Physician s sh ould avoid accepting valuable gifts (e.g., thin gs that can be sold) rom p atien ts.

X. IMPAIRED PHYSICIANS
A. Causes o im p airm en t in p h ysician s in clu de:
1. Dru g or alcoh ol u se.
2. Ph ysical or m en tal illn ess.
HELP OTHERS SO THAT GOD WILL HELP YOU.

3. Im p airm en t in u n ction in g associated with old age.


B. Removing an im p aired colleague, m edical studen t, or residen t rom con tact with p atien ts is an
ethical requirem en t because patien ts m ust be protected an d the im paired colleague should
be helped. The legal requirem en t or reportin g im paired colleagues varies am on g states.
1. An impaired medical student sh ou ld be rep orted to th e dean o th e m edical sch ool or th e
d ean o stu d en ts.
2. An impaired resident or attending physician should be reported to the person directly in charge
o him or her (e.g., the residen cy train in g director or the chie o the m edical sta , respectively).
3. Impaired physicians in p rivate p ractice sh ou ld be rep orted to th e state licen sin g board or
th e im p aired p h ysician s p rogram , u su ally ru n by p h ysician s wh o are associated with th e
state m edical society.
Tab le 23.1 p rovid es “Do’s” an d “Do Not’s” or an swerin g qu estion s on th e USMLE
in volvin g legal an d eth ical issu es.

t a b l e 23.1 Do’s and Do Not’s for Answering USMLE Ethical and Legal Questions

Do Do Not

Do tell patients the complete truth about their illness and Do not cover up the truth about a patient’s condition or explain
prognosis in language they can understand. the diagnosis and prognosis using medical terms that the
patient does not understand.
Do tell patients the truth about your qualifications (e.g., Do not cover up the true status of medical students or residents
“I am a third-year medical student”). (e.g., “I am a member of the doctor’s team”).
Do speak to competent adult patients directly. Do not discuss issues concerning patients with their relatives
(e.g., spouse, adult children) or anyone else (e.g., insurance
companies) without the patients’ permission.
Do ask competent patients to consent to their own Do not ask a relative for consent to treat a patient unless the
RISE USMLE NEPAL

treatment. relative has durable power of attorney.


Do encourage competent patients to make their own Do not make decisions about health care choices for patients; supply
health care choices (i.e., be autonomous). them with the information they need to make such decisions.
Do take care of your patient yourself. Do not refer your patient (no matter how difficult or offensive) to
another student, resident, or physician.
Do spend time with your patient. Do not delegate your responsibilities (e.g., giving lengthy medical
instructions to patients) to office staff (e.g., nurses).
Do make health care decisions based on what is best for Do not limit health care based on expense in time or money.
the health of the patient.
Do discuss all treatment options with patients, even if Do not restrict information about uncovered treatment options
their insurance companies do not cover such options. (such insurance company-generated “gag clauses” are
ethically unacceptable).
Do discuss with a pregnant patient the practical issues Do not advise a patient to have an abortion (unless she is at
of having and caring for the child. medical risk) no matter what the age of the mother (e.g.,
teenage) or the condition of the fetus (e.g., Down’s syndrome).
Do encourage a pregnant minor to make her own decision Do not accede to the demand of the pregnant woman’s parents
about whether or not to have an abortion. At the same to perform an abortion (even if the woman or her unborn child
time, encourage discussion between the woman and is intellectually disabled).
her parents about the best course of action.
Do provide medically needed analgesia to a terminally ill pat- Do not administer an analgesic overdose with the purpose of
ient even if it coincidentally may shorten the patient’s life. shortening a terminally ill patient’s life.
Review Test

Directions: Each o th e n u m b ered item s or in com p lete statem en ts in th is section is ollowed by


an swers or by com p letion s o th e statem en t. Select th e one lettered an swer or com p letion th at
is best in each case.
HELP OTHERS SO THAT GOD WILL HELP YOU.

1. A p regn an t 36-year-old m arried wom an 4. A 4th year m edical stu den t on a


asks h er p h ysician to test th e etu s or p sych iatry rotation h elp s evalu ate a p atien t
Down’s syn drom e. Sh e n otes th at i th e test is who p resen ts with extrem e an xiety in th e
p ositive, sh e will h ave an abortion . Th e test is h osp ital em ergen cy room . Th e n ext year,
p ositive. At th is tim e, th e p h ysician sh ou ld when th e stu den t is a residen t, h e sees th e
(A) con tact the wom an an d tell her th e results sam e p atien t in th e h osp ital ou tp atien t
(B) contact the husband and tell him the results dep artm en t, eels attracted to h er an d wou ld
(C) ask the wom an to com e in with her like to ask h er ou t on a date. Th is rst year
husband and tell them the results together residen t sh ou ld
(D) n ot tell eith er p aren t th e resu lts (A) wait 2 years be ore askin g th e p atien t ou t
(B) wait 10 years be ore askin g th e p atien t
2. A 7-year-old ch ild wh o h as n ever received ou t
th e m easles, m u m p s, ru bella (MMR) vaccin e (C) ask th e p atien t ou t becau se h e was on ly a
h as ju st been diagn osed with m easles. A ter stu d en t wh en h e saw h er
treatin g th e ch ild, th e p h ysician’s n ext step in (D) ask th e p atien t ou t becau se h e was n ot
th e m an agem en t o th is case is to do wh ich directly resp on sib le or th e p atien t’s care
o th e ollowin g? wh en h e saw h er
(A) Pu t th e ch ild in qu aran tin e im m ed iately. (E) n ever ask th e p atien t ou t
(B) Rep ort th e case to th e state ch ild
p rotective services agen cy. 5. A 30-year-old m an an d h is 10-year-
(C) Rep ort th e case to th e Cen ters or old son are in ju red in a com m u ter train
Disease Con trol an d Preven tion . acciden t. Both o th em n eed su rgery with in
(D) Rep ort th e case to local law en orcem en t th e n ext 12 h ou rs. Th e ath er is clearly
au th orities. m en tally com p eten t b u t re u ses th e su rgery
(E) Rep ort th e case to th e State Health or religiou s reason s or both h im sel an d h is
Dep artm en t. son . Th e p h ysician sh ou ld
RISE USMLE NEPAL

(A) get a cou rt order or th e su rgery on th e


3. Citin g religiou s reason s, th e p aren ts o a ch ild bu t d o n ot op erate on th e ath er
17-year-old patient with cancer have re used the (B) get a cou rt order or th e su rgery or both
stan dard treatm en t or her illn ess. The patient, th e ath er an d ch ild
who is in high school an d lives at hom e with (C) get p erm ission or th e su rgery or both
her parents, wants the treatm ent. The patient’s th e ath er an d ch ild rom th e m oth er
physician gives the patient the treatm ent. Which (D) h ave th e ath er m oved to an oth er
o the ollowing is the m ost likely reason that the h osp ital
doctor h as given th e treatm en t? (E) ollow th e ath er’s wish es an d do n ot
(A) Th e p atien t h as requ ested th e treatm en t. op erate on eith er th e ath er or th e ch ild
(B) Paren ts can n ot re u se n ecessary an d
stan d ard treatm en t or a m in or ch ild .
(C) Peop le can n ot re u se treatm en t or
religiou s reason s.
(D) The doctor an ticipates th at he or sh e will
be sued or m alpractice i the patien t dies.
(E) Th e doctor believes th at th e p atien t will
die with ou t th e treatm en t.

265
266 BRS Behavioral Science

6. A 60-year-old m an h as a su sp iciou s 9. A su rgeon to wh om an in tern ist h as


m ass b iop sied . He is clearly m en tally regu larly re erred p atien ts tells th e in tern ist
com p eten t b u t h as been d ep ressed over h is in con den ce th at h e (th e su rgeon ) is HIV
wi e’s recen t death . His dau gh ter asks th e p ositive. At th e sam e tim e, th e su rgeon
p h ysician n ot to tell th e p atien t th e d iagn osis assu res th e in tern ist th at h e always com p lies
i the resu lts sh ow a m align an cy becau se with accep ted p rocedu res or in ection
sh e ears th at h e will kill h im sel . I th e m ass con trol. Th e m ost ap p rop riate action or th e
p roves to b e m align an t, th e p h ysician sh ou ld in tern ist to take at th is tim e is to
(A) ollow th e dau gh ter’s wish es an d n ot tell (A) stop re errin g p atien ts to th e su rgeon
th e p atien t th e diagn osis (B) rep ort th e su rgeon to th e state h ealth
(B) tell th e p atien t th e diagn osis au th orities
HELP OTHERS SO THAT GOD WILL HELP YOU.

im m ediately (C) rep ort th e su rgeon to th e h osp ital


(C) tell th e p atien t n ot to worry an d th at h e adm in istration
will be well cared or (D) con tin u e to re er p atien ts to th e su rgeon
(D) evalu ate th e p atien t’s risk or su icide (E) con tin u e to re er p atien ts to th e su rgeon
be ore tellin g h im th e diagn osis bu t irst tell th em ab ou t h is HIV statu s
(E) h ave th e d au gh ter tell th e p atien t th e
diagn osis 10. A 25-year-old m an wh o is HIV p ositive
p resen ts to a p h ysician’s o ce or treatm en t
7. A 10-year-old boy wh o was in ju red du rin g o a skin lesion . Becau se sh e is a raid o
gym class is brou gh t to th e em ergen cy in ection , th e p h ysician re u ses to treat h im .
departm en t. He h as a severe laceration th at Th is p h ysician’s re u sal to treat th e p atien t is
requ ires im m ediate su tu rin g. His p aren ts are b est described as
on vacation an d can n ot be located an d an (A) u n eth ical an d illegal
au n t is b abysittin g or th e ch ild. Th e m ost (B) eth ical an d legal
ap p rop riate action or th e p h ysician to take (C) u n eth ical bu t legal
at th is tim e is to (D) eth ical bu t illegal
(A) ob tain con sen t rom th e au n t
(B) ob tain con sen t rom th e gym teach er 11. A legally com p eten t, term in ally ill
(C) su tu re th e laceration with ou t ob tain in g 70-year-old p atien t on li e su p p ort asks h er
con sen t p h ysician to tu rn o th e m ach in es an d let
(D) keep th e p atien t com ortab le u n til th e h er die. Th e p h ysician ollows th e p atien t’s
p h ysician reach es th e p aren ts wish es an d d iscon tin u es li e su p p ort. Th e
(E) ob tain con sen t rom th e ch ild h im sel p h ysician’s action is best described as
(A) u n eth ical an d illegal
8. On h is su rgery rotation , m edical stu den t (B) eth ical an d legal
X requ en tly sm ells alcoh ol on th e breath (C) u n eth ical bu t legal
RISE USMLE NEPAL

o m ed ical stu den t Y. Medical stu d en t Y (D) eth ical bu t illegal


den ies th at h e h as b een d rin kin g. Th e m ost
ap p rop riate action or m edical stu d en t X to
take at th is tim e is to
(A) ask m edical stu d en t Y wh y h e is d rin kin g
on th e loor
(B) warn m edical stu den t Y th at h e will be
rep orted i h e con tin u es to drin k on th e
loor
(C) rep ort m edical stu den t Y to th e dean o
stu d en ts
(D) rep ort m edical stu den t Y to th e p olice
(E) ask th at m ed ical stu d en t Y b e tran s erred
to an oth er rotation site
Chapter 23 Legal and Ethical Issues in Medicine 267

12. A legally com p eten t 65-year-old m an 14. A 55-year-old wom an u n dergoes


p rodu ces a d ocu m en t th at states th at h e does su rgery to rep air a torn kn ee ligam en t.
n ot wan t an y m easures taken to p rolon g h is A ter th e su rgery, sh e h as p artial p aralysis
li e i h e becom es n eu rologically im p aired. o the a ected leg an d su es th e su rgeon or
Five d ays later, h e h as a stroke an d requ ires m alp ractice. Th e lawsu it will be su ccess u l i
li e su p p ort. Exten sive evalu ation reveals th e p atien t can p rove th at
th at h e will n ever recover con sciou sn ess bu t (A) th e p h ysician did n ot ollow th e u su al
will in stead rem ain in a vegetative state. Th e stan d ards o p ro ession al care
p atien t’s wi e u rges th e p h ysician to keep h er (B) th e p aralysis is p erm an en t
h u sban d alive. Th e p h ysician sh ou ld (C) th e p h ysician was n ot board certi ied in
(A) get a cou rt order to start li e su p p ort orth op edic su rgery
HELP OTHERS SO THAT GOD WILL HELP YOU.

(B) ollow th e wish es o th e wi e an d start li e (D) h er m arriage is n egatively a ected by th e


su p p ort p aralysis
(C) carry ou t th e p atien t’s p rior requ est an d (E) sh e will lose a sign i ican t am ou n t o tim e
n ot start li e su p p ort rom work becau se o th e p aralysis
(D) ask th e p atien t’s ad u lt ch ild ren or
p erm ission to start li e su p p ort 15. A 35-year-old m an wh o h as
(E) tu rn th e case over to th e eth ics sch izop h ren ia an d lives in a su bway station
com m ittee o th e h osp ital is brou gh t to th e em ergen cy dep artm en t.
Th e b est reason to h osp italize th is p atien t
13. A m arried 17-year-old wom an su stain s again st h is will is i h e
b rain dam age a ter an u n su ccess u l (A) is dirty an d dish eveled
su icide attem p t. Sh e is in a com a, sh ows (B) is m aln ou rish ed
n o sp on tan eou s resp iration , an d requ ires (C) h as attem p ted to p u sh a p assen ger on to
li e su p p ort. Clin ical exam in ation an d th e train tracks
electroen cep h alogram reveal lack o (D) is h earin g voices
resp on se to extern al even ts or p ain u l (E) believes th e FBI is listen in g to h is
stim u li; absen ce o p u p illary, corn eal, con versation s
or ph aryn geal ref exes; an d absen ce o
electrical p oten tials o cerebral origin 16. Alth ou gh sh e p reviou sly agreed to allow
(i.e., “brain d eath”). Th e p atien t’s ath er a m edical stu den t to atten d th e delivery, a
in sists th at th e p h ysician n ot with d raw li e 27-year-old wom an , wh o is abou t to give
su p p ort. Th e m ost ap p rop riate action or th e birth, states th at sh e does n ot wan t th e
p h ysician to take at th is tim e is to m edical stu den t in th e delivery room . Th e
(A) exp lain to th e am ily th at th e p atien t is m ost ap p rop riate action or th e m edical
legally dead an d with draw li e su p p ort stu den t to take at th is tim e is to
(B) con tin u e li e su p p ort u n til a am ily (A) stay, b u t keep to a p art o th e room
RISE USMLE NEPAL

m em b er au th orizes its with d rawal wh ere th e p atien t can n ot see h im


(C) get a cou rt order to with draw li e su p p ort (B) tell th e p atien t th at sh e m u st let h im stay
(D) get th e p atien t’s h u sban d to au th orize becau se sh e is in a teach in g h osp ital
with drawal o li e su p p ort (C) ask th e atten d in g p h ysician or
(E) get th e p atien t’s m oth er to au th orize p erm ission to stay
with drawal o li e su p p ort (D) in orm th e atten din g p h ysician an d th en
leave th e delivery room
(E) rem in d th e p atien t th at sh e already gave
p erm ission or h im to stay
268 BRS Behavioral Science

17. A 58-year-old m an is sch edu led or op en - 20. A n u rse, wh o works in a p h ysician’s


h eart su rgery. Th e n igh t b e ore th e su rgery, o ce, asks h er em p loyer to becom e h er
th e p atien t seem s an xiou s an d worried . p rim ary care p h ysician . Th e p h ysician’s b est
Wh en th e su rgeon obtain s in orm ed con sen t resp on se is
rom th e p atien t sh e sh ou ld in clu d e (A) “I can be you r p rim ary care p h ysician
(A) th e risks o th e an esth esia on ly an d I will start a ch art or you .”
(B) th e risks o th e su rgery on ly (B) “I can be you r p rim ary care p h ysician
(C) th e risks o both th e su rgery an d bu t on ly i I am n ot p aid.”
an esth esia, om ittin g th e risk o d eath (C) “I can n ot be you r p rim ary care p h ysician
(D) th e risks o both th e su rgery an d becau se I am you r em p loyer.”
an esth esia, in clu d in g th e risk o d eath (D) “I can n ot be you r p rim ary care p h ysician
HELP OTHERS SO THAT GOD WILL HELP YOU.

(E) n on e o th e risks o th e su rgery or bu t I can treat you with ou t startin g a


an esth esia ch art.”
(E) “I can n ot be you r p rim ary care p h ysician
18. A 90-year-old p atien t wh o was recen tly bu t I can write p rescrip tion s or you
tran s erred to a n u rsin g h om e rom th e occasion ally.”
h osp ital exp erien ces card iac arrest an d a
p h ysician is called. Alth ou gh th ey believe 21. A com p eten t 30-year-old p atien t wh o is
th at th e p atien t sign ed a do n ot resu scitate 38 weeks p regn an t re u ses to h ave a cesarean
(DNR) order, th e sta can n ot locate th e d elivery desp ite th e act th at with ou t th e
p atien t’s ch art con tain in g th e ad van ce su rgery th e etu s will p robably die. Both
directive. Most ap p rop riately, th e p h ysician h er p h ysician an d a con su ltation -liaison
sh ould n ow p sych iatrist h ave ailed to con vin ce h er to
(A) resu scitate th e p atien t h ave th e su rgery. Th e m ost ap p rop riate
(B) n ot resu scitate th e p atien t action or h er p h ysician to take at th is
(C) ask th e am ily i sh e sh ou ld resu scitate tim e is to
th e p atien t (A) get p erm ission rom th e p atien t’s
(D) ask th e n u rsin g h om e sta i sh e sh ou ld h u sban d to do th e su rgery
resu scitate th e p atien t (B) ask a ju d ge to issu e a cou rt ord er to d o
(E) p rovid e su p p ortive m an agem en t on ly th e su rgery
(C) tell th e p atien t th at sh e can be crim in ally
19. A clearly com p eten t 25-year-old p rosecu ted i th e ch ild dies
wom an , wh o is 5 m on th s p regn an t, tells h er (D) deliver th e ch ild vagin ally
obstetrician th at over th e p ast year sh e h as (E) re er th e p atien t to an oth er p h ysician
been u sin g illegal in traven ou s d ru gs an d h as
h ad at least ve d i eren t sexu al p artn ers. 22. Wh ich o th e ollowin g p atien ts is m ost
Th e p h ysician exp lain s th e risk to h er etu s likely to b e in ected with h ep atitis A?
RISE USMLE NEPAL

i sh e is HIV-p ositive an d h ow p ren atal (A) A h eterosexu al Mexican m an


treatm en t with an an tiretroviral agen t can (B) A h om osexu al Mexican m an
red u ce th e risk. He th en su ggests th at sh e b e (C) A h eterosexu al Fren ch m an
tested or th e viru s. Th e wom an re u ses to b e (D) A h om osexu al Fren ch m an
tested. Th e m ost ap p rop riate action or th e (E) A h eterosexu al Can ad ian m an
p h ysician to take at th is tim e is to (F) A h om osexu al Can ad ian m an
(A) p er orm th e HIV test on a blood sam p le
ob tain ed or an oth er p u rp ose
(B) give th e p atien t a p rescrip tion or an
an tiretroviral agen t
(C) re er th e p atien t to an oth er obstetrician
(D) n ote in th e p atien t’s ch art th at sh e h as
re u sed to be tested an d con tin u e carin g
or h er
(E) get a cou rt order to do th e HIV test
Chapter 23 Legal and Ethical Issues in Medicine 269

23. A 62-year-old m an wh o h as b een in a 25. A p h ysician determ in es th at a 40-year-


seriou s au tom obile acciden t is brou gh t to old p atien t with Down’s syn drom e n eeds
th e h osp ital. A ter th e p atien t is stabilized , h ep atic su rgery with in 72 h ou rs. Wh en th e
m edical evalu ation reveals th at h e will n ever p h ysician exp lain s th is to h er, th e p atien t
recover con sciou sn ess bu t will in stead p oin ts to h er stom ach an d says, “Tu m m y
rem ain in a vegetative state. Th ere is n o h u rt.” At th is tim e, th e p h ysician sh ou ld
advan ce directive so d ecision m u st be (A) declare an em ergen cy an d op erate
m ade abou t wh eth er or n ot to con tin u e li e im m ediately
su p p ort. Prior to th e acciden t, th e p atien t (B) ask th e p atien t’s p aren ts or p erm ission
was declared th e legal gu ard ian or h is wi e to op erate
wh o h as Alzh eim er’s d isease an d lives in (C) con sid er th e p atien t com p eten t an d th en
HELP OTHERS SO THAT GOD WILL HELP YOU.

a n u rsin g h om e. Th e p atien t’s sister, with op erate


wh om th e p atien t lives, u rges th e p h ysician (D) get su bstitu ted ju dgm en t rom th e
to keep h er b roth er alive at an y cost. Th e p atien t’s p aren ts an d th en op erate
p atien t’s ad u lt son tells th e p h ysician th at h is (E) determ in e wh eth er th e p atien t h as th e
ath er wou ld n ot wan t to b e kep t alive in a cap acity to give con sen t
vegetative state. Most correctly, wh ich o th e
ollowin g will m ake th e d ecision regardin g 26. A su rgeon is ab ou t to p er orm a
con tin u in g li e su p p ort or th is p atien t? ch olecystectom y on an ob ese p atien t.
(A) Th e h osp ital eth ics com m ittee A ter th e an esth esia takes e ect, th e
(B) Th e wi e an esth esiologist lau gh s an d says, “I sh e
(C) Th e son wasn’t so at sh e wou ld n ot h ave gall bladder
(D) Th e p h ysician p roblem s.” At th is tim e th e su rgeon sh ou ld
(E) Th e sister (A) ask th e an esth esiologist to leave an d in d
a rep lacem en t
24. At an even in g goin g-away p arty, a (B) leave an d ask an oth er su rgeon to do th e
40-year-old p h ysician d rin ks 10 glasses o p rocedu re
b eer. Th e p h ysician th en gets a p age to go to (C) scold th e an esth esiologist or h is lack o
th e h osp ital. Desp ite th e act th at a colleagu e em p ath y
tells h im n ot to go, th e p h ysician says, “I am (D) carry ou t th e op eration an d th en rep ort
n e,” an d leaves or th e h osp ital. Wh at is th e th e in ciden t to th e eth ics com m ittee
m ost ap p rop riate action or th e colleagu e to (E) carry ou t th e op eration an d th en sp eak
take? to th e an esth esiologist alon e
(A) Follow th e p h ysician an d p h ysically
restrain h im . 27. A p regn an t wom an an d h er h u sban d
(B) Im m ediately n oti y th ose in ch arge at th e ask th eir p h ysician to do gen etic testin g to
h osp ital th at th e p h ysician is likely to be determ in e i th e etu s sh e is carryin g is at
RISE USMLE NEPAL

im p aired. risk or cystic b rosis. Th e test sh ows th at th e


(C) Rep ort th e p h ysician to th e h osp ital h u sban d is n ot th e ch ild’s biological ath er.
eth ics com m ittee th e n ext d ay. With resp ect to th e p atern ity n d in g th e
(D) Rep ort th e p h ysician to th e State p h ysician sh ou ld
Board o Medical Exam in ers as soon as (A) tell on ly th e m oth er
p ossible. (B) tell on ly th e h u sban d
(E) Ask to sp eak to th e p h ysician alon e as (C) tell both th e m oth er an d th e h u sban d
soon as p ossib le. (D) write it in th e ch art bu t do n ot tell th e
cou p le
(E) n eith er write it in th e ch art n or tell th e
cou p le
270 BRS Behavioral Science

28. Paren ts o a 10-year-old girl wan t to n d 31. A p h ysician h as ju st told a cou p le


ou t i sh e is a carrier o th e ragile X gen e. th at th eir 3-year-old son h as ragile X
With resp ect to gen etic testin g o h er carrier syn drom e. Th e wom an tells th e doctor th at
status, m ost ap p rop riately th e p h ysician h er sister-in -law is cu rren tly p regn an t b u t
sh ould advise th e p aren ts th at th e girl be th at sh e an d h er h u sban d d o n ot wan t to
tested tell an yon e in th eir am ily abou t th eir own
(A) wh en sh e reach es rep rod u ctive m atu rity ch ild’s d iagn osis. Wh ich o th e ollowin g
an d requ ests th e test action s by th e p h ysician is m ost ap p rop riate
(B) wh en sh e decides to get m arried con cern in g in orm in g th e sister-in -law?
(C) wh en sh e d ecid es to h ave ch ildren (A) Ask or a th ird p arty to m ed iate th e
(D) as soon as p ossible com m u n ication b etween th e cou p le an d
HELP OTHERS SO THAT GOD WILL HELP YOU.

(E) wh en sh e reach es rep rod u ctive m atu rity th e sister-in -law


(B) In sist th at th e cou p le tell th e p regn an t
29. Paren ts o a 9-year-old ch ild with wom an
leu kem ia are o ered a ch an ce to en roll (C) Keep th e diagn osis con id en tial
th e ch ild in a ran dom ized stu d y o a n ew (D) O er to con tact th e p regn an t wom an
exp erim en tal treatm en t develop ed by a directly
resp ected p ediatric on cologist. Wh en th e (E) Recom m en d am ily cou n selin g
p aren ts h ear th at th e stu d y is ran d om ized ,
th ey re u se to h ave th e ch ild p articip ate. At 32. A p h ysician is treatin g a 17-year-old
th is tim e th e ch ild’s p h ysician sh ou ld p atien t with kidn ey ailu re. Th e d octor tells
(A) ran dom ize th e p atien t with ou t th e th e p atien t’s p aren ts th at, with ou t a kid n ey
p aren ts’ p erm ission tran sp lan t, sh e will die with in 3–4 weeks.
(B) o er th e stan d ard treatm en t on ly Th e on ly p erson wh ose kid n ey m atch es
(C) o er th e exp erim en tal treatm en t on ly or a tran sp lan t is th e p atien t’s 18-year-old
(D) o er a com b in ation o th e stan dard an d b roth er. However, wh en asked to don ate h is
th e exp erim en tal treatm en t kidn ey, th e broth er gets very u p set, cryin g
(E) re er th e ch ild to a di eren t p h ysician an d sh ou tin g an d re u ses to don ate to h is
sister. Th e doctor’s best cou rse o action at
30. An atten din g su rgeon asks a rst-year th is tim e is to
su rgery residen t to obtain in orm ed con sen t (A) tell th e p aren ts to p u t m ore p ressu re on
rom a p atien t or a su rgical p roced u re. Th e th e broth er to don ate
residen t kn ows little ab ou t th e su rgery or (B) p rescribe an an tian xiety agen t or
its risks an d com p lication s. At th is tim e, th e th e broth er an d th en ask h im abou t
residen t sh ou ld don atin g h is kidn ey
(A) ob tain con sen t an d an swer an y (C) ap p ly or a cou rt ord er to d o th e
qu estion s th at h e eels able to an swer at tran sp lan t
RISE USMLE NEPAL

th is tim e (D) accep t th e broth er’s decision an d do n ot


(B) ob tain con sen t an d tell th e p atien t to do th e tran sp lan t
talk to th e atten din g su rgeon later i sh e (E) ask to sp eak to th e b roth er alon e th e n ext
h as an y qu estion s day an d discu ss h is con cern s
(C) ob tain con sen t an d th en research an d
an swer th e p atien t’s qu estion s at a later
tim e
(D) ask th e atten d in g su rgeon to get con sen t
rom th e p atien t h im sel
(E) ask th e n u rse to get con sen t rom th e
p atien t
Chapter 23 Legal and Ethical Issues in Medicine 271

33. An 84-year-old m an with can cer 36. A 73-year-old p atien t h as been in th e


wh o is receivin g ch em oth erapy an d p ain in ten sive care un it or 2 weeks a ter a stroke.
m edication tells th e doctor th at h e was told Th e p atien t h as h ad a f at lin e EEG or 24
th at d eath rom can cer is very p ain u l. He h ours, shows n o corn eal ref exes, an d is on
th en asks th e d octor i h e can h ave extra p ain ven tilator su p port. Hosp ital records n ote th at
m edication i h e n eeds it in th e u tu re. At th is at age 53, th e p atien t stated th at h e wan ted
tim e, th e p h ysician sh ou ld all m easu res taken to con tin u e h is li e as lon g
(A) tell th e p atien t th at it is u n eth ical to as p ossible. His son , with wh om h e lives, says
p rescribe h igh doses o p ain m edication h is ather would wan t li e su pp ort on ly i h e
(B) ask th e p atien t wh at else h e was told could live a n orm al li e. His dau gh ter, wh o
(C) assu re th e p atien t th at h is d eath will n ot lives in an oth er state, wan ts everyth in g don e
HELP OTHERS SO THAT GOD WILL HELP YOU.

be p ain u l to con tin u e li e su p p ort. Th e p h ysician tests


(D) reassu re th e p atien t th at h e can h ave as th e p atien t’s brain stem ref exes, an d a ter
m u ch p ain m ed ication as h e n eeds 48 hou rs, li e sup p ort is discon tin u ed. Th is
(E) tell th e p atien t th at n ot all can cer action by the doctor is ju sti ed because
p atien ts exp erien ce extrem e p ain (A) th e son lived with th e p atien t
(B) th e son h ad exp ressed h is ath er’s wish es
34. A m ildly in tellectu ally disabled 16-year- (C) th e son , dau gh ter, an d eth ics com m ittee
old girl, wh o is 16 weeks p regn an t with a ch ild o th e h osp ital decid ed to stop li e
wh o has Down’s syn drom e, wan ts to keep su p p ort
the baby. She is healthy, an d the pregn an cy is (D) th e dau gh ter did n ot live with th e p atien t
u n com p licated. Her p aren ts wan t h er to h ave (E) th e p h ysician declared th e p atien t brain
an abortion . The m ost appropriate action or dead
the physician to take at this tim e is to
(A) re er th e am ily to an adop tion agen cy 37. A 19-year-old m an wh o is HIV p ositive
(B) acilitate discu ssion between th e girl an d tells h is p h ysician th at h e is regu larly
h er p aren ts h avin g u n p rotected sex with h is 18-year-old
(C) ollow th e p aren ts’ wish es an d do th e girl rien d (also a p atien t o th is p h ysician ).
abortion Th e girl rien d d oes n ot kn ow th e p atien t’s
(D) re u se to do th e abortion HIV statu s an d th e p atien t re u ses to tell
(E) con vin ce th e p aren ts to let th e girl keep h er. Th e p h ysician h as n oti ed state h ealth
th e ch ild au thorities abou t th e situ ation bu t can n ot
con rm th at th e state h as con tacted th e
35. A 30-year-old patient with Down’s girl rien d. At th is tim e, th e p h ysician sh ou ld
syn drom e who lives in an assisted living acility (A) in orm th e p olice abou t th e p atien t’s HIV
develops pneum onia and is success ully statu s
treated. The patien t’s m other, who rarely com es (B) keep th e in orm ation ab ou t th e p atien t’s
RISE USMLE NEPAL

to visit the patien t, leaves a voice m essage HIV statu s con id en tial
or the doctor asking or in orm ation about (C) in orm th e girl rien d o th e p atien t’s HIV
her daughter’s con dition . When the patien t statu s
is in orm ed about her m other’s request, she (D) in orm th e girl rien d’s p aren ts o th e
asks that the doctor n ot give her m other an y p atien t’s HIV statu s
in orm ation. Most appropriately, the doctor (E) advise th e girl rien d n ot to h ave
should call the m other back and tell her that u n p rotected sex with th e p atien t bu t do
(A) h e will give h er th e in orm ation th at sh e n ot tell h er wh y
h as requ ested
(B) h e can n ot reveal in orm ation ab ou t h is 38. A 15-year-old p atien t wh o h as con tracted
p atien t to h er gen ital h erp es con su lts h er am ily p h ysician .
(C) th e dau gh ter h as asked th at h e n ot reveal In ad dition to treatin g h er in ection , th e
su ch in orm ation to an yon e p h ysician sh ou ld
(D) h e will give h er th e in orm ation bu t (A) n oti y h er p aren ts
requ ests th at sh e n ot tell h er dau gh ter (B) n oti y h er sexu al p artn er
th at h e did so (C) get written p erm ission or treatm en t
(E) th e acility p erson n el will give h er th e rom h er p aren ts
in orm ation th at sh e h as requ ested at a (D) cou n sel h er on sa e sex p ractices
later date (E) rep ort th e case to state h ealth au th orities
272 BRS Behavioral Science

39. A clearly com p eten t 50-year-old wom an (C) get p erm ission rom th e wom an’s am ily
wh o h as religiou s b elie s th at p reclu d e to do th e tran s u sion
blood tran s u sion is sch ed u led or m ajor (D) give th e wom an th e tran s u sion bu t n ot
su rgery. Prior to th e su rgery, sh e states th at tell h er abou t it
th e p h ysician is n ot to give h er a b lood (E) give th e wom an th e tran s u sion an d
tran s u sion , alth ou gh sh e m ay n eed it d u rin g in orm h er o it wh en sh e recovers rom
su rgery. I a tran s u sion becom es n ecessary th e an esth etic
du rin g su rgery, th e p h ysician sh ou ld
(A) rep lace lost body lu id s b u t n ot give th e
wom an th e tran s u sion
(B) get a cou rt order to do th e tran s u sion
HELP OTHERS SO THAT GOD WILL HELP YOU.
RISE USMLE NEPAL
An swers an d Exp lan ation s

Typical Board Question


The answer is C. No, becau se wh en h e is an adu lt, th e ch ild can d ecid e wh eth er or n ot to b e tested
or HD. In d ivid u als at gen etic risk m u st d ecide wh eth er th ey wan t to h ave gen etic testin g to
determ in e i th ey will or will n ot develop a gen etic disease. Becau se HD can n ot be p reven ted
HELP OTHERS SO THAT GOD WILL HELP YOU.

or atten u ated by its id en ti ication d u rin g ch ild h ood an d becau se it typ ically m an i ests in adu lt-
h ood, th is decision is m ade by th e p erson at risk a ter h e or sh e reach es th e age o adu lth ood
(age 18 years). I th e illn ess starts in ch ildh ood an d sh e will be th e caretaker, th e m oth er h as th e
righ t to kn ow abou t h er ch ild’s risk o th e illn ess. Em otion al p rep aration or likelih ood o dep res-
sion can b e ad d ressed in th e adu lt. On ly on e p aren t n eed s to con sen t to gen etic testin g.

1. The answer is A. Con tact th e wom an an d tell h er th e resu lts. Abortion is legal in th e Un ited
States, an d th e p atien t, wh o is th e p regn an t wom an , h as th e righ t to decide i sh e wan ts to
rem ain p regn an t. It is also u p to th e wom an to decide wh eth er or n ot to tell h er h u sban d o
h er p lan or abortion .
2. The answer is E. Measles is a rep ortab le d isease an d so th e p h ysician m u st rep ort th e case
to th e state h ealth d ep artm en t. Vaccin ation o ch ild ren is recom m en d ed b u t n ot legally
requ ired so rep ortin g to th e state ch ild p rotective services agen cy or law en orcem en t
au th orities is n ot ap p rop riate. Alth ou gh th e doctor sh ou ld recom m en d th at th e ch ild be
kep t away rom oth er ch ildren , m easles is n ot in clu d ed am on g th e qu aran tain able illn esses
(see Section IV.B.3).
3. The answer is B. Paren ts can n ot re u se n ecessary an d stan dard treatm en t or a m in or ch ild
(b elow age 18 years) or an y reason . Th e p atien t’s wish es are n ot relevan t sin ce sh e is a
m in or. Ad u lt p atien ts can re u se tests or treatm en t or religiou s or oth er reason . Th e doctor’s
an ticip ation o a lawsu it i th e p atien t d ies is n ot a reason or givin g a treatm en t.
4. The answer is E. Never ask th e p atien t ou t. On ce a doctor is in volved in th e m edical care o
a p atien t, even i h e or sh e was n ot d irectly resp on sible or th at care, or exam p le, a m edi-
cal stu den t, th ere exists a sp ecial relation sh ip with th e p atien t. Th u s, wh ile som e sp ecialty
b oard s m ay sp eci y a sp eci ic n u m ber o years to wait, th e best an swer is n ever start a
RISE USMLE NEPAL

rom an tic relation sh ip with a p atien t.


5. The answer is A. As n oted ab ove, a p atien t (e.g., th e ath er) wh o is legally com p eten t can
re u se li esavin g treatm en t or h im sel or religiou s or oth er reason s, even i d eath will b e
th e ou tcom e. However, a p aren t (or gu ard ian ) can n ot re u se li esavin g treatm en t or th eir
ch ild or an y reason . Wh en th ere is tim e (h ere, 12 h ou rs), a cou rt ord er sh ou ld b e ob tain ed
b e ore treatm en t is started . In an em ergen cy, th e p h ysician can p roceed with ou t a cou rt
ord er.
6. The answer is D. Sin ce th ere is som e qu estion h ere abou t th e p atien t’s em otion al state,
evalu atin g h is su icid e risk be ore tellin g h im th e resu lts is th e best ch oice. On ly th e p h ysi-
cian (n ot a am ily m em ber) sh ou ld tell th e p atien t th e resu lts o a m ed ical test. I th e p h ysi-
cian believes th at th e p atien t’s h ealth will be adversely a ected by th e n ews o a m align an cy,
h e or sh e can d elay tellin g th e p atien t th e diagn osis u n til h e or sh e is ready to receive th e
b iop sy rep ort. Th e op in ion s o am ily m em bers m ay be h elp u l to in orm th e p h ysician
ab ou t th e p atien t’s state o m in d , b u t th e determ in ation o wh eth er an d wh en to in orm th e
p atien t m u st b e m ad e by th e p h ysician .

273
274 BRS Behavioral Science

7. The answer is C. On ly th e p aren t or legal gu ard ian can give con sen t or su rgical or m edi-
cal treatm en t o a m in or. In an em ergen cy su ch as th is, i th e p aren t or gu ard ian can n ot b e
located, treatm en t m ay p roceed with ou t con sen t. Th e babysittin g au n t an d gym teach er
h ave n o legal stan din g to m ake h ealth care decision s or th is ch ild. In th is case, waitin g to
act u n til th e p aren ts are reach ed cou ld b e h arm u l to th e ch ild.
8. The answer is C. Th e m ost ap p rop riate action or m edical stu den t X to take is to rep ort
m ed ical stu den t Y to th e dean o stu d en ts. Rep ortin g o an im p aired colleagu e is requ ired
eth ically b ecau se p atien ts m u st b e p rotected an d th e im p aired colleagu e sh ou ld be h elp ed .
Even i m edical stu d en t X asks Y wh y h e is d rin kin g or warn s Y abou t h is drin kin g, th ere is
n o gu aran tee th at Y will listen an d th at th e p atien ts will be p rotected. Rep ortin g Y to th e
p olice is n ot ap p rop riate.
HELP OTHERS SO THAT GOD WILL HELP YOU.

9. The answer is D. Th e m ost ap p rop riate action or th e in tern ist to take is to con tin u e to
re er p atien ts to th e su rgeon with ou t revealin g h is HIV statu s, p rovided th at th e su rgeon
is p h ysically an d m en tally com p eten t to treat p atien ts an d th at h e com p lies with stan dard
p recau tion s or in ection con trol. Ph ysician -to-p atien t tran sm ission o HIV h as n ever been
con irm ed in th e Un ited States. Ph ysician s are n ot requ ired to in orm eith er p atien ts or th e
m ed ical estab lish m en t ab ou t a colleagu e’s HIV-p ositive statu s.
10. The answer is A. It is legal or a p h ysician to re u se to treat a p atien t or a n u m ber o rea-
son s (e.g., th e p h ysician h as n o available tim e in h is or h er p ractice). However, a ederal
ap p eals cou rt h as ru led recen tly th at it is illegal u n d er th e Am erican s with Disab ilities Act
or a h ealth care worker to re u se to treat a p atien t with HIV du e to ear o tran sm ission . It
is also u n eth ical (bu t p robably is n ot illegal) or a p h ysician to re u se to m an age p atien ts
with oth er com m u n icable diseases.
11. The answer is B. I a com p eten t p atien t requ ests cessation o arti icial li e su p p ort, it is
both legal an d eth ical or a p h ysician to com p ly with th is requ est.
12. The answer is C. In th is case, th e p h ysician sh ou ld carry ou t th e p atien t’s requ est an d n ot
p rovide li e su p p ort. Th is decision is based on th e p atien t’s p rior in stru ction s as p u t orth
in a written d ocu m en t, i.e., a livin g will. Th e wi e’s or adu lt ch ildren’s wish es are n ot rel-
evan t to th is decision . Un d er th ese circu m stan ces, th e p atien t’s wish es are clear, an d th ere
is n o n eed to app roach th e cou rt or th e eth ics com m ittee o th e h osp ital.
13. The answer is A. Th e m ost ap p rop riate action or th e p h ysician to take is to with draw li e
su p p ort. I a p atien t is legally dead (b rain dead), th e p h ysician can rem ove li e su p p ort
with ou t a cou rt order or con sen t rom am ily.
14. The answer is A. Th e lawsu it will b e su ccess u l i th e p atien t can p rove th at th e p h ysician
RISE USMLE NEPAL

did n ot ollow th e u su al stan d ard s o p ro ession al care. An u n avorable ou tcom e alon e


(e.g., p aralysis o th e leg as an u n avoid ab le com p lication o th e su rgical p rocedu re) or n eg-
ative e ects on u n ction in g b ecau se o th e in ju ry do n ot con stitu te m alp ractice. Licen sed
p h ysician s are legally p erm itted to p er orm an y m edical or su rgical p rocedu re; th ey do n ot
h ave to b e boarded in a sp ecialty.
15. The answer is C. Th e b est reason th at th is p atien t can be h osp italized in volu n tarily is
i h e p oses a sign i ican t dan ger to h im sel or to oth ers. Tryin g to p u sh a p assen ger on to
th e tracks is su ch a d an ger. Sel -n eglect (e.g., p oor groom in g, m aln u trition ) or p sych otic
sym p tom s (e.g., h earin g voices or h avin g d elu sion s—see Ch ap ter 11) can also be grou n ds
or in volu n tary h osp italization wh en th ey con stitu te a sign i ican t, im m in en t dan ger to th is
p atien t’s li e or to oth ers.
16. The answer is D. Th e m ost ap p rop riate action or th e m edical stu d en t to take wh en a
p atien t asks h im to leave th e d elivery room is to ollow th e p atien t’s wish es. Th u s, th e stu -
den t sh ou ld in orm th e atten d in g p h ysician an d th en leave. Askin g th e residen t or p erm is-
sion or argu in g with th e p atien t (e.g., tellin g h er th at sh e m u st let h im stay b ecau se it is a
teach in g h osp ital or becau se sh e already agreed) is n ot ap p rop riate. Patien ts can re u se to
h ave train ees o an y kin d p resen t at an y tim e an d or an y reason .
Chapter 23 Legal and Ethical Issues in Medicine 275

17. The answer is D. Th e su rgeon sh ou ld exp lain th e risks o com p lication s rom both th e
su rgery an d an esth esia, in clu din g th e risk o d eath . Alth ou gh p atien ts sch ed u led or m ajor
su rgery are o ten worried, th ey h ave th e righ t to be in orm ed o all risks be ore givin g con -
sen t or a p rocedu re.
18. The answer is A. In th e absen ce o oth er in stru ction s (e.g., a DNR), th e p h ysician m u st
resu scitate th e p atien t. Askin g th e am ily or n u rsin g h om e sta wh at action to take is n ot
ap p rop riate.
19. The answer is D. Th e m ost ap p rop riate action or th e p h ysician to take is to n ote in th e
p atien t’s ch art that sh e h as re u sed to be tested an d con tin u e to care or h er. Alth ou gh p ro-
vidin g zid ovu d in e (AZT) an d / or n evirap in e (Viram u n e) to an HIV-p ositive wom an du rin g
HELP OTHERS SO THAT GOD WILL HELP YOU.

p regn an cy can sign i ican tly redu ce th e dan ger o HIV tran sm ission to th e u n born ch ild
(see Tab le 19.6), a p regn an t wom an h as th e righ t to re u se m edical tests or treatm en t even
i th e etu s will die or be seriou sly in ju red as a resu lt. A ter th e ch ild is born , th e m oth er
can n ot re u se to h ave it tested or treated or HIV.
20. The answer is C. Th e p h ysician’s best resp on se is, “I can n ot be you r p rim ary p h ysician
becau se I am you r em p loyer.” Excep t in em ergen cy situ ation s, p h ysician s sh ou ld n ot m an -
age th e care o am ily m em bers, close rien ds, or em p loyees sin ce p erson al eelin gs can
in ter ere with m edical decision m akin g. Also, su ch p atien ts are likely to be u n com ortable
an swerin g qu estion s con cern in g sen sitive in orm ation , or h avin g in tim ate p h ysical exam i-
n ation s wh en n eeded. Ph ysician s sh ou ld n ot treat p atien ts with ou t keep in g ap p rop riate
records n or sh ould th ey write p rescrip tion s or in dividu als oth er th an p atien ts.
21. The answer is D. Th e m ost ap p rop riate action or th e p h ysician to take is to deliver th e
ch ild vagin ally. Com p eten t p regn an t wom en , like all com p eten t ad u lts, can re u se m ed ical
treatm en t, even i th e etu s will d ie as a resu lt. Neith er th e p atien t’s h u sban d (even i h e is
th e ath er) n or th e cou rt h as th e righ t to alter th is decision . Tryin g to righ ten th e p atien t by
tellin g h er th at sh e can be crim in ally p rosecu ted i th e ch ild dies (u n tru e) or re errin g h er
to an oth er p h ysician are n ot ap p rop riate action s (see also an swer to Qu estion 19).
22. The answer is B. Hep atitis A is related b oth to p oor water qu ality an d to oral–an al con tact.
Th u s, a Mexican m an with a h om osexu al orien tation (wh o en gages in oral–an al sex) is
m ost likely to b e in ected with th is viru s.
23. The answer is C. In th e absen ce o a written or verbal advan ce directive, th e p riority order
in which am ily m em bers m ake th is determ in ation is th e (1) sp ouse, (2) adu lt ch ildren , (3)
p aren ts, (4) siblin gs, an d (5) other relatives. Th e act th at the sp ou se has a legal guardian
in dicates th at she h as been declared in com p eten t. Th ere ore, th e son m akes th e decision .
RISE USMLE NEPAL

Th e h osp ital eth ics com m ittee m ay be called in i th ere is a di eren ce between am ily
m em bers at th e sam e p riority level (n ot n ecessary in th is case).
24. The answer is B. Th is p h ysician is likely to b e im p aired an d th u s a p oten tial d an ger to
p atien ts. Th u s, th e colleagu e sh ou ld im m ediately n oti y th ose in ch arge at th e h osp ital th at
th e p h ysician is likely to be im p aired. Rep ortin g h im or sp eakin g to h im th e n ext day will
n ot p rotect p atien ts h e is likely to see th at even in g. Also, it m ay n ot be p ossible or th e col-
leagu e to p h ysically stop th e p h ysician rom seein g p atien ts.
25. The answer is E. It is n ot clear wh eth er or n ot th is p atien t with Down’s syn drom e u n der-
stan d s en ou gh ab ou t h er con d ition to give in orm ed con sen t. However, sin ce th ere is tim e
to m ake th is determ in ation , th e situ ation is n ot em ergen t. Th ere ore, th e p h ysician sh ou ld
evalu ate th e p atien t’s cap acity (with in p u t rom con su ltan ts i n ecessary).
26. The answer is E. Most ap p rop riately, th e su rgeon sh ou ld carry ou t th e op eration an d th en
sp eak to th e an esth esiologist alon e ab ou t h is in sen sitive beh avior. Th e eth ics com m ittee
does n ot h ave to be n oti ied i th e p atien t is n ot en d an gered an d i th e an esth esiologist’s
beh avior im p roves in th e u tu re. Askin g th e an esth esiologist to leave or an oth er su rgeon to
take over can p rolon g th e p rocedu re an d en dan ger th e an esth etized p atien t. It wou ld n ot
be h elp u l or p ro ession al to scold th e oth er doctor in a p u blic ven u e.
276 BRS Behavioral Science

27. The answer is E. With resp ect to th e p atern ity in din g, th e p h ysician sh ou ld n eith er write
it in th e ch art n or tell th e cou p le. Accordin g to th e Am erican Medical Association Code o
Medical Eth ics, it is n ot ap p rop riate or p h ysician s to d ivu lge in orm ation obtain ed seren -
dip itou sly in th e cou rse o gen etic testin g an d u n related to th e p u rp ose o th e testin g.
28. The answer is A. Most ap p rop riately, th is girl’s p aren ts sh ou ld be advised th at th e girl
sh ou ld n ot b e tested or th e ragile X gen e u n til sh e is rep rodu ctively m atu re an d requ ests
th e test or h ersel . Accordin g to th e Am erican Medical Association Code o Medical Eth ics,
“Gen etic testin g or carrier statu s sh ou ld be de erred u n til eith er th e ch ild reach es m atu -
rity, or th e ch ild n eeds to m ake rep rod u ctive d ecision s.”
29. The answer is B. Th e child’s p hysician sh ould o er the stan dard treatm en t on ly. Paren ts can
HELP OTHERS SO THAT GOD WILL HELP YOU.

re use experim en tal (but n ot stan dard, accepted) treatm en t o their child or an y reason .
30. The answer is D. Th e irst-year resid en t sh ou ld ask th e atten din g su rgeon to get con sen t
rom th e p atien t h im sel . Con sen t can n ot be obtain ed u n til th e p atien t h as been in orm ed
an d u n derstan ds th e h ealth im p lication s o h er d iagn oses, h ealth risks, an d b en e its o
treatm en t, th e altern atives to treatm en t, likely ou tcom e i sh e does n ot con sen t to th e
treatm en t, an d th at sh e can with draw con sen t or treatm en t at an y tim e. It is n ot ap p rop ri-
ate or th e resid en t (or th e n u rse) to get con sen t sin ce h e can n ot p rovid e th e p atien t with
th is in orm ation at th e tim e th at con sen t is obtain ed.
31. The answer is C. Keep th e diagn osis con id en tial as th e p aren ts h ave requ ested. Th is in or-
m ation does directly th reaten th e sister-in -law so th e p h ysician is n ot com p elled to reveal
th e diagn osis.
32. The answer is E. Th e d octor’s best cou rse o action at th is tim e is to ask to sp eak to th e
broth er alon e th e n ext d ay an d d iscu ss h is con cern s. Peop le m ay be righ ten ed at irst by
th e p rosp ect o organ don ation bu t m ay com e arou n d in tim e. Th e an tian xiety agen t will
n ot in crease th e ch an ce th at h e will com p ly. Ultim ately, h owever, it m ay be n ecessary to
accep t th e broth er’s decision n ot to d on ate. Sin ce h e is an adu lt, th e b roth er can n ot b e
com p elled to d on ate by eith er h is p aren ts or th e cou rt. Pu ttin g m ore p ressu re on h im is
u n likely to ch an ge h is m in d at th is tim e.
33. The answer is B. Th e p h ysician sh ou ld irst ask th e p atien t wh at else h e was told . Th e doc-
tor can th en address all o th e p atien t’s con cern s abou t h is illn ess an d its m an agem en t.
34. The answer is B. Th e m ost ap p rop riate action or th e p h ysician to take at th is tim e is to
acilitate discu ssion between th e girl an d h er p aren ts con cern in g th eir disagreem en t.
Becau se th e p regn an cy is n ot th reaten in g h er li e or h ealth , th e p aren ts can n ot orce th e
girl to h ave an abortion . However, h elp in g th e am ily to com e to an agreem en t on th is issu e
RISE USMLE NEPAL

is a better ch oice th an sim p ly re u sin g to do th e abortion or recom m en din g adop tion .


35. The answer is B. Most ap p rop riately, th e doctor sh ou ld tell th e m oth er th at h e can n ot reveal
in orm ation about his p atien t to her. It is n ot app rop riate to reveal in orm ation about an
adu lt p atien t (even a p erson with an in tellectu al disability) to an yon e without the patien t’s
con sen t. Addition ally, th e doctor sh ou ld n ot tell th e m oth er th at th e p atien t h as requ ested
su ch con iden tiality.
36. The answer is E. Discon tin uin g li e su pp ort is justi ied in this case because the physician
declared th e p atien t brain dead. A court order, ethics com m ittee decision , or relatives’ per-
m ission is n ot n ecessary in the decision to rem ove li e su pp ort when a patien t is brain dead.
37. The answer is C. I th e p atien t re u ses to tell h is girl rien d, th e p h ysician h ersel m u st n oti y
p u blic h ealth auth orities an d, i th ey do n ot act on th is in orm ation , in som e ju risdiction s,
also in orm th e en d an gered p artn er. I th e p atien t h ad agreed to tell h is girl rien d abou t
h is HIV statu s, the p h ysician sh ou ld set u p an ap p oin tm en t to see th e p atien t an d p art-
n er togeth er to en su re th at th e p atien t discloses h is HIV statu s to th e p artn er. Th e u su al
stan dards o p h ysician –p atien t con iden tiality do n ot ap p ly h ere sin ce th e p atien t’s ailu re
to u se con d om s p oses a sign i ican t th reat to h is girl rien d’s li e (Taraso decision ). Even i
th e p atien t is u sin g con dom s, th e p h ysician sh ou ld en cou rage h im to disclose h is m edical
con d ition to h is sexu al p artn er. Not all states requ ire rep ortin g o HIV-p ositive p atien ts.
Chapter 23 Legal and Ethical Issues in Medicine 277

38. The answer is D. In add ition to treatin g th e p atien t, th e p h ysician sh ou ld cou n sel h er
on sa e sexu al p ractices. Th ere is n o n eed to break p h ysician –p atien t con iden tiality by
tellin g th e sexu al p artn er, sin ce gen ital h erp es is n ot li e th reaten in g. Paren tal con sen t is
n ot requ ired or treatin g m in ors in cases o sexu ally tran sm itted disease, p regn an cy, an d
su b stan ce u se. Gen ital h erp es is n ot gen erally rep ortable to state h ealth au th orities.
39. The answer is A. Th e p h ysician can u se m ean s to rep lace lost b ody lu id s bu t sh ou ld n ot
give th e p atien t a blood tran s u sion . Legally com p eten t p atien ts m ay re u se treatm en t even
i death will result. Gettin g a cou rt order or obtain in g p erm ission rom th e wom an’s am ily
to do th e tran s usion is n ot ap p rop riate or eth ical. Failin g to tell a p atien t th e tru th (e.g.,
givin g th e wom an th e tran s u sion b u t n ot tellin g h er abou t it), or goin g again st a com p e-
ten t p atien t’s exp ressed wish es (e.g., in orm in g h er o th e tran s u sion wh en sh e recovers
HELP OTHERS SO THAT GOD WILL HELP YOU.

rom th e an esth etic), is n ever ap p rop riate.


RISE USMLE NEPAL
Health Care in th e
c ha pte r
24 Un ited States
HELP OTHERS SO THAT GOD WILL HELP YOU.

Typical Board Question


A 38-year-old m an with diab etes an d obesity h as ap p lied or h ealth in su ran ce or h im sel , h is
wi e (wh o h as m ajor d ep ressive disord er), an d h is th ree m in or ch ildren u n der th e “Patien t
Protection an d A ord able Care Act” (ACA, colloqu ially “Obam acare”). Th e am ily in com e is
too h igh or th e am ily to qu ali y or Med icaid . Wh en th is m an’s am ily gets h ealth coverage
u n der th e ACA, wh ich o th e ollowin g is least likely to be covered by th e h ealth p lan ?
(A) Care or h is diabetes
(B) His ch ildren’s den tal care
(C) His den tal care
(D) Men tal h ealth an d su bstan ce u se services
(E) Obesity screen in g an d cou n selin g
(See “An sw ers an d Explan ation s” at en d of ch apter.)

I. HEALTH CARE DELIVERY SYSTEMS


A. Hospitals
1. Accord in g to th e Am erican Hosp ital Association , th e Un ited States h as close to 6,000 h os-
p itals with abou t 1,000,000 beds. Cu rren tly, at least on e-th ird o h osp ital beds (esp ecially
RISE USMLE NEPAL

in city h osp itals) are u n occu p ied.


2. Th e average len gth o a hospital stay is 4.8 days an d h as been steadily decreasin g. Typ es o
h osp itals an d th eir own ersh ip are listed in Table 24.1.

B. Nursing homes an d oth er h ealth care acilities


1. In 2013, th ere were abou t 15,600 nursing homes with a cap acity o ab ou t 1.7 m illion b ed s in
th e Un ited States. In th e elderly, falls leading to broken bones (e.g., h ip ractu re) com m on ly
lead to th e n eed or n u rsin g h om e care (see Ch ap ter 3).
2. Reh abilitation cen ters, visitin g n u rses association s, an d h osp ices p rovide altern atives to
h osp ital an d n u rsin g h om e care (Table 24.2).

C. Physicians
1. Cu rren tly, th ere are m ore th an 140 accredited allopathic medical schools an d 30 accredited
schools of osteopathic medicine in th e Un ited States, an n u ally gradu atin g over 18,000
m ed ical d octors (MDs ) an d 5,000 doctors o osteop ath y (DOs ).
a. Du e to th e in creasin g n eed or p h ysician s, th e n u m ber o sch ools an d th e n u m ber o
stu d en ts en rolled in th ese sch ools are in creasin g.

278
Chapter 24 Health Care in the United States 279

t a b l e 24.1 Types of Hospitals in the United States

Type Approximate Number Category Includes

Community hospitals Total = 5,000 Nonfederal and short-term general and other 


• Nongovernment not-for-profit 2,894 special hospitals (e.g., obstetrics and gynecology; 
• Investor-owned (for-profit) 1,068 rehabilitation, orthopedic), and academic medical 
• State and local government 1,037 centers or other teaching hospitals accessible to the 
general public
Federal government hospitals 211 Veterans administration (VA) and military hospitals that 
are federally owned and reserved for individuals who 
have served (veterans) or are currently serving in 
the military
HELP OTHERS SO THAT GOD WILL HELP YOU.

Nonfederal psychiatric hospitals  444 Hospitals for chronically mentally ill patients


(often owned and operated by state 
governments)
Nonfederal long-term care hospitals 117 Hospitals for chronically physically ill patients
From Health, United States 2014. United States Department of Health and Human Services, Centers for Disease Control and Prevention, National Center
for Health Statistics, Table 98.

b. Both MDs an d DOs are correctly called “p h ysician s.” Th ere are cu rren tly m ore th an
1,000,000 p h ysician s in th e Un ited States o wh ich abou t 82% are active.
c. Train in g an d practice are essen tially the sam e or DOs as or MDs; however, the philoso-
phy o osteop athic m edicin e speci ically stresses th e interrelatedness of body systems an d
th e use o musculoskeletal manipulation in the diagn osis an d treatm en t o physical illn ess.
d. Overall, p h ysician s earn an average in com e o $200,000 annually. Psych iatrists, p ediatri-
cian s, an d am ily p ractition ers typ ically earn less th an th is average igu re an d su rgeon s
typ ically earn m ore.
2. Primary care p h ysician s, in clu d in g am ily p ractition ers, in tern ists, an d p ed iatrician s, p ro-
vide in itial care to p atien ts an d accou n t or at least on e-th ird o all p h ysician s. Th is n u m -
b er is in creasin g an d soon is exp ected to m ake u p on e-h al o all p h ysician s.
3. Th e ratio of physicians to patients is h igh er in th e n orth eastern states an d in Cali orn ia
th an in th e sou th ern an d m ou n tain states.
4. Peop le in th e Un ited States average fewer yearly visits to p h ysician s th an p eop le in devel-
op ed cou n tries with system s o govern m en t- u n ded m edical care.
5. Seven ty- ive p ercen t o p eop le visit p h ysician s in a given year. In all age grou p s, th e
two most common medical conditions or wh ich treatm en t is sou gh t are upper respiratory
ailments an d injuries .
RISE USMLE NEPAL

t a b l e 24.2 Other Health Care Facilities

Type of Care or Facility Services Provided Comments

Residential-assisted living  Long-term care Average costs range from about $36,000/y 


facility, intermediate Room and board (residential-assisted living facility) to at least 
care facility, and Assistance with self-care $75,000/y (skilled care facility), depending on 
nursing home (i.e.,  Nursing care geographical area
skilled care) facility
Rehabilitation centers and Short-term care Goal is to help hospitalized patients reenter society
halfway houses Room and board
Visiting nurse association Nursing care, physical and  Funded by Medicare
occupational therapy, and Serves as a less expensive alternative to 
social work services hospitalization or nursing home placement
Care given in a patient’s own home
Hospice organization Supportive care to terminally ill  Funded by Medicare
patients (i.e., those expected to  Goal is to allow patients to die at home to be with 
live, <6 mo) their families and preserve their dignity
Care usually given in a patient’s  Pain medication is used liberally
own home
280 BRS Behavioral Science

t a b l e 24.3   Health Care Expenses and Payment

Sources of Payment for Health Care Expenses


Origins of Health Care Expenses (% of Total) (% of total)

Hospitals and clinical services (38%) Private health insurance (34%)
Physicians’ fees (24%) Medicare (22%)
Medications and medical supplies (15%) Medicaid (17%)
Nursing homes (6%) Individuals (14%)
Dental care (5%) Other sources (13%)
Other care (12%)
HELP OTHERS SO THAT GOD WILL HELP YOU.

II. COSTS OF HEALTH CARE


A. Health care expenditures
1. Health care exp en ditu res in th e Un ited States m ake u p in excess o 14% of the total econ-
omy, m ore th an in an y oth er in du strialized society.
2. Health care expenditures have increased becau se o th e increasing age o th e p op u lation ,
advan ces in m ed ical tech n ology, an d th e availability o h ealth care to the p oor an d elderly
th rou gh Medicaid an d Medicare, resp ectively (see Section III.E. an d F.).

B. Allocation of health care funds. Th e origin s o h ealth care exp en ses an d th e sou rces o p ay-
m en t or h ealth care are listed in Table 24.3.

III. PAYMENT FOR HEALTH CARE: HEALTH INSURANCE


A. Overview
1. Th e Un ited States is on e o th e ew in d u strialized cou n tries th at do n ot h ave p u blicly
m an d ated h ealth care in su ran ce coverage u n ded by th e govern m en t or all citizen s. Th is
is on e reason th at th e United States h as higher infant mortality rates (see Figu re 1.1) an d
lower life expectancies th an m ost oth er d evelop ed cou n tries.
2. Most Am erican s m u st ob tain health insurance th rou gh th eir em p loyers or on th eir own .
a. Typ ically, th e em p loyer an d th e em p loyee sh are th e cost o h ealth in su ran ce.
b. Health in su ran ce costs cu rren tly average $12,000 per year or a am ily o ou r an d h ave
RISE USMLE NEPAL

b een in creasin g.
c. Man y Am erican s h ave no health insurance , wh ile m an y oth ers are “underinsured.” Su ch
p eop le m u st p ay all or p art o th e costs o h ealth care th em selves. Fi ty p ercen t o bank-
ruptcy ilin gs are related to th e in ability to p ay m ed ical exp en ses.
3. Man y Am erican s also cannot afford p rescrip tion dru gs, eyeglasses, or den tal treatm en t.
Th e n u m ber o su ch p eop le h as decreased with th e exp an sion o Medicaid th rou gh th e
ACA (see Section III.F).
4. Certain citizen s h ave govern m en t- u n d ed h ealth care in su ran ce th rou gh Medicaid an d
Medicare (see Section III.E.).
5. No m atter wh at th e in su ran ce, th e privacy of health information (e.g., in su ran ce claim s,
re erral au th orization requ ests) th at is h eld or tran sm itted in an y orm (e.g., p ap er, elec-
tron ic) is legally p rotected by th e Health Insurance Portability and Accountability Act (HIPAA)
im p lem en ted by th e Stan d ard s or Privacy o In dividu ally Iden ti iable Health In orm ation
(i.e., Privacy Ru le).

B. Private health insurers


1. Blue Cross/Blue Shield (BC/BS), a n on p ro it p rivate in su ran ce carrier, is regu lated by in su r-
an ce agen cies in each state.
Chapter 24 Health Care in the United States 281

a. BC/ BS p ays or h osp ital costs (Blu e Cross) an d p h ysician ees an d d iagn ostic tests
(Blu e Sh ield) or u p to h al o th e workin g p eop le in th e Un ited States.
b. Alm ost h al o BC/ BS su b scrib ers are en rolled in som e typ e o m an aged care p lan .
2. In dividu als can also con tract with on e o at least 1,000 oth er p rivate in su ran ce carriers,
su ch as Aetna or Prudential.

C. Fee-for-service care versus managed care


1. Wh ich ever th e in su ran ce carrier, p atien ts u su ally can ch oose between a tradition al ee-
or-service in dem n ity p lan an d at least on e typ e o m an aged care p lan .
a. A tradition al fee-for-service in dem n ity p lan h as n o restriction s on p rovid er ch oice or
re errals. It also com m on ly h as h igh er p rem iu m s.
HELP OTHERS SO THAT GOD WILL HELP YOU.

b. A managed care p lan h as restriction s on p rovid er ch oice an d re errals an d lower


p rem iu m s.
2. Man y in su ran ce p lan s h ave a deductible (i.e., th e am ou n t th e p atien t m u st p ay ou t o
p ocket be ore th e in su ran ce com p an y b egin s to cover exp en ses), a co-payment (i.e., a p er-
cen tage, typ ically 20%, o th e total bill th at th e p atien t m u st p ay), or both .

D. Managed care
1. Managed care describ es a h ealth care delivery system in wh ich all asp ects o an in di-
vidu al’s h ealth care are coordin ated or m an aged by a grou p o p roviders to en h an ce
cost-e ectiven ess.
2. Alth ou gh cost is con trolled in m an aged care, patients are restricted in th eir ch oice o a
p h ysician . Th u s, wh ile th e n u m ber o m an aged care p lan s is in creasin g, managed care is
more popular with the government th an with th e p u b lic.
3. Becau se ewer p atien t visits resu lt in lower costs, th e p h ilosop h y o m an aged care stresses
primary, secondary, an d tertiary prevention (Table 24.4) rath er th an acu te treatm en t.
4. Typ es o m an aged care p lan s in clu d in g h ealth m ain ten an ce organ ization s (HMOs ), p re-
erred p rovider organ ization s (PPOs ), an d p oin t o service (POS) p lan s are d escribed in
Table 24.5.

E. Federal- and state-funded insurance coverage


1. Medicare and Medicaid are govern m en t- u n ded p rogram s th at p rovide m edical in su ran ce
to certain grou p s o p eop le. Medicare Part D (ad ded in 2006) covers som e, b u t n ot all, p re-
scrip tion d ru g costs. Eligibility requ irem en ts an d coverage p rovided by th ese p rogram s
are ou tlin ed in Table 24.6.
2. Diagnosis-related groups (DRGs) are u sed by Medicare an d Medicaid to p ay h osp ital bills.
Th e am ou n t p aid is based on an estim ate o th e cost o h osp italization or each diagn osis
RISE USMLE NEPAL

rath er th an th e actu al ch arges in cu rred.

t a b l e 24.4   Primary, Secondary, and Tertiary Prevention in Health Care

Type of Prevention Goal Examples

Primary To reduce the incidence of a disorder by Immunization of infants to prevent infectious illnesses


reducing its associated risk factors Improved obstetrical care to avoid premature birth and 
its associated problems
Secondary To reduce the prevalence of an existing  Early identification and management of otitis media in 
disorder by decreasing its severity children to prevent hearing loss
Mammography for the early identification and 
management of breast cancer
Tertiary To reduce the prevalence of problems  Physical therapy for stroke patients so that they can
caused by an existing disorder care for themselves
Occupational training for intellectually disabled 
persons so that they can gain the skills needed to 
join the work force
282 BRS Behavioral Science

t a b l e 24.5   Managed Care Plans

Type of Plan Definition Comments

Health Maintenance Physicians and other health care personnel are paid These plans are the most restrictive 
Organization (HMO)  a salary to provide medical services to a group of  for the patient in terms of choice
(staff model or closed  people who are enrolled voluntarily and who pay  of doctor
panel) an annual premium Patient is assigned a “gatekeeper” 
HMOs may operate their own hospitals and clinics (a primary care doctor from 
Services include hospitalization, physician services,  within the network who decides if 
preventive medicine services, and often dental,  and when a patient needs to see 
eye, and podiatric care a specialist)
HMO (independent practice  Physicians in private practice are hired by an HMO  Private practice physicians receive 
HELP OTHERS SO THAT GOD WILL HELP YOU.

association [IPA] model) to provide services to HMO patients a fee, or capitation, for each HMO


About 65% of HMOs have IPA components patient they are responsible for
Preferred provider  A third-party payer (e.g., a union trust fund,  These plans guarantee doctors in 
organization (PPO) insurance company, or corporation) contracts  private practice a certain volume 
with physicians in private practice and  of patients
with hospitals to provide medical care to its  By paying a larger share of the cost, 
subscribers patients can choose a doctor
Participants choose physicians from a list of who is not in the network
member practitioners (the network) There is no “gatekeeper” physician
Physicians in the network receive capitation for 
each patient
Point of service plan (POS) Variant of a PPO in which a third-party payer  As with a PPO, patients can choose 
contracts with physicians in private practice to  a doctor who is not in the 
provide medical care to its subscribers network by paying an extra fee
Physicians in the network receive capitation for  As with an HMO, there is a 
each patient “gatekeeper” physician

F. The Patient Protection and Affordable Care Act (ACA, colloquially Obamacare)
1. In 2010, Con gress p assed th e ACA, wh ich u ses a com bin ation o govern m en t u n din g an d
p rivate in su ran ce to p ay or h ealth care.
2. A ter th e ACA cam e in to e ect (a ter 2012), th e p ercen tage o u n in su red Am erican s
drop p ed rom 16.7% in 2013 to 11% in 2015.

t a b l e 24.6 Medicare and Medicaid

Source of Funding Eligibility Coverage


RISE USMLE NEPAL

Medicare
The federal government  People eligible for Social  Part A: Inpatient hospital care, home health care, 
(through the Social  Security benefits (e.g., those  medically necessary nursing home care (for up to 
Security system) at least 65 y of age regardless  90 days after hospitalization), hospice care
of income) Part B: Physician fees, dialysis, physical therapy, 
People of any age with chronic  laboratory tests, ambulance service, medical 
disabilities or debilitating  equipment (Part B is optional and has a 20% 
illnesses co-payment and a $100 deductible)
Covers about 49 million people Part D: Is optional and covers a share of prescription 
drug costs
Medicare does not cover long-term nursing home care
Medicaid (MediCal in California)
Both federal and state Indigent (very low-income)  Inpatient and outpatient hospital care
governments (the  people Physician services
state contribution is One-third of all funds is allocated  Home health care, e.g., hospice care, laboratory tests, 
determined by average  for nursing home care for  dialysis, ambulance service, medical equipment
per capita income of indigent elderly people Prescription drugs
the state) Covers about 70 million people Long-term nursing home care
Dental care, eyeglasses, hearing aids
No co-payment or deductible
Chapter 24 Health Care in the United States 283

3. Th e ACA in clu des:


a. Exp an d ed eligibility or Medicaid (e.g., it n ow can cover sin gle adu lts with ou t disabili-
ties or am ilies).
b. Gu aran teed coverage to in dividu als with preexisting conditions.
c. A penalty or th ose n ot covered by an in su ran ce p lan wh o do n ot bu y h ealth care
coverage.
d. Health in su ran ce exchanges , wh ich com p are in su ran ce p olicies an d sell th em .
e. Federal subsidies or low-in com e p erson s to h elp th em b u y h ealth in su ran ce.
4. Wh ile sp eci ic b en e its vary by state, in all states, th e ACA covers:
a. Hosp ital, ou tp atien t, em ergen cy room , p ren atal an d m atern ity care.
b. Prescrip tion d ru gs.
HELP OTHERS SO THAT GOD WILL HELP YOU.

c. Ph ysical an d occu p ation al th erapy.


d. Laboratory testin g.
e. Preven tive services, or exam p le, ob esity screen in g an d cou n selin g.
f. Men tal h ealth an d su bstan ce u se services.
g. Den tal an d vision care or ch ildren .

IV. DEMOGRAPHICS OF HEALTH IN THE UNITED STATES


A. Lifestyle, habits, and attitudes
1. Li estyle an d p oor dietary an d oth er h abits, p articu larly sm okin g an d drin kin g alcoh ol,
are resp on sib le or m u ch physical and mental illness .
2. Social attitudes in volvin g h ealth care issu es also a ect h ealth care delivery. For exam p le,
alth ou gh organ tran sp lan ts can save m an y lives, ewer tran sp lan t p rocedu res are don e
th an are n eeded. Th is is du e largely to th e act th at n ot en ou gh p eop le are willin g to
don ate th eir organ s at death .

B. Socioeconomic status (SES) and health


1. SES, wh ich is determ in ed by occupation and educational level as well as p lace o resid en ce
an d in com e, is directly associated with m en tal an d p h ysical h ealth .
2. Am erican s with low SES are m ore likely to b e A rican Am erican or Latin o (see Ch ap ter 18).
a. High SES patients are m ore likely to seek treatment an d to visit p rivate doctor’s o ices
th an are low SES p atien ts.
b. Low SES patients are m ore likely to seek treatm en t in h osp ital em ergen cy dep artm en ts
an d to delay seekin g treatm en t, in p art becau se o th e cost. Illn esses o ten becom e
RISE USMLE NEPAL

m ore severe wh en p atien ts d elay seekin g treatm en t.


3. Hosp itals are legally requ ired to p rovid e care to an yon e n eedin g em ergen cy m an agem en t
wh eth er th ey h ave th e m ean s to p ay or n ot via th e ederal “Em ergen cy Medical Treatm en t
an d Active Lab or Act” (EMTALA).

C. Gender and health


1. Women are more likely to seek medical treatment th an m en .
2. Men h ave sh orter li e exp ectan cies an d are m ore likely to h ave h eart d isease th an
wom en .
3. Wh ile th ey h ave less h eart d isease overall, women are m ore likely th an m en to die during
their first heart attack or to d ie d u rin g th e year a ter a h eart attack.
4. Wom en are also at h igh er risk th an m en or develop in g:
a. Au toim m u n e diseases (e.g., rh eu m atoid arth ritis).
b. Mu ltip le sclerosis.
c. Alcoh ol- an d sm okin g-related illn esses.
d. AIDS (wh en th ey are alread y HIV p ositive an d h ave th e sam e viral load as a m an ).
e. Cataracts.
f. Th yroid disease.
284 BRS Behavioral Science

D. Age and health


1. Ch ild ren an d th e eld erly are m ore likely to requ ire m edical care th an p eop le o oth er ages.
2. Th e elderly com p rise abou t 12% o th e p op u lation b u t in cu r m ore th an 30% of all health
care costs ; th is igu re is exp ected to rise to 50% by th e year 2020.
3. Th e lead in g cau ses o d eath d i er by age grou p (Table 24.7) an d h ave ch an ged in th e
last ew years. In p articu lar, th e th ird leadin g cau se o death in th e elderly an d across age
grou p s (a ter h eart d isease an d can cer) h ad been stroke an d related disorders bu t is n ow
chronic lower respiratory diseases.

24.7
HELP OTHERS SO THAT GOD WILL HELP YOU.

t a b l e   Leading Causes of Death by Age Group (across Sex and Ethnic Group)

Age Group Causes of Death (in Decreasing Order of Frequency)

Infants (<1 y of age) Congenital anomalies
Prematurity/low birth weight
Sudden infant death syndrome (SIDS)
Children (1–4 y of age) Accidents (in motor vehicles and in the home)
Congenital anomalies
Cancer (primarily leukemia and central nervous system [CNS] malignancies)
Children (5–14 y of age) Accidents (most in motor vehicles)
Cancer (primarily leukemia and CNS malignancies)
Suicide
Adolescents and young adults  Accidents (most in motor vehicles)
(15–24 y of age) Suicide
Homicide and legal intervention
Adults (25–44 y of age) Accidents (most in motor vehicles)
Cancer
Heart disease
Adults (45–64 y of age) Cancer
Heart disease
Accidents (most in motor vehicles)
Elderly (65 y of age and over) Heart disease
Cancer
Chronic lower respiratory diseases
All ages combined Heart disease
Cancer (lung, breast/prostate, and colorectal, in decreasing order)
Chronic lower respiratory diseases
RISE USMLE NEPAL
Review Test

Directions: Each o th e n u m b ered item s or in com p lete statem en ts in th is section is ollowed by


an swers or by com p letion s o th e statem en t. Select th e one lettered an swer or com p letion th at
is best in each case.

1. A su ccess u l su rgeon own s both a h osp ital Questions 5 and 6


HELP OTHERS SO THAT GOD WILL HELP YOU.

an d an ou tp atien t d iagn ostic cen ter. Wh ich


o th e ollowin g ch arges will b rin g th e A irst-year residen t wh o h as recen tly started
su rgeon th e m ost m on ey each year? workin g in a h osp ital em ergen cy dep artm en t
(A) Hosp ital ch arges sees ou r p atien ts d u rin g h is irst h ou r on th e
(B) Su rgical ees service.
(C) Ou tp atien t diagn ostic testin g ch arges
5. Wh ich o th ese p atien ts is likely to be
(D) Medication ch arges
h ealth iest wh en f rst seen by th e residen t?
(E) Reh abilitation ch arges
(A) A 45-year-old m an rom a low
2. A wealth y wom an wou ld like to d on ate socioecon om ic grou p
m on ey to u n d h ealth care or p eop le wh o (B) A 45-year-old wom an rom a low
are n eedy. Th e n eed iest p atien ts are m ost socioecon om ic grou p
likely to be wh ich o th e ollowin g? (C) A 45-year-old m an rom a h igh
socioecon om ic grou p
(A) Cardiac p atien ts
(D) A 45-year-old wom an rom a h igh
(B) Breast can cer p atien ts
socioecon om ic grou p
(C) Hom eless p atien ts
(D) Very you n g p atien ts 6. Wh ich o th ese p atien ts is likely to be th e
(E) Elderly p atien ts m ost ill wh en f rst seen by th e residen t?

3. A 79-year-old wom an wh o lives in h er (A) A 45-year-old m an rom a low


own h om e an d is in good h ealth h as ju st socioecon om ic grou p
been d iagn osed with osteop orosis. To h elp (B) A 45-year-old wom an rom a low
p reven t ractu res, th e p h ysician sh ou ld f rst socioecon om ic grou p
recom m en d th at th is p atien t (C) A 45-year-old m an rom a h igh
socioecon om ic grou p
(A) in itiate sa ety m easu res in (i.e., sa e- (D) A 45-year-old wom an rom a h igh
RISE USMLE NEPAL

p roo ) th e h om e socioecon om ic grou p


(B) take calciu m su p p lem en ts
(C) take alen dron ate sodiu m (Fosam ax) 7. In th e Un ited States, th e p ercen tage o th e
(D) begin a regu lar exercise p rogram gross dom estic p rodu ct sp en t on h ealth care
(E) in crease dairy p rodu cts in th e diet is cu rren tly abou t
(F) ap p ly or ad m ission to an assisted livin g
(A) 1%
acility
(B) 8%
(C) 14%
4. O th e ollowin g p atien ts, th e on e likely
(D) 30%
to use th e least Med icare services an d u n ds
(E) 50%
du rin g h is or h er li etim e is a(n )
(A) A rican Am erican m an 8. In th e Un ited States, th e largest
(B) A rican Am erican wom an p ercen tage o p erson al h ealth care exp en ses
(C) Wh ite m an is paid by wh ich o th e ollowin g sou rces?
(D) Wh ite wom an (A) Medicare
(E) Latin o m an (B) Medicaid
(C) Mu n icip al govern m en ts
(D) Private h ealth in su ran ce
(E) Person al u n ds
285
286 BRS Behavioral Science

9. In th e Un ited States, th e largest 13. Wh ich o th e ollowin g are th e th ree


p ercen tage o h ealth care exp en ditu res is or leadin g cau ses o d eath in th e Un ited States
(A) p h ysician ees in ord er o m agn itu d e (h igh er to lower)?
(B) n u rsin g h om e care (A) Ch ron ic lower resp iratory d iseases, h eart
(C) m ed ication s disease, can cer
(D) h osp ital an d clin ical services (B) Heart disease, can cer, ch ron ic lower
(E) den tal care resp iratory diseases
(C) Can cer, h eart disease, AIDS
10. A m oth er brin gs h er 2-m on th -old (D) Heart disease, can cer, stroke
daugh ter to th e p h ysician or a ch ecku p. In (E) Stroke, h eart disease, can cer
th e Un ited States, th e m ost com m on cau se
HELP OTHERS SO THAT GOD WILL HELP YOU.

o death in in an ts b etween b irth an d 1 year 14. In wom en in th e Un ited States, wh ich is


o age is th e m ost com m on cau se o can cer death ?
(A) leu kem ia (A) Cervical can cer
(B) su d den in an t d eath syn d rom e (SIDS) (B) Colorectal can cer
(C) con gen ital an om alies (C) Breast can cer
(D) acciden ts (D) Lu n g can cer
(E) resp iratory distress syn drom e (E) Uterin e can cer

Questions 11 and 12 15. Most p atien ts in th e Un ited States


can exp ect to receive care in wh ich o th e
A 79-year-old em ale p atien t is h osp italized ollowin g typ es o h osp ital?
or a ractu red h ip. Th e p atien t, wh o h as (A) Federal govern m en t
$100,000 in savin gs, was b rou gh t to th e h os- (B) Non govern m en t n ot- or-p ro it
p ital by am b u lan ce. Sh e stayed in th e h osp ital (C) In vestor-own ed
or 5 days an d, wh en released, requ ired p h ys- (D) Local govern m en t
ical th erapy an d a walker or h elp with m obil- (E) State govern m en t
ity or th e n ext 6 weeks.
16. An edu cation al p rogram is develop ed
11. Th is p atien t can exp ect th at Medicare to teach m en tally ill ad u lts skills n ecessary
Part A will cover wh ich o th e ollowin g costs to h elp th em get an d keep p ayin g job s. Th is
related to th is in ju ry? p rogram is an exam p le o
(A) In p atien t h osp ital care (A) p rim ary p reven tion
(B) Th e walker (B) secon dary p reven tion
(C) Am b u lan ce service (C) tertiary p reven tion
(D) Ph ysician bills (D) m an aged care
(E) Ph ysical th erapy
RISE USMLE NEPAL

17. Paren ts brin g th eir 2-year-old ch ild to


12. A ter 6 m on th s at h om e, it is determ in ed a well-ch ild clin ic. In order to p rotect th e
th at th is p atien t is u n ab le to care or h ersel ch ild’s li e an d h ealth , th e m ost im p ortan t
an d requ ires care in a residen tial n u rsin g su ggestion th e p h ysician can m ake is to tell
h om e acility, p robab ly or th e rest o h er li e. th e p aren ts to
Wh ich o th e ollowin g will p ay or th e f rst
(A) keep ip ecac in th e m ed icin e cab in et
ew years o th is care?
(B) p u t sm oke alarm s in th e h om e
(A) Medicare Part A (C) in itiate sa ety m easu res in (i.e., sa e-
(B) Medicare Part B p roo ) th e h om e
(C) Blu e Cross (D) learn cardiop u lm on ary resu scitation
(D) Blu e Sh ield (E) h ave th e ch ild im m u n ized again st
(E) Th e p atien t’s savin gs m easles, m u m p s, an d ru b ella
Chapter 24 Health Care in the United States 287

18. Paren ts b rin g th eir 8-year-old son in or Questions 19 and 20


a sch ool p h ysical. Th e p aren ts relate th at th e
ch ild is d oin g well in sch ool. Th ey also tell A 45-year-old stockbroker with th ree ch ildren
th e doctor th at a ter sch ool th ey drive th eir m u st ch oose a h ealth in su ran ce p lan at work.
son to soccer p ractice, an d on weeken ds,
th ey all go swim m in g in th e cou n ty p ool. Th e 19. In which o the ollowin g plan s will she
ch ild also likes to rid e h is bicycle arou n d th e have the m ost choice in choosin g a physician ?
n eigh borh ood . Th e p aren ts also reveal th at, (A) A h ealth m ain ten an ce organ ization
wh ile th ey are tryin g to cu t back, b oth o (HMO)
th em sm oke cigarettes in th e h om e. In ord er (B) A p re erred p rovider organ ization (PPO)
to p rotect th e ch ild’s li e an d h ealth , wh at is (C) A p oin t o service (POS) p lan
HELP OTHERS SO THAT GOD WILL HELP YOU.

th e m ost im p ortan t su ggestion th e p h ysician (D) A ee- or-service p lan


can give th e p aren ts?
(A) Stop sm okin g to redu ce th e ch ild’s 20. In wh ich o th e ollowin g p lan s will sh e
exp osu re to secon dh an d sm oke h ave the least choice in choosin g a p hysician ?
(B) Pu t sm oke alarm s in th e h om e (A) A h ealth m ain ten an ce organ ization
(C) Have th e ch ild wear a h elm et wh ile (HMO)
bicyclin g (B) A p re erred p rovider organ ization (PPO)
(D) Learn cardiop u lm on ary resu scitation (C) A p oin t o service (POS) p lan
(E) En su re th at th e ch ild wears a seat b elt in (D) A ee- or-service p lan
th e car
RISE USMLE NEPAL
An swers an d Exp lan ation s

Typical Board Question


The answer is C. Alth ou gh h is m in or ch ildren’s den tal care is con sidered an “essen tial h ealth
ben e it” an d is covered u n der th e ACA, th e ACA is u n likely to cover den tal care or th is 38- year-old
m an . I h e were eligible or Medicaid h is den tal care wou ld also be covered. Care or p reexistin g
HELP OTHERS SO THAT GOD WILL HELP YOU.

con dition s su ch as h is d iab etes, p reven tive care su ch as obesity screen in g an d cou n selin g, an d
m en tal h ealth care or h is wi e are also all con sid ered “essen tial h ealth ben e its” an d as su ch are
covered u n der th e ACA.

1. The answer is A. Hosp ital ch arges will b rin g th e su rgeon th e m ost m on ey each year. Th e
h igh est h ealth care costs in volve su ch ch arges. Doctor’s ch arges su ch as su rgical ees are
secon d in h ealth care costs an d m edication ch arges are th ird (see Table 24.3). Ou tp atien t
d iagn ostic testin g ch arges an d reh ab ilitation ch arges are lower.
2. The answer is C. Fin an cially, th e n eediest p atien ts are m ost likely to be h om eless p erson s.
Cardiac, b reast can cer, an d very you n g p atien ts are m ore likely th an th e h om eless to h ave
h ealth in su ran ce to p ay or th eir m edical costs. Peop le age 65+ typ ically h ave h ealth care
costs covered by Medicare.
3. The answer is A. Th e m ost im p ortan t recom m en d ation or th e p h ysician to m ake at
th is tim e to p reven t ractu res in th is wom an with osteop orosis is to sa e-p roo th e h om e
en viron m en t to red u ce th e likelih ood o alls (e.g., rem ove scatter ru gs, in stall sh ower grab
b ars) (see Ch ap ter 3). Calciu m su p p lem en ts, m edication s su ch as alen dron ate sodiu m
(Fosam ax), exercise, an d in creasin g d airy p rod u cts in th e diet are all im p ortan t or p rop h y-
laxis in osteop orosis; n on e will h elp p reven t ractu res in th e sh ort term . Sin ce th is p atien t
d oes well livin g on h er own , th ere is n o reason or h er to m ove to an assisted livin g acility.
4. The answer is A. Medicare p ays or h ealth care services or person s 65 years o age an d older
an d oth ers wh o are eligible to receive Social Security ben e its. These ben e its con tin ue or
th e li e o th e in dividu al. Becau se statistically h e is likely to have a sh orter li e than a White
or Latin o m an , an A rican Am erican wom an , or a Wh ite wom an , an A rican Am erican m an
is likely to u se th e least Medicare services over the course o his li etim e (see Table 3.1).
RISE USMLE NEPAL

5. The answer is D. 6. The answer is A. A wom an rom a h igh socioecon om ic grou p is likely to
b e h ealth iest wh en th e residen t irst sees h er. Wom en an d p eop le rom h igh er socioeco-
n om ic grou p s are m ore likely to seek treatm en t an d th ere ore to b e less ill wh en irst seen
by a p h ysician th an m en an d p eop le rom low socioecon om ic grou p s. A m an rom a low
socioecon om ic grou p is likely to be m ost ill wh en th e residen t irst sees h im . Low-in com e
p atien ts an d m ale p atien ts are m ore likely to d elay seekin g treatm en t. Delay in seekin g
treatm en t com m on ly resu lts in m ore severe illn ess.
7. The answer is C. Th e p ercen ta ge o th e gro ss d om estic p rod u ct (GDP) sp en t on h ealth
ca re is a b o u t 14%, a p ercen ta ge th a t is la rger th a n th at o a n y oth er d evelop ed cou n try.
8. The answer is D. Th e largest p ercen tage o p erson al h ealth care exp en ses is p aid by p rivate
h ealth in su ran ce. In decreasin g order, oth er sou rces o p aym en t or h ealth care exp en ses
are Medicare, Medicaid, an d p erson al u n ds. Mu n icip al govern m en ts p ay a relatively sm all
p ercen tage o th ese exp en ses.
9. The answer is D. In th e Un ited States, m ost h ealth care exp en ditu res are or h osp ital care.
In decreasin g order, oth er sou rces o h ealth care exp en ses are p h ysician ees, m edication s,
n u rsin g h om e care, an d d en tal services (an d see Qu estion 1).

288
Chapter 24 Health Care in the United States 289

10. The answer is C. Th e m ost com m on cau se o death in in an ts u p to 1 year o age is con -
gen ital an om alies. Prem atu rity/ low birth weigh t an d su dden in an t death syn drom e (SIDS)
are th e secon d an d th ird leadin g cau ses o d eath in th is age grou p.
11. The answer is A. 12. The answer is E. Medicare Part A will cover in p atien t h osp ital costs.
Part B covers am b u lan ce services, p h ysician ees, m edical equ ip m en t (th e walker), an d
th erapy. Th e p atien t h ersel is resp on sible or lon g-term resid en tial n u rsin g h om e acility
costs sin ce n eith er Part A or Part B o Med icare n or Blu e Cross/ Blu e Sh ield will cover su ch
costs. A ter th e p atien t’s $100,000 is exh au sted (p robably with in 1.5 years at abou t $75,000
p er year), sh e will b e in digen t an d th ere ore eligible or Medicaid. Medicaid p ays or resi-
d en tial n u rsin g h om e care an d all oth er h ealth care or in digen t p eop le.
HELP OTHERS SO THAT GOD WILL HELP YOU.

13. The answer is B. Th e lead in g cau se o death in th e Un ited States is h eart d isease, ollowed
by can cer an d ch ron ic lower resp iratory d iseases.
14. The answer is D. In wom en , as in m en , th e m ost com m on cau se o can cer death in th e
Un ited States is can cer o th e lu n g. In wom en , th is is ollowed by b reast can cer an d
colorectal can cer. Th e n u m b er o wom en gettin g lu n g can cer is in creasin g with in creased
sm okin g rates in wom en .
15. The answer is B. More p atien ts in th e Un ited States receive care in n on govern m en t, n ot-
or-p ro it h osp itals th an in ed eral, state, local-govern m en t, or in vestor-own ed h osp itals.
16. The answer is C. Th is edu cation al p rogram or adu lts with m en tal illn ess is an exam p le o
tertiary p reven tion . Tertiary p reven tion is aim ed at redu cin g th e p revalen ce o p roblem s
cau sed by an existin g disord er, m en tal illn ess in th is case. Prim ary p reven tion is aim ed at
redu cin g th e occu rren ce or in ciden ce o a disorder by redu cin g its associated risk ac-
tors (e.g., im m u n ization again st m easles). Secon dary p reven tion is aim ed at redu cin g th e
p revalen ce o an existin g disord er by redu cin g its severity (e.g., early iden ti ication an d
m an agem en t o breast can cer u sin g m am m ograp h y). Man aged care is a system o h ealth
care in wh ich all asp ects o h ealth care are coordin ated by p roviders to con trol costs.
17. The answer is C. Wh ile acciden tal p oison in g, h ou se ires, an d d rown in g are cau ses o death
in ch ild ren , in ch ildren 1–4 years o age, acciden ts in th e h om e are a m ore im p ortan t cau se
o accid en tal death . In ectiou s illn ess d u e to lack o im m u n ization rarely cau ses d eath in
Am erican ch ildren (see also Qu estion 18).
18. The answer is E. While house ires, bicyclin g acciden ts, an d drown in g cause acciden tal death
in children , ailure to wear seat belts is the m ajor cause o acciden tal death in children 4–14
years o age. While it h as been associated with in creased childhood upper respiratory sym p-
tom s, secon dh an d sm oke h as n ot been sh own to sign i ican tly a ect su rvival in children .
RISE USMLE NEPAL

19. The answer is D. 20. The answer is A. Patien ts h ave th e m ost ch oice in ch oosin g a p h ysician
in a tradition al ee- or-service in d em n ity p lan . In th is typ e o p lan , th ere are n o restriction s
on p rovider ch oice or re errals. Man aged care p lan s (e.g., h ealth m ain ten an ce organ iza-
tion s [HMOs], p re erred p rovid er organ ization s [PPOs], an d p oin t o service [POS] p lan s)
h ave restriction s on p h ysician ch oice. Patien ts h ave th e least ch oice in ch oosin g a p h ysi-
cian in an HMO. HMOs are th e m ost restrictive o m an aged care p lan s or th e p atien t in
term s o ch oice o p h ysician . Rath er th an ch oosin g a p h ysician rom th e n etwork as in a
PPO or POS, in an HMO, th e p atien t is assign ed a p h ysician .
c ha pte r
25 Medical Ep idem iology
HELP OTHERS SO THAT GOD WILL HELP YOU.

Typical Board Question


In a clin ical trial, a n ew an tih istam in e was com p ared to a p lacebo. Wh en th e stu dy was com -
p leted, th e in din gs, wh ile p rom isin g, did n ot reach statistical sign i ican ce. Th e research ers
were advised to in crease th e sam p le size. Wh ich o th e ollowin g wou ld b e th e b est way or
th e research ers to in crease th e sam p le size with ou t recru itin g m ore su bjects?
(A) Ran dom ize th e design
(B) Use a crossover d esign
(C) Use a dou b le-b lin d d esign
(D) Give all o th e con trols th e n ew an tih istam in e
(E) Th e sam p le size can n ot be in creased with ou t recru itin g m ore su bjects
(See “An sw ers an d Explan ation s” at en d of ch apter.)

I. MEDICAL EPIDEMIOLOGY: INCIDENCE AND PREVALENCE


Medical ep id em iology is th e stu d y o th e actors a ectin g th e occu rren ce an d d istrib u tion o
diseases in h u m an p op u lation s.

A. Incidence. Incidence rate is a ratio o th e n u m ber o in dividu als in th e p op u lation wh o


RISE USMLE NEPAL

develop an illness in a given tim e p eriod (com m on ly 1 year) divided by th e total n u m ber o
in d ivid u als at risk or th e illn ess d u rin g th at tim e p eriod (e.g., th e n u m b er o IV d ru g u sers
n ewly d iagn osed with AIDS in 2016 divided by th e n u m ber o IV dru g u sers in th e p op u la-
tion du rin g 2016).

B. Prevalence. Prevalence rate is a ratio o th e n u m ber o in dividu als in th e p op u lation wh o


have an illness (e.g., AIDS) d ivid ed by th e total n u m ber o in dividu als at risk or th e illn ess.
1. Point prevalence is a ratio o th e n u m b er o in dividu als wh o h ave an illn ess at a sp eci ic
point in time (e.g., th e n u m b er o p eop le wh o h ave AIDS on Au gu st 31, 2016, divided by th e
total p op u lation wh o cou ld h ave th e illn ess on th at date).
2. Period prevalence is a ratio o th e n u m b er o in dividu als wh o h ave an illn ess du rin g a
sp eci ic time period (e.g., th e n u m ber o p eop le wh o h ave AIDS in 2016 divided by th e total
p op u lation wh o cou ld h ave th e illn ess m id -year in 2016).

C. Relationship between incidence and prevalence


1. Prevalen ce rate is equ al to in cid en ce rate m u ltip lied by th e average duration o th e disease
p rocess (i in ciden ce an d d u ration are stable).

290
Chapter 25 Medical Epidemiology 291

2. Prevalen ce rate is greater th an in cid en ce rate i th e disease is lon g term . For exam p le,
becau se d iab etes lasts a li etim e, its p revalen ce is m u ch h igh er th an its in ciden ce. In con -
trast, th e p revalen ce o in lu en za, an acu te illn ess, is ap p roxim ately equ al to th e in ciden ce.
3. Health in terven tion s th at p reven t d isease (i.e., primary prevention, see Ch ap ter 24)
decrease th e in cid en ce rate o an illn ess an d u ltim ately its p revalen ce rate as well.
4. Peop le with a sp eci ic illn ess can leave th e p op u lation o p revalen t cases eith er by recover-
ing or by dying.

II. RESEARCH STUDY DESIGN


HELP OTHERS SO THAT GOD WILL HELP YOU.

Research stu d ies iden ti y relation sh ip s b etween actors or variab les. Typ es o research stu dies
in clu d e coh ort, case–con trol, an d cross-section al stu dies.

A. Cohort studies
1. Coh ort stu dies begin with th e iden ti ication o a sp eci ic p op u lation i.e., acoh ort, th at is
ree o the illness u n d er in vestigation at th e start o th e stu dy.
2. Followin g th e assessm en t o exp osu re to a risk actor (a variable lin ked to th e cau se o an
illn ess [e.g., sm okin g]), in cid en ce rates o illn ess between exp osed an d u n exp osed m em -
b ers o th e coh ort are com p ared . An exam p le o a coh ort stu dy wou ld be on e th at ollowed
h ealth y ad u lts rom early ad u lth ood th rou gh m idd le age to com p are th e h ealth o th ose
wh o sm oke versu s th ose wh o d o n ot sm oke.
3. Coh ort stu dies can be prospective (takin g p lace in th e p resen t tim e) or historical (som e
activities h ave taken p lace in th e p ast).
4. A clinical treatment trial is a sp ecial typ e o coh ort stu dy in wh ich m em bers o a coh ort
with a sp eci ic illn ess are given on e treatm en t an d oth er m em bers o th e coh ort are given
an oth er treatm en t or a p lacebo. Th e resu lts o th e two treatm en ts are th en com p ared .
An exam p le o a clin ical treatm en t trial wou ld be on e in wh ich th e di eren ces in su rvival
rates b etween m en with p rostate can cer wh o receive a n ew dru g an d m en with p rostate
can cer wh o receive a stan d ard d ru g are com p ared.

B. Case–control studies
1. Case–con trol stu d ies b egin with th e id en ti ication o su bjects wh o h ave a sp eci ic disorder
(cases) an d su b jects wh o do n ot h ave th at disorder (con trols).
2. In orm ation on th e prior exposure o cases and controls to risk actors is th en obtain ed. An
exam p le o a case–con trol stu dy wou ld be on e in wh ich th e sm okin g h istories o wom en
with an d with ou t breast can cer are com p ared.
RISE USMLE NEPAL

3. Becau se cases are iden ti ied at th e start o th e stu dy, case–con trol stu dies are p articu larly
u se u l wh en a disease is rare in th e p op u lation .

C. Cross-sectional studies
1. Cross-section al stu dies begin wh en in orm ation is collected rom a grou p o in dividu als
wh o p rovide a snapshot in time o d isease activity.
2. Su ch stu dies can p rovide in orm ation on th e relation sh ip between risk actors an d h ealth
statu s o a grou p o in divid u als at on e sp eci ic p oin t in tim e (e.g., a ran dom telep h on e
sam p le con du cted to d eterm in e i m ale sm okers are m ore likely to h ave an u p p er resp ira-
tory in ection th an m ale n on sm okers). Th ey can also be u sed to calcu late th e p revalen ce
o a d isease in a p op u lation .

III. QUANTIFYING RISK


A. Risk actors are variables that are linked to the cause o a disease.
1. Measures . Ab solu te risk, relative risk, attribu table risk, an d th e odds (or odds risk) ratio
are m easu res u sed to qu an ti y risk in p op u lation stu d ies.
a. Absolute, relative, and attributable risks are calcu lated or cohort stu dies.
b. The odds ratio is calcu lated or case–control stu d ies.
292 BRS Behavioral Science

2. Absolute risk is equ al to th e in ciden ce rate.


3. Absolute risk reduction (ARR) is th e di eren ce in absolu te risks. For exam p le, i th e in ci-
d en ce rate o lu n g can cer am on g th e p eop le in Newark an d in Tren ton , New Jersey, in 2016
is 20/ 1,000 an d 15/ 1,000, resp ectively, th e absolu te risk is 20/ 1,000 or 2.0% in Newark an d
15/ 1,000 or 1.5% in Tren ton , an d th e ARR is 2.0% m in u s 1.5% or 0.5%.
4. Relative risk. Relative risk com p ares th e in ciden ce rate o a disorder am on g in dividu als
exp osed to a risk actor (e.g., sm okin g) with th e in cid en ce rate o th e disord er in u n ex-
p osed in dividu als.
a. For exam p le, th e in ciden ce rate o lu n g can cer am on g sm okers in a city in New Jersey
is 20/ 1,000, wh ile th e in ciden ce rate o lu n g can cer am on g n on sm okers in th is city is
2/ 1,000. Th ere ore, the old increase in risk o lu n g can cer (th e relative risk) or sm okers
HELP OTHERS SO THAT GOD WILL HELP YOU.

versu s n on sm okers in th is New Jersey p op u lation is 20/ 1,000 divided by 2/ 1,000, or 10.
b. A relative risk o 10 m ean s th at in th is city, i an in dividu al sm okes, h is or h er risk o
gettin g lu n g can cer is 10 tim es (10 old) th at o a n on sm oker.
5. Attributable risk
a. Attrib u tab le risk is u se u l or d eterm in in g wh at wou ld h ap p en in a stu d y p op u lation i
th e risk actor were rem oved (e.g., d eterm in in g h ow com m on lu n g can cer wou ld be in
a stu dy i p eop le did n ot sm oke).
b. To calcu late attrib u table risk, th e in cid en ce rate o th e illn ess in u n exp osed in dividu als
is su b tracted rom th e in cid en ce rate o th e illn ess in th ose wh o h ave been exp osed to
a risk actor.
c. For th e exam p le ab ove, th e risk o lu n g can cer attribu table to sm okin g (th e attribu table
risk) in th is New Jersey city’s p op u lation is 20/ 1,000 m in u s 2/ 1,000, or 18/ 1,000.
6. Odds ratio. Sin ce in cid en ce d ata are n ot available in a case–con trol stu dy, th e odds ratio
(i.e., odd s risk ratio) can b e u sed as an estim ate o relative risk (Exam p le 25.1) in su ch
stu d ies.
7. I relative risk or odds ratio = 1, th ere is n o association between a risk actor an d a disease
(an d see Ch ap ter 26, Section II.F.)

Example 25.1. Calculating the Odds Ratio


O 200 p atien ts in th e h osp ital, 50 h ave lu n g can cer. O th ese 50 p atien ts, 45 were or cu rren tly
are sm okers. O th e rem ain in g 150 h osp italized p atien ts wh o do n ot h ave lu n g can cer, 60
were or cu rren tly are sm okers. Use th is in orm ation to calcu late th e od ds ratio or sm okin g
an d lu n g can cer in th is h osp italized p atien t p op u lation .

Smokers Nonsmokers

People with lung cancer A = 45 B=5


RISE USMLE NEPAL

People without lung cancer C = 60 D = 90

(AD) = (45)(90)
= 13.5 = Odds ratio
(BC) = (5)(60)

An odd s ratio o 13.5 m ean s th at in th is p op u lation , a p erson with lu n g can cer was 13.5
tim es m ore likely to h ave sm oked th an a p erson with ou t lu n g can cer.

B. Number needed to treat and number needed to harm


1. Nu m ber n eeded to treat (NNT)
a. NNT is th e n u m ber o p erson s wh o n eed to take a treatm en t or on e p erson to ben e it
rom th e treatm en t.
b. NNT is 1 divided by th e ARR.
c. NNT allows com p arison o th e e ectiven ess o di eren t treatm en ts or o treatm en t
versu s n o treatm en t or p lacebo (Exam p le 25.2).
2. Nu m ber n eeded to h arm (NNH)
a. NNH is th e n u m ber o p erson s wh o n eed to be exp osed to a risk actor or on e p erson
to be h arm ed wh o wou ld oth erwise n ot be h arm ed.
b. NNH is 1 divided by th e attribu table risk.
Chapter 25 Medical Epidemiology 293

Example 25.2. Number Needed to Treat


A research stu d y is d on e to d eterm in e i a n ew dru g (Dru g S) will p reven t stroke in m en
aged 55–65 years wh o h ave h yp erten sion .
Fou r th ou san d h yp erten sive m en in th is age grou p are ran dom ly assign ed to a grou p
takin g Dru g S (n = 2,000) or a p lacebo (n = 2,000).
Over 10 years, th ere were 400 strokes in th e p lacebo grou p an d 200 strokes in th e Dru g S
grou p.
Based on th ese d ata, h ow m an y m en wou ld h ave to be treated with Dru g S to p reven t on e
case o stroke?
Th e absolu te risk o stroke in th e p lacebo grou p is 400/ 2,000 = 20%.
Th e absolu te risk o stroke in th e Dru g S grou p is 200/ 2,000 = 10%.
HELP OTHERS SO THAT GOD WILL HELP YOU.

Th e absolu te risk redu ction is th ere ore 20% − 10% = 10%.


Sin ce 10% o the hyperten sive m en were saved rom stroke by the drug, the NNT is 1/ 0.1 = 10.0
There ore, 10 m en would have to be treated with the Drug S to preven t on e case o stroke.

IV. BIAS, RELIABILITY, AND VALIDITY


To be u se u l, testin g in stru m en ts m u st be b ias- ree, reliable, an d valid.

A. Bias
1. A b iased test or research stu d y is on e con stru cted so th at one outcome is more likely to
occur than another.
2. Ways th at a research stu dy or clin ical treatm en t trial can be biased can be ou n d in
Table 25.1.

t a b l e 25.1 Types of Bias

Example: Six research studies were conducted on the e ectiveness o a new ( ictitious) drug, “Flashless” vs. placebo or
relieving menopausal symptoms. One hundred women aged 50–70 years were recruited or each study, and each subject
was paid $1,000. Although Flashless ultimately proved to be a use ul drug that signi icantly reduced menopausal symptoms,
because all six studies were biased in di erent ways (see table below), all ailed to show the use ulness o Flashless.

Reason or Flashless Study Failure Is That


When Compared to the General Population,
Study # Type o Bias Explanation the Subjects
RISE USMLE NEPAL

1 Selection Rather than making random assignments, the Who had more severe symptoms to start with
subjects or the investigators are permitted to chose or were chosen to take Flashless
choose whether an individual will go into the rather than placebo and so also had more
drug group or the placebo group severe symptoms at the end of the study
2 Sampling Subjects who volunteer to be in a study are Only joined the study because they needed the
not representative of the population being money and as such did not represent the
studied because factors unrelated to the typical population of women using drugs
subject of the study (e.g., money) have led such as Flashless
them to volunteer
3 Recall Knowledge of the presence of a disorder alters Seemed to have more severe menopausal
the way the subject remembers his or her symptoms because they were asked about
history their symptoms to be in the study
4 Lead-time Early detection of disease is confused with Seemed to have had menopausal symptoms
increased survival or length of time that the for a longer period of time because they
symptoms have been present were identified early to be in the study
5 Surveillance People who are aware that they are being Went to the doctor more often because
followed for the development of a disease they were in the study which increased
are more likely to seek testing for and thus to the likelihood of being diagnosed with
be identified with the disease menopausal symptoms
6 Late-look People who are most ill are not included in the Who had severe menopausal symptoms chose
sample not to participate in the study, so those who
did participate had few symptoms
294 BRS Behavioral Science

B. Reducing bias in clinical treatment trials. Blin d stu dies, p lacebo-con trolled stu dies, crossover
stu d ies, an d ran d om ized stu d ies are u sed to red u ce b ias.
1. Blind studies. Th e exp ectation s o su bjects or clin ician -evalu ators can in lu en ce th e e ec-
tiven ess o treatm en t. Blin d stu d ies attem p t to redu ce th is in lu en ce.
a. In a single-blind study, th e su bject does n ot kn ow wh at treatm en t h e or sh e is receivin g.
b. In a double-blind study, n eith er th e su b ject n or th e clin ician –evalu ator kn ows wh at
treatm en t th e su bject is receivin g.
2. Placebo responses
a. In a b lin d d ru g stu d y, a su b ject m ay receive a p lacebo (an in active su bstan ce) rath er
th an th e active dru g.
b. Peop le receivin g th e placebo are th e control group; th ose receivin g th e active drug are
HELP OTHERS SO THAT GOD WILL HELP YOU.

th e experimental group.
c. A n u m ber o su b jects in research stu dies resp on d to th e treatm en t with p laceb os alon e
(th e p laceb o e ect—an d see Ch ap ter 4).
3. Crossover studies
a. In a crossover stu dy, su b jects are ran d om ly assign ed to on e o two grou p s. Su bjects in
grou p 1 irst receive th e dru g an d su b jects in grou p 2 irst receive th e p lacebo.
b. Later in th e crossover stu d y, th e grou p s switch —th ose in grou p 1 receive th e p lacebo
an d th ose in grou p 2 receive th e d ru g.
c. Becau se all o th e su b jects receive b oth d ru g an d p lacebo, each subject acts as his or her
own control. Havin g th ese ad d ition al d ata p oin ts can e ectively double the sample size
o p atien ts in a research stu dy.
4. Randomization. In order to en su re th at th e p rop ortion o sicker an d h ealth ier p eop le is th e
sam e in th e treatm en t an d con trol (p lacebo) grou p s, su bjects are ran dom ly assign ed to
th e grou p s. Th e n u m ber o su b jects in each grou p does n ot h ave to be equ al.

C. Reliability and validity


1. Reliab ility re ers to th e rep rodu cib ility or dep en d ab ility o resu lts.
a. Interrater reliability is a m easu re o wh eth er th e resu lts o th e test are sim ilar wh en th e
test is adm in istered by a di eren t rater or exam in er.
b. Test–retest reliability is a m easu re o wh eth er th e resu lts o th e test are sim ilar wh en th e
p erson is tested a secon d or th ird tim e.
2. Validity is a m easu re o th e ap p rop riaten ess o a test, th at is, wh eth er th e test assesses
wh at it was design ed to assess (e.g., does a n ew IQ test really m easu re IQ or d oes it in stead
m easu re edu cation al level?) (see Ch ap ter 8). Sen sitivity an d sp eci icity are com p on en ts o
validity.
RISE USMLE NEPAL

D. Sensitivity and speci icity (Exam p le 25.3)


1. Sensitivity m easu res h ow well a screen in g test iden ti ies tru ly ill p eop le.
a. True positives (TP) are ill p eop le wh om a test h as correctly iden ti ied as bein g ill.
b. False negatives (FN) are ill p eop le wh om a test h as in correctly iden ti ied as bein g well
(i.e., h ealth y).
c. Sensitivity is calcu lated u sin g on ly p eop le wh o are, in act, ill (TP an d FN) by d ivid in g
TP by th e su m o TP an d FN.
(1) Tests with h igh sen sitivity iden ti y m ost or all p ossible cases.
(2) Th ey are m ost u se u l wh en id en ti yin g an ill p erson as h ealth y can lead to severe
con sequ en ces (e.g., can cer th at can m etastasize i n ot id en ti ied early).
2. Speci icity m easu res h ow well a test iden ti ies tru ly well p eop le.
a. True negatives (TN) are well p eop le wh om a test h as correctly iden ti ied as bein g well.
b. False positives (FP) are well p eop le wh om a test h as in correctly id en ti ied as b ein g ill.
c. Speci icity is calcu lated by d ivid in g TN by th e su m o TN an d FP.
(1) Tests with h igh sp eci icity iden ti y m ost or all well p eop le.
(2) Th ey are m ost u se u l wh en iden ti yin g a h ealth y p erson as ill can lead to dan ger-
ou s, p ain u l, or u n n ecessary treatm en t (e.g., a h ealth y m an wh o tests p ositive or
p rostate can cer h as a p rostate biop sy requ irin g gen eral an esth esia).
Chapter 25 Medical Epidemiology 295

Example 25.3. Sensitivity, Specif city, Predictive Value, and Prevalence


A n ew blood test to d etect th e p resen ce o HIV was given to 1,000 p atien ts. Alth ou gh 200 o
th e p atien ts were actu ally in ected with th e viru s, th e test was p ositive in on ly 160 p atien ts
(TP); th e oth er 40 in ected p atien ts h ad n egative tests (FN) an d th u s were n ot iden ti ied
by th is n ew test. O th e 800 p atien ts wh o were n ot in ected, th e test was n egative in 720
p atien ts (TN) an d p ositive in 80 p atien ts (FP).

Use th is in orm a tion to ca lcu la te sen sitivity, sp eci icity, p ositive p red ictive va lu e,
an d n ega tive p red ictive va lu e o th is n ew b lood test an d th e p reva len ce o HIV in th is
p op u lation .
HELP OTHERS SO THAT GOD WILL HELP YOU.

Patients In ected with HIV Patients Not In ected with HIV Total Patients
Positive HIV blood test 160 (TP) 80 (FP) 240 (those with a + test)
Negative HIV blood test 40 (FN) 720 (TN) 760 (those with a − test)
Total patients 200 800 1,000

160 (TP) 160


Sensitivity = = = 80.0%
160 (TP) + 40 (FN) 200

720 (TN) 720


Specificity = = = 90.0%
720 (TN) +80 (FP) 800

160 (TP) 160


Positive predictive value = = = 66.7%
160 (TP) +80 (FP) 240

720 (TN) 720


Negative predictive value = = = 94.7%
720 (TN) + 40 (FP) 760

200 (totalof those infected)


Prevalence = = 20.0%
1,000 (totalpatients)

E. Predictive value (Example 25.3)


1. The predictive value o a test is a m easu re o th e p ercen tage o test resu lts th at m atch th e
actu al diagn osis. Pred ictive valu es (b u t n ot sen sitivity or sp eci icity) vary accordin g to th e
p revalen ce o th e d isorder in th e p op u lation .
RISE USMLE NEPAL

a. Positive predictive value (PPV) is th e p robability th at som eon e with a p ositive test is
actu ally ill. PPV is calcu lated by dividin g TP by th e su m o TP an d FP.
b. Negative predictive value (NPV) is th e p robability th at a p erson with a n egative test is
actu ally well. NPV is calcu lated by d ividin g TN by th e su m o TN an d FN.
2. Th e higher the prevalence o a disorder in th e p op u lation , th e higher the PPVan d th e lower
the NPV o a test u sed to d etect it. I th e p revalen ce o a disorder in th e p op u lation is low,
even tests with very h igh sp eci icity m ay h ave a low PPV becau se th ere are likely to be a
h igh n u m b er o FP relative to TP.

F. Receiver operating characteristic (ROC) curves


1. ROC cu rves are grap h ic rep resen tation s o th e relationship o sensitivity to speci icity.
2. Th e tru e-p ositive rate (sen sitivity) is p lotted as a u n ction o th e alse-p ositive rate
(100 m in u s sp eci icity) or d i eren t cu to p oin ts (Figu re 25.1).
3. A screen in g test with p er ect d iscrim in ation (100% sen sitivity an d 100% sp eci icity) h as an
ROC cu rve th at p asses th rou gh th e u p p er le t corn er o th e cu rve.
296 BRS Behavioral Science

0.9
A
0.8
)
y
0.7
t
vi
i
t
i
s
n
0.6 B
e
s
(
e
t
0.5
a
r
e
v
HELP OTHERS SO THAT GOD WILL HELP YOU.

FIGURE 25.1. ROC curve. Graphic rep-


i
0.4 C
t
i
s
resentation of the relationship between
o
p
0.3 sensitivity and 1 − specificity for a
e
u
screening test. The closer the curve
r
T
to the diagonal (C), the less the dis-
0.2 Us e le s s te s t criminating ability; the closer the curve
Limite d dis crimina tion “hugs” the y axis (A), the better the dis-
0.1 Be tte r dis crimina tion criminating ability of the screening test.
(Reprinted with permission from Goroll
0 AH, Muley AG. Primary Care Medicine:
Office Evaluation and Management of
0 0.1 0.2 0.3 0.4 0.5 0.6 0.7 0.8 0.9 1
the Adult Patient. 6th ed. Philadelphia,
Fa ls e pos itive ra te (1 – s pe cificity) PA: Lippincott Williams & Wilkins; 2009.)

V. CLINICAL PROBABILITY AND ATTACK RATE


A. Clin ical p robability is th e n u m b er o tim es an event actually occurs d ivid ed by th e n u m -
b er o tim es th e event can occur (Exam p le 25.4).

Example 25.4. Clinical Probability


A ter 3 years o clin ical trials o a n ew m ed ication to treat m igrain e h eadach e, it is deter-
m in ed th at 20% o p atien ts takin g th e n ew m edication develop h yp erten sion . I two
p atien ts (p atien ts A an d B) take th e dru g, calcu late th e ollowin g p robabilities.
1. The probability that both patient A and patient B will develop hypertension:
This is calculated by m ultiplyin g the probability o A developin g hyperten sion by the prob-
RISE USMLE NEPAL

ability o B developin g h yperten sion (th e m u ltiplication rule or in depen den t even ts).
Th e p robability o A d evelop in g h yp erten sion = 0.20 = 20%.
Th e p robability o B d evelop in g h yp erten sion = 0.20 = 20%.
Th e p robability o b oth A an d B d evelop in g h yp erten sion = 0.20 × 0.20 = 0.04 = 4%.
2. The probability that at least one o the two patients (either A or B or both A and B) will develop
hypertension:
Th is is calcu lated by ad d in g th e p robab ility o A develop in g h yp erten sion to th e p rob-
ability o B develop in g h yp erten sion an d th en , becau se a p atien t can n ot be in both A an d
B at th e sam e tim e, su btractin g th e p rob ab ility o both A an d B develop in g h yp erten sion
(th e add ition ru le).
0.20 + 0.20 − 0.04 = 0.36 = 36%
3. The probability that neither patient A nor patient B will develop hypertension:
Th is is calcu lated by m u ltip lyin g th e p robability o p atien t A bein g n orm oten sive by th e
p robability o p atien t B b ein g n orm oten sive: Probability o both bein g n orm oten sive
= (1 − p rob ab ility o A b ein g h yp erten sive) × (1 − p robability o B bein g h yp erten sive)
= 0.80 ´ 0.80 = 0.64 = 64%.
Chapter 25 Medical Epidemiology 297

B. Attack rate is a typ e o in cid en ce rate u sed to d escrib e d isease ou tbreaks. It is calcu lated
by dividin g th e n u m ber o p eop le wh o becom e ill du rin g a stu dy p eriod by th e n u m -
b er o p eop le at risk du rin g th e stu dy p eriod. For example, i , a ter a p icn ic, 20 ou t o 40
p eop le wh o ate ried ch icken an d 10 ou t o 50 p eop le wh o ate ried ish becom e ill, th e
attack rate is 50% or ch icken an d 20% or ish .
HELP OTHERS SO THAT GOD WILL HELP YOU.
RISE USMLE NEPAL
Review Test

Directions: Each o th e n u m b ered item s or in com p lete statem en ts in th is section is ollowed by


an swers or by com p letion s o th e statem en t. Select th e one lettered an swer or com p letion th at
is best in each case.
HELP OTHERS SO THAT GOD WILL HELP YOU.

1. A stu dy is d on e to determ in e th e 3. A typ e o gyn ecological can cer h as th e


e ectiven ess o a n ew an tih istam in e. To do sam e in ciden ce rate in Wh ite wom en an d
th is, 25 allergic p atien ts are assign ed to on e A rican -Am erican wom en in th e Un ited
o th e two grou p s, th e n ew dru g (13 p atien ts) States, but the prevalen ce rate o this typ e o
or a p laceb o (12 p atien ts). Th e p atien ts are can cer is lower in A rican -Am erican th an in
th en ollowed over a 6-m on th p eriod. Th is White wom en . The m ost likely exp lan ation
stu d y is best describ ed as a or this di eren ce in prevalen ce rates is th at
(A) coh ort stu d y wh en com p ared to Wh ite wom en , A rican -
(B) cross-section al stu d y Am erican wom en are m ore likely to
(C) case–con trol stu d y (A) recover rom th is typ e o can cer
(D) h istorical coh ort stu dy (B) h ave n atu ral im m u n ity to th is typ e o
(E) clin ical treatm en t trial can cer
(C) h ave in creased access to treatm en t or
2. Usin g th e data in th e tab le b elow, th e th is typ e o can cer
ch an ce o su rvivin g or 2 years a ter b ein g (D) be resistan t to th is typ e o can cer
diagn osed with ovarian can cer is (E) die rom th is typ e o can cer
360/ 500 = 72% an d th e ch an ce o su rvivin g
or 4 years is 135/ 500 = 27%. Wh at is th e
ch an ce o su rvivin g or 4 years a ter th e Questions 4 and 5
origin al diagn osis, given th at th e p atien t is
alive at th e en d o th e 2n d year? A town in th e western Un ited States h as a
p op u lation o 1,200. In 2012, 200 residen ts
Number o Number o Percent o
o th e town are d iagn osed with a disease. In
Women in the Women Who Women Who 2013, 100 m ore residen ts o th e town are dis-
Beginning o Died During Survived covered to h ave th e sam e disease. Th e disease
RISE USMLE NEPAL

Year the Year the Year Each Year (%) is li elon g an d ch ron ic bu t n ot atal.
0–1 500 100 80
1–2 400 40 90 4. Th e in ciden ce rate o th is disease in 2013
2–3 360 90 75 am on g th is town’s p op u lation is
3–4 270 135 50 (A) 100/ 1,200
(B) 200/ 1,200
(A) 40/ 400 = 10% (C) 300/ 1,200
(B) 135/ 500 = 27% (D) 100/ 1,000
(C) 40/ 135 = 29.6% (E) 300/ 1,000
(D) 135/ 360 = 37.5%
(E) 135/ 270 = 50% 5. Th e p revalen ce rate o th is disease in 2013
am on g th e town’s p op u lation is
(A) 100/ 1,200
(B) 200/ 1,200
(C) 300/ 1,200
(D) 100/ 1,000
(E) 300/ 1,000

298
Chapter 25 Medical Epidemiology 299

6. A stu dy is d esign ed to d eterm in e th e Questions 10–13


relation sh ip between em otion al stress an d
p ep tic u lcer. To do th is, th e research ers u se A p atien t is given a n ew screen in g test or
h osp ital records o p atien ts diagn osed with tu bercu losis. Alth ou gh th e p atien t is in ected,
p ep tic u lcer disease an d p atien ts diagn osed th e test in dicates th at th e p atien t is well.
with oth er d isorders over th e p eriod rom
Ju ly 2005 to Ju ly 2015. Th e em otion al stress 10. Th is test resu lt is kn own as
each p atien t was exp osed to was d eterm in ed (A) alse p ositive
rom th ese records an d was th en qu an ti ed (B) alse n egative
with a score o 1 (least stress) to 100 (m ost (C) tru e p ositive
stress). Th is stu d y is b est d escribed as a (D) tru e n egative
HELP OTHERS SO THAT GOD WILL HELP YOU.

(A) coh ort stu d y (E) p redictive


(B) cross-section al stu d y
(C) case–con trol stu d y 11. To iden ti y all p atien ts in ected with
(D) h istorical coh ort stu dy tu b ercu losis, th e cu to p oin t or th is test
(E) clin ical treatm en t trial sh ould be set at th e p oin t o h igh est
(A) sen sitivity
7. An in telligen ce qu otien t (IQ) test h as h igh (B) sp eci icity
in terrater reliability. Th is m ean s th at (C) p ositive p redictive valu e
(A) th e test in volves stru ctu red in terviews (D) n egative p redictive valu e
(B) a n ew assessm en t strategy is bein g u sed (E) accu racy
(C) th e test actu ally m easu res IQ an d n ot
ed u cation al level 12. I th is n ew screen in g test h as a sen sitivity
(D) th e resu lts are very sim ilar wh en th e test o 90% an d a sp eci city o 70% in a grou p
is adm in istered a secon d tim e o you n g Ru ssian p rison ers in wh ich th e
(E) th e resu lts are very sim ilar wh en th e test p revalen ce o tu bercu losis is 50%, th e
is adm in istered by a di eren t exam in er p ositive p redictive valu e o th is test is best
estim ated as
8. Th ere are 100,000 p eop le in Hobart, (A) 12.5%
Tasm an ia. On Jan u ary 1, 2015, 50 o th ese (B) 25%
p eop le h ave Disease Y. Fi ty d ivid ed by (C) 30%
100,000 on th at date gives wh ich o th e (D) 75%
ollowin g m easu res or Disease Y? (E) 90%
(A) Poin t p revalen ce
(B) Period p revalen ce 13. I th e test is given on ly to elderly
(C) In ciden ce rate p rison ers in wh om th e in cid en ce an d
(D) Odds ratio p revalen ce o tu bercu losis is h igh er th an
RISE USMLE NEPAL

(E) Relative risk in you n g p rison ers, th e p ositive p redictive


valu e an d sen sitivity o th is screen in g test
9. In wh ich o th e ollowin g in ectiou s will, resp ectively, ch an ge in wh ich o th e
illn esses is p revalen ce m ost likely to exceed ollowin g ways?
in ciden ce?
(A) Measles
(B) In lu en za Positive Predictive Value Sensitivity
(C) Lep rosy (A) Increase Increase
(D) Ru bella (B) Decrease Decrease
(E) Rabies (C) Increase Not change
(D) Not change Not change
(E) Increase Decrease
300 BRS Behavioral Science

14. A case–con trol stu d y is don e to 17. Th is stu dy is best described as a


determ in e i elderly d em en ted p atien ts (A) coh ort stu d y
are m ore likely to be in ju red at h om e th an (B) cross-section al stu d y
elderly p atien ts wh o are n ot dem en ted. Th e (C) case–con trol stu dy
resu lts o th e stu dy sh ow an odd s ratio o 3. (D) h istorical coh ort stu dy
Th is gu re m ean s th at i an elderly p atien t (E) clin ical treatm en t trial
was in ju red at h om e, th at p atien t
(A) sh ou ld be p laced in an exten d ed -care
acility Questions 18–21
(B) was on e-th ird m ore likely to be
dem en ted th an a p atien t wh o was n ot A n ew b lood test to d etect p rostate ca n cer
HELP OTHERS SO THAT GOD WILL HELP YOU.

in ju red at h om e by m ea su rin g p rosta te-sp eci ic an tigen


(C) was n o m ore likely to be dem en ted th an (PSA) was given to 1,000 m ale m em b ers o
a p atien t wh o was n ot in ju red at h om e a large HMO. Alth ou gh 50 o th e m en actu -
(D) was th ree tim es m ore likely to be ally h ad p rostate can cer, th e test was p ositive
dem en ted th an a p atien t wh o was n ot (PSA > 4 ng/ m L) in only 15; the other 35 patients
in ju red at h om e with prostate cancer had negative tests. O the
(E) sh ou ld be kep t at h om e 950 m en without prostate cancer, the test was
positive in 200 m en and negative in 750.
Questions 15–17
18. Th e sp eci city o th is test is
A stu dy is un dertaken to determ in e i p ren atal ap proxim ately
exposu re to m arijuan a is associated with low (A) 15%
birth weigh t in in an ts. Moth ers o 50 in an ts (B) 30%
weighin g less than 5 pou n ds (low birth weight) (C) 48%
an d 50 in an ts weigh in g m ore th an 7 pou n ds (D) 79%
(n orm al birth weight) are question ed about (E) 86%
their use o m arijuan a durin g pregn an cy. Th e
study in ds that 20 m oth ers o low-birth -weight 19. Th e p ositive p redictive valu e o th is
in an ts an d 2 m others o n orm al-birth-weight b lood test is
in an ts used the drug durin g pregn an cy.
(A) 7%
15. In th is stu dy, th e odds ratio associated (B) 14%
with sm okin g m ariju an a d u rin g p regn an cy is (C) 21%
(D) 35%
(A) 10
(E) 93%
(B) 16
(C) 20
RISE USMLE NEPAL

20. I th e cu to valu e in dicatin g a p ositive


(D) 30
test is lowered rom PSA 4 n g/ m L to PSA 3
(E) 48
n g/ m L, th is ch an ge wou ld
16. An odd s ratio o X, calcu lated in th e (A) in crease n egative p redictive valu e
p recedin g qu estion , m ean s th at (B) decrease sen sitivity
(A) the in ciden ce o low birth weight in in an ts (C) in crease alse-n egative rate
whose m others sm oke m arijuan a is X (D) in crease p ositive p redictive valu e
(B) an in an t o low b irth weigh t was X tim es (E) in crease sp eci icity
as likely as an in an t o n orm al birth
weigh t to h ave h ad a m oth er wh o u sed 21. With th is ch an ge in th e cu to valu e, th e
m ariju an a d u rin g p regn an cy in cid en ce an d p revalen ce o p rostate can cer
(C) a ch ild h as a 1/ X ch an ce o bein g born wou ld
o low birth weigh t i its m oth er u ses
m ariju an a Incidence Prevalence
(D) th e risk o low birth weigh t in in an ts (A) Increase Increase
wh ose m oth ers u se m ariju an a is n o (B) Decrease Decrease
di eren t rom th at o in an ts wh ose (C) Increase Not change
m oth ers d o n ot u se th e dru g (D) Not change Not change
(E) the prevalence o low birth weight in in ants (E) Increase Decrease
wh ose m oth ers sm oke m ariju an a is X
Chapter 25 Medical Epidemiology 301

22. A stu dy is d esign ed to com p are a n ew 25. In p eop le with n o kn own risk or
m edication or Croh n’s disease with a tu bercu losis, a p ositive reaction to th e
stan dard m edication . To do th is, each o 50 p u ri ed p rotein derivative (PPD) tu bercu lin
Croh n’s disease p atien ts is allowed to decide skin test requ ires 15 m m or m ore o h ard
wh ich o th ese two treatm en t grou p s to join . swellin g at th e site. A grou p o p h ysician s
Th e m ajor reason th at th e resu lts o th is decide th at th ey are goin g to ch an ge th e
stu dy m ay n ot be valid is b ecau se o criterion or a p ositive test in a grou p o
(A) selection bias p eop le with n o kn own risk or tu b ercu losis
(B) recall bias to a h ard swellin g o 10 m m or m ore at th e
(C) sam p lin g bias site. With resp ect to th e PPD test, th is ch an ge
(D) d i eren ces in th e sizes o th e two in th e cu to p oin t is m ost likely to
HELP OTHERS SO THAT GOD WILL HELP YOU.

grou p s (A) in crease sen sitivity


(E) th e sm all n u m b er o p atien ts in th e (B) decrease sen sitivity
stu d y (C) decrease n egative p redictive valu e
(D) in crease p ositive p redictive valu e
23. A ter a n ew an tid ep ressan t h as b een (E) in crease sp eci icity
on th e m arket or 5 years, it is d eterm in ed
th at o 2,400 p eop le wh o h ave taken th e 26. A research study is done to determ ine i
d ru g, 360 com p lain ed o p ersisten t n au sea. intravenous (IV) ibandronate sodium (Boniva)
I a p h ysician h as two p atien ts on th is will decrease the in ciden ce rate o hip ractures
an tidep ressan t, th e p robability th at both o in perim enopausal wom en. There are 2,600
th em will exp erien ce p ersisten t n au sea is wom en in the ibandronate sodium group, o
ap proxim ately whom 130 develop hip ractures. O the 2,600
(A) 2% wom en in the placebo group, 260 develop
(B) 9% hip ractures. Based on these data, how m any
(C) 24% wom en n eed to be treated with iban dron ate
(D) 30% sodium to preven t on e hip racture?
(E) 64% (A) 1
(B) 5
24. A b lood test reveals th at a 35-year- (C) 10
old wom an at 18 weeks o gestation h as (D) 15
in creased seru m alp h a- etop rotein (AFP). (E) 20
O th e ollowin g m easu res, wh ich h as
th e greatest in f u en ce in d eterm in in g th e 27. A n ew laboratory test to detect
p redictive valu e o th is test or n eu ral tu b e osteoporosis in wom en greater than 80 years o
d e ects in th e etu s? age has a sensitivity o 90% and a speci city o
(A) Absolu te con cen tration o AFP in th e 75%. Autopsy studies suggest that osteoporosis
RISE USMLE NEPAL

m atern al seru m has a prevalence o 30% or wom en in this age


(B) Fam ily h istory o dizygotic twin group. Using this in orm ation, the likelihood
p regn an cy that a wom an with a positive test actually has
(C) Prevalen ce o n eu ral tu b e d e ects in th e osteoporosis is best estim ated as
p op u lation in qu estion (A) 12.5%
(D) Sp eci icity o th e blood test (B) 30%
(E) Sen sitivity o th e blood test (C) 60%
(D) 70%
(E) 85%
An swers an d Exp lan ation s

Typical Board Questions


The answer is B. A crossover design wou ld in crease th e sam p le size in a research stu dy su ch as
th is on e. Su ch a d esign wou ld e ectively d ou ble th e sam p le size (th e p lacebo con trols wou ld
get th e n ew an tih istam in e an d th e n ew an tih istam in e grou p wou ld n ow receive th e p lacebo)
HELP OTHERS SO THAT GOD WILL HELP YOU.

with ou t recru itin g an y n ew su bjects. Ran dom izin g th e design or u sin g a dou ble-blin d design
wou ld n ot in crease th e sam p le size. Givin g all o th e con trols, th e n ew an tih istam in e wou ld
in crease th e sam p le size o th e n ew an tih istam in e grou p bu t wou ld n ot a ect th e sam p le size o
th e p lacebo grou p.

1. The answer is E. Th is stu d y is best d escribed as a clin ical treatm en t trial, a stu d y in wh ich a
coh ort receivin g a n ew an tih istam in e is com p ared with a coh ort receivin g a p lacebo.
2. The answer is D. Th e table sh ows th at o th e 360 ovarian can cer p atien ts wh o su rvived th e
2n d year, 135 (270 - 135 = 135) su rvived th e 4th year. Th ere ore, th e ch an ce o su rvivin g or 4
years given th at th e p atien t is alive at th e en d o th e 2n d year is 135/ 360 = 37.5%.
3. The answer is E. Prevalen ce rate o an illn ess is d ecreased eith er wh en p atien ts recover or
wh en th ey die. Becau se wh en com p ared to Wh ite p atien ts, A rican Am erican p atien ts ten d
to h ave lower in com es an d decreased access to h ealth care (see Ch ap ter 18), th ey are less
likely to receive early treatm en t or disorders su ch as can cer, an d th u s m ore likely to die.
Decreased p revalen ce in A rican Am erican wom en is th u s m ore likely to b e d u e to early
death th an to recovery rom th is typ e o can cer. Resistan ce to an illn ess or im m u n ity to an
illn ess a ects in ciden ce rate, wh ich is equ al in both grou p s o wom en in th is exam p le.
4. The answer is D. 5. The answer is C. Th e in ciden ce rate o th e disease in 2013 is 100/ 1,000,
th e n u m ber diagn osed with th e illn ess divid ed by th e n um ber o p eop le at risk or th e
illn ess. Becau se th e 200 p eop le wh o got th e disease in 2012 are n o lon ger at risk or gettin g
th e illn ess in 2013, th e d en om in ator in th e equ ation (n u m ber o p eop le at risk) is 1,000
(rath er th an 1,200). Th e p revalen ce rate o th is d isease in 2013 is 300/ 1,200. Th is igu re
rep resen ts th e p eop le wh o were diagn osed in 2013 an d still h ave th e disease (100) p lu s th e
p eop le wh o were diagn osed in 2012 an d still h ave th e disease (200) divided by th e total
RISE USMLE NEPAL

p op u lation at risk (1,200).


6. The answer is C. Case–con trol stu dies b egin with th e iden ti ication o su bjects wh o h ave
a sp eci ic d isord er (cases, i.e., u lcer p atien ts) an d su bjects wh o do n ot h ave th at disorder
(con trols, i.e., th ose diagn osed with oth er d isord ers). In orm ation on th e p rior exp osu re
o cases an d con trols to risk actors is th en ob tain ed. In th is case–con trol stu dy, th e
in vestigators u sed cases (u lcer p atien ts) an d con trols (p atien ts with oth er disorders) an d
looked in to th eir h istories (h osp ital records) to determ in e th e occu rren ce o an d qu an ti y
th e level o th e risk actor (i.e., em otion al stress) in each grou p. Coh ort stu dies begin with
th e iden ti ication o sp eci ic p op u lation s (coh orts), wh o are ree o illn ess at th e start o
th e stu dy an d can b e p rosp ective (takin g p lace in th e p resen t tim e) or h istorical (som e
activities h ave taken p lace in th e p ast). Clin ical treatm en t trials are coh ort stu dies in wh ich
m em bers o a coh ort with a sp eci ic illn ess are given on e treatm en t an d oth er m em bers o
th e coh ort are given an oth er treatm en t or a p lacebo. Th e resu lts o th e two treatm en ts are
th en com p ared . Cross-section al stu dies in volve th e collection o in orm ation on a disease
an d risk actors in a p op u lation at on e p oin t in tim e.
7. The answer is E. In terrater reliability is a m easu re o h ow sim ilar test in din gs are wh en u sed
by di eren t exam in ers.

302
Chapter 25 Medical Epidemiology 303

8. The answer is A. Poin t p revalen ce is th e n u m ber o p eop le wh o h ave an illn ess at a sp eci ic
p oin t in tim e (e.g., Jan u ary 1, 2015) divided by th e total p op u lation at th at tim e. In ciden ce
rate is th e n u m b er o in d ivid u als wh o d evelop an illn ess in a given tim e p eriod (com m on ly
1 year) divided by th e total n u m b er o in d ividu als at risk or th e illn ess du rin g th at tim e
p eriod . Period p revalen ce is th e n u m ber o in dividu als wh o h ave an illn ess du rin g a
sp eci ic tim e p eriod . Relative risk com p ares th e in cid en ce rate o a disord er am on g
in dividu als exp osed to a risk actor (e.g., sm okin g) with th e in ciden ce rate o th e disorder
in u n exp osed in dividu als. Th e od d s ratio is an estim ate o th e relative risk in case–con trol
stu d ies.
9. The answer is C. In lep rosy, a lon g-lastin g, in ectiou s illn ess, th e n u m b er o p eop le in
th e p op u lation wh o h ave th e illn ess (p revalen ce) is likely to exceed th e n u m ber n ewly
HELP OTHERS SO THAT GOD WILL HELP YOU.

develop in g th e illn ess in a given year (in ciden ce). Measles, in lu en za, ru bella, an d rabies
are sh orter-lastin g illn esses th an lep rosy.
10. The answer is B. 11. The answer is A. 12. The answer is D. 13. The answer is C. A
a lse -n ega tive resu lt o ccu rs i a test d o es n ot d etect tu b ercu losis in so m eon e wh o
tru ly is in e cted . Tru e p o sitives a re ill p eo p le wh om a test h a s co rrectly id en ti ied a s
b ein g ill. Tru e n ega tives a re well p eo p le wh o m a test h a s co rrectly id en ti ied as b ein g
we ll. Fa lse p o sitives a re we ll p eo p le wh om a test h a s in co rrectly id en ti ied a s b ein g ill.
In o rd er to id en ti y a ll tru ly in ected p eop le ( TP a n d FN), th e cu to p o in t o r th e test
sh o u ld b e set a t th e p o in t o h igh est sen sitivity, th a t is, th e p o in t a t wh ich th ere a re
th e ewest n u m b er o FN. Usin g th e d a ta p rovid ed a n d a ssu m in g th ere a re a tota l o
200 yo u n g p riso n ers, th e p o sitive p red ictive va lu e ( TP/ TP + FP) o th is test is 90/ 90 +
30 = 75%.
Disease Present Disease Absent Total

Positive test 90 (TP) 30 (FP) 120


Negative test 10 (FN) 70 (TN) 80
Total 100 100 200

Positive p red ictive valu e: 90 TP/ (90 TP + 30 FN) = 90/ 120 = 75%.
Calcu lation s sh own b elow in d icate th at i p revalen ce o th e disease is in creased in a p op u -
lation (e.g., 200 elderly m en ), p ositive p redictive valu e in creases, bu t sen sitivity does n ot
ch an ge.
Disease Present Disease Absent Total
RISE USMLE NEPAL

Positive test 172 (TP) 6 (FP) 178


Negative test 18 (FN) 14 (TN) 32
Total 180 20 200

I th e p revalen ce o th e d isease is in creased , b oth TP an d FN will in crease to th e sam e


exten t, an d sen sitivity will n ot ch an ge. However, with in creased p revalen ce, TP will
in crease an d FP will d ecrease, so p ositive p red ictive valu e will in crease. Also, b ecau se
with in creased p revalen ce, FN in creases b u t TN d ecreases, n egative p red ictive valu e will
d ecrease.
14. The answer is D. An od ds ratio o 3 m ean s th at an elderly p atien t wh o was in ju red at h om e
was th ree tim es m ore likely to be dem en ted th an a p atien t wh o was n ot in ju red at h om e.
Th is n u m ber does n ot in dicate wh eth er or n ot certain p eop le sh ou ld rem ain at h om e or be
cared or by oth ers.
15. The answer is B. 16. The answer is B. 17. The answer is C. Th e odds ratio is 16 an d is
calcu lated as ollows:
304 BRS Behavioral Science

Mother Smoked Marijuana Mother Did Not Smoke Marijuana

Low-birth-weight babies A = 20 B = 30
Normal-birth-weight babies C= 2 D = 48

Odds ratio = (AD)/ (BC) or (20)(48)/ (30)(2) = 960/ 60 = 16.


Th e od d s ratio o 16 m ean s th at an in an t o low b irth weigh t was 16 tim es as likely as an
in an t o n orm al b irth weigh t to h ave h ad a m oth er wh o u sed m ariju an a du rin g p regn an cy.
Th is stu d y is b est d escribed as a case–con trol stu dy; th e risk actor h ere is etal exp osu re to
m ariju an a.
HELP OTHERS SO THAT GOD WILL HELP YOU.

18. The answer is D. 19. The answer is A. 20. The answer is A. 21. The answer is D.
Calcu lation s sh own b elow in dicate th at th e sp eci icity o th is b lood test is 79% an d th e
p ositive p red ictive valu e is 7%.

Those Who Have Those Who Do Not


Prostate Cancer Have Prostate Cancer Total
Positive blood test 15 (TP) 200 (FP) 215
Negative blood test 35 (FN) 750 (TN) 785
Total patients 50 950 1,000

Speci icity: 750 (TN)/ [750 (TN) + 200 (FP)] = 0.789 or 78.9%.
Positive predictive value: 15 (TP)/ [15 (TP) + 200 (FP)]= 0.07 or 7.0%.
Decreasin g th e lower lim it o th is re eren ce test valu e (i.e., th e cu to valu e) can be exp ected
to both d ecrease th e n u m ber o alse n egatives an d in crease th e n u m ber o alse p ositives.
Su ch alteration s will b oth in crease sen sitivity (TP/ TP + FN) an d n egative p red ictive valu e
( TN/ TN + FN) a n d d ecrea se sp eci icity ( TN/ TN + FP) an d p ositive p red ictive valu e (TP/
TP + FP) o th e test. A ch an ge in th e re eren ce in terval wou ld n ot a ect th e in ciden ce or
p revalen ce o p rostate can cer in th e p op u lation .
22. The answer is A. Th e m ajor reason th at th e resu lts o th is stu dy are n ot valid is becau se
o selection b ias (i.e., th e su b jects were ab le to ch oose wh ich grou p to go in to). I very ill
p eop le were m ore likely to ch oose th e stan dard treatm en t, p eop le in th e exp erim en tal
treatm en t grou p (wh o were h ealth ier to begin with ) wou ld h ave h ad a better ou tcom e.
In recall bias, kn owledge o th e p resen ce o a disorder alters th e way su bjects rem em ber
th eir h istories. In sam p lin g bias, su bjects are ch osen to be in a stu dy becau se o th e actors
th at m ay be u n related to th e su bject o th e stu dy bu t distin gu ish th em rom th e rest o th e
RISE USMLE NEPAL

p op u lation . A stu dy can b e valid even th ou gh two grou p s m ay be o di eren t sizes or th ere
are a sm all n u m ber o p atien ts in a stu dy.
23. The answer is A. Th e p rob ab ility o b oth p atien ts (A an d B) takin g th is an tidep ressan t
exp erien cin g n au sea equ als th e p rob ab ility o A exp erien cin g n au sea (360/ 2,400 = 0.15)
tim es th e p robability o B exp erien cin g n au sea (360/ 2,400 = 0.15) = 0.15 × 0.15 = 0.0225,
th at is, abou t 2%.
24. The answer is C. Th e p revalen ce o n eu ral tu b e de ects in th e p op u lation in qu estion h as
th e greatest in lu en ce in determ in in g th e p red ictive valu e o th is test or th is p atien t sin ce
p revalen ce is d irectly related to p red ictive valu e. Th e h igh er th e p revalen ce, th e h igh er
th e p ositive p red ictive valu e (PPV) an d th e lower th e n egative p redictive valu e (NPV).
Sen sitivity an d speci icity relate to wh eth er th e test in dicates th at th ere is a n eu ral tu be
de ect in an a ected etu s (sen sitivity) or th e absen ce o a n eu ral tu be de ect in a h ealth y
etu s (sp eci icity). Wh ile AFP in th e m atern al seru m or am ily h istory o dizygotic twin
p regn an cy m ay be related to wh eth er or n ot th e etu s h as a n eu ral tu be de ect, th ey are n ot
related to th e p red ictive valu e o a screen in g test.
Chapter 25 Medical Epidemiology 305

25. The answer is A. With resp ect to th e PPD test, th is ch an ge in th e cu to p oin t is m ost likely
to in crease sen sitivity an d n egative p red ictive valu e. Th is is becau se th ere will be ewer
alse n egatives, th at is, ewer p eop le wh o are actu ally at risk or TB will be iden ti ied as n ot
at risk. Th is ch an ge in th e cu to p oin t will also decrease sp eci icity an d p ositive p redictive
valu e (see also an swer to Qu estion 20).
26. The answer is E. Twen ty wom en n eed to be treated with IV iban dron ate sodiu m to p reven t
on e h ip ractu re. Th e n u m ber n eeded to treat is calcu lated as 1/ absolu te risk redu ction .
O th e 2,600 wom en in th e p laceb o grou p, 260 develop h ip ractu res. O th e 2,600 wom en
in th e ib an dron ate sodiu m grou p, 130 d evelop h ip ractu res. Th e in ciden ce rate o h ip
ractu res in th e p lacebo grou p is th ere ore 260/ 2,600 (0.1 or 10%) an d th e in cid en ce rate
o h ip ractu res in th e iban dron ate sodiu m grou p is 130/ 2,600 (0.05 or 5%). Th ere ore,
HELP OTHERS SO THAT GOD WILL HELP YOU.

absolu te risk red u ction (ARR) is 10% − 5% = 5%. I 5% o wom en were p reven ted rom
h avin g a h ip ractu re by th e d ru g, th e NNT is 1.0 divided by 0.05, or 20.
27. The answer is C. Assu m in g a total o 1,000 wom en over age 80 years an d a p revalen ce rate
o 30% in th is age grou p, 300 wom en h ave osteop orosis an d 700 are well. O th e 300 wh o
h ave osteop orosis, a screen in g test with a sen sitivity o 90% wou ld iden ti y 270 TP an d 30
FN. O th e 700 who are well, a screen in g test with a sp eci icity o 75% wou ld iden ti y 525
TN an d 175 FP. Usin g th ese data (see table below), th e likelih ood th at a wom an with a
p ositive test actu ally h as osteop orosis (p ositive p redictive valu e [TP/ TP + FP]) is 270/ 270 +
175 = 60%.
Disease Present Disease Absent Total

Positive test 270 (TP) 175 (FP) 445


Negative test 30 (FN) 525 (TN) 555
Total 300 700 1,000
RISE USMLE NEPAL
c ha pte r
26 Statistical An alyses
HELP OTHERS SO THAT GOD WILL HELP YOU.

Typical Board Question


A research grou p * h yp oth esizes th at level o d aily alcoh ol con su m p tion is p ositively associ-
ated with th e risk o h ead an d n eck can cer (HNC). Th eir d ata are p resen ted in th e grap h
below. Usin g th ese data, wh ich level o alcoh ol con su m p tion is n ot associated with a sign i i-
can tly in creased risk o HNC?
Re la tive Ris k
5.00 95% confide nce inte rva l
4.50
)
4.00
R
(R
3.50
k
s
i
3.00
R
e
2.50
v
i
t
a
l
2.00
e
R
1.50
1.00
0.50
0 12 24 36 48 60 72 84 96 108 120
Alcohol cons umption, gra ms /day

(A) 12 gram s/ day


(B) 36 gram s/ day
(C) 60 gram s/ day
(D) All levels o alcoh ol con su m p tion are associated with a sign i ican t risk o HNC
(E) Risk o HNC can n ot be determ in ed rom th e data p resen ted
RISE USMLE NEPAL

*Re eren ce: Zhan g Y, Wan g R, Miao L, et al. Di eren t levels in alcoh ol an d tobacco con sum p tion
in h ead an d n eck can cer p atien ts rom 1957 to 2013. PLoS On e. 2015;10(4):e0124045. Figure 3
(See “An sw ers an d Explan ation s” at th e en d of th e ch apter.)

I. ELEMENTS OF STATISTICAL ANALYSES


A. Overview
1. Descriptive statistics su m m arize th e data obtain ed rom research stu dies.
2. Inferential statistics p rovide a way to gen eralize resu lts to an en tire p op u lation by observ-
in g a sam p le o th at p op u lation .
3. A variable is a qu an tity th at can ch an ge u n d er di eren t exp erim en tal situ ation s; variables
m ay b e in d ep en d en t or dep en d en t.
a. An independent variable is a p red ictive actor th at h as an im p act on a dep en den t vari-
able (e.g., th e am ou n t o at in th e d iet).
b. A dependent variable is th e ou tcom e th at re lects th e e ects o ch an gin g th e in dep en -
den t variab le (e.g., b ody weigh t u n der d i eren t dietary at regim en s).

306
Chapter 26 Statistical Analyses 307

t a b l e 26.1 Calculating Standard Deviation, Standard Error of the Mean, z Score, and
Confidence Interval

Measure Comment Formula

Standard deviation (S ) Average distance of observations from their mean 2


∑ (X − X)
S=
n −1

Standard error of the mean (SE) Estimate of the quality of the sample S
SE =
n
z score (z) Difference between one score in the distribution and the (X − X)
z=
HELP OTHERS SO THAT GOD WILL HELP YOU.

population mean in units of standard deviation S


Confidence interval (CI ) Specifies the high and low limits of the interval in which the Cl = X ± z (SE)
true population mean lies

n, number of subjects; X, observed value; X, mean.
From Fadem B. Behavioral Science in Medicine. 2nd ed. Philadelphia, PA: Lippincott Williams & Wilkins; 2012:318.

B. Measures of dispersion (Tab le 26.1)


1. Standard deviation (S) is th e average distan ce o observation s rom th eir m ean (X).
Stan d ard deviation is calcu lated by squ arin g each deviation rom th e m ean in a grou p
o scores, th en ad din g th e squ ared deviation s; th is su m is th en divided by th e n u m ber o
scores in th e grou p (n ) m in u s 1, an d th e squ are root o th e resu lt is d eterm in ed.
2. A stan d ard n orm al valu e, or z score , is th e di eren ce between an in dividu al variable an d
th e p op u lation m ean in u n its o stan dard deviation .
3. Standard error of the mean or stan dard error (SE) is th e stan dard deviation divided by th e
squ are root o th e n u m ber o scores in a sam p le (n ).
4. Confidence interval (CI). Th e m ean o a sam p le is on ly an estim ate. Th e CI sp eci ies th e
h igh an d low lim its between wh ich a given p ercen tage (e.g., 95% is con ven tion ally u sed in
m ed ical research ) o th e p op u lation wou ld be exp ected to all (i.e., th e in terval in wh ich
th e tru e p op u lation m ean lies). Th e CI is equ al to th e m ean o th e sam p le (X) p lu s or
m in u s th e z score m u ltip lied by th e SE.
a. For th e 95% CI, a z score o 2 is u sed.
b. For th e 99% CI, a z score o 2.5 is u sed.
c. For th e 99.7% CI, a z score o 3 is u sed.
5. In estim ating the m ean, precision re lects how reliable the estim ate is and accuracy re lects how
close the estim ate is to the true m ean. The wider the CI, the less precise the estim ate. However,
wider CIs are also m ore accurate as they have a greater likelihood o containing the true m ean.
RISE USMLE NEPAL

C. Measures of central tendency


1. Th e mean, or average, is obtain ed by ad din g a grou p o n u m bers an d dividin g th e su m by
th e qu an tity o n u m bers in th e grou p.
2. Th e median, 50th p ercen tile valu e, is th e m iddle valu e in a sequentially ordered grou p o
n u m b ers (i.e., th e valu e th at d ivid es th e data set in to two equ al grou p s).
3. Th e mode is th e valu e th at ap p ears m ost o ten in a grou p o n u m bers.

D. Normal distribution. A normal distribution , also re erred to as a gau ssian or bell-shaped distri-
bu tion , is a theoretical distribu tion o scores in wh ich the m ean , m edian , an d m ode are equal.
1. Th e h igh est p oin t in th e distribu tion o scores is th e modal peak. In a bimodal distribution,
th ere are two m odal p eaks (e.g., two d istin ct p op u lation s).
2. In a n orm al distribution , ap p roxim ately 68% o the population scores all within 1, 95% all
with in 2, an d 99.7% all with in 3 stan dard deviation s o the m ean , respectively (Figure 26.1).

E. Skewed distributions. In a skewed distribution , the m odal peak shi ts to on e side (Figure 26.2).
1. In a positively skewed distribu tion (skewed to th e righ t), th e tail is toward th e righ t an d th e
m odal p eak is toward th e le t (i.e., scores clu ster toward th e low en d).
2. In a negatively skewed distribu tion (skewed to th e le t), th e tail is toward th e le t an d th e
m odal p eak is toward th e righ t (i.e., scores clu ster toward th e h igh en d).
308 BRS Behavioral Science

y
c
n
e
u
q
e
r
F
0.15% 2.35% 13.5% 34.0% 34.0% 13.5% 2.35% 0.15%
HELP OTHERS SO THAT GOD WILL HELP YOU.

–3 –2 –1 +1 +2 +3
68%

95%

99.7%
Are a unde r the c urve
FIGURE 26.1. The normal (gaussian) distribution. The number of standard deviations (S) (−3 to +3) from the mean is shown
on the x-axis. The percentage of the population that falls under the curve within each S is shown. (From Fadem B. High-
Yield Behavioral Science. 4th ed. Baltimore, MD: Lippincott Williams & Wilkins; 2013:126.)

Norma l

S kewe d to the right


(+ s kewe d)
e
d
o
o
e
e
M
M
M
M
n
d
d
n
d
a
e
a
i
i
RISE USMLE NEPAL

S kewe d to the le ft
(– s kewe d)

Bimoda l

Me a n
FIGURE 26.2. Frequency distributions. (From Fadem B. High-Yield Behavioral Science. 4th ed. Baltimore, MD: Lippincott
Williams & Wilkins; 2013:126.)
Chapter 26 Statistical Analyses 309

II. HYPOTHESIS TESTING


A. A hypothesis is a statem en t based on in eren ce, existin g literatu re, or p relim in ary stu dies
th at p ostu lates th at a d i eren ce exists b etween grou p s. Th e p ossib ility th at th is di eren ce
occu rred by ch an ce is tested u sin g statistical p rocedu res.

B. Th e null hypothesis, wh ich p ostu lates th at n o d i eren ce exists b etween grou p s, can
eith er be rejected or n ot rejected ollowin g statistical an alysis.
E ample of the null hypothesis:
HELP OTHERS SO THAT GOD WILL HELP YOU.

1. A grou p o 20 p atien ts wh o h ave sim ilar systolic blood p ressu res at th e begin n in g o a
stu d y (tim e 1) is d ivid ed in to two grou p s o 10 p atien ts each . On e grou p is given daily
doses o an exp erim en tal dru g m ean t to lower b lood p ressu re (exp erim en tal grou p ); th e
oth er grou p is given d aily doses o a p lacebo (p lacebo grou p ). Blood p ressu re in all 20
p atien ts is m easu red 2 weeks later (tim e 2).
2. Th e n u ll h yp oth esis assu m es th at th ere are n o sign i ican t di eren ces in blood p ressu re
between th e two grou p s at tim e 2.
3. I , at tim e 2, p atien ts in th e exp erim en tal grou p sh ow systolic blood p ressu res sim ilar
to th ose in th e p lacebo grou p, the null hypothesis (i.e., th ere is n o sign i ican t di eren ce
b etween th e grou p s) is not rejected.
4. I , at tim e 2, p atien ts in th e exp erim en tal grou p h ave sign i ican tly lower or h igh er blood
p ressu res th an th ose in th e p lacebo grou p, the null hypothesis is rejected.

C. Type I (α) and type II (β) error


1. Power (1 m in u s β) is th e ability to detect a d i eren ce between grou p s i it is tru ly th ere.
Th e larger th e sam p le size, th e m ore p ower a research er h as to detect th is di eren ce.
2. A type I error occu rs wh en th e n u ll h yp oth esis is rejected, alth ou gh it is tru e (e.g., th e dru g
really does n ot lower b lood p ressu re).
3. A type II error occu rs wh en th e n u ll h yp oth esis is n ot rejected, alth ou gh it is alse (e.g., th e
dru g really does lower blood p ressu re), bu t th ere m ay n ot h ave been en ou gh p ower to
detect th is d i eren ce.

D. Statistical probability
1. α is a p reset level o significance , u su ally set at 0.05 by con ven tion .
2. Th e P (probability) value is th e ch an ce o a typ e I error occu rrin g.
3. I a P valu e is equ al to or less th an 0.05, th e p reset α level, it is u n likely th at a typ e I error
RISE USMLE NEPAL

h as been m ade (i.e., a typ e I error is m ade 5 or ewer tim es ou t o 100 attem p ts).
4. Th ere ore, a P valu e equ al to or less th an 0.05 is gen erally con sidered to be statistically
significant.

E. Statistical versus clinical significance


1. Statistical sign i ican ce itsel d oes n ot tran slate in to clin ical im p ortan ce or sign i ican ce.
2. Th ere ore, th e qu estion ab ou t wh eth er a n ew treatm en t sh ou ld b e u sed in p ractice
requ ires evalu atin g its importance in the real world.

F. Testing hypotheses involving relative risk (RR) or odds ratio (OR) (and see Chapter 25,
Section III).
1. To test th e sign i ican ce o an association between a risk actor an d a disease, con iden ce
in terval estim ates o RR or OR can b e u sed.
2. Wh en RR = 1, or OR = 1, th ere is no association b etween th e risk actor an d th e disease.
3. I 1 is in th e con iden ce in terval, th e n u ll h yp oth esis is n ot rejected; i 1 is n ot in th e con i-
d en ce in terval, th e n u ll h yp oth esis is rejected.
310 BRS Behavioral Science

E ample 26.1. In a coh ort stu d y, th e relative risk (RR) or sm okin g in relation to ch ron ic obstru c-
tive p u lm on ary disease (COPD) is d eterm in ed by level o daily sm okin g (m easu red in p acks p er
day PPD]). Th e f n d in gs (see tab le b elow) su ggest th at p eop le wh o sm oke 1 PPD or m ore h ave
a sign if can tly in creased risk or COPD b u t th at risk or COPD is n ot sign if can tly elevated or
th ose wh o sm oke 0.5 PPD.
Exp la n a tio n : Th ere is a 1 in th e co n id en ce in ter va l (CI) a t 0.5 PPD so a n a sso cia tio n
b etween sm o kin g a t th a t level a n d COPD ca n n o t b e d em o n stra ted . In co n tra st, th ere is n o
1 in th e co n id en ce in terva ls at 1.0 PPD or h igh er levels o sm okin g so a sta tistica lly sign i i-
ca n t a sso cia tio n a t th e 95% CI exists b etween sm okin g 1.0 or m o re PPD a n d COPD.

RR for COPD 95% CI


HELP OTHERS SO THAT GOD WILL HELP YOU.

1.1 0.8–1.4
0.5 PPD
1 PPD 1.4 1.1–1.7
1.5 PPD 2.0 1.7–2.3
2 PPD 3.0 2.8–3.2
>2 PPD 3.5 3.2–3.8

III. STATISTICAL TESTS


Statistical tests are u sed to an alyze data rom m edical stu dies. Th e resu lts o statistical tests in di-
cate wh eth er to reject or n ot reject th e n u ll h yp oth esis. Statistical tests can be p aram etric or
n on p aram etric.

A. Parametric statistical tests for continuous, i.e., interval data


1. Pa ra m etric tests u se p o p u la tio n p a ra m eters (e.g., m ea n scores) a n d a re u su a lly u sed
to id en ti y th e p rese n ce o sta tistica lly sign i ica n t d i eren ces b etwee n gro u p s wh en
th e d istrib u tion o sco re s in a p op u la tion is n orm a l a n d wh en th e sa m p le size is
la rge.
2. Com m on ly u sed p aram etric statistical tests in clu de t-tests, analysis of variance (ANOVA),
an d linear correlation (Exam p le 26.1).
3. Linear correlation re ers to th e degree o relation sh ip between two con tin u ou s variables
RISE USMLE NEPAL

th at can b e assessed u sin g lin ear correlation coe icien ts (r) th at ran ge b etween −1 and +1.
a. I th e two variab les m ove in th e sam e d irection , r is positive (e.g., as h eigh t in creases,
b od y weigh t in creases, or as calorie in take decreases, body weigh t decreases).
b. I th e two variables m ove in op p osite d irection s, r is negative (e.g., as tim e sp en t exer-
cisin g in creases, body weigh t decreases).
Meta-analysis is a statistical m eth od o com bin in g th e statistical resu lts o a n u m ber o stu dies
to orm an overarch in g con clu sion .

B. Nonparametric statistical tests


1. I th e distribu tion o scores in a p op u lation is n ot n orm al or i th e sam p le size is sm all,
n on p aram etric statistical tests are u sed to evalu ate th e p resen ce o statistically sign i ican t
di eren ces b etween grou p s.
2. Com m on ly u sed n on p aram etric statistical tests in clu de Wilcoxon’s (ran k su m an d sign ed
ran k), Man n -Wh itn ey, an d Kru skal-Wallis.
Chapter 26 Statistical Analyses 311

C. Categorical tests. To an alyze categorical d ata or com p are p rop ortion s, th e chi-square test
or Fisher’s e act test (wh en th e sam p le size is sm all) (Exam p le 26.1) is u sed .

ExAMPLE 26.1. Commonly Used Statistical Tests


A con su m er grou p wou ld like to evalu ate th e su ccess o th ree d i eren t com m ercial weigh t
loss p rogram s. To do th is, m ale an d em ale su bjects are assign ed to on e o th ree p rogram s
(grou p A, grou p B, an d grou p C). Th e m ean weigh t o th e su b jects is n ot sign i ican tly d i er-
en t am on g th e th ree grou p s at th e start o th e stu d y (tim e 1). Each grou p ollows a di eren t
diet regim en . At th e en d o th e 6-week stu d y (tim e 2), th e su b jects are weigh ed an d th eir
h igh -den sity lip op rotein (HDL) levels are obtain ed. Exam p les o h ow statistical tests can be
u sed to an alyze th e resu lts o th is stu dy are given below.
HELP OTHERS SO THAT GOD WILL HELP YOU.

The t-test: Difference between the means of two samples


Independent (nonpaired) test: Tests th e m ean di eren ce in body weigh ts o th e su b-
jects in grou p A an d th e su bjects in grou p B at tim e 2 (i.e., two grou p s o su bjects
are sam p led on on e occasion ).
Dependent (paired) test: Tests th e m ean di eren ce in body weigh ts o th e su b-
jects in grou p A at tim e 1 an d tim e 2 (i.e., th e sam e p eop le are sam p led on two
occasion s).
Analysis of variance (ANOVA): Differences among the means of more than two samples
One-way ANOVA: Tests th e m ean di eren ces in body weigh ts o th e su bjects in
grou p A, grou p B, an d grou p C at tim e 2 (i.e., on e variable: grou p ).
Two-way ANOVA: It tests th e m ean di eren ces in body weigh ts o m en an d wom en
an d in body weigh ts o grou p A, grou p B, an d grou p C at tim e 2 (i.e., two vari-
ables: sex an d grou p ).
Correlation: Mutual relation between two continuous variables
Tests th e relation between HDL an d body weigh t in all su bjects at tim e 2.
Correlation coe icien ts (r) are n egative (0 to −1) i th e variables m ove in op p osite
direction s (e.g., as body weigh t decreases, HDL in creases) an d p ositive (0 to +1)
i th e variables m ove in th e sam e direction (e.g., as body weigh t decreases, HDL
decreases).
Chi-square test: Differences between frequencies in a sample; and Fisher’s exact
probability: Differences between frequencies in a small sample.
RISE USMLE NEPAL

Tests th e di eren ce am on g th e p ercen tage o su bjects with body weigh t o


140 p ou n ds or less in grou p s A, B, an d C at tim e 2.
From Fadem B. Beh avioral Scien ce in Medicin e. 2n d ed. Baltim ore, MD: Lip p in cott
William s & Wilkin s; 2012:319.
Review Test

Directions: Each o th e n u m b ered item s or in com p lete statem en ts in th is section is ollowed by


an swers or by com p letion s o th e statem en t. Select th e one lettered an swer or com p letion th at
is best in each case.
HELP OTHERS SO THAT GOD WILL HELP YOU.

Questions 1 and 2 4. Wh at p ercen tage o p eop le in a


p opu lation selected at ran dom wou ld be
A research stu d y is d esign ed to id en ti y th e expected to h ave systolic blood p ressu re at
m ean b od y weigh t o wom en b etween th e or above 140 m m Hg?
ages o 30 an d 39 in Los An geles. To d o th is, (A) 1.9%
a research er ob tain s th e b ody weigh ts o an (B) 2.5%
u n biased sam p le o 81 wom en in Los An geles (C) 13.5%
in th is age grou p. Th e m ean body weigh t o (D) 34.0%
th e wom en in th e sam p le is 135 p ou n d s with (E) 64.2%
a stan d ard d eviation o 18.
5. In a p op u lation o 500 p eop le selected
1. Wh at is th e estim ated stan dard error o at ran d om , h ow m an y p eop le wou ld b e
th e m ean or th is p op u lation ? expected to h ave systolic blood p ressu re
(A) 0.05 between 110 an d 120 m m Hg?
(B) 0.10 (A) 80
(C) 1.0 (B) 100
(D) 2.0 (C) 125
(E) 3.0 (D) 170
(E) 250
2. Wh at are, resp ectively, th e 95% an d 99%
con f d en ce in tervals or th is sam p le? 6. Wh at p ercen tage o th e p op u lation can
(A) 131–139 an d 130–140 p ou n ds be exp ected to h ave blood p ressu re th at alls
(B) 130–140 an d 131–139 p ou n ds with in 1 stan dard deviation o th e m ean ?
(C) 129–141 an d 130–135 p ou n ds (A) 0.15%
(D) 130–135 an d 129–141 p ou n ds (B) 2.35%
RISE USMLE NEPAL

(E) 131–139 an d 129–141 p ou n ds (C) 34%


(D) 68%
3. Wh en com p ared to th e 99% con f den ce (E) 95%
in terval, th e 95% con f d en ce in terval is
(A) less p recise an d less accu rate 7. Wh ich o th e ollowin g statistical tests
(B) m ore p recise b u t less accu rate is m ost ap p rop riately u sed to evalu ate th e
(C) m ore p recise an d m ore accu rate di eren ce in th e p ercen tage o wom en wh o
(D) less p recise an d m ore accu rate lose weigh t on a p rotein -sp arin g diet versu s
th e p ercen tage wh o lose weigh t on a h igh -
p rotein diet?
Questions 4–6 (A) Paired t-test
(B) An alysis o varian ce
Systolic b lood p ressu re is n orm ally d istrib - (C) Ch i-squ are test
u ted with a m ean o 120 m m Hg an d a stan - (D) Correlation
dard deviation o 10. (E) In dep en den t t-test

312
Chapter 26 Statistical Analyses 313

8. Wh ich o th e ollowin g statistical tests Questions 12–14


is m ost ap p rop riately u sed to evalu ate th e
di eren ces b etween in itial bod y weigh t On a gross an atom y qu iz, test scores o 10, 10,
an d f n al b ody weigh t or each wom an on a 10, 70, 40, 20, an d 90 are obtain ed by seven
p rotein -sp arin g diet? stu d en ts in a laboratory grou p.
(A) Paired t-test
(B) An alysis o varian ce 12. Wh ich o th e ollowin g correctly
(C) Ch i-squ are test describes th ese qu iz scores?
(D) Correlation (A) Positively skewed
(E) In dep en den t t-test (B) A n orm al d istrib u tion
(C) Negatively skewed
HELP OTHERS SO THAT GOD WILL HELP YOU.

9. Wh ich o th e ollowin g statistical tests (D) Th e m ode is h igh er th an th e m ean


is m ost ap p rop riately u sed to evalu ate th e (E) Th e m ode is equ al to th e m ean
relation sh ip between body weigh t an d
systolic blood p ressu re in a grou p o 25-year- 13. Th e m edian o th ese qu iz scores is
old wom en ?
(A) 10
(A) Paired t-test (B) 20
(B) An alysis o varian ce (C) 40
(C) Ch i-squ are test (D) 70
(D) Correlation (E) 90
(E) In dep en den t t-test
14. I th e class teach in g assistan t erred an d
10. In a stu dy to determ in e th e u se u ln ess record ed th e grad e o on e stu den t wh o got
o a n ew an tih yp erten sive m ed ication , 12 a 10 as 100, th e m ean , m edian , an d m ode
h yp erten sive p atien ts are given th e n ew wou ld, resp ectively
d ru g an d 10 h yp erten sive p atien ts are given
(A) in crease, in crease, in crease
a p lacebo. Th e dep en d en t variab le in th is
(B) in crease, n ot ch an ge, n ot ch an ge
stu dy is
(C) in crease, in crease, n ot ch an ge
(A) th e exp erim en ter’s bias (D) in crease, n ot ch an ge, n ot ch an ge
(B) givin g th e p atien ts th e dru g (E) n ot ch an ge, in crease, n ot ch an ge
(C) givin g th e p atien ts a p lacebo
(D) th e p atien ts’ blood p ressu re ollowin g 15. A p h arm aceu tical com p an y claim s th at
treatm en t with th e dru g or p lacebo th eir n ew dru g or gen ital h erp es d ecreases
(E) th e daily variability in th e p atien ts’ blood th e p ersisten ce o an ou tbreak rom 5.5 to
p ressu re be ore th e dru g treatm en t 5.3 days. Th is d i eren ce is sign if can t at th e
95% con f den ce in terval. Th is resu lt
11. An alysis o th e d ata rom a large research
RISE USMLE NEPAL

(A) is clin ically sign i ican t bu t n ot


stu dy reveals a P valu e o 0.001. Th ese resu lts
statistically sign i ican t
in d icate th at th e research er
(B) is n ot clin ically sign i ican t bu t is
(A) h as com m itted a typ e I error statistically sign i ican t
(B) h as com m itted a typ e II error (C) is n eith er clin ically n or statistically
(C) can reject th e n u ll h yp oth esis sign i ican t
(D) can n ot reject th e n u ll h yp oth esis (D) is both clin ically an d statistically
(E) h as biased th e stu dy sign i ican t
314 BRS Behavioral Science

16. For a grou p o 20 elderly p atien ts aged 85 17. At an Am erican m ed ical sch ool, a stu d y
to 90 years, blood p ressu re m easu rem en ts is don e to evalu ate th e relation sh ip between
are taken an d recorded th ree tim es p er day p aren tal in com e (in th ou san ds o d ollars p er
over a 2-week p eriod . At th e en d o th e 2 year) an d USMLE Step 1 scores. Wh ich o th e
weeks, th e p atien ts’ blood p ressu res are ollowin g is m ost likely to be th e correlation
taken an d record ed ou r tim es p er d ay or th e coe f cien t (r) or th is relation sh ip as sh own
n ext 2 weeks. I th e p atien ts’ b lood p ressu res by these data?
all with in a n orm al d istribu tion , wh at will
be th e e ect o takin g th e b lood p ressu res

e
240+

r
ou r tim es p er day rath er th an th ree tim es

o
c
240

S
p er d ay on th e stan d ard deviation an d sh ap e

1
220

p
HELP OTHERS SO THAT GOD WILL HELP YOU.

o the resu ltin g cu rve?

e
200

t
S
(A) Stan d ard d eviation d ecreases; sh ap e o

E
180

L
M
th e cu rve d oes n ot ch an ge. 160

S
U
(B) Stan d ard d eviation in creases; sh ap e o 140
20 30 40 50 60 70 80 90 100+
th e cu rve d oes n ot ch an ge.
Pa re nta l income (in thous a nds of dolla rs )
(C) Stan d ard d eviation d ecreases; sh ap e o
th e cu rve is p ositively skewed . (A) 1.40
(D) Stan d ard d eviation in creases; sh ap e o (B) 0.50
th e cu rve is n egatively skewed. (C) 0
(E) Stan d ard d eviation d oes n ot ch an ge; (D) −0.25
sh ap e o th e cu rve d oes n ot ch an ge. (E) −0.75
RISE USMLE NEPAL
An swers an d Exp lan ation s

Typical Board Question


The answer is A. Th e grap h sh ows th e h igh er an d lower con iden ce level lim its o relative risk or
h ead an d n eck can cer (HNC) at di eren t levels o alcoh ol con su m p tion . Becau se th e con iden ce
in terval in clu d es 1.0 with alcoh ol con su m p tion o 12 an d 24 gram s/ day, a sign i ican tly in creased
HELP OTHERS SO THAT GOD WILL HELP YOU.

risk o HNC at th ese levels o con su m p tion can n ot be dem on strated. Becau se th e con iden ce
in terval d oes n ot in clu d e 1.0 at levels o d aily alcoh ol con su m p tion > 36 gram s/ day, a statisti-
cally sign i ican t association exists between alcoh ol con su m p tion at th is level an d risk or HNC
in th is stu dy.

1. The answer is D. 2. The answer is A. Th e estim ated stan dard error o th e m ean (SE) equ als
th e sam p le stan dard d eviation (18) divid ed by th e squ are root o 81 = 9. Th e SE is th ere ore
18/ 9 = 2. Con id en ce in terval (CI) sp eci ies th e in terval in wh ich th e tru e p op u lation m ean
lies. Th e CI is equ al to th e m ean o th e sam p le (X) p lu s or m in u s th e z score. Th e 95% CI an d
99% CI equal th e m ean plus or m in us 2.0 (SE) an d 2.5 (SE), resp ectively, th at is, 135 ± 4
(95% con iden ce in terval) an d 135 ± 5 (99% con iden ce in terval).
3. The answer is B. With resp ect to estim atin g th e m ean , p recision re lects h ow reliab le th e
estim ate is an d accu racy re lects h ow close th e estim ate is to th e tru e m ean . Th e wider
th e CI, th e less p recise an d th e m ore accu rate th e estim ate o th e m ean . Wh en com p ared
to a 99% con id en ce in terval, a 95% con id en ce in terval will be m ore p recise (sm aller
SE an d wid th o th e con iden ce in terval) bu t less accu rate (th e sam p le is less likely to be
rep resen tative).
4. The answer is B. 5. The answer is D. 6. The answer is D. Systolic blood p ressu re o 140
m m Hg is 2 stan dard d eviation s ab ove th e m ean (120 m m Hg). Th e area u n der th e cu rve
b etween 2 an d 3 stan dard d eviation s ab ove th e m ean is abou t 2.35% p lu s abou t 0.15%
(everyth in g ab ove 3 stan d ard deviation s). Th u s, a total o abou t 2.5% o th e p eop le will
h ave b lood p ressu res o 140 m m Hg an d above. Systolic blood p ressu re between 110 an d
120 m m Hg is 1 stan dard deviation below th e m ean . Th e p ercen tage o p eop le in th is area
on a n orm al cu rve is 34%. Th u s, 34% o 500 p eop le, or 170 p eop le, will h ave systolic blood
p ressu re in th e ran ge o 110–120 m m Hg. “With in” in cludes 1 stan dard deviation below
RISE USMLE NEPAL

(34%) p lu s 1 stan dard d eviation above (34%) th e m ean or a total o 68%. Th u s, a total o
68% o th e p op u lation can be exp ected to h ave blood p ressu re th at alls with in 1 stan dard
d eviation o th e m ean .
7. The answer is C. Th e ch i-squ are test is u sed to exam in e d i eren ces b etween requ en cies in
a sam p le, in th is case, th e p ercen tage o wom en wh o lose weigh t on a p rotein -sp arin g diet
versu s th e p ercen tage o wom en wh o lose weigh t on a h igh -p rotein diet.
8. The answer is A. Th e t-test is u sed to exam in e di eren ces between m ean s o two sam p les.
Th is is an exam p le o a p aired t-test becau se th e sam e wom en are exam in ed on two
d i eren t occasion s.
9. The answer is D. Correlation is u sed to exam in e th e relation sh ip between two con tin u ou s
variables—in th is case, systolic blood p ressu re an d body weigh t.

315
316 BRS Behavioral Science

10. The answer is D. Th e d ep en den t variab le is a m easu re o th e ou tcom e o an exp erim en t.


In th is case, blood p ressu re ollowin g treatm en t with th e dru g or p lacebo is th e dep en den t
variab le. Th e in dep en d en t variab le is a ch aracteristic th at an exp erim en ter exam in es
to see i it ch an ges th e ou tcom e. In th is case, givin g th e p atien t a dru g or p lacebo is th e
in dep en den t variable.
11. The answer is C. With a P valu e o 0.001 (wh ich is sm aller th an th e p reset a level o 0.05),
th e in din gs are statistically sign i ican t an d th e research er can reject th e n u ll h yp oth esis.
A typ e I error occu rs wh en th e n u ll h yp oth esis is rejected , alth ou gh it is tru e. A typ e II error
occu rs wh en th e n u ll h yp oth esis is n ot rejected , alth ou gh it is alse. Th ere is n o eviden ce
h ere o a typ e I or typ e II error or th at th e stu dy is biased (see Ch ap ter 25).
HELP OTHERS SO THAT GOD WILL HELP YOU.

12. The answer is A. 13. The answer is B. 14. The answer is C. Becau se o all th e low scores, th e
d istribu tion o th ese test scores is skewed to th e righ t (p ositively skewed). Also, th e m ode
(10) o th ese scores is lower th an th e m ean (35.7), a ch aracteristic o a p ositively skewed
d istribu tion . In a n egatively skewed distribu tion (skewed to th e le t), th e tail is toward
th e le t (i.e., scores clu ster toward th e h igh en d ). In a n orm al distrib u tion , th e m ean ,
m edian , an d m ode are equ al. Wh en th ey are sequ en tially ordered, th e m edian (m iddle
valu e) o th ese scores is 20. I th e class teach in g assistan t erred an d recorded th e grade o
on e stu den t wh o got a 10 as 100, th e m ean wou ld in crease to 48.6 an d th e m edian wou ld
in crease to 40; th e m ode wou ld stay th e sam e at 10.
15. The answer is B. It is u n likely th at p atien ts will take th e n ew d ru g (an d d eal with its side
e ects) to secu re th is sm all redu ction (i.e., 0.2 days) in len gth o ou tb reaks. Th ere ore,
alth ou gh th ese resu lts are statistically sign i ican t, th ey are u n likely to be o clin ical
sign i ican ce.
16. The answer is A. Wh en p atien ts’ blood p ressu res are m easu red ou r rath er th an th ree tim es
p er day, becau se th ere is a h igh er n (i.e., m ore data p oin ts are u sed to calcu late th e m ean ),
th e stan dard d eviation d ecreases. However, th e in creased n u m ber o data p oin ts does n ot
a ect th e sh ap e o th e cu rve.
17. The answer is B. Th e correlation between p aren tal in com e an d USMLE Step 1 scores as
sh own by th ese data is p ositive (i.e., as p aren tal in com e in creases, scores in crease). Sin ce
a correlation coe icien t (r) can n ot b e m ore th an 1, th e on ly p ossible an swer is 0.50 ( or
in orm ation on a sim ilar stu dy, see Fadem , Sch u ch m an , an d Sim rin g, Academ ic Medicin e,
1995).
RISE USMLE NEPAL
Com p reh en sive Exam in ation

Directions: Each o th e n u m b ered item s or in com p lete statem en ts in th is section is ollowed by


an swers or by com p letion s o th e statem en t. Select th e one lettered an swer or com p letion th at
is best in each case.

1. A 38-year-old wom an com es to the Questions 4 and 5


HELP OTHERS SO THAT GOD WILL HELP YOU.

em ergen cy room with a h eart rate o 180


beats/ m in an d trem ulousn ess. She reports A 25-year-old gradu ate stu den t wh o was
eelin g extrem ely an xiou s. Blood testin g diagn osed with sch izop h ren ia 1 year ago h as
reveals in creased T4 an d su p p ressed th yroid- been takin g h alop eridol 10 m g twice daily or
stim u latin g h orm on e (TSH); th e th yroid th e last m on th . Th e p atien t’s com p lain ts at
glan d is n ot p alp able. Th e p atien t tells th e th is o ice visit in clu de blu rred vision , con sti-
doctor th at sh e does n ot take an y m edication . p ation , u rin ary reten tion , an d dry m ou th .
In act, sh e is secretly takin g th yroxin e to lose
weight. Which o th e ollowin g is th e best 4. Th e u n derlyin g m ech an ism th at wou ld
diagn osis or this patien t at this tim e? best exp lain th ese sym p tom s is th at
(A) Hyp erp arath yroid ism h alop eridol blocks
(B) Graves d isease (A) h istam in ic recep tors
(C) Hash im oto th yroiditis (B) alp h a 1-ad ren ergic recep tors
(D) Factitiou s h yp erth yroidism (C) alp h a 2-ad ren ergic recep tors
(E) Som atic sym p tom disorder (D) m u scarin ic recep tors
(E) seroton ergic recep tors
2. A doctor advises a 40-year-old wom an wh o
drin ks alcohol excessively that she n eeds to 5. On e week later, th e p atien t p resen ts to
stop drin kin g. The patien t n otes that she has th e em ergen cy room with ever, tach ycardia,
already cut back an d p lan s to stop com p letely trem or, an d rigidity. Th e m ost likely
on the rst o the n ext m on th. Accordin g to exp lan ation or th is p atien t’s sym p tom s is
th e “Stage o Ch an ge” m odel, in wh ich stage (A) acu te d yston ia
o ch an ge is th is p atien t m ost likely to be? (B) tardive dyskin esia
(A) Precon tem p lation (C) agran u locytosis
(B) Con tem p lation (D) n eu rolep tic m align an t syn drom e
RISE USMLE NEPAL

(C) Prep aration (E) seroton in syn drom e


(D) Action
(E) Main ten an ce 6. A 42-year-old m an p resen ts to a p h ysician
with sign s an d sym p tom s su ggestin g
3. A p atien t with leu kem ia wh o is exp ected th at h e is exp erien cin g an ep isode o
to live ab ou t 3 m on th s, asks th e d octor m ajor dep ression . Wh ich o th e ollowin g
wh at sh e sh ou ld tell h er 6-year-old ch ild sym p tom s sh ou ld th e doctor b e m ost
when h e asks h er abou t h er con dition . Most con cern ed abou t at th is tim e?
ap p rop riately, th e doctor sh ou ld recom m en d (A) Su icidal p lan n in g
th at th e m oth er say (B) Su icidal ideation
(A) “I h ave a severe illn ess an d you can ask (C) Weigh t loss
an y qu estion s you wan t.” (D) Di icu lty sleep in g
(B) “I am d yin g b u t you sh ou ld n ot cry.” (E) Lack o en ergy
(C) “I h ave leu kem ia, a d isease o wh ite
blood cells th at m u ltip ly in a d isordered
m an n er.”
(D) “I am sick n ow b u t I will b e in e.”
(E) Noth in g abou t h er con d ition .

317
318 BRS Behavioral Science

7. A 34-year-old wom an wh o was rap ed 2 m on th s ago. Ph ysical exam in ation sh ows


as a teen ager is overp rotective o h er own eryth em atou s p ap u les an d p u stu les on th e
16-year-old dau gh ter. Sh e re u ses to allow girl’s oreh ead . Wh ich o th e ollowin g is th e
h er dau gh ter to go to classm ates’ h ou ses m ost likely cau se o th e exacerb ation o th is
a ter sch ool or to socialize with rien ds on p atien t’s acn e?
weeken d s. Th e m oth er also sp en ds m an y (A) Allergic reaction to m aterials u sed in
h ours clean in g h er own h ou se an d car. Th e road con stru ction
de en se m ech an ism th at th is wom an is u sin g (B) Allergy to th e ch in ch illa
to deal with h er own n egative early sexu al (C) Ch ocolate con su m p tion
exp erien ce is m ost likely to b e wh ich o th e (D) Excessive su n exp osu re
ollowin g? (E) Vegetarian diet
HELP OTHERS SO THAT GOD WILL HELP YOU.

(A) Su blim ation (F) Wearin g a h elm et


(B) Den ial
(C) Reaction orm ation 11. A p h ysician id en ti es a rare disease in
(D) Un doin g a 42-year-old p atien t. Th e resu lts o wh at
(E) Ration alization kin d o stu dy wou ld serve th e p h ysician best
in gettin g in orm ation abou t th is d isease to
8. Over th e p ast week, a 40-year-old u ltim ately h elp th is p atien t.
m an with sch izop h ren ia states th at h e (A) Case rep ort
h as som etim es h eard voices com in g (B) Case–con trol
rom ou tside o h is h ead wh en n o on e (C) Coh ort
was p resen t. Th e p atien t does n ot sh ow (D) Clin ical trial
delu sion al th in kin g an d h as an ap p rop riate (E) Mu ltip le case rep orts
bu t blu n ted acial exp ression . His sp eech
is clear, an d h is th ou gh ts ollow each oth er 12. I h yp erten sion is distribu ted n orm ally
logically. On th e d im en sion s o severity scale an d is de n ed as h avin g a systolic blood
or sch izop h ren ia, th is p atien t will m ost p ressu re at least 2 stan d ard deviation s
likely h ave a score closest to above th e m ean , wh at p ercen tage o p eop le
(A) 0 in a given p op u lation are likely to be
(B) 4 h yperten sive?
(C) 10 (A) 1%
(D) 12 (B) 2.5%
(E) 18 (C) 5%
(D) 10%
9. A 7-year-old b oy h as a term in al illn ess. (E) 34%
His p aren ts h ave told h im th at h e is goin g to
die. Wh ich o th e ollowin g is m ost likely to 13. Wh ile in h er teen s, a 22-year-old wom an
RISE USMLE NEPAL

ch aracterize th is ch ild’s con cep tion o d eath ? h ad an orexia n ervosa or a 5-year p eriod. Sh e
(A) Th at oth ers can die bu t h e can n ot die h as recovered bu t is n ow at h igh est risk or
(B) Th at h e can die bu t oth ers can n ot die wh ich o th e ollowin g con d ition s?
(C) Th at everyon e dies at som e tim e (A) Derm atitis
(D) Th at p eop le die bu t th en com e back (B) Osteoarth ritis
to li e (C) Am en orrh ea
(D) Osteop orosis
10. A 16-year-old girl com es to th e (E) Biliary atresia
p h ysician in late Au gu st b ecau se o
worsen in g acn e over h er oreh ead or 14. Wh ich o th e ollowin g agen ts u sed to
th e p ast ew m on th s. Sh e tells th e d octor treat p atien ts with Alzh eim er’s d isease is n ot
th at sh e b egan a vegetarian d iet 6 m on th s an acetylch olin esterase in h ibitor?
ago an d sin ce th en h as b een cravin g an d
(A) Galan tam in e
eatin g large am ou n ts o ch ocolate. Over th e
(B) Rivastigm in e
su m m er, sh e h as b een workin g ou td oors as
(C) Mem an tin e
a f ag p erson or a local road con stru ction
(D) Tacrin e
com p an y an d is req u ired to wear a h elm et.
(E) Don ep ezil
Her b roth er received a ch in ch illa or a p et
Comprehensive Examination 319

15. An overweigh t m iddle-aged wom an 19. A ter a 20-year-old wom an in gests


h as ju st been diagn osed with sleep ap n ea. red win e an d aged ch eese at a restau ran t,
Medical exam in ation an d laboratory test sh e is b rou gh t to th e em ergen cy d ep artm en t
results are u n rem arkable. Wh ich o the with elevated blood p ressu re an d a
ollowin g is th e m ost ap p rop riate m edication severe occip ital h ead ach e. Th e typ e o d ru g
to m an age sleep ap n ea in th is p atien t? m ost likely to h ave cau sed th is clin ical
(A) Diazep am p ictu re is
(B) Flu oxetin e (A) an an tid ep ressan t agen t
(C) Medroxyp rogesteron e acetate (B) an an tip sych otic agen t
(D) Im ip ram in e (C) an an tim an ic agen t
(E) Alp razolam (D) a b en zod iazep in e
HELP OTHERS SO THAT GOD WILL HELP YOU.

(E) a b arb itu rate


16. Du rin g a rou tin e p h ysical exam in ation , a
p h ysician d iscovers th at an alert 88-year-old
20. Wh ich o th e ollowin g p atien ts is at
m an in a n u rsin g h om e h as bru ises on h is
h igh est risk or su icid e?
righ t leg an d righ t arm b u t n o oth er m ed ical
n din gs. Wh en th e d octor asks th e p atien t (A) A 55-year-old divorced wom an
ab ou t th e bru ises, th e p atien t says, “At m y (B) A 55-year-old divorced m an
age wh at are you goin g to d o ab ou t it?” Th e (C) A 55-year-old m arried wom an
d octor n otes th at wh ile sp eakin g with h im , (D) A 55-year-old m arried m an
th e p atien t avoid s m akin g eye con tact. Wh at (E) A 55-year-old widowed wom an
is th e p h ysician’s n ext step in m an agem en t?
21. Wh ich o th e ollowin g is m ost likely to
(A) Ask th e p atien t abou t h is relation sh ip
be seen in h ealth y 50-year-old m en op au sal
with h is caretakers
wom en in all cu ltu res?
(B) Su ggest th at th e sta restrain th e p atien t
or h is sa ety (A) Th e “em p ty n est” syn drom e
(C) Assess th e p atien t to ru le ou t dem en tia (B) Dep ression
(D) Assess th e p atien t to ru le ou t deliriu m (C) An xiety
(E) Write an order to h ave rails p u t on th e (D) In som n ia
sid es o th e p atien t’s b ed (E) Hot lash es or lu sh es

17. A 19-year-old girl wh o h as b een at a 22. Patien ts an d p h ysician s com m on ly


p arty is b rou gh t to th e em ergen cy room a ter p re er selective seroton in reu p take in h ib itors
h avin g a seizu re. Her blood is p ositive or (SSRIs) to tricyclic an tidep ressan ts becau se
alp razolam , cocain e, an d m ariju an a. Wh ich SSRIs are m ore likely to
o th e ollowin g is m ost likely to h ave cau sed
(A) elevate m ood
th e seizu re in th is p atien t?
(B) work qu ickly
(A) Use o alp razolam
RISE USMLE NEPAL

(C) lower blood p ressu re


(B) Use o cocain e (D) en h an ce sleep
(C) Use o m ariju an a (E) be well-tolerated
(D) With drawal o alp razolam
(E) With drawal o cocain e 23. A 28-year-old m an who is a raid to drive
(F) With drawal o m ariju an a a car is taught relaxation techn iques an d is
18. Th e m oth er o a 15-year-old b oy wh o h as then shown a photograph o a m an drivin g
asthm a tells th e doctor th at th e boy re u ses a car. Later in treatm en t, while relaxed, he
to u se h is in h aler in sch ool. To in crease th e is shown people drivin g real cars. Fin ally, he
b oy’s ad h eren ce, th e d octor sh ou ld m ost drives a car. This treatm en t techn ique is best
ap prop riately described as

(A) recom m en d th at th e m oth er an d th e boy (A) im p losion


go or cou n selin g togeth er (B) bio eedb ack
(B) exp lain to th e b oy th at h e can d ie i h e (C) aversive con dition in g
does n ot u se th e in h aler (D) token econ om y
(C) sh i t th e boy to oral m ed ication (E) loodin g
(D) in stru ct th e boy to go to th e sch ool n u rse (F) system atic d esen sitization
wh en ever h e h as to u se h is in h aler (G) cogn itive th erapy
(E) p u t th e boy in con tact with a su p p ort
grou p o teen s with asth m a
320 BRS Behavioral Science

24. A 59-year-old m ale p atien t h as ju st sh e states th at th e h eadach es started 3 years


recovered rom a m yocard ial in arction . ago wh en h er n eigh bors began to com e
Du rin g a ollow-u p exam in ation , h e asks in to h er h om e an d h arass h er at n igh t.
th e p h ysician wh at is th e sa est p osition Th ere is n o evid en ce th at th e n eigh b ors are
or sexu al in tercou rse with h is wi e. Th e d oin g th is. Th e p atien t h as n o h istory o
p h ysician’s b est recom m en d ation is p sych iatric illn ess, an d p h ysical exam in ation
(A) ace to ace, lyin g on th eir sides is u n rem arkable. Sh e h as good social an d
(B) ace to ace, em ale su p erior work relation sh ip s an d , excep t or h er belie s
(C) ace to ace, m ale su p erior abou t th e n eigh bors, h er th ou gh ts seem
(D) m ale b eh in d em ale, lyin g on th eir sid es clear, logical, an d ap p rop riate. At th is tim e,
(E) avoid sexu al activity or at least 1 year th e m ost ap p rop riate diagn osis or th is
HELP OTHERS SO THAT GOD WILL HELP YOU.

wom an is
25. A m ild ly d em en ted 83-year-old m an is (A) sch izop h ren ia
brou gh t to th e em ergen cy d ep artm en t by h is (B) bip olar disord er
daugh ter, with wh om h e lives. He sm ells o (C) delu sion al d isorder
u rin e, is u n dern ou rish ed, an d h as bru ises on (D) sch izoa ective disorder
both o h is arm s an d abrasion s on on e wrist. (E) sch izoid p erson ality disord er
He seem s ear u l b u t d en ies th at an yon e
h as h arm ed h im . Th e m ost ap p rop riate rst 29. A 26-year-old wom an believes th at sh e
action or th e p h ysician to take a ter treatin g is p regn an t with Ash ton Ku tch er’s ch ild . Sh e
th e p atien t is to h as n ever m et th e actor, an d two p regn an cy
(A) sp eak to th e d au gh ter ab ou t th e tests are n egative. Th ere is n o oth er eviden ce
p ossibility th at th e m an h as been abu sed o a th ou gh t disorder. Th e m ost ap p rop riate
(B) sen d h im h om e with h is dau gh ter as d iagn osis or th is wom an is
soon as p ossib le (A) sch izop h ren ia
(C) con tact th e state social service agen cy (B) bip olar disord er
th at deals with elder abu se (C) delu sion al d isorder
(D) order a n eu rologic evalu ation (D) som atization d isorder
(E) release th e p atien t in to th e care o (E) sch izoid p erson ality disord er
an oth er relative
30. A 40-year-old wom an goes to h er
26. O th e ollowin g, th e eth n ic grou p with gyn ecologist or a yearly ch ecku p. Wh ich o
th e lon gest li e exp ectan cy is th e ollowin g is m ost likely to cau se death in
(A) Hisp an ic Am erican s a wom an o th is age?
(B) A rican Am erican s (A) Pregn an cy an d ch ildb irth
(C) Wh ite Am erican s (B) An in trau terin e device
(D) Native Am erican s (C) Oral con tracep tives
RISE USMLE NEPAL

(D) Barrier con tracep tives


27. In th e Un ited States, wh ich o th e (E) A p rogesteron e im p lan t
ollowin g is th e m ost com m on b elie
con cern in g m en tal illn ess? 31. A 29-year-old p h ysician with severe
(A) It is th erap eu tic to d iscu ss you r in tern al p soriasis on h is h an d s an d arm s asks a
em otion al p roblem s with oth ers. colleagu e h ow h e sh ou ld d eal with th e
(B) Men tal illn ess sign i ies p erson al reaction s o p atien ts wh en th ey n otice h is
weakn ess. con d ition . Th e colleagu e’s b est resp on se is
(C) Un con sciou s con licts can be m an i ested (A) “Act like n oth in g is wron g.”
as p h ysical illn ess. (B) “Wear lon g-sleeved sh irts.”
(D) Men tally ill p eop le h ave good (C) “Reassu re p atien ts th at th e skin
sel -con trol. con dition you h ave is n ot con tagiou s.”
(E) Men tally ill p eop le u su ally seek h elp. (D) “See as ew p atien ts as p ossib le.”
(E) “Tell p atien t’s th at it is you r p roblem , n ot
28. A 60-year-old wom an p resen ts with th eirs.”
ch ron ic h ead ach es. Du rin g th e in terview,
Comprehensive Examination 321

32. Th e u su al stan d ard s o doctor–p atien t 36. An in an t’s ability to roll over rom back
con den tiality are m ost likely to ap p ly to to belly an d belly to b ack u su ally begin s at
wh ich o th e ollowin g p atien ts? wh at age?
(A) A m an wh o tells h is p h ysician th at h e (A) 0–3 m on th s
p lan s to sh oot h is p artn er (B) 4–6 m on th s
(B) A recen tly b ereaved wom an wh o tells h er (C) 7–10 m on th s
p h ysician th at sh e h as h ad occasion al (D) 12–15 m on th s
th ou gh ts o su icide (E) 16–30 m on th s
(C) A m an wh o tells h is p h ysician th at h e h as
been sexu ally ab u sin g h is 10-year-old 37. A p atien t in th e em ergen cy dep artm en t
step d au gh ter h as ju st been in volved in a car acciden t.
HELP OTHERS SO THAT GOD WILL HELP YOU.

(D) An HIV-p ositive m an wh o is en gagin g in Th e p h ysician su sp ects th at sh e h as b een


sexu al in tercou rse with h is wi e with ou t drin kin g. In m ost states, th e lowest blood
u sin g con dom s alcoh ol con cen tration (BAC) to m eet th e
(E) A d ep ressed wom an wh o tells h er criterion or legal in toxication alls in to
p h ysician th at sh e h as saved u p 50 wh ich o th e ollowin g ran ges?
barb itu rate tablets an d wan ts to die (A) 0.01%–0.02%
33. Doctor A is aware th at Doctor B h as (B) 0.05%–0.15%
m ade a seriou s m istake in treatin g a very (C) 0.40%–0.50%
ill h osp italized p atien t. Doctor B re u ses (D) 1.5%–2.0%
to ad m it th at h e h as m ad e a m istake. Most (E) 2.5%–3.0%
ap prop riately, Doctor A sh ou ld
38. O th e ollowin g agen ts, wh ich is
(A) talk to Doctor B again abou t h is m istake
th e m ost ap p rop riate h eterocyclic
(B) warn Doctor B th at h e will be rep orted i
an tidep ressan t or a 45-year-old air tra c
h e con tin u es to m ake m istakes
con troller wh o m u st stay alert on th e
(C) rep ort Doctor B’s action to Doctor B’s
job?
su p erior at th e h osp ital
(D) rep ort Doctor B’s action to th e p olice (A) Selegilin e
(E) recom m en d th at Doctor B be tran s erred (B) Tran ylcyp rom in e
to an oth er h osp ital (C) Trazodon e
(D) Doxep in
34. To ollow im p rovem en t or d eterioration (E) Am oxap in e
over tim e in a 70-year-old p atien t with (F) Flu oxetin e
su sp ected n eu rologic dys u n ction , wh ich o (G) Protrip tylin e
th e ollowin g is th e m ost ap p rop riate test? (H) Nortrip tylin e
(A) Positron em ission tom ograp h y (PET) (I) Am itrip tylin e
(B) Com p u ted tom ograp h y (CT) (J ) Im ip ram in e
RISE USMLE NEPAL

(C) Am obarbital sodiu m (Am ytal) in terview


(D) Th em atic ap p ercep tion test (TAT) 39. A 79-year-old wom an rep orts th at
(E) Electroen cep h alogram (EEG) sh e h as di cu lty sleep in g th rou gh th e
(F) Wid e Ran ge Ach ievem en t Test (WRAT) n igh t becau se o p ersisten t m u scu lar
(G) Folstein Min i–Men tal State Exam in ation con traction s in h er legs. Wh ich o th e
(H) Glasgow Com a Scale ollowin g sleep d isord ers b est m atch es th is
p ictu re?
35. To evalu ate u n con sciou s con f icts in a
20-year-old m an u sin g d rawin gs d ep ictin g (A) Klein e-Levin syn drom e
am bigu ou s social situ ation s, wh ich o th e (B) Nigh tm are disorder
ollowin g is th e m ost ap p rop riate test? (C) Sleep terror disord er
(D) Sleep dru n ken n ess
(A) Positron em ission tom ograp h y (PET) (E) Circadian rh yth m sleep disorder
(B) Com p u ted tom ograp h y (CT) (F) Noctu rn al m yoclon u s
(C) Am obarbital sodiu m (Am ytal) in terview (G) Restless legs syn d rom e
(D) Th em atic ap p ercep tion test (TAT)
(E) Electroen cep h alogram (EEG)
(F) Wid e Ran ge Ach ievem en t Test (WRAT)
(G) Folstein Min i–Men tal State Exam in ation
(H) Glasgow Com a Scale
322 BRS Behavioral Science

40. A p ilot wh ose p lan e is ab ou t to cra sh d ay. O th e ollowin g e ects, wh ich is m ost
sp en d s 5 m in u tes exp lain in g th e tech n ica l likely in th is p atien t?
d etails o th e en gin e m a l u n ction to h is (A) Blood p ressu re red u ction
cop ilot. Th e d e en se m ech a n ism th at th e (B) Leth argy
p ilot is u sin g to d eal with h is own (C) Tach ycardia
an xiety is (D) Decreased gastric acid secretion
(A) rep ression (E) Dep ressed m ood
(B) su b lim ation
(C) dissociation 45. Th e social sm ile com m on ly rst ap p ears
(D) regression at wh at age in typ ical in an ts?
(E) in tellectu alization (A) 0–3 m on th s
HELP OTHERS SO THAT GOD WILL HELP YOU.

(B) 4–6 m on th s
41. Most typ ical ch ild ren b egin to walk (C) 7–10 m on th s
with ou t assistan ce at wh at age? (D) 12–15 m on th s
(A) 0–3 m on th s (E) 16–30 m on th s
(B) 4–6 m on th s 46. To evaluate readin g and arithm etic skills in
(C) 7–10 m on th s a 30-year-old hospitalized m ale patient, which
(D) 12–15 m on th s o the ollowin g is the m ost appropriate test?
(E) 16–30 m on th s
(A) Positron em ission tom ograp h y (PET)
42. A 40-year-old wom an with ten sion (B) Com p u ted tom ograp h y (CT)
h eadach es h as th e ten sion in th e ron talis (C) Am obarbital sodiu m (Am ytal) in terview
m u scle m easu red regu larly. Th e readin gs are (D) Th em atic ap p ercep tion test (TAT)
p rojected to h er on a com p u ter screen . Sh e (E) Electroen cep h alogram (EEG)
is th en tau gh t to u se m en tal tech n iqu es to (F) Wide Ran ge Ach ievem en t Test (WRAT)
decrease ten sion in th is m u scle. Wh ich o (G) Folstein Min i–Men tal State Exam in ation
th e ollowin g treatm en t tech n iqu es d oes th is (H) Glasgow Com a Scale
exam p le illu strate? 47. Th e EEG o a 32-year-old sleep in g p atien t
(A) Im p losion sh ows m ain ly slow waves. Wh at stage o
(B) Bio eed b ack sleep is th is p atien t m ost likely to be in ?
(C) Aversive con dition in g (A) Stage 1
(D) Token econ om y (B) Stage 2
(E) Floodin g (C) Stages 3 an d 4
(F) System ic desen sitization (D) REM
(G) Cogn itive th erapy
48. Typ ical in an ts b egin visu ally ollowin g
43. A p atien t rep orts th at d esp ite th e act aces an d objects with th eir eyes (trackin g) at
RISE USMLE NEPAL

th at h e goes to sleep at 11:00 p m an d wakes wh at age?


u p at 7:00 a m , h e d oes n ot eel u lly awake (A) 0–3 m on th s
u n til ab ou t n oon each d ay. His wi e states (B) 4–6 m on th s
th at h e ap p ears to b e sleep in g sou n d ly (C) 7–10 m on th s
at n igh t. Th e p atien t d en ies su b stan ce u se, (D) 12–15 m on th s
an d p h ysical exam in ation is (E) 16–30 m on th s
n orm al. Wh ich o th e ollowin g sleep
d isord ers b est m atch es th is clin ical 49. A 65-year-old p h ysician wh o h as been
p ictu re? given a diagn osis o term in al p an creatic
(A) Klein e-Levin syn drom e can cer rep eated ly discu sses th e tech n ical
(B) Nigh tm are disorder asp ects o h is case with oth er p h ysician s in
(C) Sleep terror disord er th e h osp ital. Th e d e en se m ech an ism th at
(D) Sleep dru n ken n ess th is p h ysician is u sin g is
(E) Circadian rh yth m sleep disorder (A) actin g ou t
(F) Noctu rn al m yoclon u s (B) su b lim ation
(G) Restless legs syn d rom e (C) den ial
(D) regression
44. A 33-year-old p atien t tells th e p h ysician (E) in tellectu alization
th at h e d rin ks at least 10 cu p s o co ee p er (F) reaction orm ation
Comprehensive Examination 323

50. An an xiou s, d ep ressed teen ager, wh o h as (G) Avoidan t p erson ality disorder
n ever b een in trou b le be ore, steals a car. (H) Histrion ic p erson ality disorder
Th e de en se m ech an ism th at th is teen ager is
u sin g to m an age h is an xiety an d d ep ression 54. A 35-year-old m an com es to th e
is m ost likely to b e p h ysician’s o ce dressed all in brigh t yellow.
(A) actin g ou t He rep orts th at h e elt like h e h ad “a kn i e
(B) su b lim ation in h is ear” an d says th at h e is “b u rn in g u p”
(C) den ial an d “m u st be dyin g.” Ph ysical exam in ation
(D) regression reveals m ild otitis extern a (in f am m ation o
(E) in tellectu alization th e ear can al) an d tem p eratu re o 99.6°F. Th is
(F) reaction orm ation p atien t’s beh avior is m ost closely associated
HELP OTHERS SO THAT GOD WILL HELP YOU.

with wh ich o th e ollowin g p erson ality


51. A 50-year-old h osp italized p atien t h as disorders?
ju st received a d iagn osis o breast can cer. (A) Passive–aggressive p erson ality disorder
Sh e states th at th e biop sy was in error an d (B) Sch izotyp al p erson ality disorder
ch ecks ou t o th e h osp ital again st th e advice (C) An tisocial p erson ality d isord er
o her p h ysician . Th e de en se m ech an ism (D) Paran oid p erson ality d isord er
th at th is p atien t is u sin g is (E) Sch izoid p erson ality disorder
(A) actin g ou t (F) Obsessive–com pulsive personality
(B) su b lim ation disorder
(C) den ial (G) Avoidan t p erson ality disorder
(D) regression (H) Histrion ic p erson ality disorder
(E) in tellectu alization
(F) reaction orm ation 55. A 24-year-old p atien t is exp erien cin g
in ten se h u n ger as well as tired n ess an d
52. A p atien t, alth ou gh sh e is u n con sciou sly h eadach e. Th is p atien t is m ost likely to be
an gry at h er p h ysician becau se h e can celed with d rawin g rom wh ich o th e ollowin g
h er p reviou s ap p oin tm en t at th e last m in u te, agen ts?
tells h im at h er n ext ap p oin tm en t th at sh e (A) Alcoh ol
really likes h is tie. Th e de en se m ech an ism (B) Secobarbital
th at th is p atien t is u sin g is (C) Ph en cyclidin e (PCP)
(A) actin g ou t (D) Am p h etam in e
(B) su b lim ation (E) Lysergic acid d ieth ylam ide (LSD)
(C) den ial (F) Diazep am
(D) regression (G) Heroin
(E) in tellectu alization (H) Mariju an a
(F) reaction orm ation
RISE USMLE NEPAL

56. A 55-year-old m an , wh o h as been takin g


53. A 28-year-old wom an , wh o works as m edication s or dep ression an d in som n ia, is
an an im al caretaker, lives with h er eld erly brough t to th e em ergen cy dep artm en t with
au n t an d rarely socializes. Sh e rep orts th at, sign s o severe resp iratory dep ression . Th e
alth ou gh sh e wou ld like to h ave rien d s, agen t m ost likely to be resp on sible or th ese
wh en coworkers ask h er to join th em or sym p tom s is
b reaks, sh e re u ses b ecau se sh e is a raid (A) alcoh ol
th at th ey will criticize or n ot like h er. (B) secobarb ital
Th is b eh avior is m ost closely associated (C) p h en cyclidin e (PCP)
with wh ich o th e ollowin g p erson ality (D) am p h etam in e
d isorders? (E) lysergic acid dieth ylam ide (LSD)
(A) Passive–aggressive p erson ality d isord er (F) diazep am
(B) Sch izotyp al p erson ality disorder (G) h eroin
(C) An tisocial p erson ality d isord er (H) m ariju an a
(D) Paran oid p erson ality d isord er
(E) Sch izoid p erson ality disorder
(F) Obsessive–com pulsive personality
disorder
324 BRS Behavioral Science

57. Th e p olice brin g a 25-year-old m an to 61. A 24-year-old wom an exp erien ces p elvic
th e h osp ital in a com a. His girl rien d tells th e p ain wh en sh e an d h er boy rien d attem p t
p h ysician th at p rior to h avin g a seizu re, h e to h ave sexu al in tercou rse. No ab n orm alities
becam e com b ative, sh owed abn orm al eye are ou n d du rin g p elvic exam in ation . O th e
m ovem en ts, an d said th at h e elt h is body ollowin g, th e m ost ap p rop riate diagn osis
exp an din g an d f oatin g u p to th e ceilin g. or th is p atien t is
O th e ollowin g, th e dru g m ost likely to be (A) etish ism
resp on sib le or th ese sym p tom s is (B) gen itop elvic p ain / p en etration disorder
(A) alcoh ol (C) sexu al in terest/ arou sal disord er
(B) secobarb ital (D) orgasm ic disorder
(C) p h en cyclid in e (PCP) (E) gen der dysp h oria
HELP OTHERS SO THAT GOD WILL HELP YOU.

(D) am p h etam in e
(E) lysergic acid dieth ylam ide (LSD) 62. A 65-year-old wom an wh ose h u sb an d
(F) diazep am d ied 3 weeks ago rep orts th at sh e cries
(G) h eroin o ten an d sleep s p oorly. Also, sh e states
(H) m ariju an a th at alth ou gh sh e kn ows h er h u sb an d is
d ead , sh e th ou gh t th at sh e saw h im walkin g
58. A p atien t wh o h as been a h eavy co ee d own th e street th e d ay b e ore. Th e m ost
drin ker is h osp italized an d n ot p erm itted to ap p rop riate rst action by h er p h ysician is
take an yth in g excep t water by m ou th . Wh ich to
o the ollowin g is th e p atien t m ost likely to (A) recom m en d th at sh e visit a close
dem on strate th e d ay a ter h osp italization ? relative
(A) Excitem en t (B) p rovide su p p ort an d reassu ran ce
(B) Eu p h oria (C) p rescribe an tip sych otic m edication
(C) Headach e (D) p rescribe an tidep ressan t m edication
(D) Decreased ap p etite (E) recom m en d a p sych iatric evalu ation
(E) Pu p il d ilation
63. Wh ich o th e ollowin g in dividu als
59. A 50-year-old em ale stroke p atien t h as th e h igh est risk or develop in g
op en s h er eyes in resp on se to a verb al sch izop h ren ia?
com m an d , sp eaks bu t u ses in ap p rop riate (A) Th e dizygotic twin o a p erson with
word s, an d sh ows f exion to p ain u l stim u lu s. sch izop h ren ia
For th ese resp on ses, sh e receives a test score (B) Th e ch ild o two p aren ts with
o 9. Wh ich o th e ollowin g is m ost likely to sch izop h ren ia
be th e test th at was u sed? (C) Th e m on ozygotic twin o a p erson with
(A) Positron em ission tom ograp h y (PET) sch izop h ren ia
(B) Com p u ted tom ograp h y (CT) (D) Th e ch ild o on e p aren t with
RISE USMLE NEPAL

(C) Am obarbital sodiu m (Am ytal) in terview sch izop h ren ia


(D) Th em atic ap p ercep tion test (TAT) (E) A ch ild raised in an in stitu tion al set-
(E) Electroen cep h alogram (EEG) tin g wh en n eith er biological p aren t h ad
(F) Wid e Ran ge Ach ievem en t Test (WRAT) sch izop h ren ia
(G) Folstein Min i–Men tal State Exam in ation
(H) Glasgow Com a Scale 64. Wh ich o th e ollowin g statem en ts is
m ost likely to be eviden ce o p sych otic
60. Th e electroen cep h alogram o a 28-year- eatu res in a severely dep ressed 49-year-old
old p atien t sh ows m ain ly alp h a waves. Th is m an ?
p atien t is m ost likely to b e (A) “I am an in ad equ ate p erson .”
(A) awake an d con cen tratin g (B) “I am a worth less h u m an bein g.”
(B) awake, relaxed, with eyes closed (C) “I will n ever get better.”
(C) in stage 1 sleep (D) “I am a ailu re in m y p ro ession .”
(D) in delta sleep (E) “I am p erson ally resp on sib le or th e
(E) in REM sleep Tsu n am i in Jap an .”
Comprehensive Examination 325

65. A ter a li e-th reaten in g b icycle acciden t, 69. Wh ich o th e ollowin g con dition s
a 9-year-old ch ild requ ires an im m ed iate com m on ly rst becom es ap p aren t in th e
b lood tran s u sion . I , or religiou s reason s, ou rth or th d ecade o li e?
th e p aren ts re u se to allow th e tran s u sion , (A) Alzh eim er’s d isease
th e p h ysician sh ou ld (B) Lesch -Nyh an syn drom e
(A) tell th e p aren ts th ey will be p rosecu ted i (C) Rett’s d isord er
th ey do n ot allow th e tran s u sion (D) Tou rette’s disorder
(B) give th e ch ild th e tran s u sion (E) Hu n tin gton’s disease
(C) ob tain p erm ission rom an oth er am ily
m em b er to d o th e tran s u sion 70. Du rin g an op h th alm ologic exam in ation ,
(D) h ave th e ch ild m oved to an oth er a 48-year-old em ale p atien t, wh o h as h ad
HELP OTHERS SO THAT GOD WILL HELP YOU.

h osp ital sch izop h ren ia or m ore th an 30 years, is


(E) ollow th e p aren ts’ in stru ction s an d do ou n d to h ave retin al p igm en tation . In th e
n ot give th e ch ild th e tran s u sion p ast, th is p atien t is m ost likely to h ave taken
wh ich on e o th e ollowin g an tip sych otic
66. A p atien t states th at ever sin ce a agen ts?
lon gtim e lover called to b reak u p th eir
(A) Ch lorp rom azin e
relation sh ip, th e p atien t h as h ad a severe
(B) Halop eridol
h earin g loss. No m ed ical exp lan ation can b e
(C) Perp h en azin e
ou n d . Wh ich o th e ollowin g is m ost likely
(D) Tri lu op erazin e
to be tru e ab ou t th is p atien t?
(E) Th ioridazin e
(A) Th e p atien t is old.
(B) Th e p atien t is m ale. 71. A typ ical 24-year-old wom an wh o is in
(C) Th e p atien t is well edu cated. n on -REM sleep is m ost likely to sh ow wh ich
(D) Th e p atien t’s h earin g loss ap p eared o the ollowin g?
su d den ly.
(A) Dream in g
(E) Th e p atien t is very u p set ab ou t th e h ear-
(B) In creased p u lse
in g loss.
(C) Clitoral erection
67. Lon g-term p sych iatric h osp itals in (D) Skeletal m u scle aton y
th e Un ited States are own ed an d op erated (E) Delta waves on th e
p rim arily by electroen cep h alogram
(A) u n iversities
72. O th e ollowin g d isorders, th e on e m ost
(B) p rivate in vestors
likely to sh ow th e largest sex d i eren ce in its
(C) state govern m en ts
occurren ce is
(D) m u n icip al govern m en ts
(E) th e ederal govern m en t (A) cycloth ym ic disorder
RISE USMLE NEPAL

(B) m ajor dep ressive disorder


68. A 60-year-old jan itor rom New York, wh o (C) bip olar disord er
recen tly lost h is job as a teach er, is ou n d in (D) illn ess an xiety disorder
Akron , Oh io, workin g as a salesm an . He does (E) sch izop h ren ia
n ot kn ow h ow h e arrived th ere, an d p h ysical
exam in ation is u n rem arkable. O th e 73. Negative p redictive valu e is th e
ollowin g, th e m ost ap p rop riate diagn osis or p robab ility th at a p erson with a
th is m an is (A) n egative test is actu ally well
(A) dissociative am n esia (B) p ositive test is actu ally well
(B) dissociative am n esia with u gu e (C) n egative test is actu ally ill
(C) som atic sym p tom disorder (D) p ositive test is actu ally ill
(D) con version d isorder (E) p ositive test will even tu ally sh ow sign s o
(E) dep erson alization disorder th e illn ess
326 BRS Behavioral Science

74. In a laboratory stu dy, it is sh own th at the carried to term u sed LCDCS du rin g th eir
uterus rises in the pelvic cavity durin g sexu al p regn an cies, th e odds–risk ratio associated
activity. In which stage o the sexual respon se with LCDCS in p regn an cy is ap p roxim ately
cycle does this phen om en on rst occur? (A) 2
(A) Excitem en t (B) 3
(B) Plateau (C) 10
(C) Orgasm (D) 20
(D) Resolu tion (E) 100

75. A cou p le tells th e p h ysician th at th eir 79. In a coh ort stu dy, th e ratio o th e
sex li e h as been p oor becau se th e m an in cid en ce rate o m iscarriage am on g wom en
HELP OTHERS SO THAT GOD WILL HELP YOU.

ejacu lates too qu ickly. Th e p h ysician tells wh o u se LCDCS to th e in cid en ce rate o


th em th at th e “squ eeze tech n iqu e” wou ld b e m iscarriage am on g wom en wh o do n ot u se
h elp u l. In th is tech n iqu e, th e p erson wh o LCDCS is th e
ap plies th e “squ eeze” is u su ally (A) attrib u tab le risk
(A) th e m an (B) odds–risk ratio
(B) th e p artn er (C) in ciden ce rate
(C) th e p h ysician (D) p revalen ce ratio
(D) a sex th erap ist (E) relative risk
(E) a sex su rrogate
Questions 80 and 81
76. Wh ich o th e ollowin g statistical tests
is m ost ap p rop riately u sed to evalu ate In a stu dy, th e in ciden ce rate o tu bercu losis
di eren ces am on g m ean b ody weigh ts o (TB) in p eop le wh o h ave som eon e with TB
wom en in th ree di eren t age grou p s? livin g in th eir h om e is 5 p er 1,000. Th e in ci-
den ce rate o TB in p eop le wh o h ave n o on e
(A) Dep en den t (p aired ) t-test
with TB livin g in th eir h om e is 0.5 p er 1,000.
(B) Ch i-squ are test
(C) An alysis o varian ce
80. Wh at is th e risk or gettin g TB
(D) In dep en den t (u n p aired) t-test
attribu table to livin g with som eon e wh o h as
(E) Fish er exact test
TB (attribu table risk)?
77. A 39-year-old m an (wh o h as n ever be ore (A) 1.5
h ad p rob lem s with erection s) b egin s to h ave (B) 4.5
di cu lty ach ievin g an erection du rin g sexu al (C) 7.5
activity with h is wi e. Th e very rst tim e h e (D) 9.5
h ad trou ble m ain tain in g an erection was (E) 10.0
RISE USMLE NEPAL

a ter a beach p arty wh en h e h ad “too m u ch


to drin k.” Th is m an is sh owin g evid en ce o 81. How m an y tim es h igh er is th e risk o
wh ich o th e ollowin g sexu al d ys u n ction s? gettin g TB or p eop le wh o live with a p atien t
with TB th an or p eop le wh o d o n ot live with
(A) Acqu ired erectile d isord er a TB p atien t (th e relative risk)?
(B) Situ ation al erectile disorder
(C) Delayed ejacu lation (A) 1.5
(D) Orgasm ic d isord er (B) 4.5
(E) Prem atu re ejacu lation (C) 7.5
(D) 9.5
(E) 10.0
78. A stu dy was carried ou t to d eterm in e
wh eth er exp osu re to liqu id crystal disp lay
82. To estim ate th e relative risk in a case–
com p u ter screen s (LCDCS) in th e rst
con trol stu d y, wh ich o th e ollowin g is
trim ester o p regn an cy resu lts in m iscarriage.
calcu lated ?
To do th is, 50 wom en wh o h ad m iscarriages
an d 90 wom en wh o carried to term were (A) Attrib u tab le risk
qu estion ed th e d ay a ter m iscarriage or (B) Odds–risk ratio
delivery, resp ectively, ab ou t th eir exp osu re (C) In ciden ce rate
to LCDCS d u rin g p regn an cy. I 10 wom en (D) Prevalen ce ratio
wh o h ad m iscarriages an d 10 wom en wh o (E) Sen sitivity
Comprehensive Examination 327

83. A 50-year-old em ale h igh sch ool teach er 87. “Man y p eop le eel th e way you do wh en
wh o had been o n orm al weigh t rep orts that th ey rst n eed h osp italization” is an exam p le
she has been “ eelin g very low” or the past 3 o the in terview tech n iqu e kn own as
m on ths. She o ten m isses work because she (A) con ron tation
eels tired an d h op eless, h as lost 20 p oun ds (B) valid ation
withou t dietin g, an d h as trou ble sleep in g. (C) recap itu lation
Wh en the p hysician in terviews h er, sh e says, (D) acilitation
“Doctor, th e Lord calls all h is ch ildren h om e.” (E) re lection
Physical n din gs are un rem arkable. This (F) direct qu estion
clin ical p icture is m ost closely associated with (G) su p p ort
(A) cycloth ym ic disord er
HELP OTHERS SO THAT GOD WILL HELP YOU.

(B) m ajor dep ressive d isord er 88. A ter a patient has described his sym ptom s
(C) bip olar disord er an d the tim e o day that they intensi y, the
(D) illn ess an xiety disorder interviewer says, “You say that you elt the
(E) sch izop h ren ia pain m ore in the evening?” This question is an
exam ple o the in terview techn ique kn own as
84. A 32-year-old m an su rvives a p lan e (A) con ron tation
crash in wh ich ou r p assen gers died. Two (B) valid ation
weeks later, h e rep orts th at h e h as recu rrin g (C) recap itu lation
n igh tm ares ab ou t th e crash an d eels (D) acilitation
isolated an d distan t rom oth ers. Th is p atien t (E) re lection
is m ost likely to h ave wh ich o th e ollowin g (F) direct qu estion
d isord ers? (G) su p p ort
(A) Posttrau m atic stress disorder (PTSD) 89. “You say th at you are n ot n ervou s, bu t
(B) Gen eralized an xiety disorder you are sweatin g an d sh akin g an d seem
(C) Obsessive–com p u lsive disord er (OCD) very u p set to m e” is an exam p le o th e
(D) Pan ic disorder in terviewin g tech n iqu e kn own as
(E) Acu te stress disorder
(A) con ron tation
(B) valid ation
85. On e year a ter sh e was robb ed at
(C) recap itu lation
kn i ep oin t in th e street, a 28-year-old wom an
(D) acilitation
ju m p s at every lou d n oise, h as recu rren t
(E) re lection
th ou gh ts ab ou t th e rob b ery, an d is an xiou s
(F) direct qu estion
m u ch o th e tim e. O th e ollowin g, th e best
(G) su p p ort
d iagn osis or th is p atien t is
(A) p osttrau m atic stress disorder (PTSD) 90. A p atien t relates to th e p h ysician , “I I am
RISE USMLE NEPAL

(B) gen eralized an xiety disorder seated at a table in th e cen ter o a restau ran t
(C) ob sessive–com p u lsive d isord er (OCD) rath er th an again st th e wall, I su d den ly get
(D) p an ic disorder dizzy an d eel like I can n ot breath e.” Th is
(E) acu te stress disorder p atien t is d escribin g a(n )
(A) h allu cin ation
86. At th e close o a lon g in terview with an (B) delu sion
elderly m ale p atien t, th e p h ysician says “Let’s (C) illu sion
see i I h ave taken all o th e in orm ation (D) p an ic attack with agorap h obia
correctly” an d th en su m s u p th e in orm ation (E) social an xiety disorder
th at th e p atien t h as given . Th is in terviewin g
tech n iqu e is kn own as 91. A p atien t relates to the p hysician , “Last
(A) con ron tation week, I thought I saw m y ather who died last
(B) valid ation year go aroun d the corn er, but I kn ow it wasn’t
(C) recap itu lation really him .” This p atien t is describin g a(n )
(D) acilitation (A) h allu cin ation
(E) re lection (B) delu sion
(F) direct qu estion (C) illu sion
(G) su p p ort (D) p an ic attack with agorap h obia
(E) social an xiety disorder
328 BRS Behavioral Science

92. Th e m ost ap p rop riate m eth od to wom an’s sp ou se rep orts th at sh e sn ores
determ in e th e p art o th e b rain u sed du rin g lou dly. Th is wom an is sh owin g evid en ce o
th e tran slation o a written p assage rom (A) n arcolep sy
Fren ch to En glish is (B) Klein e-Levin syn drom e
(A) com p u ted tom ograp h y (CT) (C) in som n ia
(B) dexam eth ason e su p p ression test (DST) (D) ob stru ctive sleep ap n ea
(C) evoked p oten tials (E) sleep terror d isorder
(D) electroen cep h alogram (EEG)
98. Followin g th e loss o h er job as a
(E) galvan ic skin resp on se
cash ier, a 23-year-old p atien t rep orts, in a
(F) p ositron em ission tom ograp h y (PET)
d ispassion ate way, th at sh e h as n o sen sation
in h er righ t arm . Ph ysical exam in ation ails
HELP OTHERS SO THAT GOD WILL HELP YOU.

93. Th e m ost ap p rop riate diagn ostic


to reveal a p h ysiological p rob lem . Th is
tech n iqu e to evalu ate h earin g loss in a
p atien t is sh owin g evid en ce o
3-m on th -old in an t is
(A) illn ess an xiety disorder
(A) com p u ted tom ograp h y (CT)
(B) bod y d ysm orp h ic d isord er
(B) dexam eth ason e su p p ression test (DST)
(C) con version disorder
(C) evoked p oten tials
(D) som atic sym p tom disorder
(D) electroen cep h alogram (EEG)
(E) gen eralized an xiety disorder
(E) galvan ic skin resp on se
(F) p ositron em ission tom ograp h y (PET) 99. Desp ite th e p h ysician’s reassu ran ces an d
n egative biop sies o ve di eren t m oles, a
94. A 42-year-old wom an p reten ds th at sh e is 45-year-old p atien t ap p ears very worried
p aralyzed ollowin g an au tom ob ile accid en t an d tells th e p h ysician th at h e believes th at
in ord er to collect m on ey rom th e in su ran ce h is rem ain in g m oles sh ou ld be b iop sied
com p an y. Th is wom an is dem on stratin g b ecau se th ey are “p robably m elan om as.”
(A) derealization Th is p atien t is sh owin g evid en ce o
(B) actitiou s disorder (A) illn ess an xiety disorder
(C) m alin gerin g (B) bod y d ysm orp h ic d isord er
(D) con version d isorder (C) con version disorder
(E) bod y d ysm orp h ic d isord er (D) som atic sym p tom disorder
(E) gen eralized an xiety disorder
95. A 42-year-old wom an p reten d s th at
sh e is p aralyzed ollowin g an au tom obile 100. A d og learn s to tu rn a d oorkn ob with
acciden t in order to gain atten tion rom h er its teeth becau se th is beh avior h as been
p h ysician . Th is p atien t is d em on stratin g rewarded with a treat. Th is is an exam p le o
th e typ e o learn in g best described as
(A) derealization
(A) op eran t con d ition in g
RISE USMLE NEPAL

(B) actitiou s disorder


(C) m alin gerin g (B) aversive con dition in g
(D) con version d isorder (C) sp on tan eou s recovery
(E) bod y d ysm orp h ic d isord er (D) m odelin g
(E) stim u lu s gen eralization
96. A 54-year-old wom an wh o is d ep ressed 101. Each tim e a 35-year-old m an receives
awaken s at 4:00 a m every m orn in g an d ph ysical th erapy or a sh oulder in ju ry, his
can n ot all back asleep. Sh e is th en tired all pain lessen s. Becau se o th is im p rovem en t
day. Th is wom an is m ost likely to h ave wh ich in pain , the patien t return s or m ore physical
o th e ollowin g sleep d isord ers? therapy session s. Th is p atien t’s in crease in
(A) Narcolep sy ph ysical th erapy session s is an exam p le o
(B) Klein e-Levin syn drom e which o the ollowin g?
(C) In som n ia (A) Im p losion
(D) Obstru ctive sleep ap n ea (B) Stim u lu s gen eralization
(E) Sleep terror disord er (C) System atic desen sitization
(D) Floodin g
97. A 40-year-old wom an wh o is overweigh t (E) Positive rein orcem en t
rep orts th at sh e eels tired all day d esp ite (F) Fixed ratio rein orcem en t
h avin g 9 h ou rs o sleep each n igh t. Th e (G) Negative rein orcem en t
Comprehensive Examination 329

102. A 75-year-old wom an wh o lives alon e h is n ail-bitin g h ab it. Th is is an exam p le o


develop s a h igh ever an d is b rou gh t to th e th e typ e o learn in g best describ ed as
h osp ital by a n eigh bor. Alth ou gh th e wom an (A) op eran t con d ition in g
can state h er n am e, sh e is n ot orien ted to (B) aversive con dition in g
p lace or tim e an d m istakes th e n u rsin g (C) sp on tan eou s recovery
assistan t or h er n ep h ew. Th is clin ical p ictu re (D) m odelin g
is m ost con sisten t with (E) stim u lu s gen eralization
(A) dep ression (p seu dod em en tia)
(B) Tou rette’s disorder 107. A 29-year-old wom an com es to th e
(C) Alzh eim er’s d isease p h ysician with sym p tom s o an xiety, wh ich
(D) deliriu m h ave been p resen t or over 2 years an d h ave
HELP OTHERS SO THAT GOD WILL HELP YOU.

(E) acu te stress disorder n o obviou s p recip itatin g even t. Th e p atien t


h as n ever p reviou sly taken an an tian xiety
103. A 19-year-old m an is b rou gh t to th e agen t. O th e ollowin g p sych oactive agen ts,
h osp ital by th e p olice. Th e p olicem an th e best ch oice or th is wom an is
states th at wh en stop p ed or a m in or (A) diazep am (Valiu m )
tra c violation , th e m an cu rsed an d (B) h alop eridol (Haldol)
sh owed b izarre grim acin g an d twitch in g (C) am itrip tylen e (Elavil)
m ovem en ts. Th is you n g m an is sh owin g (D) bu sp iron e (Bu Sp ar)
evid en ce o (E) lith iu m
(A) dep ression (p seu dod em en tia)
(B) Tou rette’s disorder 108. “Head Start,” an early in terven tion
(C) Alzh eim er’s d isease en rich m en t p rogram or disadvan taged
(D) deliriu m p resch oolers, is aim ed at redu cin g th e
(E) acu te stress disorder likelihood o ailu re in grade sch ool. “Head
Start” is an exam p le o
104. A 63-year-old em ale p atien t, wh o has (A) p rim ary p reven tion
been havin g m em ory problem s or the past (B) secon dary p reven tion
year, n ow can n ot iden ti y the m an sittin g n ext (C) tertiary p reven tion
to h er (h er h usban d). Ph ysical exam in ation (D) system atic d esen sitization
is un rem arkable, an d there is n o history o (E) beh avior m odi ication
drug or alcohol abuse. The patien t is alert an d
seem s to be payin g atten tion to th e p hysician . 109. Th e basic de en se m ech an ism on wh ich
This patien t is m ost likely to have all oth ers are based, an d wh ich is u sed
(A) dep ression (p seu dod em en tia) to p reven t u n accep table em otion s rom
(B) Tou rette’s disorder reachin g awaren ess, is kn own as
(C) Alzh eim er’s d isease
RISE USMLE NEPAL

(A) rep ression


(D) deliriu m (B) su b lim ation
(E) Su b stan ce/ m ed ication in d u ced m ajor (C) dissociation
n eu rocogn itive d isord er (D) regression
(E) in tellectu alization
105. A d yin g p atien t tells h is p h ysician , “I
will go to ch u rch every d ay i on ly I can get 110. In tellin g a p h ysician abou t h er you n ger
rid o th is illn ess.” Th is statem en t in dicates sister’s death in a car acciden t, a 33-year-old
th at th e p atien t is in wh ich o th e ollowin g wom an tries to sp eak bu t keep s breakin g
stages o d yin g? in to tears. Th e m ost ap p rop riate statem en t
(A) Den ial or th e p h ysician to m ake at th is p oin t is
(B) An ger (A) “It is sad bu t everybody loses a loved on e
(C) Bargain in g at som e tim e.”
(D) Dep ression (B) “Don’t cry, you will eel better in a ew
(E) Accep tan ce m on th s.”
(C) “You m u st eel terrible abou t h er death .”
106. A m oth er p u ts a bitter su b stan ce on h er (D) “Sh e was too you n g to die.”
9-year-old son’s n gern ails in ord er to b reak (E) “Please take you r tim e.”
330 BRS Behavioral Science

111. Th e p a ren ts o a 17-yea r-old you n g d evelop m en t, resp ectively, th is ch ild is best
wo m an with Down’s syn d rom e a n d a n d escribed as
in telligen ce q u otien t (IQ) o 70 b rin g h er (A) typ ical, typ ical, n eeds evalu ation
in or a sch ool p h ysica l. Th e p h ysica l (B) typ ical, n eeds evalu ation , typ ical
exam in a tio n is req u ired o r ad m ission to (C) n eeds evalu ation , typ ical, typ ical
a h igh ly recom m en d ed sp ecia l ed u ca tion (D) needs evaluation , n eeds evaluation , typical
co-ed b oa rd in g sch o ol. Th e p a ren ts a re (E) typ ical, typ ical, typ ical
wo rried a b ou t sen d in g th eir d a u gh ter (F) n eeds evalu ation , n eeds evalu ation ,
to th e sch o ol b eca u se sh e is sexu a lly n eeds evalu ation
active a n d th ey are a ra id th at sh e will (G) typical, n eeds evaluation , needs evaluation
get p regn a n t. Alth ou gh sh e h a s b een on
115. A typ ical b oy can walk u p stairs 1 oot at
HELP OTHERS SO THAT GOD WILL HELP YOU.

ora l co n tra cep tives o r th e p a st yea r, h er


m oth er m u st o ten rem in d h er to ta ke a tim e, bu t wh en told to copy a circle, h e ju st
th em . Th e p a ren ts a sk or th e p h ysicia n’s scrib bles on p ap er. At th e p laygrou n d , h e
ad vice. Th e p h ysicia n’s m o st a p p rop ria te o ten m oves away rom h is m oth er to watch
recom m en d a tion is to th e oth er ch ildren , bu t th en com es righ t
(A) do a tu b al ligation back to h er. With resp ect to verbal skills, th is
(B) do an oop h orectom y ch ild is m ost likely to be ab le to
(C) en roll h er in a local day sch ool so th at (A) sp eak in com p lete sen ten ces
sh e can live at h om e (B) u se abou t 900 words
(D) p rescrib e a lon g-actin g con tracep tive (C) u se p rep osition s
(E) sen d h er to th e board in g sch ool an d take (D) u n derstan d abou t 3,500 words
n o u rth er action (E) u se abou t 10 words

112. A 6-year-old ch ild with an in telligen ce 116. As a 60-year-old m ale p atien t is leavin g
qu otien t (IQ) o 50 is m ost likely to be able to th e hosp ital a ter su rgery or p rostate can cer,
do wh ich o th e ollowin g? h e turn s to th e p h ysician an d says, “You
kn ow doctor, I h ave a gu n in m y h ou se.” Th e
(A) Read a sen ten ce m ost ap p rop riate action or th e p h ysician to
(B) Iden ti y colors take at th is tim e is to
(C) Copy a trian gle
(D) Ride a two-wh eeled b icycle (A) call th e p atien t’s wi e an d tell h er to in d
(E) Un derstan d th e m oral di eren ce th e gu n an d rem ove it
between righ t an d wron g (B) su ggest th at th e p atien t rem ain in th e
h osp ital or u rth er evalu ation
113. A p h ysician diagn oses gen ital h erp es in (C) give th e p atien t a p rescrip tion or an
a 16-year-old m ale h igh sch ool stu d en t. Prior an tid ep ressan t
to treatin g h im , th e p h ysician sh ou ld (D) warn th e p atien t abou t takin g m edica-
RISE USMLE NEPAL

tion s th at are dan gerou s in overdose


(A) n oti y h is p aren ts (E) release th e p atien t rom th e h osp ital as
(B) get p erm ission rom h is p aren ts p lan n ed
(C) n oti y h is sexu al p artn er(s)
(D) recom m en d th at h e tell h is sexu al 117. A b on d trader states th at som etim es
p artn er(s) h e m akes m on ey an d som etim es h e loses
(E) n oti y th e ap p rop riate state agen cy m on ey. Th e trader com p lain s th at h e is so
p reoccu p ied with tradin g bon ds th at h e
114. A 9-m on th -old in an t is b rou gh t to a can n ot seem to stop ollowin g th e b on d
p ediatrician by h is m oth er. Th e ch ild can sit m arkets, even on weeken ds wh en th e
u n assisted an d p u ll h im sel u p to stan d. He m arkets are closed . Wh ich o th e ollowin g
bab bles an d m akes n oises wh en h is m oth er is m ost likely to h ave in f u en ced th is trader’s
sp eaks to h im , bu t h e can n ot say an y word s. p reoccu p ation with tradin g bon ds?
Th e m oth er tells th e p h ysician th at wh en th e (A) Con tin u ou s rein orcem en t
ch ild sees h is b abysitter on Satu rd ay n igh ts, (B) Fixed ratio rein orcem en t
h e cries an d re u ses to go to h er. With resp ect (C) Fixed in terval rein orcem en t
to p h ysical, social, an d cogn itive/ verbal (D) Variab le ratio rein orcem en t
(E) Stim u lu s gen eralization
Comprehensive Examination 331

118. A m oth er brin gs h er 3-year-old son to th e 8-year-old is an exam p le o h er u se o


th e pediatrician or a well-ch ild ch ecku p. wh ich o th e ollowin g de en se m ech an ism s?
Th e p h ysician observes th at th e ch ild relates (A) Rep ression
well to h is m oth er an d is ab le to sp eak in (B) Su blim ation
com p lete sen ten ces. In sp eakin g to th e (C) Dissociation
m oth er, th e p h ysician d eterm in es th at th e (D) Regression
ch ild h as a tricycle b u t can n ot p ed al it, an d (E) In tellectu alization
h e does n ot p lay coop eratively with oth er
ch ildren . Select th e b est d escrip tion o th is 122. O th e ollowin g p eop le, wh ich on e is
ch ild’s d evelop m en t in lan gu age, m otor, an d likely to u se th e m ost Medicare services an d
social skills, resp ectively. u n ds du rin g h is or h er li etim e?
HELP OTHERS SO THAT GOD WILL HELP YOU.

(A) Typ ical, typ ical, n eed s evalu ation (A) A rican -Am erican m ale sm oker
(B) Typ ical, n eed s evalu ation , typ ical (B) A rican -Am erican em ale sm oker
(C) Needs evalu ation , typ ical, typ ical (C) A rican -Am erican m ale n on sm oker
(D) Needs evalu ation , n eed s evalu ation , (D) A rican -Am erican em ale n on sm oker
typ ical (E) Wh ite m ale sm oker
(E) Typ ical, typ ical, typ ical (F) Wh ite em ale sm oker
(F) Needs evalu ation , n eed s evalu ation , (G) Wh ite m ale n on sm oker
n eed s evalu ation (H) Wh ite em ale n on sm oker
(G) Typ ical, n eed s evalu ation , n eed s
evalu ation 123. A typ ical ch ild u ses abou t 900 in d ivid u al
words, can stack n in e blocks, an d does well
119. A 14-year-old boy tells h is p h ysician in a p resch ool p rogram each day. Th is ch ild’s
th at h e is con cern ed becau se h e m astu rbates age is m ost likely to b e
every d ay. He is doin g well in sch ool an d is
(A) 8 m on th s
th e cap tain o th e sch ool debatin g team . Th e
(B) 12 m on th s
p h ysician’s n ext m ove sh ou ld be to
(C) 18 m on th s
(A) n oti y h is p aren ts (D) 36 m on th s
(B) re er h im to an adolescen t p sych ologist (E) 48 m on th s
(C) reassu re h im th at h is beh avior is n orm al
(D) tell h im to b ecom e m ore in volved in 124. O th e ollowin g, th e m ost likely reason
sch ool sp orts or a p h ysician to be su ed or m alp ractice is
(E) m easu re h is level o circu latin g th at th e p h ysician
testosteron e
(A) p rescribed a m edication in correctly
(B) h ad p oor rap p ort with a p atien t
120. A 49-year-old sexu ally active wom an (C) did an u n su ccess u l su rgical p rocedu re
tells h er p h ysician th at sh e is exp erien cin g
RISE USMLE NEPAL

(D) can celled an ap p oin tm en t with a p atien t


h ot f ash es an d h as n ot m en stru ated in 4 (E) m ade a p oor m ed ical decision
m on th s. Sh e asks th e p h ysician wh en sh e
can discon tin u e th e u se o b irth con trol. Th e 125. A 65-year-old wom an signs a docum ent
p h ysician’s m ost correct resp on se is that gives her neighbor durable power o
(A) 6 m on th s a ter th e last m en stru al attorney. Five days later, she has a stroke, enters
p eriod a vegetative state rom which she will never
(B) 1 year a ter th e last m en stru al p eriod recover, and requires li e support. The m ost
(C) a ter age 55 appropriate action or the physician to take at
(D) im m ediately this tim e with respect to li e support is to
(E) wh en th e h ot lash es su bside (A) get a cou rt order to con tin u e li e su p p ort
(B) n ot p rovide li e su p p ort
121. Wh en ever a n ew ch ild is ad m itted to (C) ollow th e in stru ction s o th e wom an’s
th e p ediatrics f oor, an 8-year-old girl, adu lt ch ild ren
wh o h as been h osp italized or m ore th an (D) tu rn th e case over to th e eth ics com m it-
2 m on th s, calm s th e ear u l ch ild by drawin g tee o th e h osp ital
p ictures with h im or h er. Th is beh avior by (E) ollow th e in stru ction s o th e n eigh bor
332 BRS Behavioral Science

126. An eld erly Am erican wom an is m ost 130. A wom an wh o h ad a n orm al delivery o
likely to sp en d m ost o th e last 5 years o a n orm al ch ild 2 d ays ago tells h er p h ysician
h er li e th at sh e eels sad an d cries or n o reason .
(A) in a n u rsin g h om e Sh e ap p ears well groom ed an d is takin g
(B) with am ily con gratu latory calls an d visits rom rien ds
(C) on h er own an d am ily. Her p h ysician sh ou ld
(D) in a h osp ice (A) tell h er to stop worryin g
(E) in a h osp ital (B) h ave h er call h im daily over th e n ext 2
weeks to rep ort h ow sh e is eelin g
127. Th e h u sb an d o an 85-year-old (C) recom m en d a con su ltation with a
p atien t with Alzh eim er’s d isease tells th e p sych iatrist
HELP OTHERS SO THAT GOD WILL HELP YOU.

p h ysician th at h e wan ts to keep h is wi e (D) p rescribe an an tidep ressan t


at h om e b u t is worried b ecau se sh e keep s (E) p rescribe a ben zodiazep in e
wan d erin g ou t th e ron t d oor o th e h ou se.
O th e ollowin g, th e m ost ap p rop riate 131. Alth ou gh tran sp lan ts can save m an y
recom m en d ation or th e p h ysician to m ake lives, th ere are ewer tran sp lan ts don e th an
at th is tim e is to are n eeded. O th e ollowin g, th e p rim ary
reason or th is is th at
(A) p lace th e p atien t in restrain ts
(B) label all th e doors as to th eir u n ction (A) th ere are n ot en ou gh don ors
(C) p rescribe diazep am (Valiu m ) (B) p atien ts are u su ally too ill to with stan d
(D) p rescribe don ep ezil (Aricep t) su rgery
(E) p lace th e p atien t in a n u rsin g h om e (C) tran sp lan ts are too exp en sive
(D) tran sp lan ts h ave a h igh ch an ce o
rejection
128. An 8-year-old ch ild with n orm al
(E) th e dru gs u sed to p reven t rejection are
in telligen ce reads, com m u n icates well, an d
too toxic
gets alon g well with th e oth er ch ildren in
sch ool. However, h e o ten argu es with th e
132. A 22-year-old m edical stu den t h as
teach er an d gets bad m arks or h is beh avior.
a p arotid glan d abscess an d an excessive
His p aren ts tell th e p h ysician th at h e o ten
n u m ber o den tal caries. Sh e is o n orm al
also seem s an gry toward th em an d rarely
weigh t or h er h eigh t, b u t seem s d istressed
ollows th eir ru les. Th e best descrip tion or
wh en th e p h ysician qu estion s h er abou t h er
th is ch ild’s beh avior is
eatin g h abits. Th is you n g wom an is m ost
(A) typ ical likely to h ave
(B) atten tion de icit h yp eractivity d isord er
(A) bu lim ia n ervosa
(ADHD)
(B) an orexia n ervosa
(C) au tism sp ectru m d isorder
(C) con version disorder
RISE USMLE NEPAL

(D) op p osition al d e ian t d isord er


(D) avoidan t p erson ality d isord er
(E) con d u ct d isord er
(E) p assive–aggressive p erson ality disorder

129. Wh ich o th e o llowin g a gen ts is 133. Th e adop tive p aren ts o a n ewborn ask
m ost u se u l in th e m an agem en t o a th eir p h ysician wh en th ey sh ou ld tell th e
28-yea r-o ld m ale p atien t wh o exp erien ces ch ild th at sh e is adop ted . Th e p ediatrician
ca tap lexy, h yp n a gogic h a llu cin a tio n s, an d correctly su ggests th at th ey tell h er
a very sh o rt ra p id eye m ovem en t (REM)
(A) wh en sh e qu estion s th em abou t h er
la ten cy?
backgrou n d
(A) A b en zod iazep in e (B) wh en sh e en ters sch ool
(B) A b arb itu rate (C) as soon as sh e can u n d erstan d lan gu age
(C) An op ioid (D) at 4 years o age
(D) An an tip sych otic (E) i sh e develop s an illn ess th at h as a
(E) An am p h etam in e kn own gen etic b asis
Comprehensive Examination 333

134. An in tern ist ob serves th at on e o h er rep orts th at d u rin g th e n igh t, sh e o ten


colleagu es at th e h osp ital, a su rgeon , h as wakes u p, goes in to th e kitch en , a n d
been gettin g very in toxicated at social eats large am ou n ts o ch ocolate can d y
gath erin gs. Th e n u rses, residen ts, an d ellow or m u ltip le b ags o ch ip s. A ter b in gin g
p h ysician s com p lain th at th e su rgeon’s n otes on th ese treats, sh e p u ts h er in gers
are always late, illegible, or n on sen sical. On e d own h er th roat to in d u ce vom itin g.
o th e in tern ist’s p atien ts told h er th at h e Her b od y weigh t is n orm a l, a n d p h ysica l
sm elled alcoh ol on th e b reath o th e su rgeon . exam in a tion is u n rem arka b le. O th e
A ter ceasin g to re er p atien ts to h im , th e ollowin g an tid ep ressan ts, wh ich wou ld
in tern ist’s n ext best cou rse o action is to b e th e b est in itia l ch oice or trea tin g th is
(A) su ggest to h er colleagu es th at th ey also p a tien t?
HELP OTHERS SO THAT GOD WILL HELP YOU.

stop re errin g p atien ts to th e su rgeon (A) Flu oxetin e


(B) do n oth in g. Sh e h as d on e en ou gh by (B) Sertralin e
ceasin g to re er p atien ts to th e su rgeon (C) Bu p rop ion
(C) n oti y th e p olice (D) Ph en elzin e
(D) n oti y th e state im p aired p h ysician s’ (E) Clom ip ram in e
p rogram
(E) talk to th e su rgeon an d tell h im abou t 138. A 2-year-old girl wh o h as reach ed all
h er con cern s o h er develop m en tal m ileston es at th e
typ ical ages can n ot seem to p ay atten tion to
135. Th e au top sy o a 65-year-old m an wh o a task or m ore th an 15 m in u tes at a tim e in
was killed wh en h e walked across th e street n u rsery sch ool. Sh e o ten gets ou t o h er seat
with ou t lookin g at th e tra c ligh t sh ows to walk arou n d th e room or to p lay on th e
d egen eration o ch olin ergic n eu ron s in f oor. Th e girl p lays with th e oth er ch ildren
th e h ip p ocam p u s. In li e, th is m an is m ost bu t re u ses to sh are h er toys with th em .
likely to h ave h ad wh ich o th e ollowin g Ph ysical exam in ation is u n rem arkable. Th e
d isord ers? best exp lan ation or th is ch ild’s beh avior is
(A) Bip olar d isord er (A) age-typ ical b eh avior
(B) Major d ep ressive d isorder (B) atten tion de icit h yp eractivity disorder
(C) Alzh eim er’s d isease (ADHD)
(D) Gen eralized an xiety disorder (C) au tism sp ectru m d isorder (ASD)
(E) Sch izop h ren ia (D) op p osition al d e ian t d isord er
(E) Rett’s d isord er
136. A wom an , wh o gave birth to a h ealth y
ch ild 3 weeks ago, tells h er d octor th at 139. Paren ts o a typ ical 8-m on th -old girl ask
sin ce th e b irth , sh e h as elt em otion al an d th eir p ediatrician wh at skills th ey can exp ect
righ ten ed an d h as h ad rep eated th ou gh ts th at th eir dau gh ter will develop over th e n ext
RISE USMLE NEPAL

th at th e baby wou ld be better o with ou t 2 m on th s. Th e p h ysician’s best resp on se is


h er as its m oth er. Th e p atien t is p oorly (A) sp eakin g in two-word sen ten ces
groom ed, bu t th e m edical exam in ation is (B) crawlin g on h an d s an d kn ees
u n rem arkable. Th e best exp lan ation or th is (C) sittin g u n assisted
wom an’s eelin gs is (D) clim bin g stairs
(A) typ ical p ostp artu m reaction (E) walkin g u n assisted
(B) adju stm en t disord er with d ep ressed
m ood 140. Th e Federal EMTALA act is m ost closely
(C) m ajor dep ressive d isord er associated with wh ich o th e ollowin g?
(D) adju stm en t disord er with con d u ct (A) Con trol o em ergen t p ath ogen s
distu rb an ce (B) Rep ortin g o im p aired m edical p erson n el
(E) gen eralized an xiety disorder (E) Rep ortin g o con tagiou s illn esses
(D) Treatm en t o p atien ts in h osp ital em er-
137. A 27-yea r-o ld wo m a n wh o sh ows gen cy room s
evid en ce o m a jo r d ep ressive d iso rd er (E) Con iden tiality o m edical in orm ation
334 BRS Behavioral Science

141. A com p eten t 30-year-old p atien t wh o is O th e ollowin g, th e m ost ap p rop riate


20 weeks p regn an t com es in or a sch edu led d escrip tion o th e p atien t’s beh avior at th is
p ren atal exam . Th e p h ysician observes th at tim e is
th e patien t sh ows bru isin g on th e abdom en (A) adju stm en t disord er with d ep ression
an d back. Th e p atien t’s h istory reveals th at (B) adju stm en t disord er with an xiety
sh e lost h er last p regn an cy at 21 weeks a ter (C) m ajor dep ressive disorder
h er boy rien d “got rou gh” with h er. Th e m ost (D) n orm al bereavem en t
ap p rop riate action or th e d octor to take at (E) p ersisten t dep ressive disorder
th is tim e is to do wh ich o th e ollowin g?
(A) Con tact law en orcem en t 145. Paren ts brin g th eir 12-year-old dau gh ter
(B) Ask h er to brin g h er b oy rien d in so th at to th e doctor. Th ey are worried becau se th e
HELP OTHERS SO THAT GOD WILL HELP YOU.

you can talk with h im girl re u ses to eat break ast or lu n ch an d h as


(C) State “I am con cern ed ab ou t you r h ealth been losin g weigh t over th e p ast 3 m on th s.
an d you r baby” Th e girl is 65 in ch es tall an d weigh s 110
(D) State “I th in k th e loss o you r p reviou s p ou n ds. Ph ysical exam in ation reveals th at
baby was d u e to you r boy rien d’s ab u sive th e girl is in Tan n er stage 3 an d both th is
beh avior” exam in ation an d lab oratory test resu lts are
(E) Con tact th e state ch ild p rotective agen cy u n rem arkable. Th e n ext step in m an agem en t
is or th e p h ysician to
142. A 28-year-old wom an with a am ily (A) sp eak to th e p aren ts alon e
h istory o lu p u s visits th e p h ysician . Her (B) sp eak to th e girl alon e
sym p tom s in clu d e weakn ess an d m u scle (C) sp eak to th e girl an d th e p aren ts togeth er
ach es. A ter p h ysical exam in ation an d (D) recom m en d a con su ltation with a sp e-
n egative test resu lts, th e doctor tells th e cialist in ad olescen t eatin g d isorders
p atien t th at sh e d oes n ot h ave lu p u s an d in (E) reassu re th e p aren ts th at th e girl’s beh av-
act sh ows n o sign s o rh eu m atologic illn ess. ior is n orm al
Th at a tern oon , th e p atien t calls an oth er
rh eu m atologist to m ake an ap p oin tm en t to 146. A 45-year-old m an is adm itted to th e
be evalu ated or lu p u s. Th e best exp lan ation ICU or trau m a-related in ju ries su stain ed
or th is clin ical p ictu re is in a car acciden t. Th irty-six h ou rs a ter
(A) con version d isorder adm ission , h e b ecom es agitated . He
(B) illn ess an xiety disorder is p u llin g at h is IV access lin es an d is
(C) p ost-trau m atic stress disorder disorien ted to p lace an d tim e. His blood
(D) m alin gerin g p ressu re is 190/ 110 m m Hg, an d h is p u lse is
(E) actitiou s disorder 114/ m in . A reliable h istory rom th e p atien t’s
son reveals th at th e p atien t is dep en den t on
143. Wh ich o th e ollowin g alcoh ol. Wh ich o th e ollowin g is th e m ost
RISE USMLE NEPAL

n eu rotran sm itters is m ost likely to be ap p rop riate n ext step in m an agem en t?


m etab olized to MHPG (3-m eth oxy-4- (A) Give h alop eridol
h ydroxyp h en ylglycol)? (B) Give lith iu m
(A) Seroton in (C) Give lorazep am
(B) Norep in ep h rin e (D) Give p rop ran olol
(C) Dop am in e (E) Re er to Alcoh olics An on ym ou s
(D) γ-Am in obu tyric acid (GABA)
(E) Acetylch olin e (Ach ) 147. A 59-year-old wom an tells h er p h ysician
(F) Glu tam ate th at she h as been takin g estrogen an d
p rogesteron e rep lacem en t th erapy sin ce
144. A 76-year-old m an wh ose wi e died 1 sh e stop p ed m en struatin g 5 years ago.
m on th ago tells h is p h ysician th at sin ce h er Com pared with wom en o h er age wh o h ave
death , h e som etim es eels th at h e “d oes n ot n ot taken h orm on e rep lacem en t therapy, th is
wan t to go on .” Th e p atien t den ies su icidal p atien t is likely to be at decreased risk or
p lan s an d sh ows n o evid en ce o p sych otic (A) breast can cer
th in kin g. He also rep orts th at h e sleep s (B) u terin e can cer
well m ost n igh ts an d th at h is ap p etite is (C) card iovascu lar d isease
essen tially n orm al. Ph ysical exam in ation (D) osteop orosis
an d laboratory testin g are u n rem arkab le. (E) dep ression
Comprehensive Examination 335

148. A 21-year-old stu d en t rep orts th at disch arge rom th e n ip p les. Wh ich o th e
h e becom es very an xiou s wh en h e m u st ollowin g agen ts is m ost likely to b e th e n ew
u se a p u blic restroom bu t oth erwise does m edication ?
n ot rep ort ep isodes o an xiety. Becau se h e (A) Arip ip razole (Abili y)
becom es so u n com ortab le ab ou t u sin g (B) Olan zap in e (Zyp rexa)
p u blic restroom s, h e re u ses wh en h is (C) Zip rasid on e (Geodon )
classm ates ask h im to join th em wh en th ey (D) Ilop eridon e (Fan ap t)
go out. O th e ollowin g p h arm acologic (E) Risp eridon e (Risp erdal)
agen ts wh ich is b est or th e lon g-term
m an agem en t o th is stu den t’s sym p tom s is 152. A 43-year-old wom an rep orts th at sin ce
(A) im ip ram in e (To ran il) bein g p u t on an an tid ep ressan t, sh e h as
HELP OTHERS SO THAT GOD WILL HELP YOU.

(B) ch lordiazep oxid e (Libriu m ) started to h ave di cu lty h avin g an orgasm .


(C) clom ip ram in e (An a ran il) Th e m edication th at th is p atien t is m ost
(D) ven la axin e (E exor) likely to be takin g is
(E) clon azep am (Klon op in ) (A) sertralin e
(B) vilazodon e
149. A 17-year-old college stu den t com es to
(C) m irtazap in e
th e ph ysician com p lain in g o acial swellin g
(D) du loxetin e
an d p ain . Th e stu d en t h as a BMI o 16, an d
(E) bu p rop ion
p h ysical exam in ation reveals a p arotid glan d
(F) ven la axin e
abscess. Th e p atien t n otes th at sh e on ly eats
h ealth y “diet” oods an d th en tells th e doctor
153. A 36-year-old p atien t tells th e p h ysician
th at som etim es sh e eels like h er eatin g is
th at sh e is h avin g di cu lt allin g asleep.
“ou t o con trol.” Th is clin ical p ictu re m ost
Wh ile th e p h ysical exam in ation is essen tially
closely su ggests
u n rem arkable, it reveals th at th e p atien t
(A) an orexia n ervosa is abou t 8 weeks p regn an t. I th e doctor
(B) bin ge-eatin g d isorder decides to p rescribe som eth in g to h elp th e
(C) bu lim ia n ervosa p atien t all asleep, wh ich o th e ollowin g
(D) illn ess an xiety disorder agen ts sh ou ld be avoided?
(E) acu te stress disorder
(A) Tem azep am
150. Paren ts o a 45-year-old , m ild ly (B) Bu sp iron e
in tellectu ally disabled p atien t tell th e doctor (C) Zolp id em
th at th e p atien t h as recen tly started to (D) Bu p rop ion
exp erien ce m em ory loss. Th e d octor n otes (E) Zalep lon
th at th e p atien t h as odd acial eatu res.
Th e gen etic abn orm ality resp on sible or 154. Five h ou rs a ter b irth , a m a le in an t
b egin s to sh ow excessive sa liva tion a n d
RISE USMLE NEPAL

th is clin ical p ictu re is m ost likely to be on


ch rom osom e la crim ation . Th e d o ctor n o tes th a t th e
ch ild , wh o h as a rap id h eart rate an d
(A) 1
ap p ears restless an d agitated , is sweatin g,
(B) 14
alth ou gh th e room is cool. Prior to
(C) 19
d elivery, th e m oth er o th is in an t is m ost
(D) 21
likely to h ave u sed wh ich o th e ollowin g
(E) 22
su b sta n ces?
151. Sin ce bein g started on a n ew (A) PCP
an tip sych otic m ed ication , a 25-year-old (B) Cocain e
em ale p atien t h as begu n to exp erien ce (C) Mariju an a
abn orm al m otor m ovem en ts. Sh e also (D) Alcoh ol
rep orts th at sh e h as started to h ave a (E) Heroin
336 BRS Behavioral Science

155. A 70-year old m an , wh o, over th e p ast Th e m ost likely exp lan ation or th is clin ical
year, h as d evelop ed m em ory loss, sp atial p ictu re is
im p airm en t, an d lan gu age di cu lties, (A) m ajor dep ressive disorder
h as recen tly started to sh ow a n e trem or (B) adju stm en t disord er
an d gait d istu rb an ces. Th e p atien t tells th e (C) gen eralized an xiety disorder
doctor th at h e is also d istu rb ed by vivid (D) typ ical “h om esickn ess”
visu al h allu cin ation s. Th e p atien t’s wi e (E) acu te stress disorder
rep orts th at at n igh t, th e p atien t is very
restless du rin g sleep an d o ten h its an d 158. A 48-year-old m an wh o h as been
p u n ch es h er. Two days a ter bein g started em p loyed by a com p an y or th e p ast 2 years
on risp eridon e to con trol th e h allu cin ation s, believes th at, alth ou gh th ey den y it, h is
HELP OTHERS SO THAT GOD WILL HELP YOU.

th e p atien t b egin s to sh ow severe m u scu lar ellow em p loyees are con sp irin g to get h im
rigid ity. Asid e rom th ese sym p tom s, red. He believes th at th ey h ave wires on
p h ysical n din gs an d laboratory stu dies are h is p hon e an d th at th ey ollow h im h om e.
u n rem arkable. Th is clin ical p ictu re is m ost He requ en tly ch ecks h is h om e or cam eras
con sisten t with th at h e believes h is ellow em p loyees h ave
(A) deliriu m h idden th ere an d in sists th at h is wi e also
(B) Hu n tin gton’s disease ch eck wh en h e is at work. He den ies h avin g
(C) n eu rocogn itive d isorder with Lewy au ditory h allu cin ation s an d, asid e rom
bod ies th e con sp iracy idea, h as n o eviden ce o a
(D) Alzh eim er’s d isease th ou gh t disorder. Wh ich o th e ollowin g is
(E) acqu ired im m u n od e icien cy syn d rom e th e m ost ap p rop riate diagn osis or th is m an ?
(A) Sch izop h ren ia
156. A 34-year-old em ale p olice o cer (B) Bip olar d isord er
com es to th e p h ysician or a yearly (C) Delu sion al d isord er
p h ysical. Th e o cer tells th e d octor th at (D) Paran oid p erson ality d isord er
sh e sm okes on e-h al p ackage o cigarettes (E) Sch izoid p erson ality disorder
a d ay, eats h am b u rger an d steak at least
twice a week, an d d rin ks on e glass o red 159. A 55-year-old wom an presen ts in
win e d aily with d in n er. Sh e also n otes th at the em ergen cy room with orthostatic
wh ile sh e wears a seat b elt in th e p atrol car, h yp oten sion an d p rolon ged QT in terval. The
sh e rarely wears a seat b elt in h er own car. p atien t tells the ph ysician that th e previous
Th e m ost im p ortan t recom m en d ation th e weeks he began takin g “m edicin e to m ake m e
d octor can m ake to m od i y th is p atien t’s h ap pier.” Which o th e ollowin g m edication s
lon g-term m ortality risk is to recom m en d is th is p atien t m ost likely to be takin g?
th at sh e (A) Bu p rop ion
(A) stop sm okin g (B) Flu oxetin e
RISE USMLE NEPAL

(B) start u sin g a seat belt regu larly (C) Lorazep am


(C) redu ce red m eat in h er diet (D) Sertralin e
(D) get a di eren t job (E) Am itrip tylin e
(E) stop drin kin g alcoh ol
160. A 50-year-old wh o h as ju st gon e
157. A p h ysician workin g at a large state th rough m en op au se tells h er p h ysician th at
college sees a 19-year-old stu d en t in th e an d sh e an d h er h u sban d h ave rarely b een
stu d en t h ealth service. Th e stu den t, wh o “in tim ate” in th e p ast year. Th e p h ysician’s
started college 1 m on th ago, tells th e d octor m ost ap p rop riate resp on se to th e p atien t is
th at sin ce sch ool started sh e h as been callin g (A) “Tell m e abou t you r relation sh ip with
h om e every n igh t. Sh e also n otes th at sh e you r h u sban d.”
starts cryin g as soon as h er m oth er an swers (B) “Sexu al p roblem s are n orm al a ter
th e ph on e. Sh e says th at wh ile sh e still m en op au se.”
en joys goin g ou t occasion ally with rien d s, (C) “Sexu al p roblem s are com m on a ter
sh e m isses h er am ily so m u ch th at sh e h as m en op au se.”
n ot been goin g to class an d is in dan ger (D) “A sex th erap ist can be h elp u l with th ese
o ailin g h er m id term exam s. Th ere is n o issu es.”
p reviou s h istory o em otion al p roblem s, an d (E) “Are th e sexu al p roblem s a ectin g oth er
th e stu den t den ies h avin g su icidal eelin gs. asp ects o you r relation sh ip with you r
Ph ysical exam in ation is u n rem arkable. h u sban d?”
Comprehensive Examination 337

161. Ju st p rior to a seriou s op eration , a 164. A 6-year-old ch ild sh ows cogn itive
75-year-old p atien t asks th e p h ysician n ot de cits, an d beh avior su ggestive o au tism .
to p ut h im on li e su p p ort i h e requ ires it Th e ch ild also sh ows abn orm al breath in g
du rin g or a ter th e su rgery. Th e p h ysician an d h an d -wrin gin g b eh avior. Wh ich o th e
agrees to ollow th e p atien t’s wish es an d ollowin g ch rom osom es is m ost likely to
docu m en ts th e con versation in th e p atien t’s be in volved in th e etiology o th is ch ild’s
ch art. A ter th e su rgery, th e p atien t requ ires sym p tom s?
li e sup p ort. Th e p atien t’s son p rodu ces a (A) 4
written will by th e p atien t dra ted 2 years (B) 11
p reviou sly statin g, “Do wh atever you can to (C) 12
keep m e alive.” Wh at sh ou ld th e p h ysician (D) 21
HELP OTHERS SO THAT GOD WILL HELP YOU.

do at th is tim e? (E) X
(A) Have th e h osp ital ch ap lain cou n sel th e
son 165. Th e p aren ts o a 10-year-old boy rep ort
(B) Pu t th e p atien t on li e su p p ort as th e will th at h e o ten gh ts with h is broth er an d
states sister an d h as tried to stran gle th e am ily
(C) Do n ot p u t th e p atien t on li e su p p ort cat. His teach ers rep ort th at h e sh ows
(D) Con tact th e h osp ital eth ics com m ittee p roblem atic beh avior at sch ool an d was
(E) Get a cou rt order to p u t th e p atien t on recen tly ou n d settin g a re in th e coat
li e su p p ort closet. Th e m ost likely cau se o th is p ictu re is
(A) op p osition al d e ian t d isord er
162. A 78-year-old wom an wh ose h u sb an d (B) atten tion -de icit/ h yp eractivity disord er
d ied 2 m on th s ago tells h er p h ysician th at (C) p roblem s with h is p aren ts
som etim es sh e eels th at sh e sh ou ld h ave (D) con d u ct d isord er
d ied in stead o h im . Th e p atien t d en ies (E) adju stm en t disord er
su icidal th ou gh ts or p lan s. Sh e also rep orts
th at sh e h as gon e b ack to p layin g b rid ge 166. An obese 14-year-old boy is brou gh t to
with h er rien d s an d m akin g d in n er or th e doctor by h is m oth er an d h is 18-year-
h er am ily. O th e ollowin g, th e m ost old broth er. Th e m oth er, wh o cooks all o
ap p rop riate descrip tion o th e p atien t’s th e am ily’s m eals, wou ld like in orm ation
b eh avior at th is tim e is on h ow best to p rep are ood or th e boy. His
(A) adju stm en t disord er with d ep ression broth er, wh o exercises regu larly an d is o
(B) adju stm en t disord er with an xiety n orm al body weigh t, wan ts to coach th e boy
(C) m ajor dep ressive d isord er on p h ysical tn ess. Most ap p rop riately, th e
(D) n orm al b ereavem en t doctor sh ou ld rst talk to
(E) p ersisten t d ep ressive disorder (A) th e p atien t alon e
(B) th e m oth er alon e
RISE USMLE NEPAL

163. A 29-year-old wom an wh o h as b een (C) th e m oth er an d p atien t togeth er


b rou gh t to th e em ergen cy room by a rien d (D) th e p atien t, m oth er, an d broth er
tells th e p h ysician th at sh e h as n ot slep t togeth er
in 3 d ays. Sh e says sh e is stayin g awake (E) th e broth er an d p atien t togeth er
b ecau se Jesu s an d Allah asked h er to d o a
p roject to b rin g p h ysics an d scien ce in to 167. An 18-year-old m ale p atien t wh o h as
on e en tity. Her h istory reveals th at sh e h ad a m en tal age o 2 years is en rolled in a day
an ep isod e o d ep ression at th e age o 19 care p rogram with oth er teen s wh o h ave
b u t, u n til last week, sh owed n o oth er m ood disabilities. Th e m ost likely reason th at th is
ep isod es or ab n orm al b eh avior. Th e m ost p atien t wou ld n ot h it th e oth er p atien ts in
ap p rop riate d iagn osis or th is p atien t at th is th e day care cen ter is th at h e wou ld
tim e is
(A) eel badly a terward
(A) m ajor dep ressive d isord er (B) in voke an ger in th e teach er
(B) bip olar I d isorder (C) n ot wan t to h u rt th em
(C) bip olar II disorder (D) be a raid th at th ey wou ld n ot like h im
(D) brie p sych otic disorder (E) n ot wan t th em to h it h im back
(E) sch izop h ren ia
338 BRS Behavioral Science

168. A 28-year-old wom an is red rom h igh at 4+. Th e n u rse n otes th at th e wom an
h er o ce job. As sh e leaves th e o ce a ter h as requ ested a b ed p an an d h as u rin ated
gettin g th e n ews, th e wom an slip s an d alls. n orm ally. Th e m ost likely diagn osis or th is
Th e em ergen cy m edical squ ad is called wom an at th is tim e is
wh en th e wom an rep orts th at sh e can n ot (A) h ern iation o a sp in al disc
walk. In th e em ergen cy room , m edical, (B) h em isection o th e sp in al cord
orth op ed ic an d n eu rological evalu ation s (C) a som atic sym p tom d isord er
are essen tially n orm al alth ou gh th e wom an (D) actitiou s disorder
rep orts th at sh e can n ot eel p in p ricks below (E) adju stm en t disord er
th e waist an d deep ten don ref exes are
HELP OTHERS SO THAT GOD WILL HELP YOU.
RISE USMLE NEPAL
An swers an d Exp lan ation s

1. The answer is D. In creased T4, d ecreased TSH, rap id h eart rate, trem u lou sn ess, an d an xiety
are associated with h yp erth yroid ism . I th is clin ical p ictu re resu lts rom p u rp ose u lly
takin g excessive am ou n ts o exogen ou s th yroid h orm on e (to lose weigh t in th is case), th e
con d ition is kn own as actitiou s h yp erth yroidism (see Ch ap ter 13). Hash im oto th yroid itis
is ch aracterized by excessive th yroid h orm on e cau sed by overactivity o th e th yroid glan d.
HELP OTHERS SO THAT GOD WILL HELP YOU.

Graves d isease is d iagn osed wh en th ere are lower th an n orm al levels o th yroid h orm on e in
blood . In som atic sym p tom disord er, a p erson h as p h ysical sym p tom s, b u t, in con trast to
th e p atien t in this case, th ere is n o m edical exp lan ation . Hyp erp arath yroidism sym p tom s
in clu de atigu e, dep ression , an d gastroin testin al sym p tom s, n ot th e clin ical p ictu re seen in
th is p atien t an d n ot related to takin g excessive th yroid h orm on e.
2. The answer is C. Th is wom an is in th e stage o ch an ge kn own as p rep aration . In th is stage,
th e p atien t m akes sm all im p rovem en ts in th e u n desirable beh avior (alcoh ol drin kin g in
th is case). In th e p recon tem p lation stage, th e p erson does n ot recogn ize or is in den ial
abou t th e n eed to ch an ge th e beh avior. In th e con tem p lation stage, th e p atien t is th in kin g
abou t it b u t is am b ivalen t abou t m akin g th e n eeded ch an ge. In th e action stage, th e
p atien t m akes th e n eeded ch an ge in beh avior, an d in th e m ain ten an ce stage, th e p atien t
con tin u es th e ch an ged b eh avior. In th e relap se stage, th e p atien t eels gu ilt, an ger, an d
disap p oin tm en t at reen gagin g in th e u n wan ted beh avior.
3. The answer is A. The doctor should recom m en d that the m other say “I have a severe illn ess an d
you can ask any questions you want.” Cryin g at the potential loss o a loved one is norm al, and
telling the child not to cry is n ot help ul; telling him that she will be ine or telling him nothing
is not honest. A technical explanation such as “I have leukem ia, a disease o white blood cells
that m ultiply in a disordered m an n er” is too technical an explanation or such a young child.
4. The answer is D. Blockad e o m u scarin ic recep tors is associated with blu rred vision ,
con stip ation , u rin ary reten tion , an d dry m ou th .
5. The answer is D. Th e p atien t’s sym p tom s a ter a week o treatm en t with h alop eridol,
th at is, ever, tach ycardia, trem or, an d rigidity, in dicate th at th e p atien t h as n eu rolep tic
m align an t syn d rom e, a li e-th reaten in g side e ect o an tip sych otic m edication .
6. The answer is A. The doctor should be m ost con cern ed about su icidal plan n in g in th is
RISE USMLE NEPAL

p atien t. Havin g a plan an d a m ean s to com m it suicide are o m ore im m ediate dan ger than
thin kin g about suicide (suicidal ideation ). Weight loss, di iculty sleepin g, an d lack o en ergy
are also seen in depression but are n ot as closely associated with dan ger as suicidal plan n in g.
7. The answer is D. Th is m oth er is sh owin g obsessive–com p u lsive beh avior (ch aracterized
by th e d e en se m ech an ism o “u n d oin g”) to deal with h er excessive an xiety abou t h er
dau gh ter’s sa ety b ecau se o h er own rap e as a teen ager. Usin g u n d oin g, th is m oth er seeks
to reverse or “u n do” th e dan ger to h er d au gh ter by overp rotectin g h er. Th e excessive
clean in g b eh avior also sh ows h er obsessive–com p u lsive beh avior; som eh ow, clean in g can
“u n d o” or reverse h er own rap e as a teen ager.
8. The answer is B. On th e d im en sion s o severity scale or sch izop h ren ia ( rom 0–20), th is
p atien t will h ave a score closest to 4. He wou ld score 0 (n ot p resen t) or delu sion s, 3 or
h allu cin ation s (h earin g th e voice o a n on existen t p erson ), 0 or disorgan ized sp eech (clear
sp eech ), 0 or ab n orm al p sych om otor b eh avior, an d 1 or n egative sym p tom s (ap p rop riate
bu t blu n ted acial exp ression ).
9. The answer is A. Th e con cep tion o death in a 7-year-old ch ild is th at oth ers can die bu t
th at h e can n ot die. It is n ot u n til abou t age 9 th at ch ildren begin to u n derstan d th at th ey

339
340 BRS Behavioral Science

can also d ie. Ch ildren u n der age 6 exp ect th at death is tem p orary an d th at p eop le wh o d ie
com e back to li e.
10. The answer is F. Th e m ost likely cau se o th e exacerbation o th is p atien t’s acn e is wearin g
a h elm et. Because th e exacerbation is on ly on th e oreh ead, oth er ch oices su ch as allergy
to th e am ily p et (th e ch in ch illa) or to m aterials u sed in road con stru ction are u n likely.
Her oreh ead is likely to b e covered by th e h elm et m akin g excessive su n exp osu re u n likely.
Ch ocolate con su m p tion or vegetarian diet is n ot associated with exacerbation o acn e.
11. The answer is B. A case–con trol design is best or th e stu dy o rare diseases becau se it
starts with id en ti ied cases o th e disease an d com p ares th eir exp osu re to a risk actor with
th at o p eop le wh o d o n ot h ave th e d isease (th e con trol grou p ). A case rep ort or m u ltip le
HELP OTHERS SO THAT GOD WILL HELP YOU.

case rep orts wou ld in volve p eop le with th e d isease bu t wou ld n ot in clu de a con trol grou p.
Becau se a coh ort d esign in volves a grou p o h ealth y p eop le wh o are ollowed over tim e, it
cou ld take m an y years an d m an y su b jects be ore en ou gh p eop le develop th e rare disease
to obtain m ean in g u l data. A clin ical treatm en t trial cou ld be u se u l in determ in in g treat-
m en t ch oices or m an agin g th e d isease (i an y) b u t wou ld n ot b e h elp u l in d eterm in in g
risk actors or p rogn ostic in d icators or th e d isease.
12. The answer is B. In a n orm al distribution , th e p ercen tage o people that have systolic blood
p ressure th at is m ore th an 2 stan dard deviation s above the m ean is approxim ately 2.5%.
13. The answer is D. This wom an is n ow at the highest risk or osteoporosis, a sequela o anorexia
n ervosa. Derm atitis and osteoarthritis are n ot associated with a history o anorexia nervosa. In
biliary atresia, ducts to transport bile rom the liver to the duodenum ail to develop in a etus.
Am enorrhea usually resolves when a patient with anorexia nervosa recovers rom the illness.
14. The answer is C. Mem an tin e (Nam en da), an N-m eth yl- d -aspartate (NMDA) receptor an tago-
n ist, is ap p roved to slow deterioration in p atien ts with m oderate-to-severe Alzheim er’s
disease. While also used to treat Alzh eim er’s disease, tacrin e (Cogn ex), don epezil (Aricept),
rivastigm in e (Exelon ), an d galan tam in e (Rem in yl) are all acetylcholin esterase in hibitors.
15. The answer is C. O th e listed agen ts, th e m ost ap p rop riate on e to treat sleep ap n ea in th is
p atien t is m edroxyp rogesteron e acetate (Provera). Progesteron e raises restin g ven tilation
in p atien ts with d ecreased resp iratory drive, an d overweigh t p ostm en op au sal wom en with
sleep ap n ea su ch as th is p atien t can b en e it. Wh ile th ey m ay be u sed to treat sleep ap n ea in
som e p atien ts, tricyclic an tid ep ressan ts as well as SSRIs (e.g., lu oxetin e), are less e ective
or p atien ts su ch as th is th an h orm on e treatm en t. Ben zodiazep in es su ch as diazep am an d
alp razolam can p rom ote sleep b u t are n ot u se u l in sleep ap n ea.
16. The answer is A. Th e p h ysician’s n ext step is to ask th e p atien t abou t h is relation sh ip with
RISE USMLE NEPAL

h is caretakers. Elder abu se is a real p ossibility in th is case, p articu larly sin ce th e p atien t
seem s to b e avoid in g eye con tact. Assessin g th e p atien t or a cogn itive disorder or order-
in g m easu res to p rotect th e p atien t (i n ecessary) can wait u n til th e p atien t is qu estion ed
abou t h is caretakers.
17. The answer is B. Use o cocain e is m ost likely to h ave cau sed th e seizu re in th is p atien t.
With drawal rom ben zodiazep in es su ch as alp razolam also can cau se seizu res, bu t th is
p atien t h ad alp razolam in h er system an d so is u n likely to be in with drawal. Mariju an a u se
m ariju an a with drawal, an d cocain e with drawal are n ot associated with seizu res.
18. The answer is E. As ch ild ren with ch ron ic illn esses su ch as asth m a reach th eir teen years,
th ey are less likely to adh ere to treatm en t th at sets th em ap art rom oth er teen s. Th e m ost
e ective way to in crease adh eren ce in su ch teen agers is to en cou rage in teraction with
oth er teen s wh o h ave th e sam e con dition . Frigh ten in g th e boy or recom m en din g cou n -
selin g or goin g to th e sch ool n u rse will n ot be e ective in in creasin g h is ad h eren ce. Oral
m ed ication or asth m a m ay n ot be m ed ically ap p rop riate or th is p atien t.
19. The answer is A. Th e typ e o dru g m ost likely to h ave cau sed th is p roblem is an an tidep res-
san t agen t, n am ely, a m on oam in e oxidase in h ib itor. Th ese agen ts block th e breakdown
o tyram in e (a p ressor) in th e gastroin testin al tract as well as in th e b rain , resu ltin g in
Comprehensive Examination 341

elevated b lood p ressu re, occip ital h ead ach e, an d oth er seriou s sym p tom s ollowin g in ges-
tion o tyram in e-rich oods (e.g., aged ch eese an d red win e).
20. The answer is B. Bein g divorced an d b ein g m ale are risk actors or su icide.
21. The answer is E. Becau se th eir cau se is com p letely p h ysiological, h ot lash es or lu sh es are
th e sym p tom o m en op au se m ost likely to be seen in 50-year-old wom en in all cu ltu res.
Th e “em p ty n est” syn drom e, dep ression , an xiety, an d in som n ia are m ore closely associated
with societal actors an d so are m ore likely to di er across cu ltu res.
22. The answer is E. Patien ts an d p h ysician s com m on ly p re er selective seroton in reu p take
in h ib itors (SSRIs) to tricyclic an tidep ressan ts becau se SSRIs h ave ewer side e ects an d
th u s are m ore likely to b e well-tolerated . Both grou p s o an tidep ressan ts h ave sim ilar
HELP OTHERS SO THAT GOD WILL HELP YOU.

action on m ood an d sleep. All take 3–4 weeks to work an d n eith er grou p is e ective or low-
erin g b lood p ressu re.
23. The answer is F. Th is treatm en t tech n iqu e in wh ich a p h obic p atien t is tau gh t relaxation
tech n iqu es an d is th en exp osed to in creased “doses” o th e eared ob ject is b est d escribed
as system atic desen sitization .
24. The answer is B. Becau se it is likely to cau se th e least exertion or th e p atien t, th e p h y-
sician’s best recom m en dation is ace to ace, em ale su p erior (wom an on th e top ). To
en h an ce th e p atien t’s recovery, th e cou p le sh ou ld be en cou raged to retu rn to th eir n orm al
activities (in clu din g sex) as soon as it is sa e to do so. Avoidin g sexu al activity or a p ro-
lon ged p eriod o tim e can delay th e p atien t’s recovery.
25. The answer is C. Poor physical care, bruises, and abrasions in this dem ented elderly patient
in dicate that he has been n eglected and abused. Even though he denies that anyone has
harm ed him , the m ost likely abuser is his daughter. There ore, the m ost appropriate action or
the physician to take a ter treatin g this patient is to contact the appropriate state social service
agen cy. As in cases o child abuse, speakin g to the likely abuser about the physician’s concerns
is n ot n ecessary. The physician also can n ot send the patient hom e with the likely abuser or
with an other relative. The social service agency will deal with the patient’s ultim ate placem ent.
I n ecessary, a neurologic evaluation can be done at a later date (and see Question 16).
26. The answer is A. Hisp an ic Am erican s h ave lon ger li e exp ectan cies th an A rican
Am erican s, Wh ite Am erican s, or Native Am erican s.
27. The answer is B. Wh ile som e Am erican s m ay u n derstan d th at it is th erap eu tic to discu ss
you r in tern al em otion al p rob lem s with oth ers or th at u n con sciou s con licts can be m an i-
ested as p h ysical illn ess, a sign i ican t n u m ber o Am erican s believe th at m en tal illn ess is a
sign o p erson al weakn ess or ailu re. Man y also believe th at m en tally ill p eop le h ave p oor
RISE USMLE NEPAL

sel -con trol. For th ese an d oth er reason s, m an y m en tally ill p eop le do n ot seek h elp.
28. The answer is C. In p erson s with d elu sion al disorder, delu sion s (in th e p ersecu tory typ e,
th e belie th at on e’s n eigh bors are en terin g on e’s h om e at n igh t) are p resen t with ou t
abn orm al th ou gh t p rocesses. Ab sen ce o m ood sym p tom s m akes th e diagn osis o b ip olar
disord er an d sch izoa ective d isord er u n likely. Social with drawal, bu t n o ran k delu sion s,
ch aracterizes sch izoid p erson ality d isorder (see also an swers to Qu estion 29).
29. The answer is C. Th is wom an wh o b elieves th at sh e is p regn an t with th e ch ild o a celeb -
rity p rob ab ly also h as d elu sion al d isord er, in th is case th e erotom an ic typ e (an d see
Qu estion 28).
30. The answer is A. Pregn an cy an d ch ild birth are m ore likely to cau se death in th is 40-year-
old wom an th an an y con tracep tive tech n iqu e.
31. The answer is C. Th e colleagu e’s best resp on se is “Reassu re p atien ts th at th e skin con di-
tion you h ave is n ot con tagiou s.” It is better to ace th e p roblem th an to act like n oth in g is
wron g or to p u t p atien ts o by tellin g th em it is n ot th eir p roblem . Th e p h ysician sh ou ld be
en cou raged to see p atien ts an d d eal with th e p roblem op en ly rath er th an to avoid qu es-
tion s by wearin g lon g-sleeved sh irts or ign orin g th e issu e.
342 BRS Behavioral Science

32. The answer is B. Th e u su al stan dards o doctor–patien t con iden tiality app ly to the recen tly
bereaved wom an wh o h as n ot exp ressed a p lan an d is curren tly n ot at high risk to kill
h ersel . In con trast, th e dep ressed wom an wh o tells her physician that she has saved up 50
barbiturate tablets (has a suicidal plan ) an d wan ts to die is at high risk to kill hersel . Other
excep tion s to con iden tiality in clu de p atien ts who com m it child abuse, put their sexual
p artn ers at risk or HIV in ection , or in dicate th at they plan to harm som eon e.
33. The answer is C. Th e m ost ap p rop riate action or Doctor A to take is to rep ort Doctor B
to h is su p erior at th e h osp ital. Rep ortin g o a lap se by a colleagu e is requ ired eth ically
becau se p atien ts m u st be p rotected. Talkin g to Doctor B ab ou t h is lap se again , warn in g
h im , rep ortin g to th e p olice, or recom m en din g a tran s er is n ot likely to accom p lish th e
im m ediate goal o p rotectin g p atien ts.
HELP OTHERS SO THAT GOD WILL HELP YOU.

34. The answer is G. The Folstein Min i–Men tal State Exam in ation is used to ollow im prove-
m en t or deterioration in patien ts with suspected n eurologic dys un ction . Positron em is-
sion tom ography (PET) localizes physiologically active brain areas by m easurin g glucose
m etabolism . Com puted tom ography (CT) iden ti ies organ ically based brain chan ges, such as
en larged ven tricles. Th e them atic app erception test (TAT) utilizes drawin gs depictin g am big-
u ous social situation s to evaluate u n con scious con licts. The sodium am obarbital (Am ytal)
in terview is used to determ in e whether p sych ological actors are respon sible or behavioral
sym p tom s. The electroen cephalogram (EEG), which m easures electrical activity in the cor-
tex, is use ul in diagn osin g epilepsy an d in di eren tiatin g deliriu m rom dem en tia. The Wide
Ran ge Achievem en t Test (WRAT) is used clin ically to evalu ate readin g, arithm etic, an d other
school-related skills in patien ts. The Glasgow Com a Scale quan ti ies level o con sciousn ess
on a scale o 3 to 15.
35. The answer is D. The them atic apperception test (TAT) utilizes drawings depicting am biguous
social situations to evaluate unconscious con licts in patients (see answer to Question 34).
36. The answer is B. Typ ically, in an ts begin to roll over rom back to belly an d belly to back at
abou t 5 m on th s o age.
37. The answer is B. Legal in toxication is d e in ed by blood alcoh ol con cen tration s o 0.05%–
0.15%, dep en din g on in dividu al state law.
38. The answer is G. Protrip tylin e is less sed atin g th an doxep in , n ortrip tylin e, am itrip tylin e,
an d im ip ram in e an d th u s is th e m ost ap p rop riate h eterocyclic an tidep ressan t or som eon e
wh o m u st stay alert on th e job. Wh ile lu oxetin e is also u n likely to be sedatin g, it is a selec-
tive seroton in reu p take in h ib itor (SSRI), n ot a h eterocyclic agen t. Selegilin e an d tran ylcy-
p rom in e are m on oam in e oxidase in h ibitors.
RISE USMLE NEPAL

39. The answer is F. This elderly wom an who reports that she has di iculty sleepin g through the
night because o m uscular contractions in her legs is showing evidence o nocturnal m yoclonus.
40. The answer is E. In tellectu alization , u sin g th e m in d’s h igh er u n ction s to avoid exp erien c-
in g an xiety associated with th e likelih ood o crash in g, is th e de en se m ech an ism bein g
u sed by th is p ilot.
41. The answer is D. Typically, children begin to walk unassisted between 12 and 15 m onths o age.
42. The answer is B. Th e treatm en t tech n iqu e d escrib ed h ere is bio eed back. In th is treatm en t,
th e p atien t is given on goin g p h ysiologic in orm ation abou t th e ten sion in th e ron talis
m u scle an d learn s to u se m en tal tech n iqu es to con trol th is ten sion .
43. The answer is D. Th is p atien t wh o h as a u ll n igh t’s sleep bu t does n ot eel u lly awake u n til
h ou rs a ter h e irst wakes u p is sh owin g eviden ce o a sleep disorder kn own as sleep dru n k-
en n ess. Th is con d ition is rare, an d th e diagn osis can on ly b e m ad e in th e ab sen ce o oth er
m ore com m on p rob lem s d u rin g sleep (e.g., sleep ap n ea) or su bstan ce u se.
44. The answer is C. Tach ycard ia (in creased h eart rate) is seen with th e u se o all o th e stim u -
lan t dru gs, in cludin g ca ein e. Stim u lan t dru gs also ten d to in crease en ergy, blood p res-
su re, an d gastric acid secretion an d im p rove m ood.
Comprehensive Examination 343

45. The answer is A. Th e social sm ile com m on ly irst ap p ears at ab ou t 2 m on th s o age in typ i-
cal in an ts.
46. The answer is F. Th e Wid e Ran ge Ach ievem en t Test (WRAT) is clin ically u sed to evalu ate
readin g, arith m etic, an d oth er sch ool-related skills in p atien ts (see also An swer 34).
47. The answer is C. Slow waves are ch aracteristic o delta sleep (stages 3 an d 4).
48. The answer is A. In an ts can visu ally track a h u m an ace an d objects startin g at birth .
49. The answer is E. Like th e p ilot in Qu estion 40, th is p h ysician , wh o h as been given a diag-
n osis o term in al p an creatic can cer, is u sin g th e de en se m ech an ism o in tellectu alization
(i.e., h e is u sin g h is in tellect an d kn owled ge to avoid exp erien cin g th e righ ten in g em otion s
HELP OTHERS SO THAT GOD WILL HELP YOU.

associated with h is illn ess) (an d see Qu estion 40).


50. The answer is A. By actin g ou t, th e teen ager’s u n accep table an xiou s an d dep ressed eelin gs
are exp ressed in action s (stealin g a car).
51. The answer is C. By u sin g den ial, th is wom an re u ses to believe wh at to h er is th e in toler-
able act th at she h as breast can cer.
52. The answer is F. Th is p atien t is u sin g th e d e en se m ech an ism o reaction orm ation , wh ich
in volves adop tin g b eh avior (i.e., com p lim en tin g th e p h ysician ) th at is th e op p osite o th e
way sh e really eels (i.e., an ger toward th e p h ysician ).
53. The answer is G. Th is wom an is sh owin g eviden ce o th e avoidan t p erson ality disorder.
Becau se sh e is overly sen sitive to rejection , sh e h as becom e socially with drawn . In con trast
to th e sch izoid p atien t wh o p re ers to be alon e, th is p atien t is in terested in m eetin g p eop le
bu t is u n able to d o so becau se o h er sh yn ess, eelin gs o in eriority, an d tim id ity.
54. The answer is H. Th is beh avior is m ost closely associated with th e h istrion ic p erson ality
disord er. Person s with th is disord er are dram atic wh en rep ortin g th eir sym p tom s to p h ysi-
cian s an d call atten tion to th em selves with th eir dress an d beh avior.
55. The answer is D. In ten se h u n ger, tiredn ess, an d h eadach e are all sign s o with drawal rom
am p h etam in es.
56. The answer is B. Th e h istory o in som n ia in dicates th at th is p atien t m ay h ave been given
a p rescrip tion or a b arb itu rate su ch as secob arb ital (Secon al). His h istory o dep ression
u rth er su ggests th at h e h as taken an overdose o th e dru g in a su icide attem p t.
57. The answer is C. Use o b oth p h en cyclid in e (PCP) an d lysergic acid dieth ylam ide (LSD)
resu lts in eelin gs o altered b ody state su ch as th is p atien t describes. However, in con trast
RISE USMLE NEPAL

to LSD, in creased aggressivity an d n ystagm u s (i.e., abn orm al h orizon tal or vertical eye
m ovem en ts) are m ore likely to b e seen with PCP u se.
58. The answer is C. With drawal rom ca ein e an d oth er stim u lan t d ru gs is associated with
h eadach e, leth argy, dep ression , an d in creased ap p etite. Pu p il dilation is associated with
th e u se o , rath er th an with drawal rom , stim u lan ts.
59. The answer is H. Th e Glasgow Com a Scale (scores ran ge rom 3 to 15) is u sed to evalu ate
th e level o con sciou sn ess in p atien ts (see also an swer to Qu estion 34).
60. The answer is B. Alp h a waves are associated with th e awake relaxed state with eyes closed.
61. The answer is B. Th is wom an h as sym p tom s o gen itop elvic p ain / p en etration disord er
(i.e., p h ysically u n exp lain ed p ain with sexu al in tercou rse).
62. The answer is B. With in th e irst ew m on th s o an im p orta n t loss, p eop le o ten resp on d
in ten sely. Th ey m ay even h a ve th e illu sion th at th ey see th e d ea d p erson . Th e p h ysi-
cian sh ou ld p rovid e su p p ort an d reassu ran ce sin ce th is p atien t p rob a b ly is exp erien c-
in g a n orm a l grie reaction . Wh ile lim ited u se o m ed ica tion s or sleep is ap p rop riate,
an tip sych otic or a n tid ep ressa n t m ed ica tion s are n ot in d ica ted in th e m an agem en t o
n orm a l grie .
344 BRS Behavioral Science

63. The answer is C. Th e m on ozygotic twin o a p erson with sch izop h ren ia h as abou t a 50%
ch an ce o develop in g th e disorder. Th e ch ild o on e p aren t with sch izop h ren ia or th e dizy-
gotic twin o a patien t with sch izop h ren ia h as abou t a 10% ch an ce, an d th e ch ild o two
p aren ts with sch izop h ren ia h as abou t a 40% ch an ce. En viron m en tal even ts su ch as bein g
raised in an in stitu tion al settin g are n ot risk actors or th e develop m en t o sch izop h ren ia.
64. The answer is E. Feelin g th at on e is p erson ally resp on sible or a m ajor disaster wh en on e
h ad n oth in g to do with it is a delu sion in th is dep ressed 49-year-old m an . His oth er state-
m en ts, wh ile in dicatin g eelin gs o in ad equ acy an d h op elessn ess, are com m on ly seen in
dep ression b u t do n ot in dicate p sych otic th in kin g.
65. The answer is B. Paren ts can n ot re u se li esavin g treatm en t or th eir ch ild or an y reason .
HELP OTHERS SO THAT GOD WILL HELP YOU.

Becau se th ere is n o tim e b e ore th e ch ild m u st h ave th e tran s u sion , treatm en t can p ro-
ceed on an em ergen cy b asis. Th ere is n o reason to th reaten th e p aren ts with legal action .
66. The answer is D. Th is p atien t sh ows evid en ce o con version disorder. Th is disorder
in volves n eu rological sym p tom s with n o p h ysical cau se, typ ically occu rrin g a ter a stress-
u l li e even t. Sen sory loss in p atien ts with con version disorder ap p ears su dden ly. Patien ts
with th is d isord er are m ore likely to b e you n g an d em ale. Th ey requ en tly sh ow “la belle
in di éren ce,” an u n exp ected lack o con cern abou t th e dram atic sym p tom .
67. The answer is C. In th e Un ited States, lon g-term p sych iatric h osp itals are own ed an d op er-
ated p rim arily by state govern m en ts.
68. The answer is B. Patien ts with dissociative ugue, a dissociative disorder, wan der away rom
their h om es an d do n ot kn ow how th ey got to an other destin ation . This m em ory loss an d
wan derin g o ten occur ollowin g a stress ul li e even t, in this case the patien t’s loss o his job.
69. The answer is E. Hu n tin gton’s disease com m on ly irst ap p ears between th e ages o 35
an d 45 years. Lesch -Nyh an syn d rom e an d Rett’s disorder are ap p aren t du rin g ch ildh ood;
sch izop h ren ia u su ally ap p ears in ad olescen ce or early ad u lth ood ; Alzh eim er’s d isease m ost
com m on ly ap p ears in old age.
70. The answer is E. Retin al p igm en tation is p rim arily associated with u se o th e low-p oten cy
an tip sych otic agen t th iorid azin e.
71. The answer is E. Delta waves are seen in n on -REM sleep stages 3 an d 4. Pen ile an d clitoral
erection , in creased p u lse, in creased resp iration , elevated b lood p ressu re, dream in g, an d
aton y o skeletal m u scles are all seen in REM sleep.
72. The answer is B. O th e d isord ers listed, th e largest sex di eren ce in th e occu rren ce o a
disord er is seen in m ajor d ep ressive disorder. Two tim es m ore wom en th an m en are diag-
RISE USMLE NEPAL

n osed with th is disorder. Th ere is n o sign i ican t sex di eren ce in th e occu rren ce o sch izo-
p h ren ia, cycloth ym ic disord er, illn ess an xiety disorder, or bip olar disorder.
73. The answer is A. Negative p redictive valu e is th e p robability th at a p erson with a n egative
screen in g test is actu ally well.
74. The answer is A. Risin g o th e u teru s in th e p elvic cavity with sexu al activity (i.e., “th e ten t-
in g e ect”) irst occu rs d u rin g th e excitem en t p h ase o th e sexu al resp on se cycle.
75. The answer is B. Th e sexu al p artn er, or exam p le, th e wi e, ap p lies th e squ eeze in th e
squ eeze tech n iqu e, a m eth od u sed to d elay ejacu lation in m en wh o ejacu late p rem atu rely.
In th is tech n iqu e, th e m an iden ti ies a p oin t at wh ich ejacu lation can still be p reven ted. He
th en in stru cts h is p artn er to ap p ly gen tle p ressu re to th e coron a o th e p en is. Th e erection
th en su bsid es an d ejacu lation is d elayed .
76. The answer is C. An alysis o varian ce is u sed to exam in e di eren ces am on g m ean s o m ore
th an two sam p les or grou p s. In th is exam p le, th ere are th ree sam p les (i.e., age grou p s).
77. The answer is B. This m an h as acquired erectile dys u n ction , problem s with erection occur-
rin g a ter a period o n orm al un ction in g. Alcohol use is com m on ly associated with this
con dition .
Comprehensive Examination 345

78. The answer is A. Th e od d s–risk ratio (od ds ratio) o 2 in th is case–con trol stu dy is calcu -
lated as ollows:

Liquid Crystal Display (LCD) Exposure No LCD Exposure

Women who miscarried A = 10 B = 40


Women who carried to term C = 10 D = 80

Odds ratio = (A)(D)/(B)(C) = (10)(80)/(40)(10) = 2.

79. The answer is E. In a coh ort stu dy, th e ratio o th e in ciden ce rate o a con dition (e.g., m is-
carriage) in exp osed p eop le to th e in ciden ce rate in u n exp osed p eop le is th e relative risk.
HELP OTHERS SO THAT GOD WILL HELP YOU.

80. The answer is B. 81. The answer is E. The attributable risk is the in ciden ce rate in exposed peo-
ple (5.0/ 1000) m in us the in ciden ce rate in un exposed people (0.5/ 1000) = 4.5. There ore, 4.5 is
the addition al risk o gettin g TB associated with livin g with som eon e with TB. The relative risk
is the in ciden ce rate in exposed people (5.0/ 1000) divided by the in ciden ce rate in unexposed
people (0.5/ 1000) = 10.0. There ore, the chances o gettin g TB are 10 tim es greater when livin g
with som eon e who has TB than when livin g in a household in which n o on e has TB.
82. The answer is B. The odds–risk ratio is used to estim ate the relative risk in a case–control study.
83. The answer is B. Th is p atien t is m ost likely to h ave m ajor dep ressive disorder. Eviden ce
or th is is m issin g work, eelin g h op eless an d tired, losin g >5% o body weigh t, an d h avin g
trou ble sleep in g. Su icidal ideation is sh own by h er re eren ce to death (i.e., “Doctor, th e
Lord calls all h is ch ildren h om e”).
84. The answer is E. As it is on ly 2 weeks sin ce the traum atic even t occurred, this patien t is
m ost likely to h ave acu te stress disorder. Posttrau m atic stress disorder (PTSD) can n ot be
diagn osed un til at least 1 m on th has passed a ter the traum atic even t. Obsessive–com pul-
sive disorder (OCD) is a disorder ch aracterized by obsession s an d com pulsion s, an d pan ic
disorder is ch aracterized by su dden attacks o in ten se an xiety an d a eelin g that on e is about
to die. In OCD, gen eralized an xiety disorder, an d pan ic disorder, there is n o obvious precipi-
tatin g even t.
85. The answer is A. A ter a li e-th reaten in g even t, h yp ervigilan ce (e.g., ju m p in g at every lou d
n oise), lash b acks (re-exp erien cin g o th e even t), an d p ersisten t an xiety su ggest PTSD.
Acu te stress disorder can on ly be d iagn osed with in 1 m on th o th e trau m atic even t (see
also an swer 84).
86. The answer is C. Usin g recap itu lation , th e in terviewer su m s u p all o th e in orm ation given
by th e p atien t to en su re th at it h as been correctly d ocu m en ted.
RISE USMLE NEPAL

87. The answer is B. “Man y p eop le eel th e way you do wh en th ey irst n eed h osp italization” is
an exam p le o th e in terview tech n iqu e kn own as validation . In validation , th e in terviewer
gives creden ce to th e p atien t’s eelin gs an d ears.
88. The answer is E. “You say th at you elt th e p ain m ore in th e even in g?” is an exam p le o th e
in terview tech n iqu e kn own as re lection .
89. The answer is A. Com m en tin g on bod y lan gu age in dicatin g an xiety an d n otin g in con -
sisten cies b etween verb al resp on ses an d body lan gu age dem on strate th e in terviewin g
tech n iqu e kn own as con ron tation .
90. The answer is D. Sudden ly eelin g anxious, becom ing dizzy, and eeling like one cannot breathe
when exposed to an open area are m an i estations o a panic attack with agoraphobia.
91. The answer is C. In an illu sion , an in d ivid u al m isp erceives a real extern al stim u lu s. In th is
case, th e in dividu al h as seen som eon e b u t h as in terp reted th e p erson as bein g h er ath er.
Illu sion s are n ot u n com m on in a n orm al grie reaction .
92. The answer is F. Positron em ission tom ograp h y (PET) scan s can localize m etabolically
active b rain areas in p erson s wh o are p er orm in g sp eci ic tasks.
346 BRS Behavioral Science

93. The answer is C. Au d itory evoked p oten tials, th e resp on ses o th e brain to sou n d as m ea-
su red by electrical activity, are u sed to evalu ate loss o h earin g in in an ts.
94. The answer is C. In m alin gerin g, th e p atien t p reten ds th at sh e is ill in order to realize an
obviou s (e.g., in an cial) gain .
95. The answer is B. In actitiou s disorder im p osed on sel , th e p atien t sim u lates illn ess or
m ed ical atten tion . Th e gain to th is p atien t, i.e., atten tion rom oth ers or b ein g ill, is n ot
obviou s as it is in th e m alin gerin g p atien t (see also an swer to Qu estion 94).
96. The answer is C. Early m orn in g awaken in g is a typ e o in som n ia th at is com m on ly seen in
p eop le with m ajor d ep ressive d isorder.
HELP OTHERS SO THAT GOD WILL HELP YOU.

97. The answer is D. Patien ts with ob stru ctive sleep ap n ea are requ en tly u n aware th at th ey
h ave awaken ed o ten d u rin g th e n igh t b ecau se th ey can n ot breath e. Th ey sn ore lou dly
an d o ten b ecom e ch ron ically tired .
98. The answer is C. Con version disord er in volves a dram atic loss o m otor or sen sory u n c-
tion with n o m edical cau se. Th ere is o ten a cu riou s lack o con cern (“la belle in di -
éren ce”) abou t th e sym p tom s. Illn ess an xiety disorder is an exaggerated con cern with
illn ess or n orm al bodily u n ction s. Peop le with body dysm orp h ic disorder eel th at th ere
is som eth in g seriou sly wron g with th eir ap p earan ce. In som atic sym p tom disorder,
p atien ts h ave p h ysical sym p tom s, o ten over m an y years, th at h ave n o biological cau se.
Gen eralized an xiety disorder is ch aracterized by ch ron ic (at least 6 m on th s o ) an xiety.
99. The answer is A. Th is p atien t is sh owin g evid en ce o illn ess an xiety disord er, an exagger-
ated con cern with illn ess.
100. The answer is A. In op eran t con d ition in g, a n on re lexive beh avior, su ch as a dog tu rn in g a
doorkn ob, is learn ed by u sin g rein orcem en t, su ch as a treat.
101. The answer is G. In th is exam p le o n egative rein orcem en t, a p atien t in creases h is beh av-
ior (e.g., goin g to p h ysical th erapy session s) in order to redu ce an aversive stim u lu s (e.g.,
h is sh ou lder p ain ).
102. The answer is D. Th is wom an is m ost likely to h ave deliriu m cau sed by th e h igh ever.
103. The answer is B. Facial tics, cu rsin g, an d grim acin g seen in th is you n g m an are sym p tom s
o Tou rette’s d isord er.
104. The answer is C. Th is p atien t is m ost likely to h ave Alzh eim er’s d isease. Becau se h er
level o atten tion is n orm al, th is is n ot d eliriu m . Th ere is n o evid en ce o d ep ression
(p seu d od em en tia), an d th is p atien t h as n o h istory o alcoh ol u se to su ggest su b stan ce/
RISE USMLE NEPAL

m ed ication in d u ced m ajor n eu rocogn itive d isord er.


105. The answer is C. Th is statem en t is an exam p le o th e Kü bler-Ross stage o dyin g kn own as
bargain in g.
106. The answer is B. In aversive con d ition in g, an u n wan ted beh avior (n ail bitin g) is p aired
with an u n p leasan t stim u lu s (n oxiou s-tastin g su bstan ce) an d th e beh avior ceases.
107. The answer is D. Becau se it is less likely th an th e ben zodiazep in es (e.g., diazep am ) to
cau se dep en d en ce, th e b est ch oice o m edication or th is p atien t with gen eralized an xiety
disord er (i.e., ch ron ic an xiety) is bu sp iron e. Lith iu m is u sed to treat b ip olar d isord er, an d
wh ile it can be h elp u l, am itrip tylin e h as sign i ican t side e ects an d th u s is n ot likely to be
u sed or th is p atien t.
108. The answer is A. Head Start an d ed u cation al p rogram s like it are exam p les o p rim ary
p reven tion , m ech an ism s to red u ce th e in cid en ce o a p roblem (e.g., sch ool ailu re) by
redu cin g its associated risk actors (e.g., lack o edu cation al en rich m en t).
109. The answer is A. Rep ression , th e d e en se m ech an ism in u se wh en u n accep table em otion s
are p reven ted rom reach in g awaren ess, is th e de en se m ech an ism on wh ich all oth ers are
based .
Comprehensive Examination 347

110. The answer is E. Most ap p rop riately, th e p h ysician sh ou ld tell th e p atien t th at sh e can
take h er tim e an d n ot try to sp eak wh ile sh e is cryin g.
111. The answer is D. Th e p aren t’s con cern s are real. Th ere ore, to take n o u rth er action is n ot
an accep tab le ch oice or th e p h ysician . Th e p h ysician’s m ost ap p rop riate recom m en d a-
tion is to recom m en d a lon g-actin g con tracep tive or th is you n g wom an . Perm an en t
orm s o birth con trol, su ch as tu bal ligation or oop h orectom y, are n ot ap p rop riate.
Preven tin g h er rom goin g to th e sch ool or ear o p regn an cy cou ld lim it th e social, aca-
dem ic, an d em p loym en t p oten tial o th is you n g wom an .
112. The answer is B. Usin g th e in telligen ce qu otien t (IQ) orm u la (i.e., m en tal age [MA]/
ch ron ological age [CA] × 100 = IQ), th e MA o th is ch ild is 3 years (MA/ 6 × 100 = 50). Like a
HELP OTHERS SO THAT GOD WILL HELP YOU.

typ ical 3-year-old ch ild , som eon e with a m en tal age o 3 years can iden ti y colors bu t can -
n ot read , copy a trian gle, ride a two-wh eeled bicycle, or u n derstan d th e m oral di eren ce
between righ t an d wron g.
113. The answer is D. Prior to treatin g th e 16-year-old p atien t, th e p h ysician sh ou ld recom -
m en d th at h e tell h is sexu al p artn er(s). Th ere is n o n eed to break doctor–p atien t con i-
den tiality by tellin g th e sexu al p artn er(s) sin ce th e illn ess is n ot li e-th reaten in g. Paren ts
do n ot h ave to be told or give p erm ission to treat sexu ally tran sm itted diseases in teen ag-
ers. Gen ital h erp es is n ot gen erally rep ortab le to state or ed eral h ealth au th orities.
114. The answer is E. With resp ect to p h ysical, social, an d cogn itive/ verbal develop m en t,
resp ectively, th is 9-m on th -old ch ild is best described as typ ical, typ ical, typ ical. Ch ildren
can sit u n assisted an d p u ll th em selves u p to stan d by abou t age 10 m on th s. At abou t age
7 m on th s, ch ildren begin to sh ow stran ger an xiety (th e babysitter is essen tially a stran ger
becau se th e ch ild sees h er on ly on ce a week). Ch ild ren com m on ly d o n ot sp eak u sin g
u n d erstan d ab le word s u n til th ey are ab ou t 1 year old.
115. The answer is E. Th is ch ild’s m otor skills (e.g., walkin g u p stairs 1 oot at a tim e, scrib -
blin g wh en told to copy a circle) an d social skills (e.g., m ovin g away rom an d th en toward
h is m oth er) in d icate th at th is ch ild is abou t 1½ years old. With resp ect to verbal skills,
ch ildren o th is age are ab le to u se abou t 10 in dividu al words. Ch ildren 3 years o age u se
abou t 900 word s, u n d erstan d ab ou t 3,500 words, an d sp eak in com p lete sen ten ces. At
abou t 4 years o age, ch ild ren u se p rep osition s (e.g., below, u n der) in sp eech .
116. The answer is B. A statem en t su ch as “I h ave a gu n in m y h ou se” m ade to a p h ysician is
a warn in g sign su ggestin g th at th is p atien t is p lan n in g to h arm h im sel or som eon e else.
Th ere ore, th e m ost ap p rop riate action or th e p h ysician to take at th is tim e is to su ggest
th at th e p atien t rem ain in th e h osp ital or u rth er evalu ation . I th e p atien t re u ses, h e
RISE USMLE NEPAL

can b e h eld again st h is will or a lim ited p eriod o tim e. In orm in g th e wi e o th e th reat,
rem ovin g th e gu n , an d avoid in g dan gerou s m edication s are u se u l strategies bu t will n ot
p reven t th e d an gerou s act rom occu rrin g.
117. The answer is D. Th e m ech an ism th at is likely to u n d erlie th is m an’s p reoccu p ation with
bon d trad in g is th at h e m akes m on ey on a variable ratio rein orcem en t sch edu le. Sin ce
h e n ever kn ows h ow m an y trad es h e h as to m ake to get rein orcem en t (i.e., m on ey), h is
p reoccu p ation p ersists (i.e., is resistan t to extin ction ) on weeken ds even th ou gh h e can -
n ot receive rein orcem en t b ecau se th e m arkets are closed.
118. The answer is B. Most typ ical 3-year-old ch ildren can ride a tricycle, sp eak in com p lete
sen ten ces, an d p lay in p arallel with (n ext to) oth er ch ild ren . Th ey gen erally d o n ot p lay
coop eratively with oth er ch ildren u n til abou t 4 years o age. Th u s, th is ch ild m ay n eed
evalu ation in m otor skills (e.g., h e sh ou ld be able to p edal a tricycle) bu t is typ ical in lan -
gu age an d social skills.
119. The answer is C. Th e p h ysician sh ou ld reassu re th is 14-year-old boy th at m astu rbation
is n orm al. An y am ou n t o m astu rb ation is n orm al, p rovided it does n ot p reven t a p erson
rom h avin g an active, su ccess u l li e. Th ere is n o dys u n ction in th is boy, an d it is n ot
ap p rop riate to n oti y h is p aren ts, re er h im to a p sych ologist, m easu re h is testosteron e
level, or tell h im to b ecom e in volved in sch ool sp orts.
348 BRS Behavioral Science

120. The answer is B. On e year a ter th e last m en stru al p eriod u su ally sign als th e en d o m en o-
p au se, an d th e use o birth con trol can be discon tin u ed. Th e age o m en op au se an d th e
occu rren ce o h ot lash es vary con sid erably am on g wom en an d th u s can n ot be u sed to
p redict th e en d o ertility.
121. The answer is B. Help in g oth er ch ild ren to ad ju st to th e h osp ital is an exam p le o th is
8-year-old girl’s u se o th e de en se m ech an ism o su blim ation . In su blim ation , th e ch ild
rerou tes h er own u n con sciou s, an xiou s eelin gs abou t h er h osp italization in to socially
accep tab le b eh avior (e.g., h elp in g oth er righ ten ed ch ildren ).
122. The answer is H. Because Medicare coverage lasts or li e an d because she has the lon gest
li e expectan cy, a White em ale n on sm oker is likely to use m ore Medicare services an d un ds
HELP OTHERS SO THAT GOD WILL HELP YOU.

than a Wh ite m an , A rican Am erican wom en an d m en , an d sm okers du rin g her li etim e.


123. The answer is D. Th is ch ild is m ost likely to b e 36 m on th s o age. At age 3 years, ch ildren
can u se abou t 900 words an d stack n in e b locks. Th ey are also ab le to sp en d a ew h ou rs
away rom th eir p rim ary caregiver each d ay.
124. The answer is B. Th e m ost likely reason or a p hysician to be sued or m alp ractice is that
the ph ysician had p oor rapp ort with a p atien t. Th e doctor–p atien t relation ship is th e m ost
im portan t actor in whether or n ot a patien t will sue a physician . The physician’s m edical or
su rgical skills have less to do with whether or n ot the physician will be sued by a patien t.
125. The answer is E. Th e m ost ap p rop riate action or th e p h ysician is to ollow th e wish es o
th e n eigh bor. In th is exam p le, th e n eigh bor can decide wh eth er or n ot to con tin u e li e
su p p ort sin ce sh e h as assu m ed th e p ower to sp eak or th e p atien t by virtu e o th e docu -
m en t givin g h er d u rab le p ower o attorn ey.
126. The answer is C. Most eld erly Am erican s sp en d th e last 5 years o th eir lives livin g on th eir
own in th eir own resid en ces. Sm aller n u m b ers o elderly Am erican s en d u p in n u rsin g
h om es or livin g with am ily m em bers. Hosp ice care is aim ed at p eop le exp ected to die
with in 6 m on th s. Hosp ital stays cu rren tly average less th an 1 week.
127. The answer is B. Th e m ost e ective in terven tion or th is 85-year-old p atien t with
Alzh eim er’s d isease, wh o wan d ers ou t o th e h ou se, is to label all th e doors. Sh e m ay
wan der ou t becau se sh e n o lon ger kn ows wh ere each door leads. Medication s can be
h elp u l or associated sym p tom s (e.g., diazep am or an xiety) an d to delay u rth er declin e
(e.g., don ep ezil, an acetylch olin esterase in h ib itor), b u t can n ot rep lace lost u n ction .
Nu rsin g h om e p lacem en t sh ou ld b e con sidered i th e caregiver wish es it. Lon g-term u se
o restrain ts is n ever ap p rop riate.
RISE USMLE NEPAL

128. The answer is D. Sin ce th is ch ild’s p rob lem is with au th ority igu res like h is p aren ts an d
teach ers, th e b est d escrip tion or h is b eh avior is op p osition al de ian t disorder. He reads
an d com m u n icates well, an d th ere is n o evid en ce o atten tion d e icit h yp eractivity dis-
order (ADHD) or au tism sp ectru m d isord er. Becau se th is ch ild relates well to th e oth er
ch ildren in sch ool, h e is u n likely to h ave con d u ct d isord er.
129. The answer is E. Catap lexy, h yp n agogic h allu cin ation s, an d a very sh ort rap id eye m ove-
m en t (REM) laten cy in d icate th at th is p atien t h as n arcolep sy. Am p h etam in es are m ore
likely th an b en zodiazep in es, b arb itu rates, an tip sych otics, or op ioids to be u sed in th e
m an agem en t o n arcolep sy.
130. The answer is B. Th e p h ysician’s m ost ap p rop riate action is to h ave th is p atien t call h im
over th e n ext ew weeks to rep ort h ow sh e is eelin g. Th is wom an h as th e “baby blu es”
(i.e., sad n ess or n o obviou s reason a ter a n orm al delivery). Th ere is n o sp eci ic treatm en t
or baby blu es, an d th e sym p tom s u su ally disap p ear with in 2 weeks. However, becau se
som e wom en with th e b aby blu es go on to d evelop a m ajor dep ressive ep isode requ irin g
treatm en t, th e ph ysician sh ou ld sp eak to th is p atien t daily u n til h er sym p tom s rem it.
131. The answer is A. Fewer tran sp lan ts are d on e th an are n eeded p rim arily becau se th ere are
n ot en ou gh p eop le willin g to don ate th eir organ s at death .
Comprehensive Examination 349

132. The answer is A. Th is you n g wom an is m ost likely to h ave bu lim ia n ervosa, an eatin g dis-
order ch aracterized by bin ge eatin g an d p u rgin g, bu t n orm al bod y weigh t. Parotid glan d
en largem en t an d ab scesses an d d en tal caries are seen in bu lim ia as a resu lt o th e orced
vom itin g.
133. The answer is C. Th e best tim e to tell a ch ild sh e is adop ted is as soon as p ossible, u su ally
wh en th e ch ild can irst u n derstan d lan gu age. Waitin g an y lon ger th an th is will in crease
th e p robability th at som eon e else will tell th e ch ild be ore th e p aren ts are able to.
134. The answer is D. Rep ortin g o an im p aired colleague is requ ired eth ically because p atien ts
m u st be p rotected. I , as in th is case, th e colleagu e is a licen sed p h ysician , it is ap p rop riate
to n oti y th e state im p aired p h ysician s’ p rogram . I th e in tern ist talks to th e su rgeon abou t
HELP OTHERS SO THAT GOD WILL HELP YOU.

h er con cern s, there is n o gu aran tee th at th e su rgeon will listen an d th at th e p atien ts will
b e p rotected. Rep ortin g th e su rgeon to th e p olice is n ot ap p rop riate (an d see Ch ap ter 23).
135. The answer is C. Degen eration o ch olin ergic n eu ron s in th e h ip p ocam p u s in dicates th at
th is m an is m ost likely to h ave h ad Alzh eim er’s disease. Man ia, dep ression , an xiety, an d
sch izop h ren ia are n ot sp eci ically associated with d egen eration o ch olin ergic n eu ron s.
136. The answer is C. Tear u ln ess an d overem otion ality are typical postpartum reaction s, that is,
th e “baby blu es.” However, because th is patien t has had sym ptom s in cludin g suicidality or
3 weeks, the best diagn osis is m ajor depressive disorder (see also an swer to Question 130).
137. The answer is A. Flu oxetin e is th e on ly listed agen t th at is in dicated in th e m an agem en t
o b oth m ajor dep ressive d isord er an d b u lim ia. Bu p rop ion sh ou ld be avoided in p atien ts
with eatin g disorders becau se it lowers th e seizu re th resh old.
138. The answer is A. Typ ical 2-year-old ch ild ren rarely sit still or an y len gth o tim e or sh are
th eir toys with oth er ch ildren .
139. The answer is B. Typ ical in an ts begin to crawl on h an d s an d kn ees b etween 9 an d 11
m on th s o age. In typ ical in an ts, sittin g u n assisted is seen at ab ou t 6 m on th s, walkin g
u n assisted at abou t 12 m on th s, clim bin g stairs at abou t 18 m on th s, an d sp eakin g in two-
word sen ten ces at abou t 24 m on th s (an d see Ch ap ter 1).
140. The answer is D. Hosp itals are legally requ ired to p rovide care to an yon e n eedin g em er-
gen cy m an agem en t wh eth er th ey h ave th e m ean s to p ay or n ot via th e Em ergen cy
Med ical Treatm en t an d Active Lab or Act (EMTALA).
141. The answer is C. Th e p h ysician sh ou ld tell the p atien t th at h e or sh e is con cern ed abou t
h er an d th e b aby. Sin ce sh e is a com p eten t adult, on ly sh e can con tact law en orcem en t
to rep ort th e boy rien d’s beh avior. Sin ce th e ch ild is n ot born , th e state ch ild p rotective
RISE USMLE NEPAL

agen cy can n ot in terven e. Talkin g to th e boy rien d will n ot be h elp u l an d in act m ay cau se
h im to in crease h is abu sive beh avior. Becau se th e p atien t p robably kn ows th at th e loss o
h er p reviou s p regn an cy was du e to h er boy rien d’s abusive beh avior (bu t yet h as ch osen
to con tin u e a relation sh ip with h im ), rem in din g h er o th at act is un likely to be h elp u l.
142. The answer is B. Th e best exp lan ation or th is clin ical p ictu re is illn ess an xiety disor-
d er. Desp ite n egative in d in gs, th is p atien t con tin u es to believe sh e h as lu p u s an d goes
“doctor sh op p in g,” th at is, sh e m akes an ap p oin tm en t with an oth er rh eu m atologist.
Th ere is n o in d ication th at th is p atien t is m alin gerin g (th ere is n o obviou s gain rom th e
sym p tom s) or actitiou s disorder (th ere is n o eviden ce o a desire to be con sidered a sick
p erson ), an d th ere is n o evid en ce o a p recip itatin g li e-th reaten in g stressor as in PTSD.
Con version disord er is n ot likely b ecau se th e sym p tom s are ch ron ic an d n ot n eu rological
an d th e p atien t is worried rath er th an in di eren t.
143. The answer is B. Th e n eu rotran sm itter m ost likely to b e m etabolized to MHPG
(3-m eth oxy-4-h yd roxyp h en ylglycol) is n orep in ep h rin e.
144. The answer is D. Th e m ost ap p rop riate description o th is patien t’s behavior is n orm al
bereavem en t. Occasion ally th in kin g th at on e does n ot wan t “to go on” is com m on in n or-
m al bereavem en t, an d this p atien t does n ot have suicidal p lan s. Because he sleeps an d eats
350 BRS Behavioral Science

n orm ally, m ajor dep ressive disorder is n ot likely an d his sym ptom s h ave n ot lasted lon g
en ou gh to diagn ose p ersisten t dep ressive disorder. Adju stm en t disorder (see Ch ap ter 13,
Table 13.1) can n ot be diagn osed i death o a loved on e was the li e stressor that preceded
th e sym p tom s.
145. The answer is B. Becau se th is girl is well in to p u berty (Tan n er stage 3 is th e m iddle stage
in ad olescen t sexu al develop m en t, see Ch ap ter 2), th e n ext step in m an agem en t is to
sp eak to th e girl alon e. Wh en ever th e p rob lem (h ere a p ossible eatin g disorder) in volves
p rivacy issu es in a p ostp u bescen t p atien t, th e doctor sh ou ld irst sp eak on ly to th e p atien t
(see Ch ap ter 21). It is best or th e p h ysician to take th e irst step in m an agem en t, re erral
to a sp ecialist is n ot ap p rop riate at th is tim e.
146. The answer is C. This 45-year-old m an is showin g eviden ce o alcohol withdrawal. The m ost
HELP OTHERS SO THAT GOD WILL HELP YOU.

appropriate n ext step in the acute m an agem en t o alcohol withdrawal is a ben zodiazepin e
su ch as lorazep am . His h istory o drin kin g alcohol (as provided by his son ), the delayed
(36 h ou rs) on set o agitation an d disorien tation , an d elevated blood p ressu re an d p u lse
in dicate th at h e h as becom e dep en den t on alcohol. Halop eridol, lithiu m , an d p rop ran olol
are less likely to be use ul or im m ediate m an agem en t. Re erral to Alcoholics An on ym ous
typ ically is a lon g-term , n ot an im m ediate, strategy in m an agem en t o alcohol depen den ce.
147. The answer is D. In m en op au sal wom en , estrogen rep lacem en t th erapy (ERT) is m ost
closely associated with d ecreased risk or osteop orosis. ERT h as also been associated with
in creased risk o breast can cer (wh en adm in istered in com bin ation with p rogesteron e [P])
an d u terin e can cer (wh en ad m in istered with ou t P), bu t n ot with p reven tion o cardiovas-
cu lar disease or p sych iatric illn esses su ch as d ep ression .
148. The answer is D. This student’s sym ptom s o anxiety in a public situation (e.g., using public
restroom s) but not in other situations suggest that he has social anxiety disorder. This disorder
has lim ited the patient’s ability to socialize reely. While heterocyclic antidepressants such as
im ipram ine and clom ipram ine and benzodiazepines such as chlordiazepoxide and clonaze-
pam m ay be help ul, venla axine (as well as paroxetine, sertraline, and som e MAOIs) is the only
one o the listed agents that is approved to m anage social anxiety disorder (see Chapter 16).
149. The answer is A. Th is clin ical p ictu re m ost closely su ggests an orexia n ervosa. Callu ses
on th e kn u ckles (Ru ssell sign ) an d th e p arotid glan d abscess are evid en ce o sel -in du ced
vom itin g. Becau se h er BMI is b elow 17, th is you n g wom an sh e can be diagn osed with
an orexia n ervosa (with sel -in du ced vom itin g), n ot bu lim ia n ervosa or bin ge-eatin g disor-
der. Th is p atien t n eith er worries excessively ab ou t h er h ealth , as wou ld a p erson with ill-
n ess an xiety disorder, n or does sh e rep ort exp osu re to a li e-th reaten in g stressor, as wou ld
som eon e with acu te stress d isord er (an d see also an swer to Qu estion 132).
RISE USMLE NEPAL

150. The answer is D. Mild in tellectu al disab ility an d u n u su al acial eatu res su ggest th at th is
p atien t h as Down’s syn d rom e. Down’s syn d rom e p atien ts wh o live to m iddle age com -
m on ly develop Alzh eim er’s disease. Ch rom osom e 21 is associated with both Down’s
syn drom e an d Alzh eim er’s d iseases.
151. The answer is E. Ab n orm al m otor m ovem en ts an d galactorrh ea ( lu id disch arge rom th e
n ip p les du e to in creased p rolactin ) are side e ects o risp eridon e. Arip ip razole, olan zap -
in e, zip rasid on e, an d ilop eridon e are less likely to be associated with th ese adverse e ects
(see Tab le 16.2).
152. The answer is A. Like oth er SSRIs, sertralin e is likely to cau se sexu al side e ects su ch as
delayed orgasm . Vilazod on e, m irtazap in e, d u loxetin e, bu p rop ion , an d ven la axin e h ave
lower rates o sexu al side e ects th an SSRIs (see Ch ap ter 16).
153. The answer is A. Tem azep am , a h yp n otic ben zod iazep in e, is in FDA p regn an cy category
X an d so sh ou ld b e avoided in p regn an t p atien ts. In con trast, bu sp iron e, zolp idem , an d
bu p rop ion are in category B, an d zalep lon is in category C (see Tab le 16.5).
154. The answer is E. Salivation , lacrim ation , rap id h eart rate, sweatin g, restlessn ess, an d
agitation are sign s o h eroin with drawal. Th u s, th e m oth er o th is in an t is m ost likely to
Comprehensive Examination 351

h ave been u sin g h eroin , an d th e in an t is in with drawal. With drawal rom PCP, cocain e,
m ariju an a, an d alcoh ol is u n likely to p rodu ce th is sym p tom p ictu re.
155. The answer is C. Th is p atien t is sh owin g eviden ce o n eu rocogn itive disorder with Lewy
bod ies. Patien ts with th is d isord er sh ow sign s o d em en tia sim ilar to th ose o Alzh eim er’s
disease (e.g., m em ory loss an d lan gu age d i icu lties), bu t th ey also sh ow p arkin son ian
sym p tom s (e.g., in e trem or an d gait distu rban ces), p sych otic sym p tom s (e.g., visu al h al-
lu cin ation s), m otor activity d u rin g REM sleep (REM sleep beh avior disorder [see Ch ap ter
10]), an d h yp ersen sitivity reaction s to an tip sych otic m edication s (e.g., m u scu lar rigidity).
Deliriu m is u n likely b ecau se th e sym p tom s h ave been p resen t over a lon g p eriod, an d
th ere are n o sign i ican t m edical in din gs. Hu n tin gton’s disease an d acqu ired im m u n ode i-
cien cy syn d rom e do n ot it th is clin ical p ictu re.
HELP OTHERS SO THAT GOD WILL HELP YOU.

156. The answer is B. Wh ile sm okin g, red m eat an d alcoh ol, an d workin g as a p olice o icer
are related to lon g-term m ortality, th e leadin g cau se o d eath in p eop le u n d er age 35 is
m otor veh icle accid en ts, p articu larly wh en p assen gers ail to wear seat belts (an d see
Ch ap ter 24). So, as in ch ild ren (an d see Ch ap ter 24), th e m ost im p ortan t recom m en dation
or decreasin g m ortality in th e sh ort or lon g term or th is p atien t is or h er to con sisten tly
wear a seat b elt in th e car.
157. The answer is B. Th e m ost likely exp lan ation or th is stu d en t’s b eh avior th at b egan with a
stress u l li e even t (goin g away to college) is adju stm en t disorder (with dep ressive sym p -
tom s). Th is stu den t’s b eh avior is n ot typ ical h om esickn ess becau se h er sym p tom s are
a ectin g h er ability to u n ction (e.g., sh e is in dan ger o ailin g h er cou rses). Th e ab sen ce
o a p reviou s p sych iatric h istory or su icidal th in kin g an d th e act th at sh e can en joy tim e
with rien ds in dicate th at h er sym p tom s wou ld p robably n ot u l ill criteria or an an xiety
or m ood d isorder. Th e act th at th e stressor p rovokin g th e sym p tom s was n ot li e-th reat-
en in g ru les ou t acu te stress d isord er (see also Ch ap ter 13).
158. The answer is C. Th is 48-year-old m an wh o believes th at h is ellow em p loyees are con -
sp irin g to get h im ired is m ost likely to h ave delu sion al disorder, p aran oid typ e. Th is
disord er is ch aracterized by on e ch ron ic an d ixed n on b izarre d elu sion al system su ch as
th is p atien t’s belie in a n on existen t con sp iracy. Becau se th ere is n o oth er eviden ce o a
th ou gh t disorder, sch izop h ren ia is ru led ou t. Person ality disorders are n ot ch aracterized
by ran k ixed d elu sion s su ch as th e on e th is p atien t exh ib its.
159. The answer is E. This patien t with orthostatic hypotension, and prolonged QT interval is m ost
likely to be takin g a tricyclic an tidepressant such as am itriptyline. Fluoxetine, bupropion ,
lorazepam , an d sertralin e are less likely than im ipram ine to cause these cardiac sym ptom s.
160. The answer is A. The physician’s m ost appropriate respon se to this patien t’s com m en t is
RISE USMLE NEPAL

“Tell m e about your relation ship with your husban d.” The physician should irst get all o the
in orm ation about the issue be ore reassurin g the patien t or suggestin g a course o action .
161. The answer is C. Most correctly, th e p h ysician sh ou ld ollow th e cu rren t wish es o th e
p atien t an d n ot p u t h im on li e su p p ort. Th e cu rren t p re eren ce o th e p atien t was
exp ressed directly to th e d octor, so th e p rior written in stru ction s n o lon ger ap p ly. Th e
h osp ital ch ap lain , eth ics com m ittee, an d cou rt order are n ot in volved in th is decision
sin ce th e p atien t’s wish es h ave b een directly exp ressed to an d docu m en ted by th e doctor.
162. The answer is D. Th is wom an is sh owin g n orm al bereavem en t. In th e irst ew m on th s
a ter th e d eath o a close relative, m an y p atien ts h ave occasion al th ou gh ts th at th ey
sh ou ld h ave died in stead o th e loved on e. Becau se th is p atien t is gettin g back to h er
orm er li e-style (e.g., p layin g b ridge, cookin g) it is u n likely th at h e h as m ajor dep ressive
disord er. Persisten t dep ressive d isord er in volves at least 2 years o dep ressive sym p tom s.
Adju stm en t disord er can n ot be d iagn osed wh en b ereavem en t is a m ore ap p rop riate
descrip tion .
163. The answer is B. Th e m ost ap p rop riate diagn osis or th is m an ic p atien t is b ip olar I dis-
order. Wh ile a sin gle ep isod e o m an ia d e in es th is illn ess, th is p atien t also h as a h istory
o d ep ression . Th e gran diose religiou s b elie th at th is p atien t exh ibits is a delu sion , an d
352 BRS Behavioral Science

d elu sion s are n ot seen in h yp om an ia (h yp om an ia an d dep ression ch aracterize bip olar II


d isord er). Brie p sych otic d isord er an d sch izop h ren ia both in volve p sych otic sym p tom s
su ch as delu sion s, bu t, in con trast to th is p atien t, th e delu sion s m u st occu r in dep en den t
o elevation or dep ression o m ood .
164. The answer is E. Th e X ch rom osom e is associated with Rett’s disord er (see Table 4.1), a
rare gen etic disorder th at cau ses in tellectu al disability an d au tistic beh avior. Rett’s dis-
order, seen on ly in girls, is also ch aracterized by abn orm al breath in g an d h an d-wrin gin g
m ovem en ts.
165. The answer is D. Becau se h e en gages in b eh avior th at is dan gerou s an d wou ld be con sid-
ered illegal in an ad u lt, th e m ost likely cau se o th is ch ild’s di icu lties is con d u ct disorder.
HELP OTHERS SO THAT GOD WILL HELP YOU.

Wh ile ch ildren with op p osition al de ian t disorder, atten tion -de icit/ h yp eractivity disor-
der, or adju stm en t disord er also m ay sh ow b eh avioral p roblem s, th ey are u n likely to h arm
p ets or set ires.
166. The answer is A. Most ap p rop riately, th e d octor sh ou ld talk to th e p atien t alon e. Wh ile th e
m oth er an d b roth er obviou sly wan t to h elp, th e doctor m u st irst elicit th e teen’s p ersp ec-
tive on th e issu e p rivately an d th en add ress it. Th e p atien t can in dicate wh eth er or n ot h e
wan ts h is m oth er an d broth er to be in volved . Patien ts m u st recogn ize th at th ere is a p rob-
lem an d in d icate th at th ey are willin g to ch an ge th eir beh avior be ore ch an ge can begin .
167. The answer is E. Th e m ost likely reason th at th is p atien t wou ld n ot h it th e oth er p atien ts
in th e d ay care cen ter is th at h e wou ld n ot wan t th em to h it h im back. Like a 2-year-old
ch ild (th e p atien t’s m en tal age), th is p atien t’s m ain in terest is in gettin g p leasu re an d
avoidin g p ain . He h as n ot yet d evelop ed a su p erego (typ ically develop ed a ter age 6 in
ch ildren with n orm al in telligen ce) an d so will n ot eel badly later. Like an y 2-year-old, h e
is also u n likely to care abou t h u rtin g th e oth er p atien ts, wh eth er or n ot th ey like h im , or
in vokin g an ger in th e teach er.
168. The answer is C. Th is p atien t is sh owin g eviden ce o a som atic sym p tom disorder, sp eci i-
cally con version disord er. Th is disorder is diagn osed wh en a p atien t sh ows n eu rological
sym p tom s th at are n ot exp lain ed by m ed ical in din gs bu t rath er occu r as a resu lt o a p sy-
ch ologically stress u l li e even t. Th is wom an wh o h as h ad su ch a li e even t (losin g h er job)
h as d evelop ed wh at ap p ear to b e n eu rological sym p tom s su ch as n u m bn ess below th e
waist an d th e in ability to walk. Neu rological in din gs, h owever, are absen t or n ot sp eci ic;
th e in creased d eep ten don re lexes th at sh e sh ows can also be seen in in dividu als wh o
are u n der em otion al stress. Seriou s sp in al cord in ju ry also is u n likely b ecau se th e wom an
is u rin atin g n orm ally. Factitiou s disorder is seen in p eop le p u rp osely akin g sym p tom s
to becom e th e ocu s o oth ers’ in terest; adju stm en t disorder is seen wh en a p erson h as
RISE USMLE NEPAL

p sych ological (n ot p h ysical as in th is p atien t) sym p tom s a ter a stress u l li e even t.


In d ex

Note: Page n u m ber ollowed by an d t in d icates gu re an d tab le resp ectively.

A Agin g. See also Eld erly


Abili y. See Arip ip razole cogn itive ch an ges, 26
Abn orm al grie reaction , 27, 28t dem ograp h ics o , 25
HELP OTHERS SO THAT GOD WILL HELP YOU.

Abortion , 17 li e exp ectan cy an d lon gevity, 26 , 27


Absolu te risk redu ction (ARR), 292 n eu rologic ch an ges, 26
Abu se n orm al, an d m em ory, 146t
ch ild (see Ch ild ab u se) p sych ological ch an ges, 26
dom estic p artn er, 213, 213t, 214 review test on , 29–33
eld er, 211, 212t, 214 an d sexu ality, 210
p h ysician’s role, 214 som atic ch an ges, 26
review test on , 216–218 Agorap h obia, 163
sexu al, 213 Agran u locytosis, 49, 171, 175
su b stan ce (see Su b stan ce ab u se) Alcoh ol, 211
Accep tan ce as stage o dyin g, 27 abu se, 86, 86t
Accu lturative stress, 197 e ects on sexu ality, 211, 211t
Accu racy, in m ean estim ation , 307 Alcoh olics An on ym ou s (AA), 189
Acetylch olin e (ACh ), 40–41, 100 Alcoh ol-related disord ers, in eld erly, 27
Acetylch olin esterase in h ibitors, 147 Alen d ron ate sod iu m (Fosam ax), 25
Ach ievem en t tests, 75 Alli. See Orlistat
Acquired im m un e de cien cy syn drom e Alp h a waves, 98, 99t
(AIDS), 252 Alp razolam (Xan ax), 133, 176t
Actin g ou t, 18, 59t Altru ism , 59t
Acu te in term itten t p orp h yria, 50 Alzh eim er’s d isease, 35t, 146
Acu te stress disorder (ASD), 133, 134t. See also brain ch an ges in , 147
An xiety disord ers diagn osis o , 146
Adap in . See Doxep in gen etics o , 146
Addison’s disease (h yp ocortisolism ), 50 in itial in terven tion s or, 146
ADHD. See Atten tion d e cit h yp eractivity loss o m em ory in , 146, 146t
disorder (ADHD) n eu rop h ysiological actors, 147
Adh eren ce, 233–235, 236t p h arm acologic in terven tion s or, 147
Adju stm en t disorder, 134t an d p seu dodem en tia, 146, 146t
Adolescen ce Am erican s o m idd le Eastern / n orth A rican
early, 16 descen t, 199
late, 17 Am erican su b cu ltu res, 198–199
m iddle, 16–17 Am in o acid n eu rotran sm itters, 41
RISE USMLE NEPAL

p regn an cy, 17–18, 17 Am itrip tylin e (Elavil), 172t, 173t


reaction to illn ess, 18 Am otivation al syn d rom e, 89
review test on , 20–24 Am oxap in e (Asen d in ), 172t, 173t
sexu ality, 17 Am p h etam in es, 84, 85, 85t, 211
Tan n er stages o sexu al d evelop m en t, 16t e ects on sexu ality, 211
Adop tion , 18 Am ygd ala, 36t, 211
Adren ocorticotrop ic h orm on e (ACTH), Am yloid p laqu es, 147
248 Am ytal in terview, 52
Adu lth ood An algesics, or p ain m an agem en t, 251
early, 18–19 An al in tercou rse, 212
m iddle, 19 An alysis o varian ce (ANOVA), 310
Advan ce directives, 262 An d rogen in sen sitivity syn d rom e, 205
A ect, 77t An d rogen s, an d aggression , 211
A ordable Health Care Act, 282–283 An esth esiologists, 263
A rican Am erican s An gel d u st. See Ph en cyclidin e (PCP)
cu ltural descrip tion o , 199 An gelm an syn d rom e, 35t
am ily typ es, 195 An ger as stage o dyin g, 27
in an t m ortality rate, 2t An glo Am erican s, 199
leadin g cau ses o death , 210 An h edon ic m ood, 78t
Aggression An n iversary reaction , 28
biological determ in an ts o , 210–211 An orexia n ervosa, 151
sexu al, 214–215 ch aracteristics an d m an agem en t o , 152t
social d eterm in an ts o , 210 su b typ es o , 151
353
354 Index

An tian xiety agen ts, 133, 174t, 175–176 Ben d er Visu al Motor Gestalt test, 51t
ben zodiazep in es, 174t, 175 Ben zod iazep in es (BZs), 41, 87, 175, 176t
n on -ben zodiazep in es, 174t, 175 in an xiety d isorders, 133
An ticon vu lsan ts, 126, 175 Bereavem en t, 28
An tidep ressan ts, 126, 133, 171–174, 172t, 173t β-blockers, in an xiety disorders, 133
e ects on sexu ality, 211, 211t Beta waves, 98, 99t
h eterocyclic agen ts, 172t, 173, 173t Bias, 293, 293t
MAOIs, 172t, 173, 173t redu cin g, in clin ical treatm en t trials, 294
or p ain m an agem en t, 251 Bin ge-eatin g d isorder, 151
an d seroton in , 40 Biogen ic am in es, 38–41
SNRIs, 172t, 173, 173t acetylch olin e, 40
SSRIs, 172t, 173, 173t dop am in e, 38–40
An tih yp erten sives, 176 h istam in e, 40
e ects on sexu ality, 211, 211t m easu rem en t o , 49–50
HELP OTHERS SO THAT GOD WILL HELP YOU.

An tip sych otics, 126, 147, 169–170 m etabolites o , 38, 38t


adverse e ects o , 170t, 171t n orep in ep h rin e, 40
atyp ical, 170–171, 171t overview o , 38, 38t, 39
e ects on sexu ality, 211, 211t seroton in , 40
tradition al, 170, 170t Bip olar disord er, 34, 121, 123, 170. See also
An tisocial p erson ality d isord er, 148t Dep ressive an d bip olar d isord ers
An xiety, 78t, 133, 249 an ticon vu lsan ts or, 175
in eld erly, 26 gen etics o , 125t
organ ic basis o , 133 Birth rate, 1, 17, 17
p h ysiologic m an i estation s o , 132 Blin d stu dies, 294
An xiety disord ers, 132 Blu n ted a ect, 78t
an tian xiety agen ts or, 133 Bod y d ysm orp h ic d isord er, 135t
an tidep ressan ts or, 133 Bod y m ass in d ex (BMI), 150, 151
classi cation an d occu rren ce o , 132, 134t Bord erlin e p erson ality disord er, 149t
p sych ological m an agem en t o , 133 Boston n am in g test, 51t
review test on , 137–143 Brain
Ap gar scorin g system , 2, 3t an atom y o , 35–37
Ap lastic an em ia, 175 ch an ges, in Alzh eim er disease, 146
Ap om orp h in e h ydroch lorid e (Up rim a), 209 lesion s o , 36, 36t
Arip ip razole, 170 review test on , 42–48
ASD. See Au tism sp ectru m disord ers (ASD) Breath in g-related sleep d isorder, 102, 103t
Asen ap in e (Sap h ris), 170 Brie p sych otic d isord er, 115t
Asian Am erican s, 198–199 Brin tellix. See Vortioxetin e
Attach m en t Bru xism , 101t
in an t, 4 Bu lim ia n ervosa, 151
p resch ool ch ild, 6–7 ch aracteristics an d m an agem en t o , 152t
stu d ies o , 4 su b typ es o , 151
toddler, 5–6 Bu p ren orp h in e, 87–88
Attack rate, 297 Bu p rop ion , 173t
Atten tion , 78t Bu sp iron e (Bu Sp ar), 176, 176t
Atten tion de cit h yp eractivity d isord er (ADHD), in an xiety d isorders, 133
84, 161–162, 162t BZs. See Ben zodiazep in es (BZs)
RISE USMLE NEPAL

Attribu table risk, 292


Au tism sp ectru m d isord ers (ASD), 160 C
au tism sp ectru m disorder, 160 Ca ein e, 84, 85t
n eu robiological etiology o , 161 CAGE qu estion s, or alcoh olism id en ti cation , 86
occu rren ce o , 161 Cap acity, 258–259
Aven tyl. See Nortrip tylin e Carbam azep in e (Tegretol), 126, 175
Aversive con dition in g, 66 Case-con trol stu dies, 291
Avoidan t p erson ality d isorder, 149t Catap lexy, 102
Awake state, 98 Cataton ic sch izop h ren ia, 114t
Cen ters or Disease Con trol an d Preven tion (CDC),
B 260
Babin ski ref ex, 4, 4t Cen tral n ervou s system (CNS), 35–36
Baby blu es, 2–3, 3t Cerebellu m , 66
Bad trip s, 88 Cesarean birth , 1
Barbitu rates, 41, 86–87, 211 Ch arcot-Marie-Tooth d isease, 35t
Bargain in g as stage o d yin g, 27 Ch ild abu se, 211
Bariatric su rgery, 150 p h ysical, 222t
Basal gan glia, 36t, 221 role o p h ysician in , 214
Beck Dep ression In ven tory-II (BDI-II), 77t sequ elae o , 211
Beh avioral th erap ies, 188, 188t sexu al, 211–213
Bell an d p ad ap p aratu s, 163 typ es o , 211
Bell-sh ap ed d istrib u tion . See Norm al d istrib u tion Ch ild cu stod y, 196
Index 355

Ch i-squ are test, 311 Death


Ch lord iazep oxid e (Lib riu m ), 176t ch ild’s con cep tion o , 19
Ch lorp rom azin e (Th orazin e), 170 legal stan d ard o , 262
Ch rom osom e 15 in version -du p lication syn d rom e, p h ysician’s resp on se to, 28
35t review test on , 29–33
Ch ron ological age (CA), 74 stages o , 27
Circadian rh yth m sleep disord er, 101t Declarative system , 37t
Citalop ram (Celexa), 172t, 173t De en se m ech an ism s, 58, 59t–60t
Classical con dition in g im m atu re, 58
aversive con dition in g, 66 m atu re, 58
elem en ts o , 66 rep ression , 58
im p rin tin g, 66 review test on , 61–64
learn ed h elp lessn ess, 66 Deliriu m , 144, 145t
p rin cip les, 66 Deliriu m trem en s (DTs), 86
HELP OTHERS SO THAT GOD WILL HELP YOU.

resp on se acqu isition an d extin ction , 66 Delta waves, 99t


stim u lu s gen eralization , 66 Delu sion s, 50
Clim acteriu m , 19 Delu sion al disorder, 115
Clin ical assessm en t Dem en tia, 145t, 146. See also Alzh eim er d isease
ach ievem en t tests, 75 Den ial
in telligen ce tests, 74–75 as de en se m ech an ism , 59t
m en tal statu s exam in ation , 77, 78t as stage o dyin g, 27
p erson ality tests, 76, 76t Dep en den t p erson ality disorder, 148t, 149t
p sych iatric h istory, 76 Dep erson alization disorder, 150t
p sych ological testin g, 74 Dep ression . See also Major d ep ressive disord er
review test on , 79–82 am p h etam in es or, 84
Clin ical in terview, 235–237, 237t vs. bereavem en t, 28t
Clin ical p rob ability, 296 in eld erly, 26
Clin ical treatm en t trial, 291 as stage o d yin g, 27–28
Clom ip ram in e (An a ran il), 172t, 173t Dep ression with season al p attern (DSP), 122
Clon azep am (Klon op in ), 176t Dep ressive an d bip olar d isord ers
Clorazep ate (Tran xen e), 176t classi cation o , 122–124
Clou d in g o con sciou sn ess, 78t cycloth ym ic d isord er, 122, 124
Clozap in e (Clozaril), 170, 171 de n ition o , 121–122
CNS. See Cen tral n ervou s system (CNS) ep idem iology o , 122
Cocain e, 84, 85t, 211 etiology, 124–126
e ects on sexu ality, 211 li etim e p revalen ce o , 122
Cogn itive beh avioral th erapy, in an xiety disord ers, m ajor d ep ressive d isord er, 122–123
133 m an agem en t, 126
Coh en syn drom e, 35t p ersisten t dep ressive d isorder, 122, 124
Coh ort stu dies, 291 review test on , 127–131
Com a, 78t Descrip tive statistics, 306
Com p licated bereavem en t, 27–28 Desm op ressin acetate, 163
Com p u ted tom ograp h y (CT), 51t Desven la axin e (Pristiq), 172
Con dition ed resp on se, 66 Desyrel. See Trazodon e
Con dition ed stim u lu s, 66 Develop m en t
Con du ct disorder, 161–162, 162t in an t, 5, 5t
RISE USMLE NEPAL

Con den ce in terval (CI), 307, 307t p resch ool ch ild , 6t


Con den tiality, 260 sch ool-age ch ild, 15–16
Con gen ital virilizin g adren al h yp erp lasia, 205 th eories o , 5
Con sciou s m in d, 57 Dexam eth ason e su p p ression test (DST), 50
Con servation , 16 Dextroam p h etam in e (Dexedrin e), 84
Con su ltation -liaison (CL) p sych iatrists, 250 Diabetes, an d sexu al activity, 210, 249
Con trave. See Bu p rop ion Diagn ostic an d Statistical Man u al of Men tal
Con version disorder, 135t Disorders, Fifth Edition (DSM-5), 121
Cop p er, 250t Dialectical b eh avioral th erapy, 188t
Cou n tertran s eren ce, 58, 60 Diazep am (Valiu m ), 176t
Cou rt order, 260 Disin h ibited social en gagem en t disorder, 4, 134
Crack, 84 Dissociative am n esia, 150t
Crossover stu dies, 294 Dissociative d isord ers
Cross-section al stu dies, 291 categories o , 149
Cross-toleran ce, 83 ch aracteristics o , 149, 150t
Cu ltu re, 197–198 m an agem en t o , 149
Cu sh in g’s d isease (h yp ercortisolism ), 50 Dissociative am n esia with or with ou t dissociative
Cycloth ym ic disord er, 122, 124 u gu e, 150t
Dissociative id en tity d isord er, 150t
D Divalp roex (Dep akote), 126
Dam ages, 263 Divorce, 196
Daytim e n ap s, 102 Dom estic p artn er abu se, 213, 213t, 214
356 Index

Don ep ezil (Aricep t), 41 Fem ale sexu al in terest/ arou sal disorder, 208t.
Dop am in e (DA), 38, 85, 100, 113 See also Sexu al dys u n ction
stim u lan ts e ect on , 85 Fem ale orgasm ic disorder, 208t
Doxep in , 172t Fetal alcoh ol syn drom e, 86
Dream s, 57 Fetish istic, 209t
Dru g-assisted in terview, 52 Fish er’s exact test, 311
DSP. See Dep ression with season al p attern (DSP) Fixed in terval rein orcem en t, 67t
DTs. See Deliriu m trem en s (DTs) Fixed ratio rein orcem en t, 67t
Du loxetin e (Cym b alta), 133, 172t, 173t Flat a ect, 78t
Du rab le p ower o attorn ey, 262 Flu m azen il, 87, 176
Dyssom n ias, 100 Flu oxetin e (Prozac), 133, 172t, 173t
Dyston ia m u scu loru m d e orm an s, 35t Flu p h en azin e (Prolixin ), 170
Flu p h en azin e d ecan oate, 170
E Flu razep am (Dalm an e), 176t
HELP OTHERS SO THAT GOD WILL HELP YOU.

Eatin g d isord ers, 150. See also An orexia n ervosa; Flu voxam in e (Lu vox), 172t, 173t
Bu lim ia n ervosa; Bin ge-eatin g d isorder Folstein Min i-Men tal State Exam in ation (MMSE),
Ebstein’s an om aly, 175 51, 51t, 52t, 259
Ecstasy, 84 Food an d Dru g Adm in istration (FDA),
Ego, 58t 176, 177t
Eld erly Foster care system , 4
abu se o , 211, 212t, 214 Fragile X syn drom e, 35t
alcoh ol-related disorders in , 27 Free association , 187
an xiety in , 27 Freebase, 84
deliriu m in , 146 Free f oatin g an xiety, 78t
dep ression in , 26 Freu d , Sigm u n d , 5
m em ory p roblem s in , 146t Freu d’s th eories o m in d , 57
osteop orosis in , 25 stru ctu ral th eory, 58, 58t
sleep p attern s ch an ge in , 26 top ograp h ic th eory, 57–58
Electrocon vu lsive th erapy (ECT), 177 Fron tal lob es, 36t
adm in istration o , 177–178 Fron totem p oral n eu rocogn itive disord er, 148
or dep ressive disorder, 126 Frotteu ristic, 209t
p rob lem s associated with , 178 Fu ll scale IQ (FSIQ), 75
uses o , 177 Fu n ction al MRI ( MRI), 51t
Electroen cep h alogram (EEG), 51t
Electroen cep h alograp h y, 50, 51t G
En cop resis, 163 Galan tam in e (Rem in yl), 41
En docrin e u n ction , evalu ation o , 50 Galvan ic skin resp on se, 52
En u resis, 163 γ-am in obu tyric acid (GABA), 41, 85, 211
Ep id em iology Gate con trol th eory, o p ain , 251
de n ition o , 290 Gau ssian distribu tion . See Norm al distribu tion
in ciden ce, 290–291 Gen d er d ysp h oria, 206
p revalen ce, 290–291 Gen d er iden tity, 206, 206t
review test on , 298–305 Gen itop elvic p ain -p en etration d isord er, 208t
Erectile d ys u n ction , 209, 210 Gen d er role, 206t
Erikson , Erik, 5 Gen eralized an xiety disord er (GAD), 132, 134t, 176.
Escitalop ram (Lexap ro), 133, 172t, 173t See also An xiety d isord ers
RISE USMLE NEPAL

Eszop iclon e (Lu n esta), 176, 176t Gen etic d isorders, testin g or, 260
Euth an asia, 263 Gen etics
Euth ym ic m ood, 78t o Alzh eim er d isease, 146
Evoked EEG (evoked p oten tials), 51t beh avioral, 34, 35t
Exam in ation , review exercise or, 317–352 o sch izop h ren ia, 113
Exh ibition istic, 209t Geriatrics, 25
Exp an sive m ood, 78t Geron tology, 25
Extin ction , 67, 67t Glasgow Com a Scale (GCS), 51, 51t, 52t
resistan ce to, 68 Glu tam ate, 41, 113, 147
Glycin e, 41
F Grie reaction
Factitiou s d isord er, 136, 136t typ ical, 27–28
Fam ily, 195 vs. com p licated, 28t
ch ildren in , 196 Grou p th erapy, 189, 189t
exten ded, 195
review test on , 200–204 H
sin gle-p aren t, 196 Hal way h ou se, 279t
tradition al n u clear, 195 Hallu cin ogen s, 88
Fam ily risk stu dies, or gen etics o b eh avior, 34 lab oratory n d in gs or, 90t
Fam ily th erapy, 189, 189t LSD an d PCP, 89
or con duct disorder, 162 m an agem en t o abu se o , 88t
or op p osition al de an t disorder, 162 m ariju an a, 89
Fear, 78t, 132 u se an d with d rawal o , e ects o , 89t
Index 357

Halop eridol (Haldol), 170 In orm ed con sen t, 259–260


Halop eridol decan oate, 170 In som n ia, 100–102, 103t
Harlow, Harry, 4 In telligen ce
Health care de n ed, 74
costs o , 280 n orm al, 75
delivery system s or, 278–279 test, 74–75
dem ograp h ics o h ealth an d , 283–284 In telligen ce qu otien t (IQ), 75
eth n ic d isp arities in , 198 p er orm an ce, 75
p aym en t or, 280–281, 282t verbal, 75
review test on , 285–289 In tern al em otion al state (m ood), 121
Hep atitis A, 261 In terp erson al th erapy, 189t, 190
Heroin , 87, 87t In terrater reliability, 294
e ects on sexu ality, 211 In toxication , alcoh al, 86
Heterocyclic an tid ep ressan ts (HCAs), 40, 171, 172t, Iron , 250t
HELP OTHERS SO THAT GOD WILL HELP YOU.

173, 173t Irritab le m ood , 78t


Hip p ocam p u s, 66 Isocarboxazid (Marp lan ), 172t, 173t
Hisp an ic/ Latin o Am erican s Isolation o a ect, 59t
cu ltural descrip tion o , 198
am ily typ es, 196t K
Histam in e, 40 Kallm an n’s syn drom e, 35t
Histrion ic p erson ality disord er, 148t, 149t K-com p lex, 99t
HIV in ection , 147 Ketam in e (Sp ecial K), 88
HIV-p ositive doctors, 261 Kid n ey u n ction tests, 50
HIV-p ositive p atien ts, 252, 261 Klein e-Levin syn d rom e, 101t
Hom osexu ality, 206–207 Korsako ’s syn d rom e, 86
Hom ovan illic acid (HVA), 113 Kü bler-Ross, Elizab eth , 27
Horm on es
e ects on aggression , 210–211 L
an d sexu al b eh avior, 212 Lab ile a ect, 78t
Hosp ice organ ization , 279t Lab ile m ood, 78t
Hot f ash es, 19 Late-look b ias, 293t
Hu m an im m u n ode cien cy viru s (HIV), 211–212, Lead , 250t
212t Lead -tim e b ias, 293t
Hu m or, 59t Learn ed h elp lessn ess, 66
Hu n tin gton’s disease, 35t Learn in g th eory, 65, 188
Hyp erten sive crisis, 174t classical con dition in g, 66
Hyp erth yroidism , 50 h ab itu ation , 65
Hyp n agogic/ h yp n op om p ic h allu cin ation s, 102 op eran t con d ition in g, 66–68
Hyp n osis, 209 review test on , 69–73
Hyp oactive sexu al desire disorder, 208t. See also sen sitization , 65
Sexu al dys u n ction Legal an d eth ical issu es
Hyp oth alam u s, 211 ad van ce d irectives, 262
Hyp oth esis, 309 con den tiality, 260
Hyp oth yroidism , 50 death , 262
eu th an asia, 263
I HIV in ection , 261
RISE USMLE NEPAL

Ice (m eth am p h etam in e), 84 im p aired p h ysician s, 264


Id, 58t in orm ed con sen t, 259–260
Iden ti cation , 59t legal com p eten ce, 258–259
Iden tity disru p tion , 150 m ed ical m alp ractice, 263–264
Illn ess an xiety disord er, 135t organ d on ation , 263
Ilop erid on e (Fan ap t), 170 p sych iatric h osp italization , 261
Im ip ram in e (To ran il), 163, 172t, 173t rep ortable illn esses, 260–261
Im p aired p h ysician s, 264 review test on , 265–277
Im p rin tin g, 66 Legal com p eten ce, 258–259
In ciden ce rate, 290 Lesch -Nyh an syn d rom e, 35t
In an t(s) Levom iln acip ran (Fetzim a), 172
attach m en t to p aren t, 4 Levitra. See Vard en a l
bon din g o p aren t with , 3–4 Levoth yroxin e (Syn th roid ), 173
ch aracteristics o , 4–5, 4t, 5t Lewy b od y d em en tia, 147
m otor develop m en t o , 5t Lie d etector test, 52
review test on , 8–14 Lim bic lob es, 36t
social develop m en t o , 4–5, 5t Lin ear correlation , 310
verbal an d cogn itive d evelop m en t o , 4–5, 5t Lioth yron in e (Cytom el), 173
In an t m ortality, 1–2 Lith iu m , 50, 126, 175
Ap gar score, 2, 3t Liver d ys u n ction , alcoh ol u se an d , 86
eth n icity an d , 2t Liver u n ction tests, 50
rates, 2 Livin g will, 262
In eren tial statistics, 306 Lorazep am (Ativan ), 126, 176t
358 Index

Lu rasidon e (Latu da), 170 N


Lu ria-Nebraska n eu rop sych ological b attery, 51t Narcissistic p erson ality disord er, 148t, 149t
Lysergic acid dieth ylam id e (LSD), 88, 89 Narcolep sy, 102
am p h etam in es, 84
M n aloxon e, 87, 88t
Mah ler, Margaret, 5 m an agem en t o , 103t
Major dep ressive disorder, 34, 121 Narcotics An on ym ou s (NA), 189
ch aracteristics o , 122 Native Am erican s, 199
dep ression with season al p attern , 122 Necrop h ilic, 209t
di eren tial diagn osis o , 125t Negative p red ictive valu e (NPV), 295
m asked, 122 Negative rein orcem en t, 67, 67t
p ostp artu m , 3, 3t Neu roan atom y, 35–37
p revalen ce o , 122 Neu roch em ical ch an ges in agin g, 26
sign s an d sym p tom s o , 123t Neu rocogn itive d isorders, 144
HELP OTHERS SO THAT GOD WILL HELP YOU.

su icide risk, 123, 123t Alzh eim er’s d isease, 145–147


Male h yp oactive sexu al desire d isord er, ch aracteristics an d etiologies o , 144, 145t
208, 208t deliriu m , 144, 145t
Malin gerin g, 136, 136t dem en tia, 144, 145t
MAOIs. See Mon oam in e oxidase in h ibitors du e to HIV, 147
(MAOIs) with Lewy b od ies, 147
Map rotilin e (Lu d iom il), 172t, 173t m ild or m ajor, 144, 146
Mariju an a, 89 du e to p rion disease, 148
e ects on sexu ality, 211 du e to vascu lar disease, 147
Marital/ cou p les th erapy, 189t Neu ro b rillary tan gles, 147
Marriage, 195–196 Neu ro b rom atosis-1, 35t
Masked dep ression , 122 Neu ro b rom atosis-2, 35t
Mastu rb ation , 209 Neu roim agin g, 50, 51t
Mazicon . See Flu m azen il Neu rolep tic m align an t syn drom e, 174t
Med icaid , 269, 270, 271t Neu rop ep tides, 41
Med icare, 269, 270, 271t Neu rop sych ological tests, 51, 51t, 52t
Mean , 307 Neu rotran sm ission , 37–38
Med ian , 307 Neu rotran sm itters
Med ical care, seekin g o , 232, 233t am in o acids, 37
Med ical College Ad m ission Test (MCAT), 75 biogen ic am in es, 38–41
Medical m alp ractice, 263–264 classes o , 37
Medical p ractice, 232–233, 233t, 234t p ep tid es, 37
Mem an tin e (Nam en d a), 41, 147 p resyn ap tic/ p ostsyn ap tic recep tors an d, 38
Mem ory system s, 36, 37t regu lation o activity o , 37–38, 38t
Men op au se, 206 in sleep p rod u ction , 100
Men stru al-associated syn d rom e, 101t syn ap ses an d, 37
Men tal age (MA), 74 Nevirap in e (NTP), 212
Men tal statu s exam in ation (MSE), 77, 77t Nicotin e, 84, 85t
Mep eridin e (Dem erol), 174 Nigh tm are d isord er, 101t
Meta-an alysis, 310 NMDA an tagon ist, 147
Metach rom atic leu kod ystrop h y, 35t N-m eth yl-d-asp artate (NMDA), 41
Meth adon e, 87–88, 87t Noctu rn al m yoclon u s d isord er, 101t
RISE USMLE NEPAL

e ects on sexu ality, 211 Non -declarative system , 37t


Meth am p h etam in e (Desoxyn ), 84 Non -Hisp an ic Wh ite Am erican s, 199
Meth ylp h en id ate (Ritalin ), 84 Non -REM sleep, 98
Metoclop rom ide (Reglan ), 170 Norep in ep h rin e, 40, 100
Mid li e crisis, 19 Norep in ep h rin e reu p take in h ibitor,
Min n esota Mu ltip h asic Person ality In ven tory or ADHD, 162
(MMPI-2), 76t Norm al d istrib u tion , 307, 308
Min ors, 258 Nortrip tylin e, 172t
treatm en t o , 260 Nu clear m agn etic reson an ce im agin g (NMRI), 51t
Mirtazap in e (Rem eron ), 172t, 173t Nu ll h yp oth esis, 309
Mod al p eak, 307 Nu m b er n eed ed to h arm (NNH), 292
Mod e, 307 Nu m b er n eed ed to treat (NNT), 292, 293
Mod elin g, 68 Nu rsin g h om e, 279t
Molin d on e (Mob an ), 170 Nu viva. See Varden a l
Mon oam in e oxid ase in h ib itors (MAOIs), 40, 133,
171, 172t, 173, 173t O
Mon oam in es. See Biogen ic am in es Ob am acare, 282–283
Mood , 78t Ob esity, 150
Mood stabilizers, 126, 175 am p h etam in es, 84
Mu n ch au sen syn drom e. See Factitiou s disord er Ob ject p erm an en ce, 5
Myocardial in arction (MI), an d sexu al Ob sessive-com p u lsive disord er (OCD), 133, 134t.
activity, 210 See also An xiety d isord ers
Index 359

Obsessive-com pulsive personality disorder, 148t, 149t m ed ical p ractice, 232–233, 233t, 234t
Occip ital lobes, 36t, 211 review test on , 238–247
Odds ratio, 292 Piaget, Jean , 5
Olan zap in e (Zyp rexa), 126, 170 Pickwickian syn drom e, 102
Olep tro. See Trazod on e Placebo resp on ses, 294
Op eran t con dition in g PMDD. See Prem en stru al dysp h oric d isorder
eatu res o , 67–68, 67t (PMDD)
p rin cip les, 66 PNS. See Perip h eral n ervou s system (PNS)
sh ap in g an d m odelin g, 68 Poin t p revalen ce, 290
Op ioids, 87 Porp h obilin ogen , 50
bu p ren orp h in e, 87–88 Positive p redictive valu e (PPV), 295
h eroin , 87, 87t Positive rein orcem en t, 67, 67t
laboratory n d in gs or, 90t Positron em ission tom ograp h y (PET), 51t
m an agem en t o abu se o , 88t Postp artu m m atern al reaction s, 2–3, 3t
HELP OTHERS SO THAT GOD WILL HELP YOU.

m eth adon e, 87–88, 87t Post-trau m atic stress disorder (PTSD), 133, 134t.
use an d with d rawal o , e ects o , 88t See also An xiety d isord ers
Op p osition al de an t d isorder, 161–162, 162t Power, 309
Organ don ation , 263 Prad er-Willi syn drom e, 35t
Orgasm ic disorder, 208t. See also Sexu al dys un ction Precision , in m ean estim ation , 307
Orlistat, 150 Precon sciou s m in d, 57
Osteop orosis, in elderly, 25 Pred ictive valu e, 295
Overeaters An on ym ou s (OA), 189 Pregn an cy
Oxcarbazep in e (Trilep tal), 175 cocain e u se du rin g, 84
Oxytocin , 161 p sych oactive agen ts in , 176, 177t
teen age, 17
P Prem atu re birth , 1
Pain Prem atu re ejacu lation , 208, 208t, 209. See also
in ch ildren , 251 Sexu al d ys u n ction
ch ron ic, 251 Prem en stru al d ysp h oric d isord er (PMDD), 122
disorder, 135t Presch ool ch ild
m an agem en t, 251 attach m en t, 6–7
Palip eridon e (In vega), 170 ch an ges at 6 years o age, 7
Palm ar grasp ref ex, 4, 4t ch aracteristics o , 7
Pam elor. See Nortrip tylin e m otor d evelop m en t o , 6t
Pan ic attacks, 133 reaction to illn ess, 18
Pan ic d isorder, 133, 134t review test on , 8–14
Param etric tests, 310 social d evelop m en t o , 6t
Paran oid p erson ality d isord er, 148t, 149t verb al an d cogn itive d evelop m en t o , 6t
Parap h ilias an d p arap h ilic disord ers, 209–210, 209t Prescrip tion d ru gs, e ects on sexu ality, 211, 211t
Parasom n ias, 100 Prim ary h yp ersom n ias, 101t
Parietal lob es, 211 Prim ary p rocess th in kin g, 57
Paroxetin e (Paxil), 133, 172t, 173t Progressive m yoclon ic ep ilep sy, 35t
Passive-aggressive p erson ality disord er, 148t, 149t Projection , 59t
Patien t com p lian ce, 49 Prop ran olol (In deral), 133
Patien t-con trolled an algesia (PCA), 251 Prosth etic d evices, 209
Patien t p rotection an d a ord able care act (ACA). Protrip tylin e (Vivactil), 172t, 173t
RISE USMLE NEPAL

See also Obam acare Pseu dodem en tia, 26


Pedop h ilia, 209t Psych iatric care, seekin g o , 232
Period p revalen ce, 290 Psych iatric disord ers, in ch ild ren
Perip h eral n ervou s system (PNS), 35–36 atten tion de cit/ h yp eractivity disorder, 161–162
Perp h en azin e (Trila on ), 170 elim in ation d isord ers, 163
Persisten t dep ressive d isorder, 122, 124 review test on , 164–168
Persisten t vegetative state (PVS), 262 selective m u tism , 161
Person ality disorders (PDs), 148 sep aration an xiety d isord er, 163
ch aracteristics o , 148, 148t, 149t Tou rette’s disorder, 163
classi cation o , 148, 148t, 149t Psych iatric h istory, 76
m an agem en t o , 149 Psych iatric sym p tom s
Person ality tests, 76, 76t biogen ic am in es, m easu rem en t o , 49–50
Ph en cyclidin e (PCP), 88, 89, 211 dru g-assisted in terview, 52
Ph en elzin e (Nardil), 172t, 173t electroen cep h alogram , 50–51
Ph en term in e (Ion am in ), 150 en docrin e u n ction , 50
Ph en ylketon u ria, 35t galvan ic skin resp on se, 52
Ph obias, 132, 133, 134t. See also An xiety d isorders n eu roim agin g, 50–51
Ph ysician -assisted su icide, 263 n eu rop sych ological tests, 51–52
Ph ysician -p atien t relation sh ip overview, 49
adh eren ce, 233–235, 236t p sych otrop ic d ru gs, 49–50
breakdown o , 263 review test on , 53–56
clin ical in terview, 235–237, 237t sodiu m lactate ad m in istration , 52
360 Index

Psych oactive agen ts Savan t skills, 160


in eld erly, 27 Sawtooth waves, 99t
in p regn an cy, 176, 177t Sch izoa ective d isorder, 115t
Psych oan alysis, 187–188 Sch izoid p erson ality d isorder, 148–149
Psych ological tests, 74 Sch izop h ren ia, 34, 111, 115t
ach ievem en t tests, 75 active p h ase, 112
in d ividu al vs. grou p testin g, 74 age o on set o , 111
in telligen ce test, 74–75 an tip sych otics or, 169
p erson ality tests, 76, 76t cou rse o , 112
p sych iatric evalu ation , 76–77, 77t, 78t di eren tial diagn osis o , 114
typ es o , 74 etiology o , 113
Psych ological th erap ies, 187 gen etics o , 113t
beh avioral th erap ies, 188 m an agem en t o , 114
am ily th erapy, 189 n eu ral p ath ology o , 113
HELP OTHERS SO THAT GOD WILL HELP YOU.

grou p th erapy, 190 p rod rom al p h ase, 112


p sych oan alysis an d related th erap ies, 187–188 p rogn osis or, 112
review test on , 191–194 residu al p h ase, 112
su p p ortive an d in terp erson al th erapy, 190 review test on , 116–121
Psych op h arm acologic agen ts, 169 sym p tom s o , 111, 112t
Psych otic disorder, p ostp artu m , 3, 3t Sch izop h ren i orm d isord er, 115t
Psych otic sym p tom s, 123 Sch izotyp al p erson ality disord er, 148t–149t
Psych otrop ic agen ts, 147 Sch olastic Ap titu d e Test (SAT), 75
Psych otrop ic dru gs Sch ool-age ch ild
m easu rem en t o levels o , 49–50 cogn itive ch aracteristics, 15–16
Pu berty, 16 m otor d evelop m en t, 16
Pu n ish m en t, 67, 67t reaction to illn ess, 18
P (p robability) valu e, 309 review test on , 20–24
social ch aracteristics, 15
Q Sed ation , 175
Qu etiap in e (Seroqu el), 170 Sed atives, 85–87, 86t, 126
adm in istration o , 52
R alcoh ol, 86
Ram elteon (Rozerem ), 176, 176t barb itu rates, 86–87
Ran d om ization , 294 ben zodiazep in es, 87
Rap e, 214–215 m an agem en t o abu se o , 88t
Rap id eye m ovem en t (REM) sleep, 98–100, 101t u se an d with d rawal o , e ects o , 86t
Ration alization , 59t Seizu res, 171
Reaction orm ation , 59t Selection bias, 293t
Recall bias, 293t Selective m elaton in agon ist, 176
Receiver op eratin g ch aracteristic (ROC) cu rves, Selective m u tism , 161
295, 296 Selective seroton in an d n orep in ep h rin e reu p take
Regression , 59t in h ibitors (SNRIs), 133, 171, 172t, 173, 173t
Reactive attach m en t disorder, 4 Selective seroton in reu p take in h ib itors (SSRIs), 40,
Reh abilitation cen ters, 279t 133, 171, 172t, 173, 173t, 209
Rein orcem en t Selegilin e (Em sam :tran sd erm al p atch ), 172t, 173t
n egative, 67 Sen sate- ocu s exercises, 208–209
RISE USMLE NEPAL

p ositive, 67 Sen sitivity, 294, 295


sch ed u les o , 67, 67t Sep aration an xiety d isord er, 163
Relative risk, 292 p resch ool ch ild , 7
Relaxation tech n iqu e, 209 Seriation , 16
Reliability, 294 Seroton ergic system s, 170
REM sleep. See Rap id eye m ovem en t (REM) sleep Seroton in , 40, 88, 100, 113, 161, 211
Research stu dies, 259, 291 Seroton in syn drom e, 174, 174t
Residen tial assisted livin g acility, 279t Sertralin e (Zolo t), 133, 172t, 173t
Restless legs syn drom e, 101t Sexu al ab u sers, 212–213
Restricted a ect, 77t Sexu al aversion d isord er, 208t. See also Sexu al
Reticu lar system , 36t, 211 dys u n ction
Rett’s disorder, 161 Sexu al d evelop m en t, 205–206
Risk, 291–293 Sexu al d ys u n ction , 207
Risp eridon e (Risp erdal), 126, 170 acqu ired, 208
Rivastigm in e (Exelon ), 41 cau ses o , 207–208
Rom azicon . See Flu m azen il classi cation s o , 208
Rootin g ref ex, 4, 4t li elon g, 208
Rorsch ach Test, 76t m an agem en t o , 208–209
Sexu ality
S agin g an d, 210
Sam p lin g bias, 293t biology o , 207–208
SAT. See Sch olastic Ap titu d e Test (SAT) dru gs an d, 211, 211t
Index 361

HIV an d, 211–212 h yp oth esis testin g, 309–310


illn ess an d , 210 review test on , 312–316
in ju ry an d, 210 statistical tests, 310–311
p arap h ilias, 209–210, 209t Statistical tests, 310–311
p ren atal p sych ological sexu al d evelop m en t, categorical tests, 311
205–206, 206t n on p aram etric, 310
review test on , 213–216 p aram etric, 310
sexu al dys u n ction s, 207–209, 208t Stim u lan ts, 84, 171
Sexu ally tran sm itted d iseases (STDs) am p h etam in es, 84
in ch ildren , 211 ca ein e, 84
rep ortin g o , 261 cocain e, 84
Sexu al m asoch ism , 209t e ects o u se an d with drawal o , 85t
Sexu al orien tation , 206t lab oratory n d in gs or, 90t
Sexu al resp on se cycle, 207, 208t n eu rotran sm itter association s, 85
HELP OTHERS SO THAT GOD WILL HELP YOU.

Sexu al sad ism , 209t n icotin e, 84


Sh ap in g, 68 Stran ger an xiety, 4
Sh ared p sych otic disord er, 115t Stress
“Sick role,” 232–233 AIDS p atien ts an d, 252
Silden a l citrate (Viagra), 209 ch ron ic p ain an d, 251
Sin equ an . See Doxep in actors a ectin g h ealth , 248
Sin gle-p aren t am ilies, 196 li e even ts an d , 249, 249t
Sin gle p h oton em ission tom ograp h y (SPECT), 51t p h ysiologic e ects o , 248
Skewed distribu tion s, 307 p sych ological, 250–251
Sleep review test on , 253–257
ch aracteristics o , 100t Stroop color-word test, 51t
circadian cycle, 98 Stu p or, 78t
n eu rotran sm itters in volved in , 100 Su b lim ation , 60t
stages o , 98–100, 99 , 99t Su b oxon e, 87
Sleep disorders Su b stan ce abu se
breath in g-related, 102 clin ical eatu res o , 89, 90t
classi cation o , 100 de n ition o , 83
in som n ia, 100–102 e ects on aggression , 221
m an agem en t o , 103, 103t em ergen cy dep artm en t iden ti cation
n arcolep sy, 102 o , 90t
Sleep dru n ken n ess, 101t ep idem iology an d dem ograp h ics o ,
Sleep in g p ills, 86 83–84, 84t
Sleep p aralysis, 102 h allu cin ogen s an d related agen ts,
Sleep p attern s ch an ge, in eld erly, 26 88–89, 89t
Sleep sp in dle, 99t m an agem en t o , 90, 90t
Sleep terror disord er, 101t op ioids, 87–88, 88t
Sleepwalkin g disord er, 101t review test on , 91–97
SNRIs. See Selective seroton in an d n orep in ep h rin e sedatives, 85–87, 86t
reu p take in h ibitors (SNRIs) an d sexu al dys u n ction , 207
Social an xiety d isord er, 133 stim u lan ts, 84–85, 85t
Social Read ju stm en t Ratin g Scale, 249, 249t Su b stan ce d ep en d en ce, 83
Sodiu m lactate, ad m in istration o , 52 Su ckin g ref ex, 4, 4t
RISE USMLE NEPAL

Sodom y, 214 Su icide, risk actors or, 123, 123t


Som atic sym p tom disord ers, 135 Su p erego, 58t
ch aracteristics an d classi cation o , 135, 135t Su p p ortive th erapy, 189t, 190
di eren tial diagn osis o , 135 Su p p ression , 60t
m an agem en t o , 135 Su rgeon s, 263
Som n olen ce, 78t Su rrogates, 262
Sotos syn d rom e, 35t Su rveillan ce bias, 293t
Sp eci city, 294, 295 SWAG (su icid ality, weigh t loss, an h ed on ia, gu ilt)
Sp eed, 84 sym p tom , 122, 123t
Sp in a bi da, 175 System atic d esen sitization , 209
Sp in al cord in ju ry, an d sexu al activity, 210 in an xiety d isorders, 132, 133
Sp itz, Ren é, 4
Sp littin g, 59t T
Squ eeze tech n iqu e, 209 Tactile h allu cin ation s, 84
SSRIs. See Selective seroton in reu p take in h ib itors Tadala l (Cialis), 209
(SSRIs) Tan n er stages, o sexu al d evelop m en t, 16t
Stages o ch an ge m odel, 235 Telep h on e scatologia, 209t
Stan dard deviation (SD), 307, 307t Tem azep am (Restoril), 176t
Stan dard error (SE), 307, 307t Tem gesic, 87
Startle (Moro) ref ex, 4, 4t Tem p oral lobes, 36t
Statistical an alyses Test battery, 74
elem en ts o , 306–308 Testosteron e, 206
362 Index

Test-retest reliability, 294 Varden a l, 209


Tetrah ydrocan n abin ol (THC), 89 Variable, 306
Th em atic Ap p ercep tion Test (TAT), 76t dep en den t, 306
Th erap eu tic blood levels, 49 in d ep en den t, 306
Th eta waves, 99t Variable in terval rein orcem en t, 67t
Th ioridazin e (Mellaril), 170 Variable ratio rein orcem en t, 67t
Th ioth ixen e (Navan e), 170 Vascu lar n eu rocogn itive disorder, 147
Th yroid u n ction tests, 50 Vasodilators, 209
Th yroid h orm on es, or m ood disorders Ven la axin e (E exor), 133, 172t, 173t
m an agem en t, 173 Vilazod on e (Viib ryd), 172t, 173t
Toddler(s) Vin elan d Ad ap tive Beh avior Scales, 75
attach m en t, 5–6 Vin elan d Social Matu rity Scale, 18
m otor develop m en t o , 6t Visitin g n u rse association , 279t
reaction to illn ess, 18 Vitam in an d m in eral toxicity, 249, 250t
HELP OTHERS SO THAT GOD WILL HELP YOU.

social develop m en t o , 6t Vortioxetin e (Brin tellix), 172


verbal an d cogn itive d evelop m en t o , 6t Voyeu rism , 209t
Toilet train in g, 7
Toleran ce, 83 W
Tort, 263 Wech sler Adu lt In telligen ce Scale-Fou rth Edition
Tou rette’s disorder, 163 (WAIS-IV), 75
Trackin g ref ex, 4, 4t Wech sler In telligen ce Scale or Ch ildren (WISC),
Tram ad ol (Ultram ), 174 75
Tran qu ilizers, 87 Wech sler Presch ool an d Prim ary Scale o
Tran scran ial m agn etic stim u lation (TMS), 178 In telligen ce (WPPSI), 75
Tran s eren ce, 58, 60 Wellbu trin . See Bu p rop ion
Tran svestic etish ism , 209t Wern icke’s syn drom e, 86
Tran ylcyp rom in e (Parn ate), 172t, 173t Wid e Ran ge Ach ievem en t Test (WRAT), 75
Trazod on e, 172t, 173t William’s syn drom e an d Wilson’s disease, 35t
Triazolam (Halcion ), 176t Wiscon sin card sort, 51t
Trif u op erazin e (Stelazin e), 170 With d rawal
t-tests, 310 alcoh al, 86, 86t
Tu berou s sclerosis, 35t de n ition o , 83
Twin stu dies, or gen etics o beh avior, 34 op ioids, 87, 87t
Typ e I error, 309 sedatives, 85, 86t
Typ e II error, 309 stim u lan ts, 85t
Tyram in e, 173

U X
Un con dition ed resp on se, 66 Xen ical. See Orlistat
Un con dition ed stim u lu s, 66
Un con sciou s m in d, 57 Z
Un doin g, 60t Zalep lon (Son ata), 176, 176t
Un ited States Medical Licen sin g Exam in ation Zidovu din e (AZT), 212
(USMLE), 75 Zin c, 250t
Zip rasidon e (Geodon ), 170
V Zolp idem (Am bien ), 176, 176t
RISE USMLE NEPAL

Validity, 294 z score, 307, 307t


Valp roic acid (Dep aken e, Dep akote), 175 Zyban . See Bu p rop ion

You might also like